Anda di halaman 1dari 333

Amusin S C A r y

gly
THE C2005 LOCAL GOVERNMENT REVIEWER -1-
Control Number: C2005-

OBIT ER
The Control Number is to ensure that no illegal copies were
made; illegal copies meaning copies reprinted without
permission from the APOCRYPHAL MAGGOTS.

The APOCRYPHAL MAGGOTS has always endeavored to


make readable (short) yet substantial weekly digests. This
is also our aim for the reviewer. We are making it as short
(and substantial) as possible since (1) this is a reviewer,
not a compilation; and (2) we are assuming that you’ve
read throughout the semester; hence this is just to refresh
your memories. We also included class notes and sample
exams. Class notes are represented by  or in text boxes
while sample exams are at the end of the reviewer. We
added some irreverent material whenever necessary.
The Apocryphal Maggots:
TABLE OF CONTENTS The boys (from left) Chrisgel, Rainier and Geoffry
Part I: Introduction ……………………… The girls (from left): Corina Grace and Sylvie
Blanche
….. 1
Part II: Book 1 General Provisions ……
……. 16 Par t I: Int roduc tion : History and
A. Policy and Application ……… Basic Conc epts
……16
Bas ic L aw s
B. General Power and Attributes
……19 1987 Constitution
Part II: Book 2 Local Taxation ………… ARTICLE X: LOCAL GOVERNMENT

…….202 GENERAL PROVISIONS


Part III: Local Government Units ………
……244 Sec1. The territorial and political subdivisions of the Republic of
the Philippines are the provinces, cities, municipalities, and
Part IV: Miscellaneous and Final barangays. There shall be autonomous regions in Muslim
Provisions ..283 Mindanao and the Cordilleras as hereinafter provided.
Sample Exams ………………………………… Sec2. The territorial and political subdivisions shall enjoy local
306 autonomy.

Sec3. The Congress shall enact a local government code which


shall provide for a more responsive and accountable local
government structure instituted through a system of
decentralization with effective mechanisms of recall, initiative,
and referendum, allocate among the different local government
units their powers, responsibilities, and resources, and provide
for the qualifications, election, appointment and removal, term,
salaries, powers and functions and duties of local officials, and
all other matters relating to the organization and operation of the
local units.

The Apocryphal Maggots:


Rainier, Chrisgel, Corina, Geoffry, Grace and Sylvie Blanche
The Flibbertigibbet Worms:
Golda, Gladys and Melyjane
CA VE AT: By simply r ea ding this re vi ew er a t the end o f the sem este r wi ll (hopef ully) guar antee you r pas sing this cou rse. D rink mode rat ely .
Amusin S C A r y
gly
THE C2005 LOCAL GOVERNMENT REVIEWER -2-
Sec4. The President of the Philippines shall exercise general for purposes commonly beneficial to them in accordance with
supervision over local governments. Provinces with respect to law.
component cities and municipalities, and cities and
municipalities with respect to component barangays, shall Sec14. The President shall provide for regional development
ensure that the acts of their component units are within the councils or other similar bodies composed of local government
scope of their prescribed powers and functions. officials, regional heads of departments and other government
offices, and representatives from non-governmental
Sec5. Each local government unit shall have the power to create organizations within the regions for purposes of administrative
its own sources of revenues and to levy taxes, fees and charges decentralization to strengthen the autonomy of the units therein
subject to such guidelines and limitations as the Congress may and to accelerate the economic and social growth and
provide, consistent with the basic policy of local autonomy. Such development of the units in the region.
taxes, fees, and charges shall accrue exclusively to the local
governments. AUTONOMOUS REGIONS

Sec6. Local government units shall have a just share, as Sec15. There shall be created autonomous regions in Muslim
determined by law, in the national taxes which shall be Mindanao and in the Cordilleras consisting of provinces, cities,
automatically released to them. municipalities, and geographical areas sharing common and
distinctive historical and cultural heritage, economic and social
Sec7. Local governments shall be entitled to an equitable share in structures, and other relevant characteristics within the
the proceeds of the utilization and development of the national framework of this Constitution and the national sovereignty as
wealth within their respective areas, in the manner provided by well as territorial integrity of the Republic of the Philippines.
law, including sharing the same with the inhabitants by way of
direct benefits. Sec16. The President shall exercise general supervision over
autonomous regions to ensure that laws are faithfully executed.
Sec8. The term of office of elective local officials, except barangay
officials, which shall be determined by law, shall be three years Sec17. All powers, functions, and responsibilities not granted by
and no such official shall serve for more than three consecutive this Constitution or by law to the autonomous regions shall be
terms. Voluntary renunciation of the office for any length of time vested in the National Government.
shall not be considered as an interruption in the continuity of his
service for the full term for which he was elected. Sec18. The Congress shall enact an organic act for each
autonomous region with the assistance and participation of the
Sec9. Legislative bodies of local governments shall have sectoral regional consultative commission composed of representatives
representation as may be prescribed by law. appointed by the President from a list of nominees from multi-
sectoral bodies. The organic act shall define the basic structure
Sec10. No province, city, municipality, or barangay may be of government for the region consisting of the executive
created, divided, merged, abolished, or its boundary department and legislative assembly, both of which shall be
substantially altered, except in accordance with the criteria elective and representative of the constituent political units. The
established in the local government code and subject to organic acts shall likewise provide for special courts with
approval by a majority of the votes cast in a plebiscite in the personal, family, and property law jurisdiction consistent with the
political units directly affected. provisions of this Constitution and national laws.

Sec11. The Congress may, by law, create special metropolitan The creation of the autonomous region shall be effective when
political subdivisions, subject to a plebiscite as set forth in approved by majority of the votes cast by the constituent units in
Section 10 hereof. The component cities and municipalities shall a plebiscite called for the purpose, provided that only provinces,
retain their basic autonomy and shall be entitled to their own cities, and geographic areas voting favorably in such plebiscite
local executive and legislative assemblies. The jurisdiction of the shall be included in the autonomous region.
metropolitan authority that will thereby be created shall be
limited to basic services requiring coordination. Sec19. The first Congress elected under this Constitution shall,
within eighteen months from the time of organization of both
Sec12. Cities that are highly urbanized, as determined by law, and Houses, pass the organic acts for the autonomous regions in
component cities whose charters prohibit their voters from Muslim Mindanao and the Cordilleras.
voting for provincial elective officials, shall be independent of the
province. The voters of component cities within a province, Sec20. Within its territorial jurisdiction and subject to the provisions
whose charters contain no such prohibition, shall not be of this Constitution and national laws, the organic act of
deprived of their right to vote for elective provincial officials. autonomous regions shall provide for legislative powers over:
1. Administrative organization;
Sec13. Local government units may group themselves, 2. Creation of sources of revenues;
consolidate or coordinate their efforts, services, and resources 3. Ancestral domain and natural resources;

The Apocryphal Maggots:


Rainier, Chrisgel, Corina, Geoffry, Grace and Sylvie Blanche
The Flibbertigibbet Worms:
Golda, Gladys and Melyjane
CA VE AT: By simply r ea ding this re vi ew er a t the end o f the sem este r wi ll (hopef ully) guar antee you r pas sing this cou rse. D rink mode rat ely .
Amusin S C A r y
gly
THE C2005 LOCAL GOVERNMENT REVIEWER -3-
4. Personal, family, and property relations; of local autonomy and monitor compliance thereof by said
5. Regional urban and rural planning development; units;
6. Economic, social, and tourism development; (3) Provide assistance in the preparation of national legislation
7. Educational policies; affecting local government units;
8. Preservation and development of the cultural heritage; and (4) Establish and prescribe plans, policies, programs and
9. Such other matters as may be authorized by law for the projects to strengthen the administrative, technical and
promotion of the general welfare of the people of the fiscal capabilities of local government offices and
region. personnel;
(5) Formulate and implement policies, plans, programs and
Sec21. The preservation of peace and order within the regions projects to meet national and local emergencies arising
shall be the responsibility of the local police agencies which from natural and man-made disasters; and
shall be organized, maintained, supervised, and utilized in (6) Perform such other functions as may be provided by law.
accordance with applicable laws. The defense and security of
the regions shall be the responsibility of the National Sec4. Organizational Structure.
Government.
Chapter 2 DEPARTMENT PROPER

Sec5. Office of the Secretary. - The Office of the Secretary shall


A passage from Teresa of Avila’s Conceptions of the Love of God: consist of the Secretary and his immediate staff.
Sec6. Undersecretaries and Assistant Secretaries -
But when this most wealthy Spouse desires to enrich and
Chapter 3 DEPARTMENT SERVICES
comfort the Bride still more, He draws her so closely to Him that
she is like one who swoons from excess of pleasure and joy and Sec7. Planning Service. -
seems suspended in those Divine arms and drawn near to that Sec8. Financial and Management Service. -
sacred side and to those Divine breasts. Sustained by that Sec9. Legal Service. –
Divine milk with which her Spouse continually nourishes her Sec10. Administrative Service. -
and growing in grace so that she may be enabled to receive His Sec11. Electronic Data Processing Service. -
comforts, she can do nothing but rejoice… With what to
compares this [the soul] known not, save to the caress of a
mother who so dearly loves her child and feeds and caresses it. Chapter 4 BUREAUS AND OFFICES

Sec12. Bureau of Local Government Supervision.


Sec13. Bureau of Local Government Development.
ADMINISTRATIVE CODE OF 1987, TitleXII Sec14. Office of Public Affairs.
LOCAL GOVERNMENT Sec15. Local Government Academy. -
Sec16. National Barangay Operations Office. -
Chapter 1 GENERAL PROVISIONS Sec17. Office of Project Development Services.

Sec1. Declaration of Policy. – The State shall: Chapter 5 REGIONAL AND FIELD OFFICES
(1) ensure the autonomy of local governments by providing a
more responsive and accountable local government Sec18. Regional and Field Offices.
structurethrough a system of decentralization. T
(2) guarantee LGUs their just share in national taxes and their Chapter 6 LEAGUES OF PROVINCES, CITIES AND
equitable share in proceeds from the use of natural MUNICIPALITIES
resources, and afford them a wider latitude for resources
generation. Sec19. Leagues of Provinces, Cities and Municipalities

Sec2. Mandate
Sec3. Powers and Functions of Department. - To accomplish its
mandate, the Department shall: RA 6975: An Act Establishing The Philippine National Police
(1) Advise the President on the promulgation of policies, rules, Under A Reorganized Department Of The Interior And
regulations and other issuances relative to the general Local Government, And For Other Purposes
supervision of local government units;
(2) Establish and prescribe rules, regulations and other
CHAPTER I: The Department Of The Interior And Local
issuances and implementing laws on the general
Government
supervision of local government units and on the promotion

The Apocryphal Maggots:


Rainier, Chrisgel, Corina, Geoffry, Grace and Sylvie Blanche
The Flibbertigibbet Worms:
Golda, Gladys and Melyjane
CA VE AT: By simply r ea ding this re vi ew er a t the end o f the sem este r wi ll (hopef ully) guar antee you r pas sing this cou rse. D rink mode rat ely .
Amusin S C A r y
gly
THE C2005 LOCAL GOVERNMENT REVIEWER -4-
Sec4. The Department of the Interior and Local Government. — … In times of national emergency, all elements of the PNP, the
the Department of Local Government is hereby reorganized into Bureau of Fire Protection, and the Bureau of Jail Management
the Department of the Interior and Local Government, … in and Penology shall, upon direction of the President, assist the
accordance with the provisions of this Act. Armed Forces of the Philippines in meeting the national
emergency.
Sec5. Powers and Functions of the Department. — … the
Department shall continue to exercise the powers and functions The complementary relationship between the Department of the
of the Department of Local Government in addition to the Interior and Local Government and the Department of National
powers and functions as herein provided. Defense in any of the preceding eventualities shall be jointly
prescribed by their respective Secretaries in a memorandum of
Sec6. Organization. — agreement that shall thereafter be published and implemented.
Sec7. Department Proper. — 1 Sec, 2 USecs and 3 Asst Secs

Sec8. Head of Department. —


Sec9. General Powers, Term of Office and Compensation of the RA 8551: An Act Providing For The Reform And
Secretary. — The authority and responsibility for the exercise … Reorganization Of The Philippine National Police And For
shall be vested in the Secretary, who shall hold office at the Other Purposes, Amending Certain Provisions Of RA6975
pleasure of the President…

Sec10. Specific Powers and Functions of the Secretary. —


(a) Prepare and submit periodic reports, including a Quarterly Renaissance journalist and hip porn-purveyor Aretino writes:
Anti-Crime Operations Report and such other reports as the
President and Congress may require; “What harm is there in seeing a man mounting a woman?
(b) Act as Chairman and Presiding Officer of the National Police Should beasts, then, be free-er than we are? We should wear
Commission; and that thing nature gave us for the preservation of the species on a
(c) Delegate authority to exercise any substantive or chain around our necks or as a medal on our hats; for that is the
administrative function to the members of the National Police fountain rivers of human beings come forth from… That thing
Commission or other officers of rank within the Department. made you… it created me, and I am better than bread. It
produced the Bembos, the Molzas, the Varchis, the Ugolin
Sec11. Regional Offices. — The Department shall establish, Martellis… the Titans, and the Michelangelos, and after them
operate and maintain a regional office in each of the
the Popes, emperors, and kings. It generated handsome boys
administrative regions of the country to implement the policies
and beautiful women with their ‘holy of holies.’ We should
and programs of the Department…
celebrate all this by establishing special holy days and festivities
Sec12. Relationship of the Department with the Department of in its honor rather than confining it in a small piece of cloth or
National Defense. — During a period of twenty-four (24) months silk. Men’s hands might be well hidden since they gamble
from the effectivity of this Act, the Armed Forces of the money, swear oaths, practice usury, make obscene gestures,
Philippines (AFP) shall continue its present role of preserving tear, pull, punch, wound and kill.”
the internal and external security of the State: Provided, That
said period may be extended by the President, if he finds it
justifiable, for another period not exceeding twenty-four (24) Ov er view , H is tor y, A ss es sm en t
months, after which, the Department shall automatically take
over from the AFP the primary role of preserving internal
The Local Government Code
security, leaving to the AFP its primary role of preserving
-Aquilino Pimentel
external security. However, even after the Department has
assumed primary responsibility on matters affecting internal
security, including the suppression of insurgency, and there are Local Government Code of 1991 is the result of the Legislative
serious threats to national security and public order, such as effort to flesh out the mandate embodied in Article X on Local
where insurgents have gained considerable foothold in the Government of the Constitution. It uproots decades of highly
community thereby necessitating the employment of bigger centralized decision making that have deterred the development of
tactical forces and the utilization of higher caliber armaments the country and places upon the local officials a major portion of
and better armored vehicles, the President may, upon the responsibility for the modernization of the local communities.
recommendation of the peace and order council, call upon the Before, plans and funds for development were decided mainly by
Armed Forces of the Philippines to assume the primary role and the central government in Manila and, at times, even the way
the Philippine National Police (PNP) to play the supportive role development projects and programs in the countryside were to be
in the area concerned. implemented would be pre-determined by people manning the
central government

The Apocryphal Maggots:


Rainier, Chrisgel, Corina, Geoffry, Grace and Sylvie Blanche
The Flibbertigibbet Worms:
Golda, Gladys and Melyjane
CA VE AT: By simply r ea ding this re vi ew er a t the end o f the sem este r wi ll (hopef ully) guar antee you r pas sing this cou rse. D rink mode rat ely .
Amusin S C A r y
gly
THE C2005 LOCAL GOVERNMENT REVIEWER -5-
Local Governance and Decentralisation: The Philippine
Experience, Alex B. Brilliantes, Jr Hence, LG’s proposed to DBM to set aside a stabilization fund or
augmentation fund to assist LG that could not afford the devolution
HISTORICAL ACCOUNT AND PRESENT POLITICAL CONTEXT costs at all fronts.

The movement for autonomy has its origins as early as 1898. HB 6346 was proposed which provides that the costs of devolved
Local Governments (LG) were subject to regulation including the services shall be first excluded or deducted from the total IRA
limitation of powers of taxation in order that provincial & municipal allocable to LGU and shall be equitably distributed to them. In
taxation may never be antagonistic to the system of taxation of the 1993, the amount so excluded/deducted shall be the actual cost of
State. devolution. In 1994 onwards, the amount for devolved services
When Americans arrived at start of 20 th Century, centralisation of shall increase/decrease in proportion to the increase/decrease of
the politico-administrative system was effected in order to ensure the total IRA of LGU’s.
the consolidation of powers of the colonialists. Throughout their
stay, there was a shift from “control” over LG by executive dept to B. Political Concerns - pertains to warlordism/ bossism/
“supervision” towards gen direction of some kind of autonomy for refeudalisation, intergovernmental relations, and GO-NGO
LG. From 1959-present, various laws were enacted to provide relations
greater local autonomy.
Then, as now, there is an imperative for a dominant and assertive The issue of decentralization and local autonomy is really
leadership for the consolidation and survival of weak state BUT anchored on power and how it should be dispersed from the center
there is clamor and demand for maximum autonomy from the and how it will be shared among various levels of gov’t.
central gov’t in order to become more responsive to situations
obtaining locally. Since LGCode encourages participation of NGOs. there is a
danger that the term NGO might be construed and abused in its
ISSUES AND CONCERNS broadest and most liberal context resulting in the sprouting of all
kinds of NGO’s, fly-by-night and politician-organised NGO. Some
A. Administrative Concerns – pertain to personnel, financial, see NGO’s as a ‘back door’ to local politics.
organization and management, intergovernmental relations, and
relations with the NGO’s and PO’s and the private sector There is political issue of turf and delineation of sharing of power
among various levels of gov’t (National-Local, Local-local) LGCode
Major concern: ‘administrative capacities’ or absorptive capabilities encourages partnership but problems are inevitable in the process
of LGU to assume various functions and responsibilities that will be of getting to know each other.
devolved to them
Some sectors have attempted to derail the LGCode provision of
Confusion: sectoral representation in the local sanggunian claiming budgetary
The LGCode provides that personnel of national gov’t concerns.
agencies devolved to LG shall be mandatorily absorbed by LG and
they shall not suffer any diminution in pay and rank. CSC released The concern that devolution will lead to warlordism is certainly a
memoranda to this effect. possibility but LGCode process of local autonomy also entails
BUT nationally employed personnel enjoy higher accountability with provisions for recall and initiative.
salaries than locally paid employees.
The DBM circulars prohibited LG from allocating more CONCLUSION
than 45-55% of their budgets for personnel services.
Results: Budget is strained so these contradictions must be The passage of the LGCode, while an answer to the decade-old
reconciled. problem of overcentralisation has brought with it politico-
administrative shocks and stresses upon the LG system.
Second Concern: delivery of basic services at the local level will be
disrupted largely as a result of transition But these problems and stresses are not insurmountable. Support
is needed and can be manifested by:
Because of these concerns, there seemed to be a general
hesitancy among LG esp. poorer LG’s to accept the responsibility a. non-interruption of IRA to LGU
for the delivery of a whole range of basic services. They fear that b. proper allocation of the augmentation fund identified by
they may not be able to financially support the delivery of such the DILG specifically in terms of demonstrating a bias for
services. lower income LGU
c. support for various capability-building efforts of various
Original idea in LGCode: institutions such as those conducted by academic and
Costs of devolution would be charged to IRA of LGU but IRA of nonacademic institutions
LGU is barely sufficient to support the implementation of various d. encouragement of NGO participation in the various
programs & projects that will be absorbed by LG. mechanisms made available to them.

The Apocryphal Maggots:


Rainier, Chrisgel, Corina, Geoffry, Grace and Sylvie Blanche
The Flibbertigibbet Worms:
Golda, Gladys and Melyjane
CA VE AT: By simply r ea ding this re vi ew er a t the end o f the sem este r wi ll (hopef ully) guar antee you r pas sing this cou rse. D rink mode rat ely .
Amusin S C A r y
gly
THE C2005 LOCAL GOVERNMENT REVIEWER -6-
e. Promotion of local accountabilities through the system of - broadening their taxing powers
recall. - providing them with specific share from national wealth
exploited in their area (mining, fishery, forestry charges)
The enactment of LG Code somehow marks the culmination of the - increasing share from national taxes (IRA – internal
struggle of the local institutions. The political will to implement this revenue allotments from 11% to 40%)
code is indispensable. Choices that demonstrate bias for local - increased elbow room to generate revenues from local
autonomy have to be made with central officials and agencies fees and charges
taking secondary role this time. Otherwise, it will remain another 5. Foundation for development and evolution of more
piece of legislation that will follow the well-trodden path to non- entrepreneurial-oriented LGUs – to:
implementation. - enter into B-O-T arrangements with private sector
- float bonds
- obtain loans from private institutions, etc
Local Governments In A Democratizing Polity: Trends And  to encourage them to be “more business-like” and
Prospects Alex Brillantes, Jr. competitive in their operations contradistinguished from
“traditional” government norms and operations
Introduction 6. Legal infrastructure for participation of NGOs and Pos in the
The enactment of a Local Government Code in the Philippines in process of governance by making their membership in local
1991 was the most radical and far reaching policy. The special bodies mandatory (e.g. local development council,
promulgation of the LGC in 1991 was a move welcomed by most local health board, and local school board)  enabling direct
sectors of society, which finally transferred the responsibility for the citizen participation
delivery of basic services to the local government units, including - to promote local accountability and aswerability,
appropriate personnel, assets, equipment, programs and projects. specifically through recall and people’s initiative
There were already historical attempts to decentralize power and provisions
authority to local institution (e.g. enactment of LGC in 1983) but - one mechanism to lead to people empowerment
remained only in paper. - redefinition of notion of governance where it once
refereed mostly to basic and formal structures and
Basic Features of the LGC processes of government, and now recognized key role
1. Radical transformation of the nature of relationship between of the private sector, NGOs and Pos
the national government and the LGUs.
- transfer of fundamental responsibilities and Implementing Devolution
accountabilities - has not been easy because of a number of intervening factors
- increase of financial resources available hampering smooth implementation
- move to encourage active participation of the private 1. simultaneous conduct of legal and national elections in 1992
sector, NGOs and Pos leading to a Phil. contribution in 2. resistance of certain devolved agencies to devolution (mostly
redefining notion of “governance” DOH) and subsequent moves to recentralize health sector as
2. Responsibility for delivery of basic services that earlier were advocated by certain members of Congress
the responsibility of the national government… 3. unequal distribution of financial resources (IRA), as a result of
- health (field health, hospital services and other tertiary a not-so-studied formula among various levels of loc govts.,
services) with cities gaining most, while municipalities and provinces
- social services (social welfare) are not able to afford cost of devolution
- environment (community based forestry projects) 4. “lags” in release of IRA shares of LGUs, hence proposal that
- agriculture (agricultural extension and on-site research) these should be automatically appropriated to LGUs
- public works (funded by local funds) 5. lack of guidelines to define and clarify intergovernmental
- education (school building program) relations (national-local, local-local)
- tourism (facilities, promotion and development) 6. hesitance among NGOs and Pos to participate in local
- telecommunications services and housing projects (for governance due to continuing distrust between government
provinces and cities) sectos
- other services such as investment support 7. general lack of information about LGC
3. Responsibility for enforcement of certain regulatory powers
such as: Devolved Sectors
- reclassification of agricultural lands Health (DOH)
- enforcement of environmental laws Problems encountered – career paths of devolved personnel, non-
- inspection of food products and quarantine absorption by LGUs, lower salaries, lack of adequate and
- enforcement of Nat’l Building Code appropriate medicines, purchasing procedures, late deliveries…
- operation of tricycles Would have been recentralized but bill was vetoed by the
- processing and approval of subdivision plans President upon advice of LGUs and through leadership of DOH.
- establishment of cockpits and holding of cockfights Social Services (DSWD)
4. Increase in available financial resources to LGUs by

The Apocryphal Maggots:


Rainier, Chrisgel, Corina, Geoffry, Grace and Sylvie Blanche
The Flibbertigibbet Worms:
Golda, Gladys and Melyjane
CA VE AT: By simply r ea ding this re vi ew er a t the end o f the sem este r wi ll (hopef ully) guar antee you r pas sing this cou rse. D rink mode rat ely .
Amusin S C A r y
gly
THE C2005 LOCAL GOVERNMENT REVIEWER -7-
As a consequence of devolution of health personnel, LGUs are o Relationship between LGUs and NGOs/Pos
encouraged to appoint a social welfare officer in respective o Clarifying the Roles and Powers of Oversight Committee
localities.
DSWD has devolved substantial programs and projects to LGUs,
and several dep’t orders have been issued to support and Devolution Master Plan
operationalize this devolution process. Phase1: Changeover Phase (92-93) – involved transfer to LGUs of
Environment and Natural Resources(DENR) devolved functions, with corresponding assets and personnel
Selective devolution of forest management, protected areas and Phase2: Transition Phase (94-96) – national government agencies
wildlife, environment management, mines and geosciences (NGAs) and LGUs to institutionalize their adjustments to
management, and land management. decentralized schemes introduced by LGC
Issuance of several dept circulars and implementing guidelines Phase3: Stabilization Phase (97 onwards) – assumed that LGUs
(DAO) but DAO30 clearly shows continuing primacy of DENR in will have built adequate capacities in managing local affairs, and
implementation of DENR functions. Within such context, Pimentel NGAs will provide constant support and technical assistance to
actually proposed that power and authority of LGUs over LGUs
environment should be more specific and even more extensive.
Agriculture Devolution Works: Best Practices at the Local Level
Problems encountered – rejection of regular employees (131) Indications that devolution is working:
because of possible duplication of position/personnel at local level - celebrated cases of abuses by local officials
(administrative non-viability). But before implementation, there - innovativeness and creativity at the local level has been
have been duplication at the provincial level. Some LGUs engendered by the LGC
introduce innovations (appointment of devolved personnel as - NGOs and POs have also been encouraged to be active
provincial coop officer as in Bulacan) to address the problem of participants in the process of governance at the local level
possible duplication and importantly to underscore and implement - Partnerships between various sectors have been developed
a policy thrust to cooperatives.
DOA has prepared various guides and manuals on devolution to - Gantimpalang Panglingkod Pook (Galing Pook Program)
serve as fundamental references for LGUs in the process of where winners are selected based on following criteria: (1)
absorbing devolved agricultural services. effectiveness of service delivery (extent to which program
made good its promise), (2) positive socioeconomic/
The Interagency Oversight Committee environment impact (improvement the program made on the
- constituted to support implementation of LGC life in the community, and how much the community cared for
- includes: LGUs through their respective leagues, Bureau of the environment), (3) promotion of people empowerment
Local Government Finance of DOF, Department of Budget (how many in the community were encouraged to participate
and Management, and DILG in activities meant for the common good), 94) transferability
- would address specific problems encountered in the (likelihood of program’s ability to inspire other communities to
implementation of the Code referred to it (e.g. its ability to successfully adopt it)
respond to devolution-related issues given the realities and o On health services, environmental management, public
constraints of operating as an inter-agency committee with finance, peace initiatives, integrated approach to
practically no full time staff to provide technical support and development, sociocultural development, employment
assistance) generation/livelihood, productivity improvement
- some issues and concerns could already be acted upon o E.g. Taking Care of People and Environment in Negros
without having to wait for committee to meet as a whole (e.g. Oriental (construction of a Community Primary Hospital
transition action teams, resolved by existing issuances, in hinterlands of Negros Oriental with Community Based
policies, circulars, admin and exec orders, directives and Resource Management Approach empowering fisherfolk
guidelines)  issues not settled by transition action teams will to take lead in environment protection), Energizing the
be elevated to oversight committee for resolution Purok in Sampaloc, Quezon (KapitBisig Program),
- entirely possible for it to address issues and concerns that Saving the Mangroves in Kalibo, Aklan (reforestation of
would also help define nat’l-local relations, with emphasis on 50-hec swampland), Saving the Marikina River,
areas recommended for increased nat’l government Transforming Malalag into a Provincial Agro-Industrial
participation in local development Center in Davao del Sur, Acquiring a Complete
- issues mostly pertain to the need to clarify concerns including Equipment Pool in Munoz, Nueva Ecija, Floating Bonds
the powers and latitude of various levels of local for Low Cost Housing in Victorias, Negros Occidental,
governments, classified as: Improving the Productivity in Naga City, Sustaining
o Local Finance/Local Fiscal Administration Development through an Integrated Approach in
Guagua, Pampanga, and Enriching for a Brighter
o Local Legislation/Clarifying the Role/s of the Local
Tomorrow in Bulacan (enriching cultural and historical
Legislative Bodies
heritage of Bulacan)…
o Personnel Administration
o Brought about activities in implementation of capability
o Regulatory Powers of LGUs
building programs for local governments

The Apocryphal Maggots:


Rainier, Chrisgel, Corina, Geoffry, Grace and Sylvie Blanche
The Flibbertigibbet Worms:
Golda, Gladys and Melyjane
CA VE AT: By simply r ea ding this re vi ew er a t the end o f the sem este r wi ll (hopef ully) guar antee you r pas sing this cou rse. D rink mode rat ely .
Amusin S C A r y
gly
THE C2005 LOCAL GOVERNMENT REVIEWER -8-
 Local Government Academty (LGA) shift of The World BANK, Decentralization: Rethinking
methodology and approach to experiential learning Government, in World Development Report 1999/200:
 Lakbay Aral (study visits) Entering the 21st Century 107-124 (1999)

Concluding Statement Thesis: The success of decentralization (DC for brevity) depends
Local governments play a central role in any democratizing polity. on its design.
The local governments’ success and response to the basic needs
of the people is the success of the national government. The success (or failure) of DC affects:
The burden upon the shoulders of the local governments is indeed
heavy. This should tell the national government to extend as much 1. Political Stability
support to local governments in order for them to be more  When a country is divided along geographic or ethnic
responsive to the needs of the people. lines, DC provides an institutional mechanism for
While implementation may not be a smashing success, it is not a bringing opposition groups into a formal rule-bound
failure either. Significant inroads have been made in the devolution bargaining process (ex. South Africa and Uganda)
process to deconstruct the highly centralized institutions and
processes that have long characterized the Philippine politico- 2.Public Service Performance
administrative system. Arguments for DC...
- Local institutions and processes have been set in place  DC increases efficiency and responsiveness of gov’t
- Master Plan provides guideposts and milestones to ensure because...
sustained and progressive implementation of LGC  local elected leaders know their constituents better than
- Oversight Committee promises to be vigorously involved nat’l authorities
The support of various concerned agencies to increase LGUs
 physical proximity makes it easier for citizens to hold local
absorptive capacities, through preparation of guidelines, manuals
officials accountable for performance
for devolution, and even developing various capability building
programs with appropriate institution also augurs well for
devolution.  DC creates competition among local govts (LG for
Increasing the financial resources of LGUs still play a very brevity) to better satisfy citizen’s needs if population is
important and critical role where IRA is a major source. mobile.
- Proposals to reconfigure IRA distribution formula to eliminate
bias in favor of cities and conversely against provinces and ...but supporting evidence is scanty because the causal
municipalities should be seriously considered relationships are difficult to prove since...
- Pending bills in Congress to amend Code that would initially  Govt’s perform various functions under various
subtract cost of devolution before computing IRA share of circumstances
LGUs  efficiency and responsiveness can be hard to measure
 role of central government in distributing and redistributing
wealth and resources to favor the less endowed LGUs. Thus, how DC affects access and quality of public service
There are also various innovations to increase and stretch local depends on the way it is designed and implemented. What LG can
resources. achieve depend on resources and responsibilities they are given
- granting by LGC provisions of relatively broader taxing and the power of National Governments (NG for brevity) to
powers to LGUs override LG decisions.
- creative use by LGUs of powers available to them
o revenue generation strategies such as B-O-T method to 3. Equity
build public markets and facilities Whether or not DC exacerbates income differences depends on 2
o flotation of bonds to finance construction of public factors:
housing
 Devolution and Local Autonomy is all about: unleashing the
1) Horizontal equity – the LGs capacity to generate
creative powers and resources at the local level towards the funds so as to be able to deliver an equivalent level
general objective of developing self-reliance and lessen of services to their population.
dependence upon the central government (has been one reason  As the cost of providing public service vary, most
for the state of underdevelopment of local government units in the decentralized fiscal systems include equalization grants.
Philippines) A difficulty with these grants is that LG may differ in their
willingness to raise taxes. Further, the grants create an
Local government units in the Phils are undergoing fundamental incentive for LG to understate their tax bases or relative
structural and ideological transformation as a result of devolution, wealth in order to maximize transfers.
which may be better appreciated if placed within the proper
context. 2) Within-state equity – the LGs willingness to
redistribute income w/in its borders

The Apocryphal Maggots:


Rainier, Chrisgel, Corina, Geoffry, Grace and Sylvie Blanche
The Flibbertigibbet Worms:
Golda, Gladys and Melyjane
CA VE AT: By simply r ea ding this re vi ew er a t the end o f the sem este r wi ll (hopef ully) guar antee you r pas sing this cou rse. D rink mode rat ely .
Amusin S C A r y
gly
THE C2005 LOCAL GOVERNMENT REVIEWER -9-
 In most countries, income inequality is due more to make it in the interest of the LG and NG to cooperate with each
differences among individuals w/in a province than other.
among the provinces. Thus, even dramatic redistribution
across regions will have limited results unless targeting  Moderating regional influence on the NG
is improved within regions themselves. This, in turn, Such influence is dependent on 2 factors:
depends on the ability and willingness of LG to engage 1.
the way regional interests are incorporated
in redistribution, because studies show local officials and
in the national legislature w/c determines a
community groups are better placed to identify and
LG’s ability to pressure the NG to change the
reach the poor than the NG. Studies also show that LG
rules
differ in their responsiveness to the needs of the poor.
electoral arrangements matter
Thus, the success in targeting the poor requires a 2. strength of the national executive which
combination of national and subnational efforts. influences the NG ability to withstand such
Generally, the bulk of the funding needs to remain a NG pressure
responsibility, but the better information available to local depends on:
officials can be tapped by involving LG in the delivery a) the strength of the chief executive and
and management of social services. But NG needs to whether a clear majority emerges in
retain a monitoring role to ensure that redistributive parliament; and
goals are satisfied. b) the electoral system

4. Macroeconomic stability  Creating incentives for NG and LG to cooperate


 Fiscal DC reduces the NG control over public resources. Political parties play a crucial and often underestimated role in this
Deficit spending by LG can also thwart NG efforts to cool process (ex. US and Germany vs Canada).
the economy by restraining public expenditure.
 When revenues are decentralized before expenditure Institutions can be designed to promote a commonality of interests.
responsibilities, NG are forced to maintain spending The electoral system and the resulting party structure determine
level w/in a smaller resource base, resulting in large NG the degree to w/c the political system is nationally integrated.
deficits. By separating taxing and spending powers LG
incurs only a fraction of the political and financial costs of Note, there is no single best way to divide national political power
their expenditure. between NG and LG. Nor can single const’l provision ensure that
that NG and LG elites will find it in their interest to cooperate. But
From centralized to decentralized governance whatever the system is adopted, it must not make the NG a
prisoner of LG interests.
A decentralization program must adapt to the country’s prevailing
conditions; however experience yields some universal lessons b) The structure, functions and resources of LGs
such as:
 the need for a coherent set of rules to replace the hierarchical Structure and functions
system of governance. Fiscal Federalist framework and its practical
limits
A major challenge to DC is to instutionalize the balance of power
between the NG and LG. To weather this challenge, oral or written This traditional approach calls for a LG structure with several
rules must both protect and limit the rights of LG. Making these tiers, with each tier delivering those services that provide
rules explicit and reasonably permanent reduces uncertainty benefits or those residing in the jurisdiction.
and provides a common ground for all players in the political
process. To be sustainable, it must be “self-enforcing”-all parties This model identifies 3 roles for the public sector:
must believe they have more to gain by adhering to the rules than macroeconomic stabilization, income redistribution and
by breaking them. resource allocation [in case of market failure]. The model
assigns the stabilization role to the NG because it controls
3 Major categories/sets of Rules: monetary policy and has more scope to use fiscal policy than
LG. It also assigns income redistribution to NG since local
a) Balancing political power between attempts would result in inefficient population movements. The
central and local interests model assigns a significant role to LGs in allocating resources
because local politics can approximate the efficiencies of a
Since the rules governing the LG and NG are almost always market in the allocation of local public services.
established generally by the NG, the balance between them is
dependent on the influence of regional interests on the NG. The Recent literature has recognized that while the NG should
stability of such balance hinges on the designs of institutions that continue funding and designing redistribution efforts, LG are

The Apocryphal Maggots:


Rainier, Chrisgel, Corina, Geoffry, Grace and Sylvie Blanche
The Flibbertigibbet Worms:
Golda, Gladys and Melyjane
CA VE AT: By simply r ea ding this re vi ew er a t the end o f the sem este r wi ll (hopef ully) guar antee you r pas sing this cou rse. D rink mode rat ely .
Amusin S C A r y
gly
THE C2005 LOCAL GOVERNMENT REVIEWER - 10 -
often in a good position to implement and administer in the price of goods. Overall subnational taxes are seldom a large
standardized national policies. share of subnational revenues although there is scope in improving
local revenue collection.
2 practical obstacles: 1) in developing countries where land
and labor markets may not function well and the democratic The role of transfers
tradition is in its infancy, it is unrealistic to assume that people Their design is a critical factor in DC.
can move easily between jurisdictions or make their voices Transfers are needed to fund services LG provides on behalf of the
heard; 2) establishing separate tiers of service is costly and NG while local revenues ideally scover local expenditure.
poses serious coordination problems. Governments can use transfers to influence the sectoral pattern of
local expenditure by earmarking or disbursing transfers in the form
The structure of LG of matching grants.
Although the appropriate number of tiers of government and 3 variables: 1) amount to be distributed, 2) criteria for distributing
jurisdiction in each country varies depending on its characteristics, transfers among jurisdictions, 3) conditionaities imposed on the
all countries face the same trade-off between representation and use transfers.
cost. Trends in mature decentralized countries is the reduction of Transfers should be designed according to their objectives. Those
the number of subnational units, largely on the grounds of intended to finance functions that the municipal government is
efficiency and cost. Among poor countries, the trend is otherwise performing on behalf of the NG should be earmarked. In practice,
perhaps in part because a block grant available to each LG most transfers take the form of block grants.
creates an incentive to divide jurisdictions. Some basic principles are applicable to all types of transfers:
Transfers should be determined as openly, transparently and
Clarifying the allocation of functions and allowing for shared objectively as possible. They should be kept reasonably stable
functions from year to year so that LG can plan their budgets. And they
Providing services centrally creates economies of scale and should be distributed on the basis of predetermined rules. This
captures externalities but at the cost of imposing a common policy eliminates the uncertainty and bargaining that often plague
on populations with varied preferences and priorities. This trade- intergovernmental fiscal relations.
off, w/c is the basis of the fiscal federalist approach, guides some
of the choices that must be made in allocating functions. Controlling subnational debt
Such responsibility-sharing arrangements are complex. But they In principle, subnational borrowing is a private transaction but the
work well when they are clear and each tier’s responsibilities are NG is often drawn reluctantly into the transaction because of its
relatively well-defined, and when the regulatory framework responsibility for the stability of the financial system.
anticipates that LGs are sometimes NG agents and sometimes An alternative to private financing is for the NG to provide long
principals acting on their own. Without clarity and an appropriate term credit, but the repayment record is poor. Loan allocation tends
regulatory framework, there can be no accountability. to become politicized while debt collection is lax, with national
taxpayers ultimately bearing the burden.
Assigning and controlling resources Generally, private financing is either already the primary sources of
It may perhaps be the thorniest issue in DC, experience LG credit or is meant to eventually replace NG financing. This
nevertheless provides 2 lessons in this area: requires developing means to protect the NG and the national
1) LG needs resources commensurate with their responsibilities financial system from exposure to excessive subnational debt.
4 approaches:
2) LG authorities must operate under firm budget constraints, so
that they do not spend or borrow excessively in the 1) market discipline
expectation of a NG bailout • To be effective, a laissez-faire approach requires that a
Guiding principle: finance should follow function because not only number of conditions hold-the most important being the
should resources be commensurate with what they fund, but also credibility of the NG commitment not to intervene. It also
because the type of revenue used affects consumer behavior and requires avoiding situations in which the NG would be forced
results in different patterns of incidence. to intervene.
2) cooperation between LG and NG to decide what constitutes
Overall, the appropriate structure of subnational finance depends an appropriate level of indebtedness
on the functions that have been assigned to each tier of 3) direct regulation of subnational borrowing
government.
• Subject to political bargaining and are generally at odds
with the end towards DC. Also, they make it even more
Certain forms of taxation are appropriate for financing local
difficult for the CG to refuse to intervene and rescue LG.
services with benefits that cant be confined to individual
But administrative controls are appropriate for external
consumers and must fall on residents of the jurisdiction and be a
borrowing because a LG behavior on the intl market could
direct tax (ex. realty tax). But direct taxation in developing
have contagion effects on the ratings of other national
countries often yield limited revenues. The income tax is of limited
borrowers and because managing the external debt is the
use where most of the economy operates informally. To
macroeconomic responsibility of the NG.
compensate, most rely on various forms of business taxation,
which are politically easier to impose since the effects are hidden 4) Rule-based controls (ex. ceilings on debt service ratios)

The Apocryphal Maggots:


Rainier, Chrisgel, Corina, Geoffry, Grace and Sylvie Blanche
The Flibbertigibbet Worms:
Golda, Gladys and Melyjane
CA VE AT: By simply r ea ding this re vi ew er a t the end o f the sem este r wi ll (hopef ully) guar antee you r pas sing this cou rse. D rink mode rat ely .
Amusin S C A r y
gly
THE C2005 LOCAL GOVERNMENT REVIEWER - 11 -
• more transparent and less subject to political interference Dependent on the strength of community organizations and their
especially when they set global limits that mimic markets. ability to organize
• Fundamentally however, rules and control will be Local officials must be willing to tap them
ineffective unless accompanied by market discipline and a Civil society’s formal participation has its limits: Active civic
credible no-bailout pledge by the NG. organizations are need to draw on local traditions; not always
effective and may only reflect the views of a narrow segment of the
Central regulation of LG population.
When NG in decentralizing countries tend to compensate for their
loss of direct control by steeping up their regulation of LG tends to b) Civil society and political parties
be counterproductive if NG with limited knowledge of local
conditions micromanage local functions of if they impose costs Once democratic movements achieve their immediate goals, the
they are not prepared to finance. civic energy that fueled them often dissipates. Political parties can
Personnel matters are one area in which central regulation is help maintain a continuing link between civil society and
undesirable. If a NG is concerned about nepotism or overstaffing at government. Parties aggregate the demands of a dispersed
local levels, it can address them in other ways. But NG population, represent political interests, recruit and train new
involvement in personnel matters also reflects the power of public candidates for office, ensure electoral competition, and form
sector unions and their ability to organize nationally. governments. They can help organize minorities and the poor and
NG regulation remain appropriate in a wide range of other facilitate their participation in the formal electoral process.
circumstances. When LG act as agents of the NG, regulation and
monitoring are needed to enforce national mandates and 3. Developing an effective local administration
standards. Regulation is also essential to ensure validity of the
local electoral process and to address conflicts between units of Both NG and LG can take measures to improve the effective ness
subnational government. But a free press, improved access to of local administration: 1) When NG has decentralized
information, the growth of democracy at subnational levels and the responsibilities, it can also devolve the appropriate staff; 2) LG
increasing ability of local interest groups to monitor LG should be free to hire, fire and offer appropriate packages so that
performance decrease the need for central regulation. they can attract capable local officials; 3) Privatization can reduce
the number of skilled administrators needed by LG since
c) Making LG accountable privatized services require only monitoring and regulation.
Decentralization itself, by giving LG greater responsibilities and
3 sets of complementary measures should be pursued: control over resources will then increase their incentives to invest
further in their own administrative capabilities.
1. Adopting effective electoral rules
Making elections highly visible events, facilitating participation and Policies for the transition
demonstrating that votes matter will affect electoral outcomes in
any society. DC typically takes place during periods of political and economic
upheaval. Even when DC occurs in a less dramatic context,
a) rules to improve visibility, participation, and expected questions of timing and strategy still arise
payoffs
Hold concurrent local and national elections, although this Synchronizing the elements of reform
approach carries with it the risk of national issues overshadowing The recent history of DC illustrates the dangers of not sequencing
local concerns appropriately.
Mayor or governor elected directly by the whole constituency, while
state assembly members or municipal councilors are elected by 1. Put expenditure and revenue rules in place before political
district or neighborhoods. liberalization

b) rules that promote effective governance 2. Decentralize a function and its corresponding revenue source
Effective governance requires stable coalitions and an executive simultaneously
with reasonably strong and clear powers
3. Decentralize the needed management controls
Majority voting rather than proportional represenation
Separating executive and legislative branches of LG and electing Demonstrating the hard budget constraint
the chief executive directly NG must demonstrate early on that they are committed to
imposing a hard budget constraint on LG. The mere possibility of
2. Harnessing civil society NG bailout can prompt excess spending and deficit financing at the
subnational level.
a) Civil society and formal political participation
What lessons for the future?

The Apocryphal Maggots:


Rainier, Chrisgel, Corina, Geoffry, Grace and Sylvie Blanche
The Flibbertigibbet Worms:
Golda, Gladys and Melyjane
CA VE AT: By simply r ea ding this re vi ew er a t the end o f the sem este r wi ll (hopef ully) guar antee you r pas sing this cou rse. D rink mode rat ely .
Amusin S C A r y
gly
THE C2005 LOCAL GOVERNMENT REVIEWER - 12 -
A system based on rules produces better results than one that is 1. Short Title: Section one is the short title of the code. Its long
not. Explicit rules setting out the division of functional responsibility
title is An Act Providing for a Local Government Code of
among levels of government reduce ambiguity and increase
1991
political accountability. They also provide a framework within which
interest groups can compete and negotiate without resorting to
violence 2. Effectivity Date: Under Section 536, the code shall take effect
Some rules work better than others. Revenues need to be on January 1, 1992, after its complete publication in at least
decentralized at the same time as expenditures so that finance one newspaper of general circulation (Malaya)
follows function. A “hands-off” attitude when LG defaults on their
loans may be more important in controlling debt than the most 3. Defn of Local Government Unit: is a political subdivision of a
comprehensive set of regulations and controls. nation or state which is constituted by law and has substantial
LG with multiple tiers and many small units are likely to have high control of local affairs
administrative overhead costs.
Section 2
Comments:
1. Reiteration of Constitutional Policies: It was the intent of
Fusion Sex (Herb Goldberg, Hazards of Being Male) Congress that LGUs shall possess genuine and meaningful
local autonomy to accelerate their fullest development as self-
Recently I have been exploring the possibility of different kinds reliant communities and make them more effective partners in
of intensities of male orgasms and sexual experiences. I brought the attainment of national goals
the issue up at a marathon psychotherapy session two years ago
and discovered that not all but a number of men had 2. Spirit of Liberty in Local Autonomy: Supreme Court ruled
experienced what I have come to term “fusion sex.” in the case of San Juan v Civil Service Commission (196
The experience of fusion sex is one of an intense, totally un-self- SCRA 69) that the national officials should not only comply
conscious sexual coming together during which the male is not with the constitutional provisions on local autonomy but
focusing on or aware of having sex per se but is simply a part of should also appreciate the spirit of liberty upon which these
a wholly spontaneous, ecstatic union or fusion with the female, provisions are based
one that often brings him to tears of joy.
In fusion sex there is the phenomenon of a seemingly endless 3. Meaning of Autonomy: came from the Greek language
potency, lasting sometimes for an entire weekend or several which means to live under one's own laws. Applied to the
days during which time he remains in bed making love Philippine situation, it means the power of local government
continually. Men who have reported fusion sex to me describe units to enjoy limited self-government as defined by law.
the phenomenon of ejaculating and then almost immediately
becoming erect again. They may have as many as twelve to 4. Decentralization: The principle of autonomy does not make
fifteen orgasms during a weekend’s experience of fusion sex. local govt's sovereign within the state. It simply means
Indeed, the cup overfloweth. decentralization (Basco v PAGCOR). It is a means to wean
(to be continued…) LGUs from overdependence on the central government
However, Decentralization is more akin to "Deconcentration"
whereby central government offices are transferred to regions
but remain under control of the central govt. Example:
regional offices
Lo cal G ov er nm en t, D ece nt raliz at ion, Aut onom y
5. Devolution: this is the term that more closely approximates
The Local Government Code what the Local Government Code does for LGUs. It is defined
Pimentel, (pp 13-17) by the code as "the acts by which the National Government
confers power and authority upon the various LGUs to
** Books, Titles, Chapters, and Sections provided herein pertain to perform specific functions and responsibilities"
those in the Local Government Code. Please refer accordingly. • Process proceeds from National government to LGUs.
Hence, the devolution is made possible without any
Book I General Provisions intermediary.
Title One: Basic Principles
Chapter 1. The Code: Policy and Application • Devfolution of power does not stop at the level of any one
LGU. Rather, it seeps down to all levels. Example: funds go
Section 1 directly to the barangay, not through a municipal mayor or
Comments: treasurer as was the case before.

The Apocryphal Maggots:


Rainier, Chrisgel, Corina, Geoffry, Grace and Sylvie Blanche
The Flibbertigibbet Worms:
Golda, Gladys and Melyjane
CA VE AT: By simply r ea ding this re vi ew er a t the end o f the sem este r wi ll (hopef ully) guar antee you r pas sing this cou rse. D rink mode rat ely .
Amusin S C A r y
gly
THE C2005 LOCAL GOVERNMENT REVIEWER - 13 -
must be deemed to have been repealed by Sec2 Art III of the
• Powers LGUs may exercise: Provisional Constitution.

1) those expressly conferred Held: The designations made by respondent were not valid. The
provision in the Provisional Constitution must be deemed to have
2) those implied from those expressly granted been overtaken by Section 27, Article XVIII of the 1987
Constitution. The 1987 Constitution was ratified in a plebiscite on
3) those necessary, appropriate, or incidental for efficient February 2, 1987. By that date, thus, the Provisional Constitution
and effective governance must be deemed to have been superceded. Hence, the OIC
Governor could no longer rely on a provision of this superceded
4) essential to the promotion of the general welfare of their constitution to designate respondents to the elective positions
inhabitants occupied by petitioners. Petitioners must now be held to have
acquired security of tenure
6. Accountable Local Officials: this section emphasizes that
Sarmiento dissents: the Constitution took effect on February 11,
local govt officials must be accountable for their acts, 1987, not February 2, 1987.
otherwise they can be recalled even during their term.

7. Initiative, Referendum, and Recall: These new powers are San Juan vs. Civil Service Commission (1991)
now available to the people, being exercised by registered
voters residing within a territory of an LGU under the Facts:
circumstances provided by the code. Gov. San Juan appointed Santos as Acting PBO (Provincial
Budget Officer) of Rizal Province and informed DBM Reg.IV Dir.
8. Consultation Required: consultation is now required before and asked him to endorse the appointment. However, Dir.
any project is implemented, not only with the LGUs but also recommended the appointment of Almajose as PBO since she is
with the NGOs and People's Organizations. the only CPA among the contenders. DBM Usec. signed the
appointment papers of Almajose w/out the knowledge of Gov. San
Juan. The new Reg. IV Dir. informed Gov. San Juan that Santos
9. Mandatory Consultations: Not only central government was not qualified and asked that he submit 3 other nominees.
agencies but even government owned or controlled However, Gov. San Juan learned of Almajoses’s appointment by
corporations should undergo such consultations (Art 54, the Usec and filed a letter-protest with the DBM Secretary. DBM
Rules Implementing the Local Government Code of 1991) ruled that letter-protest is not meritorious. Motion for Recon was
denied. Thus, Gov. San Juan appealed to Civil Service
10. Ecology and Environment: The maintenance and Commission (CSC) which issued a resolution dismissing Gov. San
preservation of a sound ecological balance and clean Juan’s claim.
environment are salient concerns in the code, tempering
modernization and development. Held: The DBM cannot appoint anyone it wants when the
recomendee of the Governor is unqualified.

The issue involves the application of a most important


De Leon vs. Esguerra (1987)
constitutional policy and principle, that of local autonomy. The clear
mandate on local autonomy must be obeyed. Where a law is
Facts: capable of two interpretations, one in favor of centralized
Pursuant to the Barangay Elections Act of 1982,barangay elections power in Malacañang and the other beneficial to local
were held and De Leon was elected as barangay captain, while autonomy, the scales must be weighed in favor of autonomy.
other petitioners were elected as barangay councilmen. De Leon The exercise by local gov’t of meaningful power has been a goal
received a memorandum antedated Dec 1, 1986 but signed by since the turn of the century.
OIC Governor Esguerra on Feb 8, 1987 designating Magno as
Brgy Capt of Brgy Dolores. a similar memorandum was signed by When CSC interpreted the recommending power of the Provincial
the OIC Governor for the replacement of the councilmen of Brgy Governor as purely directory, it went against the letter and spirit of
Dolores. Petitioners want these memoranda declared null and void the constitutional provisions on local autonomy. If the DBM
because there term has not yet ended and becaue of the Secretary jealously hoards the entirety of budgetary powers and
ratification of the 1987 Const, OIC had no longer authority to ignores the right of local governments to develop self-reliance and
replace them Repsondents aver that the terms of office of elective resoluteness in the handling of their own funds, the goal of
and appointive officials were abolished and that petitioners meaningful local autonomy is frustrated and set back.
continued in office by virtue of a holdover capacity and not
because their term of 6 years had not yet expired; and that the The DBM may appoint only from the list of qualified
provision in the Brgy Election Act providing for the 6-year term recommendees nominated by the Governor. If none is qualified, he
must return the list of nominees to the Governor explaining why no

The Apocryphal Maggots:


Rainier, Chrisgel, Corina, Geoffry, Grace and Sylvie Blanche
The Flibbertigibbet Worms:
Golda, Gladys and Melyjane
CA VE AT: By simply r ea ding this re vi ew er a t the end o f the sem este r wi ll (hopef ully) guar antee you r pas sing this cou rse. D rink mode rat ely .
Amusin S C A r y
gly
THE C2005 LOCAL GOVERNMENT REVIEWER - 14 -
one meets the legal requirements and ask for new recommendees
who have the necessary eligibilities and qualifications. Facts:
Basco et al petition seeking to annul the Philippine Amusement
and Gaming Corporation (PAGCOR) Charter (PD 1869). They
Ganzon vs. CA (1991) claim that PD 1869 is unconstitutional since it constitutes an
intrusion and waiver of Manila's right to impose taxes and license
Facts: fees, which is recognized by law; This is in contravention of the
Series of administrative complaints (10) were filed against Mayor constitutionally enshrined principle of local autonomy;
Ganzon by various city officials sometime in 1988. Mayor Ganzon
answered and cases were set for hearing (where a series of Held: Their contention is without merit because: (a) The City of
postponements occurred). *Sec. Santos (of Department of Local Manila, being a mere Municipal corporation has no inherent right to
Government) issued several consecutive preventive suspension impose taxes. Thus, "the Charter or statute must plainly show an
orders. Ganzon obviously complains saying that he was denied intent to confer that power or the municipality cannot assume it".
due process and President has no power to investigate/suspend Its "power to tax" therefore must always yield to a legislative act
local officials. which is superior having been passed upon by the state itself
which has the "inherent power to tax". (b) The Charter of the City
Held: The 1987 Constitution did not intend to divest the legislature of Manila is subject to control by Congress. And if Congress can
of its right, or the President of his prerogative as conferred by grant the City of Manila the power to tax certain matters, it can also
existing legislation, to provide administrative sanctions against provide for exemptions or even take back the power. (c) The City
local officials. The deletion was meant to stress, sub silencio, the of Manila's power to impose license fees on gambling, has long
objective of framers to strengthen local autonomy by severing been revoked. (d) Local governments have no power to tax
congressional control of its affairs, as observed by CA, like the instrumentalities of the National Government [note this has already
power of local legislation… BUT it did nothing more. Insofar as been dumped by SC in later cases under local taxation]. The
existing legislation authorizes the President (through Sec of Local power of local government to "impose taxes and fees" is always
Government) to proceed against local officials administratively, subject to "limitations" which Congress may provide by law. Since
Consti contains no prohibition. PD 1869 remains an "operative" law until "amended, repealed or
revoked" (Sec. 3, Art. XVIII, 1987 Constitution), its "exemption
Although President, through Sec Santos is not precluded from clause" remains as an exception to the exercise of the power of
exercising a legal power, it appears that Santos is exercising it local governments to impose taxes and fees. It cannot therefore be
oppressively and with a grave abuse of discretionby (a) inflicting on violative but rather is consistent with the principle of local
Ganzon successive suspensions when he had sufficient time to autonomy.
gather necessary evidence to build a case without suspending him
a day longer and (b) he has been cracking down on Ganzon
piecemeal – to pin him down ten times (10complaints) when he
Magtajas vs. Pryce Properties Corp. (1994)
could have pursued a consolidated effort
Facts:
RULES laid down by SC:
PAGCOR decided to expand its operations to Cagayan de Oro
1. Local autonomy, under Consti, involves a mere
city (CDO) for which it leased a portion of a building owned by
decentralization of administration, not of power, in which
Pryce for the operation of a casino. SP of CDO enacted
local officials remain accountable to the central government
ordinances (1) prohibiting the issuance of business permit and
in the manner the law may provide.
canceling existing business permit to any establishment for the
2. The new Consti does not prescribe federalism.
using and allowing to be used its premises or portion thereof for
3. The change in constitutional language (w/respect to
the operation of a casino and (2) prohibiting the operation of
supervision clause) was meant to deny legislative control
casino and providing a penalty for violation thereof. Pryce and
over local governments; it did not exempt the latter from
PAGCOR assailed the ordinances before the CA. CA declared
legislative regulations provided regulation is consistent with
ordinances invalid and issued writ of prohibition.
the fundamental premise of autonomy.
4. Since local governments remain accountable to the
Held: Enacted ordinances were invalid. For an ordinance to valid:
national authority, the latter may, and in the manner set
(1) it must not contravene the constitution or any statute; (2) it
forth therein, impose disciplinary action against local
must not be unfair and oppressive; (3) it must not be partial or
officials;
discriminatory; (4) it must not prohibit but may regulate; (5) it must
5. “Supervision” and “investigation” are not inconsistent
be general and consistent with public policy; (6) it must not be
terms; “investigation” does not signify “control” (which
unreasonable. As to the contention that it is allowed by Sec 458 to
President does not have).
prohibit all kinds of gambling, it must be noted that in Sec 458,
since the word "gambling" is associated with "and other prohibited
games of chance," the word should be read as referring only to
Basco vs. Pagcor (1991) illegal gambling which, like other prohibited games of chance, must

The Apocryphal Maggots:


Rainier, Chrisgel, Corina, Geoffry, Grace and Sylvie Blanche
The Flibbertigibbet Worms:
Golda, Gladys and Melyjane
CA VE AT: By simply r ea ding this re vi ew er a t the end o f the sem este r wi ll (hopef ully) guar antee you r pas sing this cou rse. D rink mode rat ely .
Amusin S C A r y
gly
THE C2005 LOCAL GOVERNMENT REVIEWER - 15 -
be prevented or suppressed. Such interpretation is in accordance conformity with the oversight function of the Conversion
with the rule of noscitur a sociis (a word or phrase should be Authority. cdtai (b) In case of conflict between the Subic
interpreted in relation to, or given the same meaning of, words with Authority and the local government units concerned on matters
which it is associated.) affecting the Subic Special Economic Zone other than defense
and security, the decision of the Subic Authority shall prevail.
Further, implied repeals are not lightly presumed in the absence of
clear and unmistakable showing of such intention. A reading of Some Horror Movie Quotes:
the entire repealing clause would show that it painstakingly
mentions the specific laws or the parts thereof which are repealed They will say that I have shed innocent blood. What's blood for,
or modified by the LGC. PD 1869, which created PAGCOR was
if not for shedding? (Candyman)
not one of them. There is no sufficient indication of an implied
repeal of PD 1869 as later laws (RA 7309 and RA 7648) even
mentioned PAGCOR as the source of its funding. Here we go, the world is spinning. When it stops, it's just
beginning. Sun comes up, we laugh and we cry. Sun goes down,
Lo cal G ov er nm en ts , Ad min is tr at iv e R egion s, Aut onomo us and then we all die. (The Ring)
Regio ns
Kincaid: "I'll see you in hell."
RA7227: An Act Accelerating The Conversion Of Military Freddy:"Tell 'em Freddy sent ya." (a nightmare on Elmstreet 4)
Reservations Into Other Productive Uses, Creating The
Bases Conversion And Development Authority For The
Purpose, Providing Funds Therefor And For Other Lo cal G ov er nm en ts , Ad min is tr at iv e R egion s, Aut onomo us
Purposes Regio ns

SEC12. Subic Special Economic Zone. - Subject to the Abbas vs. COMELEC (1989)
concurrence by resolution of the sangguniang panlungsod of the
City of Olongapo and the sangguniang bayan of the Facts:
Municipalities of Subic, Morong and Hermosa, there is hereby Abbas et. al. prayed that (1) the COMELEC be enjoined from
created a Special Economic and Free-port Zone consisting of conducting the plebescite and the DBM from releasing funds for
the City of Olongapo and the Municipality of Subic, Province of said purpose; and (2) RA 6734 (ARMM act) be declared
Zambales, the lands occupied by the Subic Naval Base and its unconstitutional.
contiguous extensions as embraced, covered, and defined by
the 1974 Military Bases Agreement between the Philippines and Held: RA 6734 is constitutional.
the United States of America as amended, and within the Petitioners’ allegation that RA 6734 is unconstitutional because it
territorial jurisdiction of the Municipalities of Morong and unconditionally creates an autonomous region in Muslim Mindanao
Hermosa, Province of Bataan, hereinafter referred to as the is untenable since Art 2 Sec 1 itself refers to Sec 18 Art X of Const
Subic Special Economic Zone whose metes and bounds shall which sets forth the conditions necessary for the creation of the
be delineated in a proclamation to be issued by the President of autonomous region. Also there is a specific provision in the law's
the Philippines. Within thirty (30) days after the approval of this transitory provisions which incorporates substantially the same
Act, each local government unit shall submit its resolution of requirements embodied in the Consti. These clearly indicate that
concurrence to join the Subic Special Economic Zone to the the creation of the same shall take place only in accord with the
office of the President. Thereafter, the President of the constitutional provisions.
Philippines shall issue a proclamation defining the metes and
bounds of the Zone as provided herein. The creation of the autonomous region is made to depend on the
The abovementioned zone shall be subject to the following will of the majority in each of the constituent units. Such is clearly
policies: the intention of the framers for if they intended to get the majority
xxx of the votes cast, they would simply have adopted the same
(i) Except as herein provided, the local government units phraseology as that used for the ratification of the Const where the
comprising the Subic Special Economic Zone shall retain their phrase "by the constituent units" is not found.
basic autonomy and identity. The cities shall be governed by
their respective charters and the municipalities shall operate and As to petitioner’s allegation that Congress has expanded the scope
function in accordance with Republic Act No. 7160, otherwise of ARMM which const itself prescribed to be limited, this is a
known as the Local Government Code of 1991. political issue. Any review of such would have to go into the
wisdom of the law which is violative of the principle of separation of
SEC14. Relationship with the Conversion Authority and the powers.
Local Government Units. - (a) The provisions of existing laws,
rules and regulations to the contrary notwithstanding, the Subic As to the allegation that ARMM Act violates the exercise of free
Authority shall exercise administrative powers, rule-making and Religion, there is no actual controversy between the litigants exist
disbursement of funds over the Subic Special Economic Zone in therefore the Court cannot be called upon to resolve what is

The Apocryphal Maggots:


Rainier, Chrisgel, Corina, Geoffry, Grace and Sylvie Blanche
The Flibbertigibbet Worms:
Golda, Gladys and Melyjane
CA VE AT: By simply r ea ding this re vi ew er a t the end o f the sem este r wi ll (hopef ully) guar antee you r pas sing this cou rse. D rink mode rat ely .
Amusin S C A r y
gly
THE C2005 LOCAL GOVERNMENT REVIEWER - 16 -
merely perceived as a potential conflict between the provisions of be embodied in other statutes on the same subjects as that of the
the muslim code and national law. challenged legislation.

As to the allegation that it grants the President the power not With respect to the power to merge existing administrative regions,
conferred by the Constutution, what is referred to in RA 6734 is the the standard is to be found in the same policy underlying the grant
merger of administrative region which are not territorial and to the President in R.A. No. 5435 of the power to reorganize the
political subdivisions. While the power to merge administrative Executive Department: "to promote simplicity, economy and
regions is not expressly provided for in the Consti, it is a power efficiency in the government to enable it to pursue programs
which has been traditionally lodged with the President to facilitate consistent with national goals for accelerated social and economic
the exercise of the power of general supervision over local development and to improve the service in the transaction of the
governmnets. public business."
Indeed, as the original 11 administrative regions were established
Further, the questioned provision requiring an oversight committee in accordance with this policy, it is logical to suppose that in
does not provide for a different date of effectivity. Neither would it authorizing the President to "merge by administrative
be an impediment as its creation is aimed at effecting a smooth determination the existing regions" in view of the withdrawal from
transition period for the regional government. some of those regions of the provinces now constituting the
Autonomous Region, the purpose of Congress was to reconstitute
the original basis for the organization of administrative regions.
Chiongbian vs. Orbos (1995)

Facts: Cordillera Broad Coalition vs. COA (1990)


ARMM Act, called for a plebiscite to be held. In the plebiscite held,
4 provinces (Lanao del Sur, Maguindanao, Sulu and Tawi-Tawi) Facts:
voted in favor of creating an autonomous region. In accordance After the 1996 EDSA Revolution, Balweg, broke off on ideological
with the constitutional provision, these provinces became the grounds from the CPP-NPA. After President Aquino was installed
ARMM. Pursuant to a provision in the ARMM Act those who voted into office, she advocated a policy of national reconciliation. The
against shall remain in the existing administrative regions: Cordillera People’s Liberation Army (CPLA) heeded this call.
Provided, however, that the President may, by administrative Aqiuno and Balweg arrived at a joint agreement to draft an
determination, merge the existing regions. Pursuant to the Executive Order to create a preparatory body that could perform
authority granted by this provision, Pres. Aquino issued an E.O. policy-making and administrative functions and undertake
that reorganized the administrative regions of Mindanao. The consultations and studies leading to a draft organic act for the
petitioners who were members of Congress representing the Cordilleras. Pursuant to the joint agreement, E.O. 220, creating the
various legislative districts involved in the plebiscite, wrote Pres. Cordillera Administrative Region (CAR) was signed into law.
Aquino protesting E.O. No. 429. As their protest went unheeded, During the pendency of this case, R.A. No. 6766 (Organic Act of
petitioners brought this suit for certiorari and prohibition. CAuR) was enacted and signed into law. The Act recognizes the
CAR and the offices and agencies created under E.O. No. 220 and
Held: The power to merge administrative regions is an executive its transitory nature is reinforced. Petitioners contend that the
function. Thus the creation and subsequent reorganization of issuance of the EO pre-empted Congress’ task.
administrative regions have been by the President pursuant to
authority granted to him by law. In conferring on the President the Held: NO.
power to merge by administrative determination the existing EO 220 does not create the autonomous region contemplated in
regions following the establishment of ARMM, Congress merely the Constitution. It merely provides for transitory measures in
followed the pattern set in previous legislation dating back to the anticipation of the enactment of an organic act and the creation of
initial organization of administrative regions in 1972. While the an autonomous region. In short, it prepares the ground for
power to merge administrative regions is not expressly provided for autonomy. This does not necessarily conflict with the provisions of
in the Constitution, it is a power which has traditionally been the Constitution on autonomous regions. The complex procedure
lodged with the President to facilitate the exercise of the power of for the creation of an autonomous region in the Cordilleras will take
general supervision over local governments. (Abbas v. time. The President, in 1987 still exercising legislative powers, as
COMELEC). The regions themselves are not territorial and political the first Congress had not yet convened, saw it fit to provide for
divisions like provinces, cities, municipalities and barangays but some measures to address the urgent needs of the Cordilleras in
are mere groupings of contiguous provinces for administrative the meantime that the organic act had not yet been passed and the
purposes. The power conferred on the President is similar to the autonomous region created. These measures are in E.O. No. 220,
power to adjust municipal boundaries which has been described and they do not violate the Constitution.
as "administrative in nature."
The bodies created by E.O. No. 220 do not supplant the existing
There was a standard to guide President’s discretion. A legislative local governmental structure, nor are they autonomous
standard need not be expressed. It may simply be gathered or government agencies. They merely constitute the mechanism for
implied. Nor need it be found in the law challenged because it may an "umbrella" that brings together the existing local governments,

The Apocryphal Maggots:


Rainier, Chrisgel, Corina, Geoffry, Grace and Sylvie Blanche
The Flibbertigibbet Worms:
Golda, Gladys and Melyjane
CA VE AT: By simply r ea ding this re vi ew er a t the end o f the sem este r wi ll (hopef ully) guar antee you r pas sing this cou rse. D rink mode rat ely .
Amusin S C A r y
gly
THE C2005 LOCAL GOVERNMENT REVIEWER - 17 -
the agencies of the National Government, the ethno-linguistic Effectivity of the Code
groups or tribes, and NGOs in a concerted effort to spur
development in the Cordilleras. Sec5, LGC: Rules of Interpretation – In the interpretation of the
provisions of this Code, the following rules shall apply:
Neither did E.O. 220 contravene the Constitution by creating a new xxx
territorial and political subdivision. The CAR is not a public (d) Rights and obligations existing on the date of effectivity of
corporation or a territorial and political subdivision. It does not have this Code and arising out of contracts or any other source of
a separate juridical personality, unlike provinces, cities and prestation involving a local government unit shall be governed
municipalities. Neither is it vested with the powers that are by the original terms and conditions of said contracts or the law
normally granted to public corporations (the power to sue and be in force at the time such rights were vested; and
sued, the power to own and dispose of property, the power to xxx
create its own sources of revenue, etc.). The CAR was created
primarily to coordinate the planning and implementation of Sec536, LGC: Effectivity Clause – This Code shall take effect on
programs and services in the covered areas. January 1st, 1992, unless otherwise provided herein, after its
complete publication in at least (1) newspaper of general
The CAR is in the same genre as the administrative regions circulation.
created under the Reorganization Plan, albeit under E.O. No. 220.
The operation of the CAR requires the participation not only of the
line departments and agencies of the National Government but
also the local governments, ethno-linguistic groups and NGOs in
Evardone vs. COMELEC (1991)
bringing about the desired objectives and the appropriation of
funds solely for that purpose.
Facts:
In 1990, COMELEC issued Resolution No 2272 embodying rules
Ordillo vs. COMELEC (1990)
and regulations on the recall of elective provincial, city, and
municipal officials. Pursuant to the rules under Resolution No.
Facts: 2272, COMELEC issued Resolution No. 90-0557 approving
A plebiscite was conducted in the provinces of Benguet, Mountain Election Registrar's recommendation to hold siging of petition for
Province, Ifugao, Abra and Kalinga-Apayao and Baguio City cast recall of Mayor Evardone. Evardone filed before the SC petition for
their votes in a plebiscite pursuant to R.A. No. 6766 (Oragnic Act of prohibition. His ground: Art X, Section 3 of the 1987 Consti
CAR). The COMELEC results of the plebiscite showed that only repealed BP Blg 337 (Local Government Code) in favor of one TO
the Ifugao Province wanted the CAR. DOJ Sec. issued a BE ENACTED by Congress. Since there was no LGC during the
memorandum for the President reiterating the COMELEC period material to the case, COMELEC has no basis to promulgate
resolution and provided that since only the provinces and city Resolution No.2272 and the recall proceedings are premature.
voting favorably shall be included in the CAR, the province of
Ifugao being the only province which voted favorably will alone, HELD: Resolution is constitutional. ART XVIII, Section 3 of the
legally and validly constitute the CAR. Ordillo et. al, then, 1987 Constitution provides that all existing laws not inconsistent
complains. with the Constitution shall remain operative until amended,
repealed, or revoked. On the other hand, the Local Government
Held: Ifugao cannot be the CAR alone. Code of 1991 will take effect only on January 1, 1992. Hence BP
The keywords in Art. X, Sec. 15, (Const) — provinces, cities, Blg. 337 is still the law applicable to the present case. BP 337
municipalities and geographical areas connote that "region" is to contains no special provision on the manner of conducting
be made up of more than one constituent unit. The term "region" elections of the recall of local officials. Thus, pursuant to the rule-
used in its ordinary sense means 2 or more provinces. This is making power vested in COMELEC, it promulgated Resolution No.
supported by the fact that the 13 regions into which the Philippines 2272 in 1990.
is divided for administrative purposes are groupings of contiguous
provinces. Ifugao is a province by itself. To become part of a
region, it must join other provinces, cities, municipalities, and
geographical areas. It joins other units because of their common
Secretary of Health vs. CA (1995)
and distinctive historical and cultural heritage, economic and social
structures and other relevant characteristics. The Constitutional
requirements are not present in this case. Facts:
Administrative complaint was filed with the DOH against private
respondent Administrative Officer Sibbaluca for grave misconduct,
 There is no Cordillera Autonomous Region yet since 2 dishonesty, etc. Consequently, she was put under preventive
plebiscites rejected it, but the Cordillera Administrative Region is suspension for 90 days. Pending the resolution of her motion to lift
still existent. Kalinga-Apayao is now the provinces of Kalinga and suspension, she instituted an action seeking the nullification of the
Apayao. entire administrative proceedings. She claims that when the new
Local Government Code of 1991 was enacted, the Secretary of

The Apocryphal Maggots:


Rainier, Chrisgel, Corina, Geoffry, Grace and Sylvie Blanche
The Flibbertigibbet Worms:
Golda, Gladys and Melyjane
CA VE AT: By simply r ea ding this re vi ew er a t the end o f the sem este r wi ll (hopef ully) guar antee you r pas sing this cou rse. D rink mode rat ely .
Amusin S C A r y
gly
THE C2005 LOCAL GOVERNMENT REVIEWER - 18 -
Health had lost his disciplinary powers and authority over her. and make them more effective partners in the attainment of
Such power to discipline the personnel of the Provincial Health national goals. Toward this end, the State shall provide for
Officer is now vested in the Provincial Government. a more responsive and accountable local government
structure instituted through a system of decentralization
Held: NO. Jurisdiction is determined by the statue in force at the whereby local government units shall be given more
time of the commencement of the action. The LGC of 1991 took powers, authority, responsibilities, and resources. The
effect on January 1, 1992. In the case at bar, Sibbaluca was process of decentralization shall proceed from the National
administratively charged in 1991. The operative laws then were Government to the local government units.
the Administrative Code of 1987 and EO 119. Under these laws, (b) It is also the policy of the State to ensure the accountability
the Secretary of Health exercises control, direction and supervision of local government units through the institution of effective
over his subordinates, which includes private respondent. mechanisms of recall, initiative and referendum.
(c) It is likewise the policy of the State to require all national
agencies and offices to conduct periodic consultations with
Fusion sex (part2) appropriate LGUs, nongovernmental and people’s
organizations, and other concerned sectors of the
community before any project or program is implemented
The common ingredients to fusion sex that men who have
in their respective jurisdictions.
experienced it describe are:
1) The man either recently has left a long-standing though Sec3, LGC: Operative Principles of Decentralization – The
frustrating sexual relationship or has been looking for a truly formulation and implementation of policies and measures on
satisfying one without success for a long time. In a word, he is local autonomy shall be guided by the following operative
ripe. principles:
2) The fusion sex relationship is not predicated on a future (a) There shall be an effective allocation among the different
potential. Neither party has long-range designs on the other. LGUs of their respective powers, functions, responsibilities,
The woman may be married and intending to return to her and resources;
husband or living in another part of the country or world and (b) There shall be established in every LGU an accountable,
planning to go back shortly. In other words, there are serious efficient, and dynamic organizational structure and
obstacles to permanency, the future is highly uncertain, and operating mechanism that will meet the priority needs and
therefore the relationship is totally now. service requirements of its communities;
(c) Subject to civil service law, rules and regulations, local
3) The male is flooded with and able to experience and express
officials and employees paid wholly or mainly from local
long-repressed feelings that never or very rarely emerge. That is, funds shall be appointed or removed, according to merit
during fusion sex he is able to cry in sadness, to experience his and fitness, by the appropriate appointing authority;
loneliness, to cling, to be passive, to enjoy feeling beautiful and (d) The vesting of duty, responsibility, and accountability in
to be totally transparent about his fantasies, his feelings, his LGUs shall be accompanied with provision for reasonably
past, and his future aspirations. In short, he is temporarily adequate resources to discharge their powers and
whole, reunited with long lost parts of himself. effectively carry out their functions; hence, they shall have
4) The male senses a real challenge. He has found his “magic the power to create and broaden their own sources of
lady” who is not free or readily available and he is totally revenue and the right to a just share in national taxes and
expressive in his pursuit and desire for her. an equitable share in the proceeds of the utilization and
(to be continued) development of the national wealth within their respective
areas;
(e) Provinces with respect to component cities and
municipalities, and cities and municipalities with respect to
component barangays, shall ensure that the acts of their
Par t I I: Boo k I: Gener al Pro visions component units are within the scope of their prescribed
powers and functions;
Ti tle O ne: Basi c P ri ncipl es (f) LGUs may group themselves, consolidate or coordinate
A. Polic y a nd Appli cat ion their efforts, services, and resources for purposes
commonly beneficial to them;
Sec1, LGC: Title – This Act shall be known and cited as the “Local (g) The capabilities of LGUs, especially the municipalities and
Government Code of 1991”. barangays, shall be enhanced by providing them with
opportunities to participate actively in the implementation of
Sec2, LGC: Declaration of Policy – national programs and projects;
(a) It is hereby declared the policy of the State that the (h) There shall be a continuing mechanism to enhance local
territorial and political subdivisions of the State shall enjoy autonomy not only by legislative enabling acts but also by
genuine and meaningful local autonomy to enable them to administrative and organizational reforms;
attain their fullest development as self-reliant communities

The Apocryphal Maggots:


Rainier, Chrisgel, Corina, Geoffry, Grace and Sylvie Blanche
The Flibbertigibbet Worms:
Golda, Gladys and Melyjane
CA VE AT: By simply r ea ding this re vi ew er a t the end o f the sem este r wi ll (hopef ully) guar antee you r pas sing this cou rse. D rink mode rat ely .
Amusin S C A r y
gly
THE C2005 LOCAL GOVERNMENT REVIEWER - 19 -
(i) LGUs shall share with the National Government the Greater Balanga Development Corporation (GBDC) vs.
responsibility in the management and maintenance of Municipality of Balanga (1994)
ecological balance within their territorial jurisdiction, subject
to the provisions of this Code and national policies; Facts:
(j) Effective mechanisms for ensuring the accountability of GBDC applied with the Office of the Balanga Mayor for a business
LGUs to their respective constituents shall be strengthened permit its property, certain portions of which as been "unlawfully
in order to upgrade continually the quality of local usurped and invaded" by Balanga, which had
leadership; "allowed/tolerated/abetted" the construction of shanties and market
(k) The realization of local autonomy shall be facilitated stalls while charging market fees and market entrance fees from
through improved coordination of national government the occupants and users of the area. Mayor issued a Mayor's
policies and programs and extension of adequate technical Permit granting GBDC the privilege of a "real estate
and material assistance to less developed and deserving dealer/privately-owned public market operator" under the
LGUs; registered trade name of Balanga Public Market.
(l) The participation of the private sector in local governance, However, the Sangguniang Bayan passed a Resolution annulling
particularly in the delivery of basic services, shall be the Mayor's permit issued to GBDC and advising the Mayor to
encouraged to ensure the viability of local autonomy as an revoke the permit. Mayor revoked the permit insofar as it
alternative strategy for sustainable development; and authorized the operation of a public market. GBDC filed this
(m) The National Government shall ensure that petition claiming that it had not violated any law/ordinance, thus
decentralization contributes to the continuing improvement there’s no reason to revoke the Mayor's permit. The EO and the
of the performance of LGUs and the quality of community resolution in question were quasi-judicial acts and not mere
life. exercises of police power. Respondent also failed to observe due
process in revoking the permit. Balanga argues that Mayor may
Sec4, LGC: Scope of Application – This Code shall apply to all issue, deny or revoke municipal licenses and permits. Resolution
provinces, cities, municipalities, barangays, and other political and EO were legitimate exercise of local legislative authority.
subdivisions as may be created by law, and, to the extent herein GBDC violated Section 3A-06(b) of the Balanga Revenue Code
provided, to officials, offices, or agencies of the National when it failed to disclose the true status of the area involved in the
Government. permit when it did not secure separate permits for its two
businesses.
Sec5, LGC: Rules of Interpretation – In the interpretation of the
provisions of this Code, the following rules shall apply:
(a) Any provision on a power of a LGU shall be liberally Held: No. There was no ground for revocation. The application for
interpreted in its favor, and in case of doubt, any provision Mayor's permit requires the applicant to state what type of
thereon shall be resolved in favor of devolution of powers business, profession, occupation and/or calling privileges is being
and of the lower LGU. Any fair and reasonable doubt as to applied for. Petitioner left this entry bank in its application form.
the existence of the power shall be interpreted in favor of Leaving an entry blank is not equal to false statement. There must
the LGU concerned; be proof of willful misrepresentation and deliberate intent to make
(b) In case of doubt, any tax ordinance or revenue measure a false statement. The absence of the material info in the
shall be construed strictly against the LGU enacting it, and application form was nonetheless supplied in the face of the permit
liberally in favor of the taxpayer. Any tax exemption, signed and issued by Mayor Banzon himself.
incentive or relief granted by any LGU pursuant to the
provisions of this Code shall be construed strictly against
the person claiming it; Applying for two businesses in one permit is also not a ground for
(c) The general welfare provisions in this Code shall be revocation. Par 2 Section 3A-06(b) does not expressly require two
liberally interpreted to give more powers to LGUs in permits for their conduct of two or more businesses in one place,
accelerating economic development and upgrading the but only that separate fees be paid for each business. The powers
quality of life for the people in the community; of municipal corporations are to be construed in strictissimi
(d) Rights and obligations existing on the date of effectivity of juris (strictly in its legal terms) and any doubt or ambiguity
this Code and arising out of contracts or any other source must be construed against the municipality Granting, however,
of prestation involving a LGU shall be governed by the that separate permits are actually required, the application form
original terms and conditions of said contracts or the law in does not contain any entry as regards the number of businesses
force at the time such rights were vested; and the applicant wishes to engage in.
(e) In the resolution of controversies arising under this Code
where no legal provision or jurisprudence applies, resort
my be had to the customs and traditions in the place where  Balanga case and Section 5 (a) conflict. Neither will it fall
(f) the controversies take place. under Section 5 (b) unless the business permit is a revenue
measure. Then again , the trend of SC is to limit the LGC.

The Apocryphal Maggots:


Rainier, Chrisgel, Corina, Geoffry, Grace and Sylvie Blanche
The Flibbertigibbet Worms:
Golda, Gladys and Melyjane
CA VE AT: By simply r ea ding this re vi ew er a t the end o f the sem este r wi ll (hopef ully) guar antee you r pas sing this cou rse. D rink mode rat ely .
Amusin S C A r y
gly
THE C2005 LOCAL GOVERNMENT REVIEWER - 20 -
Tano vs. Socrates (1997) In Magtajas v. Pryce Properties Corporation, well-established tests
of a valid ordinance are:
Facts: (a) It must not contravene the Constitution or any statute;
Sangguniang Panlunsod of Puerto Princesa enacted an Ordinance (b) It must not be unfair or oppressive;
banning the shipment of all live fish and lobster outside Puerto (c) It must not be partial or discriminatory;
Princesa for 5 years. To implement this, Acting City Mayor issued (d) It must not prohibit but may regulate trade;
Office Order No. 23. Next, the SP of Palawan enacted SR No.33 (e) It must be general and consistent with public policy; and
and Ordinance No. 2 series of 1993 prohibiting the catching, (f) It must not be unreasonable.
gathering, possessing, buying, selling and shipment of live marine
coral dwelling aquatic organisms in and coming from Palawan The SP of Puerto Princesa lack the authority of enacting the
waters for 5 years. The respondents implemented the said Ordinance since the subject is within the jurisdiction and
ordinances depriving all the fishermen of the whole province of responsibility of the Bureau of Fisheries and Aquatic Resources
Palawan and the City of Puerto Princesa of their only means of (BFAR) under PD704, which was not expressly repealed by the
livelihood and the petitioners Airline Shippers Association of LGCode of 1991.
Palawan and other marine merchants from performing their lawful
occupation and trade. Hence, this petition Special law must prevail over general law. LGCode is a general
law while P.D. No. 704 is a special law dealing with the protection
Held: Resolution and ordinances are valid. It is settled that and conservation of fishing and aquatic resources including those
laws, including ordinances enacted by local government units in the municipal waters.
enjoy the presumption of constitutionality. To overthrow this
presumption, there must be a clear and unequivocal breach of the Ordinance is also unenforceable for lack of approval of Secretary
Constitution, not merely a doubtful or argumentative contradiction. of Agriculture under PD 704.
Petitioners’ argument the Constitution and applicable laws were Police power is inherent in a state, it is not so in municipal
violated by the ordinances is baseless. The ordinances in question corporations or local governments. In order that a local
are meant precisely to protect and conserve our marine resources government may exercise police power, there must be a legislative
to the end that their enjoyment may be guaranteed not only for the grant which necessarily sets the limits for the exercise of the
present generation, but also for the generations to come. power. The power devolved upon the municipality under the Local
Moreover, Section 5(c) of the LGC explicitly mandates that the Government Code is the enforcement of existing fishery laws of
general welfare provisions of the LGC "shall be liberally interpreted the State and not the enactment thereof. While a local government
to give more powers to the local government units in accelerating unit may adopt ordinances upon subjects covered by law or statute
economic development and upgrading the quality of life for the (i.e., PDs 704, 1015 and 1219), such ordinances should be in
people of the community." The centerpiece of LGCode is the accordance with and not repugnant to the law.
system of decentralization as expressly mandated by the
Constitution. Indispensable to decentralization is devolution and
the LGCode expressly provides that "any provision on a power of a  Some locgovspeak:
local government unit shall be liberally interpreted in its favor, and Local autonomy: power of LGU to decide for themselves on certain
in case of doubt, any question thereon shall be resolved in favor of matters without clearance or approval from the NG.
devolution of powers and of the lower local government unit. Any Decentralization: Devolution of administration (NOT power) from
fair and reasonable doubt as to the existence of the power shall be the NG to LGU
interpreted in favor of the local government unit concerned." Deconcentration: Flow of autonomy from NG to regional agencies
Devolution: act by which NG confers power and authority from the
CONCURRING OPINION: Mendoza NG to perform specific functions and responsibilities.
There is no basis for the claim in the dissenting opinion that the
subject of these ordinances lies within the competence of the
national government. For the matter concerns a local problem, There is Memorandum Circular 53 (1993) which provides that
namely, the destruction of aquatic resources in the Province of all offices and employees including GOCCs to comply with LGC
Palawan. It would therefore set back the policy of decentralization and its IRR on consultations. However, there is no penalty for the
were this Court to sustain such a claim. breach.

The presumption of constitutionality must prevail in the absence of


some factual foundation of record for overthrowing the statute.

If the laws passed are seen to have a reasonable relation to a Some Horror Movie Quotes:
proper legislative purpose, and are neither arbitrary nor
discriminatory, the requirements of due process are satisfied and Little girl:"His name is Freddy Krueger, and he loves children,
judicial determination to that effect renders a court functus officio. especially little girls. Freddy's coming back. Soon he'll be strong
enough. It's okay to be afraid. We were all afraid. Warn your
DISSENT: Bellosillo friends. Warn everyone!" (Freddy VS. Jason}

The Apocryphal Maggots:


Rainier, Chrisgel, Corina, Geoffry, Grace and Sylvie Blanche
The Flibbertigibbet Worms:
Golda, Gladys and Melyjane
CA VE AT: By simply r ea ding this re vi ew er a t the end o f the sem este r wi ll (hopef ully) guar antee you r pas sing this cou rse. D rink mode rat ely .
Amusin S C A r y
gly
THE C2005 LOCAL GOVERNMENT REVIEWER - 21 -

Chucky:"I'll be back! I always come back.. But dying is such a Sec9, LGC: Abolition of Local Government Units. – A local
b*tch" (Child's Play) government unit may be abolished when its income, population,
or land area has been irreversibly reduced to less than the
minimum standards prescribed for its creation under Book III of
this Code, as certified by the national agencies mentioned in
Section 17 hereof to Congress or to the sanggunian concerned,
as the case may be.
A. Gener al P ower and Attr ibutes The law or ordinance abolishing a local government unit shall
specify the province, city, municipality, or barangay with which
Local Government Code the local government unit sought to be abolished will be
incorporated or merged.
Sec6, LGC: Authority to Create Local Government Units. – A
local government unit may be created, divided, merged, Sec10, LGC: Plebiscite Requirement. – No creation, division,
abolished, or its boundaries substantially altered either by law merger, abolition, or substantial alteration of boundaries of local
enacted by Congress in the case of a province, city, municipality, government units shall take effect unless approved by a majority
or any other political subdivision, or by ordinance passed by the of the votes cast in a plebiscite called for the purpose in the
sangguniang panlalawigan or sangguniang panlungsod political unit or units directly affected. Said plebiscite shall be
concerned in the case of a barangay located within its territorial conducted by the Commission on Elections (COMELEC) within
jurisdiction, subject to such limitations and requirements one hundred twenty (120) days from the date of effectivity of the
prescribed in this Code. law or ordinance effecting such action, unless said law or
ordinance fixes another date.
Sec7, LGC: Creation and Conversion. – As a general rule, the
creation of a local government unit or its conversion from one Sec11: Selection and Transfer of Local Government Site,
level to another level shall be based on verifiable indicators of Offices and Facilities. –
viability and projected capacity to provide services, to wit: (a) The law or ordinance creating or merging local government
(a) Income - It must be sufficient, based on acceptable units shall specify the seat of government from where
standards, to provide for all essential government facilities governmental and corporate services shall be delivered. In
and services and special functions commensurate with the selecting said site, factors relating to geographical
size of its population, as expected of the local government centrality, accessibility, availability of transportation and
unit concerned; communication facilities, drainage and sanitation,
(b) Population. - It shall be determined as the total number of development and economic progress, and other relevant
inhabitants within the territorial jurisdiction of the local considerations shall be taken into account.
government unit concerned; and (b) When conditions and developments in the local
(c) Land Area. - It must be contiguous, unless it comprises two government unit concerned have significantly changed
or more islands or is separated by a local government unit subsequent to the establishment of the seat of government,
independent of the others; properly identified by metes and its sanggunian may, after public hearing and by a vote of
bounds with technical descriptions; and sufficient to provide two-thirds (2/3) of all its members, transfer the same to a
for such basic services and facilities to meet the site better suited to its needs. Provided, however, That no
requirements of its populace. Compliance with the such transfer shall be made outside the territorial
foregoing indicators shall be attested to by the Department boundaries of the local government unit concerned.
of Finance (DOF), the National Statistics Office (NSO), and The old site, together with the improvements thereon, may
the Lands Management Bureau (LMB) of the Department be disposed of by sale or lease or converted to such other
of Environment and Natural Resources(DENR). use as the sanggunian concerned may deem beneficial to
the local government unit concerned and its inhabitants.
Sec8, LGC: Division and Merger. – Division and merger of (c) Local government offices and facilities shall not be
existing local government units shall comply with the same transferred, relocated, or converted to other uses unless
requirements herein prescribed for their creation: Provided, public hearings are first conducted for the purpose and the
however, That such division shall not reduce the income, concurrence of the majority of all the members of the
population, or land area of the local government unit or units sanggunian concerned is obtained.
concerned to less than the minimum requirements prescribed in
this Code: Provided, further, That the income classification of the Sec12: Government Centers. – Provinces, cities, and
original local government unit or units shall not fall below its municipalities shall endeavor to establish a government center
current income classification prior to such division. The income where offices, agencies, or branches of the national
classification of local government units shall be updated within government, local government units, or government-owned or
six (6) months from the effectivity of this Code to reflect the -controlled corporations may, as far as practicable, be located.
changes in their financial position resulting from the increased In designating such a center, the local government unit
revenues as provided herein. concerned shall take into account the existing facilities of

The Apocryphal Maggots:


Rainier, Chrisgel, Corina, Geoffry, Grace and Sylvie Blanche
The Flibbertigibbet Worms:
Golda, Gladys and Melyjane
CA VE AT: By simply r ea ding this re vi ew er a t the end o f the sem este r wi ll (hopef ully) guar antee you r pas sing this cou rse. D rink mode rat ely .
Amusin S C A r y
gly
THE C2005 LOCAL GOVERNMENT REVIEWER - 22 -
national and local agencies and offices which may serve as the reason and, in any case, not oftener than once every ten
government center as contemplated under this Section. The (10) years. The name of a local government unit or a public
national government, local government unit or government- place, street or structure with historical, cultural, or ethnic
owned or -controlled corporation concerned shall bear the significance shall not be changed, unless by a unanimous
expenses for the construction of its buildings and facilities in the vote of the sanggunian concerned and in consultation with
government center. the PHC.
(e) A change of name of a public school shall be made only
Sec13: Naming of Local Government Units and Public Places, upon the recommendation of the local school board
Streets and Structures. – concerned.
(a) The sangguniang panlalawigan may, in consultation with (f) A change of name of public hospitals, health centers, and
the Philippine Historical Commission (PHC), change the other health facilities shall be made only upon the
name of the following within its territorial jurisdiction: recommendation of the local health board concerned.
(1) Component cities and municipalities, upon the (g) The change of name of any local government unit shall be
recommendation of the sanggunian concerned; effective only upon ratification in a plebiscite conducted for
(2) Provincial roads, avenues, boulevards, thorough- the purpose in the political unit directly affected. In any
fares, and bridges; change of name, the Office of the President, the
(3) Public vocational or technical schools and other post- representative of the legislative district concerned, and the
secondary and tertiary schools; Bureau of Posts shall be notified.
(4) Provincial hospitals, health centers, and other health
facilities; and Sec14: Beginning of Corporate Existence. – When a new local
(5) Any other public place or building owned by the government unit is created, its corporate existence shall
provincial government. commence upon the election and qualification of its chief
(b) The sanggunian of highly urbanized cities and of executive and a majority of the members of its sanggunian,
component cities whose charters prohibit their voters from unless some other time is fixed therefor by the law or ordinance
voting for provincial elective officials, hereinafter referred to creating it.
in this Code as independent component cities, may, in
consultation with the Philippine Historical Commission, Sec15: Political and Corporate Nature of Local Government
change the name of the following within its territorial Units. – Every local government unit created or recognized
jurisdiction: under this Code is a body politic and corporate endowed with
(1) City barangays, upon the recommendation of the powers to be exercised by it in conformity with law. As such, it
sangguniang barangay concerned; shall exercise powers as a political subdivision of the national
(2) City roads, avenues, boulevards, thoroughfares, and government and as a corporate entity representing the
bridges; inhabitants of its territory.
(3) Public elementary, secondary and vocational or
technical schools, community colleges and non- Sec16: General Welfare. – Every local government unit shall
chartered colleges; exercise the powers expressly granted, those necessarily
(4) City hospitals, health centers and other health implied therefrom, as well as powers necessary, appropriate, or
facilities; and incidental for its efficient and effective governance, and those
(5) Any other public place or building owned by the city which are essential to the promotion of the general welfare.
government. Within their respective territorial jurisdictions, local government
(c) The sanggunians of component cities and municipalities units shall ensure and support, among other things, the
may, in consultation with the Philippine Historical preservation and enrichment of culture, promote health and
Commission, change the name of the following within its safety, enhance the right of the people to a balanced ecology,
territorial jurisdiction: encourage and support the development of appropriate and self-
(1) city and municipal barangays, upon recommendation reliant scientific and technological capabilities, improve public
of the sangguniang barangay concerned; morals, enhance economic prosperity and social justice,
(2) city, municipal and barangay roads, avenues, promote full employment among their residents, maintain peace
boulevards, thoroughfares, and bridges; and order, and preserve the comfort and convenience of their
(3) city and municipal public elementary, secondary and inhabitants.
vocational or technical schools, post-secondary and
other tertiary schools; Sec17: Basic Services and Facilities. –
(4) city and municipal hospitals, health centers and other (a) Local government units shall endeavor to be self-reliant
health facilities; and and shall continue exercising the powers and discharging
(5) Any other public place or building owned by the the duties and functions currently vested upon them. They
municipal government. shall also discharge the functions and responsibilities of
(d) None of the foregoing local government units, institutions, national agencies and offices devolved to them pursuant to
places, or buildings shall be named after a living person, this Code. Local government units shall likewise exercise
nor may a change of name be made unless for a justifiable such other powers and discharge such other functions and

The Apocryphal Maggots:


Rainier, Chrisgel, Corina, Geoffry, Grace and Sylvie Blanche
The Flibbertigibbet Worms:
Golda, Gladys and Melyjane
CA VE AT: By simply r ea ding this re vi ew er a t the end o f the sem este r wi ll (hopef ully) guar antee you r pas sing this cou rse. D rink mode rat ely .
Amusin S C A r y
gly
THE C2005 LOCAL GOVERNMENT REVIEWER - 23 -
responsibilities as are necessary, appropriate, or incidental medicines, medical supplies, and equipment
to efficient and effective provision of the basic services and needed to carry out the services herein
facilities enumerated herein. enumerated;
(b) Such basic services and facilities include, but are not (iv). Social welfare services which include
limited to, the following: programs and projects on child and youth
(1) For a Barangay: welfare, family and community welfare,
(i) Agricultural support services which include women's welfare, welfare of the elderly and
planting materials distribution system and disabled persons; community-based
operation of farm produce collection and rehabilitation programs for vagrants, beggars,
buying stations; street children, scavengers, juvenile
(ii) Health and social welfare services which delinquents, and victims of drug abuse;
include maintenance of barangay health livelihood and other pro-poor
center and day-care center; projects; nutrition services; and family
(iii) Services and facilities related to general planning services;
hygiene and sanitation, beautification, and (v). Information services which include
solid waste collection; investments and job placement information
(iv) Maintenance of katarungang pambarangay; systems, tax and marketing information
(v) Maintenance of barangay roads and bridges systems, and maintenance of a public library;
and water supply systems (vi). Solid waste disposal system or environmental
(vi) Infrastructure facilities such as multi- purpose management system and services or facilities
hall, multipurpose pavement, plaza, sports related to general hygiene and sanitation;
center, and other similar facilities; (vii). Municipal buildings, cultural centers, public
(vii) Information and reading center; and parks including freedom parks, playgrounds,
(viii) Satellite or public market, where viable; and sports facilities and equipment, and other
(2) For a municipality: similar
(i). Extension and on-site research services and facilities;
facilities related to agriculture and fishery (viii). Infrastructure facilities intended primarily to
activities which include dispersal of livestock service the needs of the residents of the
and poultry, fingerlings, and other seeding municipality and which are funded out of
materials for aquaculture; palay, corn, and municipal funds including, but not limited to,
vegetable seed farms; medicinal plant municipal roads and bridges; school buildings
gardens; fruit tree, coconut, and other kinds of and other facilities for public elementary and
seedling nurseries; demonstration farms; secondary schools; clinics, health centers and
quality control of copra and improvement and other health facilities necessary to carry out
development of local distribution channels, health services; communal irrigation, small
preferably through cooperatives; water impounding projects and other similar
interbarangay irrigation system; water and soil projects; fish ports; artesian wells, spring
resource utilization and conservation projects; development, rainwater collectors and water
and enforcement of fishery laws in municipal supply systems; seawalls, dikes, drainage and
waters including the conservation of sewerage, and flood control; traffic signals and
mangroves; road signs; and similar facilities;
(ii). Pursuant to national policies and subject to (ix). Public markets, slaughterhouses and other
supervision, control and review of the DENR, municipal enterprises;
implementation of community-based forestry (x). Public cemetery;
projects which include integrated social (xi). Tourism facilities and other tourist attractions,
forestry programs and similar projects; including the acquisition of equipment,
management and control of communal forests regulation and supervision of business
with an area not exceeding fifty (50) square concessions, and security services for such
kilometers; establishment of tree parks, facilities; and
greenbelts, and similar forest development (xii). Sites for police and fire stations and
projects; substations and the municipal jail;
(iii). Subject to the provisions of Title Five, Book I (3) For a Province:
of this Code,
(i). health services which include the A
implementation of programs
gricultural
andextension
projects and
on on-site research services and facilities which include the prevention and
primary health care, maternal
control ofand
plant
child
andcare,
animal pests and diseases; dairy farms, livestock markets, animal breeding
and communicable stations,
and non-communicable
and artificial insemination centers; and assistance in the organization of farmers' and
disease control services;fishermen's
access tocooperatives
secondary and other collective organizations, as well as the transfer of appropriate
and tertiary health services; purchase of

The Apocryphal Maggots:


Rainier, Chrisgel, Corina, Geoffry, Grace and Sylvie Blanche
The Flibbertigibbet Worms:
Golda, Gladys and Melyjane
CA VE AT: By simply r ea ding this re vi ew er a t the end o f the sem este r wi ll (hopef ully) guar antee you r pas sing this cou rse. D rink mode rat ely .
Amusin S C A r y
gly
THE C2005 LOCAL GOVERNMENT REVIEWER - 24 -
local government unit concerned, based on national
nd development services, as well as the transfer of appropriate technology;
P policies, standards and guidelines.
policies and subject to supervision, control and review of the DENR, enforcement (e) National agencies or offices concerned shall
ed to community-based forestry projects, pollution control law, small-scale mining devolve to local government units the responsibility
on the protection of the environment; and mini-hydro electric projects for local for the provision of basic services and facilities
enumerated in this Section within six (6) months
S after the effectivity of this Code. As used in this
ons of Title Five, Book I of this Code, health services which include hospitals and Code, the term "devolution" refers to the act by
services; which the national government confers power and
S authority upon the various local government units to
s which include pro grams and projects on rebel returnees and evacuees; relief perform specific functions and responsibilities.
ulation development services; (f) The national government or the next higher level of
P local government unit may provide or augment the
rovincial jails, freedom parks and other public assembly areas, and other similar basic services and facilities assigned to a lower
level of local government unit when such services
I or facilities are not made available or, if made
s intended to service the needs of the residents of the province and which are available, are inadequate to meet the requirements
ncial funds including, but not limited to, provincial roads and bridges; inter- of its inhabitants.
ks, drainage and sewerage, flood control, and irrigation systems; reclamation (g) The basic services and facilities hereinabove
facilities; enumerated shall be funded from the share of local
P government units in the proceeds of national
ts for low-cost housing and other mass dwellings, except those funded by the taxes and other local revenues and funding support
stem (SSS), Government Service Insurance System (GSIS), and the Home from the national government, its instrumentalities
l Fund (HDMF): Provided, That national funds for these programs and projects and government-owned or -controlled corporations
allocated among the regions in proportion to the ratio of the homeless to the which are tasked by law to establish and maintain
such services or facilities. Any fund or resource
I available for the use of local government units shall
ervices, including access to credit financing; be first allocated for the provision of basic services
U or facilities enumerated in subsection (b) hereof
rnization of tax information and collection services through the use of computer before applying the same for other purposes,
are and other means; unless otherwise provided in this Code.
I (h) The Regional offices of national agencies or offices
ommunications services, subject to national policy guidelines; and whose functions are devolved to local government
T units as provided herein shall be phased out within
and promotion programs; one (1) year from the approval of this Code. Said
(4) For a City: national agencies and offices may establish such
All the services and facilities of the municipality and field units as may be necessary for monitoring
province, and in addition thereto, the following: purposes and providing technical assistance to
(a) Adequate communication and transportation local government units. The properties, equipment,
facilities; and other assets of these regional offices shall be
(b) Support for education, police and fire services and distributed to the local government units in the
facilities. region in accordance with the rules and regulations
(c) Notwithstanding the provisions of subsection (b) issued by the oversight committee created under
hereof, public works and infrastructure projects and this Code.
other facilities funded by the national government (i) The devolution contemplated in this Code shall
under the annual General Appropriations Act, other include the transfer to local government units of the
special laws, pertinent executive orders, and those records, equipment, and other assets and
wholly or partially funded from foreign sources, are personnel of national agencies and offices
not covered under this Section, except in those corresponding to the devolved powers, functions,
cases where the local government unit concerned and responsibilities.
is duly designated as the implementing agency for (j) Personnel of said national agencies or offices shall
such projects, facilities, programs, and services. be absorbed by the local government units to which
(d) The designs, plans, specifications, testing of they belong or in whose areas they are assigned to
materials, and the procurement of equipment and the extent that it is administratively viable as
materials from both foreign and local sources determined by the said oversight committee:
necessary for the provision of the foregoing Provided, That the rights accorded to such
services and facilities shall be undertaken by the personnel pursuant to civil service law, rules and

The Apocryphal Maggots:


Rainier, Chrisgel, Corina, Geoffry, Grace and Sylvie Blanche
The Flibbertigibbet Worms:
Golda, Gladys and Melyjane
CA VE AT: By simply r ea ding this re vi ew er a t the end o f the sem este r wi ll (hopef ully) guar antee you r pas sing this cou rse. D rink mode rat ely .
Amusin S C A r y
gly
THE C2005 LOCAL GOVERNMENT REVIEWER - 25 -
regulations shall not be impaired: Provided, (a) A city or municipality may, through an ordinance passed by
Further, That regional directors who are career
the sanggunian after conducting public hearings for the
executive service officers and other officers of
purpose, authorize the reclassification of agricultural lands
similar rank in the said regional offices who cannot
and provide for the manner of their utilization or disposition
be absorbed by the local government unit shall be
in the following cases: (1) when the land ceases to be
retained by the national government, without any
economically feasible and sound for agricultural purposes
diminution of rank, salary or tenure.
as determined by the Department of Agriculture or (2)
(k) To ensure the active participation of the private
where the land shall have substantially greater economic
sector in local governance, local government units
value for residential, commercial, or industrial
may, by ordinance, sell, lease, encumber, or
purposes, as determined by the sanggunian concerned:
otherwise dispose of public economic enterprises
Provided, That such reclassification shall be limited to the
owned by them in their proprietary capacity. Costs
following percentage of the total agricultural land area at
may also be charged for the delivery of basic
the time of the passage of the ordinance:
services or facilities enumerated in this Section.
(1)For highly urbanized and independent component
cities, fifteen percent (15%);
Sec18: Power to Generate and Apply Resources. – Local
(2)For component cities and first to third class
government units shall have the power and authority to establish
municipalities, ten percent (10%); and
an organization that shall be responsible for the efficient and
(3)For fourth to sixth class municipalities, five percent
effective implementation of their development plans, program
(5%): Provided, further, That agricultural lands
objectives and priorities; to create their own sources of revenue
distributed to agrarian reform beneficiaries pursuant to
and to levy taxes, fees, and charges which shall accrue
Republic Act Numbered Sixty-six hundred fifty-seven
exclusively for their use and disposition and which shall be
(R.A. No. 6657), otherwise known as "The
retained by them; to have a just share in national taxes which
Comprehensive Agrarian Reform Law", shall not be
shall be automatically and directly released to them without
affected by the said reclassification and the
need of any further action; to have an equitable share in the
conversion of such lands into other purposes shall be
proceeds from the utilization and development of the national
governed by Section 65 of said Act.
wealth and resources within their respective territorial
(b) The President may, when public interest so requires and
jurisdictions including sharing the same with the inhabitants by
upon recommendation of the National Economic and
way of direct benefits; to acquire, develop, lease, encumber,
Development Authority, authorize a city or municipality to
alienate, or otherwise dispose of real or personal property held
reclassify lands in excess of the limits set in the next
by them in their proprietary capacity and to apply their resources
preceding paragraph.
and assets for productive, developmental, or welfare purposes,
(c) The local government units shall, in conformity with existing
in the exercise or furtherance of their governmental or
laws, continue to prepare their respective comprehensive
proprietary powers and functions and thereby ensure their
land use plans enacted through zoning ordinances which
development into self-reliant communities and active
shall be the primary and dominant bases for the future use
participants in the attainment of national goals.
of land resources: Provided, That the requirements for
food production, human settlements, and industrial
Sec19: Eminent Domain. – A local government unit may, through
expansion shall be taken into consideration in the
its chief executive and acting pursuant to an ordinance, exercise
preparation of such plans.
the power of eminent domain for public use, or purpose, or
(d) Where approval by a national agency is required for
welfare for the benefit of the poor and the landless, upon
reclassification, such approval shall not be unreasonably
payment of just compensation, pursuant to the provisions of the
withheld. Failure to act on a proper and complete
Constitution and pertinent laws: Provided, however, That the
application for reclassification within three (3) months from
power of eminent domain may not be exercised unless a valid
receipt of the same shall be deemed as approval thereof.
and definite offer has been previously made to the owner, and
(e) Nothing in this Section shall be construed as repealing,
such offer was not accepted: Provided, further, That the local
amending, or modifying in any manner the provisions of
government unit may immediately take possession of the
R.A. No. 6657.
property upon the filing of the expropriation proceedings and
upon making a deposit with the proper court of at least fifteen
Sec21: Closure and Opening of Roads. –
percent (15%) of the fair market value of the property based on
(a) A local government unit may, pursuant to an ordinance,
the current tax declaration of the property to be expropriated:
permanently or temporarily close or open any local road,
Provided, finally, That, the amount to be paid for the
alley, park, or square falling within its jurisdiction: Provided,
expropriated property shall be determined by the proper court,
however, That in case of permanent closure, such
based on the fair market value at the time of the taking of the
ordinance must be approved by at least two-thirds (2/3) of
property.
all the members of the sanggunian, and when necessary, an
adequate substitute for the public facility that is subject to
Sec20: Reclassification of Lands. –
closure is provided.

The Apocryphal Maggots:


Rainier, Chrisgel, Corina, Geoffry, Grace and Sylvie Blanche
The Flibbertigibbet Worms:
Golda, Gladys and Melyjane
CA VE AT: By simply r ea ding this re vi ew er a t the end o f the sem este r wi ll (hopef ully) guar antee you r pas sing this cou rse. D rink mode rat ely .
Amusin S C A r y
gly
THE C2005 LOCAL GOVERNMENT REVIEWER - 26 -
(b) No such way or place or any part thereof shall be Sec23: Authority to Negotiate and Secure Grants. – Local chief
permanently closed without making provisions for the executives may, upon authority of the sanggunian, negotiate and
maintenance of public safety therein. A property thus secure financial grants or donations in kind, in support of the
permanently withdrawn from public use may be used or basic services or facilities enumerated under Section 17 hereof,
conveyed for any purpose for which other real property from local and foreign assistance agencies without necessity of
belonging to the local government unit concerned may be securing clearance or approval therefor from any department,
lawfully used or conveyed: Provided, however, That no agency, or office of the national government or from any higher
freedom park shall be closed permanently without provision local government unit: Provided, That projects financed by such
for its transfer or relocation to a new site. grants or assistance with national security implications shall be
(c) Any national or local road, alley, park, or square may be approved by the national agency concerned: Provided, further,
temporarily closed during an actual emergency, or fiesta That when such national agency fails to act on the request for
celebrations, public rallies, agricultural or industrial fairs, or approval within thirty (30) days from receipt thereof, the same
an undertaking of public works and highways, shall be deemed approved.
telecommunications, and waterworks projects, the duration
of which shall be specified by the local chief executive The local chief executive shall, within thirty (30) days upon
concerned in a written order: Provided, however, That no signing of such grant agreement or deed of donation, report the
national or local road, alley, park, or square shall set nature, amount, and terms of such assistance to both Houses of
temporarily closed for athletic, cultural, or civic activities not Congress and the President.
officially sponsored, recognized, or approved by the
local government unit concerned. Sec24: Liability for Damages. – Local government units and their
(d) Any city, municipality, or barangay may, by a duly enacted officials are not exempt from liability for death or injury to
ordinance, temporarily close and regulate the use of any persons or damage to property.
local street, road, thoroughfare, or any other public place
where shopping malls, Sunday, flea or night markets, or
shopping areas may be established and where goods,
merchandise, foodstuffs, commodities, or articles of
commerce may be sold and dispensed to the general public. TRIVIA: Beer contains the Vitamin B group, while water does not.
Kaya mas mahal ang isang bote ng beer sa isang bote ng tubig.
Sec22: Corporate Powers. –
(a) Every local government unit, as a corporation, shall have
the following powers:
(1)To have continuous succession in its corporate MyPleasure Guide: Erotic Massage
name;
(2)To sue and be sued;
Sexual enhancement goes beyond merely using a sex toy. Erotic
(3)To have and use a corporate seal;
(4)To acquire and convey real or personal property; massage and body play can be a wonderfully exciting way to
(5)To enter into contracts; and make partner sex more sensual and loving, and can really spice
(6)To exercise such other powers as are granted to up a relationship!
corporations, subject to the limitations provided in this Perfect for anniversaries, bridal showers or bachelor parties,
Code and other laws. balms and oils are a less "taboo" form of sexual aide, making
(b)Local government units may continue using, modify, or them excellent gifts for just about everyone you know.
change their existing corporate seals: Provided, That newly (continued)
established local government units or those without corporate
seals may create their own corporate seals which shall be
registered with the Department of the Interior and Local
Cr ea tio n of L ocal Gov ern me nt Uni ts
Government: Provided, further, That any change of corporate
seal shall also be registered as provided herein.
(c)Unless otherwise provided in this Code, contract may be 1987 Constitution ARTX: Local Government
entered into by the local chief executive in behalf of the local
government unit without prior authorization by the sanggunian Sec1. The territorial and political subdivisions of the Republic of
concerned. A legible copy of such contract shall be posted at a the Philippines are the provinces, cities, municipalities, and
conspicuous place in the provincial capitol or the city, municipal barangays. There shall be autonomous regions in Muslim
or barangay hall. Mindanao and the Cordilleras as hereinafter provided.
(d)Local government units shall enjoy full autonomy in the
exercise of their proprietary functions and in the management of Sec7. Local governments shall be entitled to an equitable share in
their economic enterprises, subject to the limitations provided in the proceeds of the utilization and development of the national
this Code and other applicable laws. wealth within their respective areas, in the manner provided by

The Apocryphal Maggots:


Rainier, Chrisgel, Corina, Geoffry, Grace and Sylvie Blanche
The Flibbertigibbet Worms:
Golda, Gladys and Melyjane
CA VE AT: By simply r ea ding this re vi ew er a t the end o f the sem este r wi ll (hopef ully) guar antee you r pas sing this cou rse. D rink mode rat ely .
Amusin S C A r y
gly
THE C2005 LOCAL GOVERNMENT REVIEWER - 27 -
law, including sharing the same with the inhabitants by way of size of its population, as expected of the local government
direct benefits. unit concerned;
(b) Population. - It shall be determined as the total number of
inhabitants within the territorial jurisdiction of the local
Sec10. No province, city, municipality, or barangay may be
government unit concerned; and
created, divided, merged, abolished, or its boundary
(c) Land Area. - It must be contiguous, unless it comprises two
substantially altered, except in accordance with the criteria
or more islands or is separated by a local government unit
established in the local government code and subject to
independent of the others; properly identified by metes and
approval by a majority of the votes cast in a plebiscite in the
bounds with technical descriptions; and sufficient to provide
political units directly affected.
for such basic services and facilities to meet the
requirements of its populace. Compliance with the
Sec11. The Congress may, by law, create special metropolitan foregoing indicators shall be attested to by the Department
political subdivisions, subject to a plebiscite as set forth in of Finance (DOF), the National Statistics Office (NSO), and
Section 10 hereof. The component cities and municipalities shall the Lands Management Bureau (LMB) of the Department
retain their basic autonomy and shall be entitled to their own of Environment and Natural Resources(DENR).
local executive and legislative assemblies. The jurisdiction of the
metropolitan authority that will thereby be created shall be Sec8, LGC: Division and Merger. – Division and merger of
limited to basic services requiring coordination. existing local government units shall comply with the same
requirements herein prescribed for their creation: Provided,
however, That such division shall not reduce the income,
Sec15. There shall be created autonomous regions in Muslim population, or land area of the local government unit or units
Mindanao and in the Cordilleras consisting of provinces, cities, concerned to less than the minimum requirements prescribed in
municipalities, and geographical areas sharing common and this Code: Provided, further, That the income classification of the
distinctive historical and cultural heritage, economic and social original local government unit or units shall not fall below its
structures, and other relevant characteristics within the current income classification prior to such division. The income
framework of this Constitution and the national sovereignty as classification of local government units shall be updated within
well as territorial integrity of the Republic of the Philippines. six (6) months from the effectivity of this Code to reflect the
changes in their financial position resulting from the increased
Sec16. The President shall exercise general supervision over revenues as provided herein.
autonomous regions to ensure that laws are faithfully executed.
Sec9, LGC: Abolition of Local Government Units. – A local
government unit may be abolished when its income, population,
Sec19. The first Congress elected under this Constitution shall, or land area has been irreversibly reduced to less than the
within eighteen months from the time of organization of both minimum standards prescribed for its creation under Book III of
Houses, pass the organic acts for the autonomous regions in this Code, as certified by the national agencies mentioned in
Muslim Mindanao and the Cordilleras. Section 17 hereof to Congress or to the sanggunian concerned,
as the case may be.
The law or ordinance abolishing a local government unit shall
specify the province, city, municipality, or barangay with which
Local Government Code the local government unit sought to be abolished will be
incorporated or merged.
Sec6, LGC: Authority to Create Local Government Units. – A
local government unit may be created, divided, merged, Sec10, LGC: Plebiscite Requirement. – No creation, division,
abolished, or its boundaries substantially altered either by law merger, abolition, or substantial alteration of boundaries of local
enacted by Congress in the case of a province, city, municipality, government units shall take effect unless approved by a majority
or any other political subdivision, or by ordinance passed by the of the votes cast in a plebiscite called for the purpose in the
sangguniang panlalawigan or sangguniang panlungsod political unit or units directly affected. Said plebiscite shall be
concerned in the case of a barangay located within its territorial conducted by the Commission on Elections (COMELEC) within
jurisdiction, subject to such limitations and requirements one hundred twenty (120) days from the date of effectivity of the
prescribed in this Code. law or ordinance effecting such action, unless said law or
ordinance fixes another date.
Sec7, LGC: Creation and Conversion. – As a general rule, the
creation of a local government unit or its conversion from one Creation of the Barangay
level to another level shall be based on verifiable indicators of Sec385: Manner of Creation. – A barangay may be created,
viability and projected capacity to provide services, to wit: divided, merged, abolished, or its boundary substantially altered,
(a) Income - It must be sufficient, based on acceptable by law or by an ordinance of the sangguniang panlalawigan or
standards, to provide for all essential government facilities sangguniang panlungsod, subject to approval by a majority of
and services and special functions commensurate with the the votes cast in a plebiscite to be conducted by the Comelec in

The Apocryphal Maggots:


Rainier, Chrisgel, Corina, Geoffry, Grace and Sylvie Blanche
The Flibbertigibbet Worms:
Golda, Gladys and Melyjane
CA VE AT: By simply r ea ding this re vi ew er a t the end o f the sem este r wi ll (hopef ully) guar antee you r pas sing this cou rse. D rink mode rat ely .
Amusin S C A r y
gly
THE C2005 LOCAL GOVERNMENT REVIEWER - 28 -
the local government unit or units directly affected within such creation to less than the minimum requirements prescribed
period of time as may be determined by the law or ordinance herein.
creating said barangay. In the case of the creation of barangays (b) The territorial jurisdiction of a newly-created municipality
by the sangguniang panlalawigan, the recommendation of the shall be properly identified by metes and bounds. The
sangguniang bayan concerned shall be necessary. requirement on land area shall not apply where the
municipality proposed to be created is composed of one (1)
Sec386: Requisites for Creation. – or more islands. The territory need not be contiguous if it
(a) A barangay maybe created out of a contiguous territory comprises two (2) or more islands.
which has apopulation of at least two thousand (2,000) (c) The average annual income shall include the income
inhabitants ascertified by the National Statistics Office accruing to the general fund of the municipality concerned,
except in cities and municipalities within Metro Manila and exclusive of special funds, transfers and non-recurring
other metropolitan political subdivisions or in highly income.
urbanized cities where such territory shall have a certified (d) Municipalities existing as of the date of the effectivity of this
population of at least five thousand (5,000) inhabitants: Code shall continue to exist and operate as such. Existing
Provided, That the creation thereof shall not reduce the municipal districts organized pursuant to presidential
population of the original barangay or barangays to less issuances or executive orders and which have their
than the minimum requirement prescribed herein. respective set of elective municipal officials holding office at
To enhance the delivery of basic services in the indigenous the time of the effectivity of this Code shall henceforth be
cultural communities, barangays may be created in such considered as regular municipalities.
communities by an Act of Congress, notwithstanding the
above requirement. Creation of the City
(b) The territorial jurisdiction of the new barangay shall be Sec449: Manner of Creation. – A city may be created, divided,
properly identified by metes and bounds or by more or less merged, abolished, or its boundary substantially altered, only by
permanent natural boundaries. The territory need not be an Act of Congress, and subject to approval by a majority of the
contiguous if it comprises two (2) or more islands. votes cast in a plebiscite to be conducted by the Comelec in the
(c) The governor or city mayor may prepare a consolidation local government unit or units directly affected. Except as may
plan for barangays, based on the criteria prescribed in this otherwise be provided in such Act, the plebiscite shall be held
Section, within his territorial jurisdiction. The plan shall be within one hundred twenty (120) days from the date of its
submitted to the sangguniang panlalawigan or effectivity.
sangguniang panlungsod concerned for appropriate action.
In the case of municipalities within the Metropolitan Manila Sec450: Requisites for Creation. –
area and other metropolitan political subdivisions, the (a) A municipality or a cluster of barangays may be converted
barangay consolidation plan shall be prepared and into a component city if it has an average annual income,
approved by the sangguniang bayan concerned. as certified by the Department of Finance, of at least
Twenty million pesos (P20,000,000.00) for the last two (2)
Creation of the Municipality consecutive years based on 1991 constant prices, and if it
Sec441: Manner of Creation. – A municipality may be created, has either of the following requisites:
divided, merged, abolished, or its boundary substantially altered (i). a contiguous territory of at least one hundred (100)
only by an Act of Congress and subject to the approval by a square kilometers, as certified by the Lands
majority of the votes cast in a plebiscite to be conducted by the Management Bureau; or,
Comelec in the local government unit or units directly affected. (ii). a population of not less than one hundred fifty
Except as may otherwise be provided in the said Act, the thousand (150,000) inhabitants, as certified by the
plebiscite shall be held within one hundred twenty (120) days National Statistics Office: Provided, That, the creation
from the date of its effectivity. thereof shall not reduce the land area, population, and
income of the original unit or units at the time of said
Sec442: Requisites for Creation. – creation to less than the minimum requirements
(a) A municipality may be created if it has an average annual prescribed herein.
income, as certified by the provincial treasurer, of at least (b) The territorial jurisdiction of a newly-created city shall be
Two million five hundred thousand pesos (P=2,500,000.00) properly identified by metes and bounds. The requirement
for the last two (2) consecutive years based on the 1991 on land area shall not apply where the city proposed to be
constant prices; a population of at least twenty-five created is composed of one (1) or more islands. The
thousand (25,000) inhabitants as certified by the National territory need not be contiguous if it comprises two (2) or
Statistics Office; and a contiguous territory of at least fifty more islands.
(50) square kilometers as certified by the Lands (c) The average annual income shall include the income
Management Bureau: Provided, That the creation thereof accruing to the general fund, exclusive of special funds,
shall not reduce the land area, population or income of the transfers, and non-recurring income.
original municipality or municipalities at the time of said
Creation of the Province

The Apocryphal Maggots:


Rainier, Chrisgel, Corina, Geoffry, Grace and Sylvie Blanche
The Flibbertigibbet Worms:
Golda, Gladys and Melyjane
CA VE AT: By simply r ea ding this re vi ew er a t the end o f the sem este r wi ll (hopef ully) guar antee you r pas sing this cou rse. D rink mode rat ely .
Amusin S C A r y
gly
THE C2005 LOCAL GOVERNMENT REVIEWER - 29 -
Sec460: Manner of Creation. – A province may be created, warts, sores, or discolorations on genitals, and flu-like
divided, merged, abolished, or its boundary substantially altered, symptoms. There are other symptoms as well; see your health
only by an Act of Congress and subject to approval by a majority care provider if you have any questions.
of the votes cast in a plebiscite to be conducted by the Comelec Some STD's (such as Herpes) may be spread by contact with the
in the local government unit or units directly affected. The
general areas of the genitals, anus, or mouth. Rashes or
plebiscite shall be held within one hundred twenty (120) days
from the date of effectivity of said Act, unless otherwise provided outbreaks are often a sign of infection and contagiousness,
therein. though it is possible for some of these diseases to be transmitted
when there are no visible symptoms.
Sec461: Requisites for Creation. – (continued…)
(a) A province may be created if it has an average annual
income, as certified by the Department of Finance, of not
less than Twenty million pesos (P=20,000,000.00) based
on 1991 constant prices and either of the following Cr ea tio n
requisites:
(i). a contiguous territory of at least two thousand
Padilla, Jr. vs. COMELEC (1992)
(2,000) square kilometers, as certified by the Lands
Management Bureau; or,
(ii). a population of not less than two hundred fifty Facts:
thousand (250,000) inhabitants as certified by the In 1991, COMELEC promulgated a Resolution pursuant to RA7155
National Statistics Office: approving the creation of the Municipality of Tulay-na-Lupa in
Provided, That, the creation thereof shall not reduce Camarines Norte to be composed of 12 brgys in the Municipality of
the land area, population, and income of the original Labo subject to the approval by a majority of votes cast pursuant to
unit or units at the time of said creation to less than the Sec 10, ArtX of 1987 Constitution, and LGC. The plebiscite was to
minimum requirements prescribed herein. be held in the barangays comprising the proposed municipality and
(b) The territory need not be contiguous if it comprises two (2) the remaining areas of the mother municipality Labo. Plebiscite
or more islands or is separated by a chartered city or cities Board of Canvasser declared the rejection and disapproval of the
which do not contribute to the income of the province. proposed municipality after the turn-out where a majority voted
(c) The average annual income shall include the income against the creation. Gov. Padilla files a special civil action to set
accruing to the general fund, exclusive of special funds, aside conducted plebiscite and to undertaked a new one because
trust funds, transfers, and non-recurring income. plebiscite should have been conducted only in the 12 brgys
comprising the proposed municipality.
RA7878 (1995) Creating provinces of kalinga and apayao
RA8371 (1997), Sec18 IPRA, creation of tribal barangays for Held: Plebiscite valid.
ICCs/IPs living in contiguous zones and predominant Padilla’s contention that ArtX, Sec10 has deleted the words “unit
population, such brgy formed in accordance with LGC. or” in Sec3, ArtXI of the 1973 Constitution is untenable. As
RA9009 (2001) explained by ConCom Commissioner Davide during the 1986
RA9054 (2001), ArtIV, Sec19 ConCom debates the deletion of the words was asked because in
the plebiscite to be conducted, it must involve all the units affected.
When the law states that the plebiscite shall be conducted “in the
political units directly affected,” it means that residents of the
political entity who would be economically dislocated by the
STD Prevention and Treatment: General Comment: separation of a portion thereof have a right to vote in said
plebiscite.
Microtears in the gums (perhaps aggravated by recent brushing
or flossing), finger cuticles, penis, vagina, or anus can cause a TRIVIA: Gov Roy Padilla Jr. is the half-brother of Robin Padilla.
breach where a virus or bacterium may enter, even if the tears Magkapatid sila sa ama.
are not visible or noticeable. Pathogens can also enter the body
after landing on the eyes. Basically, one should avoid any
situation in which the body fluids of another might make their
way into one's bloodstream or contact one's mucous Torralba and Rugay vs. Municipality of Sibagat, Province of
membranes. Agusan del Sur and its Municipal Officers (1987)
Some diseases show no symptoms, and the people carrying
them may not know they are infected or show any noticeable Facts:
signs for years, though they can still transmit the disease to you. Petitioners challenge the validity of BP56 creating the municipality
of Sibagat, Province of Agusan del Sur, being violative of Sec3,
When symptoms are observed, common ones are pain when
ArtXI of the 1973Constitution. The 1983 LGC after enactment of
urinating, discharge or odd smell from genitals, itching,
BP 56. A plebiscite had been conducted among the people of the
burning, or pain in genitals (and sometime lower abdomen),

The Apocryphal Maggots:


Rainier, Chrisgel, Corina, Geoffry, Grace and Sylvie Blanche
The Flibbertigibbet Worms:
Golda, Gladys and Melyjane
CA VE AT: By simply r ea ding this re vi ew er a t the end o f the sem este r wi ll (hopef ully) guar antee you r pas sing this cou rse. D rink mode rat ely .
Amusin S C A r y
gly
THE C2005 LOCAL GOVERNMENT REVIEWER - 30 -
unit/s affected by the creation of the new municipality, who simply one of the modes by which a city may be created. Section
expressed approval. Officials had been appointed and had 10, Article X of the Constitution, allows the merger of local
assumed respective positions. Petitioners contend that BP 56 is government units to create a province city, municipality or
invalid for absence of the LGC at the time of its enactment. barangay in accordance with the criteria established by the Code.

Held: BP 56 is valid. Cawaling’s argument that the Municipality of Sorsogon alone


The 1973 Constitutional provision does not proscribe nor prohibit already qualifies to be upgraded to a component city goes into the
modification of territorial and political subdivisions before wisdom of R.A. No. 8806, a matter which we are not competent to
enactment of the 1983 LGC. What it means is that once said Code rule. In the exercise of judicial power, we are allowed only "to settle
is enacted, the creation, modification or dissolution of LGUs should actual controversies involving rights which are legally demandable
conform with the criteria thus laid down. Before enactment of 1983 and enforceable," and "may not annul an act of the political
LGC, the legislative power remains plenary except that the departments simply because we feel it is unwise or impractical”.
creation of the new LGU should be approved by the people
concerned in a plebiscite called for the purpose. Contrary to petitioner's assertion, there is only one subject
embraced in the title of the law, that is, the creation of the City of
CAB is different from Tan v Comelec since in that case, the LGC Sorsogon. The abolition/cessation of the corporate existence of the
already existed at the time the challenged statute (BP885) was Municipalities of Bacon and Sorsogon due to their merger is not a
enacted. Second, BP885 confined the plebiscite to the “proposed subject separate and distinct from the creation of Sorsogon City.
new province” to the exclusion of the voters in the remaining areas, Such abolition/cessation was but the logical, natural and inevitable
in contravention of the Const and of the LGC that plebiscite should consequence of the merger. It is well-settled that the "one title-one
be held “in the unit or units affected”. Third, the requisite area for subject" rule does not require the Congress to employ in the title of
creation of a new province was not complied with in BP885. the enactment language of such precision as to mirror, fully index
Finally, “Indecent haste” attended the enactment of BP885 and the or catalogue all the contents and the minute details therein. The
holding of the plebiscite thereafter rule is sufficiently complied with if the title is comprehensive
enough as to include the general object which the statute seeks to
Cruz, Concurring – on the assumption that the required plebiscite effect, and where, as here, the persons interested are informed of
(although not questioned) complied with the Constitutional the nature, scope and consequences of the proposed law and its
provision, and was duly held “in the unit or units affected”  not operation. Moreover, this Court has invariably adopted a liberal
only in the proposed municipality but also in the mother rather than technical construction of the rule "so as not to cripple or
municipality in line with Tan vs. Comelec, reversing Paredes vs. impede legislation."
Exec. Sec. and Lopez vs. Metro Manila Commission (where these
cases held that plebiscite could be confined only to the political unit The 120-day period within which to conduct the plebiscite starts
proposed to be created) from the date of Act’s effectivity (i.e., after publication in at least
two (2) newspapers of general and local circulation). Quite plainly,
the last sentence of Section 10 mandates that the plebiscite shall
Cawaling vs. COMELEC (2001) be conducted within 120 days from the date of the effectivity of the
law, not from its approval. While the same provision allows a law or
Facts: ordinance to fix "another date" for conducting a plebiscite, still such
Pres. Estrada signed into law R.A. No. 8806, an (Act Creating The date must be reckoned from the date of the effectivity of the law.
City Of Sorsogon). Pursuant to Section 10, Article X of the
Constitution, COMELEC conducted a plebiscite in the
Municipalities of Bacon and Sorsogon and submitted the matter for Pelaez vs. Auditor General (1965)
ratification. The Plebiscite City Board of Canvassers (PCBC)
proclaimed the creation of the City of Sorsogon as having been Facts:
ratified and approved by the majority of the votes cast in the From Sept 4 - Oct 29,1964 the President of the Philippines,
plebiscite. Cawaling, filed on 2 petitions seeking the annulment of purporting to act pursuant to Sec 68 of the Revised Administrative
the plebiscite and seeking to enjoin the further implementation of Code, issued EOs 93 to 121, 124 and 126 to 129, creating 33
R.A. No. 8806 for being unconstitutional. "During the pendency of municipalities. Soon after V.P. Pelaez instituted an action against
these cases (May 2001 elections), the newly-created Sorsogon the Auditor General, to restrain him from passing in audit any
City had the first election of its officials. Since then, the City expenditure of public funds in implementation of said executive
Government of Sorsogon has been regularly discharging its orders and/or any disbursement by said municipalities. Pelaez
corporate and political powers pursuant to its charter, R.A. No. alleges that the EOs are null and void, upon the ground that said
8806. Section 68 has been impliedly repealed by RA 2370 (Barrio
Charter Act) and constitutes an undue delegation of legislative
Held: RA 8806 constitutional and plebiscite valid. power.
Petitioner's constricted reading of Section 450(a) of the Code is
erroneous. The phrase "A municipality or a cluster of barangays Held: EOs were null and void.
may be converted into a component city" is not a criterion but

The Apocryphal Maggots:


Rainier, Chrisgel, Corina, Geoffry, Grace and Sylvie Blanche
The Flibbertigibbet Worms:
Golda, Gladys and Melyjane
CA VE AT: By simply r ea ding this re vi ew er a t the end o f the sem este r wi ll (hopef ully) guar antee you r pas sing this cou rse. D rink mode rat ely .
Amusin S C A r y
gly
THE C2005 LOCAL GOVERNMENT REVIEWER - 31 -
When Republic Act No. 2370 became effective, barrios may "not creating a barrio does not, warrant the inference of statutory
be created or their boundaries altered nor their names changed" prohibition for creating a municipality. For although municipalities
except by Act of Congress or of the corresponding provincial board consist of barrios, there is nothing in the statute that would
"upon petition of a majority of the voters in the areas affected" and preclude creation of new municipalities out of pre-existing barrios.
the "recommendation of the council of the municipality or It is not contrary to the logic of local autonomy to be able to create
municipalities in which the proposed barrio is situated." larger political units and unable to create smaller ones. The smaller
the unit of local government, the lesser is the need for the national
Respondent claims the President can upon the theory that a new government's intervention in its political affairs. Furthermore, for
municipality can be created without creating new barrios, such as, practical reasons, local autonomy cannot be given from the top
by placing old barrios under the jurisdiction of the new municipality. downwards. The national government, in such a case, could still
This theory overlooks, that the statutory denial of the presidential exercise power over the supposedly autonomous unit, e.g.,
authority to create a new barrio implies a negation of the bigger municipalities, by exercising it over the smaller units that comprise
power to create municipalities, each of which consists of several them, e.g., the barrios. A realistic program of decentralization
barrios. Founded upon logic and experience, it cannot be offset therefore calls for autonomy from the bottom upwards, so that it is
except by a clear manifestation of the intent of Congress to the not surprising for Congress to deny the national government some
contrary, and no such manifestation, subsequent to the passage of power over barrios without denying it over municipalities. For this
Republic Act No. 2370. has been brought to our attention. reason, I disagree with the majority view that because the
President could not create a barrio under Republic Act 2370, a
Respondent alleges that the power of the President to create fortiori he cannot create a municipality.
municipalities under section 68 of the Revised Administrative Code
does not amount to an undue delegation of legislative power,
relying upon the allegedly settled case of Municipality of Cardona  If a municipality entered into contracts before the declaration
vs. Municipality of Binañgonan (36 Phil. 547). Such claim is of nullity, it must still comply with it since the operative fact is that
untenable, for said case involved, not the creation of a new its rights and obligations must be complied with. (Municipality of
municipality, but a mere transfer of territory — from an already Malabang case).
existing municipality (Cardona) to another municipality
(Binañgonan), likewise, existing at the time of and prior to said
transfer in consequence of the fixing and definition, pursuant to Act  Not all LGUs created by Executive void. In the Municipality of
No. 1748, of the common boundaries of two municipalities. San Narcisco case, the municipal district was a de facto
corporaton for only after 30 year of existing was it questioned; All
It is obvious, however, that the power to fix such common those years government had acknowledged its existence plus Sec
boundary, in order to avoid or settle conflicts of jurisdiction 442(d) cures defect.
between adjoining municipalities, may partake of an administrative
nature but the authority to create municipal corporations is Aboli tion
essentially legislative in nature.
Although Congress may delegate to another branch of the Sultan Usman Sarangani vs. COMELEC (2000)
government the power to fill in the details in the execution,
enforcement or administration of a law, it is essential, to forestall a Facts:
violation of the principle of separation of powers, that said law: (a) A petition for annulment of several precincts and of book voters in
be complete in itself — it must set forth therein the policy to be Mandalum, Lanao del Sur was filed with COMELEC. Mandalum
executed, carried out or implemented by the delegate — and (b) fix Mayor Usman Sarangani and 23 brgy chairmen opposed the
a standard — the limits of which are sufficiently determinate or petition and claimed that the petition was for the purpose of
determinable — to which the delegate must conform in the diminishing the bailiwicks of the mayor. A Task Force Investigation
performance of his functions. Section 68 of the Revised Team was created to investigate the alleged ghost precincts. It
Administrative Code does not meet these requirements for a valid reported that:
delegation of the power to fix the details in the enforcement of a the supposed Brgy Padian Torogan (PT) means a cemetery
law. Even if it did not entail an undue delegation of legislative and not a residential place
powers, as it certainly does, said Section 68, as part of the only 2 structures in the area i.e. roofless concrete house and
Revised Administrative Code, approved on March 10, 1917, must wooden structure w/out walls and roof
be deemed repealed by the subsequent adoption of the no one answered affirmatively when asked if they were
Constitution, in 1935, which is utterly incompatible and inconsistent registered voters or residents of brgy PT.
with said statutory enactment. On the basis of the report, COMELEC issued an order finding PT
as ghost precinct and that it shall be excluded from the special
BENGZON, J.P., J., concurring and dissenting: elections in Mandalum. Sarangani filed this petition urging the
Since the Constitution repealed Section 68 as far back as 1935, it nullification of the COMELEC order.
is academic to ask whether Republic Act 2370 likewise has
provisions in conflict with Section 68 so as to repeal it. Suffice it to Held: COMELEC did not act with GAOD.
state, at any rate, that statutory prohibition on the President from

The Apocryphal Maggots:


Rainier, Chrisgel, Corina, Geoffry, Grace and Sylvie Blanche
The Flibbertigibbet Worms:
Golda, Gladys and Melyjane
CA VE AT: By simply r ea ding this re vi ew er a t the end o f the sem este r wi ll (hopef ully) guar antee you r pas sing this cou rse. D rink mode rat ely .
Amusin S C A r y
gly
THE C2005 LOCAL GOVERNMENT REVIEWER - 32 -
The determination of WON a certain election precinct actually The issuance of the COMELEC resolution in this case was
exists or not and whether the voters registered in said precinct are pursuant to Sec. 10 of LGCode and is thus a ministerial duty of the
real voters is a factual matter which if based on duly supported by COMELEC that is enjoined by law and part and parcel of its
evidence are conclusive upon the SC, more so in the absence of a administrative functions. It involves no exercise of discretionary
substantiated attack on the validity of the same. Upon the records, authority. It is also not an exercise of quasi-judicial power to hear
SC found that COMELEC had exerted efforts to investigate and and resolve controversies defining the rights and duties of the
verify the facts in the report, hence its conclusion that there were parties relative to the conduct of elections of public officers and the
no inhabitants in PT. If there were no inhabitants, then there can be enforcement of election laws.
no registered voters or the registered voters may have left.
The COMELEC resolution was merely an incident of its inherent
It is not impossible for a certain brgy not to actually have any administrative functions over the conduct of plebiscites, thus it may
inhabitants considering that people migrate. A barangay may not be deemed a “final order” reviewable by the SC. Any question
officially exist on record and the fact that nobody resides in the pertaining to the validity of the resolutions may well be taken in an
place does not result in its automatic cessation as a unit of local ordinary civil actions before RTC.
government. Under the LGCode, the abolition of a government unit
may be done by Congress in the case of a province, city,
municipality or any other political subdivision. In the case of a brgy, Inc ome
except in Metro Manila and cultural communities, it may be done
by the Sangguniang Panlalawigan or Panglungsod concerned Alvarez vs. Guingona (1996)
subject to the mandatory requirement of a plebiscite conducted for
the purpose in the political units affected. Facts:
Petitioners, assail the validity of RA7720 (Santiago City Act)
The right of suffrage is not tampered with when a list of fictitious because: (1) It allegedly did not originate exclusively in the House
voters is excluded from an electoral exercise. No voter is of Representatives as mandated by Sec24, Art6 of the 1987
disenfranchised because no such voter exists. Such exclusion of Constitution and (2) the Municipality of Santiago has not met the
non-existent voters all the more protects the validity and credibility minimum average annual income required under Sec450 of
of the electoral process because electoral will would not be 1991LGC in order to be converted into a component city.
rendered nugatory.
Held: RA 7720 complied with Const requirements.
Petitioners’ claim that the law did not originate exclusively in the
Salva vs. Makalintal (2000) House of Representatives because a bill of same import (SB1243)
was passed in the Senate is untenable because it cannot be
Facts: denied that HB8817 was filed in the House first (April 19, 1993)
The Sangguniang Panlalawigan (SP) of Batangas issued an before SB1243 was filed in the Senate (May 19, 1993), which they
Ordinance abolishing Brgy. San Rafael and its merger with Brgy. themselves admitted. Senate clearly held in abeyance any action
Dacanlao and instructed COMELEC to conduct the required on SB1243 until it received HB9917 already approved on 3rd
plebiscite. The Ordinance was vetoed by the Governor. Hence, the reading from the House. The filing in the Senate of a substitute bill
SP issued a Resolution affirming the ordinance and overriding the in anticipation of its receipt of the bill from the House does not
veto of the Governor. COMELEC then issued the a Resolution contravene the Constitution.
providing for the rules and regulations governing the conduct of the
plebiscite. Petitioners filed a class suit against SP of Batangas, SB Santiago has met the minimum requirement. IRAs form part of the
of Calaca for the annulment of the Ordinance and the resolutions. income of LGUs. They are items of income because they form part
TC denied issuance of TRO in an Order saying that the TRO of the gross accretion of the funds of the LGU. They regularly and
sought is directed only to the COMELEC Resolution and action automatically accrue to the local treasury without need of any
against a COMELEC resolution must be brought before further action on the part of the LGU. (Sec7, LGC) They thus
SC.Petitioners contend that RTC has jurisdiction over TRO. constitute income which the local government can invariably rely
upon as the source of much needed funds.
Held: RTC has jurisdiction over TRO.
Sec. 7, Art. IX-A of 1987 Consti provides that any decision, order or
ruling of each Commission such as COMELEC, may be brought on Sub -Pr ov in ce s
certiorari to SC by the aggrieved party w/in 30 days from receipt.
The COMELEC’s powers are classified either as 1987 Constitution, Art XVIII: Transitory Provisions
adjudicatory/quasi-judicial or administrative or ministerial. Final Sec9: A sub-province shall continue to exist and operate until it is
orders, rulings and decisions of COMELEC reviewable by SC are converted into a regular province or until its component
those rendered in actions or proceedings before COMELEC and municipalities are reverted to the mother province.
taken cognizance of said body in the exercise of its adjudicatory or
quasi-judicial powers.

The Apocryphal Maggots:


Rainier, Chrisgel, Corina, Geoffry, Grace and Sylvie Blanche
The Flibbertigibbet Worms:
Golda, Gladys and Melyjane
CA VE AT: By simply r ea ding this re vi ew er a t the end o f the sem este r wi ll (hopef ully) guar antee you r pas sing this cou rse. D rink mode rat ely .
Amusin S C A r y
gly
THE C2005 LOCAL GOVERNMENT REVIEWER - 33 -
Local Government Code (1991) – RA7160 from voting provincial officers since LGC had no specific provision
Sec462. Existing Sub-Provinces – Existing sub-provinces are that sub-province voters shall no longer be allowed to vote for
hereby converted into regular provinces upon approval by a provincial officers in case they vote against conversion
majority of the votes cast in a plebiscite to be held in the said
sub-provinces and the original provinces directly affected. The Held: COMELEC acted without jurisdiction.
plebiscite shall be conducted by the COMELEC simultaneously Although Sec 462 LGC completely addresses an eventuality where
with the national elections following the effectivity of this Code. people of both the original and the new district (to be created)
The new legislative districts created as a result of such agree to the proposed creation of the new district (by appointment
conversion shall continue to be represented in Congress by the of President, if none appointed yet, of the provincial officers of new
duly-elected representatives of the original districts out of which district), the provision is incomplete if the proposed conversion is
said new provinces or districts were created until their own rejected by those affected.
representatives shall have been elected in the next regular Whatever incumbent elective positions exist under the present set-
congressional elections and qualified. up, it appears that in case of a negative vote, these sub-provincial
The incumbent elected officials of the said sub-provinces positions shall be filled by appointment of the President. But it
converted into regular provinces shall continue to hold office failed to foresee that in the event the negative vote prevails
until June 30, 1992. Any vacancy occurring in the offices naturally, the sub-province shall continue to be a part of the original
occupied by said incumbent elected officials, or resulting from province and continue to be represented by the provincial officials
expiration of their terms of office in case of a negative vote in the of the original province.
plebiscite results, shall be filled by appointment by the
President. The appointees shall hold office until their successors However, it would be useless to undo the plebiscite
shall have been elected in the regular local elections following conducted by COMELEC.
the plebiscite mentioned herein and qualified. After effectivity of  Voters overwhelmingly voted for the approval of the conversion
such conversion, the President shall fill up the position of of Guimaras into a regular province. In this event, President shall
governor of the newly-created province through appointment if appoint the provincial officials for the newly created province of
none has yet been appointed to the same as hereinbefore Guimaras (which in fact President had already done). Guimaras
provided, and shall also appoint a vice-governor and the other being a regular and politically independent province from Iloilo,
members of the sangguniang panglalawigan, all of whom shall there is no more legal basis to call for a special election for the
likewise hold office until their successors shall have been 3municipalities in Guimaras for purpose of electing their provincial
elected in the nest regular local elections and qualified. officials.
All qualified appointive officials and employees in the career
service of the said sub-provinces at the time of their conversion TRIVIA: In Guimaras , you can eat its famous mangoes but you
into regular provinces shall continue in office in accordance with cannot take home the mango seed (“buto ng mangga”).
civil service law, rules and regulations.

Griño vs. COMELEC (1992) Selection and Transfer of Local Government Sites

Facts: Sec11, LGC: Selection and Transfer of Local Government Site,


Sub-province of Guimaras is composed of 3 municipalities which Offices and Facilities –
constitute a part of 2nd district of Iloilo. In previous elections, voters (a) The law or ordinance creating or merging local government
from the 3 municipalities were allowed to vote for the provincial units shall specify the seat of government from where
officials of entire province of Iloilo. governmental and corporate services shall be delivered. In
When 1991LGC came into effect, Sec462 called for the conversion selecting said site, factors relating to geographical
of existing subprovinces into regular provinces upon approval by a centrality, accessibility, availability of transportation and
majority of the votes cast in a plebiscite to be held in the areas communication facilities, drainage and sanitation,
directly affected by the conversion. It also directed holding of development and economic progress, and other relevant
plebiscite simultaneously with national elections following considerations shall be taken into account.
effectivity of LGC. (b) When conditions and developments in the local
COMELEC conducted a plebiscite for the conversion of Guimaras government unit concerned have significantly changed
simultaneously with the May1992 elections. It issued rules and subsequent to the establishment of the seat of government,
regulations governing the plebiscite – that all registered voters of its sanggunian may, after public hearing and by a vote of
Iloilo (except Iloilo City) and Guimaras qualified to vote in the 2/3 of all its members, transfer the same to a site better
provincial elections were qualified to vote in the plebiscite. suited to its needs: Provided, however, That no such
The ballots used contained the plebiscite question but petitioners transfer shall be made outside the territorial boundaries of
observed that ballots distributed to 3municipalities of Guimaras did the local government unit concerned.
not contain any space or provision for election of provincial The old site, together with the improvements thereon, may
officials. Petitioners allege that COMELEC acted without be disposed of by sale or lease or converted to such other
jurisdiction and with GAOD when it disallowed Guimaras voters

The Apocryphal Maggots:


Rainier, Chrisgel, Corina, Geoffry, Grace and Sylvie Blanche
The Flibbertigibbet Worms:
Golda, Gladys and Melyjane
CA VE AT: By simply r ea ding this re vi ew er a t the end o f the sem este r wi ll (hopef ully) guar antee you r pas sing this cou rse. D rink mode rat ely .
Amusin S C A r y
gly
THE C2005 LOCAL GOVERNMENT REVIEWER - 34 -
use as the sanggunian concerned may deem beneficial to Lastly, RA 8535 does not amend the Constitution. The Ordinance
the local government unit concerned and its inhabitants. attached to the Constitution merely apportions the seats of the
(c) Local government offices and facilities shall not be House of Representatives to the different legislative districts.
transferred, relocated, or converted to other uses unless Nowhere does it provide that Metro Manila shall be forever be
public hearings are first conducted for the purpose and the composed of only 0f 17 cities and municipalities.
concurrence of the majority of all the members of the
sanggunian concerned is obtained. Political and Corporate Nature of Local Government Units

Sec5, LGC: Rules of Interpretation – In the interpretation of the


Samson vs. Aguirre (1999) provisions of this Code, the following rules shall apply:
xxx
Facts: (d) Rights and obligations existing on the date of effectivity of
In 1998, President Ramos signed into law RA 8535 creating the this Code and arising out of contracts or any other source of
City of Novaliches, out of 15 barangays of QC. Petitioner prestation involving a local government unit shall be governed
challenges the constitutionality of RA 8535 because (1) RA 8535 by the original terms and conditions of said contracts or the law
failed to conform with the LGC, Sec 7, 11 (a) and 450 (a) as to in force at the time such rights were vested; and
requirements of income, population, and land area, seat of xxx
government and no adverse effect to Quezon City; (2) RA 8535
violated IRR 11(b) (1) and (2) as to furnishing a copy of the petition Sec15, LGC: Political and Corporate Nature of LGUs – Every
calling creation of Novaliches City to the QC Council and (3) RA local government unit created or recognized under this Code is
8535 will amend the Constitution a body politic and corporate endowed with powers to be
exercised by it in conformity with law. As such, it shall exercise
Held: NO, RA 8535 perfectly lawful and constitutional. powers as a political subdivision of the National Government
Rule: Every statue is presumed valid. Petitioner did not overcome and as a corporate entity representing the inhabitants of its
the presumption of validity. territory.
As to alleged lack of certifications: Petitioner did not present proof,
but only mere allegations, that no certifications were submitted to
the House of Representatives Committee on Local Government. Lidasan vs. COMELEC (1964)
So presumption stands that the law was passed had complied with
all the requisites. Facts:
On June 18, 1966, the Chief Executive signed into law known as
As to petitioner's argument that the oral manifestations made by RA 4790 creating the new municipality of Dianaton, Lanao del Sur.
the above mentioned gov't representatives were not enough Some of the barrios included in the new municipality came from
certification: the government officials were armed with official municipalities of Cotabato. Prompted by the coming elections,
statistics and reference materials. Petitioner also failed to show Comelec adopted its resolution of August 15, 1967, which affirms
that aside from the oral manifestations, these government officials the new municipality. As the law stood, twelve barrios - in two
did not submit written certifications. Hence, SC shall presume that municipalities in the province of Cotabato — are transferred to the
these requirements were met on the passage of the legislative act. province of Lanao del Sur. This brought about a change in the
boundaries of the two provinces. Apprised of this development, the
As to failure to specify the seat of government (as required by sec Office of the President, through the Assistant Executive Secretary,
11 of the LGC): Indeed, RA 8535 failed to provide for a seat of recommended to Comelec that the operation of the statute be
government for Novaliches. But this omission is not fatal to the suspended until "clarified by correcting legislation."
validity of RA 8535. Under section 12 of the LGC, Novaliches may Comelec, by resolution of September 20, 1967, stood by its own
still establish a seat of government after its creation. Note that interpretation, declared that the statute "should be implemented
while sec 12 of the LGC merely speaks of the site of government unless declared unconstitutional by the Supreme Court." Hence
centers, such site can also very well be the seat of government. the original action for certiorari and prohibition by Bara Lidasan.
As to adverse effect to QC: QC Mayor Ismael Mathay Jr. himself, Held: RA 4790 is unconstitutional.
who was present during the deliberations in the Senate committee
hearings, made no mention of any deleterious effects. It suffices if the title should serve the purpose of the constitutional
demand that it inform the legislators, the persons interested in the
As to the fact that the QC Council was not furnished a copy of the subject of the bill, and the public, of the nature, scope and
petition: the purpose of this IRR requirement is to duly inform the consequences of the proposed law and its operation. And this, to
local council of such petition and for them to have an opportunity to lead them to inquire into the body of the bill, study and discuss the
formulate comments. The matter at hand is widely publicized same, take appropriate action thereon, and, thus, prevent surprise
already in the media. Hence, the Council can not claim they were or fraud upon the legislators.
not informed. In the CAB, the title — "An Act Creating the Municipality of
Dianaton, in the Province of Lanao del Sur" — projects the

The Apocryphal Maggots:


Rainier, Chrisgel, Corina, Geoffry, Grace and Sylvie Blanche
The Flibbertigibbet Worms:
Golda, Gladys and Melyjane
CA VE AT: By simply r ea ding this re vi ew er a t the end o f the sem este r wi ll (hopef ully) guar antee you r pas sing this cou rse. D rink mode rat ely .
Amusin S C A r y
gly
THE C2005 LOCAL GOVERNMENT REVIEWER - 35 -
impression that solely the province of Lanao del Sur is affected by itself suffice for a finding of nullity by virtue of the constitutional
the creation of Dianaton. Not the slightest intimation is there that provision invoked. At the most, the statute to be free from the
communities in the adjacent province of Cotabato are incorporated insubstantial doubts about its validity must be construed as not
in this new Lanao del Sur town. The phrase "in the Province of including the barrios, located not in the municipalities of Butig and
Lanao del Sur," read without subtlety or contortion, makes the title Balabagan, Lanao del Sur, but in Parang and Buldon, Cotabato.
misleading, deceptive. For, the known fact is that the legislation The constitutional requirement is that no bill which may be enacted
has a two-pronged purpose combined in one statute: (1) it creates into law shall embrace more than one subject which shall be
the municipality of Dianaton purportedly from twenty-one barrios in expressed in the title of the bill. It is aimed against the evils of the
the towns of Butig and Balabagan, both in the province of Lanao so-called omnibus bills, and log-rolling legislation, and against
del Sur; and (2) it also dismembers two municipalities in Cotabato, surreptitious or unconsidered enactments. Where the subject of a
a province different from Lanao del Sur. bill is limited to a particular matter, the members of the legislature
The transfer of a sizeable portion of territory from one province to as well as the people should be informed of the subject of
another of necessity involves reduction of area, population and proposed legislative measures. This constitutional provision thus
income of the first and the corresponding increase of those of the precludes the insertion of riders in legislation, a rider being a
other. This is as important as the creation of a municipality. And provision not germane to the subject matter of the bill.
yet, the title did not reflect this fact. It is not to be narrowly construed though as to cripple or impede
proper legislation. The construction must be reasonable and not
RA 4790 cannot salvaged by th nullification of the portion which technical. It is sufficient if the title be comprehensive enough
took away the twelve barrios in the municipalities of Buldon and reasonably to include the general object which the statute seeks to
Parang in the other province of Cotabato. effect without expressing each and every end and means
The general rule is that where part of the statute is void, as necessary for the accomplishment of that object. Mere details need
repugnant to the Organic Law, while another part is valid, the valid not be set forth. The legislature is not required to make the title of
portion, if separable from the invalid, may stand and be enforced. the act a complete index of its contents. The constitutional
But in order to do this, the valid portion must be so far independent provision is satisfied if all parts of an act which relates to its subject
of the invalid portion that it is fair to presume that the Legislature find expression in its title.
would have enacted it by itself if they had supposed that they could It would follow therefore that the challenged legislation Republic
not constitutionally enact the other. Act No. 4790 is not susceptible to the indictment that the
Municipal corporations perform twin functions. Firstly. They serve constitutional requirement as to legislation having only one subject
as an instrumentality of the State in carrying out the functions of which should be expressed in his title was not met. The subject
government. Secondly. They act as an agency of the community in was the creation of the municipality of Dianaton. That was
the administration of local affairs. It is in the latter character that embodied in the title.
they are a separate entity acting for their own purposes and not a To avoid any doubt as to that validity of such statue, it must be
subdivision of the State. construed as to exclude from Dianaton all of such barrios
Consequently, several factors come to the fore in the consideration mentioned in RA 4790 found in municipalities outside Lanao del
of whether a group of barrios is capable of maintaining itself as an Sur. As thus interpreted, the statute can meet the test of the most
independent municipality. Amongst these are population, territory, rigid scrutiny. Nor is this to do violence to the legislative intent.
and income. It was apparently these same factors which induced What was created was a new municipality from barrios named as
the writing out of House Bill 1247 creating the town of Dianaton. found in Lanao del Sur. This construction assures precisely that.
When the foregoing bill was presented in Congress,
unquestionably, the totality of the twenty-one barrios— not nine
barrios—was in the mind of the proponent thereof. That this is so, Torio vs. Fontanilla (1978)
is plainly evident by the fact that the bill itself, thereafter enacted
into law, states that the seat of the government is in Togaig, which Facts:
is a barrio in the municipality of Buldon in Cotabato. And then the The Municipal Council of Malasiqui, Pangasinan, resolved to
reduced area poses a number of questions, which the SC may not celebrate the town fiesta and created a "Town Fiesta Executive
supply the answer to any of these. With the known premise that Committee" to undertake, manage and supervise the festivities.
Dianaton was created upon the basic considerations of The Executive Committee created a sub-committee on
progressive community, large aggregate population and sufficient "Entertainment and Stage", which constructed two stages, one for
income, we may not now say that Congress intended to create the "zarzuela" and another for "cancionan." During the program
Dianaton with only nine—of the original twenty-one—barrios, with people went up the "zarzuela" stage and before the play was over
a seat of government still left to be conjectured. For, this unduly the stage collapsed, pinning underneath one of the performers,
stretches judicial interpretation of congressional intent beyond resulting in his death.
credibility point. To do so, indeed, is to pass the line which The heirs of the deceased sued the municipality and the councilors
circumscribes the judiciary and tread on legislative premises. for damages. The municipality invoked inter alia the principal
defense that the holding of a town fiesta was an exercise of its
FERNANDO, J., dissenting: governmental function from which no liability can arise to answer
The mere fact that in the body of such statute barrios found in two for the negligence of any of its agents. The councilors maintained
other municipalities of another province were included does not of

The Apocryphal Maggots:


Rainier, Chrisgel, Corina, Geoffry, Grace and Sylvie Blanche
The Flibbertigibbet Worms:
Golda, Gladys and Melyjane
CA VE AT: By simply r ea ding this re vi ew er a t the end o f the sem este r wi ll (hopef ully) guar antee you r pas sing this cou rse. D rink mode rat ely .
Amusin S C A r y
gly
THE C2005 LOCAL GOVERNMENT REVIEWER - 36 -
that they merely acted as agents of the municipality in carrying out they may justly be regarded as its agents or servants, and the
the municipal ordinance. maxim of respondent superior applies." . . . (Dillon on Municipal
The trial court dismissed the complaint of a finding that the Corporations, 5th Ed., Vol. IV, p. 2879)
petitioners exercised due diligence and care of a good father of a
family in selecting a competent man to construct the stage and if it
collapsed it was due to forces beyond the control of the committee City of Manila vs. IAC (1989)
on entertainment and stage.
The Court of Appeals reversed. Facts:
In 1971, Vivencio Sto. Domingo died and was buried in the North
Held: The celebration a town fiesta is an undertaking in the Cemetery (NC; Lot 159), leased by the City to wife Irene for 50 yrs.
exercise of a municipality's government proprietary character Full payment of the rental fee is evidenced by official receipt which,
thus is liable. although appearing regular on its face, does not reflect the term of
The powers of a municipality are twofold in character — public, duration of the lease. City Mayor issued an Admin Order which
governmental, or political on the one hand, and corporate, private, prescribes a uniform procedure and guidelines in the processing of
or proprietary on the other. Governmental powers are those documents for the use and disposition of burial lots and plots within
exercised by the corporation in administering the powers of the NC. By virtue of said AO, it was believed that Lot 159 was leased
state and promoting the public welfare and they include the only for 5 years to the Sto. Domingos. In 1978, Vivencio’s remains
legislative, judicial, public, and political, Municipal powers on the were exhumed and placed in a bag and placed in the depository of
other hand are exercised for the special benefit and advantage of the cemetery. Sto. Domingos visited the lot but found a new lessee
the community and include those which are ministerial, private and therein. They inquired about the remains of Vivencio and was told
corporate. that she can simply look for the remains of her husband in the
warehouse of the cemetery. Irene claims that it was impossible to
In the CAB, Section 2282 of the Chapter on Municipal Law of the locate the remains of Vivencio in a depository containing
Revised Administrative Code simply gives authority to the thousands of sacks of human bones. Irene claims damages. City
municipality to accelebrate a yearly fiesta but it does not impose says it isn’t liable since operation of a public cemetery is a
upon it a duty to observe one. Holding a fiesta even if the purpose governmental function.
is to commemorate a religious or historical event of the town is in
essence an act for the special benefit of the community and not for Held: The operations and functions of a public cemetery is a
the general welfare of the public performed in pursuance of a proprietary function of the City
policy of the state. The mere fact that the celebration, as claimed, City of Manila is a political body corporate and as such is endowed
was not to secure profit or gain but merely to provide entertainment with the faculties of municipal corporations to be exercised by and
to the town inhabitants is not a conclusive test. For instance, the through its city government in conformity with law, and in its
maintenance of parks is not a source of income for the town, corporate name. It may therefore sue and be sued, and contract
nonetheless it is private undertaking as distinguished from the and be contracted with.
maintenance of public schools, jails, and the like which are for
public service. Easily, no governmental or public policy of the state Maintenance of parks, golf courses, cemeteries and airports,
is involved in the celebration of a town fiesta. among others, are recognized as municipal or city activities of a
proprietary character.
Since the injury was caused respect to the municipality’s
proprietary functions, the settled rule is that a municipal In the absence of special laws, the NC is a patrimonial property
corporation can be held liable to third persons ex contractu or ex (see Art 424 CC) of the City over which it exercises acts of
delicto if found negligent, which the CA found and held that there dominion. There is therefore no doubt that the NC is owned by the
was negligence. City in its proprietary or private character.
The municipality acting through its municipal council appointed The obligations arising from the contract of lease has the force of
Macaraeg as chairman of the sub-committee on entertainment and law between the parties in the CAB. The City’s breach of a
in charge of the construction of the "zarzuela" stage. Macaraeg contractual provision entitles the Sto. Domingos to damages
acted merely as an agent of the Municipality. Under the doctrine of
respondent superior mentioned petitioner is responsible or liable Under the doctrine of respondeat superior, the City is liable for the
for the negligence of its agent acting within his assigned tasks. tortius acts committed by its agents who failed to verify and check
". . . when it is sought to render a municipal corporation liable for the duration of the contract of lease.
the act of servants or agents, a cardinal inquiry is, whether they
are the servants or agents of the corporation. If the corporation
appoints or elects them, can control them in the discharge of their
Macasiano vs. Diokno
duties, can continue or remove them, can hold them responsible
for the manner in which they discharge their trust, and if those
duties relate to the exercise of corporate powers, and are for the Facts:
peculiar benefit of the corporation in its local or special interest, Municipality of Paranaque passed an Ordinance which authorized
the (1) closure of certain streets at Baclaran and (2) the

The Apocryphal Maggots:


Rainier, Chrisgel, Corina, Geoffry, Grace and Sylvie Blanche
The Flibbertigibbet Worms:
Golda, Gladys and Melyjane
CA VE AT: By simply r ea ding this re vi ew er a t the end o f the sem este r wi ll (hopef ully) guar antee you r pas sing this cou rse. D rink mode rat ely .
Amusin S C A r y
gly
THE C2005 LOCAL GOVERNMENT REVIEWER - 37 -
establishment of a flea market thereon. Such was passed encourage and support the development of appropriate and self-
pursuant to an MMC Ordinance authorizing and regulating the use reliant scientific and technological capabilities, improve public
of certain city streets, roads and open spaces within M.Mla. as morals, enhance economic prosperity and social justice,
sites for flea markets or vending areas. Ordinance was later promote full employment among their residents, maintain peace
approved by the M.Mla Authority subject to the certain conditions. and order, and preserve the comfort and convenience of their
Paranaque mayor entered into an agreement with Palanyag, a inhabitants.
service cooperative, for the establishment and operation of the flea
market. PNP Superintendent Macasiano ordered the confiscation Sangguniang Barangay
of stalls put up by Palanyag and the discontinuation of the Sec391, LGC: Powers, Duties, and Functions. – (a) The
operation of the flea market. TC upheld validity of Ordinance No. sangguniang barangay, as the legislative body of the barangay,
86 and enjoined Macasiano from enforcing his letter-order. shall:
(1) Enact ordinances as may be necessary to discharge the
Held: The ordinance by Paranaque authorizing the lease and responsibilities conferred upon it by law or ordinance and
use of public streets or thoroughfares as sites for flea market to promote the general welfare of the inhabitants therein;
is invalid. (2) Enact tax and revenue ordinances, subject to the
Streets are local roads used for public service and are therefore limitations imposed in this Code;
considered public properties. Properties of the local government (3) Enact annual and supplemental budgets in accordance
which are devoted to public service are deemed public and are with the provisions of this Code;
under the absolute control of Congress. Hence, local govts. Have (4) Provide for the construction and maintenance of
no authority whatsoever to control or regulate the use of public barangay facilities and other public works projects
properties unless specific authority is vested upon them by chargeable to the general fund of the barangay or such
Congress. other funds actually available for the purpose;
(5) Submit to the sangguniang panlungsod or sangguniang
Sec 10 Chapter II of the LGC, although authorizing LGUs to close bayan such suggestions or recommendations as it may
roads and similar public places, should be deemed limited by Art see fit for the improvement of the barangay or for the
424 CC which provides that properties of public dominion devoted welfare of the inhabitants thereof;
to public use and made available to the public in general are (6) Assist in the establishment, organization, and promotion
outside the commerce of man and cannot be disposed of or leased of cooperative enterprises that will improve the economic
by the LGC to private persons. condition and well-being of the residents;
(7) Regulate the use of multi-purpose halls, multi- purpose
Closure should be for the sole purpose of withdrawing the road or pavements, grain or copra dryers, patios and other post-
other public property from public use when circumstances show harvest facilities, barangay waterworks, barangay
that such property is no longer intended or necessary for public markets, parking areas or other similar facilities
use or service. When the property is already withdrawn from constructed with government funds within the jurisdiction
public use, it becomes patrimonial property of the LGU which it can of the barangay and charge reasonable fees for the use
then lawfully use or convey. thereof;
(8) Solicit or accept monies, materials and voluntary labor for
The right of the public to use the city streets may not be bargained specific public works and cooperative enterprises of the
away through contract. barangay from residents, land owners, producers and
merchants in the barangay; monies from grants-in-aid,
Even assuming that the municipality has the authority to pass the subsidies, contributions, and revenues made available to
disputed ordinance, it cannot be considered approved by the the barangays from national, provincial, city or municipal
M.Mla. Authority due to non-compliance with the imposed funds; and monies from other private agencies and
conditions. Also, the establishment of the flea market would not individuals: Provided, however, That monies or properties
help in solving the problem of congestion in Baclaran. donated by private agencies and individuals for specific
purposes shall accrue to the barangay as trust fund;
(9) Solicit or accept, in any or all the foregoing public works
General Welfare Clause and cooperative enterprises, such cooperation as is made
available by national, provincial, city, or municipal
Sec16, LGC: General Welfare. – Every local government unit agencies established by law to render financial, technical,
shall exercise the powers expressly granted, those necessarily and advisory assistance to barangays and to barangay
implied therefrom, as well as powers necessary, appropriate, or residents: Provided, however, That in soliciting or
incidental for its efficient and effective governance, and those accepting such cooperation, the sangguniang barangay
which are essential to the promotion of the general welfare. need not pledge any sum of money for expenditure in
Within their respective territorial jurisdictions, local government excess of amounts currently in the barangay treasury or
units shall ensure and support, among other things, the encumbered for other purposes;
preservation and enrichment of culture, promote health and (10) Provide compensation, reasonable allowances or per
safety, enhance the right of the people to a balanced ecology, diems as well as travel expenses for sangguniang

The Apocryphal Maggots:


Rainier, Chrisgel, Corina, Geoffry, Grace and Sylvie Blanche
The Flibbertigibbet Worms:
Golda, Gladys and Melyjane
CA VE AT: By simply r ea ding this re vi ew er a t the end o f the sem este r wi ll (hopef ully) guar antee you r pas sing this cou rse. D rink mode rat ely .
Amusin S C A r y
gly
THE C2005 LOCAL GOVERNMENT REVIEWER - 38 -
barangay members and other barangay officials, subject (24) Exercise such other powers and perform such other
to the budgetary limitations prescribed under Title Five, duties and functions as may be prescribed by law or
Book II of this Code: Provided, however, That no increase ordinance.
in the com- pensation or honoraria of the sangguniang
barangay members shall take effect until after the Sangguniang Bayan
expiration of the full term of all members of the Sec447, LGC: Powers, Duties, Functions and Compensation. –
sangguniang barangay approving such increase; (a) The sangguniang bayan, as the legislative body of the
(11) Hold fund-raising activities for barangay projects without municipality, shall enact ordinances, approve resolutions and
the need of securing permits from any national or local appropriate funds for the general welfare of the municipality and
office or agency. The proceeds from such activities shall its inhabitants pursuant to Section 16 of this Code and in the
be tax-exempt and shall accrue to the general fund of the proper exercise of the corporate powers of the municipality as
barangay: Provided, That in the appropriation thereof, the provided for under Section 22 of this Code, and shall:
specific purpose for which such fund-raising activity has (1) Approve ordinances and pass resolutions necessary for
been held shall be first satisfied: Provided, further, That an efficient and effective municipal government, and in
no fund-raising activities shall be held within a period of this connection shall:
sixty (60) days immediately preceding and after a national (i) Review all ordinances approved by the
or local election, recall, referendum, or plebiscite: sangguniang barangay and executive orders
Provided, finally, That said fund-raising activities shall issued by the punong barangay to determine
comply with national policy standards and regulations on whether these are within the scope of the
morals, health, and safety of the persons participating prescribed powers of the sanggunian and of the
therein. The sangguniang barangay, through the punong punong barangay;
barangay, shall render a public accounting of the funds (ii) Maintain peace and order by enacting measures
raised at the completion of the project for which the fund- to prevent and suppress lawlessness, disorder,
raising activity was under- taken; riot, violence, rebellion or sedition and impose
(12) Authorize the punong barangay to enter into contracts in penalties for the violation of said ordinances;
behalf of the barangay, subject to the provisions of this (iii) Approve ordinances imposing a fine not
Code;
exceeding Two thousand five hundred pesos
(13) Authorize the barangay treasurer to make direct
(P=2,500.00) or an imprisonment for a period not
purchases in an amount not exceeding One thousand
exceeding six (6) months, or both in the discretion
pesos (P1,000.00) at any one time for the ordinary and
of the court, for the violation of a municipal
essential administrative needs of the barangay;
ordinance;
(14) Prescribe fines in amounts not exceeding One thousand
(iv) Adopt measures to protect the inhabitants of the
pesos (P1,000.00) for violation of barangay ordinances;
municipality from the harmful effects of man-
(15) Provide for the administrative needs of the lupong
made or natural disasters and calamities and to
tagapamayapa and the pangkat ng tagapagkasundo;
provide relief services and assistance for victims
(16) Provide for the organization of community brigades,
during and in the aftermath of said disasters or
barangay tanod, or community service units as may be
calamities and their return to productive livelihood
necessary;
following said events;
(17) Organize regular lectures, programs, or fora on
(v) Enact ordinances intended to prevent, suppress
community problems such as sanitation, nutrition, literacy,
and impose appropriate penalties for habitual
and drug abuse, and convene assemblies to encourage
drunkenness in public places, vagrancy,
citizen participation in government;
mendicancy, prostitution, establishment and
(18) Adopt measures to prevent and control the proliferation of
maintenance of houses of ill repute, gambling and
squatters and mendicants in the barangay;
other prohibited games of chance, fraudulent
(19) Provide for the proper development and welfare of
devices and ways to obtain money or property,
children in the barangay by promoting and supporting
drug addiction, maintenance of drug dens, drug
activities for the protection and total development of
pushing, juvenile delinquency, the printing,
children, particularly those below seven (7) years of age;
distribution or exhibition of obscene or
(20) Adopt measures towards the prevention and eradication
pornographic materials or publications, and such
of drug abuse, child abuse, and juvenile delinquency;
other activities inimical to the welfare and morals
(21) Initiate the establishment of a barangay high school,
of the inhabitants of the municipality;
whenever feasible, in accordance with law;
(vi) Protect the environment and impose appropriate
(22) Provide for the establishment of a non-formal education
penalties for acts which endanger the
center in the barangay whenever feasible, in coordination
environment, such as dynamite fishing and other
with the Department of Education, Culture and Sports, ;
forms of destructive fishing, illegal logging and
(23) Provide for the delivery of basic services; and
smuggling of logs, smuggling of natural resources
products and of endangered species of flora and
fauna, slash and burn farming, and such other

The Apocryphal Maggots:


Rainier, Chrisgel, Corina, Geoffry, Grace and Sylvie Blanche
The Flibbertigibbet Worms:
Golda, Gladys and Melyjane
CA VE AT: By simply r ea ding this re vi ew er a t the end o f the sem este r wi ll (hopef ully) guar antee you r pas sing this cou rse. D rink mode rat ely .
Amusin S C A r y
gly
THE C2005 LOCAL GOVERNMENT REVIEWER - 39 -
activities which result in pollution, acceleration of (ii) Subject to the provisions of Book II of this Code
eutrophication of rivers and lakes, or of ecological and applicable laws and upon the majority vote
imbalance; of all the members of the sangguniang bayan,
(vii) Subject to the provisions of this Code and enact ordinances levying taxes, fees and
pertinent laws, determine the powers and duties charges, prescribing the rates thereof for
of officials and employees of the municipality; general and specific purposes, and granting tax
(viii) Determine the positions and the salaries, wages, exemptions, incentives or reliefs;
allowances and other emoluments and benefits of (iii) Subject to the provisions of Book II of this Code
officials and employees paid wholly or mainly and upon the majority vote of all the members
from municipal funds and provide for of the sangguniang bayan, authorize the
expenditures necessary for the proper conduct of municipal mayor to negotiate and contract loans
programs, projects, services, and activities of the and other forms of indebtedness;
municipal government; (iv) Subject to the provisions of Book II of this Code
(ix) Authorize the payment of compensation to a and applicable laws and upon the majority vote
qualified person not in the government service of all the members of the sangguniang bayan,
who fills up a temporary vacancy or grant enact ordinances authorizing the floating of
honorarium to any qualified official or employee bonds or other instruments of indebtedness, for
designated to fill a temporary vacancy in a the purpose of raising funds to finance
concurrent capacity, at the rate authorized by law; development projects;
(x) Provide a mechanism and the appropriate funds (v) Appropriate funds for the construction and
therefor, to ensure the safety and protection of all maintenance or the rental of buildings for the
municipal government property, public use of the municipality and, upon the majority
documents, or records such as those relating to vote of all the members of the sangguniang
property inventory, land ownership, records of bayan, authorize the municipal mayor to lease
births, marriages, deaths, assessments, taxation, to private parties such public buildings held in a
accounts, business permits, and such other proprietary capacity, subject to existing laws,
records and documents of public interest in the rules and regulations;
offices and departments of the municipal (vi) Prescribe reasonable limits and restraints on
government; the use of property within the jurisdiction of the
(xi) When the finances of the municipal government municipality;
allow, provide for additional allowances and other (vii) Adopt a comprehensive land use plan for the
benefits to judges, prosecutors, public elementary municipality: Provided, That the formulation,
and high school teachers, and other national adoption, or modification of said plan shall be in
government officials stationed in or assigned to coordination with the approved provincial
the municipality; comprehensive land use plan;
(xii) Provide for legal assistance to barangay officials (viii) Reclassify land within the jurisdiction of the
who, in the performance of their official duties or municipality, subject to the pertinent provisions
on the occasion thereof, have to initiate judicial of this Code;
proceedings or defend themselves against legal (ix) Enact integrated zoning ordinances in
action; and, (xiii) Provide for group insurance or consonance with the approved comprehensive
additional insurance coverage for barangay land use plan, subject to existing laws, rules
officials, including members of barangay tanod and regulations; establish fire limits or zones,
brigades and other service units, with public or particularly in populous centers; and regulate
private insurance companies, when the finances the construction, repair or modification of
of the municipal government allow said coverage. buildings within said fire limits or zones in
(2) Generate and maximize the use of resources and accordance with the provisions of the Fire
revenues for the development plans, program objectives Code;
and priorities of the municipality as provided for under (x) Subject to national law, process and approve
Section 18 of this Code with particular attention to agro- subdivision plans for residential, commercial, or
industrial development and countryside growth and industrial purposes and other development
progress, and relative thereto, shall: purposes, and collect processing fees and other
(i) Approve the annual and supplemental budgets charges, the proceeds of which shall accrue
of the municipal government and appropriate entirely to the municipality: Provided, however,
funds for specific programs, projects, services That, where approval by a national agency or
and activities of the municipality, or for other office is required, said approval shall not be
purposes not contrary to law, in order to withheld for more than thirty (30) days from
promote the general welfare of the municipality receipt of the application. Failure to act on the
and its inhabitants;

The Apocryphal Maggots:


Rainier, Chrisgel, Corina, Geoffry, Grace and Sylvie Blanche
The Flibbertigibbet Worms:
Golda, Gladys and Melyjane
CA VE AT: By simply r ea ding this re vi ew er a t the end o f the sem este r wi ll (hopef ully) guar antee you r pas sing this cou rse. D rink mode rat ely .
Amusin S C A r y
gly
THE C2005 LOCAL GOVERNMENT REVIEWER - 40 -
application within the period stated above shall thereof within the territorial jurisdiction of the
be deemed as approval thereof; municipality;
(xi) Subject to the provisions of Book II of this Code, (vii) Upon approval by a majority vote of all the
grant the exclusive privilege of constructing fish members of the sangguniang bayan, grant a
corrals or fish pens, or the taking or catching of franchise to any person, partnership,
bangus fry, prawn fry or kawag-kawag or fry of corporation, or cooperative to establish,
any species or fish within the municipal waters; construct, operate and maintain ferries,
(xii) With the concurrence of at least two-thirds (2/3) wharves, markets or slaughterhouses, or such
of all the members of the sangguniang bayan, other similar activities within the municipality as
grant tax exemptions, incentives or reliefs to may be allowed by applicable laws: Provided,
entities engaged in community growth-inducing That, cooperatives shall be given preference in
industries, subject to the provisions of Chapter the grant of such a franchise.
5, Title I, Book II of this Code; (4) Regulate activities relative to the use of land, buildings
(xiii) Grant loans or provide grants to other local and structures within the municipality in order to promote
government units or to national, provincial and the general welfare and for said purpose shall:
municipal charitable, benevolent or educational (i) Declare, prevent or abate any nuisance;
institutions: Provided, That said institutions are (ii) Require that buildings and the premises thereof
operated and maintained within the municipality; and any land within the municipality be kept and
(xiv) Regulate the numbering of residential, maintained in a sanitary condition; impose
commercial and other buildings; and, penalties for any violation thereof, or upon
(xv) Regulate the inspection, weighing and failure to comply with said requirement, have
measuring of articles of commerce. the work done and require the owner,
(3) Subject to the provisions of Book II of this Code, grant administrator or tenant concerned to pay the
franchises, enact ordinances authorizing the issuance of expenses of the same; or require the filling up of
permits or licenses, or enact ordinances levying taxes, any land or premises to a grade necessary for
fees and charges upon such conditions and for such proper sanitation;
purposes intended to promote the general welfare of the (iii) Regulate the disposal of clinical and other
inhabitants of the municipality, and pursuant to this wastes from hospitals, clinics and other similar
legislative authority shall: establishments;
(i) Fix and impose reasonable fees and charges (iv) Regulate the establishment, operation and
for all services rendered by the municipal maintenance of cafes, restaurants, beerhouses,
government to private persons or entities; hotels, motels, inns, pension houses, lodging
(ii) Regulate any business, occupation, or practice houses, and other similar establishments,
of profession or calling which does not require including tourist guides and transports;
government examination within the municipality (v) Regulate the sale, giving away or dispensing of
and the conditions under which the license for any intoxicating malt, vino, mixed or fermented
said business or practice of profession may be liquors at any retail outlet;
issued or revoked; (vi) Regulate the establishment and provide for the
(iii) Prescribe the terms and conditions under which inspection of steam boilers or any heating
public utilities owned by the municipality shall device in buildings and the storage of
be operated by the municipal government or inflammable and highly combustible materials
leased to private persons or entities, preferably within the municipality;
cooperatives; (vii) Regulate the establishment, operation, and
(iv) Regulate the display of and fix the license fees mainte nance of entertainment or amusement
for signs, signboards, or billboards at the place facilities, including theatrical performances,
or places where the profession or business circuses, billiard pools, public dancing schools,
advertised thereby is, in whole or in part, public dance halls, sauna baths, massage
conducted; parlors, and other places of entertainment or
(v) Any law to the contrary notwithstanding, amusement; regulate such other events or
authorize and license the establishment, activities for amusement or entertainment,
operation, and maintenance of cockpits, and particularly those which tend to disturb the
regulate cockfighting and commercial breeding community or annoy the inhabitants, or require
of gamecocks: Provided, That existing rights the suspension or suppression of the same; or,
should not be prejudiced; prohibit certain forms of amusement or
(vi) Subject to the guidelines prescribed by the entertainment in order to protect the social and
Department of Transportation and moral welfare of the community;
Communications, regulate the operation of (viii) Provide for the impounding of stray animals;
tricycles and grant franchises for the operation regulate the keeping of animals in homes or as

The Apocryphal Maggots:


Rainier, Chrisgel, Corina, Geoffry, Grace and Sylvie Blanche
The Flibbertigibbet Worms:
Golda, Gladys and Melyjane
CA VE AT: By simply r ea ding this re vi ew er a t the end o f the sem este r wi ll (hopef ully) guar antee you r pas sing this cou rse. D rink mode rat ely .
Amusin S C A r y
gly
THE C2005 LOCAL GOVERNMENT REVIEWER - 41 -
part of a business, and the slaughter, sale or over all territory within the drainage area of said
disposition of the same; and adopt measures to water supply and within one hundred (100)
prevent and penalize cruelty to animals; and meters of the reservoir, conduit, canal,
(ix) Regulate the establishment, operation, and aqueduct, pumping station, or watershed used
maintenance of funeral parlors and the burial or in connection with the water service; and
cremation of the dead, subject to existing laws, regulate the consumption, use or wastage of
rules and regulations. water;
(5) Approve ordinances which shall ensure the efficient and (viii) Regulate the drilling and excavation of the
effective delivery of the basic services and facilities as ground for the laying of water, gas, sewer, and
provided for under Section 17 of this Code, and in other pipes and the construction, repair and
addition to said services and facilities, shall: maintenance of public drains, sewers,
(i) Provide for the establishment, maintenance, cesspools, tunnels and similar structures;
protection, and conservation of communal regulate the placing of poles and the use of
forests and water sheds, tree parks, greenbelts, crosswalks, curbs, and gutters; adopt measures
mangroves, and other similar forest to ensure public safety against open canals,
development projects; manholes, live wires and other similar hazards
(ii) Establish markets, slaughterhouses or animal to life and property; and, regulate the
corrals and authorize the operation thereof, and construction and use of private water closets,
regulate the construction and operation of privies and other similar structures in buildings
private markets, talipapas or other similar and homes;
buildings and structures; (ix) Regulate the placing, stringing, attaching,
(iii) Authorize the establishment, maintenance and installing, repair and construction of all gas
operation of ferries, wharves, and other mains, electric, telegraph and telephone wires,
structures, and marine and seashore or offshore conduits, meters and other apparatus; and,
activities intended to accelerate productivity; provide for the correction, condemnation or
(iv) Regulate the preparation and sale of meat, removal of the same when found to be
poultry, fish, vegetables, fruits, fresh dairy dangerous, defective or otherwise hazardous to
products, and other foodstuffs for public the welfare of the inhabitants;
consumption; (x) Subject to the availability of funds and to
(v) Regulate the use of streets, avenues, alleys, existing laws, rules and regulations, establish
sidewalks, bridges, parks and other public and provide for the operation of vocational and
places and approve the construction, technical schools and similar post-secondary
improvement, repair and maintenance of the institutions and, with the approval of the
same; establish bus and vehicle stops and Department of Education, Culture and Sports,
terminals or regulate the use of the same by fix and collect reasonable fees and other school
privately-owned vehicles which serve the public; charges on said institutions, subject to existing
regulate garages and the operation of laws on tuition fees;
conveyances for hire; designate stands to be (xi) Establish a scholarship fund for poor but
occupied by public vehicles when not in use; deserving students residing within the
regulate the putting up of signs, signposts, municipality in schools located within its
awnings and awning posts on the streets; and jurisdiction;
provide for the lighting, cleaning and sprinkling (xii) Approve measures and adopt quarantine
of streets and public places; regulations to prevent the introduction and
(vi) Regulate traffic on all streets and bridges, spread of diseases;
prohibit the putting up of encroachments or (xiii) Provide for an efficient and effective system of
obstacles thereon, and, when necessary in the solid waste and garbage collection and disposal
interest of public welfare, authorize the removal and prohibit littering and the placing or throwing
of encroachments and illegal constructions in of garbage, refuse and other filth and wastes;
public places; (xiv) Provide for the care of paupers, the aged, the
(vii) Subject to existing laws, provide for the sick, persons of unsound mind, disabled
establishment, operation, maintenance, and persons, abandoned minors, juvenile
repair of an efficient waterworks system to delinquents, drug dependents, abused children
supply water for the inhabitants; regulate the and other needy and disadvantaged persons,
construction, maintenance, repair and use of particularly children and youth below eighteen
hydrants, pumps, cisterns and reservoirs; (18) years of age and, subject to availability of
protect the purity and quantity of the water funds, establish and provide for the operation of
supply of the municipality and, for this purpose, centers and facilities for said needy and
extend the coverage of appropriate ordinances disadvantaged persons;

The Apocryphal Maggots:


Rainier, Chrisgel, Corina, Geoffry, Grace and Sylvie Blanche
The Flibbertigibbet Worms:
Golda, Gladys and Melyjane
CA VE AT: By simply r ea ding this re vi ew er a t the end o f the sem este r wi ll (hopef ully) guar antee you r pas sing this cou rse. D rink mode rat ely .
Amusin S C A r y
gly
THE C2005 LOCAL GOVERNMENT REVIEWER - 42 -
(xv) Establish and provide for the maintenance and (iv) Adopt measures to protect the inhabitants of the
improvement of jails and detention centers, city from the harmful effects of man-made or
institute sound jail management programs, and natural disasters and calamities, and to provide
appropriate funds for the subsistence of relief services and assistance for victims during
detainees and convicted prisoners in the and in the aftermath of said disasters or
municipality; calamities and their return to productive livelihood
(xvi) Establish a municipal council whose purpose is following said events;
the promotion of culture and the arts, coordinate (v) Enact ordinances intended to prevent, suppress
with government agencies and non- and impose appropriate penalties for habitual
governmental organizations and, subject to the drunkenness in public places, vagrancy,
availability of funds, appropriate funds for the mendicancy, prostitution, establishment and
support and development of the same; and maintenance of houses of ill repute, gambling and
(xvii) Establish a municipal council for the elderly other prohibited games of chance, fraudulent
which shall formulate policies and adopt devices and ways to obtain money or property,
measures mutually beneficial to the elderly and drug addiction, maintenance of drug dens, drug
to the community; provide incentives for non- pushing, juvenile delinquency, the printing,
governmental agencies and entities and, distribution or exhibition of obscene or
subject to the availability of funds, appropriate pornographic materials or publications, and such
funds to support programs and projects for the other activities inimical to the welfare and morals
benefit of the elderly; and of the inhabitants of the city;
(6) Exercise such other powers and perform such other (vi) Protect the environment and impose appropriate
duties and functions as may be prescribed by law or penalties for acts which endanger the
ordinance. environment, such as dynamite fishing and other
(b) The members of the sangguniang bayan shall receive a forms of destructive fishing, illegal logging and
minimum monthly compensation corresponding to Salary Grade smuggling of logs, smuggling of natural resources
twenty-four (24) as prescribed under R.A. 6758 and the products and of endangered species of flora and
implementing guidelines issued pursuant thereto: Provided, fauna, slash and burn farming, and such other
That, in municipalities in the Metropolitan Manila Area and other activities which result in pollution, acceleration of
metropolitan political subdivisions, members of the sangguniang eutrophication of rivers and lakes, or of ecological
bayan shall receive a minimum monthly compensation imbalance;
corresponding to Salary grade twenty-five (25). (vii) Subject to the provisions of this Code and
pertinent laws, determine the powers and duties
Sangguniang Panlungsod of officials and employees of the city;
Sec458, LGC: Powers, Duties, Functions and Compensation. – (viii) Determine the positions and the salaries, wages,
(a) The sangguniang panlungsod, as the legislative body of the allowances and other emoluments and benefits of
city, shall enact ordinances, approve resolutions and appropriate officials and employees paid wholly or mainly
funds for the general welfare of the city and its inhabitants from city funds and provide for expenditures
pursuant to Section 16 of this Code and in the proper exercise necessary for the proper conduct of programs,
of the corporate powers of the city as provided for under Section projects, services, and activities of the city
22 of this Code, and shall: government;
(1) Approve ordinances and pass resolutions necessary for (ix) Authorize the payment of compensation to a
an efficient and effective city government, and in this qualified person not in the government service
connection, shall: who fills up a temporary vacancy or grant
(i) Review all ordinances approved by the honorarium to any qualified official or employee
sangguniang barangay and executive orders designated to fill a temporary vacancy in a
issued by the punong barangay to determine concurrent capacity, at the rate authorized by law;
whether these are within the scope of the (x) Provide a mechanism and the appropriate funds
prescribed powers of the sanggunian and of the therefor, to ensure the safety and protection of all
punong barangay; city government property, public documents, or
(ii) Maintain peace and order by enacting measures records such as those relating to property
to prevent and suppress lawlessness, disorder, inventory, land ownership, records of births,
riot, violence, rebellion or sedition and impose marriages, deaths, assessments, taxation,
penalties for the violation of said ordinances; accounts, business permits, and such other
(iii) Approve ordinances imposing a fine not records and documents of public interest in the
offices and departments of the city government;
exceeding Five thousand pesos (P5,000.00) or
(xi) When the finances of the city government allow,
an imprisonment for a period not exceeding one
provide for additional allowances and other
(1) year, or both in the discretion of the court, for
benefits to judges, prosecutors, public elementary
the violation of a city ordinance;

The Apocryphal Maggots:


Rainier, Chrisgel, Corina, Geoffry, Grace and Sylvie Blanche
The Flibbertigibbet Worms:
Golda, Gladys and Melyjane
CA VE AT: By simply r ea ding this re vi ew er a t the end o f the sem este r wi ll (hopef ully) guar antee you r pas sing this cou rse. D rink mode rat ely .
Amusin S C A r y
gly
THE C2005 LOCAL GOVERNMENT REVIEWER - 43 -
and high school teachers, and other national plan shall be in coordination with the approved
government officials stationed in or assigned to provincial comprehensive land use plan;
the city; (viii) Reclassify land within the jurisdiction of the city,
(xii) Provide legal assistance to barangay officials subject to the pertinent provisions of this Code;
who, in the performance of their official duties or (ix) Enact integrated zoning ordinances in
on the occasion thereof, have to initiate judicial consonance with the approved comprehensive
proceedings or defend themselves against legal land use plan, subject to existing laws, rules and
action; and regulations; establish fire limits or zones,
(xiii) Provide for group insurance or additional particularly in populous centers; and regulate the
insurance coverage for all barangay officials, construction, repair or modification of buildings
including members of barangay tanod brigades within said fire limits or zones in accordance with
and other service units, with public or private the provisions of the Fire Code;
insurance companies, when the finances of the (x) Subject to national law, process and approve
city government allow said coverage; subdivision plans for residential, commercial, or
(2) Generate and maximize the use of resources and industrial purposes and other development
revenues for the development plans, program objectives purposes, and to collect processing fees and
and priorities of the city as provided for under Section 18 other charges, the proceeds of which shall accrue
of this Code, with particular attention to agro-industrial entirely to the city: Provided, however, That
development and city-wide growth and progress, and where approval of a national agency or office is
relative thereto, shall: required, said approval shall not be withheld for
(i) Approve the annual and supplemental budgets of more than thirty (30) days from receipt of the
the city government and appropriate funds for application. Failure to act on the application within
specific programs, projects, services and the period stated above shall be deemed as
activities of the city, or for other purposes not approval thereof;
contrary to law, in order to promote the general (xi) Subject to the provisions of Book II of this Code,
welfare of the city and its inhabitants; grant the exclusive privilege of constructing fish
(ii) Subject to the provisions of Book II of this Code corrals or fish pens, or the taking or catching of
and applicable laws and upon the majority vote of bangus fry, prawn fry or kawag-kawag, or fry of
all the members of the sangguniang panlungsod, any species or fish within the city waters;
enact ordinances levying taxes, fees and (xii) With the concurrence of at least two-thirds (2/3)
charges, prescribing the rates thereof for general of all the members of the sangguniang
and specific purposes, and granting tax panlungsod, grant tax exemptions, incentives or
exemptions, incentives or reliefs; reliefs to entities engaged in community growth-
(iii) Subject to the provisions of Book II of this Code inducing industries, subject to the provisions of
and upon the majority vote of all the members of Chapter 5, Title I, Book II of this Code;
the sangguniang panlungsod, authorize the city (xiii) Grant loans or provide grants to other local
mayor to negotiate and contract loans and other government units or to national, provincial, and
forms of indebtedness; city charitable, benevolent or educational
(iv) Subject to the provisions of Book II of this Code institutions: Provided, That, said institutions are
and applicable laws and upon the majority vote of operated and maintained within the city;
all the members of the sangguniang panlungsod, (xiv) Regulate the numbering of residential,
enact ordinances authorizing the floating of bonds commercial and other buildings; and,
or other instruments of indebtedness, for the (xv) Regulate the inspection, weighing and measuring
purpose of raising funds to finance development of articles of commerce.
projects; (3) Subject to the provisions of Book II of this Code, enact
(v) Appropriate funds for the construction and ordinances granting franchises and authorizing the
maintenance or the rental of buildings for the use issuance of permits or licenses, upon such conditions and
of the city; and, upon the majority vote of all the for such purposes intended to promote the general
members of the sangguniang panlungsod, welfare of the inhabitants of the city and pursuant to this
authorize the city mayor to lease to private parties legislative authority shall:
such public buildings held in a proprietary (i) Fix and impose reasonable fees and charges for
capacity, subject to existing laws, rules and all services rendered by the city government to
regulations; private persons or entities;
(vi) Prescribe reasonable limits and restraints on the (ii) Regulate or fix license fees for any business or
use of property within the jurisdiction of the city; practice of profession within the city and the
(vii) Adopt a comprehensive land use plan for the city: conditions under which the license for said
Provided, That in the case of component cities, business or practice of profession may be
the formulation, adoption or modification of said

The Apocryphal Maggots:


Rainier, Chrisgel, Corina, Geoffry, Grace and Sylvie Blanche
The Flibbertigibbet Worms:
Golda, Gladys and Melyjane
CA VE AT: By simply r ea ding this re vi ew er a t the end o f the sem este r wi ll (hopef ully) guar antee you r pas sing this cou rse. D rink mode rat ely .
Amusin S C A r y
gly
THE C2005 LOCAL GOVERNMENT REVIEWER - 44 -
revoked and enact ordinances levying taxes (vi) Regulate the establishment and provide for the
thereon; inspection of steam boilers or any heating device
(iii) Provide for and set the terms and conditions in buildings and the storage of inflammable and
under which public utilities owned by the city shall highly combustible materials within the city;
be operated by the city government, and (vii) Regulate the establishment, operation, and
prescribe the conditions under which the same maintenance of any entertainment or amusement
may be leased to private persons or entities, facilities, including theatrical performances,
preferably cooperatives; circuses, billiard pools, public dancing schools,
(iv) Regulate the display of and fix the license fees for public dance halls, sauna baths, massage
signs, signboards, or billboards at the place or parlors, and other places for entertainment or
places where the profession or business amusement; regulate such other events or
advertised thereby is, in whole or in part, activities for amusement or entertainment,
conducted; particularly those which tend to disturb the
(v) Any law to the contrary notwithstanding, authorize community or annoy the inhabitants, or require
and license the establishment, operation, and the suspension or suppression of the same; or,
maintenance of cockpits, and regulate prohibit certain forms of amusement or
cockfighting and commercial breeding of entertainment in order to protect the social and
gamecocks: Provided, That existing rights should moral welfare of the community;
not be prejudiced; (viii) Provide for the impounding of stray animals;
(vi) Subject to the guidelines prescribed by the regulate the keeping of animals in homes or as
DOTC, regulate the operation of tricycles and part of a business, and the slaughter, sale or
grant franchises for the operation thereof within disposition of the same; and adopt measures to
the territorial jurisdiction of the city; prevent and penalize cruelty to animals; and,
(vii) Upon approval by a majority vote of all the (ix) Regulate the establishment, operation and
members of the sangguniang panlungsod: grant a maintenance of funeral parlors and the burial or
franchise to any person, partnership, corporation, cremation of the dead, subject to existing laws,
or cooperative to do business within the city; rules and regulations.
establish, construct, operate and maintain ferries, (5) Approve ordinances which shall ensure the efficient and
wharves, markets or slaughterhouses; or effective delivery of the basic services and facilities as
undertake such other activities within the city as provided for under Section 17 of this Code, and in
may be allowed by existing laws: Provided, That, addition to said services and facilities, shall:
cooperatives shall be given preference in the (i) Provide for the establishment, maintenance,
grant of such a franchise. protection, and conservation of communal forests
(4) Regulate activities relative to the use of land, buildings and water sheds, tree parks, greenbelts,
and structures within the city in order to promote the mangroves, and other similar forest development
general welfare and for said purpose shall: projects;
(i) Declare, prevent or abate any nuisance; (ii) Establish markets, slaughterhouses or animal
(ii) Require that buildings and the premises thereof corrals and authorize the operation thereof by the
and any land within the city be kept and city government; and regulate the construction
maintained in a sanitary condition; impose and operation of private markets, talipapas or
penalties for any violation thereof; or, upon failure other similar buildings and structures;
to comply with said requirement, have the work (iii) Authorize the establishment, maintenance and
done at the expense of the owner, administrator operation by the city government of ferries,
or tenant concerned; or require the filling up of wharves, and other structures intended to
any land or premises to a grade necessary for accelerate productivity related to marine and
proper sanitation; seashore or offshore activities;
(iii) Regulate the disposal of clinical and other wastes (iv) Regulate the preparation and sale of meat,
from hospitals, clinics and other similar poultry, fish, vegetables, fruits, fresh dairy
establishments; products, and other foodstuffs for public
(iv) Regulate the establishment, operation and cafes, consumption;
restaurants, beerhouses, hotels, motels, inns, (v) Regulate the use of streets, avenues, alleys,
pension houses, lodging houses, and other sidewalks, bridges, parks and other public places
similar establishments, including tourist guides and approve the construction, improvement,
and transports; repair and maintenance of the same; establish
(v) Regulate the sale, giving away or dispensing of bus and vehicle stops and terminals or regulate
any intoxicating malt, vino, mixed or fermented the use of the same by privately-owned vehicles
liquors at any retail outlet; which serve the public; regulate garages and the
operation of conveyances for hire; designate

The Apocryphal Maggots:


Rainier, Chrisgel, Corina, Geoffry, Grace and Sylvie Blanche
The Flibbertigibbet Worms:
Golda, Gladys and Melyjane
CA VE AT: By simply r ea ding this re vi ew er a t the end o f the sem este r wi ll (hopef ully) guar antee you r pas sing this cou rse. D rink mode rat ely .
Amusin S C A r y
gly
THE C2005 LOCAL GOVERNMENT REVIEWER - 45 -
stands to be occupied by public vehicles when (xi) Establish a scholarship fund for the poor but
not in use; regulate the putting up of signs, deserving students in schools located within its
signposts, awnings and awning posts on the jurisdiction or for students residing within the city;
streets; and provide for the lighting, cleaning and (xii) Approve measures and adopt quarantine
sprinkling of streets and public places; regulations to prevent the introduction and spread
(vi) Regulate traffic on all streets and bridges; prohibit of diseases;
encroachments or obstacles thereon and, when (xiii) Provide for an efficient and effective system of
necessary in the interest of public welfare, solid waste and garbage collection and disposal;
authorize the removal of encroachments and prohibit littering and the placing or throwing of
illegal constructions in public places; garbage, refuse and other filth and wastes;
(vii) Subject to existing laws, establish and provide for (xiv) Provide for the care of disabled persons,
the maintenance, repair and operation of an paupers, the aged, the sick, persons of unsound
efficient waterworks system to supply water for mind, abandoned minors, juvenile delinquents,
the inhabitants and to purify the source of the drug dependents, abused children and other
water supply; regulate the construction, needy and disadvantaged persons, particularly
maintenance, repair and use of hydrants, pumps, children and youth below eighteen (18) years of
cisterns and reservoirs; protect the purity and age; and, subject to availability of funds, establish
quantity of the water supply of the city and, for and provide for the operation of centers and
this purpose, extend the coverage of appropriate facilities for said needy and disadvantaged
ordinances over all territory within the drainage persons;
area of said water supply and within one hundred (xv) Establish and provide for the maintenance and
(100) meters of the reservoir, conduit, canal, improvement of jails and detention centers,
aqueduct, pumping station, or watershed used in institute a sound jail management, and
connection with the water service; and regulate appropriate funds for the subsistence of
the consumption, use or wastage of water and fix detainees and convicted prisoners in the city;
and collect charges therefor; (xvi) Establish a City council whose purpose is the
(viii) Regulate the drilling and excavation of the ground promotion of culture and the arts, coordinate with
for the laying of water, gas, sewer, and other government agencies and non-governmental
pipes and the construction, repair and organizations and, subject to the availability of
maintenance of public drains, sewers, cesspools, funds, appropriate funds for the support and
tunnels and similar structures; regulate the development of the same; and
placing of poles and the use of crosswalks, curbs, (xvii) Establish a City council for the elderly which shall
and gutters; adopt measures to ensure public formulate policies and adopt measures mutually
safety against open canals, manholes, live wires beneficial to the elderly and to the community;
and other similar hazards to life and property; and provide incentives for non-governmental agencies
regulate the construction and use of private water and entities and, subject to the availability of
closets, privies and other similar structures in funds, appropriate funds to support programs and
buildings and homes; projects for the benefit of the elderly; and
(ix) Regulate the placing, stringing, attaching, (6) Exercise such other powers and perform such other
installing, repair and construction of all gas duties and functions as may be prescribed by law or
mains, electric, telegraph and telephone wires, ordinance.
conduits, meters and other apparatus; and (b) The members of the sangguniang panlungsod of component
provide for the correction, condemnation or cities shall receive a minimum monthly compensation
removal of the same when found to be corresponding to Salary Grade twenty-five (25) and members of
dangerous, defective, or otherwise hazardous to the sangguniang panlungsod of highly- urbanized cities shall
the welfare of the inhabitants; receive a minimum monthly compensation corresponding to
(x) Subject to the availability of funds and to existing Salary Grade twenty-seven (27), as prescribed under R.A. 6758
laws, rules and regulations, establish and provide and the implementing guidelines issued pursuant thereto.
for the operation of vocational and technical
schools and similar post-secondary institutions Sangguniang Panlalawigan
and, with the approval of the Department of Sec468, LGC: Powers, Duties, Functions and Compensation. –
Education, Culture and Sports and subject to (a) The sangguniang panlalawigan, as the legislative body of the
existing law on tuition fees, fix and collect province, shall enact ordinances, approve resolutions and
reasonable tuition fees and other school charges appropriate funds for the general welfare of the province and its
in educational institutions supported by the city inhabitants pursuant to Section 16 of this Code and in the
government; proper exercise of the corporate powers of the province as
provided for under Section 22 of this Code, and shall:

The Apocryphal Maggots:


Rainier, Chrisgel, Corina, Geoffry, Grace and Sylvie Blanche
The Flibbertigibbet Worms:
Golda, Gladys and Melyjane
CA VE AT: By simply r ea ding this re vi ew er a t the end o f the sem este r wi ll (hopef ully) guar antee you r pas sing this cou rse. D rink mode rat ely .
Amusin S C A r y
gly
THE C2005 LOCAL GOVERNMENT REVIEWER - 46 -
(1) Approve ordinances and pass resolutions necessary for projects, services, and activities of the provincial
an efficient and effective provincial government and, in government;
this connection, shall: (ix) Authorize the payment of compensation to a
(i) Review all ordinances approved by the qualified person not in the government service
sanggunians of component cities and who fills up a temporary vacancy, or grant
municipalities and executive orders issued by the honorarium to any qualified official or employee
mayors of said component units to determine designated to fill a temporary vacancy in a
whether these are within the scope of the concurrent capacity, at the rate authorized by law;
prescribed powers of the sanggunian and of the (x) Provide a mechanism and the appropriate funds
mayor; therefor, to ensure the safety and protection of all
(ii) Maintain peace and order by enacting measures provincial government property, public documents,
to prevent and suppress lawlessness, disorder, or records such as those relating to property
riot, violence, rebellion or sedition and impose inventory, land ownership, records of births,
penalties for the violation of said ordinances; marriages, deaths, assessments, taxation,
(iii) Approve ordinances imposing a fine not accounts, business permits, and such other
records and documents of public interest in the
exceeding Five thousand pesos (P=5,000.00) or
offices and departments of the provincial
imprisonment not exceeding one (1) year, or both
government; and
in the discretion of the court, for the violation of a
(xi) When the finances of the provincial government
provincial ordinance;
allow, provide for additional allowances and other
(iv) Adopt measures to protect the inhabitants of the
benefits to judges, prosecutors, public elementary
province from the harmful effects of man-made or
and high school teachers, and other national
natural disasters and calamities, and to provide
government officials stationed or assigned to the
relief services and assistance for victims during
province.
and in the aftermath of said disasters and
(2) Generate and maximize the use of resources and
calamities and their return to productive livelihood
revenues for the development plans, program objectives
following said events;
and priorities of the province as provided for under
(v) Enact ordinances intended to prevent, suppress
Section 18 of this Code, with particular attention to agro-
and impose appropriate penalties for habitual
industrial development and country-wide growth and
drunkenness in public places, vagrancy,
progress and relative thereto, shall:
mendicancy, prostitution, establishment and
(i) Enact the annual and supplemental
maintenance of houses of ill repute, gambling and
appropriations of the provincial government and
other prohibited games of chance, fraudulent
appropriate funds for specific programs, projects,
devices and ways to obtain money or property,
services and activities of the province, or for other
drug addiction, maintenance of drug dens, drug
purposes not contrary to law, in order to promote
pushing, juvenile delinquency, the printing,
the general welfare of the province and its
distribution or exhibition of obscene or
inhabitants;
pornographic materials or publications, and such
(ii) Subject to the provisions of Book II of this Code
other activities inimical to the welfare and morals
and applicable laws and upon the majority vote of
of the inhabitants of the province;
all the members of the sangguniang
(vi) Protect the environment and impose appropriate
panlalawigan, enact ordinances levying taxes,
penalties for acts which endanger the
fees and charges, prescribing the rates thereof for
environment, such as dynamite fishing and other
general and specific purposes, and granting tax
forms of destructive fishing, illegal logging and
exemptions, incentives or reliefs;
smuggling of logs, smuggling of natural resources
(iii) Subject to the provisions of Book II of this Code
products and of endangered species of flora and
and applicable laws and upon the majority vote of
fauna, slash and burn farming, and such other
all the members of the sangguniang
activities which result in pollution, acceleration of
panlalawigan, authorize the provincial governor to
eutrophication of rivers and lakes, or of ecological
negotiate and contract loans and other forms of
imbalance;
indebtedness;
(vii) Subject to the provisions of this Code and
(iv) Subject to the provisions of Book II of this Code
pertinent laws, determine the powers and duties
and applicable laws and upon the majority vote of
of officials and employees of the province;
all the members of the sangguniang
(viii) Determine the positions and the salaries, wages,
panlalawigan, enact ordinances authorizing the
allowances and other emoluments and benefits of
floating of bonds or other instruments of
officials and employees paid wholly or mainly from
indebtedness, for the purpose of raising funds to
provincial funds and provide for expenditures
finance development projects;
necessary for the proper conduct of programs,

The Apocryphal Maggots:


Rainier, Chrisgel, Corina, Geoffry, Grace and Sylvie Blanche
The Flibbertigibbet Worms:
Golda, Gladys and Melyjane
CA VE AT: By simply r ea ding this re vi ew er a t the end o f the sem este r wi ll (hopef ully) guar antee you r pas sing this cou rse. D rink mode rat ely .
Amusin S C A r y
gly
THE C2005 LOCAL GOVERNMENT REVIEWER - 47 -
(v) Appropriate funds for the construction and jurisdiction or for students residing within the
maintenance or the rental of buildings for the use province;
of the province; and upon the majority vote of all (v) Approve measures and adopt quarantine
the members of the sangguniang panlalawigan, regulations to prevent the introduction and spread
authorize the provincial governor to lease to of diseases within its territorial jurisdiction;
private parties such public buildings held in a (vi) Provide for the care of paupers, the aged, the
proprietary capacity, subject to existing laws, rules sick, persons of unsound mind, abandoned
and regulations; minors, abused children, disabled persons,
(vi) Prescribe reasonable limits and restraints on the juvenile delinquents, drug dependents, and other
use of property within the jurisdiction of the needy and disadvantaged persons, particularly
province; children and youth below eighteen (18) years of
(vii) Review the comprehensive land use plans and age; subject to availability of funds, establish and
zoning ordinances of component cities and support the operation of centers and facilities for
municipalities and adopt a comprehensive said needy and disadvantaged persons; and
provincial land use plan, subject to existing laws; facilitate efforts to promote the welfare of families
and below the poverty threshold, the disadvantaged,
(viii) Adopt measures to enhance the full and the exploited;
implementation of the national agrarian reform (vii) Establish and provide for the maintenance and
program in coordination with the Department of improvement of jails and detention centers,
Agrarian Reform; institute a sound jail management program, and
(3) Subject to the provisions of Book II of this Code, grant appropriate funds for the subsistence of detainees
franchises, approve the issuance of permits or licenses, and convicted prisoners in the province;
or enact ordinances levying taxes, fees and charges upon (viii) Establish a provincial council whose purpose is
such conditions and for such purposes intended to the promotion of culture and the arts, coordinate
promote the general welfare of the inhabitants of the with government agencies and non-governmental
province, and pursuant to this legislative authority, shall: organizations and, subject to the availability of
(i) Fix and impose reasonable fees and charges for funds, appropriate funds for the support and
all services rendered by the provincial government development of the same;
to private persons or entities; and (ix) Establish a provincial council for the elderly which
(ii) Regulate and fix the license fees for such shall formulate policies and adopt measures
activities as provided for under this Code. mutually beneficial to the elderly and to the
(4) Approve ordinances which shall ensure the efficient and province; and subject to the availability of funds,
effective delivery of the basic services and facilities as appropriate funds to support programs and
provided for under Section 17 of this Code, and, in projects for the elderly; and provide incentives for
addition to said services and facilities, shall: non- governmental agencies and entities to
(i) Adopt measures and safeguards against pollution support the programs and projects of the elderly;
and for the preservation of the natural ecosystem and
in the province, in consonance with approved (5) Exercise such other powers and perform such other
standards on human settlements and duties and functions as may be prescribed by law or
environmental sanitation; ordinance.
(ii) Subject to applicable laws, facilitate or provide for (b) The members of the sangguniang panlalawigan shall receive
the establishment and maintenance of a a minimum monthly compensation corresponding to Salary
waterworks system or district waterworks for Grade twenty-seven (27) as prescribed under R.A. 6758 and the
supplying water to inhabitants of component cities implementing guidelines issued pursuant thereto.
and municipalities;
(iii) Subject to the availability of funds and to existing
laws, rules and regulations, provide for the RA8369: An Act Establishing Family Courts, Granting Them
establishment and operation of vocational and Exclusive Original Jurisdiction Over Child And Family
technical schools and similar post-secondary Cases, Amending Batas Pambansa Bilang 129, As
institutions; and, with the approval of the Amended, Otherwise Known As The Judiciary
Department of Education, Culture and Sports and Reorganization Act Of 1980, Appropriating Funds Therefor
subject to existing laws on tuition fees, fix And For Other Purposes
reasonable tuition fees and other school charges
in educational institutions supported by the Sec8: Supervision of Youth Detention Homes. — The judge of
provincial government; the Family Court shall have direct control and supervision of the
(iv) Establish a scholarship fund for the poor but youth detention home which the local government unit shall
deserving students in schools located within its establish to separate the youth offenders from the adult
criminals: Provided, however, That alternatives to detention and

The Apocryphal Maggots:


Rainier, Chrisgel, Corina, Geoffry, Grace and Sylvie Blanche
The Flibbertigibbet Worms:
Golda, Gladys and Melyjane
CA VE AT: By simply r ea ding this re vi ew er a t the end o f the sem este r wi ll (hopef ully) guar antee you r pas sing this cou rse. D rink mode rat ely .
Amusin S C A r y
gly
THE C2005 LOCAL GOVERNMENT REVIEWER - 48 -
institutional care shall be made available to the accused The provincial governments shall integrate the operations for the
including counseling, recognizance, bail, community continuum, agriculture extension services and shall undertake an annual
or diversions from the justice system: Provided, further, That the evaluation of all municipal extension programs.
human rights of the accused are fully respected in a manner The extension program of state colleges and universities shall
appropriate to their well-being. primarily focus on the improvement of the capability of the LGU
extension service by providing:
Sec11: Alternative Social Services. — In accordance with a) Degree and non-degree training programs;
Section 17 of this Act, in areas where no Family Court has been b) Technical assistance;
established or no Regional Trial Court was designated by the c) Extension cum research activities;
Supreme Court due to the limited number of cases, the DSWD d) Monitoring and evaluation of LGU extension projects; and
shall designate and assign qualified, trained, and DSWD e) Information support services through the tri-media and
accredited social workers of the local government units to electronics.
handle juvenile and family cases filed in the designated
Regional Trial Court of the place. Sec99: Participation of Government Agencies. — The
replication of the program shall be the responsibility of the local
government units concerned in collaboration with the
RA8425: An Act Institutionalizing The Social Reform And appropriate government agencies, and the private sector. The
Poverty Alleviation Program, Creating For The Purpose The local government units shall bear the costs of promoting and
National Anti-Poverty Commission, Defining Its Powers And monitoring the basic needs program for which their IRA shall be
Functions, And For Other Purposes increased accordingly as recommended by the Secretary of the
Department: Provided, That the appropriate national
Sec12: The Role of Local Government Units (LGUs). — The government agencies shall continue to provide the necessary
local government units, through the local development councils technical as well as financial assistance to the LGUs in the
of the province, city, municipality, or barangay shall be replication of the program.
responsible for the formulation, implementation, monitoring and The Cooperatives Development Authority shall encourage the
evaluation of the National Anti-Poverty Action Agenda in their establishment and growth of associations and cooperatives as
respective jurisdictions. The LGUs shall: vehicles for the stable expansion of basic needs enterprises.
(a) Identify the poor in their respective areas based on indicators The Department of Education, Culture and Sports, Department
such as the minimum basic needs approach and the human of Health, and the Technical Education and Skills Development
development index, their location, occupation, nature of Authority shall coordinate with the Department and Congress in
employment, and their primary resource base and formulate a the review, rationalization and reallocation of their regular
provincial/city/municipality anti-poverty action agenda; budgets as well as their budgets under the GATT-related
(b) Identify and source funding for specific social reform and measures fund to finance education, training, health and other
poverty alleviation projects; welfare services for farmers and fisherfolk.
(c) Coordinate, monitor and evaluate the efforts of local
government units with the private sector on planning and Sec101: Role of Government Agencies. — The appropriate
implementation of the local action program for social reform government agencies, under the leadership of the LGUs
and poverty alleviation; and concerned, shall provide integrated services and information to
(d) Coordinate and submit progress reports to the National Anti- prospective enterprises under the one-stop-shop concept.
Poverty Commission regarding their local action programs. Local government units are authorized to undertake investment
Nothing in this Act shall be construed as diminishing the powers and marketing missions provided that the costs of such missions
granted to the local government units under the Local are borne by the LGUs concerned. In making their land use
Government Code. plans, the LGUs, in consultation with the appropriate
government agencies concerned, shall identify areas for
industrial parks.
The Department shall coordinate with the Department of Trade
RA8435: An Act Prescribing Urgent Related Measures To
and Industry, in particular, the Board of Investments, in the
Modernize The Agriculture And Fisheries Sectors Of The
formulation of investment priorities for rural areas.
Country In Order To Enhance Their Profitability, And
The Regional Wage Boards shall consult participating
Prepare Said Sectors For The Challenges Of Globalization
enterprises in this program before they issue wage orders.
Through An Adequate, Focused And Rational Delivery Of
Necessary Support Services, Appropriating Funds Therefor
And For Other Purposes
RA8550: An Act Providing For The Development,
Management And Conservation Of The Fisheries And
Sec90: The Role of Local Government Units. — The LGUs shall
Aquatic Resources, Integrating All Laws Pertinent Thereto,
be responsible for delivering direct agriculture and fisheries
And For Other Purposes
extension services.

The Apocryphal Maggots:


Rainier, Chrisgel, Corina, Geoffry, Grace and Sylvie Blanche
The Flibbertigibbet Worms:
Golda, Gladys and Melyjane
CA VE AT: By simply r ea ding this re vi ew er a t the end o f the sem este r wi ll (hopef ully) guar antee you r pas sing this cou rse. D rink mode rat ely .
Amusin S C A r y
gly
THE C2005 LOCAL GOVERNMENT REVIEWER - 49 -
Sec16: Jurisdiction of Municipal/City Government. — The c. prior consultation, through public hearing, with the
municipal/city government shall have jurisdiction over municipal M/CFARMC has been conducted; and
waters as defined in this Code. The municipal/city government, d. the applicant vessel as well as the shipowner, employer,
in consultation with the FARMC shall be responsible for the captain and crew have been certified by the appropriate
management, conservation, development, protection, utilization, agency as not having violated this Code, environmental laws
and disposition of all fish and fishery/aquatic resources within and related laws.
their respective municipal waters. In no case shall the authorization or permit mentioned above be
The municipal/city government may, in consultation with the granted for fishing in bays as determined by the Department to
FARMC, enact appropriate ordinances for this purpose and in be in an environmentally critical condition and during closed
accordance with the National Fisheries Policy. The ordinances season as provided for in Section 9 of this Code.
enacted by the municipality and component city shall be
reviewed pursuant to Republic Act No. 7160 by the sanggunian Sec19: Registry of Municipal Fisherfolk. — The LGU shall
of the province which has jurisdiction over the same. maintain a registry of municipal fisherfolk, who are fishing or
The LGUs shall also enforce all fishery laws, rules and may desire to fish in municipal waters for the purpose of
regulations as well as valid fishery ordinances enacted by the determining priorities among them, of limiting entry into the
municipal/city council. municipal waters, and of monitoring fishing activities an/or other
The management of contiguous fishery resources such as bays related purposes: Provided, That the FARMC shall submit to the
which straddle several municipalities, cities or provinces, shall LGU the list of priorities for its consideration.
be done in an integrated manner, and shall not be based on Such list or registry shall be updated annually or as may be
political subdivisions of municipal waters in order to facilitate necessary, and shall be posted in barangay halls or other
their management as single resource systems. The LGUs which strategic locations where it shall be open to public inspection, for
share or border such resources may group themselves and the purpose of validating the correctness and completeness of
coordinate with each other to achieve the objectives of the list. The LGU, in consultation with the FARMCs, shall
integrated fishery resource management. The Integrated formulate the necessary mechanisms for inclusion or exclusion
Fisheries and Aquatic Resources Management Councils procedures that shall be most beneficial to the resident
(FARMCs) established under Section 76 of this Code shall serve municipal fisherfolk. The FARMCs may likewise recommend
as the venues for close collaboration among LGUs in the such mechanisms.
management of contiguous resources. The LGUs shall also maintain a registry of municipal fishing
vessels by type of gear and other boat particulars with the
Sec17: Grant of Fishing Privileges in Municipal Waters. — The assistance of the FARMC.
duly registered fisherfolk organizations/cooperatives shall have
preference in the grant of fishery rights by the Municipal/City Sec20: Fisherfolk Organizations and/or Cooperatives. —
Council pursuant to Section 149 of the Local Government Code: Fisherfolk organizations/cooperatives whose members are listed
Provided, That in areas where there are special agencies or in the registry of municipal fisherfolk, may be granted use of
offices vested with jurisdiction over municipal waters by virtue of demarcated fishery areas to engage in fish capture, mariculture
special laws creating these agencies such as, but not limited to, and/or fish farming: Provided, however, That an
the Laguna Lake Development Authority and the Palawan organization/cooperative member whose household is already in
Council for Sustainable Development, said offices and agencies possession of a fishery right other than for fish capture cannot
shall continue to grant permits for proper management and enjoy the fishing rights granted to the organization or
implementation of the aforementioned structures. cooperative.

Sec18: Users of Municipal Waters. — All fishery related activities Sec21: Priority of Resident Municipal Fisherfolk. — Resident
in municipal waters, as defined in this Code, shall be utilized by municipal fisherfolk of the municipality concerned and their
municipal fisherfolk and their cooperatives/organizations who organizations/cooperatives shall have priority to exploit
are listed as such in the registry of municipal fisherfolk. municipal and demarcated fishery areas of the said municipality.
The municipal or city government, however, may, through its
local chief executive and acting pursuant to an appropriate Sec22: Demarcated Fishery Right. — The LGU concerned shall
ordinance, authorize or permit small and medium commercial grant demarcated fishery rights to fishery
fishing vessels to operate within the ten point one (10.1) to organizations/cooperatives for mariculture operation in specific
fifteen (15) kilometer area from the shoreline in municipal waters areas identified by the Department.
as defined herein, provided, that all the following are met:
a. no commercial fishing in municipal waters with depth less Sec23: Limited Entry Into Overfished Areas. — Whenever it is
than seven (7) fathoms as certified by the appropriate determined by the LGUs and the Department that a municipal
agency; water is overfished based on available data or information or in
b. fishing activities utilizing methods and gears that are danger of being overfished, and that there is a need to
determined to be consistent with national policies set by the regenerate the fishery resources in that water, the LGU shall
Department; prohibit or limit fishery activities in the said waters.

The Apocryphal Maggots:


Rainier, Chrisgel, Corina, Geoffry, Grace and Sylvie Blanche
The Flibbertigibbet Worms:
Golda, Gladys and Melyjane
CA VE AT: By simply r ea ding this re vi ew er a t the end o f the sem este r wi ll (hopef ully) guar antee you r pas sing this cou rse. D rink mode rat ely .
Amusin S C A r y
gly
THE C2005 LOCAL GOVERNMENT REVIEWER - 50 -
Sec24: Support to Municipal Fisherfolk. — The Department and
the LGUs shall provide support to municipal fisherfolk through Facts:
appropriate technology and research, credit, production and The municipal council of Orion, Bataan, enacted an ordinance
marketing assistance and other services such as, but not limited which, prohibited the playing of panguingue on days not Sundays
to training for additional/supplementary livelihood. or legal holidays and penalized the violation by a casero
(housekeeper) and by jugadores (gamblers). On an evening not a
Sec25: Rights and Privileges of Fishworkers. — The Sunday or legal holiday, 7 persons, including Salaveria, the justice
fishworkers shall be entitled to the privileges accorded to other of the peace of Orion and his wife were surprised by the police
workers under the Labor Code, Social Security System and while indulging in a game of panguingue in the house of Salaveria.
other benefits under other laws or social legislation for workers: Convicted in the justice of the peace court and again in the CFI of
Provided, That fishworkers on board any fishing vessels Bataan, Salaveria appeals to the SC. Salaveria questioned among
engaged in fishing operations are hereby covered by the other things, the validity of the ordinance under which he was
Philippine Labor Code, as amended. convicted.

Held: Ordinance was valid


RA8975: An Act To Ensure The Expeditious Implementation If investigation was restricted to those portions of the Admin. Code
And Completion Of Government Infrastructure Projects By which authorize a municipal council to prohibit and penalize
Prohibiting Lower Courts From Issuing Temporary gambling, there would exist grave doubt of the validity of
Restraining Orders, Preliminary Injunctions Or Preliminary Ordinance No. 3.
Mandatory Injunctions, Providing Penalties For Violations However, the Philippine Legislature delegated to municipalities
Thereof, And For Other Purposes certain legislative powers of a discretionary nature. But in addition,
and preceding both the specific powers of a mandatory and
Sec3: Prohibition on the Issuance of Temporary Restraining discretionary character, is the general power of a municipal council
Orders, Preliminary Injunctions and Preliminary Mandatory to enact ordinances and make regulations. It is this grant that the
Injunctions. — No court, except the Supreme Court, shall issue preamble of the ordinance of Orion assigns as authority for its
any temporary restraining order, preliminary injunction or enactment. Section 2184 of the Administrative Code of 1916,
preliminary mandatory injunction against the government, or any known as the general welfare clause, delegates in statutory form
of its subdivisions, officials or any person or entity, whether the police power to a municipality. This clause has been given wide
public or private, acting under the government's direction, to application by municipal authorities and has in its relation to the
restrain, prohibit or compel the following acts: particular circumstances of the case been liberally construed by
(a) Acquisition, clearance and development of the right-of-way the courts.
and/or site or location of any national government project; It is a general rule that ordinances passed by virtue of the implied
(b) Bidding or awarding of contract/project of the national power found in the general welfare clause must be reasonable,
government as defined under Section 2 hereof; consonant with the general powers and purposes of the
(c) Commencement, prosecution, execution, implementation, corporation, and not inconsistent with the laws or policy of the
operation of any such contract or project; State.
(d) Termination or rescission of any such contract/project; and Thus, although panguingue is not named in the general law on
(e) The undertaking or authorization of any other lawful activity gambling, and although not entirely a game of chance, since it is a
necessary for such contract/project. proper subject for regulation by municipal authorities acting under
This prohibition shall apply in all cases, disputes or their delegated police power, whose laudable intention is to
controversies instituted by a private party, including but not improve the public morals and promote the prosperity of their
limited to cases filed by bidders or those claiming to have rights people, their action should be upheld by the courts. Ordinance No.
through such bidders involving such contract/project. This 3 of Orion is found to be valid.
prohibition shall not apply when the matter is of extreme
urgency involving a constitutional issue, such that unless a Dissenting (J. Fisher):
temporary restraining order is issued, grave injustice and The ordinance in question is beyond the protection of the general
irreparable injury will arise. The applicant shall file a bond, in an welfare clause for 2 reasons: (1) It is unreasonably subversive of
amount to be fixed by the court, which bond shall accrue in favor the liberty of the citizen and unnecessary and (2) the Legislature
of the government if the court should finally decide that the has spoken in well defined terms on the subject of gambling, and
applicant was not entitled to the relief sought. its pronouncement on the subject fills the field and precludes the
If after due hearing the court finds that the award of the contract possibility of stretching the authority delegated to municipalities
is null and void, the court may, if appropriate under the into the right to repeal, modify or supplement existing legislation.
circumstances, award the contract to the qualified and winning Act No. 1757 very clearly defines the intention and will of the
bidder or order a rebidding of the same, without prejudice to any Legislature with regard to gambling. Its limitation of the prohibition
liability that the guilty party may incur under existing laws. is its refusal to prohibit games of skill and games in which no value
is at stake. When the legislature authorized municipalities to
No Section 11 and 13. penalize gambling, it was aiming at the vice of risking money upon
the hazard of a game of chance. The Legislature has not
US vs. Salaveria (1918)

The Apocryphal Maggots:


Rainier, Chrisgel, Corina, Geoffry, Grace and Sylvie Blanche
The Flibbertigibbet Worms:
Golda, Gladys and Melyjane
CA VE AT: By simply r ea ding this re vi ew er a t the end o f the sem este r wi ll (hopef ully) guar antee you r pas sing this cou rse. D rink mode rat ely .
Amusin S C A r y
gly
THE C2005 LOCAL GOVERNMENT REVIEWER - 51 -
prohibited the playing of card games — in itself an innocent statute in themselves denoting habituality or a repetition of acts,
pastime — but the playing for money of games of hazard. When it and not a solitary act.
delegated like power to municipalities it had a like object in view
and no other. While assignment of police officers to insure that the funeral
procession is orderly so as not to cause great and serious
Equally untenable is the attempt to justify the statute under the inconvenience to the public is called for by every funeral
"general welfare" clause. The prohibition by ordinance of the procession, there is no explanation why the ordinance should
playing of certain card games as an amusement, without stake or collect the prescribed fees solely in the case of cadavers coming
wager, cannot be said to promote the health, safety, morals, from places outside the territory of Caloocan City for burial in
peace, good order, comfort or convenience of the inhabitants of a private cemeteries within the City.
municipality. The ordinance in question was intended to prevent
gambling, but is not warranted by the delegated authority of
municipal councils over this subject, because it is so drawn as to Balacuit vs. CFI (1988)
include harmless amusements not within the legislative definition
of gambling. Facts:
This case questions the validity and constitutionality of an
Ordinance No. 640 which penalizes anyone engaged in the
Viray vs. Caloocan (1967) business of selling admission tickets to any movie or other public
exhibitions, games, contests or other performances and to require
Facts: children between 7 and 12 years of age to pay half the cost for
The Municipal Board of Caloocan City enacted an Ordinance tickets intended for adults. Managers of 4 theaters sought to have
providing for additional fees in private cemeteries. Pursuant to the the ordinance declared invalid since (1) it is an invalid exercise of
ordinance, Viray, Cajote and Prieto, all residents of Manila, were police power and (2) and that the ordinance is not within the power
charged by and actually paid to the City Treasurer of Caloocan of the Municipal Board to enact as provided for in the Butaun City
“entrance fees” for the burial of their relatives in La Loma Charter.
cemetery, a burial place not owned by the city government.
Because the City Treasurer refused to refund the entrance fees Held: The city has no authority to enact the ordinance in
paid, the 3 filed an action in the CFI against the city government, question under its power to regulate embodied in Section
contesting the legality of the ordinance on the ground that it was a 15(n).
revenue-raising measure which is beyond the taxing authority of The power of regulation of public exhibitions and places of
the city government to enact. The city government claimed that it amusement within the city granted by the charter does not carry
was justified by the general grant of taxation to chartered cities by with it any authority to interfered with the price admission to such
Republic Act 2264 or the Local Autonomy Act and as a valid places or the resale of tickers or tokens of admission
exercise of police power.
Even if the ordinance is based on its police power as delegated
Held: Ordinance invalid under the welfare clause, ordinance is invalid. The exrcise of police
Taken in its entirety, the ordinance is clear that what is being power requires:
regulated through this ordinance is not the burying of the mortal 1. that the interest of the public generally requires an interference
remains of a person, but the exhumation and transfer of a cadaver with private rights and
— activities affecting corpses that had previously been buried. 2. the means adopted must be reasonably necessary for the
Although the ordinance which regulates the exhumation and/or accomplishment of the purpose and
transfer of corpses from other burial grounds to those located in 3. not unduly oppressive upon individuals
Caloocan is within the legislative power of the city government to
enact, the imposition of the transfer fees under the ordinance, on In the CAB, the was (1) no public necessity which demands the
the interment of the respective dead relatives of appellants in the adopting of proper measures to secure the end sought to be
La Loma cemetery, was not justified. attained by the enactment of the ordinance; (2) the means means
adopted not reasonably necessary for the accomplishment of
While sec. 2 of RA 2264 confers on chartered cities and purpose and (3) ordinance is clearly unreasonable if not unduly
municipalities "authority to impose municipal license taxes or fees oppressive upon the business of the petitioners. There is no no
upon persons engaged in any occupation or business, or discernable relation between the ordinance and the promotion of
exercising privileges in chartered cities or municipalities...by public health, safety, morals and general welfare
requiring them to secure licenses at rates fixed by the municipal
board or city council," the city government has failed to show that Gutierrez. Separate Opinion
the persons merely burying a cadaver in a private cemetery  police power is inherent in the State but not in municipal
constitutes either an occupation or business or the exercise of corporations
privileges that would justify the imposition of taxes thereon within  no rational basis for classifying children as a distinct group
the terms and intent of the enabling act, the terms employed by the insofar as paying for admission into a moviehouse is
concerned.

The Apocryphal Maggots:


Rainier, Chrisgel, Corina, Geoffry, Grace and Sylvie Blanche
The Flibbertigibbet Worms:
Golda, Gladys and Melyjane
CA VE AT: By simply r ea ding this re vi ew er a t the end o f the sem este r wi ll (hopef ully) guar antee you r pas sing this cou rse. D rink mode rat ely .
Amusin S C A r y
gly
THE C2005 LOCAL GOVERNMENT REVIEWER - 52 -

City Government of Quezon City vs. Ericta (1983)


Binay vs. Domingo (1991)
Facts:
Facts: QC passed an Ordinance regulating the establishment,
The Municipal Council of Makati issued a Resolution confirming maintenance & operation of private memorial type cemetery or
and/or ratifying the ongoing burial assistance program initiated by burial ground within the jurisdiction of QC. §9 of the Ordinance
the Office of the Mayor. The Metro Manila Commission (MMC) provides that at least 6% of the total area of a memorial park
approved Resolution No. 60. Thereafter, the municipal secretary cemetery shall be set aside for charity burial of deceased persons
certified a disbursement fund of P400,000 for the implementation of who are paupers & have been residents of QC for at least 5 yrs
the Burial Assistance Program. Resolution was then referred to the prior to their death.
Commission on Audit (COA) for its expected allowance in audit. 7 yrs after the enactment of the Ordinance, the QC Council
However, COA, after its preliminary findings, disapproved passed a resolution requesting the City Engineer to stop any
Resolution No. 60 and disallowed in audit the disbursement of further selling of memorial parks in QC where the owners have
funds for the implementation thereof. Mayor Binay filed two letters failed to donate the required 6% cemetery space. The City Engr
for reconsideration. COA denied both letters for reconsideration. notified Himlayang Pilipino, Inc. that the Ordinance would be
enforced, so Himlayan filed a petition with the CFI seeking to annul
Held: Resolution is a valid exercise of police power under the §9 of the Ordinance. CFI declared §9 null and void. Motion for
general welfare Clause recon denied.
Police power, which is an attribute of sovereignty, is inherent in the
state but not in municipal corporations. Before a municipal Held: Ordinance not authorized under QC Charter and invalid
corporation may exercise such power, there must be a valid exercise of police power
delegation of such power of the state.
An examination of the Charter does not reveal any provision that
Municipal governments exercise this power under the general would justify the ordinance in question except the provision
welfare clause of BP337 (old LGC) – exercise powers expressly granting police power to the City.
granted, necessarily implied, and necessary and proper to promote
health & safety, etc. Police power is the most essential of govt powers, at times the
most insistent, and always one of the least limitable of the powers
of govt.
COA, in saying that there is no perceptible connection, tries to It is usually exercised in the form of mere regulation or restriction
redefine the scope of police power by circumscribing its exercise to in the use of liberty or property for the promotion of the general
“public safety, general welfare, etc of the inhabitants of Makati.” welfare. It does not involve the taking or confiscation of property
with the exception of a few cases where there is necessity to
The police power of a municipal corporation is: broad; extends to confiscate private property in order to destroy it [not to devote it to
all the great public needs; in a broad sense includes all legislation public use] (i.e., in the case of confiscation of opium & firearms).
and almost every function of the municipal government. Thus, it is HERE  §9 of the Ordinance is not a mere police regulation but
deemed inadvisable to attempt to frame any definition which shall an outright confiscation. It deprives a person of his private property
absolutely indicate the limits of police power. w/o due process…even w/o compensation.
As regards COA’s additional objection, it shows that it is not
attuned to the changing of times. Public purpose is not
unconstitutional merely because it incidentally benefits a limited Tatel vs. Virac (1991)
number of persons.
Facts:
The care for the poor is general recognized as a public duty. The In 1966, complaints were received from the residents of barrio Sta.
support for the poor has long been an accepted exercise of police Elena against disturbance caused by operation of the abaca
power in the promotion of common good. bailing machine inside the warehouse of Tatel. A committee was
appointed by the municipal council of Virac to investigate on the
There is no violation of the equal protection clause in classifying matter The committee noted the crowded nature of the
paupers as subject of legislation. Paupers may be reasonably neighborhood with narrow roads and the surrounding residential
classified. houses so much so that accidental fire from the continued
operations of the warehouse and storing of inflammable materials
BUT, this decision must not be taken as a precedent, or as an created danger to the lives and properties of the people. Based on
official go-signal for municipal governments to embark on this report, the Municipal Council of passed a Resolution declaring
philanthropic ORGY of inordinate dole-outs for motives political or the warehouse a nuisance within purview of Art 694 of the Civil
otherwise. Code. Tatel's MR denied so Tatel instituted the present petition.

The Apocryphal Maggots:


Rainier, Chrisgel, Corina, Geoffry, Grace and Sylvie Blanche
The Flibbertigibbet Worms:
Golda, Gladys and Melyjane
CA VE AT: By simply r ea ding this re vi ew er a t the end o f the sem este r wi ll (hopef ully) guar antee you r pas sing this cou rse. D rink mode rat ely .
Amusin S C A r y
gly
THE C2005 LOCAL GOVERNMENT REVIEWER - 53 -
Held: Ordinance was a valid exercise of police power despite of building permit with the trial court. TC dismissed complaint.CA
its fractured syntax. reverses.
It is a settled principal of law that municipal corporations are
agencies of the State for the promotion and maintenance of local Held: Mr. Tepoot's building was commercial in nature; Hence
self-government and as such are endowed with police powers in Ordinance was not violated
order to effectively accomplish and carry out the declared objects A tax declaration is not conclusive of the nature of the property for
of their creation. zoning purposes. A property may have been declared by its owner
Authority emanates from the general welfare clause under the as residential for real estate taxation purposes but it may well be
Administrative Code: within a commercial zone.
"The municipal council shall enact such ordinance and make Even if we are to examine the evidentiary value of a tax declaration
such regulations, not repugnant to law, as may be necessary to under the Real Property Tax Code, a tax declaration only enables
carry into effect and discharge the powers and duties conferred the assessor to identify the same for assessment levels.
upon it by law and such as shall seem necessary and proper to In fact, a tax declaration does not bind a provincial/city assessor,
provide for the health and safety, promote the prosperity, for under Sec. 22 of the Real Estate Tax Code, appraisal and
improve the morals, peace, good order, comfort and assessment are based on the actual use irrespective of "any
convenience of the municipality and the inhabitants thereof, and previous assessment or taxpayer's valuation thereon," which is
for the protection of property therein." based on a taxpayer's declaration.
For an ordinance to be valid, it must not only be within the The trial court's determination that Mr. Tepoot's building is
corporate powers of the municipality to enact but must also be commercial is strengthened by the fact that the Sangguniang
passed according to the procedure prescribed by law, and must be Panlungsod has declared the questioned area as commercial or C-
in consonance with certain well established and basic principles of 2. Consequently, even if Tepoot's building was declared for
substantive nature: taxation purposes as residential, once a local government has
(i). must not contravene the Constitution or any statue reclassified an area as commercial, that determination for
(ii). must not be unfair or oppressive zoning purposes must prevail.
(iii). must not be partial or discriminatory
(iv). must not prohibit but may regulate trade The declaration of the said area as a commercial zone through a
(v). must be general and consistent with public policy, and municipal ordinance is an exercise of police power to promote the
(vi). must not be unreasonable good order and general welfare of the people in the locality.
Corollary thereto, the state, in order to promote the general
welfare, may interfere with personal liberty, with property, and with
business and occupations. Thus, persons may be subjected to
certain kinds of restraints and burdens in order to secure the
general welfare of the state and to this fundamental aim of
Patalinghug vs. CA (1994) government, the rights of the individual may be subordinated. The
ordinance which regulates the location of funeral homes has been
Facts: adopted as part of comprehensive zoning plans for the orderly
In 1982, SP of Davao City enacted an Ordinance known as the " development of the area covered thereunder.
Expanded Zoning Ordinace of Davao City" , Section 8 of which
states:
Section 8.USE REGULATIONS IN C-2 DISTRICTS (Shaded
light red in the Expanded Zoning Map) A C -2 District shall Rep. of the Philippines vs. City of Davao (2002)
be dominantly for commercial and compatible industrial uses
as provided hereunder: Facts:
3.1 Funeral Parlors/Memorial Homes with adequate off City of Davao filed an application for a Certificate of Non-Coverage
(CNC) for its proposed project, the Davao City Sports Dome with
street parking space and provided that they shall be
the Environmental Management Bureau (EMB) with the required
established not less than 50 meters from any residential
documents. EMB denied the application after finding that the
structures, churches and other institutional buildings.
proposed project was w/in an environmentally critical area. It also
Petitioner applied for certification of zoning compliance for the
held that Davao must undergo the environmental impact
construction of a funeral parlor. Certification approved. Petitioner
assessment (EIA) to secure Environmental Compliance Certificate
commenced construction. Several residents filed complaint that
(ECC) before it can proceed with construction of its project
petitioner's funeral parlor violated Ordiance since it was allegedly
pursuant to Sec 2 of PD 1586, Environmental Impact Statement
within a 50-meter radius from the Iglesia ni Kristo chapel. SP found
System, in relation to Sec 4 PD 1151, Philippine Environment
that the nearest residential structure owned by Wilfred Tepoot is
Policy. Davao complains.
only 8 inches to the south. Notwithstanding the finding, petitioner
continued and finished the construction of the funeral parlor.
Held: Davao City within the scope of the EIS law
Private respondents herein filed a case for the declaration of nullity
An LGU is a body politic and corporate endowed with powers to be
exercised inconformity with law. It has dual functions:

The Apocryphal Maggots:


Rainier, Chrisgel, Corina, Geoffry, Grace and Sylvie Blanche
The Flibbertigibbet Worms:
Golda, Gladys and Melyjane
CA VE AT: By simply r ea ding this re vi ew er a t the end o f the sem este r wi ll (hopef ully) guar antee you r pas sing this cou rse. D rink mode rat ely .
Amusin S C A r y
gly
THE C2005 LOCAL GOVERNMENT REVIEWER - 54 -
• Governmental – concerns health, safety, advancement, of RA9165: An Act Instituting The Comprehensive Dangerous
public good and welfare. It acts as an agency of national gov’t Drugs Act Of 2002, Repealing Republic Act No. 6425,
• Proprietary – seeks to obtain special corporate benefits or Otherwise Known As The Dangerous Drugs Act Of 1972, As
earn pecuniary profit and intended for private advantage and Amended, Providing Funds Therefor, And For Other
benefit. Acts as agent of community. Purposes

As a body politic endowed with governmental functions, LGU has Sec52: Abatement of Drug Related Public Nuisances. — Any
the duty to promote the people’s right to a balanced ecology (Sec. place or premises which have been used on two or more
16 LGCode) and to ensure quality of environment. Pursuant to this, occasions as the site of the unlawful sale or delivery of
Davao cannot claim exemption from the coverage of EIS law which dangerous drugs may be declared to be a public nuisance, and
has same the objectives. such nuisance may be abated, pursuant to the following
procedures:
Civil Code defines a person as either natural or juridical. The state (1) Any city or municipality may, by ordinance, create an
and its subdivisions i.e. LGU’s are juridical persons. Thus, LGU’s administrative board to hear complaints regarding the
are not excluded from EIS law. nuisances;
(2) Any employee, officer, or resident of the city or municipality
Sec 1 of EIS law intends to implement state policy to achieve a may bring a complaint before the Board after giving not less
balance between socio-economic development and environmental than three (3) days written notice of such complaint to the owner
protection. The Whereas clause of the same law stresses that this of the place or premises at his/her last known address; and
balance can only be achieved through a comprehensive and (3) After hearing in which the Board may consider any evidence,
integrated program where all the sectors of the community – gov’t including evidence of the general reputation of the place or
and private – are involved. premises, and at which the owner of the premises shall have an
Thus, LGU’s as part of the machinery of the gov’t cannot be opportunity to present evidence in his/her defense, the Board
deemed outside the scope of the EIS law. may declare the place or premises to be a public nuisance.

Since it is clear that the said project is not classified as Sec53: Effect of Board Declaration. — If the Board declares a
environmentally critical nor within critical area, DENR has no place or premises to be a public nuisance, it may declare an
choice but to issue the CNC. It is a ministerial duty that can be order immediately prohibiting the conduct, operation, or
compelled by a writ of mandamus. maintenance of any business or activity on the premises which
is conducive to such nuisance.
MyPleasure Guide to Sex Toys: Sex Toy Basics An order entered under this Section shall expire after one (1)
Sex toys come in thousands of different sizes, shapes and year or at such earlier time as stated in the order. The Board
functions -- you could have a different toy for every day of the may bring a complaint seeking a permanent injunction against
week! But to narrow the search field, think about your needs any nuisance described under this Section.
and preferences, and consider the following questions: This Article does not restrict the right of any person to proceed
under the Civil Code against any public nuisance.
What shapes appeal to you?
Do you find some sex toys silly looking?
Do you have a color preference?
Estate of Francisco vs. CA
Do you know what materials you like most: silicone, jelly,
rubber or plastic?
Facts:
Do you want stay traditional or try something new and wild? A Quonset hut standing on a lot owned by Phil. Ports Authority
Do you have trouble reaching orgasm during penetration? (PPA) in Isabela, Basilan was purchased by Gregoria Francisco in
Do you consider yourself to be "clitoral?" 1944. Proclamation 83 declared said land was for the exclusive
Do you want to use your toy alone or with a partner? use of port facilities. PPA issued to Tan Gin San the surviving
(continued) spouse of Gregoria a permit to occupy the lot of the Quonset for a
Abatement of Nuisance year for storage of copra to expire Dec 31, 1989. Mayor Valencia
notified Tan twice to relocate Quonset citing Zoning Ordinance
Local Government Code No.14. It noted its antiquated and dilapidated structure and
stressed the "clear-up campaign on illegal squatters and
Sangguniang Bayan – Sec447, LGC: Powers, Duties,
unsanitary surroundings along Strong Boulevard."
Functions and Compensation. –
Since the notifications remained unheeded by Mayor ordered the
demolition.
Sangguniang Panlungsod – Sec458, LGC: Powers, Duties,
Functions and Compensation. –
Held: Mayor cannot summarily order the demolition.
Ordinance 147 does not authorize summary removal of non-
conforming structure. Sec 16 of the ordinance provides that a
certificate of non-conformance is necessary but even if Tan failed

The Apocryphal Maggots:


Rainier, Chrisgel, Corina, Geoffry, Grace and Sylvie Blanche
The Flibbertigibbet Worms:
Golda, Gladys and Melyjane
CA VE AT: By simply r ea ding this re vi ew er a t the end o f the sem este r wi ll (hopef ully) guar antee you r pas sing this cou rse. D rink mode rat ely .
Amusin S C A r y
gly
THE C2005 LOCAL GOVERNMENT REVIEWER - 55 -
to obtain one, the provision should not be interpreted as ordered padlocking of TDI’s plant premises without previous and
authorizing the summary removal of a non-conforming building by reasonable notice upon TDI. TDI instituted an action with
the municipal gov’t. Otherwise, it would be violative of due process RTC.RTC ruled in favor of mayor. MR by TDI denied. CA affirmed.
and must be struck down.
Held: The dissolution of the writ of preliminary injunction is
LGCode imposes upon the mayor the duty "to cause to be justified by the trial court and correctly upheld by CA.
instituted judicial proceedings in connection with the violation of The following circumstances justify the dissolution:
ordinances" (LGCode, Sec. 141 [2] [t]). Thus, the violation of a 1. No Mayor’s Permit. The mayor can deny application for a
municipal ordinance neither empowers the Municipal Mayor to
permit to operate a business or close it by his police power
avail of extra-judicial remedies.
unless appropriate measures are taken to control or avoid
injury to the health of the residents of the community from
The general welfare clause authorizing the abatement of
emissions in the operation of the business.
nuisances without judicial proceedings applies to a nuisance per
While determination whether there is pollution of the environment
se, or one which affects the immediate safety of persons and
that requires control (if not prohibition) of the operation of a
property and may be summarily abated under the undefined law of
business is essentially addressed to the EMB (National Pollution
necessity
Control Commission before) of DENR, the mayor of a town has as
much responsibility to protect its inhabitants from pollution.
The storage of copra in the quonset is a legitimate business. By its
2. Acting Mayor had called the attention of TDI to the pollution
nature, it can not be said to be injurious to rights of property, of
emitted by the fumes of its plant whose offensive odor also
health or of comfort of the community. If it be a nuisance per
affects the health of the residents so that it was ordered to
accidens, it may be so proven in a hearing conducted for that
stop its operation until further orders and required to bring
purpose. It is not per se a nuisance warranting its summary
permits.
abatement without judicial intervention.
3. The action of the Mayor was in response to complaint of the
residents directed to the Provincial Governor.
While the Sangguniang Bayan may provide for the abatement of a
Although it may be true that some signatures in the 4page petition
nuisance (LGCode, Sec. 149 [ee]), it cannot declare a particular
were written by one person (particularly among members of same
thing as a nuisance per se and order its condemnation. The
family), on the whole the many signatures appear to be written by
nuisance can only be so adjudged by judicial determination.
different persons.
Certification of barrio captain that he has not received any
In CAB, Tan was in lawful possession of the lot and quonset by complaint on the matter must be because complaint was sent
virtue of PPA permit. It was not squatting on public land. Its directly to the Governor through Acting Mayor.
property was not of trifling value. It was thus entitled to an impartial 4. Closure order was issued after an investigation made by
hearing before a tribunal authorized to decide whether the quonset Guina – the fumes emitted by plant goes directly to
building did constitute a nuisance in law. surrounding houses and that no proper air pollution device
has been installed.
There was no compelling necessity for precipitate action. The 5. TDI failed to produce a building permit from Sta. Maria
mayor et al transcended their authority in abating summarily the municipality but instead presented a Makati building permit.
quonset building. They had deprived petitioner of its property 6. The temporary permit to operate from National Pollution
without due process of law. Control Commission TDI presented was good only up to
May25, 1988. TDI did not exert any effort to extend or validate
its permit nor to install any device to control the pollution and
Technology Developers, Inc. vs. CA (1991) 193SCRA147 prevent any hazard to the health of the residents of the
community.
Facts: Although TDI had spent a huge investment in its dollar-earning
Technology Developers, Inc. (TDI) is a domestic private industry, it must be stressed that concomitant with the need to
corporation engaged in the manufacture and export of charcoal promote investment and contribute to the growth of the economy is
briquette. It received a letter from acting mayor ordering full the equally essential imperative of protecting the health and lives
cessation of operation of its Sta. Maria plant and requesting Plant of the people from the deleterious effect of the pollution of the
Managaer to bring to the office of the mayor permits (Building, environment.
Mayor’s, and Pollution of Environment and Natural Resources Anti-
Pollution). As to the Anti-Pollution Permit, TDI tried to secure it
although it had previously secured before its operation a Technology Developers, Inc. vs. CA (1991) 201SCRAxi
“Temporary Permit to Operate Air Pollution Installation” issued by
Environmental Management Bureau (EMB). EMB is at a stage Facts:
trying to determine correct kind of anti-pollution devise to be TDI filed a motion for reconsideration of the SC decision denying
installed for TDI’s renewal of its permit. TDI didn’t have a mayor’s its petition for review:
permit so it tried to secure one but it was not entertained. Mayor

The Apocryphal Maggots:


Rainier, Chrisgel, Corina, Geoffry, Grace and Sylvie Blanche
The Flibbertigibbet Worms:
Golda, Gladys and Melyjane
CA VE AT: By simply r ea ding this re vi ew er a t the end o f the sem este r wi ll (hopef ully) guar antee you r pas sing this cou rse. D rink mode rat ely .
Amusin S C A r y
gly
THE C2005 LOCAL GOVERNMENT REVIEWER - 56 -
1. SC decision being anchored on factual allegation in mayor’s certified that the plant “is in accordance with the development
pleadings, which all courts assumed to be true, TDI alleges plan and zonification of the locality
the following to rebut the mayor’s factual bases:  Acting mayor may not capriciously deny a permit to operate a
a. TDI obtained a Mayor’s permit but from the wrong mayor legitimate business on the ground that its plant was causing
(in Makati where its principal office is located). Upon excessive air pollution
being apprised of the need to obtain one in Sta. Maria,  Only the EMB, after due notice and hearing may determine
TDI applied but was rebuffed. whether excessive pollution exists as held in the case Mead vs.
b. Petition against the charcoal plant was “padded” - NBI Argel that EMB is given authority by Sec6:
handwriting expert certified that several signatures in the a. to determine whether a pollution exists in any of the waters
petition by supposed residents of Brgy. Guyong were and/or atmospheric air of the Philippines
written by only one person b. to hold public hearings, make findings of facts and
c. Guina’s report that smoke from plant “contain particulate determinations all with respect to violations of the Act or
matters hazardous to health” has not been verified and orders issued by the Commission
confirmed by EMB. c. to institute or cause to be instituted in competent court legal
d. TDI has a building permit issued by Building Official of proceedings to compel compliance
Ministry of Public Works and Highways – a higher d. after due notice and hearing, revoke, suspend or modify any
authority than the building official of Sta. Maria. permit issued under the Act whenever modifications are
e. Permit issued by EMB authorizing TDI’s operation was necessary to prevent or abate pollution…
renewed and extended up to Dec.1990. It was not e. Sec8 on authority of Commission to determine existence of
renewed in 1991 because of the mayor’s closure order. pollution and to take appropriate court actions to abate or
2. SC must examine the jurisdictional issue of whether a town prevent it
mayor is empowered by law to close a plant/business (a) for f. That in matters not related to nuisance, no court action shall
lack of a mayor’s permit and (b) for alleged violation of anti- be initiated until Commission has determined existence of
pollution laws. RTC and CA turned upon the mayor’s what in law is considered pollution
justifications for closure order issuance, not his jurisdiction to
do so. It is beyond a municipal mayor’s and competence to review, revise,
reverse, or set aside a permit to operate LTD’s charcoal briquette
Held: Acting Mayor is without jurisdiction to issue closure orde. plant issued by EMB, which is the primary authority whether TDI’s
PD984 is the applicable law which created the National Pollution manufacturing process violates anti-pollution laws, rules and
Control Commission (later renamed to EMB) as the primary regulations.
agency responsible for prevention and control of environmental 1. Complaint against TDI should have been addressed to EMB
pollution in the country: which alone is empowered to investigate and determine (after
1. Sec10 and 17, PD984 affirms its primacy of jurisdiction on public hearing) whether:
matters of air, water and land pollution by providing that rules a. Charcoal plant was causing air pollution in excess of
and regulations issued by EMB for prevention of pollution permissible limits
“shall supersede and prevail over any rules or regulations as b. Discharge of smoke from TDI’s 16meter smoke stack
may heretofore have been issued by other government should be reduced or discontinued
agencies… any provision of laws…. c. Additional devices for that purpose should be installed
 Evidently, even provisions of Civil Code on nuisance (insofar as d. Its business should be temporarily suspended or totally
it is caused by pollution of air, water or land resources) are banned
deemed superseded by PD984 (a special law on subject of 2. EMB (special agency created by law for this purpose)
pollution) assisted by its staff with expertise and facilities is one
2. Sec6 enumerates powers and functions to EMB which are not competent to investigate and determine degree and extent of
conferred by law on town officials (see #4 enumerated under air pollution in plant site and around it, and determine the
Mead vs. Argel case) danger it poses to people’s health
 Never the law’s intention to place in the hands of just any
3. Sec7 requires that “public hearings shall be conducted by the layman the authority to determine if pollution exists, and the power
Commissioner xxx prior to issuance or promulgation of any to toll the death knell of a multi-million peso industry which not only
order or decision requiring discontinuance of discharge of xxx provides jobs but also brings in export dollars for the country’s
industrial wastes xxx into water, air or land resources of the dollar-starved economy.
Philippines”
 No public hearings was held by Acting Mayor
 It is the discharge of industrial wasters, not the operation of RA8749: An Act Providing For A Comprehensive Air
business, that may be discontinued Pollution Control Policy And For Other Purposes
4. TDI had been issued by EMB to operate its plant in 1987-
1988, renewed for the year 1989, and again for 1990, and the Sec7: Integrated Air Quality Improvement Framework. — The
Deputy Zoning Administrator of Sta. Maria, Bulacan had Department shall, within six (6) months after the effectivity of this
Act, establish, with the participation of LGUs, NGOs, POs, the

The Apocryphal Maggots:


Rainier, Chrisgel, Corina, Geoffry, Grace and Sylvie Blanche
The Flibbertigibbet Worms:
Golda, Gladys and Melyjane
CA VE AT: By simply r ea ding this re vi ew er a t the end o f the sem este r wi ll (hopef ully) guar antee you r pas sing this cou rse. D rink mode rat ely .
Amusin S C A r y
gly
THE C2005 LOCAL GOVERNMENT REVIEWER - 57 -
academe and other concerned entities from the private sector, from time to time, revise information on air pollution control
formulate and implement the Integrated Air Quality Improvement techniques. Such information shall include…
Framework for a comprehensive air pollution management and
control program. The framework shall, among others, prescribe Sec16: Permits. — Consistent with the provisions of this Act, the
the emission reduction goals using permissible standards, Department shall have the authority to issue permits as it may
control strategies and control measures to be undertaken within determine necessary for the prevention and abatement of air
a specified time period, including cost-effective use of economic pollution.
incentives, management strategies, collective action, and Said permits shall cover emission limitations for the regulated air
environmental education and information. pollutants to help attain and maintain the ambient air quality
The Integrated Air Quality Improvement Framework shall be standards. These permits shall serve as management tools for
adopted as the official blueprint with which all government the LGUs in the development of their action plan.
agencies must comply with to attain and maintain ambient air
quality standards. Sec20: Ban on Incineration. — …
Local government units are hereby mandated to promote,
Sec8: Air Quality Control Action Plan. — …. encourage and implement in their respective jurisdiction a
Likewise, the LGUs, with the assistance from the Department, comprehensive ecological waste management that includes
shall prepare and develop an action plan consistent with the waste segregation, recycling and composting.
Integrated Air Quality Improvement Framework to attain and With due concern on the effects of climate change, the
maintain the ambient air quality standards within their respective Department shall promote the use of state-of-the-art,
airsheds as provided in Section 9 hereof. environmentally-sound and safe non-burn technologies for the
The local government units shall develop and submit to the handling, treatment, thermal destruction, utilization, and disposal
Department a procedure for carrying out the action plan for their of sorted, unrecycled, uncomposted municipal, bio-medical and
jurisdiction. The Department, however, shall maintain its hazardous wastes.
authority to independently inspect the enforcement procedure
adopted. The Department shall have the power to closely Sec24: Pollution from Smoking. — Smoking inside a public
supervise all or parts of the air quality action plan until such time building or an enclosed public place including public vehicles
the local government unit concerned can assume the function to and other means of transport or in any enclosed area outside of
enforce the standards set by the Department. one's private residence, private place of work or any duly
A multi-sectoral monitoring team with broad public designated smoking area is hereby prohibited under this Act.
representation shall be convened by the Department for each This provision shall be implemented by the LGUs.
LGU to conduct periodic inspections of air pollution sources to
assess compliance with the emission limitations contained in Sec36: Role of Local Government Units. — Local government
their permits. units (LGUs) shall share the responsibility in the management
and maintenance of air quality within their territorial jurisdiction.
Sec9: Airsheds. — (Are these analogous to watersheds?) Consistent with Sections 7, 8 and 9 of this Act, LGUs shall
implement air quality standards set by the Board in areas within
Sec10: Management of Nonattainment Areas. — The their jurisdiction: Provided, however, That in case where the
Department shall designate areas where specific pollutants Board has not been duly constituted and has not promulgated
have already exceeded ambient standards as nonattainment its standards, the standards set forth in this Act shall apply.
areas. The Department shall prepare and implement a program The Department shall provide the LGUs with technical
that will prohibit new sources of exceeded air pollutant without a assistance, trainings and a continuing capability-building
corresponding reduction in existing sources. program to prepare them to undertake full administration of the
In coordination with other appropriate government agencies, the air quality management and regulation within their territorial
LGUs shall prepare and implement a program and other jurisdiction.
measures including relocation, whenever necessary, to protect
the health and welfare of residents in the area. Sec37: Environment and Natural Resources Office. — There
For those designated as nonattainment areas, the Department, may be established an Environment and Natural Resources
after consultation with local government authorities, Office in every province, city, or municipality which shall be
nongovernment organizations (NGOs), people's organizations headed by the environment and natural resources officer and
(POs) and concerned sectors may revise the designation of shall be appointed by the Chief Executive of every province, city
such areas and expand its coverage to cover larger areas or municipality in accordance with the provisions of Section 484
depending on the condition of the areas. of Republic Act No. 7160. Its powers and duties, among others,
are:
Sec11: Air Quality Control Techniques. — Simultaneous with the a) To prepare comprehensive air quality management programs,
issuance of the guideline values and standards, the Department, plans and strategies within the limits setforth in Republic Act No.
through the research and development program contained in 7160 and this Act which shall be implemented within its territorial
this Act and upon consultation with the appropriate advisory jurisdiction upon the approval of the sanggunian;
committees, government agencies and LGUs, shall issue, and

The Apocryphal Maggots:


Rainier, Chrisgel, Corina, Geoffry, Grace and Sylvie Blanche
The Flibbertigibbet Worms:
Golda, Gladys and Melyjane
CA VE AT: By simply r ea ding this re vi ew er a t the end o f the sem este r wi ll (hopef ully) guar antee you r pas sing this cou rse. D rink mode rat ely .
Amusin S C A r y
gly
THE C2005 LOCAL GOVERNMENT REVIEWER - 58 -
b) To provide technical assistance and support to the governor (g) A representative from the NGO sector whose principal
or mayor, as the case may be, in carrying out measures to purpose is to promote recycling and the protection of air and
ensure the delivery of basic services and the provision of water quality;
adequate facilities relative to air quality; (h) A representative from the recycling industry;
c) To take the lead in all efforts concerning air quality protection (i) A representative from the manufacturing or packaging
and rehabilitation; industry; and
d) To recommend to the Board air quality standards which shall (j) A representative of each concerned government agency
not exceed the maximum permissible standards set by national possessing relevant technical and marketing expertise as may
laws; be determined by the Board.
e) To coordinate with other government agencies and non- The Provincial Solid Waste Management Board may, from time
governmental organizations in the implementation of measures to time, call on any other concerned agencies or sectors as it
to prevent and control air pollution; and may deem necessary.
f) Exercise such other powers and perform such duties and Provided, That representatives from the NGOs, recycling and
functions as may be prescribed by law or ordinance: Provided, manufacturing or packaging industries shall be selected through
however, That in provinces/cities/municipalities where there are a process designed by themselves and shall be endorsed by the
no environment and natural resources officers, the local government agency representatives of the Board: Provided,
executive concerned may designate any of his official and/or further, That in the Province of Palawan, the Board shall be
chief of office preferably the provincial, city or municipal chaired by the chairman of the Palawan Council for Sustainable
agriculturist, or any of his employee: Provided, finally, That in Development, pursuant to Republic Act No. 7611.
case an employee is designated as such, he must have a In the case of Metro Manila, the Board shall be chaired by the
sufficient experience in environmental and natural resources chairperson of the MMDA and its members shall include:
management, conservation and utilization. (i) All mayors of its component cities and municipalities;
(ii) A representative from the NGO sector whose principal
Sec39: Public Education and Information Campaign. — purpose is to promote recycling and the protection of air and
water quality;
RA9003: AN ACT PROVIDING FOR AN ECOLOGICAL SOLID (iii) A representative from the recycling industry; and
WASTE MANAGEMENT PROGRAM, CREATING THE (iv) A representative from the manufacturing or packaging
NECESSARY INSTITUTIONAL MECHANISMS AND industry.
INCENTIVES, DECLARING CERTAIN ACTS PROHIBITED The Board may, from time to time, call on any other concerned
AND PROVIDING PENALTIES, APPROPRIATING FUNDS agencies or sectors as it may deem necessary.
THEREFOR, AND FOR OTHER PURPOSES Provided, That representatives from the NGOs, recycling and
manufacturing or packaging industries shall be selected through
Sec10: Role of LGUs in Solid Waste Management. — Pursuant a process designed by themselves and shall be endorsed by the
to the relevant provisions of R.A. No. 7160, otherwise known as government agency representatives of the Board.
the Local Government Code, the LGUs shall be primarily The Provincial Solid Waste Management Board shall have the
responsible for the implementation and enforcement of the following functions and responsibilities:
provisions of this Act within their respective jurisdictions. (1) Develop a provincial solid waste management plan
Segregation and collection of solid waste shall be conducted at from the submitted solid waste management plans of the
the barangay level specifically for biodegradable, compostable respective city and municipal solid waste management boards
and reusable wastes: Provided, That the collection of non- herein created. It shall review and integrate the submitted plans
recyclable materials and special wastes shall be the of all its component cities and municipalities and ensure that the
responsibility of the municipality or city. various plans complement each other, and have the requisite
components. The Provincial Solid Waste Management Plan
Sec11: Provincial Solid Waste Management Board. — A shall be submitted to the Commission for approval.
Provincial Solid Waste Management Board shall be established The Provincial Plan shall reflect the general program of action
in every province, to be chaired by the governor. Its members and initiatives of the provincial government in implementing a
shall include: solid waste management program that would support the
(a) All the mayors of its component cities and municipalities; various initiatives of its component cities and municipalities.
(b) One (1) representative from the Sangguniang Panlalawigan (2) Provide the necessary logistical and operational
to be represented by the chairperson of either the Committees support to its component cities and municipalities in consonance
on Environment or Health or their equivalent committees, to be with subsection (f) of Sec. 17 of the Local Government Code;
nominated by the presiding officer; (3) Recommend measures and safeguards against
(c) The provincial health and/or general services officers, pollution and for the preservation of the natural ecosystem;
whichever may be recommended by the governor; (4) Recommend measures to generate resources,
(d) The provincial environment and natural resources officer; funding and implementation of projects and activities as
(e) The provincial engineer; specified in the duly approved solid waste management plans;
(f) Congressional representative/s from each congressional (5) Identify areas within its jurisdiction which have
district within the province; common solid waste management problems and are

The Apocryphal Maggots:


Rainier, Chrisgel, Corina, Geoffry, Grace and Sylvie Blanche
The Flibbertigibbet Worms:
Golda, Gladys and Melyjane
CA VE AT: By simply r ea ding this re vi ew er a t the end o f the sem este r wi ll (hopef ully) guar antee you r pas sing this cou rse. D rink mode rat ely .
Amusin S C A r y
gly
THE C2005 LOCAL GOVERNMENT REVIEWER - 59 -
appropriate units for planning local solid waste management The City and Municipal Solid Waste Boards shall have the
services in accordance with Section 41 hereof; following duties and responsibilities:
(6) Coordinate the efforts of the component cities and (1) Develop the City or Municipal Solid Waste
municipalities in the implementation of the Provincial Solid Management Plan that shall ensure the long-term management
Waste Management Plan; of solid waste, as well as integrate the various solid waste
(7) Develop an appropriate incentive scheme as an management plans and strategies of the barangays in its area of
integral component of the Provincial Solid Waste Management jurisdiction. In the development of the Solid Waste Management
Plan; Plan, it shall conduct consultations with the various sectors of
(8) Convene joint meetings of the provincial, city and the community;
municipal solid waste management boards at least every (2) Adopt measures to promote and ensure the viability
quarter for purposes of integrating, synchronizing, monitoring and effective implementation of solid waste management
and evaluating the development and implementation of its programs in its component barangays;
provincial solid waste management plan; (3) Monitor the implementation of the City or Municipal
(9) Represent any of its component city or municipality in Solid Waste Management Plan through its various political
coordinating its resource and operational requirements with subdivisions and in cooperation with the private sector and the
agencies of the national government; NGOs;
(10) Oversee the implementation of the Provincial Solid (4) Adopt specific revenue-generating measures to
Waste Management Plan; promote the viability of its Solid Waste Management Plan;
(11) Review every two (2) years or as the need arises the (5) Convene regular meetings for purposes of planning
Provincial Solid Waste Management Plan for purposes of and coordinating the implementation of the solid waste
ensuring its sustainability, viability, effectiveness and relevance management plans of the respective component barangays;
in relation to local and international developments in the field of (6) Oversee the implementation of the City or Municipal
solid waste management; and Solid Waste Management Plan; cAHIaE
(12) Allow for the clustering of LGUs for the solution of (7) Review every two (2) years or as the need arises the
common solid waste management problems. City or Municipal Solid Waste Management Plan for purposes of
ensuring its sustainability, viability, effectiveness and relevance
Sec12: City and Municipal Solid Waste Management Board. — in relation to local and international developments in the field of
Each city or municipality shall form a City or Municipal Waste solid waste management;
Management Board that shall prepare, submit and implement a (8) Develop the specific mechanics and guidelines for the
plan for the safe and sanitary management of solid waste implementation of the City or Municipal Solid Waste
generated in areas under its geographic and political coverage. Management Plan;
The City or Municipal Solid Waste Management Board shall be (9) Recommend to appropriate local government
composed of the city or municipal mayor as head with the authorities specific measures or proposals for franchise or build-
following as members: operate-transfer agreements with duly recognized institutions,
(a) One (1) representative of the Sangguniang pursuant to R.A. 6957, to provide either exclusive or non-
Panlungsod or the Sangguniang Bayan, preferably chairpersons exclusive authority for the collection, transfer, storage,
of either the Committees on Environment or Health, who will be processing, recycling or disposal of municipal solid waste. The
designated by the presiding officer; proposals shall take into consideration appropriate government
(b) President of the Association of Barangay Councils in rules and regulations on contracts, franchises and build-operate-
the municipality or city; transfer agreements;
(c) Chairperson of the Sangguniang Kabataan (10) Provide the necessary logistical and operational
Federation; support to its component cities and municipalities in consonance
(d) A representative from NGOs whose principal purpose with subsection (f) of Sec. 17 of the Local Government Code;
is to promote recycling and the protection of air and water (11) Recommend measures and safeguards against
quality; pollution and for the preservation of the natural ecosystem; and
(e) A representative from the recycling industry; (12) Coordinate the efforts of its component barangays in
(f) A representative from the manufacturing or packaging the implementation of the city or municipal Solid Waste
industry; and Management Plan.
(g) A representative of each concerned government
agency possessing relevant technical and marketing expertise Sec13: Establishment of Multi-Purpose Environment
as may be determined by the Board. Cooperatives or Associations in Every LGU. — Multi-purpose
The City or Municipal Solid Waste Management Board may, cooperatives and associations that shall undertake activities to
from time to time, call on any concerned agencies or sectors as promote the implementation and/or directly undertake projects in
it may deem necessary. compliance with the provisions of this Act shall be encouraged
Provided, That representatives from the NGOs, recycling and and promoted in every LGU.
manufacturing or packaging industries shall be selected through
a process designed by themselves and shall be endorsed by the Sec16: Local Government Solid Waste Management
government agency representatives of the Board. Plans. — The province, city or municipality, through its local

The Apocryphal Maggots:


Rainier, Chrisgel, Corina, Geoffry, Grace and Sylvie Blanche
The Flibbertigibbet Worms:
Golda, Gladys and Melyjane
CA VE AT: By simply r ea ding this re vi ew er a t the end o f the sem este r wi ll (hopef ully) guar antee you r pas sing this cou rse. D rink mode rat ely .
Amusin S C A r y
gly
THE C2005 LOCAL GOVERNMENT REVIEWER - 60 -
solid waste management boards, shall prepare its respective disposed of within that area. The constituent materials shall be
10-year solid waste management plans consistent with the identified by volume, percentage in weight or its volumetric
national solid waste management framework: Provided, That the equivalent, material type, and source of generation which
waste management plan shall be for the re-use, recycling and includes residential, commercial, industrial, governmental, or
composting of wastes generated in their respective jurisdictions: other sources. Future revisions of waste characterization studies
Provided, further, That the solid waste management plan of the shall identify the constituent materials which comprise the solid
LGU shall ensure the efficient management of solid waste waste disposed of at permitted disposal facilities.
generated within its jurisdiction. The plan shall place primary (c) Collection and Transfer — The plan shall take into
emphasis on implementation of all feasible re-use, recycling, account the geographic subdivisions to define the coverage of
and composting programs while identifying the amount of landfill the solid waste collection area in every barangay. The barangay
and transformation capacity that will be needed for solid waste shall be responsible for ensuring that a 100% collection
which cannot be re-used, recycled, or composted. The plan shall efficiency from residential, commercial, industrial and
contain all the components provided in Sec. 17 of this Act and a agricultural sources, where necessary within its area of
timetable for the implementation of the solid waste management coverage, is achieved. Toward this end, the plan shall define
program in accordance with the National Framework and and identify the specific strategies and activities to be
pursuant to the provisions of this Act: Provided, finally, That it undertaken by its component barangays, taking into account the
shall be reviewed and updated every year by the provincial, city following concerns:
or municipal solid waste management board. (1) Availability and provision of properly designed
For LGUs which have considered solid waste management containers or receptacles in selected collection points for the
alternatives to comply with Sec. 37 of this Act, but are unable to temporary storage of solid waste while awaiting collection and
utilize such alternatives, a timetable or schedule of compliance transfer to processing sites or to final disposal sites;
specifying the remedial measures and eventual compliance (2) Segregation of different types of solid waste for re-
shall be included in the plan. use, recycling and composting;
All local government solid waste management plans shall be (3) Hauling and transfer of solid waste from source or
subjected to the approval of the Commission. The plan shall be collection points to processing sites or final disposal sites;
consistent with the national framework and in accordance with (4) Issuance and enforcement of ordinances to effectively
the provisions of this Act and of the policies set by the implement a collection system in the barangay; and
Commission: Provided, That in the Province of Palawan, the (5) Provision of properly trained officers and workers to
local government solid waste management plan shall be handle solid waste disposal.
approved by the Palawan Council for Sustainable Development, The plan shall define and specify the methods and systems for the
pursuant to R.A. No. 7611. transfer of solid waste from specific collection points to solid
waste management facilities.
Sec17: The Components of the Local Government Solid Waste (d) Processing — The plan shall define the methods and
Management Plan. — The solid waste management plan shall the facilities required to process the solid waste, including the
include, but not be limited to, the following components: use of intermediate treatment facilities for composting, recycling,
(a) City or Municipal Profile — The plan shall indicate the conversion and other waste processing systems. Other
following background information on the city or municipality and appropriate waste processing technologies may also be
its component barangays, covering important highlights of the considered provided that such technologies conform with
distinct geographic and other conditions: internationally-acceptable and other standards set in other laws
(1) Estimated population of each barangay within the city and regulations.
or municipality and population projection for a 10-year period; (e) Source reduction — The source reduction component
(2) Illustration or map of the city/municipality, indicating shall include a program and implementation schedule which
locations of residential, commercial, and industrial centers, and shows the methods by which the LGU will, in combination with
agricultural area, as well as dump sites, landfills and other solid the recycling and composting components, reduce a sufficient
waste facilities. The illustration shall indicate as well, the amount of solid waste disposed of in accordance with the
proposed sites for disposal and other solid waste facilities; diversion requirements of Sec. 20.
(3) Estimated solid waste generation and projection by The source reduction component shall describe the following:
source, such as residential, market, commercial, industrial, (1) strategies in reducing the volume of solid waste
construction/demolition, street waste, agricultural, agro- generated at source;
industrial, institutional, other wastes; and (2) measures for implementing such strategies and the
(4) Inventory of existing waste disposal and other solid resources necessary to carry out such activities;
waste facilities and capacities. (3) other appropriate waste reduction technologies that
(b) Waste characterization — For the initial source may also be considered, provided that such technologies
reduction and recycling element of a local waste management conform with the standards set pursuant to this Act;
plan, the LGU waste characterization component shall identify (4) the types of wastes to be reduced pursuant to Sec. 15
the constituent materials which comprise the solid waste of this Act;
generated within the jurisdiction of the LGU. The information
shall be representative of the solid waste generated and

The Apocryphal Maggots:


Rainier, Chrisgel, Corina, Geoffry, Grace and Sylvie Blanche
The Flibbertigibbet Worms:
Golda, Gladys and Melyjane
CA VE AT: By simply r ea ding this re vi ew er a t the end o f the sem este r wi ll (hopef ully) guar antee you r pas sing this cou rse. D rink mode rat ely .
Amusin S C A r y
gly
THE C2005 LOCAL GOVERNMENT REVIEWER - 61 -
(5) the methods that the LGU will use to determine the indicate the specific measures to be undertaken to meet the
categories of solid wastes to be diverted from disposal at a waste diversion specified under Sec. 20 of this Act.
disposal facility through re-use, recycling and composting; and Recommended revisions to the building ordinances, requiring
(6) new facilities and of expansion of existing facilities newly-constructed buildings and buildings undergoing specified
which will be needed to implement re-use, recycling and alterations to contain storage space, devices or mechanisms
composting. that facilitate source separation and storage of designated
The LGU source reduction component shall include the recyclable materials to enable the local government unit to
evaluation and identification of rate structures and fees for the efficiently collect, process, market and sell the designated
purpose of reducing the amount of waste generated, and other materials. Such recommendations shall include, but shall not be
source reduction strategies, including but not limited to, limited to separate chutes to facilitate source separation in multi-
programs and economic incentives provided under Sec. 45 of family dwellings, storage areas that conform to fire and safety
this Act to reduce the use of non-recyclable materials, replace code regulations, and specialized storage containers.
disposable materials and products with reusable materials and The Solid Waste Management Plan shall indicate the specific
products, reduce packaging, and increase the efficiency of the measures to be undertaken to meet the recycling goals pursuant
use of paper, cardboard, glass, metal, and other materials. The to the objectives of this Act.
waste reduction activities of the community shall also take into (g) Composting — The composting component shall
account, among others, local capability, economic viability, include a program and implementation schedule which shows
technical requirements, social concerns, disposition of residual the methods by which the LGU shall, in combination with the
waste and environmental impact: Provided, That, projection of source reduction and recycling components, reduce a sufficient
future facilities needed and estimated cost shall be incorporated amount of solid waste disposed of within its jurisdiction to
in the plan. comply with the diversion requirements of Sec. 20 hereof.
(f) Recycling — The recycling component shall include a The LGU composting component shall describe the following:
program and implementation schedule which shows the (1) The types of materials which will be composted under
methods by which the LGU shall, in combination with the source the programs;
reduction and composting components, reduce a sufficient (2) The methods for determining the categories of solid
amount of solid waste disposed of in accordance with the wastes to be diverted from disposal at a disposal facility through
diversion requirements set in Sec. 20. composting; and
The LGU recycling component shall describe the following: (3) New facilities, and expansion of existing facilities
(1) The types of materials to be recycled under the needed to implement the composting component.
programs; The LGU composting component shall describe methods for
(2) The methods for determining the categories of solid developing the markets for composted materials, including, but
wastes to be diverted from disposal at a disposal facility through not limited to, an evaluation of the feasibility of procurement
recycling; and preferences for the purchase of composted products. Each LGU
(3) New facilities and expansion of existing facilities may determine and grant a price preference to encourage the
needed to implement the recycling component. purchase of composted products.
The LGU recycling component shall describe methods for (h) Solid waste facility capacity and final disposal — The
developing the markets for recycled materials, including, but not solid waste facility component shall include, but shall not be
limited to, an evaluation of the feasibility of procurement limited to, a projection of the amount of disposal capacity
preferences for the purchase of recycled products. Each LGU needed to accommodate the solid waste generated, reduced by
may determine and grant a price preference to encourage the the following:
purchase of recycled products. (1) Implementation of source reduction, recycling, and
The five-year strategy for collecting, processing, marketing and composting programs required in this Section or through
selling the designated recyclable materials shall take into implementation of other waste diversion activities pursuant to
account persons engaged in the business of recycling or Sec. 20 of this Act;
persons otherwise providing recycling services before the (2) Any permitted disposal facility which will be available
effectivity of this Act. Such strategy may be based upon the during the 10-year planning period; and
results of the waste composition analysis performed pursuant to (3) All disposal capacity which has been secured through
this Section or information obtained in the course of past an agreement with another LGU, or through an agreement with
collection of solid waste by the local government unit, and may a solid waste enterprise.
include recommendations with respect to increasing the number The plan shall identify existing and proposed disposal sites and
of materials designated for recycling pursuant to this Act. waste management facilities in the city or municipality or in other
The LGU recycling component shall evaluate industrial, areas. The plan shall specify the strategies for the efficient
commercial, residential, agricultural, governmental, and other disposal of waste through existing disposal facilities and the
curbside, mobile, drop-off, and buy-back recycling programs, identification of prospective sites for future use. The selection
manual and automated materials recovery facilities, zoning, and development of disposal sites shall be made on the basis of
building code changes and rate structures which encourage internationally accepted standards and on the guidelines set in
recycling of materials. The Solid Waste Management Plan shall Secs. 41 and 42 of this Act.

The Apocryphal Maggots:


Rainier, Chrisgel, Corina, Geoffry, Grace and Sylvie Blanche
The Flibbertigibbet Worms:
Golda, Gladys and Melyjane
CA VE AT: By simply r ea ding this re vi ew er a t the end o f the sem este r wi ll (hopef ully) guar antee you r pas sing this cou rse. D rink mode rat ely .
Amusin S C A r y
gly
THE C2005 LOCAL GOVERNMENT REVIEWER - 62 -
Strategies shall be included to improve said existing sites to offered as private sector investment activity shall be identified
reduce adverse impact on health and the environment, and to and promoted as such. Appropriate incentives for private sector
extend life span and capacity. The plan shall clearly define involvement in solid waste management shall likewise be
projections for future disposal site requirements and the established and provided for in the plan, in consonance with
estimated cost for these efforts. Sec. 45 hereof and other existing laws, policies and regulations;
Open dump sites shall not be allowed as final disposal sites. If and
an open dump site is existing within the city or municipality, the (m) Incentive programs — A program providing for
plan shall make provisions for its closure or eventual phase out incentives, cash or otherwise, which shall encourage the
within the period specified under the framework and pursuant to participation of concerned sectors shall likewise be included in
the provisions under Sec. 37 of this Act. As an alternative, the plan.
sanitary landfill sites shall be developed and operated as a final
disposal site for solid and, eventually, residual wastes of a Sec18: Owner and Operator. — Responsibility for compliance
municipality or city or a cluster of municipalities and/or cities. with the standards in this Act shall rest with the owner and/or
Sanitary landfills shall be designed and operated in accordance operator. If specifically designated, the operator is considered to
with the guidelines set under Secs. 40 and 41 of this Act. have primary responsibility for compliance; however, this does
(i) Education and public information — The education not relieve the owner of the duty to take all reasonable steps to
and public information component shall describe how the LGU assure compliance with these standards and any assigned
will educate and inform its citizens about the source reduction, conditions. When the title to a disposal is transferred to another
recycling, and composting programs. person, the new owner shall be notified by the previous owner of
The plan shall make provisions to ensure that information on the existence of these standards and of the conditions assigned
waste collection services, solid waste management and related to assure compliance.
health and environmental concerns are widely disseminated
among the public. This shall be undertaken through the print and Sec19: Waste Characterization. — The Department, in
broadcast media and other government agencies in the coordination with the LGUs, shall be responsible for the
municipality. The DECS and the Commission on Higher establishment of the guidelines for the accurate characterization
Education shall ensure that waste management shall be of wastes including determination of whether or not wastes will
incorporated in the curriculum of primary, secondary and college be compatible with containment features and other wastes, and
students. whether or not wastes are required to be managed as
(j) Special waste — The special waste component shall hazardous wastes under R.A. 6969, otherwise known as the
include existing waste handling and disposal practices for Toxic Substances and Hazardous and Nuclear Wastes Control
special wastes or household hazardous wastes, and the Act.
identification of current and proposed programs to ensure the
proper handling, re-use, and long-term disposal of special Sec20: Establishing Mandatory Solid Waste Diversion. — Each
wastes. LGU plan shall include an implementation schedule which
(k) Resource requirement and funding — The funding shows that within five (5) years after the effectivity of this Act,
component includes identification and description of project the LGU shall divert at least 25% of all solid waste from waste
costs, revenues, and revenue sources the LGU will use to disposal facilities through re-use, recycling, and composting
implement all components of the LGU solid waste management activities and other resource recovery activities: Provided, That
plan. the waste diversion goals shall be increased every three (3)
The plan shall likewise indicate specific projects, activities, years thereafter: Provided, further, That nothing in this Section
equipment and technological requirements for which outside prohibits a local government unit from implementing re-use,
sourcing of funds or materials may be necessary to carry out the recycling, and composting activities designed to exceed the
specific components of the plan. It shall define the specific uses goal.
for its resource requirements and indicate its costs. The plan
shall likewise indicate how the province, city or municipality ARTICLE 2 Segregation of Wastes
intends to generate the funds for the acquisition of its resource Sec21: Mandatory Segregation of Solid Wastes. — The LGUs
requirements. It shall also indicate if certain resource shall evaluate alternative roles for the public and private sectors
requirements are being or will be sourced from fees, grants, in providing collection services, type of collection system, or
donations, local funding and other means. This will serve as combination of systems, that best meet their needs: Provided,
basis for the determination and assessment of incentives which That segregation of wastes shall primarily be conducted at the
may be extended to the province, city or municipality as source, to include household, institutional, industrial, commercial
provided for in Sec. 45 of this Act. and agricultural sources: Provided, further, That wastes shall be
(l) Privatization of solid waste management projects — segregated into the categories provided in Sec. 22 of this Act.
The plan shall likewise indicate specific measures to promote For premises containing six (6) or more residential units, the
the participation of the private sector in the management of solid local government unit shall promulgate regulations requiring the
wastes, particularly in the generation and development of the owner or person in charge of such premises to:
essential technologies for solid waste management. Specific (a) provide for the residents a designated area and
projects or component activities of the plan which may be containers in which to accumulate source separated recyclable

The Apocryphal Maggots:


Rainier, Chrisgel, Corina, Geoffry, Grace and Sylvie Blanche
The Flibbertigibbet Worms:
Golda, Gladys and Melyjane
CA VE AT: By simply r ea ding this re vi ew er a t the end o f the sem este r wi ll (hopef ully) guar antee you r pas sing this cou rse. D rink mode rat ely .
Amusin S C A r y
gly
THE C2005 LOCAL GOVERNMENT REVIEWER - 63 -
materials to be collected by the municipality or private center; including NGOs, to develop or undertake an effective solid
and waste management, or actively participate in any program
(b) notify the occupants of such buildings of the geared towards the promotion thereof as provided for in this Act.
requirements of this Act and the regulations promulgated (1) Fiscal Incentives — Consistent with the provisions of
pursuant thereto. E.O. 226, otherwise known as the Omnibus Investments Code,
the following tax incentives shall be granted:
Sec22: Requirements for the Segregation and Storage of Solid (a) Tax and Duty Exemption on Imported Capital
Waste. — The following shall be the minimum standards and Equipment and Vehicles — Within ten (10) years upon effectivity
requirements for segregation and storage of solid waste pending of this Act, LGUs, enterprises or private entities shall enjoy tax
collection: and duty-free importation of machinery, equipment, vehicles and
(a) There shall be a separate container for each type of spare parts used for collection, transportation, segregation,
waste from all sources: Provided, That in the case of bulky recycling, re-use and composting of solid wastes: Provided, That
waste, it will suffice that the same be collected and placed in a the importation of such machinery, equipment, vehicle and spare
separate and designated area; and parts shall comply with the following conditions:
(b) The solid waste container depending on its use shall (i) They are not manufactured domestically in sufficient
be properly marked or identified for on-site collection as quantity, of comparable quality and at reasonable prices;
"compostable", "non-recyclable", "recyclable" or "special waste", (ii) They are reasonably needed and will be used actually,
or any other classification as may be determined by the directly and exclusively for the above mentioned activities;
Commission. (iii) The approval of the Board of Investment (BOI) of the
DTI for the importation of such machinery, equipment, vehicle
Sec43: Guidelines for Identification of Common Solid Waste and spare parts.
Management Problems. — For purposes of encouraging and Provided, further, That the sale, transfer or disposition of such
facilitating the development of local government plans for solid machinery, equipment, vehicle and spare parts, without prior
waste management, the Commission shall, as soon as approval of the BOI, within five (5) years from the date of
practicable but not later than six (6) months from the effectivity acquisition shall be prohibited, otherwise, the LGU concerned,
of this Act, publish guidelines for the identification of those areas enterprises or private entities and the vendee, transferee or
which have common solid waste management problems and are assignee shall be solidarily liable to pay twice the amount of tax
appropriate units for clustered solid waste management and duty exemption given it.
services. The guidelines shall be based on the following: (b) Tax Credit on Domestic Capital Equipment — Within
(a) the size and location of areas which should be ten (10) years from the effectivity of this Act, a tax credit
included; equivalent to 50% of the value of the national internal revenue
(b) the volume of solid waste which would be generated; taxes and customs duties that would have been waived on the
(c) the available means of coordinating local government machinery, equipment, vehicle and spare parts, had these items
planning between and among the LGUs and for the integration been imported shall be given to enterprises, private entities,
of such with the national plan; and including NGOs, subject to the same conditions and prohibition
(d) possible lifespan of the disposal facilities. cited in the preceding paragraph.
(c) Tax and Duty Exemption of Donations, Legacies and
Sec44: Establishment of Common Waste Treatment and Gift — All legacies, gifts and donations to LGUs, enterprises or
Disposal Facilities. — Pursuant to Sec. 33 of R.A. 7160, private entities, including NGOs, for the support and
otherwise known as the Local Government Code, all provinces, maintenance of the program for effective solid waste
cities, municipalities and barangays, through appropriate management shall be exempt from all internal revenue taxes
ordinances, are hereby mandated to consolidate, or coordinate and customs duties, and shall be deductible in full from the
their efforts, services, and resources for purposes of jointly gross income of the donor for income tax purposes.
addressing common solid waste management problems and/or (2) Non-Fiscal Incentives — LGUS, enterprises or private
establishing common waste disposal facilities. entities availing of tax incentives under this Act shall also be
The Department, the Commission and local solid waste entitled to applicable non-fiscal incentives provided for under
management boards shall provide technical and marketing E.O. 226, otherwise known as the Omnibus Investments Code.
assistance to the LGUs. The Commission shall provide incentives to businesses and
industries that are engaged in the recycling of wastes and which
CHAPTER IV Incentives are registered with the Commission and have been issued
Sec45: Incentives. — (a) Rewards, monetary or otherwise, shall ECCs in accordance with the guidelines established by the
be provided to individuals, private organizations and entities, Commission. Such incentives shall include simplified procedures
including nongovernment organizations, that have undertaken for the importation of equipment, spare parts, new materials,
outstanding and innovative projects, technologies, processes and supplies, and for the export of processed products.
and techniques or activities in re-use, recycling and reduction. (3) Financial Assistance Program — Government
Said rewards shall be sourced from the Fund herein created. financial institutions such as the Development Bank of the
(b) An incentive scheme is hereby provided for the Philippines (DBP), Landbank of the Philippines (LBP),
purpose of encouraging LGUs, enterprises, or private entities, Government Service Insurance System (GSIS), and such other

The Apocryphal Maggots:


Rainier, Chrisgel, Corina, Geoffry, Grace and Sylvie Blanche
The Flibbertigibbet Worms:
Golda, Gladys and Melyjane
CA VE AT: By simply r ea ding this re vi ew er a t the end o f the sem este r wi ll (hopef ully) guar antee you r pas sing this cou rse. D rink mode rat ely .
Amusin S C A r y
gly
THE C2005 LOCAL GOVERNMENT REVIEWER - 64 -
government institutions providing financial services shall, in (c) distance of the transfer station to the waste
accordance with and to the extent allowed by the enabling management facility.
provisions of their respective charters or applicable laws, accord The fees shall be used to pay the actual costs incurred by the
high priority to extend financial services to individuals, LGU in collecting the local fees. In determining the amounts of
enterprises, or private entities engaged in solid waste the fees, an LGU shall include only those costs directly related
management. to the adoption and implementation of the plan and the setting
(4) Extension of Grants to LGUs — Provinces, cities and and collection of the local fees.
municipalities whose solid waste management plans have been
duly approved by the Commission or who have been
commended by the Commission for adopting innovative solid RA8550: An Act Providing For The Development,
waste management programs may be entitled to receive grants Management And Conservation Of The Fisheries And
for the purpose of developing their technical capacities toward Aquatic Resources, Integrating All Laws Pertinent Thereto,
actively participating in the program for effective and sustainable And For Other Purposes
solid waste management.
(5) Incentives to Host LGUs — Local government units Sec16: Jurisdiction of Municipal/City Government. — The
who host common waste management facilities shall be entitled municipal/city government shall have jurisdiction over municipal
to incentives. waters as defined in this Code. The municipal/city government,
in consultation with the FARMC shall be responsible for the
CHAPTER V Financing Solid Waste Management management, conservation, development, protection, utilization,
Sec46: Solid Waste Management Fund. — There is hereby and disposition of all fish and fishery/aquatic resources within
created, as a special account in the National Treasury, a Solid their respective municipal waters.
Waste Management Fund to be administered by the The municipal/city government may, in consultation with the
Commission. Such fund shall be sourced from the following: FARMC, enact appropriate ordinances for this purpose and in
(a) Fines and penalties imposed, proceeds of permits and accordance with the National Fisheries Policy. The ordinances
licenses issued by the Department under this Act, donations, enacted by the municipality and component city shall be
endowments, grants and contributions from domestic and reviewed pursuant to Republic Act No. 7160 by the sanggunian
foreign sources; and of the province which has jurisdiction over the same.
(b) Amounts specifically appropriated for the Fund under The LGUs shall also enforce all fishery laws, rules and
the annual General Appropriations Act. regulations as well as valid fishery ordinances enacted by the
The Fund shall be used to finance the following: municipal/city council.
(1) products, facilities, technologies and processes to The management of contiguous fishery resources such as bays
enhance proper solid waste management; which straddle several municipalities, cities or provinces, shall
(2) awards and incentives; be done in an integrated manner, and shall not be based on
(3) research programs; political subdivisions of municipal waters in order to facilitate
(4) information, education, communication and monitoring their management as single resource systems. The LGUs which
activities; share or border such resources may group themselves and
(5) technical assistance; and coordinate with each other to achieve the objectives of
(6) capability building activities. integrated fishery resource management. The Integrated
LGUs are entitled to avail of the Fund on the basis of their Fisheries and Aquatic Resources Management Councils
approved solid waste management plan. Specific criteria for the (FARMCs) established under Section 76 of this Code shall serve
availment of the Fund shall be prepared by the Commission. as the venues for close collaboration among LGUs in the
The fines collected under Sec. 49 shall be allocated to the LGU management of contiguous resources.
where the fined prohibited acts are committed in order to finance
the solid waste management of said LGU. Such allocation shall Sec17: Grant of Fishing Privileges in Municipal Waters. — The
be based on a sharing scheme between the Fund and the LGU duly registered fisherfolk organizations/cooperatives shall have
concerned. preference in the grant of fishery rights by the Municipal/City
In no case, however, shall the Fund be used for the creation of Council pursuant to Section 149 of the Local Government Code:
positions or payment of salaries and wages. Provided, That in areas where there are special agencies or
offices vested with jurisdiction over municipal waters by virtue of
Sec47: Authority to Collect Solid Waste Management Fees. — special laws creating these agencies such as, but not limited to,
The local government unit shall impose fees in amounts the Laguna Lake Development Authority and the Palawan
sufficient to pay the costs of preparing, adopting, and Council for Sustainable Development, said offices and agencies
implementing a solid waste management plan prepared shall continue to grant permits for proper management and
pursuant to this Act. The fees shall be based on the following implementation of the aforementioned structures.
minimum factors:
(a) types of solid waste; Sec18: Users of Municipal Waters. — All fishery related activities
(b) amount/volume of waste; and in municipal waters, as defined in this Code, shall be utilized by

The Apocryphal Maggots:


Rainier, Chrisgel, Corina, Geoffry, Grace and Sylvie Blanche
The Flibbertigibbet Worms:
Golda, Gladys and Melyjane
CA VE AT: By simply r ea ding this re vi ew er a t the end o f the sem este r wi ll (hopef ully) guar antee you r pas sing this cou rse. D rink mode rat ely .
Amusin S C A r y
gly
THE C2005 LOCAL GOVERNMENT REVIEWER - 65 -
municipal fisherfolk and their cooperatives/organizations who organizations/cooperatives shall have priority to exploit
are listed as such in the registry of municipal fisherfolk. municipal and demarcated fishery areas of the said municipality.
The municipal or city government, however, may, through its
local chief executive and acting pursuant to an appropriate Sec22: Demarcated Fishery Right. — The LGU concerned shall
ordinance, authorize or permit small and medium commercial grant demarcated fishery rights to fishery
fishing vessels to operate within the ten point one (10.1) to organizations/cooperatives for mariculture operation in specific
fifteen (15) kilometer area from the shoreline in municipal waters areas identified by the Department.
as defined herein, provided, that all the following are met:
a. no commercial fishing in municipal waters with depth Sec23: Limited Entry Into Overfished Areas. — Whenever it is
less than seven (7) fathoms as certified by the appropriate determined by the LGUs and the Department that a municipal
agency; water is overfished based on available data or information or in
b. fishing activities utilizing methods and gears that are danger of being overfished, and that there is a need to
determined to be consistent with national policies set by the regenerate the fishery resources in that water, the LGU shall
Department; prohibit or limit fishery activities in the said waters.
c. prior consultation, through public hearing, with the
M/CFARMC has been conducted; and Sec24: Support to Municipal Fisherfolk. — The Department and
d. the applicant vessel as well as the shipowner, the LGUs shall provide support to municipal fisherfolk through
employer, captain and crew have been certified by the appropriate technology and research, credit, production and
appropriate agency as not having violated this Code, marketing assistance and other services such as, but not limited
environmental laws and related laws. to training for additional/supplementary livelihood.
In no case shall the authorization or permit mentioned above be
granted for fishing in bays as determined by the Department to Sec25: Rights and Privileges of Fishworkers. — The
be in an environmentally critical condition and during closed fishworkers shall be entitled to the privileges accorded to other
season as provided for in Section 9 of this Code. workers under the Labor Code, Social Security System and
other benefits under other laws or social legislation for workers:
Sec19: Registry of Municipal Fisherfolk. — The LGU shall Provided, That fishworkers on board any fishing vessels
maintain a registry of municipal fisherfolk, who are fishing or engaged in fishing operations are hereby covered by the
may desire to fish in municipal waters for the purpose of Philippine Labor Code, as amended.
determining priorities among them, of limiting entry into the
municipal waters, and of monitoring fishing activities an/or other
related purposes: Provided, That the FARMC shall submit to the STD Prevention and Treatment: General Comment (Part 2)
LGU the list of priorities for its consideration.
Such list or registry shall be updated annually or as may be It is a myth that one may acquire "immunity" to STDs. Two
necessary, and shall be posted in barangay halls or other other myths are that one cannot have more than one STD at
strategic locations where it shall be open to public inspection, for
once (you can have many at once), and that one needs to have
the purpose of validating the correctness and completeness of
the list. The LGU, in consultation with the FARMCs, shall multiple partners to acquire an STD (one is sufficient).
formulate the necessary mechanisms for inclusion or exclusion HIV and hepatitis may be spread through sharing drug or
procedures that shall be most beneficial to the resident steroid needles with someone who is infected. In many cities,
municipal fisherfolk. The FARMCs may likewise recommend such as Seattle, one may go to a needle exchange program for
such mechanisms. clean needles. However, the best approach to the needle
The LGUs shall also maintain a registry of municipal fishing problem if you're an injecting drug or steroid user is to obtain a
vessels by type of gear and other boat particulars with the supply of sterile syringes and needles for yourself, and NEVER
assistance of the FARMC. use a rig that has been used by somebody else. It is true that you
can clean somebody else's works, but if you need a fix you may
Sec20: Fisherfolk Organizations and/or Cooperatives. — not be willing to sit through the sterilization procedure, which
Fisherfolk organizations/cooperatives whose members are listed takes time.
in the registry of municipal fisherfolk, may be granted use of
However, if using clean equipment is not an option, the
demarcated fishery areas to engage in fish capture, mariculture
and/or fish farming: Provided, however, That an following procedure may be used. Pour 100% bleach (i.e.
organization/cooperative member whose household is already in Clorox) into a container and repeatedly (3-4 times) draw the
possession of a fishery right other than for fish capture cannot bleach into the needle up to the top of the plunger, keep it there
enjoy the fishing rights granted to the organization or for a few minutes, and then eject it down a drain (you can
cooperative. alternatively just take your rig apart and soak it in 100% bleach
for about 10 minutes). After this step, rinse everything in cold
Sec21: Priority of Resident Municipal Fisherfolk. — Resident running water fora few minutes. If a cooker is used, wipe it with
municipal fisherfolk of the municipality concerned and their a bleach-soaked cotton ball, and then rinse the cooker with cold
water and discard the cotton. Never share any of the water,

The Apocryphal Maggots:


Rainier, Chrisgel, Corina, Geoffry, Grace and Sylvie Blanche
The Flibbertigibbet Worms:
Golda, Gladys and Melyjane
CA VE AT: By simply r ea ding this re vi ew er a t the end o f the sem este r wi ll (hopef ully) guar antee you r pas sing this cou rse. D rink mode rat ely .
Amusin S C A r y
gly
THE C2005 LOCAL GOVERNMENT REVIEWER - 66 -
bleach, or cotton once it has been used during this procedure implementation of community-based forestry
with someone else. (end) projects which include integrated social
forestry programs and similar projects;
management and control of communal forests
Basic Services and Facilities with an area not exceeding fifty (50) square
kilometers; establishment of tree parks,
Sec17: Basic Services and Facilities. – greenbelts, and similar forest development
(c) Local government units shall endeavor to be self-reliant projects;
and shall continue exercising the powers and discharging (iii). Subject to the provisions of Title Five, Book I
the duties and functions currently vested upon them. They of this Code, health services which include the
shall also discharge the functions and responsibilities of implementation of programs and projects on
national agencies and offices devolved to them pursuant to primary health care, maternal and child care,
this Code. Local government units shall likewise exercise and communicable and non-communicable
such other powers and discharge such other functions and disease control services; access to secondary
responsibilities as are necessary, appropriate, or incidental and tertiary health services; purchase of
to efficient and effective provision of the basic services and medicines, medical supplies, and equipment
facilities enumerated herein. needed to carry out the services herein
(d) Such basic services and facilities include, but are not enumerated;
limited to, the following: (iv). Social welfare services which include
(1) For a Barangay: programs and projects on child and youth
(i) Agricultural support services which include welfare, family and community welfare,
planting materials distribution system and women's welfare, welfare of the elderly and
operation of farm produce collection and disabled persons; community-based
buying stations; rehabilitation programs for vagrants, beggars,
(ii) Health and social welfare services which street children, scavengers, juvenile
include maintenance of barangay health delinquents, and victims of drug abuse;
center and day-care center; livelihood and other pro-poor
(iii) Services and facilities related to general projects; nutrition services; and family
hygiene and sanitation, beautification, and planning services;
solid waste collection; (v). Information services which include
(iv) Maintenance of katarungang pambarangay; investments and job placement information
(v) Maintenance of barangay roads and bridges systems, tax and marketing information
and water supply systems systems, and maintenance of a public library;
(vi) Infrastructure facilities such as multi- (vi). Solid waste disposal system or environmental
purpose hall, multipurpose pavement, management system and services or facilities
plaza, sports center, and other similar related to general hygiene and sanitation;
facilities; (vii). Municipal buildings, cultural centers, public
(vii) Information and reading center; and parks including freedom parks, playgrounds,
(viii) Satellite or public market, where viable; and sports facilities and equipment, and other
(2) For a municipality: similar
(i). Extension and on-site research services and facilities;
facilities related to agriculture and fishery (viii). Infrastructure facilities intended primarily to
activities which include dispersal of livestock service the needs of the residents of the
and poultry, fingerlings, and other seeding municipality and which are funded out of
materials for aquaculture; palay, corn, and municipal funds including, but not limited to,
vegetable seed farms; medicinal plant municipal roads and bridges; school buildings
gardens; fruit tree, coconut, and other kinds of and other facilities for public elementary and
seedling nurseries; demonstration farms; secondary schools; clinics, health centers and
quality control of copra and improvement and other health facilities necessary to carry out
development of local distribution channels, health services; communal irrigation, small
preferably through cooperatives; water impounding projects and other similar
interbarangay irrigation system; water and soil projects; fish ports; artesian wells, spring
resource utilization and conservation projects; development, rainwater collectors and water
and enforcement of fishery laws in municipal supply systems; seawalls, dikes, drainage and
waters including the conservation of sewerage, and flood control; traffic signals and
mangroves; road signs; and similar facilities;
(ii). Pursuant to national policies and subject to (ix). Public markets, slaughterhouses and other
supervision, control and review of the DENR, municipal enterprises;

The Apocryphal Maggots:


Rainier, Chrisgel, Corina, Geoffry, Grace and Sylvie Blanche
The Flibbertigibbet Worms:
Golda, Gladys and Melyjane
CA VE AT: By simply r ea ding this re vi ew er a t the end o f the sem este r wi ll (hopef ully) guar antee you r pas sing this cou rse. D rink mode rat ely .
Amusin S C A r y
gly
THE C2005 LOCAL GOVERNMENT REVIEWER - 67 -
(x). Public cemetery; (c) Notwithstanding the provisions of subsection (b) hereof,
(xi). Tourism facilities and other tourist attractions, public works and infrastructure projects and other facilities
including the acquisition of equipment, funded by the national government under the annual
regulation and supervision of business General Appropriations Act, other special laws, pertinent
concessions, and security services for such executive orders, and those wholly or partially funded from
facilities; and foreign sources, are not covered under this Section, except
(xii). Sites for police and fire stations and in those cases where the local government unit concerned
substations and the municipal jail; is duly designated as the implementing agency for such
(3) For a Province: projects, facilities, programs, and services.
A (d) The designs, plans, specifications, testing of materials, and
and on-site research services and facilities which include the prevention and the procurement of equipment and materials from both
animal pests and diseases; dairy farms, livestock markets, animal breeding foreign and local sources
al insemination centers; and assistance in the organization of farmers' and necessary for the provision of the foregoing services and
tives and other collective organizations, as well as the transfer of appropriate facilities shall be undertaken by the local government unit
concerned, based on national
policies, standards and guidelines.
nd development services, as well as the transfer of appropriate technology; (e) National agencies or offices concerned shall devolve to
P local government units the responsibility for the provision of
policies and subject to supervision, control and review of the DENR, enforcement basic services and facilities enumerated in this Section
ed to community-based forestry projects, pollution control law, small-scale mining within six (6) months after the effectivity of this Code.
on the protection of the environment; and mini-hydro electric projects for local As used in this Code, the term "devolution" refers to the act
by which the national government confers power and
S authority upon the various local government units to
ons of Title Five, Book I of this Code, health services which include hospitals and perform specific functions and responsibilities.
services; (f) The national government or the next higher level of local
S government unit may provide or augment the basic
s which include pro grams and projects on rebel returnees and evacuees; relief services and facilities assigned to a lower level of local
ulation development services; government unit when such services or facilities are not
P made available or, if made available, are inadequate to
rovincial jails, freedom parks and other public assembly areas, and other similar meet the requirements of its inhabitants.
(g) The basic services and facilities hereinabove enumerated
I shall be funded from the share of local government units in
s intended to service the needs of the residents of the province and which are the proceeds of national
ncial funds including, but not limited to, provincial roads and bridges; inter- taxes and other local revenues and funding support from
ks, drainage and sewerage, flood control, and irrigation systems; reclamation the national government, its instrumentalities and
facilities; government-owned or -controlled corporations which are
P tasked by law to establish and maintain such services or
ts for low-cost housing and other mass dwellings, except those funded by the facilities. Any fund or resource available for the use of local
stem (SSS), Government Service Insurance System (GSIS), and the Home government units shall be first allocated for the provision of
l Fund (HDMF): Provided, That national funds for these programs and projects basic services or facilities enumerated in subsection (b)
allocated among the regions in proportion to the ratio of the homeless to the hereof before applying the same for other purposes, unless
otherwise provided in this Code.
I (h) The Regional offices of national agencies or offices whose
ervices, including access to credit financing; functions are devolved to local government units as
U provided herein shall be phased out within one (1) year
rnization of tax information and collection services through the use of computer from the approval of this Code. Said national agencies and
are and other means; offices may establish such field units as may be necessary
I for monitoring purposes and providing technical assistance
ommunications services, subject to national policy guidelines; and to local government units. The properties, equipment, and
T other assets of these regional offices shall be distributed to
and promotion programs; the local government units in the region in accordance with
(4) For a City: the rules and regulations issued by the oversight
All the services and facilities of the municipality and committee created under this Code.
province, and in addition thereto, the following: (i) The devolution contemplated in this Code shall include the
(i). Adequate communication and transportation transfer to local government units of the records,
facilities; equipment, and other assets and personnel of national
(ii). Support for education, police and fire agencies and offices corresponding to the devolved
services and facilities. powers, functions, and responsibilities.

The Apocryphal Maggots:


Rainier, Chrisgel, Corina, Geoffry, Grace and Sylvie Blanche
The Flibbertigibbet Worms:
Golda, Gladys and Melyjane
CA VE AT: By simply r ea ding this re vi ew er a t the end o f the sem este r wi ll (hopef ully) guar antee you r pas sing this cou rse. D rink mode rat ely .
Amusin S C A r y
gly
THE C2005 LOCAL GOVERNMENT REVIEWER - 68 -
Personnel of said national agencies or offices shall be (c) Any subsequent change in national policies,
absorbed by the local government units to which they guidelines, and standards shall be subject to prior consultation
belong or in whose areas they are assigned to the extent with LGUs.
that it is administratively viable as determined by the said
oversight committee: Provided, That the rights accorded to
such personnel pursuant to civil service law, rules and Tano vs. Socrates (1997) - supra
regulations shall not be impaired: Provided, Further, That
regional directors who are career executive service officers Issue: WON the ordinances enacted are valid.
and other officers of similar rank in the said regional offices Held: Yes
who cannot be absorbed by the local government unit shall Ratio:
be retained by the national government, without any The ordinances in question are meant precisely to protect and
diminution of rank, salary or tenure. conserve our marine resources to the end that their enjoyment
(j) To ensure the active participation of the private sector in may be guaranteed not only for the present generation, but also for
local governance, local government units may, by the generations to come.
ordinance, sell, lease, encumber, or otherwise dispose of
public economic enterprises owned by them in their State policy also enshrined in the Constitution is the duty of the
proprietary capacity. State to protect and advance the right of the people to a balanced
Costs may also be charged for the delivery of basic and healthful ecology in accord with the rhythm and harmony of
services or facilities enumerated in this Section. nature.

The LGCode provision on the General Welfare Clause 1 gives flesh


Pimentel, p.15 and blood to the right of the people to a balanced and healthful
ecology.
Devolution of Central Govt. powers and funds are transferred
directly to the provinces, cities, municipalities and barangays Moreover, Section 5(c) of the LGC explicitly mandates that the
without any intermediary. The code also provides for effective general welfare provisions of the LGC "shall be liberally
allocation among the different LGUs of their respective powers, interpreted to give more powers to the local government units
functions responsibilities and resources, meaning that they will not in accelerating economic development and upgrading the
stop at any one level, but deep down to all levels of LGUs. Thus quality of life for the people of the community."
barangays exercise their powers without the need of municipal
approval, nor do they get funds from the municipality but by direct One of the devolved powers enumerated in the section of the
allocation. LGCode on devolution is the enforcement of fishery laws in
municipal waters including the conservation of mangroves. This
4 general powers exercised by the LGUs: necessarily includes the enactment of ordinances to effectively
1) those expressly granted by the code carry out such fishery laws within the municipal waters.
2) those implied from those expressly granted
3) those necessary, appropriate or incidental for their efficient and Both ordinances have two principal objectives:
effective governance
4) those that are essential to the promotion of the genera welfare (a) to establish a "closed season" for the species of fish or
of their inhabitants. aquatic animals covered therein for a period of five years
(b) to protect the coral in the marine waters of Puerto Princesa
and Palawan from further destruction due to illegal fishing
Local Government Code, IRR activities.

Article24: Devolution. — 1
(a) Consistent with local autonomy and decentralization, the SEC. 16. General Welfare. — Every local government unit shall exercise the
provision for the delivery of basic services and facilities shall be powers expressly granted, those necessarily implied therefrom, as well as powers
devolved from the National Government to provinces, cities, necessary, appropriate, or incidental for its efficient and effective governance, and
municipalities, and barangays so that each LGU shall be those which are essential to the promotion of the general welfare. Within their
responsible for a minimum set of services and facilities in respective territorial jurisdictions, local government units shall ensure and
accordance with established national policies, guidelines, and support, among other things, the preservation and enrichment of culture, promote
standards. health and safety, enhance the right of the people to a balanced ecology,
(b) For purposes of this Rule, devolution shall mean the encourage and support the development of appropriate and self-reliant scientific
transfer of power and authority from the National Government to and technological capabilities, improve public morals, enhance economic
LGUs to enable them to perform specific functions and prosperity and social justice, promote full employment among their residents,
responsibilities. maintain peace and order, and preserve the comfort and convenience of their
inhabitants.

The Apocryphal Maggots:


Rainier, Chrisgel, Corina, Geoffry, Grace and Sylvie Blanche
The Flibbertigibbet Worms:
Golda, Gladys and Melyjane
CA VE AT: By simply r ea ding this re vi ew er a t the end o f the sem este r wi ll (hopef ully) guar antee you r pas sing this cou rse. D rink mode rat ely .
Amusin S C A r y
gly
THE C2005 LOCAL GOVERNMENT REVIEWER - 69 -
The accomplishment of the first objective is well within the to erect fish corrals, oyster, mussels or aquatic beds or bangus fry
devolved power to enforce fishery laws in municipal waters, such area w/in a definite zone of the municipal waters.
as P.D. No. 1015, which allows the establishment of "closed Thus, RA 7160 DID NOT REPEAL the aforementioned laws and
seasons." The devolution of such power has been expressly DOES NOT PREVAIL over them because:
confirmed in a MOA bet DA and DILG. 1) The repeal of laws must be clear and expressed;
The realization of the second objective clearly falls within both the 2) LGC is a general law while LLDA Charter is a special law. The
general welfare clause of the LGCode and the express mandate to enactment of a later legislation w/c is a general law cant be
cities and provinces to protect the environment and impose construed to have repealed a special law unless the intent to
appropriate penalties for acts which endanger the environment. repeal is manifest.
3) Where there is a conflict bet. a general law and a special law,
The nexus between the activities barred by the ordinances and the
the latter prevails since it shows the legislative intent more
prohibited acts, on the one hand, and the use of sodium cyanide,
clearly than the latter. Implied repeals are not favored.
on the other, is painfully obvious. Thus, the public purpose and
Considering the reasons behind the establishment of the
reasonableness of the Ordinances may not then be controverted.
LLDA, there is every indication that the legislative intent is for
Petition dismissed. TRO lifted.
LLDA to proceed with its mission. [ then court says it agrees
with LLDA’s manifestation ]
Laguna Lake Dev’t Authority vs. CA (1995)
4) The power granted to the LGUs by the LGC was clearly for
revenue purposes, since it (Sec 149) is embodied in Ch 2 Bk
2 under the heading “Specific Provisions On the Taxing And
Facts: Other Revenue Raising Power of LGUs”. While te power
RA 4850 created LLDA for the purpose of environmental protection granted to the LLDA IS FOR THE PURPOSE OF
and ecology, navigational safety and sustainable development to EFFECTIVELY REGULATING & MONITORING activities in
accelerate the dev’t and balanced growth of the Laguna Lake area the Laguna Lake Region and for lake quality control and
w/in the context of nat’l and regional plans and policies for social management. It does partake of the nature of police power.
and econ dev’t.
5) Removal of such power will render nugatory its avowed
PD 813, EO 927 amended said RA which defined and enlarged the purpose.
functions and powers of LLDA and named and enumerated the
towns, cities and provinces encompassed by the term “Laguna de DGNotes:
Bay Region.” Is CAB good law?
YES. But now based on the Fisheries Act which expressly provides
When LGC 1991 was passed, the municipalities in the Laguna for the same ratio as CAB
Lake Region interpreted Secs. 149(b)[1-2] and 447 (XI) to mean
that it gave them the exclusive jurisdiction to issue fishing LTO vs. City of Butuan (2000)
privileges w/in their municipal waters.
Facts:
Big fishpen operators took advantage thereby aggravating the Because of Art X Sec 5 of Const and Secs 129 and 133 of RA
current environmental problems and ecological stress of Laguna 7160, the Sangguniang Panglungsod (SP) of Butuan passed an
Lake. ordinance regulating the operation of tricycles for hire w/c provided
for the payment of franchise fees for the grant of the franchise of
LLDA served notice to the general public that all fishpens and other tricycles, for the registration of the vehicle and permit to drive.
aqua-culture structures not unregistered or has not applied for
registration with the LLDA as illegal and be subject to demolition LTO explains that the LTFRB’s power over franchising of the
without prejudice from being criminally charged. tricycles had been transferred but not the LTO’s authority to
register all motor vehicles and to issue driver’s license.
Issue: WON the LLDA could exercise jurisdiction over the
Laguna Lake and its environs insofar as the issuance of Issue: WON LTO’s power to issue driver’s license has been
fishpen permits inspite of the passage of RA 7160 devolved to LGUs
Held: Yes, yes, show
Sec 4(k) of RA 4850, provisions of PD 813 and EO 927 Sec. 2 Held: NO
specifically provide that the LLDA shall have exclusive jurisdiction The DOTC through the LTO and LTFRB has been tasked with
to issue permits for the use or all surface water for any projects or implementing laws pertaining to land transpo.
activities in or affecting the said region, including navigation,
construction and operation of fishpens, fish enclosures and the In fine. registration and licensing functions are vested with the LTO
like. On the other hand, RA 7160 has granted to the municipalities while the franchising and regulatory responsibilities had been
the exclusive authority to grant fishery privileges in municipal vested in the LTFRB. Under the LGC (Sec 458[3] {VI}) , LGUs now
waters. The Sangguniang Bayan (SB) may grant fishery privileges have the power to regulate the operation of tricycles for hire and to

The Apocryphal Maggots:


Rainier, Chrisgel, Corina, Geoffry, Grace and Sylvie Blanche
The Flibbertigibbet Worms:
Golda, Gladys and Melyjane
CA VE AT: By simply r ea ding this re vi ew er a t the end o f the sem este r wi ll (hopef ully) guar antee you r pas sing this cou rse. D rink mode rat ely .
Amusin S C A r y
gly
THE C2005 LOCAL GOVERNMENT REVIEWER - 70 -
grant franchises for its operation “To regulate” means to fix, in the exercise or furtherance of their governmental or
establish or control. proprietary powers and functions and thereby ensure their
development into self-reliant communities and active
A “franchise” is a special privilege to do certain things conferred by participants in the attainment of national goals.
the government. In contrast, “to register” means to record formally
and exactly, to enroll, or to enter precisely in a list or the like and a any
“driver’s license” is the certificate w/c authorizes a person to Eminent Domain
operate a motor vehicle.
Sec19: Eminent Domain. — A local government unit may, through
The devolution of the functions of the LTFRB is aimed at curbing its chief executive and acting pursuant to an ordinance, exercise
the accidents in national highways involving tricycles. the power of eminent domain for public use, or purpose or
Nevertheless, under Sec 458 (a) [3-VI] of the LGC, the LGUs welfare for the benefit of the poor and the landless, upon
power to regulate the operation of tricycles and to grant franchises payment of just compensation, pursuant to the provisions of the
for its operation is subject to DOTC guidelines. Constitution and pertinent laws: Provided, however, That the
power of eminent domain may not be exercised unless a valid
Clearly unaffected is the LTOs power under RA 4136 Sec 5 and definite offer has been previously made to the owner, and
requiring the registration of all kinds of motor vehicles “used or such offer was not accepted: Provided, further, That the local
operated on or upon any public highway” in the country. LTO is government unit may immediately take possession of the
additionally charged with being the central repository and property upon the filing of the expropriation proceedings and
custodian of all records of all motor vehicles (Sec 4[d]{2}). upon making a deposit with the proper court of at least fifteen
percent (15%) of the fair market value of the property based on
The reliance of City on the broad taxing powers of the LGUs (Sec the current tax declaration of the property to be expropriated:
133 LGC) is tangential. Police power and taxation, along with Provided, finally, That, the amount to be paid for the
eminent domain are inherent powers of sovereignty which State expropriated property shall be determined by the proper court,
might share with LGUs by delegation under the law. But the based on the fair market value at the time of the taking of the
similarity ends there. The aim of police power is public good and property.
welfare while Taxation focuses on the power of govt to raise
revenue in order to support its existence. Although correlative, the
grant of one does not mean the grant of the other. The power
granted under the LGC is the power to regulate their operation and
Rule67, 1997 Rules of Civil Procedure
to grant franchises.
EXPROPRIATION
The exclusionary clause contained in the tax provisions of the LGC Sec1: The complaint.
must not be held to have had the effect of withdrawing the express File: verified complaint which shall
power of the LTO to register all motor vehicles and to issue driver’s * state with certainty the right and purpose of expropriation,
licenses. These functions are essentially regulatory in nature, * describe the real or personal property sought to be
exercised pursuant to police power of the State. expropriated
join as defendants all persons owning or claiming to own, or
occupying, any part thereof or interest therein, showing, so far
as practicable, their separate interest.
Power to Generate Revenue – to be discussed as part of PartII
Aver in complaint if:
Sec18: Power to Generate and Apply Resources. — Local property is owned by the Republic but occupied by private
government units shall have the power and authority to establish individuals, or
an organization that shall be responsible for the efficient and the title is otherwise obscure or doubtful so that the plaintiff
effective implementation of their development plans, program cannot with accuracy or certainty specify who are the real
objectives and priorities; to create their own sources of revenues owners.
and to levy taxes, fees, and charges which shall accrue
exclusively for their use and disposition and which shall be Sec2: Entry of plaintiff upon depositing value with authorized
retained by them; to have a just share in national taxes which government depositary.
shall be automatically and directly released to them without REQS:
need of further action; to have an equitable share in the complaint has already been filed.
proceeds from the utilization and development of the national due notice to the defendant
wealth and resources within their respective territorial deposit with the authorized government depositary an amount
jurisdictions including sharing the same with the inhabitants by equivalent to the assessed value of the property for purposes of
way of direct benefits; to acquire, develop, lease, encumber, taxation to be held by such bank subject to the orders of the
alienate, or otherwise dispose of real or personal property held court.
by them in their proprietary capacity and to apply their resources
and assets for productive, developmental, or welfare purposes,

The Apocryphal Maggots:


Rainier, Chrisgel, Corina, Geoffry, Grace and Sylvie Blanche
The Flibbertigibbet Worms:
Golda, Gladys and Melyjane
CA VE AT: By simply r ea ding this re vi ew er a t the end o f the sem este r wi ll (hopef ully) guar antee you r pas sing this cou rse. D rink mode rat ely .
Amusin S C A r y
gly
THE C2005 LOCAL GOVERNMENT REVIEWER - 71 -
deposit shall be in money, unless in lieu thereof the court Objections to the appointment filed with the court within ten (10)
authorizes the deposit of a certificate of deposit of a government days from service, and shall be resolved within thirty (30) days
bank of the Republic of the Philippines. after all the commissioners shall have received copies of the
objections.
If personal property is involved, its value shall be provisionally
ascertained and the amount to be deposited shall be promptly Sec6: Proceedings by commissioners.
fixed by the court. The commissioners shall assess the consequential damages to
After such deposit is made the court shall order the sheriff or the property not taken and deduct from such consequential
other proper officer to forthwith place the plaintiff in possession damages the consequential benefits to be derived by the owner
of the property involved and promptly submit a report thereof to from the public use or purpose of the property taken, the
the court with service of copies to the parties. operation of its franchise by the corporation or the carrying on of
the business of the corporation or person taking the property.
Sec3: Defenses and objections. But in no case shall the consequential benefits assessed
Defendant has no objection or defense to the action or the exceed the consequential damages assessed, or the owner be
taking of his property: file and serve a notice of appearance and deprived of the actual value of his property so taken.
a manifestation to that effect, specifically designating or
identifying the property in which he claims to be interested,
within the time stated in the summons. Thereafter, he shall be Sec7: Report by commissioners and judgment thereupon.
entitled to notice of all proceedings affecting the same. Except as otherwise expressly ordered by the court, such report
shall be filed within sixty (60) days from the date the
Defendant has any objection or defense to the filing of commissioners were notified of their appointment, which time
complaint: serve his answer within the time stated in the may be extended in the discretion of the court
summons containing- specific designation or identification of the Parties have 10 days from receipt to appeal from said report.
property in which he claims to have an interest, nature and
extent of the interest claimed, all his objections and defenses to Sec8: Action upon commissioners’ report.
the taking of his property. No counterclaim, cross-claim or third- Upon the expiration of the period of appeal, or even before the
party complaint shall be alleged or allowed in the answer or any expiration of such period but after all the interested parties have
subsequent pleading. filed their objections to the report or their statement of
agreement therewith, the court may, after hearing, accept the
A defendant waives all defenses and objections not so alleged report and render judgment; or may recommit the same to the
but the court, in the interest of justice, may permit amendments commissioners for further report of facts; or it may set aside the
to the answer to be made not later than ten (10) days from the report and appoint new commissioners; or it may accept the
filing thereof. However, at the trial of the issue of just report in part and reject it in part; and it may make such order or
compensation, whether or not a defendant has previously render such judgment as shall secure the plaintiff’s right of
appeared or answered, he may present evidence as to the expropriation, and the defendant’s right to just.
amount of the compensation to be paid for his property, and he
may share in the distribution of the award. Sec9: Uncertain ownership; conflicting claims.
If there is uncertainty as to ownership of the property, the court
Sec4: Order of expropriation. may order any sum or sums awarded as compensation for the
If the objections to and the defenses against the right of the property to be paid to the court for the benefit of the person
plaintiff to expropriate the property are overruled, or when no adjudged in the same proceeding to be entitled thereto.
party appears to defend as required by this Rule, the court may
issue an order of expropriation Sec10: Rights of plaintiff after judgment and payment.
payment of just compensation to be determined as of the date of Upon payment by the plaintiff to the defendant of the
the taking of the property or the filing of the complaint, compensation fixed by the judgment, with legal interest thereon
whichever came first. from the taking of the possession of the property, or after tender
to him of the amount so fixed and payment of the costs, the
A final order sustaining the right to expropriate the property may plaintiff shall have the right to enter upon the property.
be appealed by any party aggrieved thereby. Such appeal, If the defendant and his counsel absent themselves from the
however, shall not prevent the court from determining the just court, or decline to receive the amount tendered, the same shall
compensation to be paid. be ordered to be deposited in court and such deposit shall have
the same effect as actual payment to the defendant.
Sec5: Ascertainment of compensation.
Upon the rendition of the order of expropriation, the court shall Sec11: Entry not delayed by appeal; effect of reversal.
appoint not more than three (3) competent and disinterested The right of the plaintiff to enter upon the property not delayed
persons as commissioners to ascertain and report to the court by an appeal from the judgment unless appellate court
the just compensation for the property sought to be taken determines the plaintiff to be without the right to expropriate.
Copies of the order shall be served on the parties.

The Apocryphal Maggots:


Rainier, Chrisgel, Corina, Geoffry, Grace and Sylvie Blanche
The Flibbertigibbet Worms:
Golda, Gladys and Melyjane
CA VE AT: By simply r ea ding this re vi ew er a t the end o f the sem este r wi ll (hopef ully) guar antee you r pas sing this cou rse. D rink mode rat ely .
Amusin S C A r y
gly
THE C2005 LOCAL GOVERNMENT REVIEWER - 72 -
Sec12: Costs, by whom paid. property. In the main, its subject is the govt.’s exercise of eminent
The fees of the commissioners shall be taxed as a part of the domain, a matter which is incapable of pecuniary estimation.
costs of the proceedings. All costs, except those of rival
claimants litigating their claims, shall be paid by the plaintiff, The value of the property to be expropriated is indeed estimated in
unless an appeal is taken by the owner of the property and the monetary terms but this is merely incidental to the expropriation
judgment is affirmed, in which event the costs of the appeal shall suit. In fact, the amount is determined only after the court is
be paid by the owner. satisfied with the propriety of the expropriation.

Sec13: Recording judgment, and its effect.


When real estate is expropriated, a certified copy of such DGNotes
judgment shall be recorded in the registry of deeds of the place DIFFERENCE BETWEEN EXERCISE OF EMINENT DOMAIN BY:
in which the property is situated, and its effect shall be to vest in LGU National Govt.
the plaintiff the title to the real estate so described for such 15% of value of property Assessed value of the proerty
public use or purpose. required to be deposited in required to be deposited in
order to take immediate order to take immediate
possession possession
Barangay San Roque vs. Heirs of Pastor (2000)
Formal offer required No formal offer required
An ordinance must be
Facts:
enacted first by the
Petitioner filed before the MTC a Complaint to expropriate a
Sanggunian
property of the respondents.

MTC dismissed on the ground of lack of jurisdiction as eminent


domain is an exercise of the power to take private property for Suguitan vs. City of Mandaluyong (2000)
public use after payment of just compensation. RTC has exclusive
original jurisdiction over the action as it is one incapable of Facts:
pecuniary estimation. Sangguniang Panlungsod of Mandaluyong issued Res. No. 396
authorizing Mayor Abalos to institute expropriation proceedings
RTC also dismissed the Complaint holding that an action for over the property of Saguitan for expansion of the Mandaluyong
eminent domain affected title to real property. Therefore, the value Medical Center.
of the property would determine whether the case should be filed
before the MTC or RTC. Cited retired Justice Feria’s Petitioners contend- City may only exercise its delegated power of
pronouncement. eminent domain by means of an ordinance as required by Sec. 19
LGC and not by a mere resolution.
Issue: Which court, MTC or RTC, has jurisdiction over cases for
eminent domain or expropriation Respondents contend- it validly and legally exercised its power of
eminent domain. Pursuant to Art 36 Rule VI of the IRR of the LGC,
Held: RTC a resolution is sufficient for the filing of the expropriation
Sec 19(1) BP 129 provides that RTCs shall exercise exclusive proceedings. An ordinance is only required in order to appropriate
original jurisdiction over all civil actions incapable of pecuniary funds for the payment of the just compensation.
estimation.
Issue: WON a resolution is sufficient to authorize the filing of the
In determining whether an action is one the subject matter of which expropriation complaint.
is incapable of pecuniary estimation, the nature of the principal
action or remedy sought must be ascertained first. Held: NO
The exercise of the right of eminent domain is necessarily in
 If it is primarily for the recovery of a sum of money, derogation of private rights, and the rule in that case is that the
the claim is considered capable of pecuniary estimation. authority is to be construed strictly.
 If the basic issue is something other than the right to
recover a sum of money, or where the money claim The power of eminent domain is essentially legislative in nature.
is purely incidental to, or a consequence of, the However, it may be validly delegated to LGUs, other public entities
principal relief sought such action is deemed as and public utilities, although the scope of this delegated power is
having a subject matter incapable of pecuniary necessarily narrower and may only be exercised in strict
estimation over which the RTC have exclusive original compliance with the terms of the delegating law.
jurisdiction.
The basis for the exercise of the power of eminent domain by the
In CAB, the primary consideration of the expropriation suit is LGUs is Sec 19 of the LGC and it is clear that it requires an
whether the govt. has complied with the requisites for taking ordinance and not a resolution.

The Apocryphal Maggots:


Rainier, Chrisgel, Corina, Geoffry, Grace and Sylvie Blanche
The Flibbertigibbet Worms:
Golda, Gladys and Melyjane
CA VE AT: By simply r ea ding this re vi ew er a t the end o f the sem este r wi ll (hopef ully) guar antee you r pas sing this cou rse. D rink mode rat ely .
Amusin S C A r y
gly
THE C2005 LOCAL GOVERNMENT REVIEWER - 73 -
would be entitled to recover in any aspect of the facts or any
Terms ordinance and resolution are not synonymous. combination of the facts alleged, if they were to be proved,
then the MTD should be denied.
ORDINANCE RESOLUTION • The same liberality should be applied in the CAB where an
a law merely a declaration of the examination of the city’s “Comment and Opposition” to the
sentiment or opinion of a law- MTD leaves no room for doubt that the city had indeed made
making body on a specific matter “a valid and definite order” to Cardeno as required by law.
• The general rule is that a MTD hypothetically admits the truth
Possesses a general and Temporary in nature of the facts alleged in the complaint. Thus, Ordinance No.
permanent character 1418 is not only incorporated into the complaint for eminent
A 3rd reading is necessary 3rd reading not necessary, unless domain filed by the city, but is also deemed admitted by
decided otherwise by a majority of Cardeno.
all the Sanggunian members.
• A perusal of the ordinance shows that the fact of the city’s
Rule 67 ROC that an ordinance promulgated by the local having made a previous valid and definite offer to Cardeno is
legislative body authorizing its local chief executive to exercise the categorically stated in its 2nd whereas clause: “WHEREAS,
power of eminent domain is necessary prior to the filing of the the city government has made a valid and definite offer to
complaint with the court, and not only after the court has purchase subject lots for the public use aforementioned but
determined the amount of just compensation. the registered owner Mrs. Merlita Cardeno has rejected such
The reliance w Art 36(a) Rule VI IRR of LGC which requires only a offer.”
resolution to authorize and LGU to exercise eminent domain- • NEVERTHELESS, the dismissal of the complaint did not
Ra 7160 , the law itself, prevails over the rule which merely seeks bar the petitioner city from filing another eminent domain
to implement it. The clear letter of the law is controlling and cannot case and from correcting its alleged error by the mere
be amended by a mere administrative rule issued for its expedient of changing para VII.
implementation.
Discrepancy seems to indicate a mere oversight in the wording of
the IRR since Art 32 of the same Rule also requires that the chief City of Cebu vs. Apolonio (2002)
executive of the LGU must act pursuant to an ordinance.
Facts:
• City of Cebu filed a complaint for eminent domain against the
City of Cebu vs. CA (1996) Dedamo spouses

Facts: • The Dedamos filed a MTD (purpose for expropriation not for
• City of Cebu filed a complaint for eminent domain against public purpose, City can just buy their lot, price offered was
Cardeno with the RTC, seeking to expropriate parcel of land too low, they have no other land in Cebu).
she owns. • The parties then submitted to the court an agreement wherein
• The complaint was initiated pursuant to Resolution No. 404 they declared that they have partially settled the case.
and Ordinance No. 1418 of the Sangguniang Panlungsod of • Trial court appointed 3 commissioners to determine the just
Cebu City, authorizing the City Mayor to expropriate the said compensation of the lots.
parcel of land for the purpose of providing a socialized o The commissioners submitted their report on the basis of
housing project for the landless and low-income city which TC rendered its decision.
residents. City interposed objections to assessment by the commissioners.
• Cardeno filed a MTD on the ground of lack of cause of action.
And failure to make a valid and definite offer. When must just compensation be determined? Time of actual
• City alleged compliance with said requirement. It was only taking or filing of complaint?
after Cerdeno repeatedly refused their offer did they file the • The applicable law as to the point of reckoning for the
expropriation case. determination of just compensation is Sec. 19, LGC, which
• RTC dismissed the complaint expressly provides that just compensation shall be
• CA affirmed and held that an allegation of repeated determined as of the time of actual taking.
negotiations made with Cardeno cannot be equated with the
clear and specific requirement that the city should have • The city has misread the ruling in Napocor v. CA. It was not
previously made a valid and definite offer to purchase. categorically ruled in that case that just compensation should
be determined as of the filing of the complaint. It was there
Issue: WON the complaint states a cause of action – YES stated that although the general rule in determining just
• Though the allegations in the complaint are ambiguous, compensation in eminent domain is the value of the
indefinite or uncertain but, nevertheless, a cause of action property as of the date of the filing of the complaint, the
can, in any manner, be made out therefrom, and the plaintiff rule admits of an exception: where the SC fixed the value

The Apocryphal Maggots:


Rainier, Chrisgel, Corina, Geoffry, Grace and Sylvie Blanche
The Flibbertigibbet Worms:
Golda, Gladys and Melyjane
CA VE AT: By simply r ea ding this re vi ew er a t the end o f the sem este r wi ll (hopef ully) guar antee you r pas sing this cou rse. D rink mode rat ely .
Amusin S C A r y
gly
THE C2005 LOCAL GOVERNMENT REVIEWER - 74 -
of the property as of the date it was taken and not at the
date of the commencement of the expropriation  old concept: condemned property must actually be used by
proceedings. (Case didn’t explain why this is an exception) the general public (roads, bridges, public plazas) before the
• While Sec. 4, Rule 67 provides that just compensation shall taking thereof could satisfy the constitutional requirement of
be determined at the time of the filing of the complaint for public use
expropriation, such law cannot prevail over the LGC, which is
a substantive law. 2. establishment of pilot development center would inure to the
direct benefit and advantage of the people of the province
WON the city is bound by the compensation fixed by the
commissioners – YES Issue: Whether the expropriation of agricultural lands by LGUs is
subject to the prior approval of the Secretary of Agrarian Reform,
• More than anything else, the parties, by a solemn
as the implementor of the agrarian reform program.
document freely and voluntarily agreed upon by them, Held: NO
agreed to be bound by the report of the commission and 1. section 9 of BP 337 does not intimate in the least that LGUs
approved by the trial court. The agreement is a contract must first secure the approval of the Department of Land reform for
between the parties. It has the force of law between them the conversion of lands before they can institute the necessary
and should be complied with in good faith. expropriation proceedings.
• Furthermore, during the hearing, the city did not interpose a
serious objection. It is therefore too late for the city to 2. No provision in CARL which subjects the expropriation of
question the valuation now without violating the principle of agricultural lands by LGUs to the control of the DAR.
equitable estoppel.
3. rules on conversion of agricultural lands found in Section 4 (k)
and 5 (1) of Executive Order No. 129-A, Series of 1987 cannot be
the source of the authority of the DAR to determine the suitability
of a parcel of agricultural land for the purpose to which it would be
Province of Camarines Sur vs. CA (1993) devoted by the expropriating authority. Said rules merely vest on
the DAR the exclusive authority to approve or disapprove
Facts: conversions of agricultural lands for residential, commercial or
The Sangguniang Panlalawigan passed resolution No. 129, industrial uses, such authority is limited to the applications for
authorizing the Provincial Governor to purchase or expropriate reclassification submitted by the land owners or tenant
property contiguous to the provincial capital site, in order to beneficiaries.
establish a pilot farm and a housing project for provincial
government employees 5. Ordinarily, it is the legislative branch of the local government unit
that shall determine whether the use of the property sought to be
Governor Villafuerte filed two separate cases for expropriation expropriated shall be public, the same being an expression of
against Ernesto & Efren San Joaquin. legislative policy.

The San Joaquins moved to dismiss the complaints on the ground  courts defer to such legislative determination and will
of inadequacy of the price offered for their property. The TC denied intervene only when a particular undertaking has no real or
the motion substantial relation to the public use

Solicitor General: under Section 9 of the LGC, there was no need


for the approval by the Office of the president of the exercise by 6. fears of private respondents that they will be paid on the basis
the Sangguniang Panlalawigan of the right of eminent domain of the valuation declared in the tax declarations of their property,
 but Province must first secure the approval of the DAR of the are unfounded.
plan to expropriate the lands of petitioners for use as a
housing project (Section 65 of RA 6657: Comprehensive  This Court has declared as unconstitutional the Presidential
Agrarian Reform Law requires the approval of the DAR before Decrees fixing the just compensation in expropriation cases
a parcel of land can be reclassified from an agricultural t a to be the value given to the condemned property either by the
non-agricultural land owners or the assessor, whichever was lower

Issue1: WON the expropriation is for a public purpose or public


use  Municipality of Talisay v. Ramirez: the rules for determining
Held: YES just compensation are those laid down in Rule 67 of the Rules
1. New concept of public use: means public advantage, of Court, which allow private respondents to submit evidence
convenience or benefit, which tends to contribute to the general on what they consider shall be the just compensation for their
welfare and the prosperity of the whole community, like a resort property.
complex for tourists or housing projects

The Apocryphal Maggots:


Rainier, Chrisgel, Corina, Geoffry, Grace and Sylvie Blanche
The Flibbertigibbet Worms:
Golda, Gladys and Melyjane
CA VE AT: By simply r ea ding this re vi ew er a t the end o f the sem este r wi ll (hopef ully) guar antee you r pas sing this cou rse. D rink mode rat ely .
Amusin S C A r y
gly
THE C2005 LOCAL GOVERNMENT REVIEWER - 75 -
inseparable transaction involving the development of the 808
Heirs of Ardona vs. Reyes (1983) hectares tourism complex.

Facts: Issue3: WON there is impairment of obligations of contracts


* The Philippine Tourism Authority wishes to expropriate 282 h. of Held3: Invocation of the contracts clause has no merit
land for development into integrated complexes of areas with • The non-impairment clause has never been a barrier to the
potential tourist value. exercise of police power and likewise eminent domain. Parties
* The defendant filed a motion to dismiss on grounds of Public by entering into contracts may not estop the legislature from
Use. They contend that the land is already for land reform and that
it should seek the approval of the Court of Agrarian Relations. DGNotes:
* CFI granted writs of possession. SC held that the power of eminent domain of LGUs is more
* Petitioners brought instant petition before SC to enjoin execution. superios than land reform-
SIR: Duh? Land reform is constitutionally mandated. Tourism,
Issue1: WON there is non-compliance with the “public use” creation of parks, etc are not.
requirement under the eminent domain provision of the Bill of
Rights
Held1: NO. the state’s power of eminent domain extends to the Filstream International Inc. vs. CA (1998)
expropriation of land for tourism purposes although this specific
objective is not expressed in the Constitution
Facts:
• The policy objective of the framers can be expressed only in I. Ejectment
general terms such as social justice, local autonomy, Filstream is the owner of parcels of land in Tondo. It instituted
conservation and development of the national patrimony, ejectment proceedings against the occupants of the land and was
public interest, and general welfare among others. able to obtain a judgment from the MTC ordering the occupants to
To include specific programs like tourism as express vacate the premises & to pay back rentals. RTC & CA affirmed.
constitutional provisions would make the Constitution more Ejectment order was never effected.
prolix than a bulk code and would require the framers to be Later, the city wanted to expropriate the land by virtue of an
impossibly prescient. The particular mention in the Constitution ordinance which was questioned by petitioner since there is
of agrarian reform, among others, merely underscores the already a judgment in their favor and also, the price offered was
magnitude of the problems sought to be remedied by these too low, the ordinance was without hearing and there is no
programs. They do not preclude nor limit the exercise of the certification of municipal funds available. Contentions were
power of eminent domain for such purposes like tourism and denied.
other development programs. Filstream wanted to demolish the place but the occupants were
• The concept of public use is not limited to traditional able to secure an injunction.
purposes. The idea that “public use is strictly limited to clear The case was reshuffled and the order of demolition was granted.
cases of “use by the public” has been discarded. CA issued an injunction again.
Filstream filed a petition questioning the injunction.
Issue2: WON there is disregard of the land reform nature of the
property being expropriated SC HELD:
Held2: NO Procedural – Did the court put too much weight on
1. land being developed into a tourism complex consists of procedure?
more than 808 hectares, almost all of which is not affected by Yes. What it should have done was to allow a clearer copy of the
the land reform program petition to be submitted and not dismiss the case outright.. This
2. the portion being expropriated consists of hilly and amounted to a deprivation of property without due process as this
unproductive land where even subsistence farming of crops was an expropriation proceeding.
other than rice and corn can hardly survive Substantive
3. Only 8,970 square meters of 283 hectares affected is part of Filstream claims: ownership & existence of a final & executory
the operation land transfer judgment against the occupants vs.
4. this 8,970 square meters is not even within the sports Occupants claim: supervening event (expropriation proceedings)
complex proper but form parts of the 32 hectares rendering the enforcement of Filstream’s rights moot
resettlement area 1) Manila has an undeniable right to exercise its power of eminent
5. of the 40 defendants, only 2 have emancipation patents for domain w/in its jurisdxn (§19 LGC). It has the power to expropriate
the less than one hectare of land affected private property in pursuit of its urban land reform & housing
Under the facts of this petition, there is no need to rule on program (§§3,100 of RA 409 - Revised Charter of the City of
whether one public purpose is superior or inferior to another Manila)
purpose or engage in a balancing of competing public In Phil. Columbian Association v Panis, the SC has upheld the City
interests. Petitioners have failed to overcome the showing of Manila’s right to exercise these prerogatives notwithstanding the
that the taking of the 8,970 square meters covered by the
Operation Land Transfer forms a necessary part of an

The Apocryphal Maggots:


Rainier, Chrisgel, Corina, Geoffry, Grace and Sylvie Blanche
The Flibbertigibbet Worms:
Golda, Gladys and Melyjane
CA VE AT: By simply r ea ding this re vi ew er a t the end o f the sem este r wi ll (hopef ully) guar antee you r pas sing this cou rse. D rink mode rat ely .
Amusin S C A r y
gly
THE C2005 LOCAL GOVERNMENT REVIEWER - 76 -
existence of a final & executory judgment over the property to be First District in Tondo  to be sold and distributed pursuant to
expropriated. Land Use Development Program of the City of Manila

2) Nevertheless, LGUs are not given unbridled authority when • City filed an amended complaint for expropriation against
exercising their power of eminent domain  subject to: present owners of lot  the Serranos, children of Demetria.
• CA held that Lot 1-C is not exempt from expropriation as it
a. basic rules in the Consti: due process (Art 3 §1), equal
exceeds 300 square meters. However, citing Filstream, the
protection (Art 3 §1), just compensation (Art 3 §9) other modes of acquisition of land enumerated in Sections 9
b. §19 of LGC “An LGU may…exercise the power of and 10were not shown to have been resorted to by the city
eminent domain…pursuant to the provisions of the I.
Constitution & pertinent laws”. ↓ Petitioner contends that the CA erroneously presumed that Lot 1C
has been ordered condemned in its favor when the fact is that the
3) The governing law that deals with the subject of expropriation order of the trial court merely authorized the issuance of the writ of
for purposes of urban land reform & housing is RA 7279 (Urban possession and petitioner's entry pursuant to Rule 67. At that
Development & Housing Act of 1992), §92 and §103 of which set stage, it was premature whether the requirements of section 9 and
out the limitations with respect to the order of priority in acquiring 10 have been complied with since no evidentiary hearing had yet
private lands & in resorting to expropriation proceedings as a been conducted by the trial court.
means to acquire the same.
HERE  There is no showing that the City of Manila complied with SC: Contention well taken (YEEEEY!!!)
§§9,10. Filstream’s properties were expropriated without any Citing Rule 67 section 2, SC ruled that a writ of execution may
showing that resort to other modes of acquiring lands listed in §9 be issued by a court upon a filing by the government of a complaint
have proved futile. for expropriation sufficient in form and substan.ce and upon
The State has a paramount interest in exercising its power of deposit made by the government of the amount equivalent to the
eminent domain for the general good…however…individual rights assessed value of the property. Upon compliance, the issuance of
affected by the exercise of such right are also entitled to the writ becomes ministerial.
protection….vigilance over compliance with due process CA erroneously applied Filstream ruling. In that case, such
requirements is in order. ruling was necessitated because an order of condemnation has
Petitions granted. CA resolutions reversed & set aside. already been issued by the trial court. Hence, the judgment of the
City of Manila vs. Serrano (2001) TC in Filstream case was already final. In CAB, the TC has not yet
gone beyond the issuance of the writ of possession. Hearing is s
till to be held to determine WON petitioner complied with the
Facts:
requirements provided in RA 7279.
• City Council of Manila enacted Ordinance No. 7833,
authorizing the expropriation of certain properties in Manila's HENCE, CA ruling REVERSED, TC order REINSTATED, case
REMANDED to the trial court for further proceedings.
2
Sec. 9. Priorities in the Acquisition of Land. — Lands for socialized housing DGNotes:
shall be acquired in the following order: Its not that Filstream does not apply, its just that its application
(a) Those owned by the Government or any of its subdivisions, instrumentalities, was premature. WON Secs. 9 & 10 were complied with still has to
or agencies, including government-owned or -controlled corporations and their be determined.
subsidiaries;
(b) Alienable lands of the public domain;
(c) Unregistered or abandoned and idle lands; City of Mandaluyong vs. Aguilar (2001)
(d) Those within the declared Areas for Priority Development, Zonal Improvement
Program sites, and Slum Improvement and Resettlement Program sites which Facts:
have not yet been acquired; • Subject of expropriation: 3 adjoining parcels of land leased
(e) Bagong Lipunan Improvement of Sites and Services or BLISS sites which have
out to tenants by Aguilars
not yet been acquired; and
• 1983: lots were classified by Resolution No 125 of the Board
(f) Privately-owned lands [last in the order of priority]
of the Housing and Urban Development Coordinating Council
3 as an area for priority development for urban land reform,
Sec. 10. Modes of Land Acquisition. — The modes of acquiring lands for pursuant to Proclamation Nos 1967 and 2284 of then
purposes of this Act shall include, among others, community mortgage, land President Marcos.
swapping, land assembly or consolidation, land banking, donation to the • Tenants offered to purchase the land but respondents refused
Government, joint-venture agreement, negotiated purchase, and expropriation: • 1996: Sangguniang Panlungsod of Mandaluyong, upon
Provided, however, That expropriation shall be resorted to only when other models petition of an association of tenants (Kapitbisig) adopted
of acquisition have been exhausted… Resolution No. 516 -S96 authorizing Mayor Abalos to initiate

The Apocryphal Maggots:


Rainier, Chrisgel, Corina, Geoffry, Grace and Sylvie Blanche
The Flibbertigibbet Worms:
Golda, Gladys and Melyjane
CA VE AT: By simply r ea ding this re vi ew er a t the end o f the sem este r wi ll (hopef ully) guar antee you r pas sing this cou rse. D rink mode rat ely .
Amusin S C A r y
gly
THE C2005 LOCAL GOVERNMENT REVIEWER - 77 -
action for expropriation and construction of medium-rise a. It should be noted that the property in question where
condominium
originally 2 residential lots. Respondents acquired properties
• Mayor Abalos sent a letter to Aguilars offering to purchase, from their parents through intestate succession. In 1986, they
Aguilars did not answer. agreed to an extrajudicial partition. This was only effected in
• August 1997, petitioner City of Mandaluyong filed with the 1998. Coming from this background, petitioner contends
RTC of Pasig a complaint for expropriation that respondents partitioned the property in bad faith, being
• Trial court dismissed the complaint: made 6 months after the complaint for expropriation was filed.
• City moved for Recon, denied. SC held: No, partition was done in good faith. When the siblings
inherited the properties, they are considered as co-owners. The
ISSUE: WON the trial court erred in upholding respondents' right of the co-owners to have the property partitioned cannot be
contention that they qualify as small property owners and are thus questioned, as "no co-owner shall be obliged to remain in the co-
exempt from expropriation. ownership" (Art 494, Civil Code). The partition was a necessary
Main Argument of Petitioner: The size of the lots in litigation does incident of the co-ownership. Absent any evidence to the contrary,
not exempt the same from expropriation in view of the fact that the partition is presumed to have been done in good faith.
said lots have been declared to be within the Area for Priority
Development (APD) of Mandaluyong by virtue of Proclamation No b. Upon partition, siblings received the following:
1967 as amended by Proc No. 2284 in relation to PD 15 17 (Urban Francisco, Thelma, Rodolfo and Antonio: each with 300
Land Reform Act. These authorize Mandaluyong to expropriate the square meters
property ipso facto, regardless of area of land. Virginia: 89
SC: No, trial court did not err. Petition Denied Eusebio: 347
• under section 9 of the UDHA (see law) , the lots in litigation SC: ops! Eusebio's share exceeds 300 sq meters! However,
Eusebio died in 1995 survived by 5 children. When there are
are privately owned and therefore last in the order of priority
several co-owners and some of them die, the heirs of those who
acquisition. However, the lots are also declared APD which is
die, with respect to that part belonging to the deceased, become
4th in the order of priority.
also co-owners also of that property. So 347 / 5 = 69.4 sq meters.
• Now, section 9 (which speaks of priorities in acquisition)
should be read in connection with section 10 (see law) which SC's discussion on the second element:
provides for the modes of land acquisition. The acquisition of Antonio Aguilar himself testified that that he and other co-owners of
the lands must be made subjects to the modes and conditions the property do not reside in the lots in question but in their
set forth in section 10. ancestral home in Paco, Manila. Nonetheless, records do not show
that the ancestral home and the land on which it stands are owned
1. Petitioner claims that it had faithfully observed the modes of by respondents. City of Mandaluyong did not present any title or
land acquisition proof of this fact.
SC: City, however, failed to state with particularity that it exhausted Hence, Petition denied.
the other modes of acquisition. City alleged only one mode of
acquisition: negotiated purchase.
Bardillon vs. Masili (2003)
2. City claims the exercise of the power of eminent domain is
not anymore conditioned on the size of the land. Facts:
SC: the UDHA introduced a limitation on the size of land sought to Brgy. Masili in Calamba, Laguna filed 2 complaints for eminent
be expropriated for socialized housing. It exempted "small property domain for the purpose of expropriating a parcel of land within
owners" as defined in the law (see law again for definition, Section Brgyy Masili, owned by Bardillon for the purpose of providing
3(q) ) Bargy. Masili a multi-purpose hall for the use and benefit of its
constituents.
3. SC then talks to itself out loud… question now is whether
respondents qualify as small property owners? And SC held, First complaint was filed at MTC after the failure of Brgy. Masili to
yes they are! (oooohh…) reach an agreement with Bardillon on the purchase offer of
P200,000. – DISMISSED
Two elements of "small property owner" definition:
1) Those owners of real property consists of residential lands Second complaint for eminent domain was filed at RTC seeking
with an area of not more than 300 sq meters in highly the expropriation of the said lot for the same purpose.
urbanized cities (800 in other urban cities)
2) That they do not own real property other than the same. Bardillon opposed by Motion to Dismiss, alleging that it violated
Both these elements are possessed by respondents! (aaaaahh…) Section 19(f) of Rule 16 in that cause of action is barred by prior
SC's Discussion on the first element: judgment, pursuant to the doctrine of res judicata.

The Apocryphal Maggots:


Rainier, Chrisgel, Corina, Geoffry, Grace and Sylvie Blanche
The Flibbertigibbet Worms:
Golda, Gladys and Melyjane
CA VE AT: By simply r ea ding this re vi ew er a t the end o f the sem este r wi ll (hopef ully) guar antee you r pas sing this cou rse. D rink mode rat ely .
Amusin S C A r y
gly
THE C2005 LOCAL GOVERNMENT REVIEWER - 78 -
RTC: Denied MTD. Held that MTC which ordered the dismissal of Held: The requirements for the issuance of a writ of possession in
1st complaint has no jurisdiction over the said expropriation an expropriation case are expressly and specifically governed by
proceeding. Issued Order in favor of Brgy. Masili. Section 2 of Rule 67 of the 1997 Rules of Civil Procedure.
On the part of LGU’s, expropriation is also governed by Sec 19 of
CA: RTC did not commit grave abuse of discretion in issuing the the LGCode.
assailed Orders. 2nd Complaint was not barred by res judicata
since MTC had no jurisdiction over the action. In expropriation proceedings, the requisites for authorizing
immediate entry are as follows:
1. WON the MTC had jurisdiction over the expropriation case the filing of a complaint for expropriation sufficient in form and
Held: An expropriation suit does not involve the recovery of a sum substance; and
of money. Rather, it deals with the exercise by the government of the deposit of the amount equivalent to 15 percent of the fair
its authority and right to take property for public use As such, it is market value of the property to be expropriated based on its
incapable of pecuniary estimation and should be filed with the current tax declaration.
regional trial courts.
In CAB, the issuance of the Writ of Possession in favor of Brgy.
The primary consideration in an expropriation suit is whether the Masili was proper because it had complied with the foregoing
government or any of its instrumentalities has complied with the requisites.
requisites for the taking of private property. Hence, the courts
determine the authority of the government entity, the necessity of The issue of the necessity of the expropriation is a matter properly
the expropriation, and the observance of due process. addressed to the RTC in the course of the expropriation
proceedings. If petitioner objects to the necessity of the takeover
The value of the property to be expropriated is estimated in of her property, she should say so in her Answer to the Complaint.
monetary terms and the court is duty-bound to determine the just The RTC has the power to inquire into the legality of the exercise
compensation for it. This, however, is merely incidental to the of the right of eminent domain and to determine whether there is a
expropriation suit. Indeed, that amount is determined only after the genuine necessity for it.
court is satisfied with the propriety of the expropriation. Petition denied.

An expropriation suit is within the jurisdiction of the RTC


regardless of the value of the land, because the subject Reclassification of Lands
of the action is the government’s exercise of eminent
domain -- a matter that is incapable of pecuniary --- through an ordinance
estimation. --- conversion to—residential, commercial, or industrial
purposes
2. whether the dismissal of that case before the MTC constituted
res judicata Sec20, LGC: Reclassification of Lands. —
Held: Res judicata literally means a matter adjudged, judicially (a) A city or municipality may, through an ordinance passed by the
acted upon or decided, or settled by judgment. It provides that a sanggunian after conducting public hearings for the purpose,
final judgment on the merits rendered by a court of competent authorize the reclassification of agricultural lands and provide for
jurisdiction is conclusive as to the rights of the parties and their the manner of their utilization or disposition in the following cases:
privies; and constitutes an absolute bar to subsequent actions (1) when the land ceases to be economically feasible and sound
involving the same claim, demand or cause of action. for agricultural purposes as determined by the Department of
Requisites of res judicata: Agriculture or (2) where the land shall have substantially greater
the former judgment must be final; economic value for residential, commercial, or industrial
the court that rendered it had jurisdiction over the subject matter purposes, as determined by the sanggunian concerned:
and the parties; Provided, That such reclassification shall be limited to the
it is a judgment on the merits; and following percentage of the total agricultural land area at the
there is -- between the first and the second actions -- an identity of time of the passage of the ordinance:
parties, subject matter and cause of action. (1) For highly urbanized and independent component
cities, fifteen percent (15%);
Since the MTC had no jurisdiction over expropriation proceedings, (2) For component cities and first to the third class
the doctrine of res judicata finds no application even if the Order of municipalities, ten percent (10%); and
dismissal may have been an adjudication on the merits. (3) For fourth to sixth class municipalities, five percent
(5%): Provided, further, That agricultural lands distributed to
3. Whether the CA erred when it ignored the issue of entry upon agrarian reform beneficiaries pursuant to Republic Act
the premises Numbered Sixty-six hundred fifty-seven (R.A. No. 6657).
otherwise known as "The Comprehensive Agrarian Reform
Law", shall not be affected by the said reclassification and the

The Apocryphal Maggots:


Rainier, Chrisgel, Corina, Geoffry, Grace and Sylvie Blanche
The Flibbertigibbet Worms:
Golda, Gladys and Melyjane
CA VE AT: By simply r ea ding this re vi ew er a t the end o f the sem este r wi ll (hopef ully) guar antee you r pas sing this cou rse. D rink mode rat ely .
Amusin S C A r y
gly
THE C2005 LOCAL GOVERNMENT REVIEWER - 79 -
conversion of such lands into other purposes shall be In all cases, applications for conversion involving lands
governed by Section 65 of said Act. protected from and non-negotiable for conversion shall not be
(b) The President may, when public interest so requires and upon given due course by the DAR.
recommendation of the National Economic and Development
Authority, authorize a city or municipality to reclassify lands in 2. The following areas shall be highly restricted from conversion:
excess of the limits set in the next preceding paragraph. a. Lands classified as 'Highly Restricted from Conversion' in the
(c) The local government units shall, in conformity with existing Network of Protected Areas for Agriculture as delineated by
laws, continue to prepare their respective comprehensive land use the DA, as follows:
plans enacted through zoning ordinances which shall be the a.1 Irrigable lands not covered by irrigation projects
primary and dominant bases for the future use of land resources: with firm funding commitments;
Provided. That the requirements for food production, human a.2 Agro-industrial croplands, or lands presently
settlements, and industrial expansion shall be taken into planted to industrial crops that support the economic
consideration in the preparation of such plans. viability of existing agricultural infrastructure and agro-
(d) Where approval by a national agency is required for based enterprises; and
reclassification, such approval shall not be unreasonably withheld. a.3 Highlands, or areas located in elevations of 500
Failure to act on a proper and complete application for meters or above and have the potential for growing semi-
reclassification within three (3) months from receipt of the same temperate and usually high value crops.
shall be deemed as approval thereof.
(e) Nothing in this Section shall be construed as repealing, b. Lands issued a Notice of Acquisition/Valuation under the
amending, or modifying in any manner the provisions of R.A. No. agrarian reform program or subject of a perfected agreement
6657. between the landowner and the beneficiaries under Voluntary
Land Transfer (VLT) or Direct Payment Scheme (DPS) under
CARP, as determined by the DAR; and
Administrative Order No.363 (Oct9, 1997) - Prescribing c. Areas identified as environmentally critical as determined by
Guidelines For The Protection Of Areas Non-Negotiable For the DENR, pursuant to PD 1586 (1978) and its implementing
Conversion And Monitoring Compliance With Section 20 Of rules and regulations;
The Local Government Code
Lands classified as highly restricted from conversion may be
Sec1. Declaration of Principles and Governing Policies converted only upon compliance with existing laws, rules and
A. General Principles regulations. An additional requirement of the social benefit cost
xxx analysis approved by the DA shall also be required before these
B. Governing Policies lands may be approved for conversion.
1. The following areas shall not be subject to or non-negotiable for Applications for conversion covering areas under 2 (c) above
conversion: shall be subject to the Environmental Impact Assessment (EIA)
a. Protected areas designated under the National Integrated and/or Environmental Compliance Certificate (ECC) of the
Protected Areas (NIPAS), including watershed and recharge DENR.
areas of aquifers, as determined by the Department of
Environment and Natural Resources (DENR), pursuant to RA 3. Conversion of priority areas under Executive Order 124 (1993),
7586 (1992); as identified below, falling within the areas highly restricted from
b. All irrigated lands, as delineated by the Department of conversion may be allowed and the social benefit cost analysis
Agriculture (DA) and/or the National Irrigation Administration for these areas may be waived; PROVIDED that the
(NIA) and approved by the President, where water is requirement of an EIC or ECC shall always be required;
available to support rice and other crop production, and all PROVIDED FURTHER that in no case shall conversion be
irrigated lands where water is not available for rice and other allowed if these sites fall under those classified as non-
crop production but are within areas programmed for irrigation negotiable areas:
facility rehabilitation by the DA and the NIA, pursuant to a. specific sites in regional agri-industrial centers/regional
Presidential Administrative Order 20 (1992); and industrial centers (RAICs/RICs) identified by the Department
c. All irrigable lands already covered by irrigation projects with of Trade and Industry (DTI) and the Department of Agriculture
firm funding commitments, as delineated by the DA and/or (DA), attached as Annex A;
NIA and approved by the President. b. tourism development areas (TDAs) identified by the
Department of Tourism (DOT), attached as Annex B; and
For this purpose, the Network of Protected Areas for Agriculture c. sites identified by the local government units (LGUs) for
(as of 1991), as determined by the DA and/or NIA shall serve as socialized housing.
guide in determining non-negotiable areas. The Network may
only be revised upon the approval of the President, upon 4. In all cases, farmers or prospective beneficiaries of the agrarian
favorable recommendation by the Cabinet Cluster on Agro- reform program affected by the conversion shall be paid
Industrial Development. sufficient disturbance compensation. In addition, the owners and
or developers of the land shall be encouraged to provide capital

The Apocryphal Maggots:


Rainier, Chrisgel, Corina, Geoffry, Grace and Sylvie Blanche
The Flibbertigibbet Worms:
Golda, Gladys and Melyjane
CA VE AT: By simply r ea ding this re vi ew er a t the end o f the sem este r wi ll (hopef ully) guar antee you r pas sing this cou rse. D rink mode rat ely .
Amusin S C A r y
gly
THE C2005 LOCAL GOVERNMENT REVIEWER - 80 -
which will enable the affected farmers and other legitimate shall be required to replace the areas affected by an equal area
stakeholders to shift to another livelihood, skills training, of irrigated lands, whether within or outside the area/locality
relocation sites, and priority in employment for them and their being applied for conversion; and PROVIDED FURTHER that
children. Investment arrangements which give affected farmers such conversion shall not adversely affect the irrigation system.
and other legitimate stakeholders a stake in the development of
the land, such as, but not limited to, joint ventures and Sec2: Definition of Terms. — xxx
partnerships, shall also be encouraged.
Sec3: Monitoring Compliance with Sec. 20 of RA 7160. xxx
5. No application for reclassification by LGUs shall be given due Sec4: Penalties and Sanctions. — xxx
course by HLURB without the approved Comprehensive Land
Use Plan approved by the HLURB for provinces, highly Sec5: Transitory Provisions. — xxx
urbanized cities, independent component cities and the cities (deleted this na kasi 1997 pa date of effectivity nito. If you guys
and municipalities of Metropolitan Manila, or the Sangguniang want to check this out for intellectual growth (o para sa medal, di
Panlalawigan for component cities and municipalities, after 1 ba mike?) see our daily digests. 
January 1989. The following requirements or certifications from
various agencies shall also be required:
a. Certification from the local HLURB specifying the total area of DGNotes:
zoned agricultural lands in the local government concerned CONVERSION CLASSIFICATION
based on the approved Comprehensive Land Use Plan or
Change in actual use of land Classifying land in paper
Zoning Ordinance prior to the application for conversion;
b. Certification from the NIA that the area to be reclassified is
Done with approval of DAR Done by Sanggunian
not covered under Presidential A.O. 20, s. 1992;
c. Certification from the DAR indicating that such lands are not Classification of Sanggunian
distributed or covered by a Notice of Valuation under CARP; • Validity depends on compliance with requisites in Sec 20
and • LGC however, does not provide how often this can be
d. Certification from DENR that the area applied for
reclassification has been classified as alienable and done (sir thinks this can be used to create loopholes
disposable, and is not needed for forestry purposes in case with respect to the requirements of the law)
the area applied for falls within public lands. • Conversion is an additional safeguard for power to
reclassify.
6. No application for conversion shall be given due course by DAR Sec 20 LGC – Sanggunian may classify lands into
without the following certifications from various agencies: 1. residential
a. Certification of the Viability or Non Viability of Agricultural 2. commercial
Land from the DA and that the land is not part of the area 3. industria
identified as non-negotiable for conversion or a certification
as to whether the land is classified as highly restricted from
conversion or not; RA 8435 (1997) - An Act Prescribing Urgent Related
b. Certification that the land does not fall under the NIPAS area Measures To Modernize The Agriculture And Fisheries
or is not classified as environmentally critical from the DENR. Sectors Of The Country In Order To Enhance Their
For applications for conversion involving environmentally Profitability, And Prepare Said Sectors For The Challenges
critical areas, the DAR may issue an Order of Conversion, Of Globalization Through An Adequate, Focused And
subject to the issuance of an ECC by the DENR. Rational Delivery Of Necessary Support Services,
The DENR, in coordination with the DAR, shall institute an Appropriating Funds Therefor And For Other Purposes
Environmental Guarantee Fund to ensure environment
protection and to provide government the financial capability to Sec9: Delineation of Strategic Agriculture and Fisheries
handle negative impacts of the conversion; Development Zones. — The Department, in consultation with
c. Certification from the NIA stating that the area is not covered the Department of Agrarian Reform, the Department of Trade
under Presidential A.O. 20, S. 1992; and and Industry, the Department of Environment and Natural
d. Certification from the HLURB that the land has been Resources, Department of Science and Technology, the
reclassified and that said reclassification is within or outside concerned LGUs, the organized farmers and fisherfolk groups,
the maximum allowable limits set by law. the private sector and communities shall, without prejudice to
the development of identified economic zones and free ports,
7. In all cases, the decision of the DAR Secretary shall be establish and delineate, based on sound resource accounting,
appealable to the Office of the President. The President may the SAFDZ within one (1) year from the effectivity of this Act.
allow the conversion of areas considered non-negotiable for All irrigated lands, irrigable lands already covered by irrigation
conversion only upon the favorable recommendation of the projects with firm funding commitments, and lands with existing
Cabinet Cluster on Agro-Industrial Development; PROVIDED or having the potential for growing high-value crops so
that where lands affected are irrigated, the owner/developer delineated and included within the SAFDZ shall not be

The Apocryphal Maggots:


Rainier, Chrisgel, Corina, Geoffry, Grace and Sylvie Blanche
The Flibbertigibbet Worms:
Golda, Gladys and Melyjane
CA VE AT: By simply r ea ding this re vi ew er a t the end o f the sem este r wi ll (hopef ully) guar antee you r pas sing this cou rse. D rink mode rat ely .
Amusin S C A r y
gly
THE C2005 LOCAL GOVERNMENT REVIEWER - 81 -
converted for a period of five (5) years from the effectivity of this (b) Conversion may be allowed, in accordance with Section 65 of
Act: Provided, however, That not more than five percent (5%) of RA 6657, when the land has ceased to be economically feasible
the said lands located within the SAFDZ may be converted upon and sound for agricultural purposes; or the locality has become
compliance with existing laws, rules, regulations, executive urbanized and the land will have greater economic value for
orders and issuances, and administrative orders relating to land residential, commercial, industrial or other non-agricultural
use conversion: Provided, further, That thereafter: 1) a review of purposes;
the SAFDZ, specifically on the productivity of the areas, (c) Conversion of lands within SAFDZs, as provided in Rule 9.5.2
improvement of the quality of life of farmers and fisherfolk, and of DA Administrative Order No. 6, series of 1998, shall take into
efficiency and effectiveness of the support services shall be account the following factors:
conducted by the Department and the Department of Agrarian (1) The conversion of land use is consistent with the natural
Reform, in coordination with the Congressional Oversight expansion of the municipality or locality, as contained in the
Committee on Agricultural and Fisheries Modernization; 2) approved physical framework and land use plan;
conversion may be allowed; if at all, on a case-to-case basis (2) The area to be converted in use is not the only remaining
subject to existing laws, rules, regulations, executive orders and food production area of the community;
issuances, and administrative orders governing land use (3) The land use conversion shall not hamper the availability of
conversion; and, 3) in case of conversion, the land owner will irrigation to nearby farmlands;
pay the Department the amount equivalent to the government's (4) The areas with low productivity will be accorded priority for
investment cost including inflation. use conversion; and
(5) Sufficient disturbance compensation shall be given to the
Sec11: Penalty for Agricultural Inactivity and Premature farmers whose livelihoods are negatively affected by the land
Conversion. — Any person or juridical entity who knowingly or use conversion as provided for by existing laws and
deliberately causes any irrigated agricultural lands seven (7) regulations.
hectares or larger, whether contiguous or not, within the (d) Conversion may be allowed when environmental impact
protected areas for agricultural development, as specified under assessment or initial environmental examination, as may be
Section 6 in relation to Section 9 of this Act, to lie idle and appropriate, shall have determined that it shall not adversely
unproductive for a period exceeding one (1) year, unless due to affect air and water quality and the ecological stability of the
force majeure, shall be subject to an idle land tax of Three area.
Thousand Pesos (P3,000.00) per hectare per year. In addition,
the violator shall be required to put back such lands to Sec9: Who May Apply for Conversion. — The following persons
productive agricultural use. Should the continued agricultural may apply for conversion:
inactivity, unless due to force majeure, exceed a period of two (a) Owners of private agricultural lands or other persons duly
(2) years, the land shall be subject to escheat proceedings. authorized by the landowner;
Any person found guilty of premature or illegal conversion shall (b) Beneficiaries of the agrarian reform program after the lapse
be penalized with imprisonment of two (2) to six (6) years, or a of five (5) years from award, reckoned from the date of issuance
fine equivalent to one hundred percent (100%) of the of the Certificate of Land Ownership Award (CLOA), and who
government's investment cost, or both, at the discretion of the have fully paid their obligations and are qualified under these
court, and an accessory penalty of forfeiture of the land and any Rules, or persons duly authorized by them; or,
improvement thereon. (c) Government agencies, including government-owned or
In addition, the DAR may impose the following penalties, after controlled corporations, and local government units, which own
determining, in an administrative proceedings, that violation of agricultural land as their patrimonial property.
this law has been committed:
a) Cancellation or withdrawal of the authorization for land use Sec10: Documentary Requirements. —xxx
conversion; and Sec11: Filing of Application. — The procedures in applying for
b) Blacklisting, or automatic disapproval of pending and conversion are as follows:
subsequent conversion applications that they may file with the (a) The applicant secures application form either from the DAR
DAR. Regional Office or the DAR Central Office.
(b) The applicant accomplishes the application and
secures/prepares the documents required under Section 10
Department of Agrarian Reform Administrative Order No.1 hereof.
(1999) - III (f) and (k), 10 NAR 398 (c) The applicant files three (3) sets of the application under oath
(inclusive of original) together with the required documents
ARTICLE III Conversion Procedures before the Regional Office or the Central Office.
Sec8: Criteria for Conversion. — The following criteria shall (d) The receiving office shall review the completeness of the
guide the resolution of applications for conversion: application. If found complete, the applicant shall be advised to
(a) Conversion may be allowed if the land subject of application is pay the filing fee and post the cash bond prescribed herein.
not among those considered non-negotiable for conversion as Applications with incomplete requirements shall not be
provided in Section 4 of these Rules; accepted.

The Apocryphal Maggots:


Rainier, Chrisgel, Corina, Geoffry, Grace and Sylvie Blanche
The Flibbertigibbet Worms:
Golda, Gladys and Melyjane
CA VE AT: By simply r ea ding this re vi ew er a t the end o f the sem este r wi ll (hopef ully) guar antee you r pas sing this cou rse. D rink mode rat ely .
Amusin S C A r y
gly
THE C2005 LOCAL GOVERNMENT REVIEWER - 82 -
Sec12: Filing Fees. — xxx the government in the event actual conversion activities are
Sec13: Processing of Application. — conducted by the applicant prior to approval of the application.
(a) Posting of Notice for 2 by MARO upon receipt of Notice of The forfeiture shall be without prejudice to the filing of criminal
application from central office or regional office. charges against those responsible for the premature conversion.
(b) Field Investigation to be conducted on the property on the (c) On the other hand, the performance bond shall be posted
eighth day from posting of the notice of application and shall be within five (5) days from issuance of the order of conversion in
completed within seven (7) days from start of the inspection and the form of either of the following:
the report shall be submitted within three (3) days from (i) Cash, manager's check, cashier's check, irrevocable letter of
completion thereof. credit, bank draft equivalent to two point five percent (2.5%) of
(c) Deliberation and Approval of Application on the 20 th day from the total zonal value of the land; or
posting of the notice and after completion of inspection by the (ii) Bank guarantee equivalent to five percent (5%) of the total
Regional Office/Central Office. zonal value of the land; or
The recommendation on the application for conversion shall be (iii) Surety equivalent to fifteen percent (15%) of the total zonal
forwarded to the Regional Director, the Undersecretary value of the land.
concerned, or the Secretary, as the case may be, within ten (10) The performance bond shall be in favor of DAR to guarantee the
days from the date of deliberation. payment of the amount of security as penalty in the event it is
Within ten (10) days from the submission of the established that the applicant/developer is in default of their
recommendation, the approving authority shall resolve the obligations under the order of conversion. It shall be co-
application furnishing copies thereof to the applicant and the terminous with the final completion of the project and shall be
oppositors, if any. forfeited in favor of the government in case of violation of any of
the conditions of the conversion order.
Sec14: Processing of Applications Involving Priority
Development Areas. — The following steps shall be observed Sec16: Disturbance Compensation. —
in the processing of applications involving RAICs/RICs, (a) Disturbance compensation, in cash or in kind or both, shall be
socialized housing, tourism development, and PEZA-approved paid by the landowner or the developer, as may be appropriate,
ECOZONES: to tenants, farmworkers, as bona fide occupants to be affected
(a) The applicant submits the duly accomplished application by the conversion in such amounts or under such terms as may
form together with the supporting documents to the Regional be mutually agreed upon between them and the landowner or
Office/Central Office. Only applications with complete the developer, but which shall not be less than five (5) times the
requirements shall be accepted and given due course. average of the gross harvests on their landholding during the
(b) The Regional Office/Central Office shall immediately prepare last five (5) preceding calendar years, pursuant to Section 36 of
the Notice of Application and transmit the same to the DAR RA 3844, as amended by Section 7 of RA 6389, particularly in
Municipal Office for posting. the case of tenants.
(c) The MARO posts the Notice of Application within three (3) (b) Compensation in kind may consist of free housing, homelots,
days from receipt thereof in two (2) conspicuous places in the employment, and other benefits. The DAR shall approve the
municipality, in the barangay(s) where the property is located, terms of any agreement for the payment of disturbance
and in the property itself. compensation and monitor compliance therewith. In no case
(d) An ocular inspection shall be conducted by a team assigned shall compliance with the terms and conditions thereof extend
for the purpose within five (5) days from acceptance of the beyond sixty (60) days from the date of approval of the
application. The inspection shall be completed within three (3) application for conversion.
days. (c) In the event the parties do not agree on the amount of
(e) The team shall submit its report together with a draft order disturbance compensation, the issue may be brought by either
involving the application within three (3) days from inspection. of them before the DAR Adjudication Board for resolution
(f) The approving authority shall resolve the application within pursuant to existing rules.
two (2) days from receipt of the draft order. EAICTS Sec17: Role of PARC Land Use Technical Committee. — xxx

Sec15: Cash Bond and Performance Bond. — AND NO W F OR SIR’ S FAVORI TE CASE S
(a) To guarantee that the application shall not undertake premature
Years later, we’re still very much upset.
conversion pending consideration of the application and ensure
But before that:
faithful compliance with the conditions of the conversion order
by the applicant/developer, a cash bond and a performance
bond shall be required pursuant to Section 35, Chapter 6, Book Fusion Sex (Part 3)
IV of the Administrative Code of 1987.
(b) The cash bond shall be posted upon filing of the application The most important aspects of this apparently exquisite
equivalent to two point five percent (2.5%) of the total zonal relationship-experience are that it is totally in the present. There
value of the land. It shall be refundable upon issuance of the are no strings attached and no future plans. The male feels
order of conversion or convertible into performance bond at the sufficiently safe and accepted. He can become emotionally
applicant's option. The cash bond shall be forfeited in favor of defenseless and allow a total flow of blocked and typically

The Apocryphal Maggots:


Rainier, Chrisgel, Corina, Geoffry, Grace and Sylvie Blanche
The Flibbertigibbet Worms:
Golda, Gladys and Melyjane
CA VE AT: By simply r ea ding this re vi ew er a t the end o f the sem este r wi ll (hopef ully) guar antee you r pas sing this cou rse. D rink mode rat ely .
Amusin S C A r y
gly
THE C2005 LOCAL GOVERNMENT REVIEWER - 83 -
controlled feelings to emerge. In most cases that I have heard of, Thus, NQSRMDC file application for conversion. The said
such relationships did not gain permanence. They never had to industrial area will be composed of the Development Academy of
meet the long range test of “reality.” Mindanao, Bukidnon Agro-Industrial Park, Forest development and
support facilities (hotels, restaurants, dorms and housing project)
I have also heard of a few instances of fusion sex occurring for a
man who has been in a relationship with a woman for many
In 1994:
years. Usually it seems to happen for him after a personal DAR, thru sec. Garilao, issued an Order denying application for
growth crisis which has allowed him to achieve new levels of conversion [it was prime agricultural land!!!] and instead placed the
awareness. In these periods he is able to experience new levels same under compulsory coverage of CARP. It also directed its
of trust leading to greater self-expression which then allows him distribution to qualified beneficiaries. DAR Secretary ordered Reg.
to surrender himself more completely to the sexual moment and Dir. to proceed with the compulsory acquisition and distribution.
therefore to experience the ecstasy of fusion sex. Gov. Carlos Fortich appealed the order of denial to the Office of
The experience of fusion sex is certainly deserving of further the President (OP).
exploration. It may very well have been experienced by men Pres. Asst. for Mindanao and DILG recommended the conversion
under conditions quite dissimilar to the ones outlined here, and of the land to the President and requested that the enforcement of
I don’t want to imply that the dimensions I’ve discussed are the the DAR Sec’s Order be put on hold.
only ones. However, hearing descriptions of this experience
In 1995:
leads me to believe that most men have not experienced nearly
NQSRMDC filed with CA petition for certiorari, prohibition to
the ecstatic potential of their sexuality. While I am not prevent the enforcement of DAR Secretary’s Order.
suggesting that the male set fusion sex as a constant standard of CA issued Resolution ordering parties to observe status quo
expectation, description of the experience of fusion sex certainly pending case resolution.
do contain important implications for the untapped sexual
potential of the male. (end) In 1996
OP, through Executive Secretary Ruben Torres issued an Order on
Fortich vs. Corona (1998) 289scra624 March 29, 1996, (OP Order ‘96 for brevity) in favor the conversion.
It reversed DAR Secretary Garilao’s decision which denied the
[notes in italics kasi daming side comments si sir]
conversion.
Facts:
In 1991: • conversion would bring about real development [not
DAR placed the 144-hectare property owned by NQSRMDC significant]
(Norberto Quisumbing S. Management and Development
Corporation) under compulsory acquisition for P2.38M. Such • the land is not irrigated since it was for 10 yrs planted with
property was leased as pineapple plantation to Del Monte for 10 pineapples [DAR was the fact-finding body if said land is
yrs. irrigated]
NQSRMDC resisted and DARAB (DAR Adjudication Board)
ordered DAR Region X Director to desist from the activity til further
• the notice of compulsory acquisition covering land since 1991
orders. was null and void as land could not be subjected to such cos
Regional Dir ignored DARAB order and issued memorandum it was still leased to Del Monte [lease had already expired]
directing Landbank to open trust account for P2.38M in • no beneficiaries since the land was not tenanted [CLOAS
NQSRMDC and to conduct summary proceedings for the just were already cancelled]
compensation of the property. • Sec 20 LGCode grants LGU power to re-classify land.
NQRSRMDC filed omnibus motion with DARAB which ordered
DAR Reg. Dir to comply with the previous order and nullified the
DAR filed motion for recon (MFR) of the OP Order ’96 on May 20,
Director’s memorandum.
1996.
Landbank complied with the DARAB order.
NQSRMDC discovered that title over the subject property was no
longer in its name. DAR had caused the cancellation of their title in
In 1993:
1995 and had caused the issuance of Certificates of Land
Prov’l Development Council (PDC) of Bukidnon passed Resolution
Ownership Award (CLOA) and had it registered in the name of 137
6 designating certain areas where the property is located as
farmer-beneficiaries.
Bukidnon Agro-Industrial Zones.
Pursuant to Sec 20 of LGCode, the Sangguniang Bayan of
In 1997
Sumilao, Bukidnon enacted Ordinance No. 24 converting or re-
NQSRMDC filed complaint with RTC for the cancellation and
classifying the 144-hectare land from agricultural to
annulment of title. RTC issued TRO and Writ of Prelim Injunction
industrial/institutional to attract investors who can provide more
restraining DAR and others from occupying the said land.
jobs and raise income of its people.
Meanwhile, on June 23, 1997 OP, issued an Order (OP Order ’97
The Ordinance was approved by Bukidnon Provincial Land Use
for brevity) thru Sec Torres, denying MFR of DAR for having been
Committee. Bukidnon provincial Board expressed support through
Resolution No. 94-95

The Apocryphal Maggots:


Rainier, Chrisgel, Corina, Geoffry, Grace and Sylvie Blanche
The Flibbertigibbet Worms:
Golda, Gladys and Melyjane
CA VE AT: By simply r ea ding this re vi ew er a t the end o f the sem este r wi ll (hopef ully) guar antee you r pas sing this cou rse. D rink mode rat ely .
Amusin S C A r y
gly
THE C2005 LOCAL GOVERNMENT REVIEWER - 84 -
filed beyond the reglementary period of 15 days. The Order further
stated that the OP Order ’96 had become final and executory. (Martinez and Mendoza deny; Puno and Melo to grant MR)
Some farmers began their hunger strike in front of DAR compound Motions for reconsideration (MR) by respondents and applicants
to protest the OP Order ‘96. Pres. Ramos held dialogue with them for intervention seeking reversal of April24, 1998 SC Decision
and created an 8-man Fact Finding Task Force (FFTF). which:
- nullified “win-win” Resolution (November7, 1997) issued by
OP resolved the strikers protest by issuing the “WIN-WIN” the Office of the President (OP) and
Resolution penned by Deputy exectutive Secretary Renato C. - denied applicants’ Motion for Leave to Intervene
Corona. SC struck down as void the act of OP in reopening the case (OP
• NQSRMDC’s application for conversion is approved only for 96-C 6424) through issuance of “win-win” Resolution which
44-hectare portion substantially modified its March29, 1996 OP Decision that had
• The remaining 100 hectare suitable for agriculture is long become final and executory – as a gross disregard of rules
distributed to qualified farmer-beneficiaries. and basic legal precept that afford finality to administrative
Gov. Fortich and NQRMDC filed petition for Certiorari in SC with determinations.
TRO against Corona and DAR Sec Garilao. They argue that: Same March29, 1996 OP Decision was declared by same office as
• WIN-Win Resolution was a political decision prompted by the final and executory in its June23, 1997 Order which denied
hunger strike by fake farmer beneficiaries respondent DAR’s MR of OP decision for having been filed beyond
the 15-day reglementary period.
• Corona committed grave abuse of discretion (GAOD) as the
SC deemed it necessary to write an extended resolution (when a
WIN-WIN Reso was patently illegal and contrary to law
minute one denying MRs with finality would have been sufficient) in
Corona, SolGen:
view of the wide publicity and media coverage the case has
• proper remedy is certiorari on appeal in CA generated…
• file motion for recon first before certiorari
• NQSRMDC is guilty of forum shopping As to Intervention Applicants’ MR:
Farmer-beneficiaries filed Motion to Intervene claiming they are 1. Other issues which they describe as “substantial”
real parties in interest. SC: These have thoroughly and properly been disposed of in the
March OP Decision. Factual findings of admin agencies which
MAIN ISSUE: WON the final executory decision of OP can still have acquired expertise in their field are binding and conclusive.
be substantially modified by the WIN-WIN resolution? – NO Office of President presumed competent in matters within its
In appealing to OP, sec 7 of Admin Order No. 18 provides that domain.
decisions/resolutions/orders of OP become final after the lapse of
15 days from receipt of a copy thereof by the parties unless a
a. Whether subject land is considered prime agricultural land
motion for recon is filed within that period. Sec 9 provides ROC with irrigation facility  “while it is true that there is, indeed,
shall apply in a suppletory character. an agricultural facility in the area, it merely passes thru the
property (as a right of way) to provide water to the ricelands
When OP issued OP Order ’97 declaring OP Order ’96 final and located on the lower portion. Land itself is not irrigated as it
executory, no one has seasonably filed an MFR. Thus the OP has was, for years, planted with pineapple by Phil. Packing Corp.
lost its jurisdiction to re-open the case, more so modify its decision. b. Whether land has long been covered by a Notice of
Compulsory Acquisition (NCA)  declared null and void by
Having lost its jurisdiction, OP has no more authority to entertain DAR Adjud. Board deciding in favor of NQSR that property
the 2nd MFR filed by DAR secretary, which 2 nd MFR became the could not validly be subject of compulsory acquisition until
basis for the WIN-WIN resolution. expiration of lease contract with Del Monte… and
Quisumbing family has already contributed substantially…
Only one MFR is allowed from the OP Order ’96 and even if a 2 nd
MFR is permitted in exceptionally meritorious cases, still the 2nd c. whether land is tenanted, and if not, whether the applicants
MFR would not have been entertained since the 1 st MFR was not for intervention are qualified to become beneficiaries thereof
seasonably filed. Thus OP Order ’96 had lapsed into finality.  there are no beneficiaries for the land is not tenanted
d. whether the Sangguniang Bayan of Sumilao has legal
The orderly administration of justice requires that the authority to reclassify the land into industrial/institutional use
judgment/resolutions of a court of quasi-judicial body must reach a  purpose of law granting autonomy to LGUs in the
point of finality set by the law, rules and regulation. management of their local affairs and autonomy includes
power to convert portions of their agricultural lands and
WIN-WIN Resolution is void – “a lawless thing which can be provide for the manner of their utilization and disposition to
treated as an outlaw and slain at sight, or ignored wherever and enable them to attain their fullest development as self-reliant
whenever it exhibits its head.” communities.
To be sure, conversion would open great opportunities for
employment and bring about real development in the area
Fortich vs. Corona (Nov.17 1998) 298scra678 towards a sustained economic growth of the municipality.

The Apocryphal Maggots:


Rainier, Chrisgel, Corina, Geoffry, Grace and Sylvie Blanche
The Flibbertigibbet Worms:
Golda, Gladys and Melyjane
CA VE AT: By simply r ea ding this re vi ew er a t the end o f the sem este r wi ll (hopef ully) guar antee you r pas sing this cou rse. D rink mode rat ely .
Amusin S C A r y
gly
THE C2005 LOCAL GOVERNMENT REVIEWER - 85 -
Distributing the land to would-be beneficiaries (not even
tenants, as there are none) does not guarantee such benefits 2. Petitioners are estopped from assailing authority of OP to re-
open case and resolve it based on the report of PFTF.
2. In the interest of justice, have mercy please… - they participated in the process conducted by task force
SC: Affirmation of finality of OP Decision is precisely pro-poor without raising any objection – bars them from raising any
considering that more of the impoverished members of society will jurisdictional infirmity
be benefited by the agro-economic development of the land. Plan - Petitioners (Bukidnon Governor Fortich and Sumilao Mayor
was supposed to have following components: Baula) were named members of task force
a. Development Academy of Mindanao… - NSQR did not question authority of President to constitute
b. Bukidnon Agro-Industrial Park… food processing task force despite its express adherence to Torres declaration
c. Forest development… open spaces and parks for recreation as to finality of his decision – confident that its interests would
d. Support facilities… hotel, resto, dorm, and housing project be promoted and protected by Fortich and Baula
Project was supported by Mun. of Sumilao (enacting Ord.24) and “Win-win” resolution by OP being adverse to petitioners, they now
Bukidnon Prov’l Brd. on a Joint Committee Report. assail authority of President to modify Torres decision, but
- DTI, Bukidnon Provincial Office adopted it as one of its circumstances show application of estoppel.
flagship projects…
- recommended by Provincial Development Council of 3. Considering special circumstances of case, it would better
Bukidnon, DAR municipal and regional office, Regional serve ends of justice to obtain a definitive resolution of issues
office of DENR which issued environmental compliance in instant petition and remand to CA where jurisdiction over
certificate… appeal lies, also where two or more cases involving Sumilao
- National Irrigation Admin interposed NO OBJECTION property is pending
- Kisolon-San Vicente Irrigators Multi Purpose Coop
interposed no objection – as it will provide more DGNotes:
economic benefits SC doctrine: “fealty to procedure is in the bill of rights.”
- During a public consultation held, people of affected - where in the (expletives deleted) Bill of Rights is it
barangay rallied behind their respective officials in stated??!
endorsing the project.
MRs denied. why are we upset?
The respondents allege that the Torres decision was without legal
Separate Opinion: PUNO – to grant MRs and remand case to CA
basis. But the SC did not pass upon this allegation because (oops)
1. President has suspended the 15day rule.
they were late and missed the deadline.
 Closer scrutiny of records reveals that the 15day rule was
suspended by the President when he constituted (on Oct15, 1997
And why did the SC cite the Camarines case? It was never
or 6months after promulgation of Torres decision) the Presidential
Fact-Finding Task Force (PFTF) to conduct a comprehensive provided therein that LGUs have the power to convert. It only
review of the proper lands use of the Sumilao property. held that LGUs have the power to classify. Court cannot therefore
Even though Torres had already denied 1st MR, President treated use said case as a basis to say that the question regarding the
case as still open and stated in his memorandum that findings of power of the LGUs to convert land is no longer novel.
PFTF “will be inputs to resolution of case now pending at OP.”
President took cognizance of special circumstances surrounding
tardy filing by DAR of its MR: Fortich vs. Corona (Aug.19 1999) 312scra751
- DAR lawyers assigned received Torres decision only after
lapse of reglementary 15-day period for appeal Respondents and intervenors in their MR:
- Copy of decision intended for them was passed from one
office to another
- argue that Nov17, 1998 SC Resolution where vote turn-out
- DAR would not be the only to lose a significant case because was 2 to 2 on separate MRs of earlier April24, 1998 SC
of bureaucratic lapses Decision  did not effectively resolve MRs inasmuch as the
 President as admin head is vested by Admin Code of 1987 to matter should have been referred to Court en banc (pursuant
promulgate rules relating to governmental operation including to ArtVIII, Sec4(3) of Consti)
admin procedure, also with power to suspend its effectivity when - assail Jan27, 1999 SC Resolution which noted without
they hamper, defeat or undermine effective enforcement of laws. action the intervenors’ MR with Motion to Refer Matter to
 This act is also under the corollary principles of presidential Court En Banc” filed on Dec3, 1998:
power of control and qualified political agency. o movants have no legal personality to further seek
 This act is not arbitrary considering the fundamental issues the redress before SC after denial of their motion for leave to
problem raises – conflict between land reform and the intervene in the April24, 1998 Decision
industrialization of the countryside – resolution of which would o their subsequent MR of said decision (praying to resolve
have far-reaching implications on success of land reform programs motion by Court en banc) also denied in Nov17, 1998
and peace or rebellion in the countryside. Resolution

The Apocryphal Maggots:


Rainier, Chrisgel, Corina, Geoffry, Grace and Sylvie Blanche
The Flibbertigibbet Worms:
Golda, Gladys and Melyjane
CA VE AT: By simply r ea ding this re vi ew er a t the end o f the sem este r wi ll (hopef ully) guar antee you r pas sing this cou rse. D rink mode rat ely .
Amusin S C A r y
gly
THE C2005 LOCAL GOVERNMENT REVIEWER - 86 -
o MR of Dec3, 1998 in the nature of a 2nd MR which is a have already been raised before and passed upon by SC in the
Resolution.
forbidden motion (1997 Rules of Civil Procedure)
o All respondents in the case did not anymore join them in
Reminiscing the Nov17, 1998 Resolution:
seeking reconsideration of Nov17, 1998 Resolution
a. “Win-win” Resolution is void and of no legal effect considering
that OP Decision had already become final and executory
1. Should the court decide en banc pursuant to Consti?
even prior to the filing of MRs as basis of “win-win”
SC (Ynares-Santiago as ponente): NO.
Resolution.
[sir: why the hell is Ynares-Santiago sitting in the 2nd
b. While it may be true that on its face the nullification of the
division?!]
“win-win” Resolution was grounded on a procedural rule
pertaining to the reglementary period to appeal or move for
Careful rea0ding of ArtVIII, Sec4(3) of Consti reveals intention of
reconsideration, the underlying consideration was the
framers to draw a distinction between cases and matters. Cases
protection of the substantive rights of petitioners as well as
are “decided”, while Matters, which include motions, are
the people who stand to be benefited by development of the
“resolved.”  The word “decided” must refer to “cases”; while the property, which was vested by the OP Decision. The issue is
word “resolved” must refer to “matters”, applying the rule of therefore not a question of technicality but of substance and
reddendo singular singulis, as is true in the interpretation of other merit.
provisions of the Constitution where these words appear. c. As to legal standing of intervenors, see Martinez decision…
It is clear that only cases are referred to the Court en banc for
decision whenever the required number of votes is not obtained. Separate Opinion: MELO – votes with Puno that case be referred
Rule does not apply where the required votes is not obtained in the to CA for further proceedings.
resolution of MR. Hence, second sentence of the provision speaks Bound to abide by Court En Banc’s Resolution 99-1-09-SC which
only of “case” and not “matter”. settled issue of an even (2-2) vote in a division – still constrained to
REASON: The provision pertains to disposition of cases by a vote with majority in denying all motions:
division. If there is a tie in the voting, there is no decision. The only 1. ArtVIII, Sec4(3) mandate that cases heard by a division when
way to dispose of the case then is to refer it to the Court en banc. the required majority of at least 3votes in the division is not
If a case has already been decided by the division and the losing obtained are to be heard and decided by Court En Banc.
party files MR, failure of the division to resolve the motion because
 1986 Constitutional Commission deliberations disclose that if
of a tie in the voting does not leave the case undecided. There is
case is not decided in a division by a majority vote, it goes to the
still the decision which must stand in view of the failure of the
Court En Banc and not to a larger division. Moreover, elevation to
members of the decision to muster the necessary vote for its
En Banc shall be automatic.
reconsideration.
Explicit is the requirement that at least 3members must concur in
Plainly, if the voting results in a tie, MR is lost. Decision is not
any case or matter heard by a division. Failing thus, case or matter
reconsidered and must therefore be deemed affirmed, as such was
will have to be decided by the Court En Banc. This also applies in
ruling of SC in the Nov17, 1998 Resolution.
a division of 5 but only 4members can vote (1 being inhibited or
unable to participate).
2. Should be decided En Banc because the issues submitted in
their separate motions are of first impressions… That Nov.17,
2. Distinction between “cases” and “matters” is not true all the
1998 Resolution did not dispose of the earlier MRs of April24,
time.
1998 Decision.
It can only be true in original cases, as opposed to appealed
SC: Justice Martinez in the Nov17, 1998 Resolution already
cases, filed with SC. Due to hierarchy of courts doctrine, only
expressed that the issues presented were no longer novel…
original cases taken cognizance by SC are those wherein it has
having been decided in case Province of Camarines Sur vs. CA.
exclusive jurisdiction. Such cases are required to be held En Banc.
The contention is flawed. The present MRs necessarily partake of
Cases taken cognizance by division are either petitions under
the nature of a 2nd MR which, according to clear and unambiguous
Rule45 or 65.
language of Rule56, Sec4 in relation to Rule52, Sec2 of Rules of
Under Rule45, appeal by way of petition for review on certiorari is
Civil Procedure, is prohibited.
not a matter of right. Should there be a tie in the voting on
deliberation of a “case” by the division, although apparently no
There are exceptional cases when Court may enforce a 2 nd MR,
action is passed, a decision may still be rendered – DENIED due
such as where there are extraordinarily persuasive reasons. But it
course and DISMISSED, which is definitely in consonant with
has been ruled that such must be filed with express leave of court
majority’s line of reasoning in the 2-2 vote on MRs.
first obtained.
But why is it that, 2-2 vote in deliberation of “case” at first instance
Movants failed to ask for prior leave of court. Moreover, They have
should still be referred to the Court En banc?  the express
been unable to show that there are exceptional reasons to give
provision of Consti requires a vote of at least 3 justices for there to
due course to their 2nd MRs.
be a valid and binding decision of the Court. It should equally be
appled to MRs. In all instances, whether it be in the deliberations
 Stripped of arguments for referral to Court en banc, the present
of a cases at first instance or on MR, a division having a 2-2 vote
motions are nothing more but rehashes of MRs which have been
cannot pass action.
denied in Nov17, 1998 Resolution. Allegations contained therein

The Apocryphal Maggots:


Rainier, Chrisgel, Corina, Geoffry, Grace and Sylvie Blanche
The Flibbertigibbet Worms:
Golda, Gladys and Melyjane
CA VE AT: By simply r ea ding this re vi ew er a t the end o f the sem este r wi ll (hopef ully) guar antee you r pas sing this cou rse. D rink mode rat ely .
Amusin S C A r y
gly
THE C2005 LOCAL GOVERNMENT REVIEWER - 87 -
Held: NO
 The requirement of 3votes equally applies to MRs because the On at least 2 occasions, the SC, sitting En Banc has refused to
provision contemplates “cases” or “matters” (with no material take cognizance of this case:
distinction insofar as divisions are concerened) heard by a division, 1. Dec 14, 1998 – case was referred to the En Banc and the ff
and MR cannot be divorced from decision in a case that it seeks to day was referred back to the 2nd Division, indicating a clear
be reconsidered. denial of the motion to refer the matter to the Court En Banc
Consequently, if required minimum majority of 3votes I not met,
matter of MR has to be heard by Court En Banc, as mandated by 2. Dec 7, 1999 – the Court En Banc declined to accept the case,
the Consti. despite repeated motions for automatic referral by
To argue that MR is not a “case” but only a “matter” which does not respondents and intervenors
concern a case is to engage in a lot of unfounded hairsplitting.
The controlling resolution is A.M. 99-1-09-SC,”In the matter
3. The theory of leaving the issue hanging on a 2-2vote or any clarifying the rule resolving motions for reconsideration” which
even vote may be sustained only in cases where there is no states:
recourse to a higher assemblage. There is still recourse to the A MOTION FOR RECON OF DECISION OR RESOLUTUION
Court En Banc in CAB… more so, being expressly directed OF THE COURT EN BANC OR OF A DIVISION MAY BE
by the Constitution. GRANTED UPON A VOTE OF A MAJORITY OF THE
MEMBERS OF THE EN BANC OR OF A DIVISION, AS THE
CASE MAY BE, WHO ACTUALLY TOOK PART IN THE
Fortich vs. Corona (Dec, 1999) GR131457 DELIBERATION OF THE MOTION
IF THE VOTING RESULTS IN A TIE, THE MOTION FOR
Facts: RECON IS DEEMED DENIED
Respondents and intervenors begged to file a “Joint First Motion This Resolution was applied by in the case of Yale Land
for Reconsideration of the Denial of the Motion for Automatic Development Corporation v Pedro Caragao. The foregoing is
Referral of the Case to the Supreme Court Sitting En Banc in View consistent with the interpretation of the SC in the Aug 1999
of a Court Division’s 2-2 Vote and Clarificatory Motion”, where they Resolution.
reiterate their earlier contention that the case should be referred to
the SC En Banc pursuant to Art 8 Sec 4 par 3 of the Constitution There is nothing repugnant in this rule to Art 8 Sec 4(3) of the
and that it involves several novel questions of law invoking SC Constitution. Only the decision of cases require the concurrence of
Resolution February 9, 1993 in Bar Matter 209. They also argue at least 3 members of the SC. Insofar as the resolutions of MRs
that the SC Resolution of the case on Aug 1999 [the 3rdtich case] are concerned, the failure to muster the required vote of 3 simply
reflected the interpretation of that Const provision of the ponente means that MR is lost and decision stands. There is nothing in that
alone. Const provision that requires an automatic referral to the SC En
Banc in such instance. Even the deliberations of the 1986 ConCom
Petitioners then filed a Petition to Cite for Indirect Contempt do not equivocably show that the resolutions of MRs, where the
because the DAR and applicants for intervention for filing 2 nd, 3rd required vote of 3 is not obtained, should be referred to the En
and, in the case of intervenors, 4th motions for reconsideration in Banc.
violation of the admonition that no further pleading or motion will be
entertained in this case. Petitioners posit that since all 4 Justices of
the Special 2ND Division concurred in the result of the Aug 1999 Roxas & Co., Inc. vs. CA (1999)
Resolution, there is no more 2-2 vote but a 4-0 vote.
Facts:
Citing Santiago v COMELEC and Krivenko v Register of Deeds, Petitioner is the registered owner of 3 haciendas, namely,
they also contend that the filing of a Motion for Reconsideration Haciendas Palico, Banilad and Caylaway, all located in Nasugbu,
(MR) does not vacate the decision sought. Simply put, they state Batangas.
that the 2-2 vote resulted in the denial of the MRS and the In February 1986, President Aquino issued Proclamation No. 3
affirmation of the 24 April 1998 Decision. Respondents rely heavily promulgating a Provisional Constitution. In the exercise of this
on Justice Melo’s separate opinion in assailing the SC’s legislative power the President signed on July 22, 1987,
interpretation of Art 8 Sec 4(3). They also invoke pertinent Proclamation No 131 instituting a Comprehensive Agrarian Reform
provisions of the deliberations of the 1986 ConCom in support of Program and Executive Order No. 229 providing the mechanisms
their contention and where the concurrence of at least 3 members necessary to initially implement the program.
of the SC is not obtained, the case is automatically referred to the On July 27, 1987, the Congress of the Philippines formally
SC En Banc. convened and took over legislative power from the President. This
Congress passed Republic Act No. 6657, CARL of 1988.
Issue: WON a MR decided by a 2-2 vote by a division should Before the law's effectivity, on May 6, 1988, petitioner filed with
be referred to the Court En Banc for resolution respondent DAR a voluntary offer to sell Hacienda Caylaway
pursuant to the provisions of E.O. No. 229. Haciendas Palico and

The Apocryphal Maggots:


Rainier, Chrisgel, Corina, Geoffry, Grace and Sylvie Blanche
The Flibbertigibbet Worms:
Golda, Gladys and Melyjane
CA VE AT: By simply r ea ding this re vi ew er a t the end o f the sem este r wi ll (hopef ully) guar antee you r pas sing this cou rse. D rink mode rat ely .
Amusin S C A r y
gly
THE C2005 LOCAL GOVERNMENT REVIEWER - 88 -
Banilad were later placed under compulsory acquisition by address the Notice may be sent by personal delivery or registered
respondent DAR in accordance with the CARL. mail, the law does not distinguish. The DAR Administrative Orders
On August 24, 1993, petitioner instituted a case with respondent also do not distinguish. In the proceedings before the DAR the
DAR Adjudication Board (DARAB) praying for the cancellation of distinction between natural and juridical persons in the sending of
the CLOA's issued by DAR in the name of several persons. notices may be found in the Revised Rules of Procedure of the
Petitioner alleged that the Municipality of Nasugbu, where the DAR Adjudication Board (DARAB). Service of pleadings before the
haciendas are located, had been declared a tourist zone, that the DARAB is governed by Section 6, Rule V of the DARAB Revised
land is not suitable for agricultural production, and that the Rules of Procedure.
Sangguniang Bayan of Nasugbu had reclassified the land to non-
agricultural. Jaime Pimentel is not the president, manager, secretary,
In a Resolution dated October 14, 1993, respondent DARAB held cashier or director of petitioner corporation. Is he, as
that the case involved the prejudicial question of whether the administrator of the two Haciendas, considered an agent of
property was subject to agrarian reform, hence, this question the corporation?
should be submitted to the Office of the Secretary of Agrarian
Reform for determination. The purpose of all rules for service of process on a corporation is
On October 29, 1993, petitioner filed with the CA. It questioned the to make it reasonably certain that the corporation will receive
expropriation of its properties under the CARL and the denial of prompt and proper notice in an action against it. Service must be
due process in the acquisition of its landholdings. made on a representative so integrated with the corporation as to
Meanwhile, the petition for conversion of the three hectares was make it a priori supposable that he will realize his responsibilities
denied by the MARO on November 8, 1993. Petitioner's petition and know what he should do with any legal papers served on him,
was dismissed by the Court of Appeals on April 28, 1994. and bring home to the corporation notice of the filing of the action.
Petitioner moved for reconsideration but the motion was denied on
January 17, 1997 by CA. Petitioner's evidence does not show the official duties of Jaime
Pimentel as administrator of petitioner's haciendas. The evidence
Issue: WON the acquisition proceedings over the three does not indicate whether Pimentel's duties is so integrated with
haciendas were valid and in accordance with law the corporation that he would immediately realize his
Held: NO, there was notice provided by law responsibilities and know what he should do with any legal papers
In the CAB, DAR claims that it sent a letter of invitation to served on him.
petitioner. The invitation was received on the same day it was sent
as indicated by a signature and the date received at the bottom left At the time the notices were sent and the preliminary conference
corner of said invitation. conducted, petitioner's principal place of business was listed in
respondent DAR's records as "Soriano Bldg., Plaza Cervantes,
With regard to Hacienda Banilad, respondent DAR claims that the Manila," and "7th Flr. Cacho-Gonzales Bldg., 101 Aguirre St.,
administrator of Hacienda Banilad, was notified and sent an Makati, Metro Manila." Pimentel did not hold office at the principal
invitation to the conference and actually attended the conference place of business of petitioner. Neither did he exercise his
and signed the Minutes of the meeting on behalf of petitioner functions in Plaza Cervantes, Manila nor in Cacho-Gonzales Bldg.,
corporation. The Minutes was also signed by the representatives of Makati, Metro Manila. He performed his official functions and
the BARC, the LBP and farmer beneficiaries. No letter of invitation actually resided in the haciendas in Nasugbu, Batangas, a place
was sent or conference meeting held with respect to Hacienda over two hundred kilometers away from Metro Manila.
Caylaway because it was subject to a Voluntary Offer to Sell to
respondent DAR. Curiously, respondent DAR had information of the address of
petitioner's principal place of business. The Notices of Acquisition
When respondent DAR, through the Municipal Agrarian Reform over Haciendas Palico and Banilad were addressed to petitioner at
Officer (MARO), sent to the various parties the Notice of Coverage its offices in Manila and Makati. These Notices were sent barely
and invitation to the conference, DAR A.O. No. 12, Series of 1989 three to four months after Pimentel was notified of the preliminary
was already in effect more than a month earlier. conference.

The Operating Procedure in DAR Administrative Order No. 12 Nevertheless, assuming that Pimentel was an agent of petitioner
does not specify how notices or letters of invitation shall be sent to corporation, and the notices and letters of invitation were validly
the landowner, the representatives of the BARC, the LBP, the served on petitioner through him, there is no showing that Pimentel
farmer beneficiaries and other interested parties. The procedure in himself was duly authorized to attend the conference meeting with
the sending of these notices is important to comply with the the MARO, BARC and LBP representatives and farmer
requisites of due process especially when the owner, as in this beneficiaries for purposes of compulsory acquisition of petitioner's
case, is a juridical entity. landholdings. Even respondent DAR's evidence does not indicate
this authority.
The Notice of Acquisition in Sec 16 of the CARL is required to be
sent to the landowner by "personal delivery or registered mail." On the contrary, petitioner claims that it had no knowledge of the
Whether the landowner be a natural or juridical person to whose letter-invitation, hence, could not have given Pimentel the authority

The Apocryphal Maggots:


Rainier, Chrisgel, Corina, Geoffry, Grace and Sylvie Blanche
The Flibbertigibbet Worms:
Golda, Gladys and Melyjane
CA VE AT: By simply r ea ding this re vi ew er a t the end o f the sem este r wi ll (hopef ully) guar antee you r pas sing this cou rse. D rink mode rat ely .
Amusin S C A r y
gly
THE C2005 LOCAL GOVERNMENT REVIEWER - 89 -
to bind it to whatever matters were discussed or agreed upon by beneficiaries. But this does not mean that these requirements may
the parties at the preliminary conference or public hearing. Notably, be dispensed with regard to VOS filed before June 15, 1988.
one year after Pimentel was informed of the preliminary
conference, DAR A.O. No. 9, Series of 1990 was issued and this First of all, the same E.O. 229, like Section 16 of the CARL,
required that the Notice of Coverage must be sent "to the requires that the land, landowner and beneficiaries of the land
landowner concerned or his duly authorized representative." subject to agrarian reform be identified before the notice of
acquisition should be issued. Hacienda Caylaway was voluntarily
Assuming further that petitioner was duly notified of the CARP offered for sale in 1989. The Hacienda has a total area of
coverage of its haciendas, the areas found actually subject to 867.4571 hectares and is covered by four (4) titles. In two separate
CARP were not properly identified before they were taken over by Resolutions both dated January 12, 1989, respondent DAR,
respondent DAR. Respondents insist that the lands were identified through the Regional Director, formally accepted the VOS over two
because they are all registered property and the technical of these four titles. The land covered by the two titles has an area
description in their respective titles specifies their metes and of 855.5257 hectares, but only 648.8544 hectares thereof fell
bounds. Respondents admit at the same time, however, that not all within the coverage of R.A. 6657. Petitioner claims it does not
areas in the haciendas were placed under the comprehensive know where these portions are located.
agrarian reform invariably by reason of elevation or character or
use of the land. Respondent DAR, on the other hand, avers that surveys on the
land covered by the four titles were conducted in 1989, and that
The acquisition of the landholdings did not cover the entire petitioner, as landowner, was not denied participation therein. The
expanse of the two haciendas, but only portions thereof. Hacienda results of the survey and the land valuation summary report,
Palico has an area of 1,024 hectares and only 688.7576 hectares however, do not indicate whether notices to attend the same were
were targetted for acquisition. Hacienda Banilad has an area of actually sent to and received by petitioner or its duly authorized
1,050 hectares but only 964.0688 hectares were subject to CARP. representative.
The haciendas are not entirely agricultural lands. In fact, the
various tax declarations over the haciendas describe the Issue: Assuming the haciendas may be reclassified from
landholdings as "sugarland," and "forest, sugarland, pasture land, agricultural to non-agricultural, whether this court has the
horticulture and woodland." power to rule on this issue.
Held: NO
Under Section 16 of the CARL, the sending of the Notice of At the time petitioner filed its application for conversion, the Rules
Acquisition specifically requires that the land subject to land reform of Procedure governing the processing and approval of
be first identified. The two haciendas in the instant case cover vast applications for land use conversion was the DAR A.O. No. 2,
tracts of land. Before Notices of Acquisition were sent to petitioner, Series of 1990. Under this A.O., the application for conversion is
however, the exact areas of the landholdings were not properly filed with the MARO where the property is located. The MARO
segregated and delineated. Upon receipt of this notice, therefore, reviews the application and its supporting documents and conducts
petitioner corporation had no idea which proportions of its estate field investigation and ocular inspection of the property. The
were subject to compulsory acquisition, which portions it could findings of the MARO are subject to review and evaluation by the
rightfully retain, whether these retained portions were compact or Provincial Agrarian Reform Officer (PARO). The PARO may
contiguous, and which portions were excluded from CARP conduct further filed investigation and submit a supplemental
coverage. Even respondent DAR's evidence does not show that report together with his recommendation to the Regional Agrarian
petitioner, through its duly authorized representative, was notified Reform Officer (RARO) who shall review the same. For lands less
of any ocular inspection and investigation that was to be conducted than five hectares, the RARO shall approve or disapprove
by respondent DAR. Neither is there proof that petitioner was given applications for conversion. For lands exceeding five hectares, the
the opportunity to at least choose and identify its retention areas in RARO shall evaluate the PARO Report and forward the records
those portions to be acquired compulsorily. The right of retention and his report to the Undersecretary for Legal Affairs. Applications
and how this right is exercised, is guaranteed in Section 6 of the over areas exceeding fifty hectares are approved or disapproved
CARL. by the Secretary of Agrarian Reform.

Petitioner was also left in the dark with respect to Hacienda The DAR's mandate over applications for conversion was first laid
Caylaway, which was the subject of a Voluntary Offer to Sell down in Section 4 (j) and Sections 5 (l) of Executive Order No,
(VOS). The VOS was made before the effectivity of CARL. VOS 129-A, Series of 1987 and reiterated in the CARL and
transactions were first governed by DAR Administrative Order No. Memorandum Circular No. 54, Series of 1993 of the Office of the
19, series of 1989, and under this order, all VOS filed before June President.
15, 1988 shall be heard and processed in accordance with the
procedure provided for in Executive Order No. 229. Applications for conversion were initially governed by DAR A.O.
No. 1, Series of 1990 entitled "Revised Rules and Regulations
The E.O. is silent as to the procedure for the identification of the Governing Conversion of Private Agricultural Lands and Non-
land, the notice of coverage and the preliminary conference with Agricultural Uses," and DAR A.O. No. 2, Series of 1990 entitled
the landowner, representatives of the BARC, the LBP and farmer "Rules of Procedure Governing the Processing and Approval of

The Apocryphal Maggots:


Rainier, Chrisgel, Corina, Geoffry, Grace and Sylvie Blanche
The Flibbertigibbet Worms:
Golda, Gladys and Melyjane
CA VE AT: By simply r ea ding this re vi ew er a t the end o f the sem este r wi ll (hopef ully) guar antee you r pas sing this cou rse. D rink mode rat ely .
Amusin S C A r y
gly
THE C2005 LOCAL GOVERNMENT REVIEWER - 90 -
Applications for Land Use Conversion." These A.O.'s and other
implementing guidelines, including Presidential issuances and Finally, failure of respondent DAR to comply with the requisites of
national policies related to land use conversion have been due process in the acquisition proceedings does not give this Court
consolidated in DAR A.O. No. 07, Series of 1997. Under this recent the power to nullify the CLOA's already issued to the farmer
issuance, the guiding principle in land use conversion is: beneficiaries. To assume the power is to short-circuit the
"to preserve prime agricultural lands for food production while, administrative process, which has yet to run its regular course.
at the same time, recognizing the need of the other sectors of Respondent DAR must be given the chance to correct its
society (housing, industry and commerce) for land, when procedural lapses in the acquisition proceedings. In Hacienda
coinciding with the objectives of the Comprehensive Agrarian Palico alone, CLOA's were issued to 177 farmer beneficiaries in
Reform Law to promote social justice, industrialization and 1993. Since then until the present, these farmers have been
the optimum use of land as a national resource for public cultivating their lands. It goes against the basic precepts of justice,
welfare." fairness and equity to deprive these people, through no fault of
their own, of the land they till. Anyhow, the farmer beneficiaries
"Land Use" refers to the manner of utilization of land, including its hold the property in trust for the rightful owner of the land.
allocation, development and management.
THE CASE IS REMANDED TO DAR FOR PROPER
"Land Use Conversion" refers to the act or process of changing the ACQUISITION PROCEEDINGS AND DETERMINATION OF
current use of a piece of agricultural land into some other use as PETITIONER'S APPLICATION FOR CONVERSION
approved by the DAR. The conversion of agricultural land to uses
other than agricultural requires field investigation and conferences [Sorry blockmates, lost ako kung pano to ieedit for the reviewer.]
with the occupants of the land. They involve factual findings and
highly technical matters within the special training and expertise of
the DAR. DAR A.O. No. 7, Series of 1997 lays down with Book Review on A Hand in the Bush: The Fine Art of Vaginal
specificity how the DAR must go about its task. This time, the field Fisting by D. Addington
investigation is not conducted by the MARO but by a special task
force, known as the Center for Land Use Policy Planning and
This book is a wonderful reference to the wide world of fisting. I
Implementation (CLUPPI-DAR Central Office). The procedure is
loved what it did for my sex life, not only do I know how to fist
that once an application for conversion is filed, the CLUPPI
prepares the Notice of Posting. The MARO only posts the notice myself properly, but now I know how to fist others properly. I
and thereafter issues a certificate to the fact of posting. The am so glad that someone finally ventured out and wrote openly
CLUPPI conducts the field investigation and dialogues with the about fisting, I was ashamed to ask people how to do it
applicants and the farmer beneficiaries to ascertain the information properly, but I very well knew that everyone does it, its nothing
necessary for the processing of the application. The Chairman of to be ashamed of. We all fist at one point in our lives. This book
the CLUPPI deliberates on the merits of the investigation report breaks through the barriers and it brings it IN YOUR FACE, and
and recommends the appropriate action. This recommendation is leaves no questions asked. Fisting can be a fun and pleasurable
transmitted to the Regional Director, thru the Undersecretary, or past time if you KNOW how to do it RIGHT. And this book
Secretary of Agrarian Reform. Applications involving more than fifty gives you step by step instructions while doing it, although I do
hectares are approved or disapproved by the Secretary. The advise that you have someone that you feel you can trust to
procedure does not end with the Secretary, however. The Order
either read to you as you are doing it, or have someone perform
provides that the decision of the Secretary may be appealed to the
it for you and tell you step by step what they are doing. This
Office of the President or the Court of Appeals, as the case may
be, viz: enhances the experience and makes it almost more pleasurable.
• "Appeal from the decision of the Undersecretary shall be I am glad that Deborah Addington has chosen to break the
made to the Secretary, and from the Secretary to the Office of taboos of fisting. It is time that this particular sexual experience
the President or the Court of Appeals as the case may be. be brought forth to the public in all its glory.
The mode of appeal motion for reconsideration, and the
appeal fee, from Undersecretary to the Office of the Secretary
shall be the same as that of the Regional Director to the Closure and Opening of Roads
Office of the Secretary."
Indeed, the doctrine of primary jurisdiction does not warrant a court Sec21: Closure and Opening of Roads. —
to arrogate unto itself authority to resolve a controversy the (a) A local government unit may, pursuant to an ordinance,
jurisdiction over which is initially lodged with an administrative body permanently or temporarily close or open any local road, alley,
of special competence. Respondent DAR is in a better position to park, or square falling within its jurisdiction: Provided, however,
resolve petitioner's application for conversion, being primarily the That in case of permanent closure, such ordinance must be
agency possessing the necessary expertise on the matter. The approved by at least two-thirds (2/3) of all the members of the
power to determine whether Haciendas Palico, Banilad and sanggunian, and when necessary, an adequate substitute for
Caylaway are non-agricultural, hence, exempt from the coverage the public facility that is subject to closure is provided. casia
of the CARL lies with the DAR, not with this Court.

The Apocryphal Maggots:


Rainier, Chrisgel, Corina, Geoffry, Grace and Sylvie Blanche
The Flibbertigibbet Worms:
Golda, Gladys and Melyjane
CA VE AT: By simply r ea ding this re vi ew er a t the end o f the sem este r wi ll (hopef ully) guar antee you r pas sing this cou rse. D rink mode rat ely .
Amusin S C A r y
gly
THE C2005 LOCAL GOVERNMENT REVIEWER - 91 -
(b) No such way or place or any part thereof shall be permanently Provincial board, after all, has the duty of maintaining the roads for
closed without making provisions for the maintenance of public the comfort and convenience of the inhabitants of the province.
safety therein. A property thus permanently withdrawn from Also, its authority is inferable from the grant by the national
public use may be used or conveyed for any purpose for which legislature of the funds to the province for the construction of
other real property belonging to the local government unit roads.
concerned may be lawfully used or conveyed: Provided,
however, That no freedom park shall be closed permanently 3) WON petitioner is entitled to damages
without provision for its transfer or relocation to a new site. Held: NO
(c) Any national or local road, alley, park, or square may be General Rule: One whose property does not abut on the closed
temporarily closed during an actual emergency, or fiesta section of the street has no right of compensation for its closure if
celebrations, public rallies, agricultural or industrial fairs, or an he still has reasonable access to the general system of streets.
undertaking of public works and highways, telecommunications, To warrant recovery, the property owner must show that the
and waterworks projects, the duration of which shall be specified situation is such that he has sustained special damage differing in
by the local chief executive concerned in a written order: kind, and not merely in degree, from those sustained by the public
Provided, however, That no national or local road, alley, park, or generally.
square shall be temporarily closed for athletic, cultural, or civic
activities not officially sponsored, recognized, or approved by The constitution does not undertake to guarantee to a property
the local government unit concerned. owner the public maintenance of the most convenient route to his
(d) Any city, municipality, or barangay may, by a duly enacted door. The law will not permit him to be cut off from the public
ordinance, temporarily close and regulate the use of any local thoroughfares, but he must content himself with such route for
street, road, thoroughfare, or any other public place where outlet as the regularly constituted public authority may deem most
shopping malls, Sunday, flea or night markets, or shopping compatible with the public welfare. His acquisition of city property
areas may be established and where goods, merchandise, is a tacit recognition of these principles.
foodstuffs, commodities, or articles of commerce may be sold
and dispensed to the general public. In CAB, the injury suffered by petitioner is the price he and others
like him must pay for the welfare of the entire community. The
inconvenience he suffered pales in comparison to the greater
convenience the new road has been giving to the general public.

DGNotes:
Cabrera vs. CA (1991) Q&A: Can a province close a municipal road in a municipality?
Debatable.
Facts: YES. Municipal plans must be in accordance with provincial plans.
Provincial Board of Catanduanes issued Res. No. 158 for the But then land use plans are very general so this may not be a very
closure of a road leading to the Capitol Bldg. strong argument.
NO. because of autonomy. Even if the province reviews municipal
Owners of the properties traversed by the new road were given ordinances, it can only review its legality.
portions of the old road in exchange for their properties.

Petitioner, upon learning about the resolution, filed a complaint for Dacanay vs. Asistio (1992)
the abatement of nuisance and annulment of resolutions and
documents with damages. He alleged that the land fronting his
Facts:
house was a public road owned by the province in its
1979- MMC Ordinance No. 79-02 was enacted by the MMC,
governmental capacity and therefore beyond the commerce of
designating certain streets, roads and open spaces as sites for flea
man.
markets.
1) WON there is an order for closure or a mere order for the barter
Caloocan city mayor, pursuant to the Ordinance, opened up 7 flea
or exchange of lands.
markets in the city.
Held: Order for closure
Closure of road is clearly ordered by Res. 158 which provides "it is
City mayor and city engineer issued licenses for the conduct of
hereby resolved to close the road"
vending activities upon application of some vendors.
2) WON provincial board has power to order closure
1987- OIC mayor Martinez caused the demolition of market stalls
Held: YES
on certain streets. Stall-owners filed an action for prohibition
Decision in Favis v. Baguio upheld the power of the city council to
praying that the court issue a writ of preliminary injunction.
close city streets and withdraw them from public use. This
decision applies to CAB.

The Apocryphal Maggots:


Rainier, Chrisgel, Corina, Geoffry, Grace and Sylvie Blanche
The Flibbertigibbet Worms:
Golda, Gladys and Melyjane
CA VE AT: By simply r ea ding this re vi ew er a t the end o f the sem este r wi ll (hopef ully) guar antee you r pas sing this cou rse. D rink mode rat ely .
Amusin S C A r y
gly
THE C2005 LOCAL GOVERNMENT REVIEWER - 92 -
RTC issued writ prayed for but later dismissed the petition and
lifted the writ of preliminary injunction it had issued earlier. It found The path climbed upward amongst the jagged hills. Below us
that the streets were of public dominion and hence, outside the lay the town and, far off, the sea, glittering in the moonlight. It
commerce of man. was a warm night, yet the pale rays of the moon cast a chill aura
of malignant evil over the scene. We rode through a cemetery
Shortly after the decision came out, the city administration
changed hands. However, the new city mayor Asistio Jr. did not where the whitewashed tombstones flitted past like serried
pursue the prior administration's policy of clearing and cleaning up ranks of ghosts, then the dark shape of trees rose up again on
the city streets. either side, stretching their gnarled branches in our way.

Petitioner wrote a letter to Asistio asking for the demolition of the And all the while the hollow booming of the drums rang in our
illegally constructed stalls and invoking the decision in prior civil ears; now nearer, now further off, rising and falling in subtle
case. His letters however, were not acted on. cadences… The drums were calling, they drugged the will until
all resistance died. I realized with impotent horror that it was
Issue: WON licenses issued to the stall holders are valid impossible to turn back; the power of the drums was too great.
Held: NO
The disputed areas from which the market stalls are sought to be Suddenly we emerged into a wide clearing. In the middles was
evicted in CAB are undeniably public streets. Being outside the
a huge fire and round it were at least two hundred negroes and
commerce of man, they may not be the subject of lease or other
contract. negresses…
(continued…)
The leases or licenses are null and void for being contrary to law.
The right of the public to use the streets may not be bargained
away through contract. The interests of a few should not prevail
over the good of the greater number in the community.
Book Review on A Hand in the Bush: The Fine Art of Vaginal
Executive Order issued by the Acting mayor Robles authorizing the Fisting
use of said streets as a vending area contravenes the general law
that reserves the city streets and roads for public use. One searches long and hard for a book like this, until the one
day that he finds it and all of his dreams come true. I would be
DGNotes: lying if I said that did not love fisting. But I would also be lying
Who else may order closure of roads if I said that I knew it as a true art. I always saw it as a science.
• Local chief executive – therefore, an ordinance is not Spread legs. Insert fist. Twist around for 15-30 minutes or until
always necessary for the order of closure of roads orgasm. Remove fist. Wash hand. Repeat upon request.
• Close roads for shopping malls, night markets, etc? only But now that I've discovered Ms. Addington's fantastic book
temporarily with it's detailed descriptions and explanations, I am a changed
If the closure was due to an emergency, once the emergency man. Can I get a witness? Praise Jesus!
situation ceases, closure will no longer be valid. After reading the book, my partner of many years has never
been the same. I have never seen her eyes roll back into her head
that way they do since we found this book. Also she learned
that it doesn't have to be all what I'm doing. She guides me to
her spots, teaches me where she wants to be fisted, and
sometimes I apply force.
Excerpt from Voodoo Fire in Haiti by Richard Loederer: So I give this book 5 enthusiastic stars. Before my partner of
many years gave my technique high marks, but looking back I
As we rode through the night the drums were beating again – was an amateur. Now every night she gives me the multiple-
but with a new rhythm that I had never heard before. I was orgasm stamp of approval.
keyed up to a pitch of perspiring excitement, fearing what was
to come and yet unwilling to turn back. We were about to
participate in a monstrous performance, an orgy which not one
white man in a million has ever seen. Tonight was a Voodoo Corporate Powers
Fire, and we were to be present.
Sec22: Corporate Powers. —
I shivered as I rode along. I was horribly afraid; afraid of the (a) Every local government unit, as a corporation, shall have the
night, afraid of the menacing drums, and above all, afraid of following powers:
seeing too much. (1) To have continuous succession in its corporate name;

The Apocryphal Maggots:


Rainier, Chrisgel, Corina, Geoffry, Grace and Sylvie Blanche
The Flibbertigibbet Worms:
Golda, Gladys and Melyjane
CA VE AT: By simply r ea ding this re vi ew er a t the end o f the sem este r wi ll (hopef ully) guar antee you r pas sing this cou rse. D rink mode rat ely .
Amusin S C A r y
gly
THE C2005 LOCAL GOVERNMENT REVIEWER - 93 -
(2) To sue and be sued; 2-fold powers-
(3) To have and use a corporate seal; 1. governmental or political- powers exercised in
(4) To acquire and convey real or personal property;
administering the powers of the state and promoting the
(5) To enter into contracts; and
public welfare
(6) To exercise such other powers as are granted to
corporations, subject to the limitations provided in this Code and 2. municipal powers- those exercised for the special benefit
other laws. and advantage of the community and include ministerial,
(b) Local government units may continue using, modify, or change private and corporate powers.
their existing corporate seals: Provided, That newly established
local government units or those without corporate seals may With respect to proprietary functions, a municipal corporation
create their own corporate seals which shall be registered with can be held liable to third persons ex contractu or ex delicto.
the Department of the Interior and Local Government: Provided,
further, That any change of corporate seal shall also be Maintenance of parks, golf courses, cemeteries and airports,
registered as provided hereon. among others, are recognized as municipal or city activities of a
(c) Unless otherwise provided in this Code, no contract may be proprietary character.
concerned entered into by the local chief executive in behalf of
the local government unit without prior authorization by the In the absence of special laws, the NC is a patrimonial property4 of
sanggunian. A legible copy of such contract shall be posted at a the City over which it exercises acts of dominion. There is
conspicuous place in the provincial capitol or the city, municipal therefore no doubt that the NC is owned by the City in its
or barangay hall. proprietary or private character.
(d) Local government units shall enjoy full autonomy in the
exercise of their proprietary functions and in the limitations The obligations arising from the contract of lease has the force of
provided in this Code and other applicable laws law between the parties in the CAB. The City’s breach of a
contractual provision entitles the Sto. Domingos to damages

City of Manila vs. IAC (1989) - supra Under the doctrine of respondeat superior, the City is liable for the
tortius acts committed by its agents who failed to verify and check
Facts: the duration of the contract of lease.
1971- Vivencio Sto. Domingo died and was buried in Lot 159 in the
North Cemetery, leased by the City to Irene (wife) for 50 yrs.
NAWASA vs. Dator (1967)
City Mayor issued Admin Order No. 5 which prescribes a uniform
procedure and guidelines in the processing of documents for the
use and disposition of burial lots and plots within NC.
• The SC had affirmed the decision of the CFI of Luzon
declaring the municipality of Lucban, Quezon, the owner of
By virtue of said AO, it was believed that Lot 159 was leased only the “Apolinario de la Cruz Waterworks System”, subject,
for 5 years to the Sto. Domingos. however, to the jurisdiction, control and supervision of
NAWASA. This decision had become final and executory.
1978- Lot 159 was exhumed and the remains of Vivencio were • Hobart Dator, in his capacity as Municipal Mayor of Lucban,
placed in a bag and placed in the depository of the cemetery. issued
o a Memorandum directing the Municipal Treasurer to
Irene claims that it was impossible to locate the remains of designate some of the clerks in his office, as temporary
Vivencio in a depository containing thousands of sacks of human waterworks collectors, to receive the water rentals paid
bones. Sto. Domingos instituted an action for damages against by the users of water; and
the City. o a Proclamation enjoining the consuming public to pay
their water fees to the office of the Municipal Treasurer.
TC ordered the City to allow the Sto. Domingos to make use of • The treasurer proceeded to collect water fees from
another lot. CA affirmed but also awarded damages. consumers in the municipality.
• NAWASA filed with the same CFI of Quezon, a petition to
Issue: WON the operations and functions of a public cemetery are declare the mayor in contempt, alleging that the acts of the
a governmental or a corporate or proprietary function of the City
Held: Corporate or proprietary
City of Manila is a political body corporate and as such is endowed 4
Art 424 CC enumerates properties for public use- provincial roads, city streets,
with the faculties of municipal corporations to be exercised by and
municipal streets, squares, fountains, public waters, promenades and public
through its city government in conformity with law, and in its
works for public service paid for by the provinces, cities and municipalities. All
corporate name. It may therefore sue and be sued, and contract
other propertie s are patr im onia l without prejudice to provisions of special
and be contracted with.
laws.

The Apocryphal Maggots:


Rainier, Chrisgel, Corina, Geoffry, Grace and Sylvie Blanche
The Flibbertigibbet Worms:
Golda, Gladys and Melyjane
CA VE AT: By simply r ea ding this re vi ew er a t the end o f the sem este r wi ll (hopef ully) guar antee you r pas sing this cou rse. D rink mode rat ely .
Amusin S C A r y
gly
THE C2005 LOCAL GOVERNMENT REVIEWER - 94 -
latter in ordering for the collection of fees are in defiance of
the SC decision. Dismissed.
• RA 3039 was approved amending Sec. 50 of CA 39 by
providing that “All buildings, properties and assets belonging
WON the mayor’s order to collect water bills in the name of the to the former province of Z and located within the City of Z are
municipality constitutes contempt of court (as an encroachment hereby transferred, free of charge, in favor of the said City of
upon NAWASA’s supervisory power over the municipality’s Z.”
waterworks system) – NO • The Sec. of Finance ordered the CIR to stop effecting further
• The authority of a municipality to fix and collect rents for water payments to Z del Norte and to return to Z City the sum taken
supplied by its waterworks system is expressly granted by from it out of the internal revenue allotment of Z del Norte as
law. (Sec. 2317 of the Revised Administrative Code and Sec. payment for said properties.
2, RA No. 2264) • Z del Norte then filed a complaint in the CFI against Z City,
the Sec. of Finance and the CIR. Among others, it prayed
• Even without these express provisions, the authority of the that RA 3039 be declared unconstitutional for depriving it of
municipality to fix and collect fees from its waterworks property without due process and just compensation and that
would be justified from its inherent power to administer the Sec. of Finance and CIR be enjoined from reimbursing
what it owns privately. Although NAWASA may regulate and the P57,373 to Z City.
supervise the water plants owned and operated by cities and • Judgment was rendered, declaring RA 3039 unconstitutional
municipalities, the ownership thereof is vested in the and ordered Z City to pay to the province the sum of
municipality and in the operation thereof the municipality P704,220.
acts in its proprietary capacity.
• Like any private owner, the municipality enjoys the attributes WON RA 3039 is valid – YES, insofar as 24 of the 50 lots are
of ownership under the New Civil Code. One such attribute is concerned
the right to use or enjoy the property • If the property is owned by the municipality in its public
• If a governmental entity, like the NAWASA, were allowed to and governmental capacity, the property is public and
collect the fees that the consuming public pay for the water Congress has absolute control over it. But if the property
supplied to them by the municipality, the latter, as owner, is owned in its private or proprietary capacity, then it is
would be deprived of the full enjoyment of its property. patrimonial and Congress has no absolute control. The
Ownership is nothing without the inherent rights of municipality cannot be deprived of it without due process
possession, control and enjoyment. and payment of just compensation.
• The National Government can not appropriate • The capacity in which the property is held is, however,
patrimonial property of municipal corporations without dependent on the use to which it is intended and
just compensation and due process of law. As a devoted.
consequence, neither may the National Government
assume the power of administration of patrimonial Which of 2 norms, i.e., that of the Civil Code or that obtaining
property of municipal corporations, if such action is under the law of Municipal Corporations, must be used in
based upon the appropriation of said property by the classifying the properties in question?
State. 2 views:
• In fact, it may not, by operation of law, assume such The CC classification applies.
administration, without appropriating the title to the Result: All properties, except for 2, patrimonial.
property, if the same or the income derived from its • The Civil Code classification is embodied in its Arts. 423 and
operation will be co-mingled with other property, either of 424:
the National Government or of other municipal o ART. 423. The property of provinces, cities and
corporations, in such a way to permit the use of the municipalities, is divided into property for public use and
property or income belonging to one of such patrimonial properly.
corporations for the benefit of another municipal o ART. 424. Property for public use, in the provinces,
corporation or of the State itself. cities, and municipalities, consists of the provincial
roads, city streets, municipal streets, the squares,
fountains, public waters, promenades, and public works
Zamboanga Province vs. Zamboanga City (1968) for public service paid for by said provinces, cities, or
municipalities.
• CA 39 converted the municipality of Zamboanga into o All other property possessed by any of them is
Zamboanga City. patrimonial and shall be governed by this Code, without
• The properties and buildings consisted of 50 lots and some prejudice to the provisions of special laws.
buildings constructed thereon, located in the City of • Applying the provisions above, all the properties, except the 2
Zamboanga and covered by Torrens certificates of title in the lots used as High School playgrounds, could be considered
name of Zamboanga Province. as patrimonial properties of the former Zamboanga province.
Even the capitol site, the hospital and leprosarium sites, and

The Apocryphal Maggots:


Rainier, Chrisgel, Corina, Geoffry, Grace and Sylvie Blanche
The Flibbertigibbet Worms:
Golda, Gladys and Melyjane
CA VE AT: By simply r ea ding this re vi ew er a t the end o f the sem este r wi ll (hopef ully) guar antee you r pas sing this cou rse. D rink mode rat ely .
Amusin S C A r y
gly
THE C2005 LOCAL GOVERNMENT REVIEWER - 95 -
the school sites will be considered patrimonial for they are not • Moreover, the fact that these 26 lots are registered
for public use. They would not fall under the phrase "public strengthens the proposition that they are private in nature.
works for public service" for under the ejusdem generis rule, On the other hand, that the 24 lots for governmental purposes
such public works must be for free and indiscriminate use by are also registered is of no significance since registration
anyone, just like the preceding enumerated properties in the cannot convert public property to private.
first paragraph of Art. 424. The playgrounds, however, would
fit into this category. This was the norm applied by the lower Reasons for adopting 2nd view:
court. • The controversy here is more along the domains of the Law
• In previous cases, it was held that the capitol site and the of Municipal Corporations — State v. Province — than along
school sites in municipalities constitute their patrimonial that of Civil Law.
properties. This result is understandable because, unlike in
the classification regarding State properties, properties for • Moreover, municipal property held and devoted to public
public service in the municipalities are not classified as public. service cannot be said to be in the same category as
ordinary private property, because the consequences are
The principles constituting the law of Municipal Corporations dire. As ordinary private properties, they can be levied
applies. upon and attached. They can even be acquired thru
Result: All those of the 50 properties which are devoted to adverse possession — all these to the detriment of the
public service are deemed public; the rest remain patrimonial. local community.
• Lastly, the classification of properties other than those for
• Under this norm, to be considered public, it is enough that
public use in the municipalities as patrimonial under Art. 424
the property be held and devoted for governmental of the Civil Code is "...without prejudice to the provisions of
purposes like local administration, public education, public special laws." For purposes of this article, the principles
health, etc. obtaining under the Law of Municipal Corporations can be
• Municipality of Batangas v. Cantos: A municipal lot which had considered as "special laws".
always been devoted to school purposes is one dedicated to • Hence, the classification of municipal property devoted for
public use and is not patrimonial property of a municipality. governmental purposes as public should prevail over the Civil
Code classification in this particular case.
SC adopts 2nd view:
• Following this classification, RA 3039 is valid insofar as it
affects the lots used as capitol site, school sites and its
grounds, hospital and leprosarium sites and the high school
playground sites — a total of 24 lots — since these were held Rabuco vs. Villegas
by the former Z province in its governmental capacity and
therefore are subject to the absolute control of Congress.
Facts:
• Regarding the several buildings existing on the lots, the In January 1965, the City mayor and the City Engineer of Manila
records do not disclose whether they were constructed at the ordered the demolition of petitioners’ houses.
expense of the former Province of Z. Considering however
the fact that said buildings must have been erected even Said officials justified their actions by saying that even if the
before the enactment of CA 39 and the further fact that petitioners were already owners of the land on which their
provinces then had no power to authorize construction of respected houses are erected, they could still cause the removal
buildings such as those in the CAB at their own expense, it thereof:
can be assumed that said buildings were erected by the  as they were constructed in violation of city ordinances and
National Government, using national funds. Hence, Congress
 constitute public nuisance
could very well dispose of said buildings in the same manner
that it did with the lots in question.
Petitioners alleged that the officials’ actions were unlawful and
• But even assuming that provincial funds were used, still the clearly prohibited by Republic Act 3120 which:
buildings constitute mere accessories to the lands, which are
public in nature, and so, they follow the nature of said lands,  converted the lot in question which are reserved as
i.e., public. communal property into disposable or alienable lands of the
• Moreover, said buildings, those located in the city, will not be State to be placed under the administration and disposal of
for the exclusive use and benefit of city residents for they the Land tenure Administration for subdivision into small lots
could be availed of also by the provincial residents. xxx for sale on installment basis to the tenants of bona fide
• But RA 3039 cannot be applied to deprive Zamboanga del occupants thereof and
Norte of its share in the value of the rest of the 26 remaining  expressly prohibited ejectment and demolition of petitioners’
lots which are patrimonial properties since they are not being homes under section 2 of the Act
utilized for distinctly governmental purposes.

The Apocryphal Maggots:


Rainier, Chrisgel, Corina, Geoffry, Grace and Sylvie Blanche
The Flibbertigibbet Worms:
Golda, Gladys and Melyjane
CA VE AT: By simply r ea ding this re vi ew er a t the end o f the sem este r wi ll (hopef ully) guar antee you r pas sing this cou rse. D rink mode rat ely .
Amusin S C A r y
gly
THE C2005 LOCAL GOVERNMENT REVIEWER - 96 -
Issue: WON RA 3120 deprives the City of manila of the properties • Upon appeal by the municipal Board, The Court of Tax
without payment of just compensation. Appeals dismissed the case on the ground that the City of
Held: YES Cebu cannot appeal from the decision of the Board of
Salas vs. Jarencio: Assessment Appeals.. The City of Cebu, being a
1. Regardless of the source or classification of land in the governmental agency, is not among those who may appeal to
possession of a municipality, (except those acquired with its own the Court of Tax Appeals.
funds in its private and corporate capacity) such property is held in
trust for the State for the benefit of its inhabitants, whether it be for Issue: WON the City of Cebu can appeal from the decision of
governmental or proprietary purposes. the Board of Assessment Appeals
 that it has in its name a registered title is not questioned, but Held: Yes
this title should be deemed to be held in trust for the State • Sec. 11 of R.A. No. 1125 states:
2. Municipality holds such lands subject to the paramount power of Sec. 11. who May Appeal. Any person, association or
the legislature to dispose of the same: corporation adversely affected by a decision or ruling of the
 for after all it owes is creation to it as an agent for the CIR, Coll. Of Customs or any provincial or city Board of
performance of a part of its public work (the municipality Assessment Appeals may filed an appeal in the CTA within 30
being but a subdivision thereof for purposes of local days after the receipt of such decision or ruling.”
administration • The City of Cebu constitutes a political body corporate
3. legal situation is the same as of the State itself holds the created by a special charter endowed with the powers which
property and puts it to a different use pertain to a municipal corporation. As such, it possesses the
capacity to sue and be sued. It is authorized to levy real
Court further says: estate taxes for its support.
1. RA 3120 was intended to implement: • In the decision of the Board of Assessment Appeals
 the social justice policy of the Constitution and the exempting the lots in question from the payment of real
 governmental program of land for the landless and property tax, no entity is more adversely affected than the City
 that they were not intended to expropriate the property of Cebu, for it stands to lose a yearly income equivalent to the
involved realty tax.
 but merely t confirm its character as communal land of the • As to the personality of the municipal Board to represent the
State and City of Cebu in this suit, suffice it to say that Sec. 58 of the
 to make it available for disposition by the National Commonwealth Act No. 58 expressly vests in the Municipal
Government Board the authority to appeal from the decision of the City
2. Subdivision of communal property and subsequent sale: Assessor to the Board of Assessment Appeals. This indicates
 does NOT operate as an exercise of the power of eminent legislative intent to lodge in the Municipal Board the right to
domain without just compensation represent the City in an appeal from an adverse decision of
 BUT SIMPLY as a manifestation of its right and power to deal the City Board of AA.
with state property Decision set aside. Case remanded to CTA.

Authority to Negotiate and Secure Grants

Sec23: Authority to Negotiate and Secure Grants. — Local


chief executives may, upon authority of the sanggunian,
negotiate and secure financial grants or donations in kind, in
support of the basic services or facilities enumerated under
Municipal Board vs. CTA (1964) Section 17 hereof, from local and foreign assistance agencies
without necessity of securing clearance or approval therefor
Facts: from any department, agency, or office of the national
• The University of Southern Philippines Foundation, through government of from any higher local government unit: Provided,
its president, applied for inclusion in the list of real estate That projects financed by such grants or assistance with
exempt from real property taxation several parcels of land national security implications shall be approved by the national
which it leased from various persons for school purposes. agency concerned: Provided, further, That when such national
• City Assessor informed the University that, except for 6 lots, agency fails to act on the request for approval within thirty (30)
all the remaining lots were exempt from real property taxation. days from receipt thereof, the same shall be deemed approved.
Cdasia
• On appeal, the Board of Assessment Appeals of Cebu
Citygranted The local chief executive shall, within thirty (30) days upon
exemption to the 6 lots. signing of such grant agreement or deed of donation, report the

The Apocryphal Maggots:


Rainier, Chrisgel, Corina, Geoffry, Grace and Sylvie Blanche
The Flibbertigibbet Worms:
Golda, Gladys and Melyjane
CA VE AT: By simply r ea ding this re vi ew er a t the end o f the sem este r wi ll (hopef ully) guar antee you r pas sing this cou rse. D rink mode rat ely .
Amusin S C A r y
gly
THE C2005 LOCAL GOVERNMENT REVIEWER - 97 -
nature, amount, and terms of such assistance to both Houses of
Congress and the President. RA 8749

Sec43: Suits and Strategic Legal Actions Against Public


Liability for Damages Participation and the Enforcement of this Act. — Where a
suit is brought against a person who filed an action as provided
Sec24: Liability for Damages. — Local government units and in Section 41 of this Act, or against any person, institution or
their officials are not exempt from liability for death or injury to government agency that implements this Act, it shall be the duty
persons or damage to property. of the investigating prosecutor or the court, as the case may be,
to immediately make a determination not exceeding thirty (30)
days whether said legal action has been filed to harass, vex,
New Civil Code exert undue pressure or stifle such legal recourses of the person
complaining of or enforcing the provisions of this Act. Upon
Art34: When a member of a city or municipal police force refuses determination thereof, evidence warranting the same, the court
or fails to render aid or protection to any person in case of shall dismiss the case and award attorney's fees and double
danger to life or property, such peace officer shall be primarily damages.
liable for damages, and the city or municipality shall be This provision shall also apply and benefit public officers who
subsidiarily responsible therefor. The civil action herein are sued for acts committed in their official capacity, there being
recognized shall be independent of any criminal proceedings, no grave abuse of authority, and done in the course of enforcing
and a preponderance of evidence shall suffice to support such this Act.
action.

Art2180: The obligation imposed by article 2176 is demandable Mendoza vs. De Leon (1916)
not only for one's own acts or omissions, but also for those of
persons for whom one is responsible. Facts:
The father and, in case of his death or incapacity, the mother, * Mendoza was awarded the lease of an exclusive ferry privilege
are responsible for the damages caused by the minor children by the municipality of Villasis, Pangasinan under the provisions of
who live in their company. Act 1634 of the Phil. Commission.
Guardians are liable for damages caused by the minors or * After operation for a little more than a year, he was forcibly
incapacitated persons who are under their authority and live in ejected in pursuance of a resolution adopted by the Villasis
their company. municipal council, which awarded the franchise for the same ferry
The owners and managers of an establishment or enterprise are to another person.
likewise responsible for damages caused by their employees in * This is an action for damages against the individual members of
the service of the branches in which the latter are employed or the municipal council (De Leon et al) for the revocation.
on the occasion of their functions.
Employers shall be liable for the damages caused by their Issue1: What are the rules governing the liability of a municipality?
employees and household helpers acting within the scope of
their assigned tasks, even though the former are not engaged in 1) Municipalities have both governmental & corporate functions.
any business or industry.
The State is responsible in like manner when it acts through a A. In so far as its governmental functions are concerned, a
special agent; but not when the damage has been caused by municipality is not liable at all, unless expressly made so by
the official to whom the task done properly pertains, in which statute; nor are its officers, so long as they perform their duties
case what is provided in article 2176 shall be applicable. honestly and in food faith.
Lastly, teachers or heads of establishments of arts and trades Reason: Govtl affairs do not lose their govtl character by being
shall be liable for damages caused by their pupils and students delegated to the municipal govts…The state, being immune for
or apprentices, so long as they remain in their custody. injuries suffered by private individuals in the administration of
The responsibility treated of in this article shall cease when the strictly govtl functions, like immunity is enjoyed by the municipality
persons herein mentioned prove that they observed all the in the performance of the same duties, unless it is expressly made
diligence of a good father of a family to prevent damage. liable by statute…The exemption is based upon the sovereign
(1903a) charac of the state & its agencies and upon the absence of
obligation.
Art2189: Provinces, cities and municipalities shall be liable for
damages for the death of, or injuries suffered by, any person by B. In the administration of its patrimonial property [corporate
reason of the defective condition of roads, streets, bridges, functions], a municipality is to be regarded as a private corpo or
public buildings, and other public works under their control or individual in so far as its liability to third persons on contract or in
supervision. (n) tort is concerned.

The Apocryphal Maggots:


Rainier, Chrisgel, Corina, Geoffry, Grace and Sylvie Blanche
The Flibbertigibbet Worms:
Golda, Gladys and Melyjane
CA VE AT: By simply r ea ding this re vi ew er a t the end o f the sem este r wi ll (hopef ully) guar antee you r pas sing this cou rse. D rink mode rat ely .
Amusin S C A r y
gly
THE C2005 LOCAL GOVERNMENT REVIEWER - 98 -
Its contracts, validly entered into, may be enforced & damages
may be collected from it for the torts of its officers or agents within San Fernando vs. Firme (1991)
the scope of their employment in precisely the same manner and
to the same extent as those of private corporations or individuals. Facts:
As to such matters the principles of respondeat superior applies. Morning of December 1965, collision occurred involving:
 Passenger jeepney driven by Balagot and owned by
[ Addtl requirement] To create such liability, it is necessary that Estate of Nieveras
the act done which is injurious to others must be within the scope
 Gravel and sand truck driven by Marandang and owned
of the corporate powers…it must not be ultra vires (must not lie
by Velasquez
wholly outside of the general or special powers of the corpo as
conferred in its charter or by statute).  Dump truck of Municipality
Reason: A corpo can’t be impliedly liable to a greater • Casualties: several passengers of the jeep including
extent that it could make itself by express corporate vote Laureano, Sr. died
or action. • Dec 1966, private respondents instituted a compliant for
damages against the Estate of Nieveras and Balagot, in the
2) HERE  the leasing of a municipal ferry to the highest bidder is CFI of La Union, Br I. However, defendants filed Third Party
a corporate function. Such a lease…constitutes a contract…which Complaint against Municipality and the driver of dump truck
the municipality is bound to respect. (Bislig).
Were the municipality a party to this action…a judgment for
damages against it for the rescission of the contract would be Issue: WON the municipality is liable for the torts committed by its
proper. But the present action is against the members of the employee:
municipal council personally. So… TEST: depends on whether or not the driver, acting in behalf
of the municipality, is performing governmental or proprietary
Issue2: Are the individual municipal council members liable? functions.
SC notes that in permitting such entities to be sued (through
A. Gen. Rule: In administering the patrimonial property of allowance in the municipal charter), the State merely gives
municipalities, the municipal council occupies the position of a the claimant the right to show that the defendant was not
board of directors of a private corpo. They are not liable for the acting in its governmental capacity when the injury was
mismanagement of corpo affairs where such mismanagement is a committed or that the case comes under the exceptions
mistake of judgment. recognized by law. Failing this, the claimant cannot recover.
And in the CAB, the driver was indeed performing
B. Exception: Directors & managing officers may be liable for governmental functions!
mismanagement…if it can be plainly shown that their actions are  Bislig insists that "he was on his way to the Naguilian
so far opposed to the true interests of the corpo itself as to lead to river to get a load of sand and gravel for the repair of
the clear inference that no one thus acting could have been San Fernando's municipal streets." SC ruled in Palafox,
influenced by any honest desire to secure such interests, but that et. al. v. Province of Ilocos Norte, the District Engineer,
he must have acted with an intent to subserve some outside and the Provincial Treasurer (102 Phil 1186) that "the
purpose, regardless of the consequences to the corpo, and in a construction or maintenance of roads in which the truck
manner inconsistent with its interests. and the driver worked at the time of the accident are
admittedly governmental activities".
HERE There was no valid reason for forcibly evicting the • In the absence of any evidence to the contrary, the regularity
plaintiff. He had operated the ferry for over a year with knowledge
of the performance of official duty is presumed (RoC). Hence,
of the councilors. Thus, the argument that he had been operating a
the driver of the dump truck was performing duties or tasks
ferry other than the one leased to him is untenable.
pertaining to his office. Hence, no liability.
In rescinding the contract, thereby making the municipality liable
to an action for damages for no valid reason at all, the defendant DGNotes:
councilors are not honestly acting for the interests of the • Is this no longer good law with passage of LGC? No. LGC
municipality. They are liable solidarily for the damages sustained by should be read civil code provisions.
the plaintiff. • Differentiated bet. Suability and liability. Determine first if
Affirmed. suable. If yes, then determine if liable.
Sec 24- LGUs and its officials are not exempt from liability for
DGNotes: death or injury.
If a municipality acts in a governmental capacity, how can they be All government units can be sued because of Sec 24. Won one
liable? can recover is another question.
1. bad faith of public officer - officer is liable
2. law provides it can beliable – Sec 24 LGC; Art 2189 CC

The Apocryphal Maggots:


Rainier, Chrisgel, Corina, Geoffry, Grace and Sylvie Blanche
The Flibbertigibbet Worms:
Golda, Gladys and Melyjane
CA VE AT: By simply r ea ding this re vi ew er a t the end o f the sem este r wi ll (hopef ully) guar antee you r pas sing this cou rse. D rink mode rat ely .
Amusin S C A r y
gly
THE C2005 LOCAL GOVERNMENT REVIEWER - 99 -
Fernando vs. CA (1992) old hand in this kind of service, who is presumed to know the
hazards of the job. His failure, therefore, and that of his men to
Facts: take precautionary measures for their safety was the proximate
• A certain Bascon won the bid for the re-emptying of the septic cause of the accident.
tank in Agdao.
3. Petitioners: insist on the applicability of Article 24 of the New
• losing bidder Bertulano with four other companions were
Civil Code:
found dead inside the septic tank. "Art. 24. In all contractual, property or other relations, when one
• The City Engineer's office investigated, findings: the victims of the parties is at a disadvantage on account of his moral
entered the tank without clearance from it nor with the dependence, ignorance, indigence, mental weakness, tender age
knowledge and consent of the market master. The tank was or other handicap, the courts must be vigilant for his protection."
found to be almost empty and the victims were presumed to
be the ones who did the re-emptying. SC: Untenable. We approve of the appellate court's ruling that
• Autopsy reveals: cause of death of all five victims as "(w)hile one of the victims was invited to bid for said project, he did
`asphyxia' caused by the diminution of oxygen supply. The not win the bid, therefore, there is a total absence of contractual
lungs of the five victims burst due to their intake of toxic gas, relations between the victims and the City Government of Davao
produced from the waste matter in the tank. City that could give rise to any contractual obligation, much less,
• Heirs of the deceased filed suit against City of Davao. any liability on the part of Davao City."
• Trial court dismissed, CA awarded damages. But upon MR,
CA reversed its original ruling and dismissed the case.
Guillergan vs. Ganzon (1966)
Issue: WON Davao City guilty of negligence in the case at bar?
Held: NO. Facts:
To be entitled to damages for an injury resulting from the Petitioners are laborers or employees in the unclassified service,
negligence of another, a claimant must establish the relation assigned as market-sweepers of the City of Iloilo. They had been
between the omission and the damage. He must drove under working for some time, ranging from 9-25 years.
Article 2179 of the New Civil Code that the defendant's negligence
was the immediate and proximate cause of his injury. Proximate Claiming that they had been illegally dismissed or separated, they
cause has been defined as that cause, which, in natural and filed suit against the Mayor, Treasurer et al of Iloilo City, to compel
continuous sequence unbroken by any efficient intervening cause, these officers to reinstate them to their former positions.
produces the injury, and without which the result would not have
occurred Mayor claimed that he had the right to remove or separate the
petitioners from the service because they were not civil service
1. Petitioners fault the city government of Davao for failing to clean eligibles and were paid on a daily basis.
a septic tank for the period of 19 years resulting in an accumulation
of hydrogen sulfide gas which killed the laborers. They contend Issue: WON the lower court erred in holding the City of Iloilo jointly
that such failure was compounded by the fact that there was no liable with the other respondents for the back salaries of petitioners
warning sign of the existing danger and no efforts exerted by the Held: Yes
public respondent to neutralize or render harmless the effects of 1. The Charter of Iloilo City (Sec. 3 of Commonwealth Act No.
the toxic gas. They submit that the public respondent's gross 158), expressly provides that the City of Iloilo may "sue and be
negligence was the proximate cause of the fatal incident. sued".
SC: No. While it may be true that the public respondent has been
remiss in its duty to re-empty the septic tank annually, such 2. The operation of a market, in the cleaning of which petitioners
negligence was not a continuing one. Upon learning from the herein are engaged, is not strictly a governmental function.
report of the market master about the need to clean the septic tank
of the public toilet in Agdao Public Market, the public respondent 3. It has been settled that municipal corporations may be held
immediately responded by issuing invitations to bid for such liable for back pay or wages of employees or laborers illegally
service. separated from the service, including those involving primarily
governmental functions, such as those of policemen.
2. In view of this factual milieu, it would appear that an accident Decision affirmed.
such as toxic gas leakage from the septic tank is unlikely to
happen unless one removes its covers. The accident in the case at
bar occurred because the victims on their own and without Pilar vs. Sangguniang Bayan ng Dasol, Pangasinan (1984)
authority from the public respondent opened the septic tank.
Considering the nature of the task of emptying a septic tank
Facts:
especially one which has not been cleaned for years, an ordinarily
Expedito Pilar was elected as Vice mayor of Dasol, Pangasinan.
prudent person should undoubtedly be aware of the attendant
risks. The victims are no exception; more so with Mr. Bertulano, an

The Apocryphal Maggots:


Rainier, Chrisgel, Corina, Geoffry, Grace and Sylvie Blanche
The Flibbertigibbet Worms:
Golda, Gladys and Melyjane
CA VE AT: By simply r ea ding this re vi ew er a t the end o f the sem este r wi ll (hopef ully) guar antee you r pas sing this cou rse. D rink mode rat ely .
Amusin S C A r y
gly
THE C2005 LOCAL GOVERNMENT REVIEWER - 100 -
SB of Dasol adopted Resolution No. 1 increasing the salaries of that thresher-owner-operator voluntarily agree to donate 1%
mayor and municipal treasurer but not that of the vice mayor. Pilar of all palay threshed within jurisdiction of municipality… and
questioned it. agree to report weekly the total number of palay threshed…

The provincial and national gov’t endorsed compliance with Private respondent Jurado sent his agent to municipal treasurer’s
Circular 9-A of the Joint Commission on Local Government and office to pay license fee of 285P for thresher operators but Mapagu
Personal administration in giving the revised rate of salary to the refused to accept payment and required him to secure a mayor’s
vice mayor. The executive secretary of the Commission sent a permit first. Mayor Tuzon said that he should first comply with
letter to Mayor advising him to pay vice mayor salary equal to the Resolution9 and sign the agreement before permit could be
treasurer. issued.
Jurado ignored requirement and sent 285P license fee by postal
SB enacted resolution appropriating the amount of P15,144 as money order to the office of municipal treasurer. Mapago returned
unpaid salaries of Pilar from Jan 1, 1981- Dec. 31, 1982. amount because of failure to comply with Resolution No.9.

Mayor vetoed the resolution. Hence, Pilar filed for a writ of Special civil action for mandamus with damages to compel
mandamus. issuance of mayor’s permit and license filed with CFI. then
declaratory judgment against resolution and implementing
Issue: WON Pilar is entitled to damages agreement for being illegal either as a donation or as a tax
Held: Yes measure.
Pilar is entitled to damages and attorney's fees because the facts
show that Issue: W/N Mayor and Treasurer are liable for damages.
1. he was forced to litigate in order to claim his lawful salary Held: NO.
which was unduly denied him for 3 years and Civil Code provision (Art27) has been remarked:
2. the Mayor acted in gross and evident bad faith in refusing to To have a purpose to end the bribery system, where public
satisfy petitioner's plainly valid, just and demandable claim. official, for some flimsy excuse, delays or refuses the
performance of his duty until he gets some kind of pabagsak”
WHO WILL PAY DAMAGES? (Paras on Civil Code)
Mayor Lodovico Espinosa alone should be held liable and To presuppose that the refusal or omission of a public official
responsible for the miserable plight of Pilar. to perform his official duty is attributable to malice or
• Mayor vetoed without just cause SB Resolution appropriating inexcusable negligence (Phil. Match Co. vs. City of Cebu)
the salary of Pilar In any event, the erring public functionary is justly punishable
• Mayor exceeded his authority in an arbitrary manner when he under it for whatever loss or damage complainant has
vetoed the resolution since there exists sufficient municipal sustained.
funds from which the salary of the petitioner could be paid.  CAB:
• Mayor's refusal, neglect or omission in complying with the It has not been alleged that Mayor’s refusal to act on his
directives of the Provincial Budget Officer and the Director of application was an attempt to compel him to resort to bribery to
the Bureau of Local Government that the salary of the obtain approval of his application.
petitioner be provided for and paid the prescribed salary rate, It cannot be said also that mayor and treasurer were motivated by
is reckless and oppressive personal spite or were grossly negligent in refusing to issue permit
Mayor is liable personally to the petitioner for exemplary or and license to Jurado.
corrective damages as well as actual damages and costs of No evidence has been offered to show that they singled out
litigation for mental anguish, serious anxiety, wounded feelings, Jurado for persecution.
moral shock, social humiliation and similar injury. Attorney’s fees Neither does it appear that they stood to gain personally from
was also awarded. refusing to issue to Jurado the permit and license he needed.
They were not his business competitors nor has it been
established that they intended to favor his competitors.
Tuzon vs. CA (1992)
On the contrary, record discloses that resolution was uniformly
Facts: applied to all threshers in the municipality without discrimination or
Sangguniang Bayan of Camalaniugan, Cagayan unanimously preference.
adopted Resolution No.9 where 1% donation from thresher
operators who will apply for a permit to thresh within its jurisdiction Petitioners acted within scope of their authority and in consonance
will be solicited to help finance construction of Sports and Nutrition with their honest interpretation of the resolution. In the absence of
Center. a judicial decision declaring it invalid, the legality of challenged
Petitioner municipal treasurer Mapagu prepared a document for measures would have to be presumed. As executive officials of
signature of all thresher applying for a mayor’s permit to implement the municipality, they had the duty to enforce it as long as it had
the resolution: not been repealed by Sangguniang Bayan or annulled by the
courts.

The Apocryphal Maggots:


Rainier, Chrisgel, Corina, Geoffry, Grace and Sylvie Blanche
The Flibbertigibbet Worms:
Golda, Gladys and Melyjane
CA VE AT: By simply r ea ding this re vi ew er a t the end o f the sem este r wi ll (hopef ully) guar antee you r pas sing this cou rse. D rink mode rat ely .
Amusin S C A r y
gly
THE C2005 LOCAL GOVERNMENT REVIEWER - 101 -
Respondents’ contention: (1) By seeking specific performance of
As to Jurado’s contention that he was prevented from deed of donation as their primary cause of action, they cannot at
operating his business and profit by petitioners’ acts… the same time claim ownership over property subject of donation
SC: Petitioners are correct that he should have taken prudent by virtue of laches or acquisitive prescription – inconsistent causes
course of signing the agreement under protest and later of action. (2) Trial court already found deed to have been validly
challenging it in court to relieve him of the obligation to “donate.” revoked so the primary cause of action was already declared
inexistent  Hence, correct dismissal of complaint by CA.
Held: YES. See (2)
Baluyot et al. vs. CA (1999) 1. As to RTC ruling that there is no cause of action for specific
performance because of revocation of donation but UP was
Facts: barred to contest petitioners’ right to remain in possession on
Petitioners are residents of Brgy. Cruz-na-Ligas, Diliman, QC and ground of laches. W/N it is correct
members of a non-stock corporation Cruz-na-Ligas Homesite SC: NO. RTC is erroneous.
Association, Inc. They filed a complaint for specific performance a. UP cannot be barred by laches
and damages against UP before RTC, QC, which was later While prescription does not run against registered lands,
amended to include QC government. nonetheless a registered owner’s action to recover
They allege: possession of his land may be barred by laches (as held in
UP through Board of Regents approved donation of land in Mejia de Lucas vs. Gamponia).
their favor and of its willingness to proceed with it, and However, laches is a defense against a registered owner
Association manifested consent in writing for benefit of suing to recover possession of the land registered in its
bonafide residents and agreement to comply with its terms name. UP is not suing in CAB.
and conditions
UP backed-out and resumed negotiations on donation thru
b. Petitioners did not invoke laches. What they alleged in
QC government under terms disadvantageous to residents their complaint is their occupancy of the land from time
immemorial, adversely, and continuously in concept of owner.
UP and Petitioners have come to the agreement for the
execution of Deed of Donation by UP to QC gov for the c. They may have claimed prescription but it is untenable.
benefit of qualified residents under the conditions to be Land in question is registered land.
complied with by QC gov’t. d. Validity of UP’s title to land cannot be questioned being a
QC immediately prepared groundworks to comply but UP collateral attack on registered land which is not
failed to deliver certificate of title covering property to enable permitted.
QC to register Deed of Donation and ownership and comply
with the terms and conditions of the Deed (Par18 of 2. As to CA ruling that complaint fails to state a cause of
complaint) action. W/N complaint states a cause of action.
Despite requests and several conferences made, UP did not SC: YES. CA is erroneous.
comply with its duty to deliver certificate (Par19) All the elements of a cause of action are found contained in
Upon expiration of 18months, UP declared revocation of amended complaint. ((1) right in favor of plaintiff by whatever
Deed for alleged noncompliance with terms and conditions means and under whatever law it arises or is created; (2) an
(Par20) obligation by defendant to respect or not to violate such right; (3)
act or omission on part of defendant in violation of right of plaintiff
Issue1: W/N Petitioners’ amended complaint alleges facts or constituting a breach of obligations of defendant to plaintiff for
constituting a cause of action. which latter may maintain an action for recovery of damages)
Petitioners’ argument: Citing paragraphs 185, 196 and 207 of their
complaint questioning the validity of revocation of the donation and a. While they were not parties to deed of donation, their right to
seek enforcement of donation through specific performance. seek its enforcement upon their allegation that they are
intended beneficiaries of donation to QC gov is supported by
5 Art1311, CC (stipulation pour autrui) which exists in CAB:
That QC government immediately prepared the groundworks in compliance with
the terms and conditions of the donation, but UP had failed to deliver certificate of (1) That there must be a stipulation in favor of a 3 rd person
title covering property to be donated to enable QC to register Deed of Donation  Par17 of complaint (that deed of donation contains a
and corresponding certificate of title be issued under its name. stipulation that QC gov, as donee, is required to transfer
6
UP had continuously and unlawfully refused, despite requests and several to qualified residents, by way of donations, the lots
conferences made, to comply with its reciprocal duty to deliver certificate of tile to occupied by them)
enable Donee (QC gov) to register the ownership so that it can legally and fully (2) Stipulation must be a part, not the whole of the contract
comply with their obligation under the deed of donation.  same Par17 (that the stipulation is part of conditions
7
Upon expiration of 18months for alleged non-compliance of QC gov with terms and obligations imposed by UP, as donor, upon QC gov,
and conditions of Deed of Donation, UP thru President Abueva unilaterally, as donee)
capriciously, whimsically, and unlawfully issued Admin Order 21 declaring deed of
donation revoked and donated property be reverted to UP.

The Apocryphal Maggots:


Rainier, Chrisgel, Corina, Geoffry, Grace and Sylvie Blanche
The Flibbertigibbet Worms:
Golda, Gladys and Melyjane
CA VE AT: By simply r ea ding this re vi ew er a t the end o f the sem este r wi ll (hopef ully) guar antee you r pas sing this cou rse. D rink mode rat ely .
Amusin S C A r y
gly
THE C2005 LOCAL GOVERNMENT REVIEWER - 102 -
(3) Contracting parties must have clearly and deliberately causes of action where one of them clearly states a sufficient
cause of action against defendant.
conferred a favor upon 3rd person, not a mere incidental
CA reversed. Case remanded to RTC.
benefit or interest  Par15 and 16 (that intent of parties
to deed of donation was to confer a favor upon
petitioners by transferring to latter lots occupied by them)
(4) 3rd person must have communicated his acceptance to Voodoo Fire in Haiti (part 2):
obligor before its revocation  Par19 (that conferences
were held between parties to convince UP to surrender To the left of the fire a row of stakes had been driven into the
certificates of title to QC gov, implying that donation had ground. Fixed horizontally across them at the height of a man’s
been accepted by petitioners by demanding fulfillment head were crossbars from which depended five long conical
thereof and that UP were aware of such acceptance)
drums. A gigantic naked negro stood in front of each, working
(5) Neither of contracting parties bears legal representation like a fiend. Two of the men used short wooden sticks but the
or authorization of 3rd party  all allegations considered other three evoked a peculiar rhythm by gliding their fingers
together (it can be fairly inferred that neither acted in and palms over the tightly stretched goatskins. The drums
representation of the other; each had its own obligations, responded to the efforts of these sweating blacks with a
in view of conferring a favor upon petitioners) shattering resonance of sound.
b. Amended complaint further alleged:
That UP has an obligation to transfer the land to city
Suddenly a negress wearing a white chemise and a scarlet sash
government so that QC gov can in turn comply with its
obligations to make improvements and then transfer it to stood up. It was the Mamaloi…
petitioners,
That in breach of this obligation, UP failed to deliver title and Backwards and forwards danced the Mamaloi. In and out
then revoked deed of donation city failed to fulfill its obligation between the rows of squatting figures. Her eyes were fixed in a
within time allowed rigid sightless stare and the sweat poured down her body.
 These allegations must be deemed to be hypothetically Saliva ran from her mouth, trickling down her neck and
true for purpose of determining sufficiency of cause of action. between her breasts. She approached the fire…
It is for trial court to determine ruling on the merits. (continued…)

3. As to Respondents’ contention that trial court has already


found that the donation has already been revoked
SC: No merit.
The ruling on that point was made in connection with the
application for a writ of preliminary injunction to stop UP from Ch ap te r 3: In terg overnmen ta l R el at io ns –
ejecting petitioners. TC denied injunction on the ground that
donation had already been revoked and therefore petitioners had
Nat ion al G overnmen t a nd L oc al G overnmen t
no clear legal right to be protected. Uni ts
Evident that such ruling was only tentative, without prejudice to the
final resolution of the question after presentation by the parties of Ex ec ut iv e Supe rv isio n
their evidence.
1987 Constitution
Issue2: W/N amended complaint alleges inconsistent causes of
action for specific performance of deed of donation. Article X
Respondents claim: While petitioners claim to be beneficiaries- Sec2: The territorial and political subdivisions shall enjoy local
donees of 15.8hectares subject of the deed, they at the same time autonomy.
seek recovery/delivery of title to 42hectares of land included in
UP’s certificate of title. Sec4: The President of the Philippines shall exercise general
supervision over local governments. Provinces with respect to
Held: NO. Such allegations are not inconsistent but, rather, component cities and municipalities, and cities and
alternative causes of action which Rule8, Sec2 RoC allows. municipalities with respect to component barangays shall
1. Parties are allowed to plead as many separate claims as they ensure that the acts of their component units are within the
may have, regardless of consistency, provided that no rules scope of their prescribed powers and functions.
regarding venue and joinder of parties are violated.
2. Moreover, subjects of these claims are not exactly and Article XI
entirely the same parcel of land, when their causes of action Sec25 (no such section)
consist of 2 definite and distinct claims. Rule is that a trial
judge cannot dismiss a complaint which contained 2 or more

The Apocryphal Maggots:


Rainier, Chrisgel, Corina, Geoffry, Grace and Sylvie Blanche
The Flibbertigibbet Worms:
Golda, Gladys and Melyjane
CA VE AT: By simply r ea ding this re vi ew er a t the end o f the sem este r wi ll (hopef ully) guar antee you r pas sing this cou rse. D rink mode rat ely .
Amusin S C A r y
gly
THE C2005 LOCAL GOVERNMENT REVIEWER - 103 -
Local Government Code unlawfully refused and still refused to surrender said office to
Reyes, who claimed to be entitled thereto.
Sec25: National Supervision over Local Government Units. —
(a) Consistent with the basic policy on local autonomy, the Issue: WON a municipal mayor, not charged with disloyalty to
President shall exercise general supervision over local the Republic of the Philippines, may be removed or
government units to ensure that their acts are within the scope suspended directly by the President of the Philippines,
of their prescribed powers and functions. regardless of the procedure set forth in sections 2188 to 2191
The President shall exercise supervisory authority directly over of the Revised Administrative Code.
provinces, highly urbanized cities, and independent component Held: NO
cities; through the province with respect to component cities and 1. President has no "inherent power to remove or suspend" local
municipalities; and through the city and municipality with respect elective officers. In the same case, this Court also declared
to barangays. that there is no statutory or consti provision granting the
(b) National agencies and offices with project implementation President sweeping authority to remove municipal officials.
functions shall coordinate with one another and with the local
government units concerned in the discharge of these functions. What is the procedure prescribed by law for the suspension
They shall ensure the participation of local government units of elective municipal officials?
both in the planning and implementation of said national Provincial governor receives and investigates complaints. If a
projects. more severe punishment (not mere reprimand) is required, he
(c) The President may, upon request of the local government unit shall submit the charges to the provincial board. He may
concerned, direct the appropriate national agency to provide suspend the officer pending action by the board. The
financial, technical, or other forms of assistance to the local preventive suspension shall not be for more than 30 days.
government unit. Such assistance shall be extended at no extra The provincial board shall then conduct a trial on the charges.
cost to the local government unit concerned. If the officer wishes to appeal an adverse decision of the
(d) National agencies and offices including government-owned or board, he may appeal to the Secretary of the Interior. (based
controlled corporations with field units or branches in a province, on section 2188 to 2191 of the Revised Admin Code)
city, or municipality shall furnish the local chief executive
concerned, for his information and guidance, monthly reports It is, likewise, well settled that laws governing the suspension
including duly certified budgetary allocations and expenditures. or removal of public officers, especially those chosen by the
direct vote of the people, must be strictly construed in their
favor.
Hebron vs. Reyes (1958)
Accordingly, when the procedure for the suspension of an
Facts: officer is specified by law, the same must be deemed
→ In the 1951 general elections, Hebron, a LP member, and mandatory and adhered to strictly, in the absence of express
Reyes, of the NP, were elected mayor and vice-mayor, of or clear provision to the contrary — which does not exist with
Carmona, Cavite for a term of 4 years. respect to municipal officers.

Hebron discharged the duties and functions of mayor What is more, the language of sections 2188 to 2191 of the
continuously until May 22 or 24, 1954, when he received a Revised Administrative Code leaves no room for doubt that
letter from the Office of the President stating the ff: the law - in the words of Mr. Justice Tuason — "frowns upon
 That the President has decided to assume directly the prolonged or indefinite suspension of local elective officials"
investigation of the administrative charges against him. (Lacson vs. Roque). → In CAB, Hebron was suspended in
 That he shall be suspended from office, effective may 1954. The records of the case were forwarded to the
immediately, to last until the final termination of the Exec. Secretary since July 15, 1954. Yet no decision had
administrative proceedings (Hebron was suspended for been made as of May 1955, when this complaint was filed or
more than a year and seven months) before the expiration of Hebron’s term on December 31,
 That Vice-Mayor Reyes would assume the office of 1955.
Acting Mayor during that period.
→ Thereupon Reyes acted as mayor of Carmona and the 2. Next, respondent cites Sections 79 (C) and 86 of the RAC.
Provincial Fiscal of Cavite investigated the charges. After These provisions state that the Dept. Head (referring to
holding hearings in connection with said charges, the Secretary of Interior) shall have direct control…over all
provincial fiscal submitted his report thereon on July 15, 1954. bureaus and OFFICES…
Since then, the matter has been pending in the Office of the
President for decision. Inasmuch as a decision did not appear SC: In the case of Mondano vs. Silvosa, this Court has
to be forthcoming, and the term of petitioner, who remained already said that although the Dept. Head as agent of the
suspended, was about to expire, he instituted the present president has direct control over all bureaus and offices, he
action for quo warranto, upon the ground that respondent was does not have the same control of local governments.
illegally holding the Office of Mayor of Carmona, and had

The Apocryphal Maggots:


Rainier, Chrisgel, Corina, Geoffry, Grace and Sylvie Blanche
The Flibbertigibbet Worms:
Golda, Gladys and Melyjane
CA VE AT: By simply r ea ding this re vi ew er a t the end o f the sem este r wi ll (hopef ully) guar antee you r pas sing this cou rse. D rink mode rat ely .
Amusin S C A r y
gly
THE C2005 LOCAL GOVERNMENT REVIEWER - 104 -
These provisions were inserted in the RAC during the would, in effect, place local governments under the control of
American regime, when the Executive (Gov-Gen) still had the Executive and consequently conflict with the Constitution
control over all executive functions of government. However,
this authority has been constricted under the new Consti, If neither the Secretary of the Interior nor the President may
which limits the Exec’s powers to general supervision. disapprove a resolution of the Provincial Board of Pangasinan
(as held in the case of Rodriguez vs. Montinola), passed
If the same sections would be applied to local governments, within the jurisdiction thereof, because such disapproval
the President could alter or modify or nullify any duly enacted would connote the assumption of control, which is denied by
municipal ordinance. Yet, it is well settled that he cannot do the Constitution, it is manifest that greater control would be
so. Also, the word “offices” as used in 79 (C) was not wielded by said officers of the national government if they
deemed to include local governments, even before the could either assume the powers vested in said provincial
adoption of the Consti. board or act in substitution thereof, such as by suspending
municipal officials, without the administrative proceedings
3. Respondent also cites section 64 (C) of the RAC, which gives prescribed in sections 2188 to 2190 of the Administrative
the President authority to order an investigation of any action Code, before said board.
or conduct of any person in the government.

SC: The powers specified in this provision are given to the


7. As early as April 7, 1900, President McKinley, in his
Instructions to the Second Philippine Commission, laid down
Pres. “in addition to his general supervisory authority.” It
the policy that our municipal governments should be "subject
follows that the application of these powers to municipal
to the least degree of supervision and control" on the part of
corporations would contravene the consti provision restricting
the national government; that said supervision and control
the authority of the Pres. over local governments to “general
should be "confined within the narrowest limits"; that in the
supervision.”
distribution of powers among the governments to be
organized in the Philippines, "the presumption is always to be
4. Respondent cites 64 (B) of the RAC, which empowers the
in favor of the smaller subdivision"; that the organization of
Executive to remove officials from office conformably to law
local governments should follow "the example of the
…for disloyalty…
distribution of powers between the states and the national
government of the United States"; and that, accordingly, the
SC: First of all, it is not claimed that Hebron is charged with
national government "shall have no direct administration
disloyalty so such provision is not applicable to his case.
except of matters of purely general concern."
Even then, the provision states that the power of removal
must be exercised “conformably to law,” which as regards
If such were the basic principles underlying the organization
municipal officers, is found in sections 2188-2191 of the RAC.
of our local governments, at a time when the same were
Finally, this Court held in the case of Alejandrino vs. Quezon
under the control of the Governor-General (the representative
that the power of removal does not imply the authority to
of the United States, which has delegated to us some
suspend for a substantial period of time (which, in that case,
governmental powers, to be exercised in the name of the
was only 1 year)
United States), with more reason must those principles be
observed under the Constitution of the Philippines, pursuant
5. Re: conflict between section 64, 79 and 86 of the RAC and
to which "sovereignty resides in the (Filipino) people and all
sections 2188-2191 of the same code
government authority emanates from them" and the power of
the President over local governments is limited to "general
SC: Sections 2188-2191, being specific provisions, setting
supervision . . . as may be provided by law."
forth the procedure for disciplinary action that may be taken
against municipal officials, must prevail over sections 64, 79
and 86, which are general provisions dealing with the powers 8. Respondent argues that the authority of the President over
of the President and the dept. heads over the officers of the our municipal corporations is not identical to that of State
government. Governors in the United States, for the former is the
Executive, with more comprehensive powers than those of
6. The alleged authority of the Executive to suspend a municipal the latter, who are merely chief executives
mayor directly, without any opportunity on the part of the
SC: Although this view is accurate (see Severino vs.
provincial governor and the provincial board to exercise the
Governor General case), it is immaterial to the CAB. The
administrative powers of both under sections 2188 to 2190 of
Severino case referred to the authority of the American
the Administrative Code, cannot be adopted without
Governor-General over local governments established in the
conceding that said powers are subject to repeal or
Philippines, as an unincorporated territory or insular
suspension by the President. Obviously, this cannot, and
possession of the United States, which local governments
should not, be done without a legislation of the most explicit
had been placed by McKinley's Instructions under the
and categorical nature, and there is none to such effect.
"control" of said officer. The case at bar deals with the
Moreover, as stated in Mondano vs. Silvosa, said legislation

The Apocryphal Maggots:


Rainier, Chrisgel, Corina, Geoffry, Grace and Sylvie Blanche
The Flibbertigibbet Worms:
Golda, Gladys and Melyjane
CA VE AT: By simply r ea ding this re vi ew er a t the end o f the sem este r wi ll (hopef ully) guar antee you r pas sing this cou rse. D rink mode rat ely .
Amusin S C A r y
gly
THE C2005 LOCAL GOVERNMENT REVIEWER - 105 -
authority of the President of the Philippines, as a full too narrow in scope, it is for the Legislature, and not the
sovereign state, over local governments created by Philippine courts, to expand it.
laws, enacted by representatives of the Filipino people, who
elected said representatives and are the ultimate repository Besides, as said earlier, the issue of WON the executive may
of our sovereignty, in the exercise of which they adopted and suspend municipal officials has never been squarely
promulgated a Constitution, and ordained therein, that, in lieu presented and decided before this Court. All the cases cited
of the power of control of the former Governors-General, our by respondent (as shown above) are not identical to the CAB.
Executive shall merely exercise "general supervision over all
local governments as may be provided by law." CONCLUSIONS:
 Under the present law, the procedure prescribed in Sections
9. Respondent points out that municipal corporations in the US 2188-2191 of the RAC is MANDATORY and EXCLUSIVE
have the power of “local self-government,” which is not given  The Executive may conduct investigations to determine
to our political subdivisions. This means simply that, whereas whether municipal officials are guilty of acts or omissions
the former may not be deprived of their right to local "self- warranting admin. action, as a means to ascertain whether
government", the latter have only such autonomy, if any, as the provincial governor and the board should take such action
the central government may deem fit to grant thereto, and  The Executive may take appropriate measures to compel the
that said autonomy shall be under the control of the national provincial governor and board to take such action, but if they
government, which may decree its increase, decrease, or, fail to do so, the Executive may not deprive the governor and
even, complete abolition the board of the powers under sections 2188-2190 of the
RAC
SC: True. But who shall exercise this power on behalf of the  The authority vested in the Executive under section 2191 is
state? Not the Executive, but the legislature, as an incident of merely appellate in character
its authority to create or abolish municipal corporations, and,
consequently, to define its jurisdiction and functions. (Dean
Vicente Sinco: Local governments are subject to the control Ganzon vs. CA (1991) - supra
of Congress which has the authority to prescribe the
procedure by which the President may perform his Facts:
constitutional power of general supervision.) Series of administrative complaints (10) were filed against Mayor
Ganzon by various city officials sometime in 1988…
10. Respondent claims that the power of general supervision of Mayor Ganzon answered and cases were set for hearing (where a
the President imposes upon him the duty of non-interference series of postponements occurred).
in purely corporate affairs of local governments, but such *Sec. Santos (of Department of Local Government) issued
limitation does not apply to its political affairs. preventive suspension order for 60days after finding probable
grounds and reasons in said hearings.
SC: The limitation of the President’s authority to general *A prima facie evidence found to exist in arbitration case filed by
supervision only is unqualified, and hence, it applies to all Erbite so another preventive suspension for 60days was ordered
powers of municipal corporations, corporate and political by Sec. Santos.
alike. Besides, there was no need to qualify the consti Meanwhile, Sec. Santos issued a third order of preventive
powers of the pres. as regards corporate functions of local suspension for another 60days and designated ViceMayor as
governments, inasmuch as the Executive never had any acting mayor. Ganzon commenced a petition for prohibition in CA.
control over said functions anyway. The same are not, and
never have been, under the control of Congress. In the Ganzon claims:
exercise of corporate functions, municipal corporations are on 1. denial of due process
the same level vis-à-vis the National Government, as private Held: No denial of due process. Ganzon did not show very
corporations. Thus, the limitation of the President’s powers clearly in what manner he might have been deprived of his rights
could have no other purpose than to affect his authority over by Sec. Santos
political functions. NOTE: This theory was already rejected 2. Sec. of Local Government, as President’s alter ego, is
by the Court in Villena vs. Secretary of Interior. without authority to suspend/remove local officials
Ganzon contends: 1987 Constitution (see Sec4) no longer allows
11. Respondent claims that the action of the Executive was President to exercise power of suspension/removal over local
based on the demand of public interest and the seeming officials because:
implication of some of the former decisions of this Court 1. it is meant to strengthen self-rule by local government units

SC: The question is not one of necessity or usefulness, but


2. by deleting the phrase “as may be provided by law” in
of authority or prerogative. If public interest demands it, the previous Constitutions, the President was stipped of power of
remedy is to have the legislative branch correct by control over local governments  since the power of the
appropriate enactment. If the law (in this case, the RAC) is President is “provided by law”, and hence, no law may
provide for it any longer in 1987 Consti

The Apocryphal Maggots:


Rainier, Chrisgel, Corina, Geoffry, Grace and Sylvie Blanche
The Flibbertigibbet Worms:
Golda, Gladys and Melyjane
CA VE AT: By simply r ea ding this re vi ew er a t the end o f the sem este r wi ll (hopef ully) guar antee you r pas sing this cou rse. D rink mode rat ely .
Amusin S C A r y
gly
THE C2005 LOCAL GOVERNMENT REVIEWER - 106 -
3. their view that finds support in Constitutional Commission Has the Constitution repealed Sec62 and 63 of LGC?
debates  NO… “supervision” and “removal” are not incompatible and
Note: Sec. of Local Government meted out suspensions in may stand with the other notwithstanding the stronger expression
consonance with Secs.62 (Notice of Hearing), and 63 (Preventive of local autonomy under the new Constitution, that in spite of the
Suspension) of BP337 (Local Government Code). approval of the Charter, LGC (Batas#337) is still in force and effect
Did the 1987 Constitution, in deleting the phrase”, intend to
divest the President of power to investigate/suspend/ Are the successive orders of suspensions proper?
discipline/remove local officials? Considerations in preventive suspensions:
 NO, notwithstanding the change, charter did not intend to divest 1. A preventive suspension of a duly elected officer may be
the legislature of its right, or the President of his prerogative as justified but its continuance for an unreasonable length of
conferred by existing legislation, to provide administrative time as to outrun the bounds of reason and result in sheer
sanctions against local officials oppression raises a due process question where his right to
- the omission only signifies underscoring local governments’ hold office will be nullified and injustice may be inflicted on the
autonomy from congress and to break Congress “control” people which elected him.
over their affairs 2. It is also because it is out of the ordinary to have a vacancy in
- Consti did not intend, for sake of local autonomy, to deprive local government
legislature of all authority over municipal corporations, in 3. Its sole objective is simply “to prevent the accused from
particular, concerning discipline
hampering the normal cause of the investigation with his
- The deletion was meant to stress, sub silencio, the objective influence and authority over possible witnesses” or to keep
of framers to strengthen local autonomy by severing him off “the records and other evidence.” It is a means to
congressional control of its affairs, as observed by CA, like assist prosecutors in firming up a case, if any, against an
the power of local legislation… BUT it did nothing more. erring local official.
Insofar as existing legislation authorizes the President 4. Under LGC, it cannot exceed 60days, which is to say that it
(through Sec of Local Government) to proceed against local need not be exactly 60days long if a shorter period is
officials administratively, Consti contains no prohibition. otherwise sufficient, and also to say that it ought to be lifted if
prosecutors have achieved their purpose in a shorter span.
What is LOCAL AUTONOMY? 5. It is not a penalty and not unlike preventive imprisonment in
Under the Constitution, it is not instantly self-executing, but which accused is held to insure his presence at trial. Both
subject to, among other things, the passage of a local enjoy a presumption of innocence.
government code (ArtX S3), a local tax law (S5,6), income 6. It is temporary. A longer suspension is unjust and
distribution legislation (S7), and a national representation law unreasonable, and nothing less than tyranny.
(S9), and measures (S14) designed to realize autonomy at the  Although President (through Sec) is not precluded from
local level. exercising a legal power, it appears that Sec of Interior is
In spite of autonomy, the Constitution places the local exercising it oppressively and with a grave abuse of
government under the general supervision of the Executive. discretion…
The charter also allows Congress to include in the local Held: Ganzon to suffer duration of 3rd suspension (which is the one
government code provisions for removal of local officials  under question), but as to the remaining 7complaints, Department
suggesting that Congress may exercise removal powers. As the of Local Government is urged to undertake steps to expedite them,
existing Local Government Code has done, it has been subject to Ganzon’s usual remedies if warranted. Meanwhile, Sec.
delegated to the President (see ArtX, Sec3). is precluded from meting out further suspensions based on
remaining complaints, notwithstanding findings of prima facie
But the power of investigation and disciplinary authority are evidence.
not included in supervisory powers? Ganzon may serve suspension so far ordered, but may no longer
 A mistaken impression, because legally, “supervision” is not be suspended for offenses he was charged originally, provided: (1)
incompatible with disciplinary authority as held in Mondano vs. delays in investigation due to his fault, neglect or request shall not
Silvosa… be counted in computing time of suspension (S63(3), LGC); (2) if
- the impression has apparently been worsened by rulings in during, or after expiration of, his suspension, he commits another
Lacson vs. Roque, Hebron vs. Reyes, Mondano vs. Silvosa, crime/abuse for which proper charges are filed against him, his
and Pelaez vs. Auditor General. The Court denied the previous suspension shall not be a bar to another preventive
President power (to suspend/remove) but it was not because suspension, if warranted under Sec63(b) of LGC.
of the thought that the President cannot exercise it on
account of his limited power, but because the law lodged the RULES laid down by SC:
power elsewhere. 1. Local autonomy, under Consti, involves a mere
- Where in cases in which the law gave the President power decentralization of administration, not of power, in which
like Ganzon vs. Kayanan, it found little difficulty in sustaining local officials remain accountable to the central government
him. in the manner the law may provide.
2. The new Consti does not prescribe federalism.

The Apocryphal Maggots:


Rainier, Chrisgel, Corina, Geoffry, Grace and Sylvie Blanche
The Flibbertigibbet Worms:
Golda, Gladys and Melyjane
CA VE AT: By simply r ea ding this re vi ew er a t the end o f the sem este r wi ll (hopef ully) guar antee you r pas sing this cou rse. D rink mode rat ely .
Amusin S C A r y
gly
THE C2005 LOCAL GOVERNMENT REVIEWER - 107 -
3. The change in constitutional language (w/respect to performing their functions is accordance with law, that is, with the
supervision clause) was meant to deny legislative control prescribed procedure for the enactment of tax ordinances and the
over local governments; it did not exempt the latter from grant of powers to the city government under the Local
legislative regulations provided regulation is consistent with Government Code. As we see it, that was an act not of control but
the fundamental premise of autonomy. of mere supervision.
4. Since local governments remain accountable to the
national authority, the latter may, and in the manner set An officer in control lays down the rules in the doing of an act. It
forth therein, impose disciplinary action against local they are not followed, he may, in his discretion, order the act
officials; undone or re-done by his subordinate or he may even decide to do
5. “Supervision” and “investigation” are not inconsistent it himself. Supervision does not cover such authority. The
terms; “investigation” does not signify “control” (which supervisor or superintendent merely sees to it that the rules are
President does not have). followed, but he himself does not lay down such rules, nor does he
have the discretion to modify or replace them. If the rules are not
observed, he may order the work done or re-done but only to
conform to the prescribed rules. He may not prescribe his own
Drilon vs. Lim (1994)
manner for the doing of the act. He has no judgment on this matter
except to see to it that the rules are followed. In the opinion of the
Facts: Court, Secretary Drilon did precisely this, and no more nor less
Pursuant to Sec 187 of the LGC, the Secretary of Justice had, on than this, and so performed an act not of control but of mere
appeal to him of four oil companies and a taxpayer, declared supervision.
Ordinance No. 7794, otherwise known as the Manila Revenue
Code, null and void for non-compliance with the prescribed
procedure in the enactment of tax ordinances and for containing
certain provisions contrary to law and public policy.
Consultations
Secretary argues that the annulled Section 187 is constitutional
and that the procedural requirements for the enactment of tax
ordinances as specified in the Local Government Code has indeed Sec2, LGC: Declaration of Policy.
not been observed. xxx
(c) It is likewise the policy of the State to require all national
Issue: Constitutionality of Sec. 187 of the LGC agencies and offices to conduct periodic consultations with
Held: Constitutional appropriate local government units, nongovernmental and
Every court, including this Court, is charged with the duty of a people's organizations, and other concerned sectors of the
purposeful hesitation before declaring a law unconstitutional, on community before any project or program is implemented in
the theory that the measure was first carefully studied by the their respective jurisdictions.
executive and the legislative departments and determined by them
to be in accordance with the fundamental law before it was finally Sec26: Duty of National Government Agencies in the
approved. Maintenance of Ecological Balance. — It shall be the duty of
every national agency or government-owned or controlled
To doubt is to sustain. The presumption of constitutionality can be corporation authorizing or involved in the planning and
overcome only by the clearest showing that there was indeed an implementation of any project or program that may cause
infraction of the Constitution, and only when such a conclusion is pollution, climatic change, depletion of non-renewable
reached by the requipped majority may the Court pronounce, in the resources, loss of crop land, rangeland, or forest cover, and
discharge of the duty it cannot escape, that the challenged act extinction of animal or plant species, to consult with the local
must be struck down. government units, nongovernmental organizations, and other
sectors concerned and explain the goals and objectives of the
In CAB, the RTC was rather hasty in invalidating the provision. project or program, its impact upon the people and the
Section 187 authorizes the Secretary of Justice to review only the community in terms of environmental or ecological balance, and
constitutionality or legality of the tax ordinance and, if warranted, to the measures that will be undertaken to prevent or minimize the
revoke it on either or both of these grounds. When he alters or adverse effects thereof. aisa dc
modifies or sets aside a tax ordinance, he is not also permitted to
substitute his own judgment for the judgment of the local Sec27: Prior Consultations Required. — No project or program
government that enacted the measure. shall be implemented by government authorities unless the
consultations mentioned in Sections 2 (c) and 26 hereof are
Secretary Drilon did set aside the Manila Revenue Code, but he complied with, and prior approval of the sanggunian concerned
did not replace it with his own version of what the Code should be. is obtained: Provided, That occupants in areas where such
He did not pronounce the ordinance unwise or unreasonable as a projects are to be implemented shall not be evicted unless
basis for its annulment. He did not say that in his judgment it was a appropriate relocation sites have been provided, in accordance
bad law. What he found only was that it was illegal. All he did in with the provisions of the Constitution.
reviewing the said measure was determine if the petitioners were

The Apocryphal Maggots:


Rainier, Chrisgel, Corina, Geoffry, Grace and Sylvie Blanche
The Flibbertigibbet Worms:
Golda, Gladys and Melyjane
CA VE AT: By simply r ea ding this re vi ew er a t the end o f the sem este r wi ll (hopef ully) guar antee you r pas sing this cou rse. D rink mode rat ely .
Amusin S C A r y
gly
THE C2005 LOCAL GOVERNMENT REVIEWER - 108 -

Memorandum Circular No.52 (1993) - Calling Attention To


And Enjoining Strict Compliance With The Provisions Of
The Local Government Code Of 1991 (R.A. No. 7160)
Requiring Mandatory Coordination And Consultation With
Local Government Units, Non-Governmental And People's
Organizations And Other Concerned Sectors Lina vs. Paño (2001)

"SECTION 26. Duty of National Government Agencies in the Facts:


Maintenance of Ecological Balance. — It shall be the duty of every Respondent Calvento was appointed agent by the PCSO to install
national agency or government-owned or controlled corporation a terminal for the operation of lotto in San Pedro, Laguna.
authorizing or involved in the planning and implementation of any He applied for a mayor's permit but this was denied due to an
project or program that may cause pollution, climatic change, ordinance passed by the Sangguniang Panlalawigan entitled
depletion of non-renewable resources, loss of crop land, Kapasiyahan BLG. 508.
rengeland, or forest cover, and extinction of animal or plant
species, to consult with the local government units, non- Respondent filed a complaint for declaratory relief with prayer for
governmental organizations, and other sectors concerned and preliminary mandatory injunction and TRO.
explain the goals and objectives of the project or program, its
impact upon the people and the community in terms of TC rendered decision in favor of respondent. Petitioners filed
environmental or ecological balance, and the measures that will be M4Recon which was denied.
undertaken to prevent or minimize the adverse effects thereof."
Issue: WON prior consultations and approval by the concerned
"SECTION 27. Prior Consultations Required. — No project or Sanggunian are needed before the operation of a lotto system can
program shall be implemented by government authorities unless be allowed.
the consultations mentioned in Sections 2(c) and 26 hereof are Held: NO
complied with, and prior approval of the sanggunian concerned is Sec 2 (c) and 27 apply only to national programs and?or projects
obtained: Provided, That occupants in areas where such projects which are ot be implemented ona particular local community. Lotto
are to be implemented shall not be evicted unless appropriate is neither a program nor a project of the NG but of a charitable
relocation sites have been provided, in accordance with the institution, the PCSO.
provisions of the Constitution." Section 27, read in conjunction with Sec 26 would show that the
projects and programs mentioned in Sec 27 are those that
Accordingly, all officers and employees of national government a) may affect pollution
agencies and offices, including concerned government owned and b) may bring climatic change
controlled corporations are hereby enjoined to strictly comply with c) may cause the depletion of non-renewable resources,
the foregoing provisions of the Local Government Code as well as d) may result in loss of crop land, range-land, or forest
the pertinent provisions of its Implementing Rules and Regulations. cover
DONE in the City of Manila, this 2nd day of June, in the year of e) may eradicate certain animal or plant species from the
Our Lord, Nineteen Hundred and Ninety-Three. face of the planet
f) other programs and projects that may call for the eviction
of a particular group of people
RA 8975 (2000) - An Act To Ensure The Expeditious None of these effects will be produced by the introduction of the
Implementation And Completion Of Government lotto in Laguna.
Infrastructure Projects By Prohibiting Lower Courts From Argument of lack of prior consultation and approval is a mere
Issuing Temporary Restraining Orders, Preliminary afterthought as it was not indicated in the letter of refusal to issue a
Injunctions Or Preliminary Mandatory Injunctions, mayor's permit.
Providing Penalties For Violations Thereof, And For Other
Purposes DGNotes:
not a sound decision! Sec 2 is a broader provision which should
Sec7: Issuance of Permits. — Upon payment in cash of the not be limited to provisions of Secs. 26 and 27.
necessary fees levied under Republic Act No. 7160, as
amended, otherwise known as the Local Government Code of
1991, the governor of the province or mayor of a highly-
urbanized city shall immediately issue the necessary permit to
extract sand, gravel and other quarry resources needed in Relations with Philippine National Police
government projects. The issuance of said permit shall consider
environmental laws, land use ordinances and the pertinent Sec28: Powers of Local Chief Executives over the Units of the
provisions of the Local Government Code relating to Philippine National Police. — The extent of operational
environment.

The Apocryphal Maggots:


Rainier, Chrisgel, Corina, Geoffry, Grace and Sylvie Blanche
The Flibbertigibbet Worms:
Golda, Gladys and Melyjane
CA VE AT: By simply r ea ding this re vi ew er a t the end o f the sem este r wi ll (hopef ully) guar antee you r pas sing this cou rse. D rink mode rat ely .
Amusin S C A r y
gly
THE C2005 LOCAL GOVERNMENT REVIEWER - 109 -
supervision and control of local chief executives over the police (1) of whom shall be designated as vice chairperson by the
force, fire protection unit, and jail management personnel President. The fourth regular commissioner shall come from the
assigned in their respective jurisdictions shall be governed by law enforcement sector either active or retired: Provided, That
the provisions of Republic Act Numbered Sixty-nine hundred an active member of a law enforcement agency shall be
seventy-five (R.A. No. 6975), otherwise known as "The considered resigned from said agency once appointed to the
Department of the Interior and Local Government Act of 1990", Commission: Provided, further, That at least one (1) of the
and the rules and regulations issued pursuant thereto. Commissioners shall be a woman. The Secretary of the
Department shall be the ex-officio Chairperson of the
Commission, while the Vice Chairperson shall act as the
RA 6975 (1990) as amended by RA 8551 (1998) executive officer of the Commission."

Sec5: Section 14 of Republic Act No. 6975 is hereby amended to


RA 8551 (1998) – An Act Providing For The Reform And
read as follows:
Reorganization Of The Philippine National Police And For
"SEC. 14. Powers and Functions of the Commission. — The
Other Purposes, Amending Certain Provisions Of Republic
Commission shall exercise the following powers and functions:
Act Numbered Sixty-Nine Hundred And Seventy-Five
"(a) Exercise administrative control and operational
Entitled, "An Act Establishing The Philippine National
supervision over the Philippine National Police which shall mean
Police Under A Re-Organized Department Of The Interior
the power to:
And Local Government, And For Other Purposes
"1) Develop policies and promulgate a police manual
prescribing rules and regulations for efficient
TITLE I: TITLE AND DECLARATION OF POLICY organization, administration, and operation, including
Sec1. Title. —xxx criteria for manpower allocation, distribution and
deployment, recruitment, selection, promotion, and
Sec2: Declaration of Policy and Principles. xxx retirement of personnel and the conduct of qualifying
[rationale of law: to institutionalize police force and prevent entrance and promotional examinations for uniformed
bosissm. members;
"2) Examine and audit, and thereafter establish the
TITLE II: THE ROLE OF THE PNP IN COUNTER-INSURGENCY standards for such purposes on a continuing basis, the
FUNCTIONS performance, activities, and facilities of all police
Sec3: Section 12 of Republic Act No. 6975 is hereby amended to agencies throughout the country;
read as follows: "3) Establish a system of uniform crime reporting;
"SEC. 12. Relationship of the Department with the "4) Conduct an annual self-report survey and compile
Department of National Defense. — The Department of the statistical data for the accurate assessment of the crime
Interior and Local Government shall be relieved of the primary situation and the proper evaluation of the efficiency and
responsibility on matters involving the suppression of insurgency effectiveness of all police units in the country;
and other serious threats to national security. The Philippine "5) Approve or modify plans and programs on education and
National Police shall, through information gathering and training, logistical requirements, communications,
performance of its ordinary police functions, support the Armed records, information systems, crime laboratory, crime
Forces of the Philippines on matters involving suppression of prevention and crime reporting;
insurgency, except in cases where the President shall call on the "6) Affirm, reverse or modify, through the National Appellate
PNP to support the AFP in combat operations. Board, personnel disciplinary actions involving demotion
"In times of national emergency, the PNP, the Bureau of Fire or dismissal from the service imposed upon members of
Protection, and the Bureau of Jail Management and Penology the Philippine National Police by the Chief of the
shall, upon the direction of the President, assist the armed Philippine National Police;
forces in meeting the national emergency." "7) Exercise appellate jurisdiction through the regional
appellate boards over administrative cases against
TITLE III: THE NATIONAL POLICE COMMISSION policemen and over decisions on claims for police
Sec4: Section 13 of Republic Act No. 6975 is hereby amended to benefits;
read as follows: "8) Prescribe minimum standards for arms, equipment, and
"SEC. 13. Creation and Composition. — A National Police uniforms and, after consultation with the Philippine
Commission, hereinafter referred to as the Commission, is Heraldry Commission, for insignia of ranks, awards, and
hereby created for the purpose of effectively discharging the medals of honor. Within ninety (90) days from the
functions prescribed in the Constitution and provided in this Act. effectivity of this Act, the standards of the uniformed
The Commission shall be an agency attached to the Department personnel of the PNP must be revised which should be
for policy and program coordination. It shall be composed of a clearly distinct from the military and reflective of the
Chairperson, four (4) regular Commissioners, and the Chief of civilian character of the police;
PNP as ex-officio member. Three (3) of the regular "9) Issue subpoena and subpoena duces tecum in matters
commissioners shall come from the civilian sector who are pertaining to the discharge of its own powers and duties,
neither active nor former members of the police or military, one

The Apocryphal Maggots:


Rainier, Chrisgel, Corina, Geoffry, Grace and Sylvie Blanche
The Flibbertigibbet Worms:
Golda, Gladys and Melyjane
CA VE AT: By simply r ea ding this re vi ew er a t the end o f the sem este r wi ll (hopef ully) guar antee you r pas sing this cou rse. D rink mode rat ely .
Amusin S C A r y
gly
THE C2005 LOCAL GOVERNMENT REVIEWER - 110 -
and designate who among its personnel can issue such incapacity of the chairperson, the Vice chair shall serve as
processes and administer oaths in connection therewith; Chairperson until the Chairperson is present or regains capacity
"10) Inspect and assess the compliance of the PNP on the to serve. In case of death or permanent incapacity or
established criteria for manpower allocation, distribution, disqualification of the chairperson, the acting chairperson shall
and deployment and their impact on the community and also act as such until a new chairperson shall have been
the crime situation, and thereafter formulate appropriate appointed by the President and qualified."
guidelines for maximization of resources and effective
utilization of the PNP personnel; Sec10: Section 20 of Republic Act No. 6975 is hereby amended to
"11) Monitor the performance of the local chief executives as read as follows:
deputies of the Commission; and "SEC. 20. Organizational Structure. — The Commission shall
"12) Monitor and investigate police anomalies and consist of the following units:
irregularities. "(a) Commission Proper. — This is composed of the
"b) Advise the President on all matters involving police offices of the Chairman and four (4) Commissioners.
functions and administration; "(b) Staff Services. — The staff services of the
"c) Render to the President and to the Congress an Commission shall be as follows:
annual report on its activities and accomplishments during the "(1) The Planning and Research Service, which shall provide
thirty (30) days after the end of the calendar year, which shall technical services to the Commission in areas of overall
include an appraisal of the conditions obtaining in the policy formulation, strategic and operational planning,
organization and administration of police agencies in the management systems or procedures, evaluation and
municipalities, cities and provinces throughout the country, and monitoring of the Commission's programs, projects and
recommendations for appropriate remedial legislation; internal operations; and shall conduct thorough research
"d) Recommend to the President, through the Secretary, and analysis on social and economic conditions affecting
within sixty (60) days before the commencement of each peace and order in the country;
calendar year, a crime prevention program; and "(2) The Legal Affairs Service, which shall provide the
"e) Perform such other functions necessary to carry out Commission with efficient and effective service as legal
the provisions of this Act and as the President may direct." counsel of the Commission; draft or study contracts
affecting the Commission and submit appropriate
Sec6: Section 15 of Republic Act No. 6975 is hereby amended to recommendations pertaining thereto; and render legal
read as follows: opinions arising from the administration and operation of
"SEC. 15. Qualifications. — No person shall be appointed the Philippine National Police and the Commission;
regular member of the Commission unless: "(3) The Crime Prevention and Coordination Service, which
"(a) He or she is a citizen of the Philippines; shall undertake criminological researches and studies;
"(b) A member of the Philippine Bar with at least five (5) formulate a national crime prevention plan; develop a
years experience in handling criminal or human rights cases or crime prevention and information program and provide
a holder of a master's degree but preferably a doctorate degree editorial direction for all criminology research and crime
in public administration, sociology, criminology, criminal justice, prevention publications;
law enforcement, and other related disciplines; and "(4) The Personnel and Administrative Service, which shall
"(c) The regular member coming from the law perform personnel functions for the Commission,
enforcement sector should have practical experience in law administer the entrance and promotional examinations
enforcement work for at least five (5) years while the three (3) for policemen, provide the necessary services relating to
other regular commissioners must have done extensive records, correspondence, supplies, property and
research work or projects on law enforcement, criminology or equipment, security and general services, and the
criminal justice or members of a duly registered non-government maintenance and utilization of facilities, and provide
organization involved in the promotion of peace and order." services relating to manpower, career planning and
development, personnel transactions and employee
Sec7: Section 16 of Republic Act No. 6975 is hereby amended to welfare;
read as follows: "(5) The Inspection, Monitoring and Investigation Service,
"SEC. 16. Term of Office. — The four (4) regular and full-time which shall conduct continuous inspection and
Commissioners shall be appointed by the President for a term of management audit of personnel, facilities and operations
six (6) years without re-appointment or extension." at all levels of command of the PNP, monitor the
implementation of the Commission's programs and
Sec8: Expiration of the Terms of Office of Current projects relative to law enforcement; and monitor and
Commissioners. xxx held unconstitutional under Canonizado. investigate police anomalies and irregularities;
"(6) The Installations and Logistics Service, which shall
Sec9: Section 17 of Republic Act No. 6975 is hereby amended to review the Commission's plans and programs and
read as follows: formulate policies and procedures regarding acquisition,
"SEC. 17. Temporary or Permanent Incapacity of the inventory, control, distribution, maintenance and disposal
Chairperson. — In case of absence due to the temporary of supplies and shall oversee the implementation of

The Apocryphal Maggots:


Rainier, Chrisgel, Corina, Geoffry, Grace and Sylvie Blanche
The Flibbertigibbet Worms:
Golda, Gladys and Melyjane
CA VE AT: By simply r ea ding this re vi ew er a t the end o f the sem este r wi ll (hopef ully) guar antee you r pas sing this cou rse. D rink mode rat ely .
Amusin S C A r y
gly
THE C2005 LOCAL GOVERNMENT REVIEWER - 111 -
programs on transportation facilities and installations "c) Must have passed the psychiatric/psychological, drug
and the procurement and maintenance of supplies and and physical tests to be administered by the PNP or by any
equipment; and NAPOLCOM accredited government hospital for the purpose of
"(7) The Financial Service, which shall provide the determining physical and mental health;
Commission with staff advice and assistance on "d) Must possess a formal baccalaureate degree from a
budgetary and financial matters, including the recognized institution of learning;
overseeing of the processing and disbursement of funds "e) Must be eligible in accordance with the standards set
pertaining to the scholarship program and surviving by the Commission;
children of deceased and/or permanently incapacitated "f) Must not have been dishonorably discharged from
PNP personnel. military employment or dismissed for cause from any civilian
"(c) Disciplinary Appellate Boards — The Commission position in the Government;
shall establish a formal administrative disciplinary appellate "g) Must not have been convicted by final judgment of an
machinery consisting of the National Appellate Board and the offense or crime involving moral turpitude;
regional appellate boards. "h) Must be at least one meter and sixty-two centimeters
"The National Appellate Board shall decide cases on appeal (1.62 m.) in height for male and one meter and fifty-seven
from decisions rendered by the PNP chief, while the regional centimeters (1.57 m.) for female;
appellate boards shall decide cases on appeal from decisions "i) Must weigh not more or less than five kilograms (5
rendered by officers other than the PNP chief, the mayor, and kgs.) from the standard weight corresponding to his or her
the People's Law Enforcement Board (PLEB) created height, age, and sex; and
hereunder." "j) For a new applicant, must not be less than twenty-one
(21) nor more than thirty (30) years of age: except for the last
Sec11: Section 22 of Republic Act No. 6975 is hereby amended to qualification, the above-enumerated qualifications shall be
read as follows: continuing in character and an absence of any one of them at
"SEC. 22. Qualifications of Regional Directors. — XXX any given time shall be a ground for separation or retirement
from the service: Provided, That PNP members who are already
Sec12: Qualifications Upgrading Program. — XXX in the service upon the effectivity of this Act shall be given at
least two (2) more years to obtain the minimum educational
TITLE IV: THE PHILIPPINE NATIONAL POLICE qualification and one (1) year to satisfy the weight requirement.
A. REORGANIZATION "For the purpose of determining compliance with the
Sec13: Authority of the Commission to Reorganize the PNP. — requirements on physical and mental health, as well as the non-
Notwithstanding the provisions of Republic Act No. 6975 on the use of prohibited drugs, the PNP by itself or through a
organizational structure and rank classification of the PNP, the NAPOLCOM accredited government hospital shall conduct
Commission shall conduct a management audit, and prepare regular psychiatric, psychological drug and physical tests
and submit to Congress a proposed reorganization plan of the randomly and without notice.
PNP not later than December 31, 1998, subject to the limitations "After the lapse of the time period for the satisfaction of a
provided under this Act and based on the following criteria: a) specific requirement, current members of the PNP who will fail
increased police visibility through dispersal of personnel from to satisfy any of the requirements enumerated under this
the headquarters to the field offices and by the appointment and Section shall be separated from the service if they are below
assignment of non-uniformed personnel to positions which are fifty (50) years of age and have served in Government for less
purely administrative, technical, clerical or menial in nature and than twenty (20) years or retired if they are from the age of fifty
other positions which are not actually and directly related to (50) and above and have served the Government for at least
police operation; and b) efficient and optimized delivery of police twenty (20) years without prejudice in either case to the
services to the communities. payment of benefits they may be entitled to under existing laws."
The PNP reorganization program shall be approved by
Congress through a joint resolution. Sec15: Waivers for Initial Appointments to the PNP. — The
age, height, weight, and educational requirements for initial
appointment to the PNP may be waived only when the number
of qualified applicants fall below the minimum annual quota:
B. QUALIFICATIONS UPGRADING Provided, That an applicant shall not be below twenty (20) nor
Sec14: Section 30 of Republic Act No. 6975 is hereby amended to over thirty-five (35) years of age: Provided, further, That any
read as follows: applicant not meeting the weight requirement shall be given
"SEC. 30. General Qualifications for Appointment. — No reasonable time but not exceeding six (6) months within which
person shall be appointed as officer or member of the PNP to comply with the said requirement: Provided, furthermore, That
unless he or she possesses the following minimum only applicants who have finished second year college or have
qualifications: earned at least seventy-two (72) collegiate units leading to a
"a) A citizen of the Philippines; bachelor's decree shall be eligible for appointment: Provided,
"b) A person of good moral conduct; furthermore, That anybody who will enter the service without a
baccalaureate degree shall be given a maximum of four (4)

The Apocryphal Maggots:


Rainier, Chrisgel, Corina, Geoffry, Grace and Sylvie Blanche
The Flibbertigibbet Worms:
Golda, Gladys and Melyjane
CA VE AT: By simply r ea ding this re vi ew er a t the end o f the sem este r wi ll (hopef ully) guar antee you r pas sing this cou rse. D rink mode rat ely .
Amusin S C A r y
gly
THE C2005 LOCAL GOVERNMENT REVIEWER - 112 -
years to obtain the required educational qualification: Provided, "SEC. 32. Examinations of Policemen. — The National Police
finally, That a waiver for height requirement shall be Commission shall administer the entrance and promotional
automatically granted to applicants belonging to the cultural examinations for policemen on the basis of the standards set by
communities. the Commission."

Sec16: Selection Criteria Under the Waiver Program. — The Sec22: Section 34 of Republic Act No. 6975 is hereby amended to
selection of applicants under the Waiver Program shall be read as follows:
subject to the following minimum criteria: "SEC. 34. Qualifications of Chief of City and Municipal
a) Applicants who possess the least disqualification shall Police Stations. — No person shall be appointed chief of a city
take precedence over those who possess more police station unless he/she is a graduate of Bachelor of Laws
disqualifications. or has finished all the required courses of a master's degree
b) The requirements shall be waived in the following program in public administration, criminology, criminal justice,
order: (a) age, (b) height, (c) weight, and (d) education. law enforcement, national security administration, defense
The Commission shall promulgate rules and regulations to studies, and other related disciplines from a recognized
address other situations arising from the waiver of the entry institution of learning. No person shall be appointed chief of a
requirements. municipal police station unless he or she has finished at least
second year Bachelor of Laws or has earned at least twelve (12)
Sec17: Nature of Appointment Under a Waiver Program. — units in a master's degree program in public administration,
Any PNP uniformed personnel who is admitted due to the waiver criminology, criminal justice, law enforcement, national security
of the educational or weight requirements shall be issued a administration, and other related disciplines from a recognized
temporary appointment pending the satisfaction of the institution of learning: Provided, That members of the Bar with at
requirement waived. Any member who will fail to satisfy any of least five (5) years of law practice, licensed criminologists or
the waived requirements within the specified time periods under graduates of the Philippine National Police Academy and who
Section 13 of this Act shall be dismissed from the service. possess the general qualifications for initial appointment to the
PNP shall be qualified for appointment as chief of a city or
Sec18: Re-application of Dismissed PNP Members Under a municipal police station: Provided, further, That the appointee
Waiver Program. — Any PNP member who shall be dismissed has successfully passed the required field training program and
under a waiver program shall be eligible to re-apply for has complied with other requirements as may be established by
appointment to the PNP: Provided, That he or she possesses the Commission: Provided, furthermore, That the chief of police
the minimum qualifications under Section 14 of this Act and his shall be appointed in accordance with the provisions of Section
or her reappointment is not by virtue of another waiver program. 51, paragraph (b), subparagraph 4(i) of this Act."

Sec19: The Field Training Program. — All uniformed members of Sec23: Qualifications Upgrading Program. — The Commission
the PNP shall undergo a Field Training Program for twelve (12) shall design and establish a qualifications upgrading program for
months involving actual experience and assignment in patrol, the Philippine National Police officers and members in
traffic, and investigation as a requirement for permanency of coordination with the Civil Service Commission, and the
their appointment. Commission on Higher Education through a distance education
program and/or an in-service education program or other similar
Sec20: Increased Qualifications for Provincial Directors. — No programs within ninety (90) days from the effectivity of this Act.
person may be appointed Director of a Provincial Police Office
unless: C. ATTRITION SYSTEM FOR UNIFORMED PERSONNEL
a) he or she holds a master's degree in public Sec24: Attrition System. — There shall be established a system
administration, sociology, criminology, criminal justice, law of attrition within the uniformed members of the PNP within one
enforcement, national security administration, defense studies, (1) year from the effectivity of this Act to be submitted by the
or other related discipline from a recognized institution of PNP to the Commission for approval. Such attrition system shall
learning; and include but is not limited to the provisions of the following
b) has satisfactorily passed the required training and sections.
career courses necessary for the position as may be established
by the Commission. Sec25: Attrition by Attainment of Maximum Tenure in Position.
Any PNP personnel who is currently occupying the position but — The maximum tenure of PNP members holding key positions
lacks any of the qualifications mentioned above shall be given is hereby prescribed as follows:
three (3) years upon the effectivity of this Act to comply with the POSITION MAXIMUM TENURE
requirements; otherwise he or she shall be relieved from the Chief four (4) years
position. Deputy Chief four (4) years
Director of the Staff Services four (4) years
Sec21: Section 32 of Republic Act No. 6975 is hereby amended to Regional Directors six (6) years
read as follows: Provincial/City Directors nine (9) years

The Apocryphal Maggots:


Rainier, Chrisgel, Corina, Geoffry, Grace and Sylvie Blanche
The Flibbertigibbet Worms:
Golda, Gladys and Melyjane
CA VE AT: By simply r ea ding this re vi ew er a t the end o f the sem este r wi ll (hopef ully) guar antee you r pas sing this cou rse. D rink mode rat ely .
Amusin S C A r y
gly
THE C2005 LOCAL GOVERNMENT REVIEWER - 113 -
Other positions higher than Provincial Director shall have the D. PROMOTION SYSTEM
maximum tenure of six (6) years. Unless earlier separated, Sec31: Rationalized Promotion System. — Within six (6) months
retired or promoted to a higher position in accordance with the after the effectivity of this Act, the Commission shall establish a
PNP Staffing Pattern, police officers holding the above- system of promotion for uniformed and non-uniformed personnel
mentioned positions shall be compulsorily retired at the of the PNP which shall be based on merits and on the
maximum tenure in position herein prescribed, or at age fifty-six availability of vacant positions in the PNP staffing pattern. Such
(56), whichever is earlier: Provided, That in times of war or other system shall be gender fair and shall ensure that women
national emergency declared by Congress, the President may members of the PNP shall enjoy equal opportunity for promotion
extend the PNP Chief's tour of duty: Provided, further, That PNP as that of men.
members who have already reached their maximum tenure
upon the effectivity of this Act may be allowed one (1) year more Sec32: Promotion by Virtue of Position. — Any PNP personnel
of tenure in their positions before the maximum tenure provided designated to any key position whose rank is lower than that
in this Section shall be applied to them, unless they shall have which is required for such position shall, after six (6) months of
already reached the compulsory retirement age of fifty-six (56), occupying the same, be entitled to a rank adjustment
in which case the compulsory retirement age shall prevail. corresponding to the position: Provided, That the personnel shall
Except for the Chief, PNP, no PNP member who has less than not be reassigned to a position calling for a higher rank until
one (1) year of service before reaching the compulsory after two (2) years from the date of such rank adjustment:
retirement age shall be promoted to a higher rank or appointed Provided, further, That any personnel designated to the position
to any other position. who does not possess the established minimum qualifications
therefor shall occupy the same temporarily for not more than six
Sec26: Attrition by Relief . — A PNP uniformed personnel who (6) months without reappointment or extension.
has been relieved for just cause and has not been given an
assignment within two (2) years after such relief shall be retired Sec33: Section 38 (a) and (b) of Republic Act No. 6975 is hereby
or separated. amended to read as follows:
"SEC. 38. Promotions. — (a) A uniformed member of the PNP
Sec27: Attrition by Demotion in Position or Rank. — Any PNP shall not be eligible for promotion to a higher position or rank
personnel, civilian or uniformed, who is relieved and assigned to unless he or she has successfully passed the corresponding
a position lower than what is established for his or her grade in promotional examination given by the Commission, or the Bar,
the PNP staffing pattern and who shall not be assigned to a or the corresponding board examinations for technical services
position commensurate to his or her grade within eighteen (18) and other professions, has satisfactorily completed the
months after such demotion in position shall be retired or appropriate and accredited course in the PNPA or equivalent
separated. training institutions, and has satisfactorily passed the required
psychiatric/psychological and drug tests. In addition, no
Sec28: Attrition by Non-promotion. — Any PNP personnel who uniformed member of the PNP shall be eligible for promotion
has not been promoted for a continuous period of ten (10) years during the pendency of his or her administrative and/or criminal
shall be retired or separated. case or unless he or she has been cleared by the People's Law
Enforcement Board (PLEB), and the Office of the Ombudsman
Sec29: Attrition by Other Means. — A PNP member or officer of any complaints proffered against him or her, if any. prcd
with at least five (5) years of accumulated active service shall be "(b) Any uniformed member of the PNP who has exhibited
separated based on any of the following factors: acts of conspicuous courage and gallantry at the risk of his/her
a) inefficiency based on poor performance during the last life above and beyond the call of duty, shall be promoted to the
two (2) successive annual rating periods; next higher rank: Provided, That such acts shall be validated by
b) inefficiency based on poor performance for three (3) the Commission based on established criteria."
cumulative annual rating periods;
c) physical and/or mental incapacity to perform police E. UPGRADING OF SALARIES AND BENEFITS
functions and duties; or Sec34. Section 75 of the same Act is hereby amended to read as
d) failure to pass the required entrance examinations follows:
twice and/or finish the required career courses except for "SEC. 75. Retirement Benefits. — XXX
justifiable reasons.
Sec35: Section 73 of the same Act is hereby amended to read as
Sec30: Retirement or Separation Under the Preceding follows:
Sections. — Any personnel who is dismissed from the PNP "SEC. 73. Permanent Physical Disability. — An officer or non-
pursuant to Sections 25, 26, 27, 28 and 29 hereof shall be officer who is permanently and totally disabled as a result of
retired if he or she has rendered at least twenty (20) years of injuries suffered or sickness contracted in the performance of his
service and separated if he or she has rendered less than duty as duly certified by the National Police Commission, upon
twenty (20) years of service unless the personnel is disqualified finding and certification by the appropriate medical officer, that
by law to receive such benefits. the extent of the disability or sickness renders such member
unfit or unable to further perform the duties of his position, shall

The Apocryphal Maggots:


Rainier, Chrisgel, Corina, Geoffry, Grace and Sylvie Blanche
The Flibbertigibbet Worms:
Golda, Gladys and Melyjane
CA VE AT: By simply r ea ding this re vi ew er a t the end o f the sem este r wi ll (hopef ully) guar antee you r pas sing this cou rse. D rink mode rat ely .
Amusin S C A r y
gly
THE C2005 LOCAL GOVERNMENT REVIEWER - 114 -
be entitled to one year's salary and to lifetime pension d) submit a periodic report on the assessment, analysis,
equivalent to eighty percent (80%) of his last salary, in addition and evaluation of the character and behavior of PNP personnel
to other benefits as provided under existing laws. and units to the Chief PNP and the Commission;
"Should such member who has been retired under permanent e) file appropriate criminal cases against PNP members
total disability under this section die within five (5) years from his before the court as evidence warrants and assist in the
retirement, his surviving legal spouse or if there be none, the prosecution of the case;
surviving dependent legitimate children shall be entitled to the f) provide assistance to the Office of the Ombudsman in
pension for the remainder of the five (5) years guaranteed cases involving the personnel of the PNP.
period." The IAS shall also conduct, motu proprio, automatic
investigation of the following cases:
Sec36: Section 36 of Republic Act No. 6975 is hereby amended to a) incidents where a police personnel discharges a
read as follows: firearm;
"SEC. 36. Status of Members of the Philippine National b) incidents where death, serious physical injury, or any
Police. — The uniformed members of the PNP shall be violation of human rights occurred in the conduct of a police
considered employees of the National Government and shall operation;
draw their salaries therefrom. They shall have the same salary c) incidents where evidence was compromised,
grade level as that of public school teachers: Provided, That tampered with, obliterated, or lost while in the custody of police
PNP members assigned in Metropolitan Manila, chartered cities personnel;
and first class municipalities may be paid financial incentive by d) incidents where a suspect in the custody of the police
the local government unit concerned subject to the availability of was seriously injured; and
funds." e) incidents where the established rules of engagement
have been violated.
Sec37: Early Retirement Program. — Within three (3) years after Finally, the IAS shall provide documents or recommendations as
the effectivity of this Act, any PNP officer or non-commissioned regards to the promotion of the members of the PNP or the
officer may retire and be paid separation benefits corresponding assignment of PNP personnel to any key position.
to a position two (2) ranks higher than his or her present rank
subject to the following conditions: Sec40: Organization. — National, regional, and provincial offices
a) that at the time he or she applies for retirement, he or of the Internal Affairs shall be established. Internal Affairs
she has already rendered at least ten (10) years of continuous Service shall be headed by an Inspector General who shall be
government service; assisted by a Deputy Inspector General. The area offices shall
b) the applicant is not scheduled for separation or be headed by a Director while the provincial offices shall be
retirement from the service due to the attrition system or headed by a Superintendent: Provided, That the head of the
separation for cause; Internal Affairs Service shall be a civilian who shall meet the
c) he or she has no pending administrative or criminal qualification requirements provided herein.
case; and The commission shall establish a rationalized staffing pattern in
d) he or she has at least three (3) more years in the the Reorganization Plan as provided for in Section 13 hereof.
service before reaching the compulsory retirement age and at
least a year before his or her maximum tenure in position. Sec41: Appointments. — The Inspector General shall be
appointed by the President upon the recommendation of the
Sec38: Rationalization of Retirement and Separation Benefits. Director General and duly endorsed by the Commission.
— The Commission shall formulate a rationalized retirement and Appointments of personnel who shall occupy various positions
separation benefits schedule and program within one (1) year shall be made by the Inspector General and shall be based on
from the effectivity of this Act for approval by Congress: an established career pattern and criteria to be promulgated by
Provided, That the approved schedule and program shall have the Commission.
retroactive effect in favor of PNP members and officers retired or
separated from the time specified in the law, unless the Sec42: Entry Qualifications to IAS. — Entry to the Internal
retirement or separation is for cause and the decision denies the Affairs Service shall be voluntary and subject to rigid screening
grant of benefits. where only PNP personnel who have at least five (5) years
experience in law enforcement and who have no derogatory
TITLE V: INTERNAL AFFAIRS SERVICE service records shall be considered for appointment: Provided,
Sec39: Creation, Powers, and Functions. — An Internal Affairs That members of the Bar may enter the service laterally.
Service (IAS) of the PNP is hereby created which shall:
a) pro-actively conduct inspections and audits on PNP Sec43: Initial Appointments to the National, Directorial, and
personnel and units; Provincial Internal Affairs Service Offices. — Initial
b) investigate complaints and gather evidence in support appointments of the heads of the offices in the Internal Affairs
of an open investigation; Service shall be made by the President upon recommendation
c) conduct summary hearings on PNP members facing by the Commission. Thereafter, appointments and promotions to
administrative charges;

The Apocryphal Maggots:


Rainier, Chrisgel, Corina, Geoffry, Grace and Sylvie Blanche
The Flibbertigibbet Worms:
Golda, Gladys and Melyjane
CA VE AT: By simply r ea ding this re vi ew er a t the end o f the sem este r wi ll (hopef ully) guar antee you r pas sing this cou rse. D rink mode rat ely .
Amusin S C A r y
gly
THE C2005 LOCAL GOVERNMENT REVIEWER - 115 -
the Service shall follow the established requirements and authority who fails to act or who acts with abuse of discretion on
procedures. the recommendation of the IAS shall be made liable for gross
neglect of duty. The case of erring disciplinary authority shall be
Sec44: Promotions. — The Commission shall establish the submitted to the Director General for proper disposition.
promotion system within the IAS which shall follow the general
principles of the promotion system in the PNP. Sec50: Appeals. — Decisions rendered by the provincial
inspectors shall be forwarded to the area internal affairs office
Sec45: Prohibitions. — Any personnel who joins the IAS may not for review within ten (10) working days upon the receipt thereof.
thereafter join any other unit of the PNP. Neither shall any Decisions of the area office may be appealed to the national
personnel of the IAS be allowed to sit in a committee office through the Office of Inspector General. Decisions
deliberating on the appointment, promotion, or assignment of rendered by the National IAS shall be appealed to the National
any PNP personnel. Appellate Board or to the court as may be appropriate:
Provided, That the summary dismissal powers of the Director
Sec46: Career Development and Incentives. — (1) Personnel of General and Regional Directors as provided in Section 42 of
the Internal Affairs Service shall in addition to other allowances Republic Act No. 6975 shall remain valid: Provided, further, That
authorized under existing laws be granted occupational the existing jurisdiction over offenses as provided under
specialty pay which shall not exceed fifty percent (50%) of their Republic Act No. 6975 shall not be affected.
basic pay. This pay shall not be considered a forfeiture of other
remuneration and allowances which are allowed under existing Sec51: Complaints Against the IAS. — A complaint against any
laws. personnel or office of IAS shall be brought to the Inspector
(2) IAS members shall also have priorities in the quota General's Office or to the Commission as may be appropriate.
allocation for training and education.
TITLE VI: DISCIPLINARY MECHANISMS
Sec47: Records Management of the IAS. — Local Internal Sec52: Section 41 of Republic Act No. 6975 is hereby amended to
Affairs Offices shall be responsible for the maintenance and read as follows:
update of the records of the members of the PNP within their "SEC. 41 (a). Citizen's Complaints. — Any complaint by a
jurisdiction. natural or juridical person against any member of the PNP shall
When a PNP personnel is reassigned or transferred to another be brought before the following:
location or unit outside the jurisdiction of the current Internal "(1) Chiefs of Police, where the offense is punishable by
Affairs Office, the original records of such personnel shall be withholding of privileges, restriction to specified limits,
transferred over to the Internal Affairs Office that will acquire suspension or forfeiture of salary, or any combination
jurisdiction over the transferred personnel while copies will be thereof, for a period not exceeding fifteen (15) days;
retained by the former Internal Affairs Office. In cases where a "(2) Mayors of cities and municipalities, where the offense is
PNP personnel has been relieved of his/her position and has not punishable by withholding of privileges, restriction to
been given an assignment, the Internal Affairs Office where the specified limits, suspension or forfeiture of salary, or any
person has been assigned last shall continue to have jurisdiction combination thereof, for a period of not less than sixteen
over his or her records until such time that the officer or member (16) days but not exceeding thirty (30) days;
shall have been given a new assignment where the records will "(3) People's Law Enforcement Board, as created under
be forwarded to the Internal Affairs Office acquiring jurisdiction Section 43 hereof, where the offense is punishable by
over the PNP personnel. withholding of privileges, restriction to specified limits,
suspension or forfeiture of salary, or any combination
Sec48: Inclusion of Supervisors and Superiors in IAS thereof, for a period exceeding thirty (30) days; or by
Investigations. — The immediate superior or supervisor of the dismissal.
personnel or units being investigated under the preceding "The Commission shall provide in its implementing rules and
section shall be automatically included in the investigation of the regulations a scale of penalties to be imposed upon any
IA to exclusively determine lapses in administration or member of the PNP under this Section.
supervision. "(b) Internal Discipline. — On dealing with minor offenses
involving internal discipline found to have been committed by
Sec49: Disciplinary Recommendations of the IAS. — (a) Any any regular member of their respective commands, the duly
uniformed PNP personnel found guilty of any of the cases designated supervisors and equivalent officers of the PNP shall,
mentioned in Section 39 of this Act and any immediate superior after due notice and summary hearing, exercise disciplinary
or supervisor found negligent under Section 48 shall be powers as follows:
recommended automatically for dismissal or demotion, as the "(1) Chiefs of police or equivalent supervisors may
case may be. summarily impose the administrative punishment of
(b) Recommendations by the IAS for the imposition of admonition or reprimand; restriction to specified limits;
disciplinary measures against an erring PNP personnel, once withholding of privileges; forfeiture of salary or
final, cannot be revised, set-aside, or unduly delayed by any suspension; or any of the combination of the foregoing:
disciplining authority without just cause. Any disciplining

The Apocryphal Maggots:


Rainier, Chrisgel, Corina, Geoffry, Grace and Sylvie Blanche
The Flibbertigibbet Worms:
Golda, Gladys and Melyjane
CA VE AT: By simply r ea ding this re vi ew er a t the end o f the sem este r wi ll (hopef ully) guar antee you r pas sing this cou rse. D rink mode rat ely .
Amusin S C A r y
gly
THE C2005 LOCAL GOVERNMENT REVIEWER - 116 -
Provided, That, in all cases, the total period shall not "(b) that to the best of his knowledge, no such action or
exceed fifteen (15) days; proceeding is pending in other police administrative
"(2) Provincial directors or equivalent supervisors may disciplinary machinery or authority;
summarily impose administrative punishment of "(c) that if there is any such action or proceeding which is
admonition or reprimand; restrictive custody; withholding either pending or may have been terminated, he must
of privileges; forfeiture of salary or suspension, or any state the status thereof; and
combination of the foregoing: Provided, That, in all "(d) that if he should thereafter learn that a similar action or
cases, the total period shall not exceed thirty (30) days; proceeding has been filed or is pending before any other
"(3) Police regional directors or equivalent supervisors shall police disciplinary authority, he must undertake to report
have the power to impose upon any member the that fact within five (5) days therefrom to the disciplinary
disciplinary punishment of dismissal from the service. He authority where the original complaint or pleading has
may also impose the administrative punishment of been filed."
admonition or reprimand; restrictive custody; withholding
of privileges; suspension or forfeiture of salary; Sec53. Section 42 of Republic Act No. 6975 is hereby amended to
demotion; or any combination of the foregoing: Provided, read as follows:
That, in all cases, the total period shall not exceed sixty "SEC. 42. Summary Dismissal Powers of the National Police
(60) days; Commission, PNP Chief and PNP Regional Directors. — The
"(4) The Chief of the PNP shall have the power to impose the National Police Commission, the chief of the PNP and PNP
disciplinary punishment of dismissal from the service; regional directors, after due notice and summary hearings, may
suspension or forfeiture of salary; or any combination immediately remove or dismiss any respondent PNP member in
thereof for a period not exceeding one hundred eighty any of the following cases:
(180) days: Provided, further, That the chief of the PNP "(a) When the charge is serious and the evidence of guilt
shall have the authority to place police personnel under is strong;
restrictive custody during the pendency of a grave "(b) When the respondent is a recidivist or has been
administrative case filed against him or even after the repeatedly charged and there are reasonable grounds to believe
filing of a criminal complaint, grave in nature, against that he is guilty of the charges; and
such police personnel. "(c) When the respondent is guilty of a serious offense
"(c) Exclusive Jurisdiction. — A complaint or a charge filed involving conduct unbecoming of a police officer.
against a PNP member shall be heard and decided exclusively "Any member or officer of the PNP who shall go on absence
by the disciplining authority who has acquired original without official leave (AWOL) for a continuous period of thirty
jurisdiction over the case and notwithstanding the existence of (30) days or more shall be dismissed immediately from the
concurrent jurisdiction as regards the offense: Provided, That service. His activities and whereabouts during the period shall
offenses which carry higher penalties referred to a disciplining be investigated and if found to have committed a crime, he shall
authority shall be referred to the appropriate authority which has be prosecuted accordingly."
jurisdiction over the offense.
"For purposes of this Act, a 'minor offense' shall refer to any act Sec54: Section 44 of Republic Act No. 6975 is hereby amended to
or omission not involving moral turpitude, but affecting the read as follows:
internal discipline of the PNP, and shall include, but not limited "SEC. 44. Disciplinary Appellate Boards. — The formal
to: administrative disciplinary machinery of the PNP shall be the
"(1) Simple misconduct or negligence; National Appellate Board and the regional appellate boards.
"(2) Insubordination; "The National Appellate Board shall be composed of the four (4)
"(3) Frequent absences and tardiness; regular commissioners and shall be chaired by the executive
"(4) Habitual drunkenness; and officer. The Board shall consider appeals from decisions of the
"(5) Gambling prohibited by law. Chief of the PNP.
"(d) Forum shopping of multiple filing of complaints. — "The National Appellate Board may conduct its hearings or
When an administrative complaint is filed with a police sessions in Metropolitan Manila or any part of the country as it
disciplinary authority, such as the People's Law Enforcement may deem necessary.
Board (PLEB), no other case involving the same cause of action "There shall be at least one (1) regional appellate board per
shall be filed with any other disciplinary authority. administrative region in the country to be composed of a senior
"In order to prevent forum shopping or multiple filing of officer of the regional Commission as Chairman and one (1)
complaints, the complainant or party seeking relief in the representative each from the PNP, and the regional peace and
complaint shall certify under oath in such pleading, or in a sworn order council as members. It shall consider appeals from
certification annexed thereto and simultaneously filed therewith, decisions of the regional directors, other officials, mayors, and
to the truth of the following facts and undertaking: the PLEBs: Provided, That the Commission may create
"(a) that he has not heretofore commenced any other action additional regional appellate boards as the need arises."
or proceeding involving the same issues in other
disciplinary forum; Sec55: Section 47 of Republic Act No. 6975 is hereby amended to
read as follows:

The Apocryphal Maggots:


Rainier, Chrisgel, Corina, Geoffry, Grace and Sylvie Blanche
The Flibbertigibbet Worms:
Golda, Gladys and Melyjane
CA VE AT: By simply r ea ding this re vi ew er a t the end o f the sem este r wi ll (hopef ully) guar antee you r pas sing this cou rse. D rink mode rat ely .
Amusin S C A r y
gly
THE C2005 LOCAL GOVERNMENT REVIEWER - 117 -
"Sec. 47. Preventive Suspension Pending Criminal Case. — from the effectivity of this Act to include but not limited to the
Upon the filing of a complaint or information sufficient in form establishment of equal opportunities for women in the PNP, the
and substance against a member of the PNP for grave felonies prevention of sexual harassment in the workplace, and the
where the penalty imposed by law is six (6) years and one (1) prohibition of discrimination on the basis of gender or sexual
day or more, the court shall immediately suspend the accused orientation.
from office for a period not exceeding ninety (90) days from
arraignment: Provided, however, That if it can be shown by Sec60: Administrative Liability. — Any personnel who shall
evidence that the accused is harassing the complainant and/or violate the established rules and regulations regarding gender
witnesses, the court may order the preventive suspension of the sensitivity and gender equality shall be suspended without pay
accused PNP member even if the charge is punishable by a for not less than thirty (30) days and shall undergo gender
penalty lower than six (6) years and one (1) day: Provided, sensitivity seminar or training: Provided, That any personnel
further, That the preventive suspension shall not be more than who violates the rules more than twice shall be recommended
ninety (90) days except if the delay in the disposition of the case for demotion or dismissal from the PNP.
is due to the fault, negligence or petitions of the respondent:
Provided, finally, That such preventive suspension may be Sec61: Non-prohibition for Promotion. — Nothing in this title
sooner lifted by the court in the exigency of the service upon shall be construed as a restriction on the assignment of
recommendation of the chief, PNP. Such case shall be subject policewomen to other positions in the PNP nor shall any
to continuous trial and shall be terminated within ninety (90) provisions of this title be used for the non-promotion of a PNP
days from arraignment of the accused." female personnel to higher position.

Sec56: Section 49 of Republic Act No. 6975 is hereby amended to TITLE VIII: PARTICIPATION OF LOCAL GOVERNMENT
read as follows: EXECUTIVES IN THE ADMINISTRATION OF THE PNP
"SEC. 49. Legal Assistance. — The Secretary of Justice, the Sec62: The provisions of the second, third, fourth and fifth
chairman of the Commission or the Chief of the PNP may paragraphs of subparagraph (b) (1), Section 51, Chapter III-D of
authorize lawyers of their respective agencies to provide legal Republic Act No. 6975 are hereby amended to read as follows:
assistance to any member of the PNP who is facing before the "The term 'operational supervision and control' shall mean the
prosecutor's office, the court or any competent body, a charge or power to direct, superintend, and oversee the day-to-day
charges arising from any incident which is related to the functions of police investigation of crime, crime prevention
performance of his official duty: Provided, That government activities, and traffic control in accordance with the rules and
lawyers so authorized shall have the power to administer oaths: regulations promulgated by the Commission.
Provided, further, That in such cases, when necessary, as "It shall also include the power to direct the employment and
determined by the Commission, a private counsel may be deployment of units or elements of the PNP, through the station
provided at the expense of the Government. The Secretary of commander, to ensure public safety and effective maintenance
Justice, the Chairman of the Commission and the Chief of the of peace and order within the locality. For this purpose, the
PNP shall jointly promulgate rules and regulations to implement terms 'employment' and 'deployment' shall mean as follows:
the provisions of this Section." "'Employment' refers to the utilization of units or elements of the
PNP for purposes of protection of lives and properties,
TITLE VII: CREATION OF WOMEN'S DESKS IN ALL POLICE enforcement of laws, maintenance of peace and order,
STATIONS AND THE FORMULATION OF A GENDER prevention of crimes, arrest of criminal offenders and bringing
SENSITIVITY PROGRAM the offenders to justice, and ensuring public safety, particularly
Sec57: Creation and Functions. — The PNP shall establish in the suppression of disorders, riots, lawlessness, violence,
women's desks in all police stations throughout the country to rebellious and seditious conspiracy, insurgency, subversion or
administer and attend to cases involving crimes against chastity, other related activities.
sexual harassment, abuses committed against women and "'Deployment' shall mean the orderly and organized physical
children and other similar offenses: Provided, That municipalities movement of elements or units of the PNP within the province,
and cities presently without policewomen will have two (2) years city or municipality for purposes of employment as herein
upon the effectivity of this Act within which to comply with the defined."
requirement of this provision.
Sec63: Section 51 (b) (4) of Republic Act No. 6975 is hereby
Sec58: Prioritization of Women for Recruitment. — Within the amended to read as follows:
next five (5) years, the PNP shall prioritize the recruitment and "(4) Other Powers. In addition to the aforementioned
training of women who shall serve in the women's desk. powers, city and municipal mayors shall have the following
Pursuant to this requirement, the PNP shall reserve ten percent authority over the PNP units in their respective jurisdictions:
(10%) of its annual recruitment, training, and education quota for "(i) Authority to choose the chief of police from a list of five
women (5) eligibles recommended by the provincial police
director, preferably from the same province, city or
Sec59: Gender Sensitivity Program. — The Commission shall municipality: Provided, however, That in no case shall an
formulate a gender sensitivity program within ninety (90) days officer-in-charge be designated for more than thirty (30)

The Apocryphal Maggots:


Rainier, Chrisgel, Corina, Geoffry, Grace and Sylvie Blanche
The Flibbertigibbet Worms:
Golda, Gladys and Melyjane
CA VE AT: By simply r ea ding this re vi ew er a t the end o f the sem este r wi ll (hopef ully) guar antee you r pas sing this cou rse. D rink mode rat ely .
Amusin S C A r y
gly
THE C2005 LOCAL GOVERNMENT REVIEWER - 118 -
days: Provided, further, That the local peace and order "The PLEB shall be the central receiving entity for any citizen's
council may, through the city or municipal mayor, complaint against the officers and members of the PNP. Subject
recommend the recall or reassignment of the chief of to the provisions of Section 41 of Republic Act No. 6975, the
police when, in its perception, the latter has been PLEB shall take cognizance of or refer the complaint to the
ineffective in combating crime or maintaining peace and proper disciplinary or adjudicatory authority within three (3) days
order in the city or municipality: Provided, finally, That upon the filing of the complaint."
such relief shall be based on guidelines established by
the NAPOLCOM; Sec67: Number (3) of Paragraph (b), Section 43 of Republic Act
"(ii) Authority to recommend to the provincial director the No. 6975 is hereby amended to read as follows:
transfer, reassignment or detail of PNP members outside "(3) Three (3) other members who are removable only for
of their respective city or town residences; and cause to be chosen by the local peace and order council from
"(iii) Authority to recommend from a list of eligibles previously among the respected members of the community known for
screened by the peace and order council the their probity and integrity, one (1) of whom must be a woman
appointment of new members of the PNP to be assigned and another a member of the Bar, or, in the absence thereof, a
to their respective cities or municipalities without which college graduate, or the principal of the central elementary
no such appointments shall be attested: Provided, That school in the locality."
whenever practicable and consistent with the
requirements of the service, PNP members shall be Sec68. The last paragraph of Section 43 (b)(3) of Republic Act No.
assigned to the city or municipality of their residence. 6975 shall be amended to read as follows:
"The control and supervision of anti-gambling operations shall "The Chairman of the PLEB shall be elected from among its
be within the jurisdiction of local government executives." members. The term of office of the members of the PLEB shall
be for a period of three (3) years from assumption of office.
Sec64: Automatic Deputation of Local Government Executives Such member shall hold office until his successor shall have
as Commission Representatives. — Governors and mayors, been chosen and qualified."
upon having been elected and living qualified as such, are
automatically deputized as representatives of the National Sec69: Compensation and Benefits. — Paragraph c, Section 43
Police Commission in their respective jurisdiction. As deputized of Republic Act No. 6975 shall be amended to read as follows:
agents of the Commission, local government executives can "(c) Compensation. — Membership in the PLEB is a civic
inspect police forces and units, conduct audit, and exercise duty. However, PLEB members shall be paid per diem and shall
other functions as may be duly authorized by the Commission. be provided with life insurance coverage as may be determined
by the city or municipal council from city or municipal funds. The
Sec65: Section 52 of Republic Act No. 6975 is hereby amended to DILG shall provide for the per diem and insurance coverage of
read as follows: PLEB members in certain low income municipalities."
"SEC. 52. Suspension or Withdrawal of Deputation. —
Unless reversed by the President, the Commission may, after Sec70: Budget Allocation. — The annual budget of the Local
consultation with the provincial governor and congressman Government Units (LGU) shall include an item and the
concerned, suspend or withdraw the deputation of any local corresponding appropriation for the maintenance and operation
executive for any of the following grounds: of their local PLEBs.
"(a) Frequent unauthorized absences; The Secretary shall submit a report to Congress and the
"(b) Abuse of authority; President within fifteen (15) days from the effectivity of this Act
"(c) Providing material support to criminal elements; or on the number of PLEBs already organized as well as the LGUs
"(d) Engaging in acts inimical to national security or which still without PLEBs. Municipalities or cities without a PLEB or
negate the effectiveness of the peace and order campaign. with an insufficient number of organized PLEBs shall have thirty
"Upon good cause shown, the President may, directly or through (30) more days to organize their respective PLEBs. After such
the Commission, motu proprio restore such deputation period, the DILG and the Department of Budget and
withdrawn from any local executive." Management shall withhold the release of the LGU's share in
the national taxes in cities and municipalities still without
TITLE IX: STRENGTHENING THE PEOPLE'S LAW PLEB(s).
ENFORCEMENT BOARD
Sec66: Paragraph (a), Section 43 of Republic Act No. 6975 is Sec71: Request for Preventive Suspension. — The PLEB may
hereby amended to read as follows: ask any authorized superior to impose preventive suspension
"SEC. 43. People's Law Enforcement Board (PLEB). — (a) against a subordinate police officer who is the subject of a
Creation and Functions — The sangguniang panlungsod/bayan complaint lasting up to a period as may be allowed under the
in every city and municipality shall create such number of law. A request for preventive suspension shall not be denied by
People's Law Enforcement Boards (PLEBs) as may be the superior officer in the following cases:
necessary: Provided, That there shall be at least one (1) PLEB a) when the respondent refuses to heed the PLEB's
for every five hundred (500) city or municipal police personnel summons or subpoena;
and for each of the legislative districts in a city.

The Apocryphal Maggots:


Rainier, Chrisgel, Corina, Geoffry, Grace and Sylvie Blanche
The Flibbertigibbet Worms:
Golda, Gladys and Melyjane
CA VE AT: By simply r ea ding this re vi ew er a t the end o f the sem este r wi ll (hopef ully) guar antee you r pas sing this cou rse. D rink mode rat ely .
Amusin S C A r y
gly
THE C2005 LOCAL GOVERNMENT REVIEWER - 119 -
b) when the PNP personnel has been charged with With Art. XVI Sec. 6 Consti.8 in mind, Congress passed RA 6975,
offenses involving bodily harm or grave threats, An Act establishing the PNP under a reorganized DILG.
c) when the respondent is in a position to tamper with Petitioner filed a petition for review seeking the Court’s declaration
the evidence; and of unconstitutionality of RA 6975.
d) when the respondent is in a position to unduly
influence the witnesses. 1. WON RA 6975 emasculated the NPC by limiting its power to
Any superior who fails to act on any request for suspension administrative control over the PNP, with control over the PNP and
without valid grounds shall be held administratively liable for the NPC remaining with the DILG Sec.
serious neglect of duty. Xxx Held: NO. merely an exercise by the President of his control
powers through his the DILG Sec.
Doctrine of Qualified Political Agency – The President has to
Executive Order No.292 (1987) delegate some of his control powers to his cabinet members as he
cannot be expected to exercise such all at the same time and in
Book IV, Chapter 7 person.
Sec38: Definition of Administrative Relationship. — Unless
otherwise expressly stated in the Code or in other laws defining The President’s power of control is directly exercised by him over
the special relationships of particular agencies, administrative the members of the cabinet who, in turn, and by his authority,
relationships shall be categorized and defined as follows: control the bureaus and other offices under their respective
(3) Attachment. — (a) This refers to the lateral relationship jurisdictions in the exec dept.
between the department or its equivalent and the attached
agency or corporation for purposes of policy and program Placing NAPOLCOM and the PNP under the reorganized DILG is
coordination. The coordination may be accomplished by having merely an administrative realignment that would bolster a system
the department represented in the governing board of the of coordination and cooperation among the citizenry, local execs
attached agency or corporation, either as chairman or as a and the integrated law enforcement agencies.
member, with or without voting rights, if this is permitted by the
charter; having the attached corporation or agency comply with Organizational set-up does not violate the mandate of the Consti.
a system of periodic reporting which shall reflect the progress of As Sec 14 of RA 6975 specifically provides for administration and
programs and projects; and having the department or its control at the commission level.
equivalent provide general policies through its representative in
the board, which shall serve as the framework for the internal 2. WON the following are in derogation of the power of control of
policies of the attached corporation or agency; the NAPOLCOM over the PNP:
(b) Matters of day-to-day administration or all those 1. power to choose the PNP Provincial Director and the Chief of
pertaining to internal operations shall be left to the discretion or Police vested in the Governors and Mayors, respectively (Sec
judgment of the executive officer of the agency or corporation. In 51)
the event that the Secretary and the head of the board or the 2. power of operational supervision and control over police units
attached agency or corporation strongly disagree on the vested in city and municipal mayors (Sec 51)
interpretation and application of policies, and the Secretary is 3. disciplinary powers over the PNP members vested in the
unable to resolve the disagreement, he shall bring the matter to People’s Law Enforcement Boards and in the city and
the President for resolution and direction; municipal mayors. (Sec 51)
(c) Government-owned or controlled corporations 4. participation in appointments to positions of Senior
attached to a department shall submit to the Secretary Superintendent to Deputy Dir. Gen. as well as administration
concerned their audited financial statements within sixty (60) of qualifying entrance exams vested in CSC (Sec 31)
days after the close of the fiscal year; and Held: NO DEROGATION
(d) Pending submission of the required financial Nos. 1 and 2 – No usurpation of the power of control because
statements, the corporation shall continue to operate on the under said provisions, the local executives are only acting as
basis of the preceding year's budget until the financial representatives of the NAPOLCOM. As such deputies, they are
statements shall have been submitted. Should any government- answerable to the NAPOLCOM for their actions in the exercise of
owned or controlled corporation incur an operation deficit at the their functions
close of its fiscal year, it shall be subject to administrative No. 3 – Commission exercises appellate jurisdiction, through the
supervision of the department; and the corporation's operating regional appellate boards, over decisions of both the PLEB and the
and capital budget shall be subject to the department's said mayors.
examination, review, modification and approval. It is also the Commission which shall issue the implementing
guidelines and procedures to be adopted by the PLEB for the
conduct of its hearings.
Carpio vs. Executive Secretary (1992)
8
The State shall establish and maintain one police force, which shall be national
Facts: and scope and civilian in character, to be administered and controlled by a
national police commission. xxx

The Apocryphal Maggots:


Rainier, Chrisgel, Corina, Geoffry, Grace and Sylvie Blanche
The Flibbertigibbet Worms:
Golda, Gladys and Melyjane
CA VE AT: By simply r ea ding this re vi ew er a t the end o f the sem este r wi ll (hopef ully) guar antee you r pas sing this cou rse. D rink mode rat ely .
Amusin S C A r y
gly
THE C2005 LOCAL GOVERNMENT REVIEWER - 120 -
As a disciplinary board primarily created to hear and decide
citizen’s complaints against erring officers and members of the Andaya vs. RTC (1999)
PNP, the establishment of the PLEBs would all the more help
professionalize the police force. • The position of City Director, Cebu City Police Command
On No. 4 – It precisely underscores the civilian character of the
(chief of police) became vacant.
national police force
• Andaya, Regional Director of the Regional Police Command
9
3. WON Sec 12 RA 6975 constitutes an encroachment upon, No. 7, submitted to the City Mayor of Cebu a list of 5 eligibles
interference with, and an abdication by the Pres. of executive for the mayor to choose one to be appointed as the chief of
control and commander-in-chief powers. police.
Held: NO • The mayor did not choose anyone from the list because the
Provision precisely gives muscle to and enforces the proposition name of P/Chief Inspector Andres Sarmiento was not
that the national police force does not fall under the Commander- included therein.
in-Chief powers of the President. This is necessarily so since the • Andaya refused to agree to Mayor Garcia’s request to include
police force, not being integrated with the military, is not part of the the name of Major Sarmiento in the list of recommendees.
AFP. His refusal was based on his contention that Sarmiento was
Sec 12 simply provides for the transition period or process during not qualified for the position.
which the national police would gradually assume the civilian
function of safeguarding the internal security of the State. WON the mayor may require the Regional Director to include the
mayor’s protégé in the list of 5 eligibles to be recommended by the
4. WON the creation of the Special Oversight Committee under Regional Dir. from which the mayor shall choose the chief of police
Sec 84 is an unconstitutional encroachment upon the President’s – NO
power of control over all executive depts., bureaus and offices • RA 6975, Sec. 51 gives authority to the Cebu City mayor to
Held: NO choose the chief of police from a list of 5 eligibles to
The SOC is merely a transitory or ad hoc body, established and recommended by the Regional Director, Regional Police
tasked solely with planning and overseeing the immediate transfer, Command No. 7.
merger and absorption into the DILG of the involved agencies. • Under RA 6975, Sec. 51, the mayor of Cebu City shall be
Once its functions are carried out, it would cease to exist. deputized as representative of NAPOLCOM in his territorial
Moreover, no executive dept. bureau or office is placed under its jurisdiction and as such, the mayor shall have authority to
control and authority. choose the chief of police from a list of 5 eligibles
recommended by the Police Regional Director. The City
Police Station of Cebu City is under the direct command and
The commission is not in the same category as the independent control of the PNP Regional Director, Regional Police
Constitutional Commissions created by the Constitution which are Command No. 7, and is equivalent to a provincial office.
independent of the Executive. It was even stressed during the Then, the Regional Director, Regional Police Command No. 7
CONCOM deliberations that this commission would be under the appoints the officer selected by the mayor as the chief of
President’s, and ultimately the DILG Sec.’s control. police.
• It is the prerogative of the Regional Police Director to name
DGNotes: the 5 eligibles from a pool of eligible officers screened by the
Art XVI Sec 6 Consti Senior Officers Promotion and Selection Board,
- 1st sentence qualifies the 2nd. Law governing power of local Headquarters, PNP, Camp Crame, without interference from
chief execs. must be consistent with the 1st. local executives.
• In case of disagreement between the Regional Police Director
Under CAB, both NAPOLCOM and PNP were under the DILG. and the Mayor, the question shall be elevated to the Regional
- GOOD LAW? Director, NAPOLCOM, who shall resolve the issue within 5
NO. Under RA 8551, the DILG and the NAPOLCOM are working days from receipt and whose decision on the choice
of the Chief of Police shall be final and executory.
attached agencies, both under the President. The PNP is
now under the NAPOLCOM. • As deputy of the Commission, the authority of the mayor is
very limited. In reality, he has no power of appointment; he
has only the limited power of selecting one from among the
list of 5 eligibles to be named the chief of police.

9 • Actually, the power to appoint the chief of police is vested in


During a period of 24 mos. From the effectivity of this Act, the AFP shall
the Regional Director, Regional Police Command No. 7.
continue its present role of preserving the internal and external security of the
Much less may the mayor require the Regional Director,
State: Provided, that the said period may be extended by the Pres if he finds it
Regional Police command, to include the name of any officer,
justifiable, for another period not exceeding 24 mos, after which, the Dept. shall
no matter how qualified, in the list of 5 to be submitted to the
automatically take over from the AFP the primary role of preserving external
mayor. The purpose is to enhance police professionalism
security

The Apocryphal Maggots:


Rainier, Chrisgel, Corina, Geoffry, Grace and Sylvie Blanche
The Flibbertigibbet Worms:
Golda, Gladys and Melyjane
CA VE AT: By simply r ea ding this re vi ew er a t the end o f the sem este r wi ll (hopef ully) guar antee you r pas sing this cou rse. D rink mode rat ely .
Amusin S C A r y
gly
THE C2005 LOCAL GOVERNMENT REVIEWER - 121 -
and to isolate the police service from political must be made in good faith, not for political or personal
domination. reasons, or in order to circumvent the constitutional security
of tenure of civil service ees.
Petition granted.
• Mayor v. Macaraig: Abolition of an office is not the same
as the declaration that that office is vacant. While it is a
prerogative of the legislature to abolish certain offices, it
Canonizado vs. Aguirre (2000)
cannot be conceded the power to simply pronounce those
offices vacant and thereby effectively remove the occupants
• The NAPOLCOM was originally created under RA 6975 or holders thereof from the civil service. Such an act would
entitled “An Act Establishing the Philippine National Police constitute, on its face, an infringement of the constitutional
Under A Reorganized Department of the Interior and Local guarantee of security of tenure, and will have to be struck
Government, and for Other Purposes.” down on that account.
• RA 8551 later on took effect. It declared that the terms of the • This is precisely what RA 8551 seeks to do – declare the
current Commissioners were deemed as expired upon its offices of petitioners vacant, by declaring that “the terms
effectivity. of office of the current Commissioners are deemed
• Pursuant to this, Pres. Ramos appointed Cairme and Adiong expired,” thereby removing petitioners from the civil
to the NAPOLCOM. Completing the membership were service. Congress may only be conceded this power if it
Magahum and Factoran, who were appointed by Pres. is done pursuant to a bona fide abolition of the
Estrada. NAPOLCOM.
• According to petitioners, secs. 4 and 8 of RA 8551 are • RA 8551 did not expressly abolish petitioners’ positions. In
unconstitutional: order to determine whether there has been an implied
o Sec. 4 – See Ra 8551 abolition, it becomes necessary to examine the changes
o Sec. 8: Upon the effectivity of this Act, the terms of office introduced by the new law in the nature, composition and
of the current Commissioners are deemed expired which functions of the NAPOLCOM.
shall constitute a bar to their reappointment or an • It is pointed out that under RA 6975, the NAPOLCOM was
extension of their terms in the Commission except for described as a collegial body within the DILG, whereas under
current Commissioners who have served less than 2 RA 8551, it is made an agency attached to the Dept. for policy
years of their terms of office who may be appointed by and program coordination. However, this change does not
the Pres. for a maximum term of 2 years. result in the creation of a new office.
• The organizational structure of NAPOLCOM remains
WON the removal of petitioners from office by virtue of Sec. 8 essentially the same and except for the addition of the PNP
violates their security of tenure – YES Chief as ex-officio member, the composition of NAPOLCOM
• Petitioners are members of the civil service, which embraces is also substantially identical under the 2 laws. Also, under
all branches, subdivisions, instrumentalities and agencies of both laws, the Sec. of the Dept. shall act as the ex-officio
the government. As such, they cannot be removed or Chairman of NAPOLCOM and the Vice-Chairman shall be
suspended from office, except for cause provided by law. The one of the Commissioners designated by the Pres.
phrase “except for cause provided by law” refers to reasons • Also, the powers and duties of the NAPOLCOM remain
which the law and sound public policy recognize as sufficient basically unchanged by the amendments. (see pp. 322-324
warrant for removal, that is, legal cause, and not merely for enumeration of functions in RA 6975 and RA 8551.)
causes which the appointing power in the exercise of Clearly, the NAPOLCOM continues to exercise substantially
discretion may deem sufficient. the same administrative, supervisory, rule-making, advisory
and adjudicatory functions.
Aguirre, et. al.: The express declaration in Sec. 8, RA 8551 that
the terms of petitioners’ offices are deemed expired discloses the Aguirre, et. al.: The fact that the NAPOLCOM is now vested with
legislative intent to impliedly abolish the NAPOLCOM created administrative control and operational supervision over the PNP,
under RA 6975 pursuant to a bona fide reorganization. The whereas under RA 6975, it only exercised administrative control
various changes introduced by RA 8551 in the functions, should be construed as evidence of legislative intent to abolish
composition and character of the NAPOLCOM is proof of such office.
Congress’ intention to abolish the body created under RA 6975 in • Control means the power of an officer to alter or modify or set
order to replace it with a new NAPOLCOM which is more civilian in aside what a subordinate officer had done in the performance
nature. of his duties and to substitute the judgment of the former for
• The creation and abolition of public officers is primarily a that of the latter. On the other hand, to supervise is to
legislative function. Congress may abolish any office it oversee, to have oversight of, to superintend the execution of
creates without impairing the officer’s right to continue in the or the performance of a thing, or the movements or work of a
position held and such power may be exercised for various person, to inspect with authority; it is the power or authority of
reasons, such as the lack of funds or in the interest of the an officer to see that subordinate officers perform their duties.
economy. However, in order for the abolition to be valid, it

The Apocryphal Maggots:


Rainier, Chrisgel, Corina, Geoffry, Grace and Sylvie Blanche
The Flibbertigibbet Worms:
Golda, Gladys and Melyjane
CA VE AT: By simply r ea ding this re vi ew er a t the end o f the sem este r wi ll (hopef ully) guar antee you r pas sing this cou rse. D rink mode rat ely .
Amusin S C A r y
gly
THE C2005 LOCAL GOVERNMENT REVIEWER - 122 -
• Thus, the power of control necessarily encompasses the such body. These questions also go into the very wisdom of
power of supervision and adding the phrase “operational the law.
supervision” under the powers of the NAPOLCOM would not
bring about a substantial change in its functions. Sec. 8 unconstitutional and petitioners are entitled to be
reinstated.
• There was no reorganization resulting in the abolition of
petitioners’ offices. Reorganization takes place when there
is an alteration of the existing structure of government Canonizado vs. Aguirre (2001)
offices or units therein, including the lines of control,
authority and responsibility between them. It involves a
Facts:
reduction of personnel, consolidation of offices, or
• Respondents are seeking a reconsideration of the SC
abolition thereof by reason of economy or redundancy of
functions. It may result in the loss of one’s position decision declaring sec. 8 of RA 8551 to be violative of
through the abolition of an office. However, for a petitioners’ constitutionally mandated right to security of
reorganization to be valid, it must also pass the test of tenure.
good faith. • During the pendency of the case, Pres. Estrada had
appointed Alexis Canonizado to the position of Inspector
• As a general rule, a reorganization is carried out in “good General of the Internal Affairs Service (IAS) of the PNP.
faith” if it is for the purpose of economy or to make • By accepting such position, respondents contend that
bureaucracy more efficient. In that event, no dismissal or Canonizado is deemed to have abandoned his claim for
separation actually occurs because the position itself ceases reinstatement to the NAPOLCOM since the offices of
to exist. Be that as it may, if the “abolition”, which is NAPOLCOM Commissioner and Inspector General of the IAS
nothing more but a separation or removal, is done for are incompatible.
political reasons or purposely to defeat security of
tenure, or otherwise not in good faith, no valid “abolition” WON Canonizado’s appointment to and acceptance of his position
takes place. An example is where there is merely a change of Inspector Gen. should result in an abandonment of his claim for
of nomenclature of positions, or where claims of economy are reinstatement to the NAPOLCOM – NO
belied by the existence of ample funds. • Abandonment of an office is the voluntary relinquishment of
• RA 8551 effected a reorganization of the PNP, not of the an office by the holder, with the intention of terminating his
NAPOLCOM. They are 2 separate and distinct bodies, with possession and control thereof. There are, therefore 2
one having supervision and control over the other. In fact, it essential elements of abandonment:
is the NAPOLCOM that is given the duty of submitting a o An intention to abandon
proposed reorganization plant of the PNP to Congress. o An overt or “external” act by which the intention is
• The basic structure of the NAPOLCOM has been carried into effect.
preserved by the amendatory law. There has been no • Generally, a person holding a public office may abandon such
revision in its lines of control, authority and office by nonuser or acquiescence. Non-user refers to a
responsibility, neither has there been a reduction in its neglect to use a right or privilege or to exercise an office.
membership, nor a consolidation or abolition of the However, nonperformance of the duties of an office does
offices constituting the same. not constitute abandonment where such nonperformance
• No bona fide reorganization of NAPOLCOM having been results from temporary disability or from involuntary
mandated by Congress, RA 8551, insofar as it declares the failure to perform.
terms of office of the incumbent Commissioners as expired • Abandonment may also result from an acquiescence by the
and resulting in their removal from office, removes civil officer in his wrongful removal or discharge. Where, while
service ees from office without legal cause and must therefore desiring and intending to hold the office, and with no willful
be struck down for being constitutionally infirm. desire or intention to abandon it, the public officer vacates it in
deference to the requirements of a statute which is afterwards
Canonizado, et. al.: Sec. 4 unconstitutional insofar as it limits the declared unconstitutional, such a surrender will not be
law enforcement sector to only 1 position on the Commission and deemed an abandonment and the officer may recover the
categorizes the police as being part of the law enforcement sector office.
despite the provision in the Consti. Which provides that the police • By accepting the position of Inspector General during the
force shall be civilian in character.
pendency of the present case, Canonizado cannot be
They also claim that the requirement in Sec. 4 that one of the
deemed to have abandoned his claim for reinstatement to
Commissioners shall be a woman has no rational basis and is
the latter position. Canonizado did not voluntarily leave
therefore discriminatory, amounting to class legislation and an
his post as Commissioner, but was compelled to do so
undue restriction on the appointing power of the Pres.
on the strength of Sec. 8 of RA 8551, which was struck
• There is no longer any need to pass upon these remaining down for being violative of petitioners’ constitutionally
constitutional questions. The legislator has the power to guaranteed right to security of tenure.
provide for the composition of NAPOLCOM since it created

The Apocryphal Maggots:


Rainier, Chrisgel, Corina, Geoffry, Grace and Sylvie Blanche
The Flibbertigibbet Worms:
Golda, Gladys and Melyjane
CA VE AT: By simply r ea ding this re vi ew er a t the end o f the sem este r wi ll (hopef ully) guar antee you r pas sing this cou rse. D rink mode rat ely .
Amusin S C A r y
gly
THE C2005 LOCAL GOVERNMENT REVIEWER - 123 -
• The removal of petitioners from their positions by virtue 1. NAPOLCOM National Appellate Board dismissed petitioner’s
of a constitutionally infirm act necessarily negates a appeal and denied the motion for reconsideration.
finding of voluntary relinquishment. 2. CA denied petitioner’s petition for lack of merit.
• It is well-settled that he who, while occupying one office,
accepts another incompatible with the first, ipso fact vacates Issue: WON the Court of Appeals erred when it dismissed the
the first office and his title is thereby terminated without any petition for certiorari and mandamus filed by petitioner PFC
other act or proceeding. Public policy considerations dictate Rodolfo Rodriguez.
against allowing the same individual to perform inconsistent Held: NO, Court of Appeals committed no reversible error of law in
and incompatible duties. dismissing petitioner’s special civil action for certiorari and
• The incompatibility contemplated is not the mere physical mandamus.
impossibility of one person’s performing the duties of the 2 1. Pursuant to the Civil Service Law which outlines the procedure
offices due to a lack of time or the inability to be in 2 places at for dismissal and Rule XIV of the Omnibus Rules Implementing
the same moment, but that which proceeds from the nature Book V of the Administrative Code of 1987 provides:
and relations of the 2 positions to each other as to give rise to - where a police officer is dismissed by the PNP Director
contrariety and antagonism should one person attempt to General and the dismissal is affirmed by the NAPOLCOM
faithfully and impartially discharge the duties of one toward National Appellate Board, the proper remedy is to appeal the
the incumbent of the other. dismissal with the DILG Secretary
- That the NAPOLCOM Chairman is also the DILG Secretary is
• The positions of NAPOLCOM Commissioner and Inspector
of no moment
Gen. of the IAS are incompatible with each other. RA 8551
o under the aforecited laws and regulations, only the DILG
prohibits any personnel of the IAS from sitting in a committee
Secretary can act on the appeal.
charged with the task of deliberating on the appointment,
- Besides, what is involved here is not the sole act of the
promotion, or assignment any PNP personnel, whereas the
NAPOLCOM Chairman, but the decision of the Commission.
NAPOLCOM has the power of control and supervision over
o Should the DILG Secretary’s decision prove adverse to
the PNP.
However, the rule on incompatibility of duties will not apply to appellant, then he as the aggrieved party may bring an
the CAB because at no point did Canonizado discharge the appeal to the Civil Service Commission.
functions of the 2 offices simultaneously. Canonizado was - In instances where the CSC denies the appeal,
forced out of his first office by the enactment of Sec. 8, RA 8551. o the remedy under R.A. No. 7902 would be to appeal the
Thus, when he was appointed as Inspector Gen., he had ceased to adverse decision to the Court of Appeals.
discharge his official functions as NAPOLCOM Commissioner. As - In the instant case, petitioner had three opportunities to
a matter of fact, on the same date of his appointment as Inspector appeal the decision of the NAPOLCOM.
Gen., the appointments of 4 regular members of the NAPOLCOM o He chose not to avail of them, but instead opted to file
were completed. Thus, the incompatibility of duties rule never had an action for certiorari and mandamus with the appellate
a chance to come into play for petitioner never occupied the 2 court.
positions nor discharged their respective functions
-- Neither certiorari nor mandamus can substitute for appeal where
the latter is the proper remedy. The extraordinary remedies of
Rodriguez vs. CA (2002) certiorari, prohibition, and mandamus will lie only when there is no
appeal or any plain, speedy, and adequate remedy in the ordinary
Facts: course of law
In an entrapment Rodriguez, et. al. were caught asking for money
from an alleged traffic violator. 2. Why appeal to the CSC?
Republic v. Asuncion: “the civilian character of the PNP is
3 cases were filed: unqualified, unconditional, and all embracing.”
1. administrative case for grave misconduct - Members of the PNP are deemed civilian personnel of the
- filed with NAPOLCOM government.
- Police officers and personnel are part of the civil service. This
2. administrative case for summary dismissal is expressly recognized by R.A. No. 6975 when it provided for
- filed with NAPOLCOM the applicability of civil service laws to all its personnel in
Section 91 thereof, which states:
3. A charge for robbery/extortion SEC. 91. Application of Civil Service Laws. –
- filed with Headquarters, PC-INP The Civil Service Law and its implementing rules
and regulations shall apply to all personnel of
February 7, 1991: 3 policemen were summarily dismissed by PNP the Department.
Chief Major General Nazareno Petition denied.
- petitioner appealed to the NAPOLCOM National
Appellate Board

The Apocryphal Maggots:


Rainier, Chrisgel, Corina, Geoffry, Grace and Sylvie Blanche
The Flibbertigibbet Worms:
Golda, Gladys and Melyjane
CA VE AT: By simply r ea ding this re vi ew er a t the end o f the sem este r wi ll (hopef ully) guar antee you r pas sing this cou rse. D rink mode rat ely .
Amusin S C A r y
gly
THE C2005 LOCAL GOVERNMENT REVIEWER - 124 -
People vs. Velarde (2002) During the cross-examination of the investigator (witness for
prosecution), he even went so far as to state that Atty. Domingo
Facts: had not acted as appellant’s lawyer. If this were so, then
• On May 11, 1997, Crispin Velarde was arrested by the appellant had absolutely no counsel when his extra-judicial
confession was taken.
barangay tanods and officials of Tikay, Malolos in connection
It is clear that appellant was not assisted by a competent and
with the rape and murder of an 8-eight year old child named
independent counsel during the custodial investigation and the
Brenda Candelaria.
taking of this extrajudicial confession. Hence, the Court is duty
• He was tagged by some witnesses as the person last seen
bound to disregard the extra-judicial confession.
with the child, whose dead body was later found in a grassy
vacant lot.
• Velarde was initially brought to the Barangay Hall of Barangay
Inter-Local Government Relations
Tikay, and then to the Malolos Police Station, where he was
incarcerated and allegedly mauled.
Sec3: Operative Principles of Decentralization.
• On May 14, 1997, his case was referred by the Malolos police
(e) Provinces with respect to component cities and
to the incumbent mayor of Malolos, Bulacan, Atty. Danilo
municipalities, and cities and municipalities with respect to
Domingo, who asked that Velarde be brought to him. Upon
component barangays, shall ensure that the acts of their
advice of the mayor, Velarde’s written extrajudicial confession
component units are within the scope of their prescribed powers
was taken. During the investigation, Velarde was assisted by
and functions;
the mayor as counsel. Armed police officers were also
present during the investigation.
Sec29: Provincial Relations with Component Cities and
Municipalities. — The province, through the governor, shall
Issue1: WON Atty. Domingo, incumbent mayor of Malolos,
ensure that every component city and municipality within its
could be considered a competent and independent counsel
territorial jurisdiction acts within the scope of its prescribed
qualified to assist Velarde who was then under custodial
powers and functions. Highly urbanized cities and independent
investigation
component cities shall be independent of the province.
Held1: No
• Under the circumstances, Atty. Domingo cannot be Sec30: Review of Executive Orders. —
considered as an independent counsel. He was the mayor of (a) Except as otherwise provided under the Constitution and
Malolos at the time. As such, he exercised “operational special statutes, the governor shall review all executive orders
supervision and control” over the PNP unit in that municipality. promulgated by the component city or municipal mayor within
His powers included the utilization of the elements thereof for his jurisdiction. The city or municipal mayor shall review all
the maintenance of peace and order, the prevention of executive orders promulgated by the punong barangay within
crimes, the arrest of criminal offenders and the bringing of his jurisdiction. Copies of such orders shall be forwarded to the
offenders to justice. governor or the city or municipal mayor, as the case may be,
As mayor of Malolos, his duties were inconsistent with those of within three (3) days from their issuance. In all instances of
his responsibilities to appellant, who was already incarcerated review, the local chief executive concerned shall ensure that
and tagged as the main suspect in the rape-slay case. Serving such executive orders are within the powers granted by law and
as counsel of appellant placed him in direct conflict with his duty in conformity with provincial, city, or municipal ordinances.
of “operational supervision and control “ over the police. (b) If the governor or the city or municipal mayor fails to
• What the constitution requires in Art. III Sec. 12(1) is the act on said executive orders within thirty (30) days after their
present of competent and independent counsel, one who will submission, the same shall be deemed consistent with law and
effectively undertake his client’s defense without any therefore valid.
intervening conflict of interest.
Evidently, Atty. Domingo, being the mayor of the place where the Sec31: Submission of Municipal Questions to the Provincial
investigation was taken, could not act as counsel, independent Legal Officer or Prosecutor. — In the absence of a municipal
or otherwise, of appellant. legal officer, the municipal government may secure the opinion
The desired role of counsel in the process of custodial of the provincial legal officer, and in the absence of the latter,
investigation is rendered meaningless if the lawyer gives that of the provincial prosecutor on any legal question affecting
perfunctory advice as opposed to a meaningful advocacy of the the municipality.
rights of the person undergoing questioning. If the advice given
is so cursory as to be useless, voluntariness is impaired. Sec32: City and Municipal Supervision over Their Respective
• During the investigation, Atty. Domingo failed to act as the Barangays. — The city or municipality, through the city or
independent and competent counsel envisioned by the municipal mayor concerned, shall exercise general supervision
constitution. He failed to give any meaningful advice to over component barangays to ensure that said barangays act
protect the rights of appellant. The former did not even bother within the scope of their prescribed powers and functions.
to inform the latter of the consequences of an extrajudicial
confession.

The Apocryphal Maggots:


Rainier, Chrisgel, Corina, Geoffry, Grace and Sylvie Blanche
The Flibbertigibbet Worms:
Golda, Gladys and Melyjane
CA VE AT: By simply r ea ding this re vi ew er a t the end o f the sem este r wi ll (hopef ully) guar antee you r pas sing this cou rse. D rink mode rat ely .
Amusin S C A r y
gly
THE C2005 LOCAL GOVERNMENT REVIEWER - 125 -
Sec33: Cooperative Undertakings Among Local Government (4) Two (2) representatives of non-governmental
Units. — Local government units may, through appropriate organizations that are represented in the local
ordinances, group themselves, consolidate, or coordinate their development council concerned, to be chosen by the
efforts, services, and resources for purposes commonly organizations themselves; and
beneficial to them. In support of such undertakings, the local (5) Any practicing certified public accountant from the
government units involved may, upon approval by the private sector, to be designated by the local chapter of
sanggunian concerned after a public hearing conducted for the the Philippine Institute of Certified Public Accountants, if
purpose, contribute funds, real estate, equipment, and other any.
kinds of property and appoint or assign personnel under such Representatives of the Commission on Audit shall observe the
terms and conditions as may be agreed upon by the proceedings of such committee and shall certify that the rules
participating local units through Memoranda of Agreement. and procedures for prequalification, bids and awards have been
complied with.
(b) The agenda and other information relevant to the
Relations with Peoples’ and Non-governmental meetings of such committee shall be deliberated upon by the
Organizations committee at least one (1) week before the holding of such
meetings.
Sec34: Role of People's and Non-governmental Organizations. (c) All meetings of the committee shall be held in the
— Local government units shall promote the establishment and provincial capitol or the city or municipal hall. The minutes of
operation of people's and non-governmental organizations to such meetings of the committee and any decision made therein
become active partners in the pursuit of local autonomy. shall be duly recorded, posted at a prominent place in the
provincial capitol or the city or municipal hall, and delivered by
Sec35: Linkages with People's and Non-governmental the most expedient means to elective local officials concerned.
Organizations. — Local government units may enter into joint
ventures and such other cooperative arrangements with Sec38: Local Technical Committee. —
people's and non-governmental organizations to engage in the (a) There is hereby created a local technical committee in every
delivery of certain basic services, capability-building and province, city and municipality to provide technical assistance to
livelihood projects, and to develop local enterprises designed to the local prequalification, bids and awards committees. It shall
improve productivity and income, diversity agriculture, spur rural be composed of the provincial, city or municipal engineer, the
industrialization, promote ecological balance, and enhance the local planning and development coordinator, and such other
economic and social well-being of the people. cdt officials designated by the local prequalification, bids and
awards committee.
Sec36: Assistance to People's and Non-governmental (b) The chairman of the local technical committee shall
Organizations. — A local government unit may, through its local be designated by the local prequalification, bids and awards
chief executive and with the concurrence of the sanggunian committee and shall attend its meeting in order to present the
concerned, provide assistance, financial or otherwise, to such reports and recommendations of the local technical committee.
people's and non-governmental organizations for economic,
socially-oriented, environmental, or cultural projects to be
implemented within its territorial jurisdiction.
Customers who bought A Hand in the Bush: The Fine Art of
Vaginal Fisting also bought the following:
Local Pre-qualification, Bids and Awards Committee • Anal Pleasure & Health: A Guide for Men and Women by Jack
Morin
Sec37: Local Prequalification, Bids and Awards Committee
(Local PBAC). —
• The Ultimate Guide to Cunnilingus: How to Go Down on a
(a) There is hereby created a local prequalification, bids and Woman and Give Her Exquisite Pleasure by Violet Blue
awards committee in every province, city, and municipality, • Trust, the Hand Book: A Guide to the Sensual and Spiritual Art
which shall be primarily responsible for the conduct of of Handballing by Bert Herrman
prequalification of contractors, bidding, evaluation of bids, and
the recommendation of awards concerning local infrastructure
• Jay Wiseman's Erotic Bondage Handbook by Jay Wiseman
projects. The governor or the city or municipal mayor shall act • The New Bottoming Book by Janet W. Hardy, Dossie Easton
as the chairman with the following as members:
(1) The chairman of the appropriations committee of the
sanggunian concerned;
(2) A representative of the minority party in the sanggunian TITLE TWO: ELECTIVE O FF ICIALS
concerned, if any, or if there be none, one (1) chosen by
said sanggunian from among its members; Qualifications and Elections
(3) The local treasurer;

The Apocryphal Maggots:


Rainier, Chrisgel, Corina, Geoffry, Grace and Sylvie Blanche
The Flibbertigibbet Worms:
Golda, Gladys and Melyjane
CA VE AT: By simply r ea ding this re vi ew er a t the end o f the sem este r wi ll (hopef ully) guar antee you r pas sing this cou rse. D rink mode rat ely .
Amusin S C A r y
gly
THE C2005 LOCAL GOVERNMENT REVIEWER - 126 -
Sec39: Qualifications. — cities and municipalities shall serve as ex officio members of the
(a) An elective local official must be a citizen of the Philippines; a sangguniang panlalawigan concerned. The presidents of the
registered voter in the barangay, municipality, city, or province "liga ng mga barangay and the pederasyon ng mga
or, in the case of a member of the sangguniang panlalawigan, sangguniang kabataan" elected by their respective chapters, as
sangguniang panlungsod, or sangguniang bayan, the district provided in this Code, shall serve as ex officio members of the
where he intends to be elected; a resident therein for at least sangguniang panlalawigan, sangguniang panlungsod, and
one (1) year immediately preceding the day of the election; and sangguniang bayan. (Has been amended by RA8553)
able to read and write Filipino or any other local language or (c) In addition thereto, there shall be one (1) sectoral
dialect. representative from the women, one (1) from the workers, and
(b) Candidates for the position of governor, vice-governor, one (1) from any of the following sectors: the urban poor,
or member of the sangguniang panlalawigan, or mayor, vice- indigenous cultural communities, disabled persons, or any other
mayor or member of the sangguniang panlungsod of highly sector as may be determined by the sanggunian concerned
urbanized cities must be at least twenty-one (21) years of age within ninety (90) days prior to the holding of the next local
on election day. elections as may be provided for by law. The COMELEC shall
(c) Candidates for the position of mayor or vice-mayor of promulgate the rules and regulations to effectively provide for
independent component cities, component cities, or the election of such sectoral representatives.
municipalities must be at least twenty-one (21) years of age on
election day. Sec42: Date of Election. — Unless otherwise provided by law, the
(d) Candidates for the position of member of the elections for local officials shall be held every three (3) years on
sangguniang panlungsod or sangguniang bayan must be at the second Monday of May.
least eighteen (18) years of age on election day.
(e) Candidates for the position of punong barangay or Sec43: Term of Office. —
member of the sangguniang barangay must be at least eighteen (a) The term of office of all local elective officials elected after
(18) years of age on election day. the effectivity of this Code shall be three (3) years, starting from
(f) Candidates for the sangguniang kabataan must be at noon of June 30, 1992 or such date as may be provided for by
least fifteen (15) years of age but not more than twenty-one (21) law, except that of elective barangay officials: Provided, That all
years of age on election day. local officials first elected during the local elections immediately
following the ratification of the 1987 Constitution shall serve until
Sec40: Disqualifications. — The following persons are noon of June 30, 1992.
disqualified from running for any elective local position: (b) No local elective official shall serve for more than
(a) Those sentenced by final judgment for an offense three (3) consecutive terms in the same position. Voluntary
involving moral turpitude or for an offense punishable by one (1) renunciation of the office for any length of time shall not be
year or more of imprisonment, within two (2) years after serving considered as an interruption in the continuity of service for the
sentence; full term for which the elective official concerned was elected.
(b) Those removed from office as a result of an (c) The term of office of barangay officials and members of
administrative case; the sangguniang kabataan shall be for three (3) years,
(c) Those convicted by final judgment for violating the which shall begin after the regular election of barangay
oath of allegiance to the Republic; officials on the second Monday of May 1994.
(d) Those with dual citizenship; Me mbe rs of the Sa ng gu ni an
(e) Fugitives from justice in criminal or non-political cases
here or abroad; RA7160 – Local Government Code
(f) Permanent residents in a foreign country or those who
have acquired the right to reside abroad and continue to avail of Sec41: Manner of Election. —
the same right after the effectivity of this Code; and (a) The governor, vice-governor, city mayor, city vice-mayor,
(g) The insane or feeble-minded. municipal mayor, municipal vice-mayor, and punong barangay
shall be elected at large in their respective units by the qualified
Sec41: Manner of Election. — voters therein. However, the sangguniang kabataan chairman
(a) The governor, vice-governor, city mayor, city vice-mayor, for each barangay shall be elected by the registered voters of
municipal mayor, municipal vice-mayor, and punong barangay the katipunan ng kabataan, as provided in this Code.
shall be elected at large in their respective units by the qualified (b) The regular members of the sangguniang
voters therein. However, the sangguniang kabataan chairman panlalawigan, sangguniang panlungsod, and sangguniang
for each barangay shall be elected by the registered voters of bayan shall be elected by district, as may be provided for by law.
the katipunan ng kabataan, as provided in this Code. Sangguniang barangay members shall be elected at large. The
(b) The regular members of the sangguniang presidents of the leagues of sanggunian members of component
panlalawigan, sangguniang panlungsod, and sangguniang cities and municipalities shall serve as ex officio members of the
bayan shall be elected by district, as may be provided for by law. sangguniang panlalawigan concerned. The presidents of the
Sangguniang barangay members shall be elected at large. The "liga ng mga barangay and the pederasyon ng mga
presidents of the leagues of sanggunian members of component sangguniang kabataan" elected by their respective chapters, as

The Apocryphal Maggots:


Rainier, Chrisgel, Corina, Geoffry, Grace and Sylvie Blanche
The Flibbertigibbet Worms:
Golda, Gladys and Melyjane
CA VE AT: By simply r ea ding this re vi ew er a t the end o f the sem este r wi ll (hopef ully) guar antee you r pas sing this cou rse. D rink mode rat ely .
Amusin S C A r y
gly
THE C2005 LOCAL GOVERNMENT REVIEWER - 127 -
provided in this Code, shall serve as ex officio members of the shall have twelve (12) councilors each to be elected at large by
sangguniang panlalawigan, sangguniang panlungsod, and the qualified voters of the said city or municipality. All the other
sangguniang bayan. (Has been amended by RA8553) municipalities within the Metropolitan Manila area shall have ten
(c) In addition thereto, there shall be one (1) sectoral (10) councilors each, with the exception of the Municipality of
representative from the women, one (1) from the workers, and Pateros which shall have eight (8) councilors, to be elected at
one (1) from any of the following sectors: the urban poor, large by their respective qualified voters.
indigenous cultural communities, disabled persons, or any other
sector as may be determined by the sanggunian concerned Sec3: Other Cities. — The provision of any law to the contrary
within ninety (90) days prior to the holding of the next local notwithstanding the City of Cebu, City of Davao, and any other
elections as may be provided for by law. The COMELEC shall city with more than one representative district shall have eight
promulgate the rules and regulations to effectively provide for (8) councilors for each district who shall be residents thereof to
the election of such sectoral representatives. be elected by the qualified voters therein, provided that the cities
of Cagayan de Oro, Zamboanga, Bacolod, Iloilo and other cities
comprising a representative district shall have twelve (12)
RA6636 (1987) - An Act Resetting The Local Elections From councilors each and all other cities shall have ten (10)
November 9, 1987 To January 18, 1988, Amending For This councilors each to be elected at large by the qualified voters of
Purpose Executive Order Numbered Two Hundred And the said cities: Provided, That in no case shall the present
Seventy (EO270) number of councilors according to their charters be reduced.

Sec1: Section 1 of Executive Order No. 270 is hereby amended to Sec4: Provinces and Municipalities. — First and second class
read as follows: provinces shall each have ten (10) elective members; third and
"Sec. 1. Election of local official. — There shall be elections fourth class provinces, eight; and fifth and sixth class provinces,
for provincial governors, provincial vice-governor, city and six to be elected at large by the qualified voters therein.
municipal mayors, city and municipal vice-mayors, and All other municipalities shall have the same number of elective
members of each Sangguniang Panlalawigan, Sangguniang members as provided in existing laws.
Panlungsod, and Sangguniang Bayan, including all members of November 6, 1987.
the city or municipal boards or councils in the Metropolitan
Manila area to be held on Monday, January 18, 1988.
Certificates of candidacy for the aforesaid local elective RA7166 – An Act Providing For Synchronized National And
positions shall be filed not later than sixty (60) days prior to the Local Elections And For Electoral Reforms, Authorizing
said elections. Appropriations Therefor, And For Other Purposes
All local officials, whether elected, acting or officers-in-charge,
shall be deemed automatically resigned from their positions Sec3: Election of Members of the Sangguniang Panlalawigan,
effective upon the filing of their certificates of candidacy for any Sanggunian Panlungsod and Sangguniang Bayan. — The
local position which shall not be later than forty-five (45) days elective members of the Sangguniang Panlalawigan,
prior to the said elections. If the governor or the city or municipal Sangguniang Panlungsod and Sangguniang Bayan shall be
mayor or the officer-in-charge of that office is a candidate, and elected as follows:
unless the Secretary of Local Government designates another (a) For provinces with two (2) or more legislative districts,
person, the following local officials shall act as officer-in-charge the elective members of the Sangguniang Panlalawigan shall be
of the position vacated in a concurrent capacity in the order elected by legislative districts. For this purpose, the number of
hereinbelow provided: seats shall be apportioned equitably: Provided, That, if equal
"a) Chief, Senior and Local Government Officers for division is not possible, the remaining member or members shall
provinces, cities and municipalities, respectively; be elected in the district or districts with the greater number of
"b) Provincial/City/Municipal Administrator; population or, if they be the same, with the greater number of
"c) Provincial/City/Municipal Health Officer. voters: Provided further, That, if a legislative district includes a
"In case of vacancies in the Sangguniang Panlalawigan, city that does not vote in the election of provincial officials, the
Sangguniang Panlungsod, or Sangguniang Bayan on account of Commission on Elections, hereinafter referred to as the
the candidacies of their members, the Secretary of Local Commission, shall allocate the number of seats among the
Government shall designate acting members from qualified districts in proportion to the population of the constituencies
voters in the province, city or municipality to fill such vacancies." voting for the Sangguniang Panlalawigan;
(b) For provinces with only one (1) legislative district, the
Sec2: Metro Manila Area. — For purposes of the Local Elections Commission shall divide them into two (2) districts for purposes
on January 18, 1988, the City of Manila, Quezon City and the of electing the members of the Sangguniang Panlalawigan, as
City of Caloocan shall have six (6) councilors for each of their nearly as practicable according to the number of inhabitants,
representative districts who shall be residents thereof to be each district comprising a compact, contiguous and adjacent
elected by the qualified voters therein. The City of Pasay and territory, and the number of seats of elective members of their
the Municipalities of Makati, Parañaque, Pasig, Marikina, and respective sanggunian shall be equitably apportioned between
Valenzuela, each of which comprises a representative district,

The Apocryphal Maggots:


Rainier, Chrisgel, Corina, Geoffry, Grace and Sylvie Blanche
The Flibbertigibbet Worms:
Golda, Gladys and Melyjane
CA VE AT: By simply r ea ding this re vi ew er a t the end o f the sem este r wi ll (hopef ully) guar antee you r pas sing this cou rse. D rink mode rat ely .
Amusin S C A r y
gly
THE C2005 LOCAL GOVERNMENT REVIEWER - 128 -
the districts in accordance with the immediately preceding
paragraph; RA8553 – An Act Amending Section 41(B) Of RA7160,
(c) The number and election of elective members of the Otherwise Known As The Local Government Code Of 1991
Sangguniang Panlungsod and Sangguniang Bayan in the Metro
Manila Area, City of Cebu, City of Davao and any other city with Sec1: Section 41(b) of Republic Act No. 7160, otherwise known as
two (2) or more legislative districts shall continue to be governed the Local Government Code of 1991, is hereby amended to
by the provisions of Sections 2 and 3 of Republic Act No. 6636: read as follows:
Provided, That, the Municipalities of Malabon, Navotas, San "(b) The regular members of the sangguniang
Juan, Mandaluyong, Muntinlupa, Las Piñas and Taguig shall panlalawigan, sangguniang panlungsod, and sangguniang
have twelve (12) councilors, and Pateros, ten (10): Provided, bayan shall be elected by district as follows:
further, That, the Commission shall divide each of the "First and second-class provinces shall have ten (10) regular
municipalities in Metro Manila Area into two (2) districts by members; third and fourth-class provinces, eight (8); and fifth
barangay for purposes of representation in the Sangguniang and sixth-class provinces, six (6): Provided, That in provinces
Bayan as nearly as practicable according to the number of having more than five (5) legislative districts, each district shall
inhabitants, each comprising a compact, contiguous and have two (2) sangguniang panlalawigan members, without
adjacent territory; and prejudice to the provisions of Section 2 of Republic Act No.
(d) For purposes of the regular elections on May 11, 6637. Sangguniang barangay members shall be elected at
1992, elective members of the Sangguniang Panlungsod and large. The presidents of the leagues of sanggunian members of
Sangguniang Bayan shall be elected a large in accordance with component cities and municipalities shall serve as ex officio
existing laws. However, beginning with the regular elections in members of the sangguniang panlalawigan concerned. The
1995, they shall be elected by district. presidents of the liga ng mga barangay and the pederasyon ng
The Commission shall promulgate rules and regulations to mga sangguniang kabataan elected by their respective
effective provide for the election of sectoral representatives in chapters, as provided in this Code, shall serve as ex officio
the implementation of the Local Government Code. members of the sangguniang panlalawigan, sangguniang
Approved: November 26, 1991 panlungsod, and sangguniang bayan."

Sec2: Upon the petition of the provincial board, the election for any
RA7887 – An Act Instituting Electoral Reforms For The additional regular member to the sangguniang panlalawigan as
Purpose Of Amending Section 3, Paragraphs (C) And (D) Of provided for under this Act, shall be held not earlier than six (6)
RA7166 months after the May 11, 1998 national and local elections.
Approved: February 25, 1998
Sec1: Section 3, paragraphs (c) and (d) of Republic Act No. 7166,
is hereby amended to read as follows:
"(c) The number and election of elective members of the Herrera vs. COMELEC (1999)
sangguniang panlungsod and sangguniang bayan in the Metro
Manila area, City of Cebu, City of Davao and any other city with FACTS:
two (2) or more legislative districts shall be elected by districts • Sangguniang Panlalawigan of Guimaras passed Resolution
and in accordance with the provisions of Sections 2 and 3 of
No. 68 requesting COMELEC to bring about the division of
Republic Act No. 6636: Provided, That, all cities with one (1)
the Province into two provincial districts, in view of the
legislative district and all municipalities in the Metro Manila area
addition of 2 new municipalities, San Lorenzo & Sibunag.
shall have twelve (12) councilors each: Provided, further, That,
• Disagreement ensued on how the districts therein should be
the Commission shall divide all cities with one legislative district
and each of the municipalities in Metro Manila area into two (2) divided for purpose of electing members of the Sangguniang
districts by barangay for purposes of representation in the Panlalawigan: according to number of inhabitants, or
sangguniang bayan as nearly as practicable according to the according to number of registered voters
number of inhabitants, each district comprising a compact,
contiguous and adjacent territory; and APPLICABLE LAWS:
"(d) For purposes of the regular elections on May 11, 1992
and all general elections thereafter, the regular elective §4 RA6636 : allotment of elective members to provinces &
members of the sangguniang panlungsod and sangguniang municipalities must be made on the basis of its classification as a
bayan, shall be elected at large in accordance with existing province or municipality. Thus, a 4th class province like Guimaras,
laws. shall have 8 elective members.
"The Commission shall promulgate rules and regulations to
effectively implement the provisions of law which may hereafter §3(b) RA 7166: For provinces with only 1 legislative district, the
be enacted providing for the election of sectoral COMELEC shall divide them into 2 districts for purposes of
representatives." electing the members of the Sangguniang Panglalawigan.
Approve: February 20, 1995 Apportionment shall be done in the ff manner:
1) as nearly as practicable

The Apocryphal Maggots:


Rainier, Chrisgel, Corina, Geoffry, Grace and Sylvie Blanche
The Flibbertigibbet Worms:
Golda, Gladys and Melyjane
CA VE AT: By simply r ea ding this re vi ew er a t the end o f the sem este r wi ll (hopef ully) guar antee you r pas sing this cou rse. D rink mode rat ely .
Amusin S C A r y
gly
THE C2005 LOCAL GOVERNMENT REVIEWER - 129 -
2) accdng to the number of inhabitants  Ground: Ermelita made a false representation in her
3) each district comprising a compact, contiguous & certificate of candidacy, stating that her surname was
adjacent territory Salcedo. She had no right to do so as she was not
4) number of seats of elective members equitably legally married to Neptali.
apportioned between the districts
Issue: WON Ermelita’s use of such surname constitutes a material
Issue: On what basis shall apportionment of districts shall be misrepresentation under the Omnibus Election Code so as to
made? justify cancellation of her certificate of candidacy.
Held: NO, it does NOT constitute material misrepresentation
Held: Under RA 7166 & COMELEC Res. No. 2313 Rule: the
basis for division into districts is the number of inhabitants of the 1. Test of Materiality: Jurisprudence provides that the material
province misrepresentation contemplated by section 78 of the
( not the number of listed or registered voters as theorized by Omnibus Election Code refer to the qualifications of
the petitioners) elective officials. It could not have been the intention of the
Petition dismissed. law to deprive a person of a such a basic and substantive
political right to be voted for a public office upon just any
innocuous mistake.
Qua lifica tion s • In CAB, aside from the allegation of wrong surname,
petitioner does not claim that respondent lacks the
RA9164 (2002) – An Act Providing For Synchronized requisite residency, age, citizenship, or any other legal
Barangay And Sangguniang Kabataan Elections, Amending qualification necessary to run for a local elective office
RA7160, As Amended, Otherwise Known As The "Local as provided for in the LGC.
Government Code Of 1991", And For Other Purposes
2. Test of Deliberate Attempt: Aside from the requirement of
Sec6: Section 424 of Republic Act No. 7160, otherwise known as materiality, a false representation under Section 78 must
the Local Government Code of 1991, is hereby amended to consist of a DELIBIRATE ATTEMPT to mislead, misinform, or
read as follows: hide a fact which would otherwise render a candidate
"Sec424. Katipunan ng Kabataan. — The katipunan ng ineligible.
kabataan shall be composed of Filipino citizens actually residing • IN CAB No showing that the inhabitants of Sara were
in the barangay for at least six (6) months, who are fifteen (15) deceived by the use of such surname by private
but less than eighteen (18) years of age on the day of the respondent.
election, and who are duly registered in the list of the
sangguniang kabataan or in the official barangay list in the !!! Dan Gat's notes: Both tests should be used. For example, a
custody of the barangay secretary." aCHcIE name may pass the first test (materiality), it not being a
material fact referring to qualification. However, it may be
Sec7: Section 428 of Republic Act No. 7160, otherwise known as patently deceiving that it doesn't pass the second test. Hence,
the Local Government Code of 1991, is hereby amended to the candidate must still be disqualified.
read as follows:
"Sec428. Qualifications. — An elective official of the
sangguniang kabataan must be a Filipino citizen, a qualified
voter of the katipunan ng kabataan, a resident of the barangay RA9225 (2003) – Citizenship Retention and RE-acquisition
for at least one (1) year immediately prior to election, at least Act of 2003
fifteen (15) years but less than eighteen (18) years of age on the
day of the election, able to read and write Filipino, English, or Sec2. Declaration of Policy - It is hereby declared the policy of
the local dialect, and must not have been convicted of any crime the State that all Philippine citizens of another country shall be
involving moral turpitude." deemed not to have lost their Philippine citizenship under the
conditions of this Act.

Salcedo II vs. COMELEC (1999) Sec3. Retention of Philippine Citizenship - Any provision of law
to the contrary notwithstanding, natural-born citizenship by
Facts: reason of their naturalization as citizens of a foreign country are
hereby deemed to have re-acquired Philippine citizenship upon
 Neptali Salcedo married Agnes Celiz
taking the following oath of allegiance to the Republic:
 without first marriage dissolved, Neptali married private "I _____________________, solemny swear (or affrim)
respondent Ermelita Cacao that I will support and defend the Constitution of the
 Petitioner Victorino Salcedo and respondent Ermelita both ran Republic of the Philippines and obey the laws and legal
for the position of Sara, Iloilo orders promulgated by the duly constituted authorities of
 Victorino filed with the Comelec petition seeking for the the Philippines; and I hereby declare that I recognize and
cancellation of Ermelita's certificate of candidacy.

The Apocryphal Maggots:


Rainier, Chrisgel, Corina, Geoffry, Grace and Sylvie Blanche
The Flibbertigibbet Worms:
Golda, Gladys and Melyjane
CA VE AT: By simply r ea ding this re vi ew er a t the end o f the sem este r wi ll (hopef ully) guar antee you r pas sing this cou rse. D rink mode rat ely .
Amusin S C A r y
gly
THE C2005 LOCAL GOVERNMENT REVIEWER - 130 -
accept the supreme authority of the Philippines and will • Ortega, also running for the same position, filed a
maintain true faith and allegiance thereto; and that I disqualification proceeding against Labo before the
imposed this obligation upon myself voluntarily without Comelec. Ground: false representation that Labo was a
mental reservation or purpose of evasion." "natural-born" citizen.
Natural born citizens of the Philippines who, after the effectivity
of this Act, become citizens of a foreign country shall retain their Issue: WON Labo is eligible to run
Philippine citizenship upon taking the aforesaid oath. Held: No.
1. Labo: he has reacquired his Filipino citizenship by citing his
Sec4. Derivative Citizenship - The unmarried child, whether application for reacquisition of Philippine citizenship filed
legitimate, illegitimate or adopted, below eighteen (18) years of before the Office of the Solicitor General pursuant to PD 725
age, of those who re-acquire Philippine citizenship upon and Letter of Instruction No. 270
effectivity of this Act shall be deemed citizenship of the SC: No
Philippines. Rule: In the absence of any official action or approval by the
proper authorities a mere application for repatriation does not, and
Sec5. Civil and Political Rights and Liabilities - Those who cannot, amount to an automatic reacquisition of the applicant's
retain or re-acquire Philippine citizenship under this Act shall Philippine citizenship.
enjoy full civil and political rights and be subject to all attendant In CAB: To date, however, and despite favorable recommendation
liabilities and responsibilities under existing laws of the by the Solicitor General, the Special Committee on Naturalization
Philippines and the following conditions: had not yet acted upon said application for repatriation.
(1) Those intending to exercise their right of surffrage must Meet
the requirements under Section 1, Article V of the Constitution,
Republic Act No. 9189, otherwise known as "The Overseas
2. Ortega avers that the candidate receiving the next highest
Absentee Voting Act of 2003" and other existing laws; number of votes should be declared Mayor of Baguio City.
(2) Those seeking elective public in the Philippines shall meet SC: No.
the qualification for holding such public office as required by the General Rule: The ineligibility of a candidate receiving majority
Constitution and existing laws and, at the time of the filing of the votes does not entitle the eligible candidate receiving the next
certificate of candidacy, make a personal and sworn highest number of votes to be declared elected. A minority or
renunciation of any and all foreign citizenship before any public defeated candidate cannot be deemed elected to the office.
officer authorized to administer an oath; Exception: if the electorate, fully aware in fact and in law of a
(3) Those appointed to any public office shall subscribe and candidate's disqualification so as to bring such awareness within
swear to an oath of allegiance to the Republic of the Philippines the realm of notoriety, would nonetheless cast their votes in favor
and its duly constituted authorities prior to their assumption of of the ineligible candidate.
office: Provided, That they renounce their oath of allegiance to Ratio: In such case, the electorate may be said to have waived the
the country where they took that oath; validity and efficacy of their votes by notoriously misapplying their
(4) Those intending to practice their profession in the Philippines franchise or throwing away their votes, in which case, the eligible
shall apply with the proper authority for a license or permit to candidate obtaining the next higher number of votes may be
engage in such practice; and deemed elected.
(5) That right to vote or be elected or appointed to any public
office in the Philippines cannot be exercised by, or extended to, In the case at bar: It has not been shown, and none was alleged,
those who: that petitioner Labo was notoriously known as an ineligible
(a) are candidates for or are occupying any public office candidate, much less the electorate as having known of such fact.
in the country of which they are naturalized citizens;
and/or Hence, as a consequence of petitioners' ineligibility, a
(b) are in active service as commissioned or non- permanent vacancy in the contested office has occurred. This
commissioned officers in the armed forces of the country should now be filled by the vice-mayor, in accordance with Sec. 44
which they are naturalized citizens. of the Local Government Code

Concurring and Dissenting Opinion of Justice Gutierrez:


In deciding cases involving citizenship, I believe that the
Labo vs. COMELEC and Ortega (1992)
presumptions should be in favor of its retention and against its
loss.
Facts
• Labo was married to an Australian wife. Based on this
marriage, Labo took an oath as citizen of Australia. Later on Frivaldo vs. COMELEC (2000)
however, this marriage was nullified, as the wife still had a
subsisting marriage when she married Labo.
Facts:
• Labo filed his certificate of candidacy for Mayor of Baguio for
• On March 20, 1995, Frivaldo filed his Certificate of Candidacy
the May 11, 1992 elections.
for Governor

The Apocryphal Maggots:


Rainier, Chrisgel, Corina, Geoffry, Grace and Sylvie Blanche
The Flibbertigibbet Worms:
Golda, Gladys and Melyjane
CA VE AT: By simply r ea ding this re vi ew er a t the end o f the sem este r wi ll (hopef ully) guar antee you r pas sing this cou rse. D rink mode rat ely .
Amusin S C A r y
gly
THE C2005 LOCAL GOVERNMENT REVIEWER - 131 -
• Lee, another candidate, filed a petition w/ the COMELEC  If the law intended the citizenship qualification to be possessed
praying that Frivaldo be disqualified prior to election consistent with the requirement of being a
 Reason: of not yet being a citizen of the Philippines registered voter, then it would not have made citizenship a
• Frivaldo garnered the highest number of votes in the SEPARATE qualification. The law abhors a redundancy. It
elections. therefore stands to reason that the law intended CITIZENSHIP to
• Lee filed a petition praying for his proclamation as the elected be a qualification distinct from being a VOTER, even if being a
governor. At 8:30 pm of June 30, 1995, Lee was proclaimed voter presumes being a citizen first.
governor.
DISSENT: DAVIDE
• Frivaldo filed a new petition w/ COMELEC seeking the
annulment of the June 30 proclamation of Lee and his own
proclamation. !!! Dan Gat's notes: Frivaldo would have benefited from RA
9225. Hence, we will never again have a Frivaldo problem.
 He alleged that on June 30, 1995, at 2:00pm, he took
his oath of allegiance as a citizen of the Phil after his
petition for repatriation under P.D. 725 w/c he filed with
the Special Committee on Naturalization in September Torayno vs. COMELEC (2000)
1994 had been granted.
Facts:
Issue1: FRIVALDO’S REPATRIATION
1st Question: Was the repatriation of Frivaldo valid and legal?
• On June 14, 1997, on his last term, the three-time governor of
Held: YES Misamis Oriental, Vicente Emano, executed a Voter
Under Phil law, citizenship may be reacquired by direct act of Registration Record in Cagayan de Oro City (CDO) which is
Congress, by naturalization or by repatriation. located in the same province. He claimed to have resided
in CDO for 20 yrs.
2nd Question: Did Frivaldo’s repatriation seasonably cure his lack • Petitioners filed a petition for disqualification at the
of citizenship as to qualify him to be proclaimed and hold office as COMELEC on the ground that he had failed to meet the 1 yr
governor? residency requirement. However, prior to the resolution of
Held: YES, he possessed the citizenship reqt on the day the law petition, Emano was proclaimed by COMELEC as the duly
mandates his term of office to begin. elected mayor.
LAW:
LGC, Sec. 39: An elective local official must be: Issue: WON Emano has duly established his residence in CDO at
a. citizen of the Philippines; least 1 yr prior to May 11 elections to qualify him to run for the
b. a registered voter in the barangay, municipality, city, or mayorship
province . . . where he intends to be elected; Held: Yes
c. a resident therein for at least one 1 yr immediately preceding 1. The 1 yr residency reqt is aimed at preventing the possibility
the day of the election; of a stranger or newcomer unacquainted with the conditions
d. able to read and write Filipino or any other local language or and needs of a community from and not identified with the
dialect. latter from seeking an elective office to serve that community
and at excluding outsiders from taking advantage of favorable
1. circumstances existing in that community for electoral gain.
A. LIBERAL INTERPRETATION : IN CAB: The facts show that Emano was a resident of CDO for a
The law does not specify any particular date or time when the period of time sufficient to qualify him to run for public office
candidate must possess citizenship, unlike that for residence and therein.
age. 2. CDO, after all, is an integral part of the province and is in
Now, an official begins to govern or to discharge his functions only fact located at its heart. It is also the seat of prov’l gov’t. Thus,
upon his proclamation and on the day the law mandates his term the provincial officials who carry out their functions cannot
of office to begin. avoid living in CDO.
B. LITERAL CONSTRUCTION Also, the city’s residents, politics, commerce and businesses are
Sec 39 speaks of "Qualifications" of "ELECTIVE OFFICIALS", not not isolated from that of the province's, since the city is located at
of candidates. Literally, such qualifications should thus be the very heart of the province itself.
possessed when the elective official begins to govern, i.e., at the
time he is proclaimed
!!! Dan Gat's notes: only the RESIDENCE requirement is not a
continuing requirement. ALL OTHER requirements are
2.
continuing requirements.
RE Lee’s argument that the citizenship qualification should be
possessed at the time the candidate registered as a voter as
under the law a "voter" must be a citizen of the Philippines.
SC: NO. Papandayan vs. COMELEC (2002)

The Apocryphal Maggots:


Rainier, Chrisgel, Corina, Geoffry, Grace and Sylvie Blanche
The Flibbertigibbet Worms:
Golda, Gladys and Melyjane
CA VE AT: By simply r ea ding this re vi ew er a t the end o f the sem este r wi ll (hopef ully) guar antee you r pas sing this cou rse. D rink mode rat ely .
Amusin S C A r y
gly
THE C2005 LOCAL GOVERNMENT REVIEWER - 132 -
Facts:
• In the May, 2001 elections, 3 candidates ran for mayor of A) either separately (when one acquires status of a resident
Tubaran, Lanao del Sur – petitioner Papandayan, Jr., alien before aquiring Philippine citizenship – through an
respondent Balt (incumbent mayor seeking reelection), and immigrant visa and an immigrant certificate of residence
Hassan Bantuas. to waive status as non-resident), OR
• Balt sought the disqualification of Papandayan alleging that
he was not a resident of Brgy. Tangcal in Tubaran, but a B) at the same time when one acquires Philippine citizenship –
permanent resident of Bayang, Lanao del Sur. through naturalization under CA473 or by an act of
• Comelec agreed with Balt Congress.
In CAB,
Issue: W/N COMELEC erred in declaring Papandayan as a. his repeated entrance in the country since 1998 was
disqualified on ground that he is not a resident of Tubaran. only on the basis of a visa-free balikbayan visitor
Held: YES. Evidence is insufficient to sustain COMELEC whose stay as such was valid for one year only.
resolution.
1) Papandayan has duly proven that, although he was formerly
b. Coquilla appeared to have waived his status as an alien
a resident of Bayang, he later transferred residence to and as a non-resident only on Nov10, 2000 upon taking
Tangcal in Tubaran as shown by his actual and physical his oath
presence for 10years prior to May2001 elections.
2. Frivaldo vs. COMELEC case not applicable where residency
2) Principle of animus revertendi has been used to determine was not an issue in the case but compliance with the
whether a candidate has an ”intention to return” to the place citizenship requirement.
where he seeks to be elected. Corollary to this is a
determination whether there has been an “abandonment” of 3. Coquilla: His registration as voter of Butnga, Oras in January
his former residence which signifies an intention to depart 2001 is conclusive of his residency as a candidate because
therefrom. Sec17 of Omnibus Election Code requires that a voter must
3) Romualdez vs. RTC, Tacloban – domicile and residence as have resided in the Philippines for at least 1yr
synonymous. SC: NO.
4) In order to acquire a new domicile by choice, there must Nuval vs. Guray has held that registration as a voter does not bar
concur (1) residence or bodily presence in the new locality, (2) the filing of a subsequent case questioning candidate’s lack of
intention to remain there, and (3) intention to abandon old residency.
domicile

Coquilla vs. COMELEC (2002) MyPleasure Guide: Sex Toy Basics (Part 2)

Facts; There's no right answer to any of these questions -- whatever


• Coquilla (Filipino) joined the US Navy. He was subsequently floats your boat is fine with us! However, you'll have an easier
naturalized as a US citizen. time selecting your pocket pal if you give it some thought before
• 1998, he came to Phils and took out a residence certificate, you try to buy.
although he continued making several trips to US (last took Whichever toy you decide to try, let your desires be your guide.
place on July6, 2000 upto Aug5, 2000). Don't let other people's inhibitions bring you down. There is
• He then applied for repatriation under RA8171, was approved absolutely nothing wrong with buying, owning and using sex
on Nov7, 2000. toys! Whether you use your toys alone or with a partner, you
• On Nov10, he took his oath as a Phil. Citizen are enhancing your life and pumping up your sex drive.
• Nov21, he applied for registration as voter of Butnga, Oras Furthermore, you're keeping an open mind and trying
• Feb27, 2001, he filed his certificate of candidacy stating that something new, which is the key to life-long learning. So relax
he had been a resident of Oras for “2years”. and enjoy yourself. Who knows? Perhaps some day you'll have
• Alvarez sought cancellation of certificate – Ground: Coquilla a whole treasure trove of titillating toys to tickle your fancy!
made a material misrepresentation in his certificate of (end)
candidacy by stating that he had been a resident of Oras for
2years, when in truth, his residency was for only about Dis qu alifica tion
6months since Nov10, 2000 when he took his oath.
RA8295 (1997) – An Act Providing For The Proclamation Of
Issue: W/N Coquilla had been a resident of Oras at least 1year
A Lone Candidate For Any Elective Office In A Special
before the elections, as he represented in his cert. of cand.
Election, And For Other Purposes
Held: NO.
1. Status of being an alien and an non-resident can be waived:

The Apocryphal Maggots:


Rainier, Chrisgel, Corina, Geoffry, Grace and Sylvie Blanche
The Flibbertigibbet Worms:
Golda, Gladys and Melyjane
CA VE AT: By simply r ea ding this re vi ew er a t the end o f the sem este r wi ll (hopef ully) guar antee you r pas sing this cou rse. D rink mode rat ely .
Amusin S C A r y
gly
THE C2005 LOCAL GOVERNMENT REVIEWER - 133 -
Sec4: Disqualification. — In addition to the disqualifications (g) All candidates for public office whether appointed or
mentioned in Sections 12 and 68 of the Omnibus Election Code elected both in the national or local government shall undergo a
and Section 40 of Republic Act No. 7160, otherwise known as mandatory drug test.
the Local Government Code, whenever the evidence of guilt is In addition to the above stated penalties in this Section, those
strong, the following persons are disqualified to run in a special found to be positive for dangerous drugs use shall be subject to
election called to fill the vacancy in an elective office, to wit: the provisions of Section 15 of this Act.
a) Any elective official who has resigned from his office
by accepting an appointive office or for whatever reason which !!!Dan Gat's notes on RA 9165:
he previously occupied but has caused to become vacant due to • If candidate does not undergo drug test, it would be a
his resignation; and ground for disqualification, since drug test is mandatory
b) Any person who, directly or indirectly, coerces, bribes, • BUT if candidate found positive of drugs, this is NOT a
threatens, harasses, intimidates or actually causes, inflicts or ground for disqualification. The candidates will just be
produces any violence, injury, punishment, torture, damage, loss subject to the penalties in RA 9165
or disadvantage to any person or persons aspiring to become a
candidate or that of the immediate member of his family, his
honor or property that is meant to eliminate all other potential
candidate. De La Torre vs. COMELEC (1996)

Sec5: Prohibited Acts, Election Offenses and Penalties. — Any Issue: WON Fencing involves moral turpitude
act of coercion, bribery, threat, harassment, intimidation, Held: YES
terrorism, or actually causing, inflicting or producing violence, • Court consistently adopts Black's Law Dictionary definition of
injury, punishment, torture, damage, loss or disadvantage to "moral turpitude"
discourage any other person or persons from filing a certificate ". . . an act of baseness, vileness, or depravity in the private
of candidacy in order to eliminate all other potential candidate duties which a man owes his fellow men, or to society in
from running in a special election shall constitute as an election general, contrary to the accepted and customary rule of right
offense. Violations of this provision shall be prosecuted and and duty between man and woman or conduct contrary to
penalized in accordance with the provision of Section 264 of the justice, honesty, modesty, or good morals."
Omnibus Election Code. • A crime involves moral turpitude is ultimately a question of
fact and frequently depends on all the circumstances
!!! Dan Gat's notes: why will there be a special election called to surrounding the violation of the statute.
fill the vacancy when there is such a thing as principle of • Moral turpitude is deducible from the third element of the
succession? crime. Actual knowledge [ or merely "should have known" the
origin of the property received] by the "fence" of the fact that
property received is stolen displays the same degree of
malicious deprivation of one's rightful property as that which
RA9165 (2002) – An Act Instituting The Comprehensive animated the robbery or theft which, by their very nature, are
Dangerous Drugs Act Of 2002, Repealing RA6425, crimes of moral turpitude.
Otherwise Known As The Dangerous Drugs Act Of 1972, As
Amended, Providing Funds Therefor, And For Other Issue2: WON PROBATION AFFECTS SEC 40(a)
Purposes H2: NO
• The legal effect of probation is only to suspend the execution
Sec36: Authorized Drug Testing. — Authorized drug testing shall of the sentence.
be done by any government forensic laboratories or by any of
the drug testing laboratories accredited and monitored by the !!! Dan Gat's Notes:
DOH to safeguard the quality of test results. The DOH shall take Q: Is moral turpitude also qualified by within 2 years after
steps in setting the price of the drug test with DOH accredited service of offense in Section 40 (a) of LGC?
drug testing centers to further reduce the cost of such drug test.
The drug testing shall employ, among others, two (2) testing A:Yes it is. So moral turpitude is not a perpetual disqualification.
methods, the screening test which will determine the positive
result as well as the type of the drug used and the confirmatory
test which will confirm a positive screening test. Drug test
Magno vs. COMELEC and Ortega (2002)
certificates issued by accredited drug testing centers shall be
valid for a one-year period from the date of issue which may be
used for other purposes. The following shall be subjected to Issue: WON Direct Bribery involves moral turpitude
undergo drug testing: Held: YES
xxx • Moral turpitude can be inferred from the third element of
direct bribery. The fact that the offender agrees to accept a
promise or gift and deliberately commits an unjust act or

The Apocryphal Maggots:


Rainier, Chrisgel, Corina, Geoffry, Grace and Sylvie Blanche
The Flibbertigibbet Worms:
Golda, Gladys and Melyjane
CA VE AT: By simply r ea ding this re vi ew er a t the end o f the sem este r wi ll (hopef ully) guar antee you r pas sing this cou rse. D rink mode rat ely .
Amusin S C A r y
gly
THE C2005 LOCAL GOVERNMENT REVIEWER - 134 -
refrains from performing an official duty in exchange for some Issue2: WON Garcia should be proclaimed mayor.
favors, denotes a malicious intent on the part of the offender Held: NO
to renege on the duties which he owes his fellowmen and To simplistically assume that the 2nd placer would have received
society in general. the other voted would be to substitute the court’s judgment for the
Malinao vs. Reyes (1996) mind of the voter.
Votes cast for Reyes are presumed to have been cast in the belief
Facts: that he was qualified and for that reason cannot be treated as
• Malinao filed an administrative case against Red for abuse of stray, void or meaningless.
authority and denial of due process.
• In an Aug 12 session, Members of the Sanggunian, by a vote Lingating vs. COMELEC (2002)
of 5 to 3, found Red guilty, vote signed only by presiding
chairman Facts:
• September 5: Sanggunian, voting 7 to 2, acquitted Red of the • Sulong ran for the 2001 election mayoralty race
charges against him. This vote was embodied in a Decision • Petitioner Lingating filed a petition for the disqualification of
which was signed by all the members. Cesar B. Sulong, pursuant to Sec. 40 (b) of the LGC
• Malinao: First Sanggu Decision had already become final and • Ground: Sulong was administratively charged during his first
executory for failure of Red to appeal. term as mayor in 1991 with various offenses, and that in
1992, the Sangguniang Panlalawigan of Zamboanga del Sur
Issue: WON the second Decision IS VALID. found him guilty of the charges and ordered his removal from
Held: office.
Yes, second decision of ACQUITAL is valid. • Sulong: this decision has not yet become final and executory:
In any case, this issue is already Moot and Academic o he filed a motion for reconsideration
CASE IS NOW MOOT AND ACADEMIC as a result of the • Lingating (complainant) had not yet complied therewith and
expiration of Red’s term during which the act complained of was his MR had consequently remained pending.
allegedly committed, and further proceedings are barred by his
reelection. [Sec 66(b) LGC] Issue: WON Sulong should be disqualified from becoming mayor
of Lapuyan
Held: NO
Reyes vs. COMELEC (1996)
• the Reyes v COMELEC decision cannot be applied to this
Facts: case because it appears that the 1992 decision of the SP has
not until now become final.(as correctly averred by Sulong)
• An admin case was filed against Mayor Reyes
• While RA 7160 on disciplinary actions is silent on the filing of
• Sangguniang Panlalawigan found Reyes guilty and ordered a MR, the same cannot be interpreted as a prohibition against
his removal from office. the filing of a MR.
• For the next elections, Reyes filed a certificate of candidacy
!!!Dan Gat's notes:
with the COMELEC but de Castro sought his disqualification Section 40 (b) LGC is a PERPETUAL disqualification.
based on Sec 40 (b) LGC10 BUT, the cause of removal may be petty-- nagnakaw ng ball
• Due to the absence of any contrary order from COMELEC pen, repeated absences, etc.
Reyes was voted into office in the subsequent election. On the other hand, Section 40 (a) on moral turpitude
• COMELEC then issued a resolution holding that Reyes is is NOT a perpetual disqualification!
disqualified from running for office and his COC is thereby Apparent ratio:
cancelled.
• Municipal Board of Canvassers, unaware of the COMELEC
• Sec 40 (b)  goes into the competency of carrying out
Resolution, proclaimed Reyes as the duly-elected mayor. functions.
(Garcia is second-placer) • Sec 40 (a)  you can be a competent, albeit immoral,
leader 
Issue1: WON the reelection of Reyes rendered the admin charges
against him moot and academic.
Held: NO
Section 40 (b) of the LGC operates. Grego vs. COMELEC (1997)

Facts:
10 • Basco was removed from his position as Deputy Sheriff by
Sec 40. Disqualification- The ff. persons are disqualified from running from any
the SC upon a finding of serious misconduct in an
elective local position: xxx
(b) Those removed from office as a result of an administrative case.

The Apocryphal Maggots:


Rainier, Chrisgel, Corina, Geoffry, Grace and Sylvie Blanche
The Flibbertigibbet Worms:
Golda, Gladys and Melyjane
CA VE AT: By simply r ea ding this re vi ew er a t the end o f the sem este r wi ll (hopef ully) guar antee you r pas sing this cou rse. D rink mode rat ely .
Amusin S C A r y
gly
THE C2005 LOCAL GOVERNMENT REVIEWER - 135 -
administrative complaint lodged by a certain Nena Held: Admittedly, by operation of law, Manzano possessed dual
Tordesillas.(New LGC NOT yet in effect then) citizenship since he was:
• Subsequently, Basco ran as a candidate for councilor and - born in the US which follows the doctrine of jus sanguinis but
won (New LGC already in effect) of Filipino parents (jus soli under Philippine law)
• After his term, Basco sought reelection twice and won on both However, Court held that Manzano had validly elected Philippine
reelections citizenship and effectively repudiated his American citizenship:
• Petition for disqualification was filed against Basco, using - by filing a certificate of candidacy , he elected Philippine
section 40 (b) of the LGC… but dismissed. citizenship and in effect renounced his American citizenship

Issue: WON the petition for disqualification was correctly


o for every certificate of candidacy contains an oath of
allegiance to the Philippine government
dismissed
- mere filing is enough considering that their condition is the
Held: YES
unavoidable consequence of conflicting laws of different
1. WON Sec. 40 (b) of the LGC applies retroactively to those
states
removed from office before it took effect on Jan. 1, 1992 as
contended by Grego– NO
!!! Dan Gat's Notes:
2. WON Basco’s election to office as city councilor in the 1988, We should be very unhappy with this Manzano case.
1992 and 1995 elections wipe away and condone the First, is Section 40 (d) vague? Very clear naman…
administrative penalty against him, thus restoring his eligibility DUAL CITIZENSHIP
for public office – YES Second, assuming the law is vague, is the
• There was nothing to condone in the first place. Basco interpretation of the SC still correct? NO:
was NOT subject to any disqualification at all under Sec. 40  SC cited ConCoM proceedings, not the deliberations fot
(b) of the LGC, which applies only to those removed from the LGC
office on or after Jan. 1, 1992. In view of the irrelevance of  Section 40 ( C) is the one referring to dual allegiance (and f
the issue posed by petitioner, there is no more reason to is for green card holders.)
dwell on the matter.
This SC decision made Section 40 (d) inexistent -- this
disqualification will never exist because once a candidate files
for candidacy he is deemed to have renounced his foreign
Mercado vs. Manzano (1999)
citizenship.
Facts:
• Mamaril filed a petition for disqualification against Edu
!!! Dan Gat's Notes:
Manzano on the ground that the latter was not a citizen of the
Common MISTAKE of students: citing Frivaldo for dual
Philippines but of the United States. This is based on Section
citizenship problems. NO! Use Manzano case. Manzano is a
40(d) of the LGC which disqualifies persons with dual
Filipino. Frivaldo and Labo were stateless.
citizenship from running for a local position.

Issue: How do we interpret "Dual Citizenship: as a ground for


Disqualification in Section 40 (d)? Valles vs. COMELEC (2000)
Held: term in the law must be understood as referring to DUAL
ALLEGIANCE. Facts:
DUAL CITIZENSHIP: arises when, as a result of the concurrent • Rosalind Lopez was born in 1934 in Western Australia. Her
application of the different laws of 2 or more states, a person is father was a Filipino citizen, while her mother was an
simultaneously considered a national by the said states Australian. In 1949, at the age of 15, she left Australia and
DUAL ALLEGIANCE: a person simultaneously owes, by some came to settle in the Philippines where she married a Filipino
positive act, loyalty to 2 or more states citizen.
- result of the individual’s volition
- considered as inimical to the national interest and shall be • Since then, she has continuously participated in the electoral
dealt with by law (Consti) process not only as a voter but as a candidate. She ran for
- implies double allegiance under a double sovereignty which and was elected Governor of Davao Oriental in the 1992,
would be repugnant to the sovereignty which pervades the 1995 and 1998 elections.
Consti and to citizenship itself
• In all three elections, her respective opponents filed petitions
Issue2: WON Manzano possesses dual citizenship and is
for her disqualification, contesting her Filipino citizenship.
disqualified from being a candidate for vice mayor of Makati City

The Apocryphal Maggots:


Rainier, Chrisgel, Corina, Geoffry, Grace and Sylvie Blanche
The Flibbertigibbet Worms:
Golda, Gladys and Melyjane
CA VE AT: By simply r ea ding this re vi ew er a t the end o f the sem este r wi ll (hopef ully) guar antee you r pas sing this cou rse. D rink mode rat ely .
Amusin S C A r y
gly
THE C2005 LOCAL GOVERNMENT REVIEWER - 136 -
Issue: Did Lopez renounce her Filipino citizenship in 1988 the US authorities after he learned of the charges against him
when she applied for an Alien Certificate of Registration and does not make him a “fugitive from justice”. Going back to the
Immigrant Certificate of Resident, and was issued an US in the middle of his term would only violate the very
Australian passport? functions of his office and jeopardize public interest.
Held: No
• SC reiterated the principle that a renunciation to effectively VITUG DISSENTS:
result in the loss of citizenship, it must be express.

Issue: Assuming that Lopez had dual citizenship, was she MyPleasure Guide: Erotic Massage (Part 2):
disqualified to run for governor?
Held: No Massage Oils
• SC reiterated Mercado v. Manzano ruling : it was ruled that for Massage oils and lotions do a variety of useful things: they add
candidates with dual citizenship, it is enough that they elect zest to a dull night, lend a sensual dimension to a boring old
Philippine citizenship upon the filing of their certificate of back rub, and reduce friction, keeping your body silky, slippery
candidacy, to terminate their status as persons with dual and slick.
citizenship. The filing of the certificate of candidacy sufficed to
To use, pour a small amount in the palm of your hand and rub
renounce foreign citizenship, effectively removing any
your hands together before gently applying to your lover's
disqualification as a dual citizen.
body. Never pour the oil directly on your lover's body -- it will
• Recognizing situations in which a Filipino citizen may, without
feel cold and unpleasant. Rub the lotion or oil into your lover's
performing any act, and as an involuntary consequence of
conflicting laws of different countries, be also a citizen of body, reapplying as necessary. Feel free to rub oil into your
another state, the SC held that dual citizenship as a lover's back, arms, legs, buttocks, chest and neck, but keep it
disqualification must refer to citizens with dual allegiance. away from genitals unless the bottle specifically says the lotion
“Dual citizenship” which is a ground for disqualification in is safe for internal use. Even if it is safe, test a small amount on
LGCode must be understood as referring to citizens with the skin first, ensuring you and your lover won't have an
“dual allegiance”. Consequently, persons with mere dual allergic or "burning" reaction to it. Massage oils and lotions are
citizenship do not fall under the disqualification in the LGC. not safe to use with latex unless the bottle specifically says
• It is significant to note that Lopez executed in 1992 a otherwise.
Declaration of Renunciation of Australian citizenship, as a (continued…)
result of which her Australian passport was cancelled.

Rodriguez vs. COMELEC (1996)

Issue1: WON intent to evade is a necessary element of the


definition of a “fugitive from justice” – Yes
A “fugitive from justice” includes not only those who, after Oth er Gr ou nd s f or Disq ua lifica tion
conviction, flee to avoid punishment but likewise those who, after
being charged, flee to avoid prosecution.” BP881 – Omnibus Election Code
- The definition indicates that the intent to evade is the
compelling factor that animates one’s flight from a particular Sec68: Disqualifications. — Any candidate who, in an action or
jurisdiction. protest in which he is a party is declared by final decision of a
- Obviously there can only be intent to evade when there is competent court guilty of, or found by the Commission of having
knowledge by the fleeing subject of an already instituted (a) given money or other material consideration to influence,
indictment or of a promulgated judgment of conviction. induce or corrupt the voters or public officials performing
electoral functions;
IN CAB  Rodriguez’ arrival in the Philippines (July 95) preceded (b) committed acts of terrorism to enhance his candidacy;
the filing of the felony complaint in Los Angeles (Nov 95). When he (c) spent in his election campaign an amount in excess of that
left the US, there was as yet no complaint & arrest warrant – much allowed by this Code;
less conviction – that he could run away from. (d) solicited, received or made any contribution prohibited under
1) The circumstantial fact that the charges against Rodriguez Sections 89, 95, 96, 97 and 104; or
were filed 17days after his departure can’t overturn the (e) violated any of Sections 80, 83, 85, 86 and 261, paragraphs
presumption of good faith in his favor d, e, k, v, and cc, subparagraph 6, shall be disqualified from
continuing as a candidate, or if he has been elected, from
2) It is immaterial to determine the exact time when Rodriguez
holding the office. Any person who is a permanent resident of or
was made aware of the charges against him, having an immigrant to a foreign country shall not be qualified to run for
established that he was not aware of such charges when he any elective office under this Code, unless said person has
left the US  His failure to submit himself to the jurisdiction of waived his status as permanent resident or immigrant of a

The Apocryphal Maggots:


Rainier, Chrisgel, Corina, Geoffry, Grace and Sylvie Blanche
The Flibbertigibbet Worms:
Golda, Gladys and Melyjane
CA VE AT: By simply r ea ding this re vi ew er a t the end o f the sem este r wi ll (hopef ully) guar antee you r pas sing this cou rse. D rink mode rat ely .
Amusin S C A r y
gly
THE C2005 LOCAL GOVERNMENT REVIEWER - 137 -
foreign country in accordance with the residence requirement to delegate its authority to its Law Department as partial solution to
provided for in the election laws. (Sec. 25, 1971 EC) the problem.
However: The May 8, 1995 elections, did not result in a surfeit of
Sec74: Contents of certificate of candidacy. disqualification cases which the COMELEC cannot handle.
XXX
3. Blanco: Comelec erred in using Summary Proceedings to
Sec78: Petition to deny due course to or cancel a certificate of resolve the disqualification case
candidacy. — A verified petition seeking to deny due course or SC: Ok lang!
to cancel a certificate of candidacy may be filed by the person • The COMELEC action is safely anchored on section 4 of its
exclusively on the ground that any material representation Rules of Procedure which expressly provides that petitions for
contained therein as required under Section 74 hereof is false. disqualification "shall be heard summarily after due notice."
The petition may be filed at any time not later than twenty-five
days from the time of the filing of the certificate of candidacy and • Vote-buying has its criminal and electoral aspects. Its criminal
shall be decided, after due notice and hearing, not later than aspect to determine the guilt or innocence of the accused
fifteen days before the election. cannot be the subject of summary hearing.
• However, its electoral aspect to ascertain whether the
Election Offenses
offender should be disqualified from office can be determined
Sec261: Prohibited Acts. — The following shall be guilty of an
in an administrative proceeding that is summary in character.
election offense: XXX
(Please see law for a VERY long list of election offenses)
Concurring and Dissenting Opinion: Bellosillo

Nolasco vs. COMELEC (1997)


MyPleasure Guide: Erotic Massage (Part 3):
Facts:
• May 8, 1995:Election of mayor of Meycauayan, Bulacan was Warming Oils
held. Blanco won over Alarilla Warming oils work about the same way massage oils work:
• May 9, 1995: Alarilla filed with the Comelec a petition to pour a small puddle in one hand, rub both hands together to
disqualify Blanco, alleging that a search has been conducted distribute, then apply in a thin layer to the desired area. Gently
on Blanco's house which yielded unlicensed firearms and rub into the skin, applying more as necessary. As you continue
evidence of systematized massive vote-buying. to massage, the warming oil will begin to heat up, causing your
• After submission of position papers, Comelec resolved to partner to feel a pleasantly warm sensation. Try blowing on the
disqualify Blanco oil-doused skin, too, for that fabulous icy and hot feel!
While not the best option for full-body massages, warming oils
Issues posed by Blanco: are great for genital massage, or for massaging small,
1. Blanco: was denied due process when Comelec suspended concentrated areas, such as breasts, buttocks, thighs and arms.
his proclamation pending determination of his disqualification Some warming oils can't be used with latex products -- make
SC: No sure to read the label before using. Warming oils aren't
1. Section 6 of R.A. No. 6646 and sections 4 and 5 of the Rule appropriate for internal use -- don't use them for vaginal or anal
25 of the Comelec Rules of Procedure merely require that penetration. However, you can apply them to the surface of
evidence of guilt should be strong to justify the COMELEC your genitals. Be warned: some of the warmth-inducing
in suspending a winning candidate's proclamation. It ought to elements may irritate skin; do a skin patch test before using
be emphasized that the suspension order is provisional in during sex or your genitals may end up hotter than you
nature and can be lifted when the evidence so warrants. intended.
2. Blanco was given all opportunity to prove that the evidence
(continued…)
on his disqualification was not strong : he was given chance
to file Motion to Lift Order, Answer, and position paper. Commo n P robl ems in Lo cal Gov ern men t Ele ctio ns

2. Blanco: Comelec departed from procedure laid down by Ramas vs. COMELEC (1998)
Comelec Resolution 2050 wherein a complaint should be
referred for preliminary investigation to the Law Department Facts:
SC: Untenable. • Petitioners were proclaimed as the duly elected municipal
COMELEC cannot always be straitjacketed by this procedural rule. officials
Comelec's Reason for this Resolution: • Respondents, the losing candidates filed election protests
Resolution 2050 was passed to take care of the with the RTC of Pagadian.
proliferation of disqualification cases at that time. It deemed it wise • RTC rendered decision declaring all respondents as winners

The Apocryphal Maggots:


Rainier, Chrisgel, Corina, Geoffry, Grace and Sylvie Blanche
The Flibbertigibbet Worms:
Golda, Gladys and Melyjane
CA VE AT: By simply r ea ding this re vi ew er a t the end o f the sem este r wi ll (hopef ully) guar antee you r pas sing this cou rse. D rink mode rat ely .
Amusin S C A r y
gly
THE C2005 LOCAL GOVERNMENT REVIEWER - 138 -
• Respondents filed a Motion for Immediate Execution of COMELEC held correctly that: Shortness of term alone and by
Decision Pending Appeal. itself cannot justify premature execution. It must be manifest in the
• RTC granted. Grounds for granting: decision sought to be executed that the defeat of the protestee and
a) public interest the victory of the protestant has been established.
b) near expiration of term of office In addition, RA 8524, which took effect in 1998, has extended the
c) pendency of the protest for one year term of office of the barangay officials to 5 yrs (expire in 2002).
This negates or removes the factual basis for the finding of the
Issue: WON RTC and the COMELEC acted with grave abuse of MTC that the term of the contested office had past almost midway
discretion in granting execution of RTC decision pending appeal. of the whole term.
Held: No grave abuse of discretion
I.
1. Is execution of judgment pending appeal still provided for by Miranda vs. Abaya (1999)
law? Yes.
a) for elective municipal and barangay officials, application of Facts:
Section 2 of Rule 39 of the RoC are permissible pursuant • Jose “Pempe” (the pimp, hehe) Miranda, incumbent mayor,
to Rule 143 of the RoC, which is now Section 4, Rule filed his certificate of candidacy (COC) for Mayor of Santiago
1 of the 1997 Rules of Civil Procedure. City for 1998 elections.
b) for regional, provincial and city officials, the COMELEC  BUT: Pempe was DISQUALIFIED as he can not run for
Rules of Procedure provide for the suppletory application the 4th time.
for the Rules of Court in the absence of any applicable • Joel Miranda filed COC for the mayoralty post supposedly as
provision a substitute for his father Pempe
• Joel won over Abaya.
2. Now, what are the recognized reasons for execution of Abaya thus filed petition to declare null and void the
judgement pending appeal? substitution
The following constitute good reasons, and a • Comelec ANNULLED the substitution, election and
combination of two or more will suffice to grant execution proclamation of Joel
pending appeal:
a. public interest involved or the will of the electorate Issue1: WON Pempe aside from being disqualified, his COC had
b. shortness of the remaining portion of the term of the been denied due course and cancelled
contested office Held: Yes, it was cancelled and denied due course.
c. the length of time that the election contest has been
pending Issue2: WON there was valid substitution
Held: No. There was no valid substitution
<*filing of a bond is not a good reason, but it may be required A disqualified candidate may only be substituted if he
by the court to answer for payment of damages which the had a valid COC in the first place because if he did
aggrieved party may suffer by reason of the execution pending not have a valid COC, he is and was not a candidate
appeal.> at all.
In the case at bar, the reasons cited by the RTC are good
reasons for the issuance of the execution of judgement pending If a person was not a candidate, he cannot be
appeal. substituted under Sec. 77.
1. Under Sec. 77 of Omnibus Election Code 11, substitution is only
!!! Dan Gat's Notes: He has a problem with the "good reasons" allowed in death, disqualification or withdrawal. This does not
for granting Motion for Execution Pending Appeal (MEPA). include those cases where the COC of the person to be substituted
Indeed, there is a need for concurrence of two reasons, but the had been denied due course.
reasons are easy!
 "Public Interest": already existent in any case! 11
Sec. 77. Candidates in case of death, disqualification or withdrawal. – If after
the last day for the filing of certificates of candidacy, an official candidate of a
So, does this mean one who files a MEPA will always win? registered or accredited political party dies, withdraws or is disqualified for any
No. Look at Fermo case. BUT Fermo lost because he was cause, only a person belonging to, and certified by the same political party may
stupid: only cited one ground and didn't know what the term is. file a certificate of candidacy to replace the candidate who died, withdrew or was
disqualified. The substitute candidate nominated by the political party concerned
Fermo vs. COMELEC (2000) may file his certificate of candidacy for the office affected in accordance with the
preceding sections not later than mid-day of the election. If the death, withdrawal
Issue: WON the Motion for execution Pending Appeal should be or disqualification should occur between the day before the election and mid-day
granted of election day, said certificate may be filed with any board of election inspectors
in the political subdivision where he is a candidate, or in the case of candidates to
Held: No. be voted for by the entire electorate of the country, with the Commission.

The Apocryphal Maggots:


Rainier, Chrisgel, Corina, Geoffry, Grace and Sylvie Blanche
The Flibbertigibbet Worms:
Golda, Gladys and Melyjane
CA VE AT: By simply r ea ding this re vi ew er a t the end o f the sem este r wi ll (hopef ully) guar antee you r pas sing this cou rse. D rink mode rat ely .
Amusin S C A r y
gly
THE C2005 LOCAL GOVERNMENT REVIEWER - 139 -
IN CAB, since Pempe’s COC had been denied and cancelled, Joel • Assuming that all 3 were fielded-in by same political party, at
could not have validly substituted him. time Recabo Jr filed his certificate, there was no more void to
fill in as Reyes had already filed his certificate as official
2. Under the provision of Sec 77, not just any person but only “an candidate of Lakas. No more vacancy to be substituted for.
official candidate of a registered or accredited political party” may  Disunity and discord among members of a political
be substituted. party should not be allowed to create a mockery of
Any person who attempts to run for an elective office but does not the electoral process, which envisions one
file a COC (just like what Pempe is considered to be IN CAB) is not candidate from a political party for each position.
a candidate at all.
IN CAB, since Pempe’s COC had been denied and cancelled, he
was NOT an “official candidate” under Sec. 77 and therefore, Joel Rulloda vs. COMELEC (2003)
could not have validly substituted him.
Facts:
DISSENT: Romero
DISSENT: Panganiban • Romeo Rulloda and Remegio Placido were contending
candidates for Brgy. Chairperson
!!! Dan Gat's Notes: • Romeo passed away so his widow Petronila Rulloda
A practical application of the Miranda case is: if you (petitioner) wrote a letter to COMELEC to seek permission to
do not want your opponent to be substituted, ask for the run as candidate in lieu of her dead husband.
cancellation of his COC, not mere disqualification. • Widow Rulloda won
Sir notes though… the reason why you file for • However, COMELEC issued Resolution 4801 and Resolution
cancellation is because your opponent is disqualified? BUT the 5217 insofar as they prohibiting her from running as substitute
SC distinguishes disqualification from cancellation, so wala na candidate. Placido proclaimed winner.
tayong magagawa dun.
Issue: W/N Rulloda’s COC should be given due course
Held: YES.
Recabo vs. COMELEC (1999) 1. Placido’s argument that there can be no substitution because
there is no political party in brgy. elections (which is non-
partisan) from which to designate the substitute holds no
Facts: water. Contrary to respondent’s claim, the absence of a
• Candelaria Recabo who filed her COC as Vice-mayor of specific provision governing substitution of candidates in
Mainit, Surigao but later withdrew it. barangay elections can not be inferred as a prohibition
• The COC of Candelaria has a formal defect: signed by only against said substitution.
one representative of her alleged party • Indeed, there is more reason to allow the substitution of
• Reyes filed his COC as official candidate of Lakas as vice- candidates where no political parties are involved than when
mayor of Mainit, Surigao. political considerations or party affiliations reign, a fact that
• Kaiser Recabo Jr. also filed his COC, claiming to be the must have been subsumed by law.
official candidate of Lakas also as vice-mayor of Mainit as a 2. Placido’s contention that the votes in petitioner’s favor can not
substitute candidate for the office of Candelaria Recabo be counted because she did not file any certificate of
• Recabo won the elections, but… candidacy so he was the only candidate for Barangay
• COMELEC cancelled Recabo’s certificate of candidacy Chairman is untenable.
Comelec Resolution: letter-request is allowed, treated as a
Issue: W/N Recabo is correct in arguing that his election cannot certificate of candidacy.
be annulled based on formal defects in his Certificate of Candidacy
because the electorate has voted clearly in his favor. “Popular will
MyPleasure Guide: Erotic Massage (Part 4):
as clearly expressed in votes cast and counted should prevail.”
Held: NO, the doctrine that a mere technicality cannot be used to
Balms, Dusts and Body Play
frustrate the people’s will finds no application in CAB considering
that the results of the election have not been duly established. These potions and lotions are the most innovative and
• “Certified List of Candidates with their Votes Obtained” and interesting of the topical sexual enhancers. Spanning a wide
Undated “Certified List of Winning Candidates” submitted by range that includes body paint, edible body frosting, honey
Recabo do not sufficiently establish the real results of the dust, nipple cream and "Good Head gel", body balms are a
election. creative addition to anyone's pleasure chest.
• only acceptable evidence of election results: election returns When using any kind of body balm, keep a few common-sense
pursuant to BP881. facts in mind. Most sensual unguents contain oil, which means
they'll stain just about any fabric they come into contact with.
Issue: WON Recabo could still be allowed to run We recommend using an old set of sheets. Also, body play can
Held: No

The Apocryphal Maggots:


Rainier, Chrisgel, Corina, Geoffry, Grace and Sylvie Blanche
The Flibbertigibbet Worms:
Golda, Gladys and Melyjane
CA VE AT: By simply r ea ding this re vi ew er a t the end o f the sem este r wi ll (hopef ully) guar antee you r pas sing this cou rse. D rink mode rat ely .
Amusin S C A r y
gly
THE C2005 LOCAL GOVERNMENT REVIEWER - 140 -
be very messy. While we don't expect you to throw down a The barangay elections shall be nonpartisan and shall be
drop cloth, you also don't want to stain your couch, carpeting or conducted in an expeditious and inexpensive manner.
towels. Think about how your cream-colored chenille throw
Sec3: The construction or maintenance of provincial, city,
blanket will look when smeared with blue body paint, and
municipal and barangay-funded roads and bridges shall be
consider draping the couch with an old towel before getting out
prohibited for a period of ten (10) days immediately preceding
the paintbrushes. (end) the date of election, the provisions of Section 261, paragraphs
(v) and (w), of the Omnibus Election Code to the contrary
notwithstanding.
Term of Office
Sec5: There shall be a sangguniang barangay in every duly
Sec43: Term of Office. — constituted barangay which shall be the legislative body and
(a) The term of office of all local elective officials elected after shall be composed of seven (7) kagawads to be elected by the
the effectivity of this Code shall be three (3) years, starting from registered voters of the barangay. The candidate who obtains
noon of June 30, 1992 or such date as may be provided for by the highest number of votes shall be the punong barangay and
law, except that of elective barangay officials: Provided, That all in the event of a tie, there shall be a drawing of lots under the
local officials first elected during the local elections immediately supervision of the Commission on Elections. The chairman of
following the ratification of the 1987 Constitution shall serve until the kabataang barangay who shall hereafter be elected in
noon of June 30, 1992. accordance with law shall be an ex officio member of the
(b) No local elective official shall serve for more than sangguniang barangay. In the event of any vacancy in the office
three (3) consecutive terms in the same position. Voluntary of the punong barangay, whether temporary or permanent, or in
renunciation of the office for any length of time shall not be case of disqualification or refusal to assume office, an order of
considered as an interruption in the continuity of service for the succession among the six (6) members of the sangguniang
full term for which the elective official concerned was elected. barangay based upon the number of votes received from the
(c) The term of office of barangay officials and members highest to the lowest shall be followed to fill up the vacancy. In
of the sangguniang kabataan shall be for three (3) years, which default of any duly elected member of the sangguniang
shall begin after the regular election of barangay officials on the barangay, the vacancy may be filled up by the city or municipal
second Monday of May 1994. mayor concerned.
Approved: November 4, 1988

1987 Constitution – Article X


RA8524 (1998) – An Act Changing The Term Of Office Of
Sec8: The term of office of elective local officials, except barangay Barangay Officials And Members Of The Sangguniang
officials, which shall be determined by law, shall be three years Kabataan From Three (3) Years To Five (5) Years, Amending
and no such official shall serve for more than three consecutive For The Purpose Section 43 Of RA7160, Otherwise Known
terms. Voluntary renunciation of the office for any length of time As The Local Government Code Of 1991, And For Other
shall not be considered as an interruption in the continuity of his Purposes
service for the full term for which he was elected.
Sec1: Section 43 of Republic Act No. 7160, otherwise known as
the Local Government Code of 1991, is hereby amended to
RA6679 (1988) - An Act To Amend Republic Act No. 6653 To read as follows:
Postpone The Barangay Elections To March 28, 1989, "SEC. 43. Term of Office. — (a) The term of office of all elective
Prescribing Additional Rules Governing The Conduct Of officials elected after the effectivity of this Code shall be three
Barangay Elections And For Other Purposes (3) years, starting from noon of June 30, 1992 or such date as
may be provided for by law, except that of elective barangay
officials and members of the sangguniang kabataan: Provided,
Sec1: The elections of barangay officials set on the second
That all local officials first elected during the local elections
Monday of November 1988 by Republic Act No. 6653 are
immediately following the ratification of the 1987 Constitution
hereby postponed and reset to March 28, 1989. They shall
shall serve until noon of June 30, 1992.
serve a term which shall begin on the first day of May 1989 and
"(b) No local elective official shall serve for more than
ending on the thirty-first day of May 1994.
three (3) consecutive terms in the same position. Voluntary
There shall be held a regular election of barangay officials on
renunciation of the office for any length of time shall not be
the second Monday of May 1994 and on the same day every
considered as an interruption in the continuity of service for the
five (5) years thereafter. Their term shall be for five (5) years
full term for which the elective official concerned was elected.
which shall begin on the first day of June following the election
"(c) The term of barangay officials and members of the
and until their successors shall have been elected and qualified:
sangguniang kabataan shall be for five (5) years, which shall
Provided, That no barangay official shall serve for more than
begin after the regular election of barangay officials on the
three (3) successive terms. acd
second Monday of May 1997: Provided, That the sangguniang

The Apocryphal Maggots:


Rainier, Chrisgel, Corina, Geoffry, Grace and Sylvie Blanche
The Flibbertigibbet Worms:
Golda, Gladys and Melyjane
CA VE AT: By simply r ea ding this re vi ew er a t the end o f the sem este r wi ll (hopef ully) guar antee you r pas sing this cou rse. D rink mode rat ely .
Amusin S C A r y
gly
THE C2005 LOCAL GOVERNMENT REVIEWER - 141 -
kabataan members who were elected in the May 1996 elections (a) To prevent the establishment of political dynasties is not the
shall serve until the next regular election of barangay officials." only policy embodied in the constitutional provision in
question. The other policy is that of enhancing the freedom of
Sec2: The provisions of this Act shall apply to the incumbent choice of the people. To consider only the stay in office
barangay officials and members of the sangguniang kabataan. regardless of how the official concerned came to that office
Approved: February 14, 1998 would be to disregard one of the purposes of Article X, §8 of
the Constitution.
(b) The first sentence speaks of "the term of office of elective
RA9006 (2001) – An Act To Enhance The Holding Of Free, local officials" and bars "such official[s]" from serving for more
Orderly, Honest, Peaceful And Credible Elections Through than three consecutive terms. The second sentence, in
Fair Election Practices explaining when an elective local official may be deemed to
have served his full term of office, states that "voluntary
Sec14: Repealing Clause. — Sections 67 and 85 of the Omnibus renunciation of the office for any length of time shall not be
Election Code (BP881) and Sections 10 and 11 of RA6646 are considered as an interruption in the continuity of his service
hereby repealed. As a consequence, the first proviso in the third for the full term for which he was elected." The term served
paragraph of Section 11 of RA8436 is rendered ineffective. All must therefore be one "for which [the official concerned] was
laws, presidential decrees, executive orders, rules and elected." The purpose of this provision is to prevent a
regulations, or any part thereof inconsistent with the provisions circumvention of the limitation on the number of terms an
of this Act are hereby repealed or modified or amended elective local official may serve. Conversely, if he is not
accordingly. serving a term for which he was elected because he is simply
continuing the service of the official he succeeds, such official
cannot be considered to have fully served the term
notwithstanding his voluntary renunciation of office prior to its
RA9164 (2002) – An Act Providing For Synchronized
expiration.
Barangay And Sangguniang Kabataan Elections, Amending
Ra7160, As Amended, Otherwise Known As The "Local Lonzanida vs. COMELEC (1999)
Government Code Of 1991", And For Other Purposes
Facts:
Sec2: Term of Office. — The term of office of all barangay and • Lonzanida served two consecutive terms as Mayor prior to
sangguniang kabataan officials after the effectivity of this Act the 1995 elections. In the May 1995 elections Lonzanida ran
shall be three (3) years. for and won as mayor.
No barangay elective official shall serve for more than three (3) • His proclamation in 1995 was however contested by his then
consecutive terms in the same position: Provided, however, that opponent Alvez who filed an election protest
the term of office shall be reckoned from the 1994 barangay • COMELEC declared Alvez the duly elected mayor of San
elections. Voluntary renunciation of office for any length of time Antonio, Zambales.
shall not be considered as an interruption in the continuity of • Alvez assumed office for the remainder of Lonzanida's term.
service for the full term for which the elective official was • In 1998 elections Lonzanida again ran for mayor of San
elected. Antonio. His opponent Muli filed a petition to disqualify
Lonzanida from running on the ground that he had served
three consecutive terms in the same post.
Borja vs. COMELEC (1998)
Issue: WON Lonzanida's assumption of office from May 1995 to
Facts: March 1998 may be considered as service of one full term for the
• Capco was elected Pateros vice-mayor of Pateros on 1988 purpose of applying the three-term limit for elective local
for a term ending June 1992. On September 2, 1989, he government officials
became mayor, upon the death of the incumbent. Held: NO
• Capco ran for and won as mayor in the 1992 and 1995 • 2 conditions for the application of the disqualification must
elections.
concur:
• Capco filed a certificate of candidacy for mayor of Pateros for 1) that the official concerned has been elected for three
the 1998 elections. Borja who was also a candidate for consecutive terms in the same local government post
mayor, sought Capco's disqualification on the theory that the and
latter would have already served as mayor for three 2) that he has fully served three consecutive terms.
consecutive terms (counting 1989-1992 term)
• IN CAB: The two requisites are absent.
Issue: WON Capco’s service as mayor from September 2, 1989 to
June 30, 1992 is considered as service for one full term. 1. Not elected for three terms: Petitioner cannot be considered
Held: NO as having been duly elected to the post and did not fully serve
the 1995-1998 mayoral term by reason of involuntary

The Apocryphal Maggots:


Rainier, Chrisgel, Corina, Geoffry, Grace and Sylvie Blanche
The Flibbertigibbet Worms:
Golda, Gladys and Melyjane
CA VE AT: By simply r ea ding this re vi ew er a t the end o f the sem este r wi ll (hopef ully) guar antee you r pas sing this cou rse. D rink mode rat ely .
Amusin S C A r y
gly
THE C2005 LOCAL GOVERNMENT REVIEWER - 142 -
relinquishment of office. After a re-appreciation and revision of been elected to the same position for the same number of times
the contested ballots the COMELEC itself declared by final before the disqualification can apply.
judgment that petitioner Lonzanida lost in the May 1995 For the disqualification to apply, 2 conditions must concur:
mayoral elections and his previous proclamation as winner 1. that the official has been elected for 3 consecutive terms
was declared null and void. His assumption of office as in the same local govt. post
mayor cannot be deemed to have been by reason of a 2. that he has fully served the 3 consecutive terms.
valid election but by reason of a void proclamation. A
proclamation subsequently declared void is no proclamation For nearly 2 years, Talaga was a private citizen. The continuity of
at all. his mayorship was disrupted by his defeat in the 1998 elections.
2. Did not fully served the May 1995 to 1998 term : because he Fr. Bernas’ comment (if one is elected representative to serve the
unexpired term of another, that unexpired, no matter how short,
was ordered to vacate his post before the expiration of the
will be considered 1 term xxx) is pertinent only to members of the
term.
House of Rep.
• Voluntary v. Involuntary Renunciation: Voluntary renunciation
of a term does not cancel the renounced term in the
computation of the three term limit; conversely, involuntary Socrates vs. COMELEC (2002)
severance from office for any length of time short of the full
term provided by law amounts to an interruption of continuity Facts:
of service. The petitioner vacated his post not by • July 2, 2002- Puerto Princessa barangay officials
voluntary renunciation but in compliance with the legal convened into a Preparatory Recall Assembly to initiate the
process of writ of execution issued by the COMELEC to recall of Mayor Socrates. Resolution No. 01-02 was passed.
that effect. Such involuntary severance from office is an • Socrates questioned Resolution but Comelec upheld it.
interruption of continuity of service.
• Hagedorn filed his CoC for mayor in said recall election.
• Respondents filed a petition to disqualify him and to cancel
!!! Dan Gat's Notes:
his CoC on the ground that such would then be his fourth
SC has made a distinction between:
consecutive term having been elected for 3 consecutive terms
a. voluntary renunciation  term deemed fully served
b. involuntary renunciation  term not deemed fully served Issue: WON Hagedorn is qualified to run for mayor
Held: YES
3-term limit rule for elective local officials is found in Sec 8 Art X of
Sir asks…what if the official, on his third term of office, was
the Consti and on Sec. 43(b) of the LGC. These constitutional and
suspended for one day  involuntary renunciation? So he can statutory provisions have 2 parts:
run again in the next election? 1. provides that an elective local official cannot serve for more than
3 consecutive terms. The clear intent is that only consecutive
terms count in determining the 3-term rule limit.
Adormeo vs. COMELEC (2002) 2. voluntary renunciation of the office for any length of time does
not interrupt the continuity of service. The clear intent is that
Facts: involuntary severance from office for any length of time interrupts
• Talaga, served as mayor: continuity of service and prevents the service before and after the
a. May 1992 – served the full term interruption from being joined together from a continuous service
b. 1995 – served a full term or consecutive terms.
c. 1998- lost to Tagaro
d. 2000 recall election – served the unexpired portion of The prohibited election refers to the next regular election of the
Tagarao same office following the end of the 3rd consecutive term.
• Talaga and Adormeo filed their COCs for the 2001 elections
• Adormeo filed a petition to cancel the COC of and disqualify Any subsequent election is no longer covered by the prohibition for
2 reasons:
Talaga on the ground that it had served as a city mayor for 3
1. A subsequent election like a recall election is no longer an
consecutive terms which is in violation of Sec 8 Art X of the
immediate reelection after 3 consecutive terms
Consti.
2. The intervening period constitutes an involuntary interruption in
the continuity of service.
Issue: WON Talaga is qualified to run as Mayor of Lucena.
Held: YES
Deliberation of the ConCom shows that what is prohibited is an
Talaga has not yet served for 3 consecutive terms.
immediate reelection for a 4th term immediately after the end of
The term limit for elective local officials must be taken to refer to
the 3rd consecituve term.
the right to be elected as well as the right to serve in the same
elective position. It is not enough that an individual has served 3
consecutive terms in an elective local office, he must also have

The Apocryphal Maggots:


Rainier, Chrisgel, Corina, Geoffry, Grace and Sylvie Blanche
The Flibbertigibbet Worms:
Golda, Gladys and Melyjane
CA VE AT: By simply r ea ding this re vi ew er a t the end o f the sem este r wi ll (hopef ully) guar antee you r pas sing this cou rse. D rink mode rat ely .
Amusin S C A r y
gly
THE C2005 LOCAL GOVERNMENT REVIEWER - 143 -
Hagedorn's candidacy in the recall election does not fall under the • In order that the 3-consecutive term limit can apply, 2
said prohibition. He was elected for 3 consecutive terms in the conditions must concur:
1992, 1995, and 1998 elections and served in full his 3 o That the elective local official concerned has been
consecutive terms as mayor of Puerto Princesa. Under the Consti elected for 3 consecutive terms to the same local
and the LGC he can no longer run for the 2001 elections because government position, and
of which it was Socrates who ran and won the said position. o That he has served 3 consecutive full terms.
Hagedorn then became a private citizen for nearly 15 months. • A winner who dislodges in a recall election an incumbent
Said period is clearly an interruption in the continuity of elective local official merely serves the balance of the latter’s
HAgedorn's service as mayor not because of voluntary term of office; it is not a full 3-year term.
renunciation but because of a legal prohibition.
• < It also goes without saying that an incumbent elective local
official against whom a recall election is initiated and who
Constitution does not require that the interruption be a full term of 3
nevertheless wins in a recall election must be viewed as
years. The clear intent is that interruption for any length of time is
being a continuing term of office and not as a break in
sufficient to break an elective local official's continuity of service.
reckoning his 3 consecutive terms.>
Hagedorn's recall term does not retroact to include the tenure in • If involuntary severance from the service which results in the
office of Socrates. He does not serve the full term of the incumbent’s being unable to finish his term of office because
predecessor but only the expired term. of his ouster through valid recall proceedings negates “one
term” for purposes of applying the 3-term limit, it stands to
!!! Dan Gat's Notes: reason that the balance of the term assumed by the newly
elected local official in a recall election should not also be
Ah! So an LGU elective official can stay in power forever! held to be one term in reckoning the 3-term limit. In both
Serve three terms; then be sure your successor gets recalled; situations, neither the elective local official who is unable to
then be sure to win in the recall election; then you have a fresh finish his term nor the elected local official who only assumes
three terms again after serving the recall term. the balance of the term of the ousted official following the
recall election could be considered to have served a full 3-
year term set by the Constitution.

Mendoza vs. COMELEC (2001) Mendoza, J: (joined by Quisumbing, J.):


(sorry kung mahaba ito. Many important arguments - geof)
Facts: WON Roman’s last election violates the 3-term limit in the Consti.
Considering that at that time he had served for only 7 years – NO
• Leonardo B. Roman held the post of governor of Bataan
• To summarize, in applying the 3-term limit, the term during
province for several terms:
which succession takes place or a recall election is held
Years served Manner of Assumption
should not be counted, either with the 3 consecutive terms
1986-1988 Appointed OIC Governor of Bataan province by
preceding, or with the 3 consecutive terms succeeding, such
former Pres. Aquino and served up to 1988
term. It should not be counted, not because of any
1988-1992 Elected gov. and served up to 1992
interruption in the continuity of the service but because such
1994-1995 Elected gov. during the recall election in 1993, term is for less than 3 years. Hence, the unexpired portion of
assumed office on June 28, 1994 and served a term, whether filled by succession or by election in a recall,
up to 1995 cannot be considered 1 full term. In the CAB, since Roman’s
1995-1998 Elected gov. and served up to 1998 first election in 1993 was in consequence of a recall and not a
1998-2001 Elected gov. and served up to 2001 regular election and he had not fully served 3 consecutive
2001-2004 Elected gov. and presently the incumbent gov. terms when he was elected in 2001, his last election is valid.
• Petitioners filed with the COMELEC a petition for quo The cases of Borja and Arcos are on all fours with the CAB.
• Succession and recall are alike. They are both modes of
warranto, alleging that Roman has served as governor of
Bataan for 3 consecutive terms counted from his assumption succession for the purpose of automatically filling permanent
of office by virtue of the 1993 recall election. As such, he is vacancies in elective local offices to prevent a hiatus in office.
disqualified/ ineligible to seek a 4th term for the same position The local official who succeeds to the office or is elected in a
in 2001. recall simply finishes the term of his predecessor.
• After due deliberation, the SC voted 8-7 to dismiss the
Panganiban, J.: (joined by Puno, J.) Dismisses Petition
petition.
An Apparent Distinction That Does Not Make a Real Difference
• To argue and differentiate – that in one case there was
Vitug, J: (joined by Ynares-Santiago, J.)
WON Roman’s incumbency to the post of Governor following the succession to office and in the other a recall election – would
recall elections be included in determining the 3-consecutive term be to quibble over an apparent distinction that does not make
limit - NO a real difference.

The Apocryphal Maggots:


Rainier, Chrisgel, Corina, Geoffry, Grace and Sylvie Blanche
The Flibbertigibbet Worms:
Golda, Gladys and Melyjane
CA VE AT: By simply r ea ding this re vi ew er a t the end o f the sem este r wi ll (hopef ully) guar antee you r pas sing this cou rse. D rink mode rat ely .
Amusin S C A r y
gly
THE C2005 LOCAL GOVERNMENT REVIEWER - 144 -
• Petitioners opine that in establishing term limits, the Consti. know that such term is in itself 1 term although less than 3
intended to prevent a local official from holding the same years. This is the inherent limitation he takes by running and
office for a period longer than 3 consecutive terms or a total winning in the recall election. It now boggles my mind why
of 9 years. However, whether the initial accession to office the majority has made a complete turn-around and totally
was by virtue of succession/ operation of law or by virtue of a disregarded this significant pronouncement which could have
recall election, the same evil (monopoly of political power) given life to the constitutional mandate.
might still arise at some point down the road.
Carpio, J.: (joined by Carpio-Morales, J.)
• One unusual mode of entry into public office would be WON a recall term is considered as 1 term for purposes of
simplistically favored over another if one official is allowed to counting the 3-term limit of elective local officials – YES
serve more than 3 terms, on the ground that the excess was
by virtue of a legal succession to a vacant office; and to • The instant case is not a situation where the official
disallow another from so serving, simply because the excess succeeded by operation of law to the office and served the
was by reason of a recall election. Simplistically, assumption unexpired term of his predecessor as in Borja. Here, Roman
of office by operation of law would be favored over that was elected to serve the unexpired term of his predecessor.
by recall election. To say that the recall term is a stray term, belonging to no
• Such line of reasoning puts a higher premium on an elected official in counting the 3-term limit, is to ignore reality.
accidental or opportunistic succession to office (for ex., A recall term arises from a special election for a fixed term of
through the death of the incumbent) over a collective and office – the unexpired term of the recalled official. The
earnest expression of the people’s sovereign will (as through official elected in a recall election has the same functions
a recall election). and powers as an official elected to the same office in a
regular election. The recall term is a legal and political
• The SC’s pronouncement in Socrates is of no avail either. The fact that cannot just be dismissed as a stray term.
analysis therein cannot be regarded as controlling insofar as • In Adormeo and Socrates, it was ruled that the recall term is
the instant case is concerned. In that case, the main issue not consecutive to the previous terms of one who wins the
was whether a recall election that took place after the 4th recall election against the recalled official. The term of office
consecutive election had taken place was to be deemed an of the incumbent or recalled official serves to break the
“immediate reelection” to a 4th term. The SC answered “No,” continuity of service of the comebacking official who wins a
there was “no immediate reelection after 3 consecutive recall election. But a recall term of an official who is re-
terms.” elected in the next 2 regular elections, like that of Roman, is
• The issues in Socrates did not include the question whether a not interrupted by any term of another official. Thus, such
recall term should be considered 1 term for purposes of recall term should be counted in computing the 3-term limit.
reckoning term limits. Therefore, the SC’s ratiocination that a • To consider a recall term as a stray term will encourage a
recall term is 1 term for purposes of counting the 3-term limit person disqualified because of the 3-term limit to agitate for
may be regarded merely as an obiter dictum. the recall of his immediate successor.

Sandoval-Gutierrez, J.: Azcuna, J.: (joined by Bellosillo, J.)


(joined by Davide, C.J, Austria-Martinez, Corona, and Callejo, Sr.) • In the present case, Roman’s election as governor in the
WON a governor, elected in a recall election and who has held recall election should not be counted as 1 full term. For the
office for the unexpired term of his predecessor, is considered to disqualification to attach, 3 consecutive terms must be served
have served a full term for the purpose of applying the 3-term limit in full. This is the exception to the rule, so it must be strictly
under the Consti – YES complied with. Service for less than the full term, except only
• Said constitutional and statutory provisions on term limits in case of voluntary renunciation, should not be counted to
make no distinction as to the nature of the election – whether determine the existence of the disqualification.
regular, special or recall elections. Where the law does not
distinguish, the courts should make no distinction. These Voted to DISMISS pet: Voted to GRANT pet.:
provisions do not confine the 3-term rule to regular elections Vitug Sandoval-Gutierrez
only. They include any election (such as recall election) for Ynares-Santiago Davide
the same position. Mendoza Austria-Martinez
• To exclude the service of such official who won the recall Quisumbing Corona
election would certainly permit a circumvention of the purpose Panganiban Callejo, Sr.
of the 3-term rule Puno Carpio
• The SC, just a month ago, declared in Socrates that although Azcuna Carpio-Morales
an official elected in a recall election serves the unexpired Bellosillo
term of the recalled official, this unexpired term is in itself 1 David vs. COMELEC (1997)
term for purposes of counting the 3-term limit. It went further
in saying that a local official who serves a recall term should

The Apocryphal Maggots:


Rainier, Chrisgel, Corina, Geoffry, Grace and Sylvie Blanche
The Flibbertigibbet Worms:
Golda, Gladys and Melyjane
CA VE AT: By simply r ea ding this re vi ew er a t the end o f the sem este r wi ll (hopef ully) guar antee you r pas sing this cou rse. D rink mode rat ely .
Amusin S C A r y
gly
THE C2005 LOCAL GOVERNMENT REVIEWER - 145 -
Section 43(c) of R.A. 7160 reads: The term of office of barangay Unforeseen and unforeseeable, since the vacating is expressly
officials and members of the sangguniang kabataan shall be for 3 made as of the moment of the filing of the COC.
years, which shall begin after the regular election of barangay
officials on the second Monday of May 1994. Aguinaldo vs. COMELEC (1999)

Issue: WON the law which governs the term of office of bgy. • Petitioners Aguinaldo et al were incumbent provincial or
officials is RA 7160 (and not 6679)
municipal officials in Cagayan. They seek to prevent the
Held: YES, RA 7160
Comelec from enforcing during the 1998 elections Section 67
• In light of the historical background, the intent and design of of the Omnibus Election Code (BP 881).
the legislature to limit the term of barangay officials to only 3
years as provided under the LGC emerges. • Section 67 of the Omnibus Election Code provides:
Sec. 67. Any elective official, whether national or local,
Issue: WON the 3-year term is repugnant to the Constitution running for any office other than the one which he is holding
Held: NO in a permanent capacity, except for President and Vice
(Liga ng mga Barangay posits that by excepting barangay officials President, shall be considered ipso facto resigned from his
whose "term shall be determined by law" from the general office upon the filing of his certificate of candidacy
provision fixing the term of "elective local officials" at 3 years, the Issue: WON Section 67 is unconstitutional for violating the equal
Constitution thereby impliedly prohibits Congress from legislating a protection clause
3-year term for such officers.) Held: No
• The Constitution did not expressly prohibit Congress from
fixing any term of office for barangay officials. It merely left the
• In Dimaporo v Mitra, the Supreme Court ruled that
determination of such term to the lawmaking body, without 1. Section 67 seeks to ensure that elective public officials serve
any specific limitation or prohibition, thereby leaving to the out their entire term of office by discouraging them from
lawmakers full discretion to fix such term in accordance with running for another public office and thereby cutting short
the exigencies of public service. their tenure by making it clear that should they fail in their
candidacy, they cannot go back to their former position.
2. Section 67 was crafted with the intention of giving flesh to the
Monroy vs. CA (1967) constitutional mandate that public service is a public trust.
This means that all elective officials should honor the
Facts: mandate they have gotten from the people. They should not
be allowed to deviate from this mandate by running for
• Roberto Monroy was the incumbent mayor of Navotas, Rizal another position during his term of office, unless he
when he filed his certificate of candidacy as representative of relinquishes or abandons his office.
the 1st district of Rizal. If a Batasan member, governor or mayor who was mandated
• 3 days later, he filed a petition with the Comelec withdrawing to serve for a fixed number of years is allowed to run for an
said certificate of candidacy. The Comelec approved the office other than the one he was elected to serve, then that
withdrawal. clearly shows that he does not have the intention to service
• However, then Vice mayor Felipe del Rosario took his oath of the mandate of the people which was placed upon him and
office as municipal mayor on the theory that: therefore he should be considered as ipso facto resigned.
- Monroy had forfeited his office upon his filing of the (from the records of the deliberations of the Batasang
certificate of candidacy in question. Pambansa)
3. Section 67 does not unduly cut short the term of office of local
Issue: WON withdrawal of COC restored petitioner to his former officials. The situation that results with the application of
position Section 67 is covered by the term “voluntary renunciation.”
Held: NO The filing of a certificate of candidacy for another office
1. The Law: Section 27 of the Revised Election Code: constitutes an overt, concrete act of voluntary renunciation of
Any elective provincial, municipal or city official running the elective office presently being held by the concerned
for an office. Other than the one which he is actually holding, shall official.
be considered resigned from his office from the moment of the
filing of his COC. Hunting the Elusive G-Spot by Tamar Love
2. Forfeiture is therefore automatic and permanently effective upon
the filing of the COC for another office. Virtually everyone has heard stories about a magic spot inside a
- consider only the MOMENT and ACT of filing woman that, if properly stimulated, will send her to paroxysms
3. JBL Reyes in Castro vs. Gatuslao: of sexual ecstasy. Known as the G-Spot, this area has been the
The wording of the law plainly indicates that only the subject of several books and countless magazine articles. But
date of the filing xxx should be taken into account. The law does
does it really exist? If so, how do you find it?
not make the forfeiture dependent upon future contingencies.

The Apocryphal Maggots:


Rainier, Chrisgel, Corina, Geoffry, Grace and Sylvie Blanche
The Flibbertigibbet Worms:
Golda, Gladys and Melyjane
CA VE AT: By simply r ea ding this re vi ew er a t the end o f the sem este r wi ll (hopef ully) guar antee you r pas sing this cou rse. D rink mode rat ely .
Amusin S C A r y
gly
THE C2005 LOCAL GOVERNMENT REVIEWER - 146 -
The simple answer is yes, the G-Spot does exist, but but there's (b) Except for the sangguniang barangay, only the
nothing mysterious about it. It's simply an area of increased nominee of the political party under which the sanggunian
sensitivity that many women like to have stimulated. Named member concerned had been elected and whose elevation to
the position next higher in rank created the last vacancy in the
after Ernst Grafenberg, a German medical doctor who wrote
sanggunian shall be appointed in the manner hereinabove
about "an erotic zone located on the anterior wall of the vagina
provided. The appointee shall come from the same political
along the course of the urethra that would swell during sexual party as that of the sanggunian member who caused the
stimulation," the G-Spot was popularized during the Women's vacancy and shall serve the unexpired term of the vacant office.
Movement in The G Spot, a best-selling book by Alice K. Ladas, In the appointment herein mentioned, a nomination and a
Beverly Whipple and John D. Perry. (continued…) certificate of membership of the appointee from the highest
Vacancies and Succession official of the political party concerned are conditions sine qua
non, and any appointment without such nomination and
Sec44: Permanent Vacancies in the Offices of the Governor, certification shall be null and void ab initio and shall be a ground
Vice-Governor, Mayor, and Vice-Mayor. — If a permanent for administrative action against the official responsible
vacancy occurs in the office of the governor or mayor, the vice- therefore.
governor or vice-mayor concerned shall become the governor or (c) In case or permanent vacancy is caused by a
mayor. If a permanent vacancy occurs in the offices of the sanggunian member who does not belong to any political party,
governor, vice-governor, mayor, or vice-mayor, the highest the local chief executive shall, upon recommendation of the
ranking sanggunian member or, in case of his permanent sanggunian concerned, appoint a qualified person to fill the
inability, the second highest ranking sanggunian member, shall vacancy.
become the governor, vice-governor, mayor or vice-mayor, as (d) In case of vacancy in the representation of the youth
the case may be. Subsequent vacancies in the said office shall and the barangay in the sanggunian, said vacancy shall be filled
be filled automatically by the other sanggunian members automatically by the official next in rank of the organization
according to their ranking as defined herein. concerned.
(b) If a permanent vacancy occurs in the office of the
punong barangay, the highest ranking sanggunian barangay Sec46: Temporary Vacancy in the Office of the Local Chief
member or, in case of his permanent inability, the second Executive. —
highest ranking sanggunian member, shall become the punong (a) When the governor, city or municipal mayor, or punong
barangay. barangay is temporarily incapacitated to perform his duties for
(c) A tie between or among the highest ranking physical or legal reasons such as, but not limited to, leave of
sanggunian members shall be resolved by the drawing of lots. absence, travel abroad, and suspension from office, the vice-
(d) The successors as defined herein shall serve only the governor, city or municipal vice-mayor, or the highest ranking
unexpired terms of their predecessors. sangguniang barangay member shall automatically exercise the
For purposes of this Chapter, a permanent vacancy arises when powers and perform the duties and functions of the local chief
an elective local official fills a higher vacant office, refuses to executive concerned, except the power to appoint, suspend, or
assume office, fails to qualify, dies, is removed from office, dismiss employees which can only be exercised if the period of
voluntarily resigns, or is otherwise permanently incapacitated to temporary incapacity exceeds thirty (30) working days.
discharge the functions of his office. (b) Said temporary incapacity shall terminate upon
For purposes of succession as provided in the Chapter, ranking submission to the appropriate sanggunian of a written
in the sanggunian shall be determined on the basis of the declaration by the local chief executive concerned that he has
proportion of votes obtained by each winning candidate to the reported back to office. In cases where the temporary incapacity
total number of registered voters in each district in the is due to legal causes, the local chief executive concerned shall
immediately preceding local election. also submit necessary documents showing that said legal
causes no longer exist.
Sec45: Permanent Vacancies in the Sanggunian. — (c) When the incumbent local chief executive is traveling
(a) Permanent vacancies in the sanggunian where automatic within the country but outside his territorial jurisdiction for a
succession provided above do not apply shall be filled by period not exceeding three (3) consecutive days, he may
appointment in the following manner: designate in writing the officer-in-charge of the said office. Such
(1) The President, through the Executive Secretary, in the authorization shall specify the powers and functions that the
case of the sangguniang panlalawigan and the local official concerned shall exercise in the absence of the local
sangguniang panlungsod of highly urbanized cities and chief executive except the power to appoint, suspend, or dismiss
independent component cities; employees.
(2) The governor, in the case of the sangguniang (d) In the event, however, that the local chief executive
panlungsod of component cities and the sangguniang concerned fails or refuses to issue such authorization, the vice-
bayan; governor, the city or municipal vice-mayor, or the highest
(3) The city or municipal mayor, in the case of sangguniang ranking sangguniang barangay member, as the case may be,
barangay, upon recommendation of the sangguniang shall have the right to assume the powers, duties, and functions
barangay concerned. of the said office on the fourth (4th) day of absence of the said

The Apocryphal Maggots:


Rainier, Chrisgel, Corina, Geoffry, Grace and Sylvie Blanche
The Flibbertigibbet Worms:
Golda, Gladys and Melyjane
CA VE AT: By simply r ea ding this re vi ew er a t the end o f the sem este r wi ll (hopef ully) guar antee you r pas sing this cou rse. D rink mode rat ely .
Amusin S C A r y
gly
THE C2005 LOCAL GOVERNMENT REVIEWER - 147 -
local chief executive, subject to the limitations provided in Held: YES
subsection (c) hereof. 1 Under the circumstances of this case:
(e) Except as provided above, the local chief executive a.Silence of the LGC as to the mode of succession in the
shall in no case authorize any local official to assume the
event of a temporary vacancy in the office of Vice-Gov;
powers, duties, and functions of the office, other than the vice-
and
governor, the city or municipal vice-mayor, or the highest
b. Necessity for the appointment of Vice-Gov demanded by
ranking sangguniang barangay member, as the case may be.
the exigencies of public service - to prevent a delay in
the delivery of basic services and the disruption in the
Sec47: Approval of Leaves of Absence. —
proper management of the affairs of Leyte during the
(a) Leaves of absence of local elective officials shall be
time when no Gov had yet been proclaimed
approved as follows:
…the President, through her alter ego, the Sec of Local Govt, may
(1) Leaves of absence of the governor and the mayor of a
remedy the situation [by declaring the temporary appointment of
highly urbanized city or an independent component city
Menzon as Acting Vice-Gov].
shall be approved by the President or his duly authorized
representative;
2 CA 588 & the Revised Admin Code of 1987, authorizing the Prex
(2) Leaves of absence of vice-governor or a city or
to make temporary appointments in case of vacancies in
municipal vice-mayor shall be approved by the local
appointive positions, may be similarly applied in the present case.
chief executive concerned: Provided, That the leaves of
- in the absence of any contrary provision in the LGC
absence of the members of the sanggunian and its
- in the best interest of public service
employees shall be approved by the vice-governor or
city or municipal vice-mayor concerned;
(3) Leaves of absence of the component city or municipal
mayor shall be approved by the governor; and
(4) Leaves of absence of a punong barangay shall be
approved by the city or municipal mayor: Provided, That
leaves of absence of sangguniang barangay members Gamboa vs. Aguirre (1999)
shall be approved by the punong barangay.
(b) Whenever the application for leave of absence Facts:
hereinabove specified is not acted upon within five (5) working • Negros Occidental Governor Coscolluela designated
days after receipt thereof, the application for leave of absence petitioner Gamboa as acting governor for the duration of the
shall be deemed approved. governor's official trip abroad
• Subsequently, when the SP held its regular session,
respondents (members of the Sangguniang Panlalawigan)
Menzon vs. Petilla (1991) questioned the authority of Gamboa to preside therein in
view of his designation as Acting Governor
Facts:
• No Governor was proclaimed for the province of Leyte due to Issue: WON an incumbent Vice-Governor, while concurrently the
a pending election case before the COMELEC. Acting Governor, continue to preside over the sessions of the
Sangguniang Panlalawigan
• Sec of Local Govt Santos designated Vice-Gov Petilla as
Held:
Acting Gov of Leyte, and Sangguniang Panlalawigan Member
NO. There results a temporary vacancy in the office of the Vice-
Menzon as Acting Vice-Gov.
Governor whenever the latter acts as Governor by virtue of a
• The Sangguniang Panlalawigan issued Resolution 505, temporary vacancy.
resolving NOT to recognize the designation of Menzon as 1. unlike under the old LGC where the Governor is not only
Acting Vice-Gov. the provincial chief executive but also the presiding officer
of the local legislative body, the new LGC deprived the
Issue1: WON there was a vacancy in the office of Vice-Gov Governor of the power to preside over the SP
Held: YES
 this is clear from the provision of the LGC that the "local
There is a vacancy when there is no person lawfully authorized to
legislative power shall be vested in the SP" (Sec 48) which
assume and exercise at present the duties of the office.
is comprised of:
HERE  The office of Vice-Gov was left vacant when the duly 1.) Vice Governor, as presiding officer
elected Vice-Gov, Petilla was appointed as Acting Gov. In the eyes 2.) Regular elective SP members
of the law, the office to which he was elected was left barren of a 3.) Three elective sectoral representatives
legally qualified person to exercise the duties of the office of Vice- 4.) ex-officio members, nameley:
Gov. a.) president of the provincial chapter of the liga ng
mga barangay
Issue2: WON the Sec. of Interior had the authority to designate b.) president of the panlalawigang pederasyon ng
Menzon as Acting Vice-Gov mga sangguniang kabataan

The Apocryphal Maggots:


Rainier, Chrisgel, Corina, Geoffry, Grace and Sylvie Blanche
The Flibbertigibbet Worms:
Golda, Gladys and Melyjane
CA VE AT: By simply r ea ding this re vi ew er a t the end o f the sem este r wi ll (hopef ully) guar antee you r pas sing this cou rse. D rink mode rat ely .
Amusin S C A r y
gly
THE C2005 LOCAL GOVERNMENT REVIEWER - 148 -
c.) president of the provincial federation of SEC. 50. Permanent Vacancies in Local Sanggunians. Except for
sanggunian members of municipalities and the sangguniang barangay, the appointee shall come from the
component cities political party of the sanggunian member who caused the vacancy,
 Governor is not in the list! when the law enumerates, the and shall serve the unexpired term of the vacant office.
law necessarily excludes. Consequently, the union of 1. From the tenor of the appointment extended to Docena on
legislative-executive powers in the Governor has been November 19, 1990, there is no question that it was intended to be
disbanded permanent. Petitioner's appointment had already become
complete and enforceable at the time it was supposed to have
2. It has been held that if a Mayor who is out of country is been "superseded" by the appointment in favor of Alar.
considered effectively absent, the Vice-Mayor should  Docena had already acquired security of tenure in the
discharge the duties of the mayor. This should equally position and could be removed therefrom only for any of the
apply in the CAB. causes, and conformably to the procedure, prescribed by the
Local Government Code. These requirements could not be
 Although it is difficult to lay down a definite rule as to what
circumvented by the simple process of recalling his
constitutes "absence", this term should be reasonably appointment.
construed to mean "effective" absence  one that renders
the officer concerned powerless for the time being to 2. The respondents are ambivalent about the power of the
discharge the powers and prerogatives of his office Secretary of Local Government to recall his appointments.
 Being the Acting Gov, the Vice Gov cannot continue to  They described the December 19 recall of Alar as
simultaneously exercise the duties of the latter office, since "whimsical, capricious and wishy-washy" for lack of a
the nature of the duties of the provincial Governor call for a previous hearing (although they had no similar complaints
full time occupant to discharge them. about the recall of Docena's appointment although also made
without hearing).
3. Remedy: Article 49 (b) of LGC: election of a temporary
presiding officer.
!!! Dan Gat's Notes:
Docena still good law?

Victoria vs. COMELEC (1994)

Issue: How should the ranking of SP members be computed for


the purpose of succession?
Docena vs. Sangguniang Panlalawigan of Eastern Samar Petitioner’r argument: ranking of the SP members should not only
(1991) be based on the number of votes obtained in relation to the total
number of registered voters, but also on the number of voters in
Facts the district who actually voted therein (which will result in petitioner
• Luis Capito, elected and serving as member of the Victoria ranking 1st)
Sangguniang Panlalawigan (SP) died in office
Held:
• Petitioner Agustin Docena was appointed to succeed Capito Sec 44 of LGCode last par provides:
• This appontment was issued by Department of Local . . . . "For purposes of succession as provided in this Chapter,
Government Secretary Santos on November 19, 1990 ranking in the sanggunian shall be determined on the basis of the
• November 27: For unknown reasons, respondent Socrates proportion of votes obtained by each winning candidates to the
Alar was also appointed by Secretary Santos to the position total number of registered voters in each district in the immediately
already occupied by Docena preceding local election."
• December 18: the SP passed Resolution No. 75 recognizing The law is clear that the ranking in the SP shall be determined on
Alar rather than Docena as legitimate successor of the late the basis of the proportion of the votes obtained by each winning
Capito candidate to the total number of registered voters of each district.
• December 19: Secretary Santos sent a letter to Alar, It does not mention anything about factoring the numbers of voters
informing the latter of the prior appointment of Docena and who actually voted.
recalling Alar's appointment

Issue: WON Docena should be the appointed member to the Fariñas vs. Barba (1996)
Sangguniang Panlalawigan
Held: Yes Issue1: In case of a permanent vacancy in the Sangguniang
The pertinent legal provision is Section 50, LGC: Bayan caused by the cessation from office of a member who does

The Apocryphal Maggots:


Rainier, Chrisgel, Corina, Geoffry, Grace and Sylvie Blanche
The Flibbertigibbet Worms:
Golda, Gladys and Melyjane
CA VE AT: By simply r ea ding this re vi ew er a t the end o f the sem este r wi ll (hopef ully) guar antee you r pas sing this cou rse. D rink mode rat ely .
Amusin S C A r y
gly
THE C2005 LOCAL GOVERNMENT REVIEWER - 149 -
not belong to any political party, who can appoint the replacement of a member must be made by the mayor upon the
and in accordance with what procedure? recommendation of that Sanggunian. The reason is that members
Held: Governor upon the recommendation of SB concerned of the Sangguniang Barangay are not allowed to have party
Since the vacancy in this case was created by a SB member who affiliations.
did not belong to any political party, the specific provision involved
is par. (c) of Sec. 45 to wit: Issue2: Is the appointing authority limited to the appointment of
In case the permanent vacancy is caused by a sanggunian those "recommended" to him?
member who does not belong to any political party, the local Held: YES
chief executive shall, upon recommendation of the The appointing authority is not bound to appoint anyone
sanggunian concerned, appoint a qualified person to fill the recommended to him by the Sanggunian concerned. The power of
vacancy. appointment is a discretionary power. On the other hand, neither is
To be sure the President of the Philippines can not be referred to the appointing power vested with so large a discretion that he can
as "local chief executive" in Sec. 45(c). It is apparent that the disregard the recommendation of the Sanggunian concerned.
phrase is a misnomer and that the choice of this phrase was Since the recommendation takes the place of nomination by
simply dictated by the need to avoid, for stylistic reasons, political party, the recommendation must likewise be considered a
interminably repeating the officials on whom the power to appoint condition sine qua non for the validity of the appointment, by
is conferred. Perhaps "authorities concerned" would have been a analogy to the provision of Sec. 45(b).
more accurate generic phrase to use. IN CAB, since neither Nacino nor Palafox was appointed in the
manner indicated in the SB of San Nicolas. For while Nacino was
The phrase "sanggunian concerned" in Sec. 45(c) should more appointed by Gov, he was not recommended by the SB of San
properly be understood as referring to the Sanggunian in which the Nicolas. On the other hand, Palafox was recommended by the
vacancy is created. This is in keeping with the policy implicit in mayor and not the provincial governor who appointed him.
Sec. 45(a)(3).

CONSTRUCTION OF SEC. 45: Navarro vs. CA (2001)

Where the Permanent Vacancy is Caused by a Sanggunian Facts:


Member belonging to a Political Party May11, 1997 local elections in Mapandan, Pangasinan, following
Sangguniang Panlalawigan and Sanguniang Panlungsod of officials were elected to office…
highly urbanized cities and independent component cities — Calimlim – Mayor – Lakas NUCD-KAMPI
The President, through the Executive Secretary, upon the Aquino – Vice-Mayor – Lakas…
nomination and certification of the political party to which the As members of Sangguniang Bayan ranked accordingly…
member who caused the vacancy belonged, as provided in Sec. 1. Tamayo – REFORMA-LM
45(b). 2. Soriano – REFORMA…
Sangguniang Panlungsod of component cities and Sangguniang 3. Biagtan – REFORMA
Bayan — The Governor upon the nomination and certification of 4. Lalas – REFORMA
the political party to which the member who caused the vacancy 5. Eden, Jr. – REFORMA
belonged, as provided in Sec. 45(b). 6. Lalangan – Lakas…
7. Pascual – REFORMA
Where the Vacancy is Caused by a Sanggunian Member Not 8. Lalas – Lakas…
Belonging to a Political Party
Sangguniang Panlalawigan and Sangguniang Panlungsod of • Mayor Calimlim died.
highly urbanized and independent component cites — The • Vice-Mayor Aquino succeeded as Mayor by operation of law
President, through the Executive Secretary, upon (Sec44, LGC) filling up the vacancy created. Accordingly,
recommendation- of the Sangguniang Panlalawigan or Tamayo, the highest ranking member of the Sangguniang
Sangguniang Panlungsod as the case may be. Bayan (the one who garnered the highest number of votes)
Sangguniang Panlungsod of component cities and Sangguniang was elevated to the position of Vice-Mayor, pursuant to same
Bayan — The Governor upon recommendation of the law.
Sangguniang Panlungsod or Sangguniang Bayan as the case
• Since a vacancy occurred in Sang. Bayan by elevation of
may be.
Tamayo to Vice-Mayor office, Governor Agbayani appointed
Where the Vacancy is Caused by a Member of the Sangguniang Navarro (belonging to and recommended by REFORMA
Barangay — City or Municipal Mayor upon recommendation of party, party of Tamayo) as Member.
the Sangguniang Barangay • Private respondents (from Lakas) filed civil case to nullify
appointment of Navarro before RTC.

Issue1: Who created the last vacancy?


There is only one rule governing appointments to the Sangguniang
Held: Elevation of Tamayo to Office of Vice Mayor.
Barangay. Any vacancy therein caused by the cessation from office

The Apocryphal Maggots:


Rainier, Chrisgel, Corina, Geoffry, Grace and Sylvie Blanche
The Flibbertigibbet Worms:
Golda, Gladys and Melyjane
CA VE AT: By simply r ea ding this re vi ew er a t the end o f the sem este r wi ll (hopef ully) guar antee you r pas sing this cou rse. D rink mode rat ely .
Amusin S C A r y
gly
THE C2005 LOCAL GOVERNMENT REVIEWER - 150 -
Under Sec44, a permanent vacancy arises when an elective • ViceMayor Navarro, while serving as Acting Mayor of
official: Santiago City because of Mayor Miranda’s suspension,
(1) fills a higher vacant office appointed respondents to positions in the city gov’t. – with
(2) refuses to assume office permanent status and based on the evaluation made by the
(3) fails to qualify City Personnel Selection and Promotion Brd (PSPB),
(4) dies approved by CSC.
(5) is removed from office • When Mayor Miranda reassumed his post on March5, 1998,
(6) voluntarily resigns he considered the composition of PSPB irregular since
(7) otherwise permanently incapacitated to discharge the majority party was not properly represented.
functions of his office • 3months after (June10), Miranda issued an order terminating
Crucial to the interpretation of Sec45(b) providing that “xxx only the Respondents' services effective June15 because they
nominee of the political party under which the Sanggunian member performed poorly during the probationary period.
concerned has been elected and whose elevation to the position
next higher in rank created the last vacancy in the Sanggunian Issue: W/N respondents’ services were illegally terminated.
shall be appointed in the manner hereinabove provided. The Held: YES. Respondents already under Civil Service, so the Civil
appointee shall come from the political party as that of the Service Law applies.
Sanggunian member who caused the vacancy xxx.” I. Observance of Condition of Probationary Period
Reason behind the right given to a political party to nominate a To be able to gauge whether a subordinate is inefficient or
replacement where a permanent vacancy occurs in the incompetent requires enough time on the part of his immediate
Sanggunian  to maintain the party representation as willed superior within which to observe his performance.
by the people in the election. Civil Service Law provides specific grounds for dismissing a
government officer of employee from service, among which are
In CAB, inefficiency and incompetence in performance of official duties.
• with the elevation of Tamayo (from REFORMA) to position of Dismissal was on the ground of poor performance. Poor
ViceMayor, a vacancy occurred in the Sanggunian that should performance falls within the concept of inefficiency and
be filled up with someone who should belong to same political incompetence in the performance of official duties which are
party. grounds from dismissing a government official or employee from
• Otherwise, REFORMA’s representation in Sanggunian would service. This concept requires passage of sufficient time
be diminished. (probationary period of 6 months) for it to be determined.
• To argue that vacancy created was that formerly held by
CAB: However, the condition was not observed – where it is
Lalas (from Lakas) as concluded by CA, would result in the improbable that Mayor Miranda could finally determine their
increase of that party’s representation at the expense of performance for only the 1st 3 months of probationary period.
REFORMA. This is contrary to the letter and spirit of the law
and thus violative of a fundamental rule in statutory II. Due Process
construction  to ascertain and give effect to the intent and Omnibus Guidelines on Appointments and Other Personnel
purpose of the law. Actions (CSC Memo Circular) provides that an official or employee
may be dropped from rolls based on poor performance in a rating
• The “last vacancy” in the Sanggunian refers to that created by after due notice – that he be informed in writing of the status
the elevation of the member formerly occupying the next of his performance not later than the 4th month of that period
higher in rank which in turn also had become vacant by any of with sufficient warning that failure to improve his performance
the causes already enumerated. within the remaining period of semester shall warrant
• The term is thus used in Sec45(b) to differentiate it from the separation from service… with sufficient information to enable
other vacancy previously created. employee to prepare an explanation.
• The term by no means refers to the vacancy in No.8 position
CAB: Respondents were denied due process.
1. They vehemently asserted that they were never notified in
which occurred with the election of Lalas to 7th position. Such writing regarding the status of their performance, nor warned
construction will result in absurdity. that they will be dismissed for failure to improve which Joel
did not refute.
2. Records show that they received only the termination order.

Miranda vs. Carreon (2003)

Facts:

The Apocryphal Maggots:


Rainier, Chrisgel, Corina, Geoffry, Grace and Sylvie Blanche
The Flibbertigibbet Worms:
Golda, Gladys and Melyjane
CA VE AT: By simply r ea ding this re vi ew er a t the end o f the sem este r wi ll (hopef ully) guar antee you r pas sing this cou rse. D rink mode rat ely .
Amusin S C A r y
gly
THE C2005 LOCAL GOVERNMENT REVIEWER - 151 -
professional relationship or any relation by affinity or
Local Legislation - LGC consanguinity within the fourth civil degree, which he may have
with any person, firm, or entity affected by any ordinance or
Sec48: Local Legislative Power. — Local legislative power shall resolution under consideration by the sanggunian of which he is
be exercised by the sangguniang panlalawigan for the province; a member, which relationship may result in conflict of interest.
the sangguniang panlungsod for the city; the sangguniang Such relationship shall include:
bayan for the municipality; and the sangguniang barangay for (1) Ownership of stock or capital, or investment, in the entity
the barangay. cdtai or firm to which the ordinance or resolution may apply;
and
Sec49: Presiding Officer. — (2) Contracts or agreements with any person or entity which
(a) The vice-governor shall be the presiding officer of the the ordinance or resolution under consideration may
sangguniang panlalawigan; the city vice-mayor, of the affect.
sangguniang panlungsod; the municipal vice-mayor, of the In the absence of a specific constitutional or statutory provision
sangguniang bayan; and the punong barangay, of the applicable to this situation, "conflict of interest" refers in general
sangguniang barangay. The presiding officer shall vote only to to one where it may be reasonably deduced that a member of a
break a tie. sanggunian may not act in the public interest due to some
(b) In the event of the inability of the regular presiding private, pecuniary, or other personal considerations that may
officer to preside at a sanggunian session, the members present tend to affect his judgment to the prejudice of the service or the
and constituting a quorum shall elect from among themselves a public.
temporary presiding officer. He shall certify within ten (10) days (b) The disclosure required under this Act shall be made
from the passage of ordinances enacted and resolutions in writing and submitted to the secretary of the sanggunian or
adopted by the sanggunian in the session over which he the secretary of the committee of which he is a member. The
temporarily presided. disclosure shall, in all cases, form part of the record of the
proceedings and shall be made in the following manner:
Sec50: Internal Rules of Procedure. — (1) Disclosure shall be made before the member
(a) On the first regular session following the election of its participates in the deliberations on the ordinance or
members and within ninety (90) days thereafter, the sanggunian resolution under consideration: Provided, That, if the
concerned shall adopt or update its existing rules of procedure. member did not participate during the deliberations, the
(b) The rules of procedure shall provided for the following: disclosure shall be made before voting on the ordinance
(1) The organization of the sanggunian and the election of or resolution on second and third readings; and
its officers as well as the creation of standing (2) Disclosure shall be made when a member takes a
committees which shall include, but shall not be limited position or makes a privilege speech on a matter that
to, the committees on appropriations, women and family, may affect the business interest, financial connection, or
human rights, youth and sports development, professional relationship described herein.
environmental protection, and cooperatives; the general
jurisdiction of each committee; and the election of the Sec52: Sessions. —
chairman and members of each committee; (a) On the first day of the session immediately following the
(2) The order and calendar of business for each session; election of its members, the sanggunian shall, by resolution, fix
(3) The legislative process; the day, time, and place of its regular sessions. The minimum
(4) The parliamentary procedures which include the conduct numbers of regular sessions shall be once a week for the
of members during sessions; sangguniang panlalawigan, sangguniang panlungsod, and
(5) The discipline of members for disorderly behavior and sangguniang bayan, and twice a month for the sangguniang
absences without justifiable cause for four (4) barangay.
consecutive sessions, for which they may be censured, (b) When public interest so demands, special sessions
reprimanded, or excluded from the session, suspended may be called by the local chief executive or by a majority of the
for not more than sixty (60) days, or expelled: Provided, members of the sanggunian.
That the penalty of suspension or expulsion shall require (c) All sanggunian sessions shall be open to the public
the concurrence of at least two-thirds (2/3) vote of all the unless a closed-door session is ordered by an affirmative vote of
sanggunian members: Provided, further, That a member a majority of the members present, there being a quorum, in the
convicted by final judgment to imprisonment of at least public interest or for reasons of security, decency, or morality.
one (1) year for any crime involving moral turpitude shall No two (2) sessions, regular or special, may be held in a single
be automatically expelled from the sanggunian; and day.
(6) Such other rules as the sanggunian may adopt. (d) In the case of special sessions of the sanggunian, a
written notice to the members shall be served personally at the
Sec51: Full Disclosure of Financial and Business Interests of member's usual place of residence at least twenty-four (24)
Sanggunian Members. — hours before the special session is held.
(a) Every sanggunian member shall, upon assumption to office, Unless otherwise concurred in by two-thirds (2/3) vote of the
make a full disclosure of his business and financial interests, or sanggunian members present, there being a quorum, no other

The Apocryphal Maggots:


Rainier, Chrisgel, Corina, Geoffry, Grace and Sylvie Blanche
The Flibbertigibbet Worms:
Golda, Gladys and Melyjane
CA VE AT: By simply r ea ding this re vi ew er a t the end o f the sem este r wi ll (hopef ully) guar antee you r pas sing this cou rse. D rink mode rat ely .
Amusin S C A r y
gly
THE C2005 LOCAL GOVERNMENT REVIEWER - 152 -
matters may be considered at a special session except those items of an appropriations ordinance, an ordinance or resolution
stated in the notice. adopting a local development plan and public investment
(e) Each sanggunian shall keep a journal and record of program, or an ordinance directing the payment of money or
its proceedings which may be published upon resolution of the creating liability. In such a case, the veto shall not affect the item
sanggunian concerned. or items which are not objected to. The vetoed item or items
shall not take effect unless the sanggunian overrides the veto in
the manner herein provided; otherwise, the item or items in the
appropriations ordinance of the previous year corresponding to
Sec53: Quorum. — those vetoed, if any, shall be deemed reenacted.
(a) A majority of all the members of the sanggunian who have (c) The local chief executive may veto an ordinance or
been elected and qualified shall constitute a quorum to transact resolution only once. The sanggunian may override the veto of
official business. Should a question of quorum be raised during the local chief executive concerned by two-thirds (2/3) vote of all
a session, the presiding officer shall immediately proceed to call its members, thereby making the ordinance effective even
the roll of the members and thereafter announce the results. without the approval of the local chief executive concerned.
(b) Where there is no quorum, the presiding officer may
declare a recess until such time as a quorum is constituted, or a Sec56: Review of Component City and Municipal Ordinances
majority of the members present may adjourn from day to day or Resolutions by the Sangguniang Panlalawigan. —
and may compel the immediate attendance of any member (a) Within three (3) days after approval, the secretary to the
absent without justifiable cause by designating a member of the sanggunian panlungsod or sangguniang bayan shall forward to
sanggunian to be assisted by a member or members of the the sangguniang panlalawigan for review, copies of approved
police force assigned in the territorial jurisdiction of the local ordinances and the resolutions approving the local development
government unit concerned, to arrest the absent member and plans and public investment programs formulated by the local
present him at the session. development councils.
(c) If there is still no quorum despite the enforcement of (b) Within thirty (30) days after the receipt of copies of
the immediately preceding subsection, no business shall be such ordinances and resolutions, the sangguniang panlalawigan
transacted. The presiding officer, upon proper motion duly shall examine the documents or transmit them to the provincial
approved by the members present, shall then declare the attorney, or if there be none, to the provincial prosecutor for
session adjourned for lack of quorum. prompt examination. The provincial attorney or provincial
prosecutor shall, within a period of ten (10) days from receipt of
Sec54: Approval of Ordinances. — the documents, inform the sangguniang panlalawigan in writing
(a) Every ordinance enacted by the sangguniang panlalawigan, of his comments or recommendations, which may be considered
sangguniang panlungsod, or sangguniang bayan shall be by the sangguniang panlalawigan in making its decision.
presented to the provincial governor or city or municipal mayor, (c) If the sangguniang panlalawigan finds that such an
as the case may be. If the local chief executive concerned ordinance or resolution is beyond the power conferred upon the
approves the same, he shall affix his signature on each and sangguniang panlungsod or sangguniang bayan concerned, it
every page thereof; otherwise, he shall veto it and return the shall declare such ordinance or resolution invalid in whole or in
same with his objections to the sanggunian, which may proceed part. The sangguniang panlalawigan shall enter its action in the
to reconsider the same. The sanggunian concerned may minutes and shall advise the corresponding city or municipal
override the veto of the local chief executive by two-thirds (2/3) authorities of the action it has taken.
vote of all its members, thereby making the ordinance or (d) If no action has been taken by the sangguniang
resolution effective for all legal intents and purposes. panlalawigan within thirty (30) days after submission of such an
(b) The veto shall be communicated by the local chief ordinance or resolution, the same shall be presumed consistent
executive concerned to the sanggunian within fifteen (15) days with law and therefore valid.
in the case of a province, and ten (10) days in the case of a city
or a municipality; otherwise, the ordinance shall be deemed Sec57: Review of Barangay Ordinances by the Sangguniang
approved as if he had signed it. Panlungsod or Sangguniang Bayan. —
(c) Ordinances enacted by the sangguniang barangay (a) Within ten (10) days after its enactment, the sangguniang
shall, upon approval by the majority of all its members, be barangay shall furnish copies of all barangay ordinances to the
signed by the punong barangay. sangguniang panlungsod or sangguniang bayan concerned for
review as to whether the ordinance is consistent with law and
Sec55: Veto Power of the Local Chief Executive. — city or municipal ordinances.
(a) The local chief executive may veto any ordinance of the (b) If the sangguniang panlungsod or sangguniang
sanggunian panlalawigan, sangguniang panlungsod, or bayan, as the case may be, fails to take action on barangay
sanggunian bayan on the ground that it is ultra vires or ordinances within thirty (30) days from receipt thereof, the same
prejudicial to the public welfare, stating his reasons therefor in shall be deemed approved.
writing. (c) If the sangguniang panlungsod or sangguniang
(b) The local chief executive, except the punong bayan, as the case may be, finds the barangay ordinances
barangay, shall have the power to veto any particular item or inconsistent with law or city or municipal ordinances, the

The Apocryphal Maggots:


Rainier, Chrisgel, Corina, Geoffry, Grace and Sylvie Blanche
The Flibbertigibbet Worms:
Golda, Gladys and Melyjane
CA VE AT: By simply r ea ding this re vi ew er a t the end o f the sem este r wi ll (hopef ully) guar antee you r pas sing this cou rse. D rink mode rat ely .
Amusin S C A r y
gly
THE C2005 LOCAL GOVERNMENT REVIEWER - 153 -
sanggunian concerned shall, within thirty (30) days from receipt consecutive days in a newspaper of local circulation: Provided,
thereof, return the same with its comments and however, That in provinces, cities and municipalities where there
recommendations to the sangguniang barangay concerned for are no newspapers of local circulation, the same may be posted
adjustment, amendment, or modification; in which case, the in at least two (2) conspicuous and publicly accessible places.
effectivity of the barangay ordinance is suspended until such
time as the revision called for is effected. Sec511: Posting and Publication of Ordinances with Penal
Sanctions. —
Sec58: Enforcement of Disapproved Ordinances or (a) Ordinances with penal sanctions shall be posted at
Resolutions. — Any attempt to enforce any ordinance or any prominent places in the provincial capitol, city, municipal or
resolution approving the local development plan and public barangay hall, as the case may be, for a minimum period of
investment program, after the disapproval thereof, shall be three (3) consecutive weeks. Such ordinances shall also be
sufficient ground for the suspension or dismissal of the official or published in a newspaper of general circulation, where
employee concerned. available, within the territorial jurisdiction of the local government
unit concerned, except in the case of barangay ordinances.
Unless otherwise provided therein, said ordinances shall take
effect on the day following its publication, or at the end of the
Sec59: Effectivity of Ordinances or Resolutions. — period of posting, whichever occurs later.
(a) Unless otherwise stated in the ordinance or the resolution (b) Any public officer or employee who violates an
approving the local development plan and public investment ordinance may be meted administrative disciplinary action,
program, the same shall take effect after ten (10) days from the without prejudice to the filing of the appropriate civil or criminal
date a copy thereof is posted in a bulletin board at the entrance action.
of the provincial capitol or city, municipal, or barangay hall, as (c) The secretary to the sanggunian concerned shall
the case may be, and in at least two (2) other conspicuous transmit official copies of such ordinances to the chief executive
places in the local government unit concerned. officer of the Office Gazette within seven (7) days following the
(b) The secretary to the sanggunian concerned shall approval of the said ordinance for publication purposes. The
cause the posting of an ordinance or resolution in the bulletin Official Gazette may publish ordinances with penal sanctions for
board at the entrance of the provincial capitol and the city, archival and reference purposes.
municipal, or barangay hall in at least two (2) conspicuous
places in the local government unit concerned not later than five
(5) days after approval thereof. Implementing Rules

The text of the ordinance or resolution shall be disseminated See Implementing Rules
and posted in Filipino or English and in the language understood RULE XVII: Local Legislative Bodies and Local Legislation
by the majority of the people in the local government unit Article 92 - Article 114
concerned, and the secretary to the sanggunian shall record
such fact in a book kept for the purpose, stating the dates of
approval and posting.
Ortiz vs. Posadas (1931)
(c) The gist of all ordinances with penal sanctions shall
be published in a newspaper of general circulation within the Facts:
province where the local legislative body concerned belongs. In • 13 out of 16, including president of the municipal council of
the absence of any newspaper of general circulation within the Tabaco Albay, were present and voted for Ordinance No. 25
province, posting of such ordinances shall be made in all concerning cockpits in this manner:
municipalities and cities of the province where the sanggunian 7 in favor
of origin is situated. 6 voted against
3 absent.
(d) In the case of highly urbanized and independent • One judge of first instance of the province held that it was
component cities, the main features of the ordinance or valid, while another judge of first instance of the province held
resolution duly enacted or adopted shall, in addition to being an ordinance enacted under similar circumstances invalid.
posted, be published once in a local newspaper of general
circulation within the city: Provided, That in the absence thereof Issue: WON Ordinance is valid
the ordinance or resolution shall be published in any newspaper Held: NO, invalid
of general circulation. • Section 2224 of the Administrative Code is clear. It needs only
application, not interpretation. The ayes and noes are taken
Sec188: Publication of Tax Ordinances and Revenue upon (1) the passage of all ordinances, (2) all propositions to
Measures. — Within ten (10) days after their approval, certified create any liability against the municipality, and (3) any other
true copies of all provincial, city, and municipal tax ordinances or proposition, upon the request of any member. The same idea
revenue measures shall be published in full for three (3) is carried into the succeeding sentence. For the passage of

The Apocryphal Maggots:


Rainier, Chrisgel, Corina, Geoffry, Grace and Sylvie Blanche
The Flibbertigibbet Worms:
Golda, Gladys and Melyjane
CA VE AT: By simply r ea ding this re vi ew er a t the end o f the sem este r wi ll (hopef ully) guar antee you r pas sing this cou rse. D rink mode rat ely .
Amusin S C A r y
gly
THE C2005 LOCAL GOVERNMENT REVIEWER - 154 -
(1) any ordinance or (2) any proposition creating matter. In the enactment of ordinances in general, the
indebtedness, the affirmative vote of a majority of all the application of the aforementioned laws cannot be disputed.
members of the municipal council shall be necessary. Other Undeniably, however, Section 6.44 of said ordinance
measures prevail upon the majority vote of the members regarding amendments thereto is a specific and particular
present. "Creating indebtedness" refers to "proposition" and provision for said ordinance and explicitly provides for a
not to "ordinance." The contention that only ordinances different number of votes.
creating indebtedness require the approval of a majority of all
the members of the municipal council, is devoid of merit.
• Rule: Where there is in the same statute a particular
• The basic idea of the legislative body to make impossible the enactment and also a general one which in its most
comprehensive sense would include what is embraced in the
approval of ordinances or of propositions creating
former, the particular enactment must be operative, and the
indebtedness by minority votes of municipal councils, at
general statement must be taken to affect only such cases
meetings hastily called is wise. Legislative intention should be
within its language as are not within the provisions of the
effectuated.
particular enactment.
• Section 2224 of the Administrative Code, requiring in
mandatory language the affirmative vote of a majority of all
the members of the municipal council for the passage of any !!!Dan Gat's Notes:
ordinance, whether or not an ordinance creating
indebtedness, an ordinance passed by less than that majority Ortiz and Casino decisions are bad precedents. Sanggunian
is invalid. may no require a higher OR LOWER (1 vote!?) voting
requirement?
Casino vs. CA (1991)
Malonzo vs. Zamora (1999)
• Petitioner was a licensee of a cockpit under Sections 2285 to
2286 of the Revised Administrative Code. Issue: WON Ord. 0254 was enacted in violation of Secs. 50 & 52
of the LGC
• Sangguniang Panlungsod (SP) of Gingoog City issued
PETITIONERS: Ordinance 0254 was passed in accordance with
Resolution No. 49 in 1984 classifying certain areas of the city
Sec. 50 &52 of the LGC12. Matter of adoption or updating the
as residential zones, including the cockpit. The classification
house rules was taken up and the council arrived at the decision to
led to the cancellation of petitioner's license to operate the
create an ad hoc committee to study the rules. Even if it failed to
cockpit.
comply with said sections, it would only mean that the old rules will
• Resolution No. 378, in 1985 the area was reclassified as continue to be in force
within the recreational zone, allegedly amending Resolution RESPONDENTS: Sanggunian violated Sec 50 & 52 LGC as it
No. 49. conducted 3 readings on Ordinance 0254 on the 1st day of its
9 SP members participated session without 1st organizing itself and adopting its rules of
4 voting procedure
4 voted against
1abstained. Held: NO
The vice-mayor, as presiding officer, broke the deadlock by It cannot be inferred that the Code mandates that no other
voting for the amendment. business may be transacted on the 1st regular session except to
• By virtue of said Resolution No. 378, the succeeding city take up the matter of adopting or updating rules.
mayor issued to petitioner the permit to operate a cockpit in There is nothing in the language of the provision which restricts the
1986 which was renewed by another permit issued in 1987. matters to be taken up during the 1st regular session.
Adoption or updating of house rules would necessarily entail work
Issue: WON the mayor's permits issued in 1986 and 1987 are null beyond the day of the 1st regular session but this cannot mean that
and void because Resolution 378 did not amend Section 6.44 of the local council could not act till then.
Resolution 49, the three-fourths (3/4) votes not having been There was sufficient compliance with the Code when the
obtained in passing said Resolution 378. Sanggunian adopted a set of house rules with the request to
Held: YES, void create an ad hoc committee to study the existing house rules.
• Although the charter of the City of Gingoog and the Local
KAPUNAN dissenting
Government Code require only a majority for the enactment
of an ordinance, Resolution No. 49 cannot be validly
amended by the resolution in question without complying with
the categorical requirement of a three-fourths vote 12
which requires that on the 1st regular session following
incorporated in the very same ordinance sought to be
amended. The pertinent provisions in the aforesaid city the election of its members and within 90 days
charter and the Local Government Code obviously are of thereafter, the Sanggunian shall adopt or update its
general application and embrace a wider scope or subject existing rules of procedure.

The Apocryphal Maggots:


Rainier, Chrisgel, Corina, Geoffry, Grace and Sylvie Blanche
The Flibbertigibbet Worms:
Golda, Gladys and Melyjane
CA VE AT: By simply r ea ding this re vi ew er a t the end o f the sem este r wi ll (hopef ully) guar antee you r pas sing this cou rse. D rink mode rat ely .
Amusin S C A r y
gly
THE C2005 LOCAL GOVERNMENT REVIEWER - 155 -
a year after the effectivity of the ordiance in 1996. Clearly, the
De Los Reyes vs. Sandiganbayan (1997) Sec. of Justice correctly dismissed it for being time-barred.
• The timeframe fixed by law for parties to avail of their legal
Facts: remedies before competent courts is not a “mere technicality”
• Mayor De los Reyes and 2 SB members were charged with that can be easily brushed aside. The periods stated in Sec.
falsification of a public document, Resolution 57-S-92. 187 of the LGC are mandatory.
• Resolution alleged to be anomalous for it was not approved
by the SB. The minutes of their proceedings made no !!! Dan Gat's Notes:
reference to its supposed approval.
• Petitioner De Los Reyes: deliberations and passage of Section 187 LGC is indeed mandatory. But if you want to
question the CONSTITUTIONALITY of the tax ordinance, you
resolution are legislative in nature. As the local chief exec. he
can bypass appeal to the Secretary and Section 187 and go
has neither the official custody of nor the duty to prepare the
directly to the courts.
resolution. He therefore could not have taken advantage of
his official position in committing the crime.

Issue: WON the final step in the approval of an ordinance or


resolution, where the local chief exec affixes his signature, is
purely ministerial.
Held: NO
The grant of the veto power confers authority beyond the simple Marahomsalic vs. Lim (2000)
mechanical act of signing an O/R as a requisite to its enforceability.
Such power accords the local chief exec the discretion to sustain Facts:
the O/R at the 1st instance or to return it to the sanggunian with his • Then Mayor Alfredo Lim launched a campaign against so-
objections. called “known” and “confirmed” drug pushers in Manila in an
Hagonoy Market Vendor Assn. vs. Municipality of Hagonoy, effort to curb the drug problem in Manila.
Bulacan (2002) • He ordered the Western Police District Command authorities
to spray-paint the houses of “confirmed” drug pushers in the
Facts: city with written words and remarks
• 1 Oct. 1996: The Sangguniang Bayan (SB) of Hagonoy,
Bulacan, enacted an ordinance, Kautusan Blg. 28, which Issue: WON Manila City Ordinance No. 7926 is unconstitutional
increased the stall rentals of the market vendors in Hagonoy. Held:YES
It also provided that it shall take effect upon approval. • According to the Art. 7 of the Civil Code, acts, orders and
Ordiance was also duly posted. regulations of administrative or executive agencies must be in
• In the last week of Nov., 1997, the petitioner’s members were harmony with the valid existing laws and primarily with the
given copies of the approved ordinance and were informed Constitution.
that it will be enforced in Jan., 1998. • Test of a valid ordinance:
• Dec. 8, 1997: The association filed an appeal with the Sec. of o It must not contravene the Constitution or any statute;
Justice, assailing the constitutionality of the ordinance o It must not be unfair or oppressive;
• The DOJ Sec. dismissed the appeal on the ground that it was o It must not be partial or discriminatory;
filed out of time, i.e. beyond 30 days from the effectivity of the o It must not prohibit but may regulate trade;
ordinance on 1 Oct. 1996 as prescribed under the LGC. o It must be general and consistent with public policy; and
• Appeal to CA, CA dismissed. o It must not be unreasonable.

Issue: WON the petition should be dismissed


Held: YES Moday vs. CA (1995)
• The appeal of the petitioner with the Sec. of Justice is already
time-barred. Facts:
• Sec. 187 (Procedure for Approval and Effectivity of Tax • 1989: SB of the Municipality of Bunawan passed resolution
Ordinances and Revenue Measures; Mandatory Public 43-89:
Hearings) of the LGC requires that an appeal of a tax - authorized the mayor to initiate the petition for expropriation
ordinance or revenue measure should be made to the Sec. of of a 1 hectare land along the national highway
Justice within 30 days from effectivity of the ordinance and - owned by Percival Moday
even during its pendency, the effectivity of the assailed - for the site of Bunawan Farmers Center and Other
ordinance shall not be suspended. Government Sports Facilities
• In the CAB, Municipal Ordinance No. 28 took effect in Oct. • Said resolution was approved by Mayor Bustillo but
1996. Petitioner filed its appeal only in Dec. 1987, more than subsequently disapproved by the Sangguniang Panlalawigan:

The Apocryphal Maggots:


Rainier, Chrisgel, Corina, Geoffry, Grace and Sylvie Blanche
The Flibbertigibbet Worms:
Golda, Gladys and Melyjane
CA VE AT: By simply r ea ding this re vi ew er a t the end o f the sem este r wi ll (hopef ully) guar antee you r pas sing this cou rse. D rink mode rat ely .
Amusin S C A r y
gly
THE C2005 LOCAL GOVERNMENT REVIEWER - 156 -
- expropriation is unnecessary considering that there are still legs and as it flew through the air in dizzy circles it spread its
available lots in Bunawan for the establishment o the wings wide in the last convulsions of death. As though carried
government center through the air by the beating pinions the negress whirled
• The Municipality then filed a petition for eminent domain and forwards in frantic ecstasy. The drums rose to their shattering
its motion to take or enter upon the possession of subject finale, the woman stopped motionless, and then – a miracle –
matter was subsequently granted by the RTC.
the dying cock twisted its neck convulsively and crowed –
crowed loud and raucous into the surrounding night. It was the
Issue: WON a municipality may expropriate private property by
virtue of a municipal resolution which was disapproved by the final touch of horror. (265-271)
Sangguniang Panlalawigan

Held: YES
1. The Municpality of Bunawan’s power to exercise the right of
eminent domain is expressly provided for in BP 337 (LGC) in force Hunting the Elusive G-Spot (Part2)
at the time expropriation proceedings were initiated:
The G-Spot is embryonically analogous to the male prostate. In
Sec. 9. Eminent Domain- A local government unit may, through its
plain English, certain embryonic cells develop one way if the
head and acting pursuant to a resolution of its sanggunian,
exercise the right of eminent domain and institute condemnation child is female and another way if the baby turns out to be male.
proceedings for public use or purpose. In boys, the prostate gland is responsible for the production of
fluid that makes up semen. In girls, the area that would have
2. Section 153 of BP 337 grants the SP the power to declare a been the prostate becomes the G-Spot. The reason that some
municipal resolution invalid on the sole ground that: medical authorities have said that it doesn't exist is because the
- it is beyond the power of the SB or Mayor to issue G-Spot engorges only with stimulation, making it very hard to
- Velazco vs.Blas: If a provincial board passes these limits, it locate in autopsy studies, the primary source for new
usurps the legislative functions of the municipal council or anatomical research.
president
The G Spot cannot possibly live up to all the hype it has
THUS, the SP was: received; it is simply one more pleasant place to stimulate in
- without authority to disapprove the resolution for the
some women. While all women have a G-Spot, not all women
Municipality of Buanwan clearly has the power to exercise the
right of eminent domain and its SB the capacity to promulgate notice anything different when it is stimulated--and some
said resolution women actually dislike the sensation. So don't be discouraged if
you can't find your own G-Spot, or that of a partner. And don't
3. Resolution 43-89 is valid and binding and could be used as be alarmed if you don't like the sensation. Remember, every
lawful authority to petition for the condemnation of petitioners’ body is different!
property (continued…)

Voodoo Fire in Haiti (Part 3):


Ju dicia l I nte rv ent ion
A violent fit of trembling shook the woman. It was as if a demon
lover had taken possession of her and were exploring her limbs Rule 63, Section 4, 1997 Rules of Civil Procedure
to their very extremities. Someone handed her a black cock
which she raised high above her head. The scene was diabolical. SECTION 4. Local government ordinances. — In any action
There stood the naked negress, her sweat-streaked body involving the validity of a local ordinance, the corresponding
glistening in the ruddy firelight, and at arm’s length she held a prosecutor or attorney of the local governmental unit involved
terrified black cock, the very symbol of Satan, squawking and shall be similarly notified and entitled to be heard. If such
flapping its wings, while the feathers flew in all directions. An ordinance is alleged to be unconstitutional, the Solicitor General
awful sickness gripped me. I felt as if I were looking into the shall be notified and entitled to be heard. (4a, R64)
very depths of evil. The thunder of the drums grew to an
avalanche of deafening reverberations.
Perez vs. Dela Cruz (1969)
Slowly the body of the Mamaloi relaxed from its rigid posture.
Facts:
She began to turn; faster—faster—faster. Like a madman she
Vice Mayor Perez (Naga City), who was presiding a private
spun round on her toes. She swung the cock now only by its
conference with 7 city councilors in the matter of selecting the

The Apocryphal Maggots:


Rainier, Chrisgel, Corina, Geoffry, Grace and Sylvie Blanche
The Flibbertigibbet Worms:
Golda, Gladys and Melyjane
CA VE AT: By simply r ea ding this re vi ew er a t the end o f the sem este r wi ll (hopef ully) guar antee you r pas sing this cou rse. D rink mode rat ely .
Amusin S C A r y
gly
THE C2005 LOCAL GOVERNMENT REVIEWER - 157 -
secretary of the municipal board and the chairman of various Homeowner’s Association of the Phils., Inc. vs. Municipal
standing committees of the board, expressed her intention to vote Board of City of Manila (1968)
on these matters – to create a tie vote and thereafter to exercise
her power as presiding officer to break the deadlock. Facts:
Four concilors sought to prevent the Vice Mayor by filing with CFI Homeowner’s Association brought action for declaratory relief to
of Camarines Sur. nullify Manila Municipal Ordinance4841 declaring that a state of
emergency existed in the matter of housing accommodations in
Held: Manila, in view of prevailing scarcity of land and buildings for
1. The Vice Mayor of Naga City as presiding officer of the residential purposes there.
Municipal Board cannot be a member of the same board: Sec1 and 2 declared that lessors and sublessor of land and
In the absence of any statutory authority constituting the vice- buildings primarily devoted to residential purposes cannot increase
mayor as a member of the municipal board, in addition to being the their rentals beyond certain conditions.
presiding officer thereof, it cannot be read into the law something
that is not there. Differences in law beget differences in legal Held:
effects.
The mere fact that the vice-mayor was made the “presiding officer” 1. Assuming that the City had such powers and assuming the
of the board did not ipso jure make him a member thereof; and existence of the emergency, ordinance is illegal and
even if he “is an integral part of the Municipal board” such fact unconstitutional (Note: Court did not decide if the city has
does not necessarily confer on him” either the status of a regular power to declare a state of emergency and if such emergency
member of its municipal board or the powers and attributes of a existed)
municipal councilor. The police power of municipal corporations is subject to
CAB: There is nothing in the Charter of Naga City which provides constitutional limitations. Individual rights may be adversely
that the vice-mayor of said city is a member of the municipal board. affected by the exercise of police power only to the extent that may
He was not even designated as “Acting Mayor” in case of be fairly required by the legitimate demands of public interest or
temporary incapacity of the Mayor. public welfare.
When the demands of public interest are brought about by a state
2. Vice-Mayor is allowed to vote only in case of a tie. He is not of emergency, the interference upon individual rights must be co-
allowed to vote twice (to create a tie and break the deadlock). terminus with the existence of the state of emergency. The statute
CAB: Since there are 7 councilors, a tie is not possible. Vice passed to meet a given emergency, should limit the period of its
Mayor’s vote in case of a tie is no longer necessary. effectivity.
Otherwise, that which was intended to meet a temporary
emergency may become a permanent law  When the cause for
3. Courts with jurisdiction over the case to prohibit Vice Mayor
the grant of power was temporary, so should the grant be, for the
from exercising to vote twice effect cannot remain in existence upon the removal of its cause.
Doctrine in Vera vs. Avelino that prohibition refers only to
proceedings of any tribunal, corporation, board or person not 2. Lower court proceedings are NOT INVALID on the ground
exercising legislative functions is based on the principle of that the Solicitor General had not been heard on the issue of
separation of powers and checks and balances which is not the constitutionality of the ordinance.
applicable to local governments. The case is irrelevant to the issue The issue was not raised in the CFI.
in CAB in addition to the actuality that executives at the local or In any event, this requirement is merely directory and does not
municipal level are vested with both legislative and sometimes affect the jurisdiction of the trial court nor the validity of the
judicial functions, in addition to their purely executive duties. proceedings therein.
By explicit statutory command, courts are given authority to
determine the validity of municipal proceedings. And in CAB, the
petitioner, in insisting to exercise the right to vote twice in the Dis cip lina ry A ct ions
municipal board, acted without jurisdiction and power to do so, and
may be validly prevented and restrained by a writ of prohibition. RA 9165 (2002) – Comprehensive Dangerous Drugs Act Of
In reply to petitioner’s assertion that the acts sought to be 2002
restrained are mere “probable individual actuations” beyond the
reach of a prohibitory writ, suffice it to state that prohibition is Sec27. Criminal Liability of a Public Officer or Employee for
essentially a preventive remedy and is not intended to provide for Misappropriation, Misapplication or Failure to Account for
remedy for acts already accomplished. the Confiscated, Seized and/or Surrendered Dangerous
Petitioner’s threat of voting twice was not an empty or meaningless Drugs, Plant Sources of Dangerous Drugs, Controlled
gesture for record shows that she voted twice for the approval of Precursors and Essential Chemicals, Instruments/
the alleged amendment to the rules of procedure. Paraphernalia and/or Laboratory Equipment Including the
Proceeds or Properties Obtained from the Unlawful Act
Committed. — The penalty of life imprisonment to death and a
fine ranging from Five hundred thousand pesos (P500,000.00)
to Ten million pesos (P10,000,000.00), in addition to absolute

The Apocryphal Maggots:


Rainier, Chrisgel, Corina, Geoffry, Grace and Sylvie Blanche
The Flibbertigibbet Worms:
Golda, Gladys and Melyjane
CA VE AT: By simply r ea ding this re vi ew er a t the end o f the sem este r wi ll (hopef ully) guar antee you r pas sing this cou rse. D rink mode rat ely .
Amusin S C A r y
gly
THE C2005 LOCAL GOVERNMENT REVIEWER - 158 -
perpetual disqualification from any public office, shall be Sec61. Form and Filing of Administrative Complaints. — A
imposed upon any public officer or employee who verified complaint against any erring local elective official shall
misappropriates, misapplies or fails to account for confiscated, be prepared as follows:
seized or surrendered dangerous drugs, plant sources of (a) A complaint against any elective official of a province,
dangerous drugs, controlled precursors and essential a highly urbanized city, an independent component city or
chemicals, instruments/paraphernalia and/or laboratory component city shall be filed before the Office of the President;
equipment including the proceeds or properties obtained from (b) A complaint against any elective official of a
the unlawful acts as provided for in this Act. municipality shall be filed before the sangguniang panlalawigan
Any elective local or national official found to have benefited whose decision may be appealed to the Office of the President;
from the proceeds of the trafficking of dangerous drugs as and
prescribed in this Act, or have received any financial or material (c) A complaint against any elective barangay official
contributions or donations from natural or juridical persons found shall be filed before the sangguniang panlungsod or
guilty of trafficking dangerous drugs as prescribed in this Act, sangguniang bayan concerned whose decision shall be final
shall be removed from office and perpetually disqualified from and executory.
holding any elective or appointive positions in the government,
its divisions, subdivisions, and intermediaries, including Sec62. Notice of hearing. —
government-owned or -controlled corporations. (a) Within seven (7) days after the administrative
complaint is filed, the Office of the President or the sanggunian
Sec28. Criminal Liability of Government Officials and concerned, as the case may be, shall require the respondent to
Employees. — The maximum penalties of the unlawful acts submit his verified answer within fifteen (15) days from receipt
provided for in this Act shall be imposed, in addition to absolute thereof, and commence the investigation of the case within ten
perpetual disqualification from any public office, if those found (10) days after receipt of such answer of the respondent.
guilty of such unlawful acts are government officials and (b) When the respondent is an elective official of a
employees. province or highly urbanized city, such hearing and investigation
shall be conducted in the place where he renders or holds office.
For all other local elective officials, the venue shall be the place
where the sanggunian concerned is located.
(c) However, no investigation shall be held within ninety
Local Government Code (90) days immediately prior to any local election, and no
preventive suspension shall be imposed within the said period. If
Sec58. Enforcement of Disapproved Ordinances or preventive suspension has been imposed prior to the 90-day
Resolutions. — Any attempt to enforce any ordinance or any period immediately preceding local election, it shall be deemed
resolution approving the local development plan and public automatically lifted upon the start of aforesaid period.
investment program, after the disapproval thereof, shall be
sufficient ground for the suspension or dismissal of the official or Sec63. Preventive Suspension. —
employee concerned. (a) Preventive suspension may be imposed:
(1) By the President, if the respondent is an elective official
Sec60. Grounds for Disciplinary Actions. — An elective local of a province, a highly urbanized or an independent
official may be disciplined, suspended, or removed from office component city;
on any of the following grounds: (2) By the governor, if the respondent is an elective official
(a) Disloyalty to the Republic of the Philippines; of a component city or municipality; or
(b) Culpable violation of the Constitution; (3) By the mayor, if the respondent is an elective official of
(c) Dishonesty, oppression, misconduct in office, gross the barangay.
negligence, or dereliction of duty; (b) Preventive suspension may be imposed at any time
(d) Commission of any offense involving moral turpitude after the issues are joined, when the evidence of guilt is strong,
or an offense punishable by at least prision mayor; and given the gravity of the offense, there is great probability
(e) Abuse of authority; that the continuance in office of the respondent could influence
(f) Unauthorized absence for fifteen (15) consecutive the witnesses or pose a threat to the safety and integrity of the
working days, except in the case of members of the records and other evidence: Provided, That, any single
sangguniang panlalawigan, sangguniang panlungsod, preventive suspension of local elective officials shall not extend
sangguniang bayan, and sangguniang barangay; beyond sixty (60) days: Provided, further, That in the event that
(g) Application for, or acquisition of, foreign citizenship or several administrative cases are filed against an elective official,
residence or the status of an immigrant of another country; and he cannot be preventively suspended for more than ninety (90)
(h) Such other grounds as may be provided in this Code days within a single year on the same ground or grounds
and other laws. existing and known at the time of the first suspension.
An elective local official may be removed from office on the (c) Upon expiration of the preventive suspension, the
grounds enumerated above by order of the proper court. suspended elective official shall be deemed reinstated in office
without prejudice to the continuation of the proceedings against

The Apocryphal Maggots:


Rainier, Chrisgel, Corina, Geoffry, Grace and Sylvie Blanche
The Flibbertigibbet Worms:
Golda, Gladys and Melyjane
CA VE AT: By simply r ea ding this re vi ew er a t the end o f the sem este r wi ll (hopef ully) guar antee you r pas sing this cou rse. D rink mode rat ely .
Amusin S C A r y
gly
THE C2005 LOCAL GOVERNMENT REVIEWER - 159 -
him, which shall be terminated within one hundred twenty (120) exoneration, he shall be paid his salary and such other
days from the time he was formally notified of the case against emoluments during the pendency of the appeal.
him. However, if the delay in the proceedings of the case is due
to his fault, neglect, or request, other than the appeal duly filed,
the duration of such delay shall not be counted in computing the Rules and Procedures on the Investigation of
time of termination of the case. Administrative Disciplinary Cases Against Elective Local
(d) Any abuse of the exercise of the power of preventive Officials (AO23 as amended by AO159, and AO66)
suspension shall be penalized as abuse of authority.
Pursuant to Sec60-66 of 1991 LGC, in relation to Rule XIX,
Sec64. Salary of Respondent Pending Suspension. — The Art124-130 of the IRR
respondent official preventively suspended from office shall RULE 1: PRELIMINARY PROVISIONS
receive no salary or compensation during such suspension; but Sec1. Coverage. — These rules and procedures shall apply to
upon subsequent exoneration and reinstatement, he shall be administrative disciplinary cases filed against: (a) the governors,
paid full salary or compensation including such emoluments and members of the sangguniang panlalawigan; (b) the mayors,
accruing during such suspension. vice mayors, and members of the sangguniang panlungsod of
highly urbanized cities, independent component cities, and
Sec65. Rights of Respondent. — The respondent shall be component cities; and (c) the mayors, vice mayors, and
accorded full opportunity to appear and defend himself in person members of the sangguniang panlungsod or bayan of cities or
or by counsel, to confront and cross-examine the witnesses municipalities in Metropolitan Manila.
against him, and to require the attendance of witnesses and the
production of documentary process of subpoena or subpoena Sec2. Disciplinary Authority. — The President, who may act
duces tecum. through the Executive Secretary, shall be the Disciplining
Authority for all administrative complaints against elective local
Sec66. Form and Notice of Decision. — officials covered therein. (As amended by AO66)
(a) The investigation of the case shall be terminated
within ninety (90) days from the start thereof. Within thirty (30) Sec3. Investigating Authority. — The Secretary of the Interior
days after the end of the investigation, the Office of the and Local Government is hereby designated as the Investigating
President or the sanggunian concerned shall render a decision Authority. As such, he is authorized to investigate complaints
in writing stating clearly and distinctly the facts and the reasons filed against elective officials covered herein and act
for such decision. Copies of said decision shall immediately be appropriately thereon as hereinafter provided. He may,
furnished the respondent and all interested parties. thereafter, constitute an Investigating Committee in the
(b) The penalty of suspension shall not exceed the Department of Interior and Local Government for the conduct of
unexpired term of the respondent or a period of six (6) months investigation.
for every administrative offense, nor shall said penalty be a bar The preceding paragraph notwithstanding, the Disciplining
to the candidacy of the respondent so suspended as long as he Authority may, in the interest of the service, constitute a Special
meets the qualifications required for the office. Investigating Committee in lieu of the Secretary of the Interior
(c) The penalty of removal from office as a result of an and Local Government. Nothing herein provided, however, shall
administrative investigation shall be considered a bar to the prevent the President from assuming jurisdiction at any stage of
candidacy of the respondent for any elective position. the proceedings over cases to be preliminarily investigated by
the Department of Interior and Local Government. In such
Sec67. Administrative Appeals. — Decisions in administrative event, the same shall immediately be forwarded to the Special
cases may, within thirty (30) days from receipt thereof, be Investigating Committee after it may have been constituted by
appealed to the following: the Disciplining Authority. (As amended by AO66)
(a) The sangguniang panlalawigan, in the case of
decisions of the sangguniang panlungsod of component cities RULE 2: GROUNDS FOR ADMINISTRATIVE DISCIPLINARY
and the sangguniang bayan; and ACTION
(b) The Office of the President, in the case of decisions of Sec1. Grounds. — An elective local official may be disciplined,
the sangguniang panlalawigan and the sangguniang suspended, or removed from office on any of the following
panlungsod of highly urbanized cities and independent grounds:
component cities. a) Disloyalty to the Republic of the Philippines;
Decisions of the Office of the President shall be final and b) Culpable violation of the Constitution;
executory. c) Dishonesty, oppression, misconduct in office, gross
negligence, or dereliction of duty;
Sec68. Execution Pending Appeal. — An appeal shall not d) Commission of any offense involving moral turpitude
prevent a decision from becoming final or executory. The or any offense punishable by at least prision mayor, which is
respondent shall be considered as having been placed under from six (6) years and one (1) day to twelve (12) years
preventive suspension during the pendency of an appeal in the imprisonment;
event he wins such appeal. In the event the appeal results in an e) Abuse of authority;

The Apocryphal Maggots:


Rainier, Chrisgel, Corina, Geoffry, Grace and Sylvie Blanche
The Flibbertigibbet Worms:
Golda, Gladys and Melyjane
CA VE AT: By simply r ea ding this re vi ew er a t the end o f the sem este r wi ll (hopef ully) guar antee you r pas sing this cou rse. D rink mode rat ely .
Amusin S C A r y
gly
THE C2005 LOCAL GOVERNMENT REVIEWER - 160 -
f) Unauthorized absence for fifteen (15) consecutive Sec2. Form of answer. — The answer, accompanied by affidavits
working days in case of local chief executives and four (4) of witnesses or evidences in support of the defense, shall be
consecutive sessions in the case of members of the addressed to the President and shall be drawn in clear, simple,
sangguniang; and concise language and in methodical manner as to traverse
g) Application for, or acquisition of, foreign citizenship or the charge.
residence of the status of an immigrant of another country; and
h) Such other grounds as may be provided by the Local Sec3. Where filed. — The answer shall be submitted to the
Government Code of 1991; Republic Act No. 6713; Republic Act Secretary of the Interior and Local Government or the Office of
No. 3019; Administrative Code of 1987; Revised Penal Code; the President as the case may be. However, for cases against
and all other applicable general and special laws. elective officials of LGUs concerned outside Metropolitan
Manila, the answer may be submitted through the concerned
RULE 3: COMPLAINT Regional Director of the DILG, who shall transmit the same to
Sec1. How initiated. — An administrative case may be initiated by the Secretary of the Interior and Local Government, within forty-
any private individual or any government officer or employee by eight (48) hours from receipt thereof. In this regard, the Regional
filing a sworn written complaint against any elective local official Director concerned shall authenticate all the pertinent
enumerated under Sec. 1, Rule I hereof. It may also be initiated documents presented to him.
motu proprio by the Office of the President or any government A copy of the answer shall be furnished to each of the following:
agency duly authorized by law to ensure that local government a) The complainant;
units (LGUs) act within their prescribed powers and functions. b) The Office of the Governor in the case of component
cities; and
Sec2. Form of complaint. — The complaint, accompanied by c) The Metropolitan Manila Development Authority in the
affidavits of witness or evidences in support of the charge, shall case of cities and municipalities in Metro Manila. (As amended
be addressed to the President. It shall be drawn in clear, simple, by AO66)
and concise language and in methodical manner as to apprise
the respondent of the nature of the charge against him and to Sec4. Failure to answer. — Unreasonable failure of respondent to
enable him to prepare his defense. The party filing the complaint file his verified answer within fifteen (15) days from receipt of the
shall be called the complainant, while the official against whom complaint against him shall be considered as waiver of his right
the complaint is filed shall be called the respondent. to present evidence in his behalf.

Sec3. Where filed. — The complaint shall be filed with the RULE 5: PRELIMINARY INVESTIGATION
Records Office, Office of the President, Malacañang, Manila. Sec1. Commencement. — Within twenty (20) days from receipt of
However, for cases against elective officials of LGUs concerned the answer, the Investigating Authority shall commence the
outside Metropolitan Manila, the complaint may be filed through investigation of the case. (As amended by AO66)
the concerned Regional Director of the DILG, who shall transmit
the same to the Secretary of the Interior and Local Government, Sec2. Failure to commence preliminary investigation. —
within forty-eight (48) hours from receipt thereof. In this regard, Unreasonable failure to commence the preliminary investigation
the Regional Director concerned shall authenticate all the within the prescribed period by the person or persons assigned
pertinent documents presented to him. to investigate shall be a ground for administrative disciplinary
Upon receipt of the said documents, the Secretary of the Interior action.
and Local Government shall transmit the same to the Office of
the President, within forty-eight (48) hours from receipt of the Sec3. Evaluation. — After the filing of the answer, the
same. Investigating Authority shall, within fifteen (15) days from receipt
A copy of the complaint shall be furnished to each of the thereof, determine the existence of a probable cause.
following: Should probable cause exists, the Investigating Authority shall
a) the Office of the Governor in the case of component set the case for preliminary conference and formal
cities; administrative proceedings shall thereafter be conducted. If
b) the Metropolitan Manila Authority in the case of cities warranted, the investigating Authority may recommend to the
and municipalities in Metropolitan Manila; and Disciplining Authority the imposition of a preventive suspension
c) the DILG in all cases. on the respondent/s.
Sec4. Filing fee. — (repealed by AO66)
Sec4. Dismissal motu propio. — If the Investigating Authority
RULE 4: ANSWER determines that there is no prima facie case to warrant the
Sec1. Notice. — Within seven (7) days after the complaint is filed, institution of formal administrative proceedings, it shall, within
the Investigating Authority or the Disciplining Authority as the the same period prescribed under the preceding Section, submit
case may be shall issue an order requiring the respondent to its recommendation to the Disciplining Authority for the motu
submit his verified answer within fifteen (15) days from receipt proprio dismissal of the case, together with the recommended
thereof. (As amended by AO66) decision, resolution, and order.

The Apocryphal Maggots:


Rainier, Chrisgel, Corina, Geoffry, Grace and Sylvie Blanche
The Flibbertigibbet Worms:
Golda, Gladys and Melyjane
CA VE AT: By simply r ea ding this re vi ew er a t the end o f the sem este r wi ll (hopef ully) guar antee you r pas sing this cou rse. D rink mode rat ely .
Amusin S C A r y
gly
THE C2005 LOCAL GOVERNMENT REVIEWER - 161 -
Sec5. Preliminary conference. — If the Investigating Authority
determines that there is prima facie case to warrant the Sec3. Grounds. — Preventive suspension may be imposed at any
institution of formal administrative proceedings, it shall, within time after the issues are joined, that is, after respondent has
the same period prescribed under the preceding section, answered the complaint, when the evidence of guilt is strong
summon the parties to a preliminary conference to consider the and, given the gravity of the offense, there is a great probability
following: that the continuance in office of the respondent could influence
a) Whether the parties desire a formal investigation or the witnesses or pose a threat to the safety and integrity of the
are willing to submit the case for resolution on the basis of the records and other evidence.
evidence on record; and
b) If the parties desire a formal investigation, to consider Sec4. Duration. — Any single preventive suspension of local
the simplification of issues, the possibility of obtaining stipulation elective officials shall not extend beyond sixty (60) days;
or admission of facts and of documents, specifically affidavits provided that, in the event that several administrative cases are
and depositions, to avoid unnecessary proof, the limitation of filed against an elective official, he cannot be preventively
number of witnesses, and such other matters as may aid the suspended for more than ninety (90) days within a single year
prompt disposition of the case. on the same ground or grounds existing and known at the time
The Investigating Authority shall encourage the parties and their of the first suspension.
counsels to enter, at any stage of the proceedings, into amicable
settlement, compromise and arbitration, the terms and Sec5. Automatic reinstatement. — Upon expiration of the
conditions of which shall be subject to the approval of the preventive suspension, the suspended elective official shall be
Disciplining Authority. deemed reinstated in office without prejudice to the continuation
After the preliminary conference, the Investigating Authority shall of the proceedings against him, which shall be terminated within
issue and Order reciting the matters taken up thereon, including one hundred twenty (120) days from the time he was formally
the facts stipulated and the evidences marked, if any. Such notified of the case against him. However, if the delay in the
order shall limit the issues for hearing to those not disposed of proceeding of the case is due to his fault, or request, other than
by agreement or admission of the parties, and shall schedule the appeal duly filed, the duration of such delay shall not be
the formal investigation within ten (10) days from its issuance, counted in computing the time of termination of the case.
unless a later date is mutually agreed in writing by the parties
concerned. Sec6. Salary of respondent pending suspension. — The
respondent, who is preventively suspended from office, shall
Sec6. Venue of hearing. — When the respondent is an elective receive no salary or compensation during such suspension; but,
official of a province or highly urbanized city, the preliminary upon subsequent exoneration and reinstatement, he shall be
investigation as contemplated in this Rule shall be conducted in paid his full salary or compensation, including such emoluments
the place where he renders or holds office. For all other local accruing during such suspension.
elective officials, the venue shall be the place where the
sanggunian concerned is located. RULE 7: FORMAL INVESTIGATION
Sec1. Procedural due process. — The respondent shall be
Sec7. 90-day ban. — No preliminary investigation shall be accorded full opportunity to appear and defend himself in person
conducted within ninety (90) days immediately prior to any local or by counsel, to confront and cross-examine the witnesses
election. (As amended by AO159) against him, and to require the attendance of witnesses and the
production of documents through the compulsory process of
RULE 6: PREVENTIVE SUSPENSION subpoena or subpoena duces tecum.
Sec1. Power to suspend. — Preventive suspension may be
imposed by the Disciplining Authority in cases where the Sec2. Who conducts the hearing. — The formal administrative
respondent is an elective official of the following LGUs: investigation shall be conducted by the Investigating Authority.
a) provinces;
b) highly urbanized cities; Sec3. Failure to commence formal investigation. —
c) independent component cities; and Unreasonable failure to commence the formal investigation
d) cities and municipalities in Metropolitan Manila. within the prescribed period in the preliminary conference order
The governor shall, upon the direct order of the Disciplining by the person or persons assigned to investigate shall be
Authority, preventively suspend an elective official of a aground for administrative disciplinary action.
component city, who is under formal administrative investigation
by the Office of the President. Sec4. Power to take testimony or receive evidence. — The
Investigating Authority is hereby authorized to take testimony or
Sec2. 90-day ban. — No preventive suspension shall be imposed receive evidence relevant to the administrative proceedings,
within ninety (90) days immediately prior to any local election. If which authority shall include the power to administer oaths,
the preventive suspension has been imposed prior to the 90-day summon witnesses, and require the production of documents by
period immediately preceding a local election, it shall be subpoena duces tecum pursuant to Book 1, Chapter 9, Section
deemed automatically lifted upon the start of aforesaid period. 37 of the Administrative Code of 1987.

The Apocryphal Maggots:


Rainier, Chrisgel, Corina, Geoffry, Grace and Sylvie Blanche
The Flibbertigibbet Worms:
Golda, Gladys and Melyjane
CA VE AT: By simply r ea ding this re vi ew er a t the end o f the sem este r wi ll (hopef ully) guar antee you r pas sing this cou rse. D rink mode rat ely .
Amusin S C A r y
gly
THE C2005 LOCAL GOVERNMENT REVIEWER - 162 -
Anyone who, without lawful excuse, fails to appear upon d) Re-cross examination by the opponent.
summons issued under authority of the preceding paragraph or
who, appearing before the Investigating Authority exercising the Sec12. Termination of formal investigation. — The formal
power therein defined, refuses to make oath, give testimony or investigation of the case shall be terminated by the Investigating
produce documents for inspection, when lawfully required, shall Authority within ninety (90) days from the start thereof.
be subject to discipline as in case of contempt of court and, Unreasonable failure to complete the formal investigation after
upon application by the Investigating Authority, shall be dealt the said period by the person or persons assigned to investigate
with by the judge of the proper regional trial court in the manner shall be a ground for disciplinary action.
provided for under Book VII, Chapter 3, Section 13, in relation to
Chapter 1, Section 2 (1), of the Administrative Code of 1987. Sec13. Memoranda. — The Investigating Authority may allow the
parties to submit their respective memoranda, together with their
Sec5. Notice of hearing. — The parties and their witnesses shall respective draft resolutions and orders for consideration of the
be notified by subpoena of the scheduled hearing at least five Investigating Authority, within fifteen (15) days after the
(5) days before the date thereof, stating the date, time and place termination of the formal investigation.
of the hearing.
Sec. 14. 90-day ban. — No formal investigation shall be
Sec6. Venue of hearing. — When the respondent is an elective conducted within ninety (90) days immediately prior to any local
official of a province or highly urbanized city, the formal election. (As added by AO159)
investigation as contemplated in this Rule shall be conducted in
the place where he renders or holds office. For all other local RULE 8: EVIDENCE
elective officials, the venue shall be the place where the Sec1. Rules of evidence. — In administrative disciplinary
sanggunian concerned is located. proceedings —
a) The Investigating Authority may admit and give
Sec7. Request for subpoena. — If a party desires the probative value to evidence commonly accepted by reasonably
attendance of a witness or the production of documents, he prudent men in the conduct of their affairs;
should make formally request for the issuance of the necessary b) Documentary evidence may be received in the form of
subpoena or subpoena duces tecum at least three (3) days copies or excepts, if the original is not readily available. Upon
before the scheduled hearing. request, the parties shall be given opportunity to compare the
copy with the original. If the original is in the official custody of a
Sec8. Postponement. — Postponement of investigation shall be public officer, a certified copy thereof may be accepted; and
discouraged and shall be allowed only in meritorious cases, like c) The Investigating Authority may take notice of
illness of the parties or counsels and other similar case. No judicially cognizable facts and of generally technical or scientific
postponement for a period longer than seven (7) days shall be facts within it specialized knowledge. The parties shall be
allowed, and in no case shall the total number of postponements notified and afforded an opportunity to contest the facts so
for one party be more than twenty (20) days. noticed.

Sec9. Stenographic record of proceedings. — The testimony of Sec2. Marking. — All documentary evidence or exhibits shall be
each witness and the manifestation of the parties and counsels properly marked by letter (A, B, C, etc.), if presented by the
during an investigation shall be taken in shorthand or stenotype. complainants, and by numbers (1, 2, 3, etc.), if presented by the
A transcript of the proceedings made by the official stenographer respondent. They shall be attached to the records or, if
or stenotypist and duly certified by him shall be prima facie a voluminous, kept in a separate folder marked "Folder of
correct statement of such proceedings. Exhibits", which shall also be attached to the records.

Sec10. Order of hearing. — Unless otherwise directed by the RULE 9: REPORT OF INVESTIGATING AUTHORITY AND
Investigating Authority, the order of a hearing shall be as follows: TRANSMISSION OF RECORDS
a) The complaint shall produce the evidence on his part; Sec1. Transmission of Records to Disciplining Authority. —
b) The respondent shall then offer evidence in support of The Investigating Authority shall forward to the Disciplining
his defense; and Authority its findings and recommendations, together with the
c) The parties may then respectively offer rebutting following:
evidence, unless the Investigating Authority, for good reasons a) the draft decision, resolution and order;
and in the furtherance of justice, permits them to offer evidence b) the complete records with each page consecutively
upon their original case. numbered and initialed by the custodian of the records;
c) a summary of proceedings thereon from the filing of
Sec11. Order of Examination. — The order in which a witness the complaint to the transmittal of the records in chronological
may be examined shall be as follows: order indicating the action taken on the incidents involved; and
a) Direct examination by the proponent; d) a list of all pleadings, motions, manifestations,
b) Cross examination by the opponent; annexes, exhibits, and other papers or documents filed by the
c) Re-direct examination by the proponent; and

The Apocryphal Maggots:


Rainier, Chrisgel, Corina, Geoffry, Grace and Sylvie Blanche
The Flibbertigibbet Worms:
Golda, Gladys and Melyjane
CA VE AT: By simply r ea ding this re vi ew er a t the end o f the sem este r wi ll (hopef ully) guar antee you r pas sing this cou rse. D rink mode rat ely .
Amusin S C A r y
gly
THE C2005 LOCAL GOVERNMENT REVIEWER - 163 -
contending parties, as well as the corresponding orders or months for every administrative offense, nor shall said penalty
resolutions. be a bar to the candidacy of the respondent so suspended as
Such documents shall be forwarded to the Disciplining Authority long as he meets the qualifications required for the office.
within twenty (20) days — When the respondent has been meted two (2) or more penalties
a) from receipt of the last pleading and evidence, if any, of suspension for two (2) or more administrative offenses, such
in case the respondent does not elect a formal investigation; penalties shall be served successively. (As amended by AO159)
b) after the expiration of the period within which to
submit the same; or after the termination of the formal Sec3. Removal. — An elective local official may be removed from
investigation; or office on the grounds enumerated in Rule 2 hereof by order of
c) after the parties have submitted their respective the proper court or the Disciplining Authority whichever first
Memoranda if so allowed. acquires jurisdiction to the exclusion of the other.
The transcript of the proceedings shall be paged consecutively The penalty or removal from office as a result of an
and in chronological order, sewed on the left-hand side, and administrative investigation shall be considered a bar to the
properly indexed, showing the page on which the testimony of candidacy of the respondent for any elective position.
each witness begins.
RULE 12: EXECUTIVE CLEMENCY
Sec2. Records classification. — Records in administrative Sec1. Removal of administrative penalties or disabilities. — In
disciplinary cases are classified as confidential in nature and meritorious cases, the President may, after his decision has
any information as to the charges, accusation, or facts adduced become final and executory, commute or remove administrative
may not be released, and such records may not be available, penalties and disabilities imposed upon elective local officials in
except to the proper authorities and, upon request, to the administrative disciplinary cases, subject to such terms and
parties-in-interest or their authorized representatives on the conditions as he may imposed in the interest of the service.
"need-to-know" basis pursuant to Memorandum Circular No. 78
dated August 14, 1964, as amended by Memorandum Circular RULE 13: MISCELLANEOUS PROVISIONS
No. 196 dated July 19, 1968, prescribing rules governing Sec1. Effects and application of relevant laws. — xxx In all
security of classified matter in government offices. matters not provided in this Administrative Order, the Rules of
Court and the 1987 Administrative Code shall apply in a
RULE 10: DECISION suppletory character.
Sec1. Rendition of decision. — Within thirty (30) days after
receipt of the report of the Investigating Authority and the
transmittal of records, the Disciplining Authority shall render a
decision in writing stating clearly and distinctly the facts and Espiritu vs. Melgar (1992)
reasons for such decision. Copies of said decision shall
immediately be furnished the respondent and all interested Facts:
parties. Three similar complaints were filed with the DILG, Office of the
Provincial Governor, and Office of the President, accusing Mayor
Sec2. Finality of decision. — The decision of the Disciplining Melgar of physically assaulting and arresting him without any
Authority shall immediately be final and executory upon receipt reason.
of a copy thereof by the complainant or the respondent, as the Sangguniang Panlalawigan, after evaluation, passed a resolution
case may be. (As amended by AO159) recommending the Provincial Governor to preventively suspend
him pending the administrative case so Mayor Melgar was
Sec3. Motion for reconsideration. — A motion for suspended by Governor Espiritu.
reconsideration shall not stay the execution of a decision. In the Melgar filed petition with RTC which enjoined the Governor from
event that a decision is reconsidered as to result in an implementing the order of suspension.
exoneration, the respondent shall be paid his salary and such
other emoluments accruing during the period of his suspension Held: RTC has no jurisdiction to enjoin the governor from
of removal. (As amended by AO159) preventively suspending the mayor.
Clearly, under Sec63 of the (old) LGC, the provincial governor of
RULE 11: PENALTIES Oriental Mindoro is authorized by law to preventively suspend the
Sec1. Suspension or removal. — A respondent found guilty of municipal mayor of Naujan at anytime after the issues had been
any of the offenses enumerated in Rule 2 hereof may be meted joined and any of the following grounds were shown to exist:
the penalty of suspension or removal depending on the 1. When there is reasonable ground to believe that
evidence presented and the aggravating or mitigating the respondent has committed the act or acts complained of;
circumstances that may be considered by the Disciplining 2. When the evidence of culpability is strong;
Authority. 3. When the gravity of the offense so warrants; or
4. When the continuance in office of the respondent
Sec2. Suspension. — The penalty of suspension shall not exceed could influence the witnesses or pose a threat to the safety
the unexpired term of the respondent, or a period of six (6) and integrity of the records and other evidence.

The Apocryphal Maggots:


Rainier, Chrisgel, Corina, Geoffry, Grace and Sylvie Blanche
The Flibbertigibbet Worms:
Golda, Gladys and Melyjane
CA VE AT: By simply r ea ding this re vi ew er a t the end o f the sem este r wi ll (hopef ully) guar antee you r pas sing this cou rse. D rink mode rat ely .
Amusin S C A r y
gly
THE C2005 LOCAL GOVERNMENT REVIEWER - 164 -
Ratio of preventive suspension: so that the respondent may not Sec5 of AO23 provides that at the preliminary conference, the
hamper the normal course of the investigation through the use of Investigating Authority shall summon the parties to consider
his influence and authority over possible witnesses (Lacson vs. whether they desire a formal investigation. DILG has no discretion
Roque, 92 Phil. 456). to determine whether a formal investigation would be conducted.
There is nothing in the LGC, its IRR, nor in AO23 that provide that
Also, Melgar failed to exhaust administrative remedies: Since administrative cases can be decided on the basis of position
respondent mayor believed that his preventive suspension was papers. Respective memoranda may be required ONLY AFTER
unjustified and politically motivated, he should have sought relief the formal investigation and hearing has been held.
first from the Secretary of Interior and Local Government, not from
the courts.
Berces vs. Guingona (1995)
However, once the 60-day preventive suspension has been
served, the official is deemed reinstated in office without prejudice Facts:
to the continuation of the administrative investigation of the After the filing of 2 administrative cases against Mayor Corral of
charges against him. Tiwi, Sangguniang Panlalawigan ordered:
In the first case – suspension for 2months, and payment to Berces
In the second case – suspension for 3months and reimbursement
Joson vs. Executive Secretary (1998) Upon appeal of the decisions by Corral, Office of the President
stayed the execution of the decisions based on LGC and AO18.
Facts: Berces sought reconsideration.
It was alleged in the complaint of the vice governor and members
of the Sangguniang Panlalawigan that Governor Joson, with some Held: The Office of the President has authority to stay the
armed men, barged into the session hall of the SP, angrily execution of decisions made by Sangguniang Panlalawigan.
kicking the chairs and tables and threatening the members The LGC in its repealing clause Sec530(f) has not expressly
present – alleged done to harass the SP into approving a pending repealed Sec6 of AO18 because it failed to identify or designate
legislative measure obtaining a P150 million from the PNB. the laws or executive orders that are intended to be repealed.
DILG found Joson guilty and ordered 6 months suspension on the In the absence of an express repeal, a subsequent law cannot be
ground that the affidavits of respondent complainants were more construed as repealing a prior law unless an irreconcilable
natural, reasonable and probable than those of Joson's. inconsistency and repugnancy exists in the terms of the new and
old laws.
Held: Governor Joson was not validly suspended. CAB: Sec 68 of LGCode and Sec. 6 AO No. 18 are not
Verification requirement in the complaint and the joint affidavit is irreconcilably inconsistent and repugnant and the two laws must in
only a formal and not a jurisdictional requisite which is required fact be read together.
only to insure good faith in making the allegations. The court may The 1st sentence of Sec 68 merely provides that an "appeal shall
order correction of the pleading if unverified or act upon the not prevent a decision from becoming final or executory." As
unverified pleading if attendant circumstances are such that worded, there is room to construe said provision as giving
dispenses the requirement. CAB: Verification requirement was discretion to the reviewing officials to stay the execution of the
waived when the president himself acted on the complaint. appealed decision.
The execution of decisions pending appeal is procedural and in the
The power to discipline an elective official is lodged in the absence of a clear legislative intent to remove from the reviewing
President (AO 23, Sec2) which includes the power to investigate. officials the authority to order a stay of execution, such authority
AO23 (Sec3) however delegates the power to investigate to the can be provided in the rules and regulations governing the appeals
DILG. of elective officials in administrative cases.
Also, such power of the DILG to investigate administrative
complaints is based on the alter-ego principle and the doctrine of The term "shall" in Sec68, LGC may be read either as mandatory
qualified political agency. or directory depending upon a consideration of the entire provision
in which it is found, its object and the consequences that would
Non-compliance with AO23 in that the Office of the President follow from construing it one way or the other.
should have first required Joson to file his answer then refer the CAB: There is no basis to justify the construction of the word as
complaint and answer to the Investigative Authority (DILG) for mandatory. Office of the President found that the execution of the
further proceedings is not a fatal defect, being merely procedural. decision of the Sangguniang Panlalawigan suspending Mayor
The answer is required only for the President to make a Corral from office might be prejudicial to the public interest. Thus,
preliminary assessment of the case, and he found the complaint in in order not to disrupt the rendition of service by the mayor to the
CAB already sufficient in form and substance. public, a stay of the execution of the decision is in order.

HOWEVER, the 6month suspension is not valid because of the


denial of Joson’s Motion to Conduct Formal Investigation which
denied him procedural due process. Ganzon vs. CA (1991)

The Apocryphal Maggots:


Rainier, Chrisgel, Corina, Geoffry, Grace and Sylvie Blanche
The Flibbertigibbet Worms:
Golda, Gladys and Melyjane
CA VE AT: By simply r ea ding this re vi ew er a t the end o f the sem este r wi ll (hopef ully) guar antee you r pas sing this cou rse. D rink mode rat ely .
Amusin S C A r y
gly
THE C2005 LOCAL GOVERNMENT REVIEWER - 165 -
However, such rule finds no application to criminal cases pending
Facts: against petitioner for acts he may have committed during the failed
DLG Secretary issued against Mayor Ganzon 3 separate orders of coup.
60-day preventive suspensions dated Aug11 1988, Oct11 1988,
and May3 1990. A fourth order was issued on July3, 1991. 2. The 1987 Constitution has not repealed the power of DLG
On August 5, 1991, SC issued order that the first 3 suspensions Sec to suspend or remove local government officials as alter
are affirmed provided that Ganzon may not be made to serve ego of the President.
future suspensions on account of any of the remaining admin The Sec’s power to remove is anchored on the Constitution and a
charges against him. statutory grant from the legislature:
Ganzon filed a petition for mandamus with "manifestation and - Constitutional basis: in Articles VII (17) and X (4) – which vest
compliance," alleging that he had already fully served the in the President the power of control over all executive
suspension orders issued against him, in compliance with the departments, bureaus and offices and the power of general
August 5 SC Order, and that he should be allowed to re-assume supervision over local governments, and by the doctrine that
his office starting September 4 1991. the acts of the department head are presumptively the acts of
the President unless expressly rejected by him
Held: Simultaneous service of the 3rd and 4th orders of suspension - The statutory grant in BP337 has constitutional roots - having
can be allowed been enacted by the Batas Pambansa pursuant to Art XI,
Under the bizarre circumstances of CAB, It would work in favor of Sec2 of 1973 Constitution which states that the National
Ganzon, an elective official, and it presumably will favor the local Assembly shall... provide for the qualifications, election and
constituency and certainly lessen if not offset the harsh effects of removal... of local government officials
whatever motive may be behind the intriguing action of DLG Sec in - a similar provision is found in Art X, Sec3 of 1987 Constitution
issuing the successive suspension orders especially when he that the Congress shall... provide for the qualifications,
could have pursued a consolidated effort. election, appointment, and removal... and all other matters
SC takes judicial notice of recently-approved LGC which provides, relating to the organization and
under Sec63 as to imposition of preventive suspensions, that operation of the local units.
Sec63 (b). . . that, any single preventive suspension of local Inasmuch as the power and authority of the legislature to enact a
elective official shall not extend beyond sixty (60) days: local government code, which provides for the manner of removal
Provided, further that in the event that several administrative of local government officials, is found in the 1973 and 1987
cases are filed against an elective official, he cannot be Constitutions, then it cannot be said that BP337 was repealed by
preventively suspended for more than ninety (90) days within a the effectivity of the present Consitution.
single year on the same ground or grounds existing and known
at the time of the first suspension." Moreover, the Court had stated in the case Bagabuyo vs. Davide
that BP337 remained in force despite the effectivity of the present
Constitution, until such time as the proposed 1991 LGC is
Aguinaldo vs. Santos (1992) approved. The power of DLG Sec to remove local elective
government officials is found in Sec60 and 61 of BP337.
Facts:
Governor Aguinaldo of Cagayan province was found guilty by DLG
Sec for disloyalty to the Republic and culpable violation of the People vs. Toledano (2000)
Constitution and was ordered removed from office.
Pending Aguinaldo’s petition in SC, he filed his certificate of Facts:
candidacy for position of Governor for the 1992 elections. Ombudsman dismissed administrative cases against Bunao
COMELEC disqualified Aguinaldo. (member of Sangguniang Bayan), but recommended the
SC annulled COMELEC Resolution disqualifying Aguinaldo on the prosecution of Bunao under Sec41, par(1) in relation to Sec221 of
ground that DLG Sec. decision has not yet attained finality and is 1983 LGC (Officials not to engage in Business Transactions or
still pending review with SC. Possess Pecuniary Interest).
Aguinaldo won in the 1992 elections as Governor. An information was filed against Bunao before the RTC.
Bunao moved to dismiss the information on the ground that the
Held: charge had already become moot and academic and that any
1. Aguinaldo’s re-election to Governor position has rendered the criminal liability he may have incurred has been extinguished
administrative case pending before SC moot and academic. (Administrative charges had been dismissed; Bunao was re-
RULE: A public official can not be removed for administrative elected).
misconduct committed during a prior term, since his re-election to
office operates as a condonation of the officer's previous Held: It was not proper for judge to issue order of dismissal on the
misconduct to the extent of cutting off the right to remove him ground of extinction of criminal liability. There is Grave Abuse of
therefor. It is assumed that the electorate did this with knowledge Discretion amounting to lack of jurisdiction.
of his life and character.

The Apocryphal Maggots:


Rainier, Chrisgel, Corina, Geoffry, Grace and Sylvie Blanche
The Flibbertigibbet Worms:
Golda, Gladys and Melyjane
CA VE AT: By simply r ea ding this re vi ew er a t the end o f the sem este r wi ll (hopef ully) guar antee you r pas sing this cou rse. D rink mode rat ely .
Amusin S C A r y
gly
THE C2005 LOCAL GOVERNMENT REVIEWER - 166 -
It is a fundamental principle of administrative law that from the service; or (c) the respondent’s continued stay in
administrative cases are independent from criminal actions for the office may prejudice the case filed against him.
same act or omission. xxx
The reliance made by respondent judge on the re-election of  “Under his authority” has been previously interpreted to mean
private respondent as Kagawad in the May 1992 election so as to that the Ombudsman can preventively suspend all officials under
warrant the dismissal of the information filed against him, citing investigation by his office, regardless of the branch of government
Aguinaldo vs. Santos is misplaced. The ruling in said case which in which they are employed, except those removable by
forbids the removal from office of a public official for administrative impeachment, members of Congress and the Judiciary.
misconduct committed during a prior term, finds no application to  This power is also available to the Deputy Ombudsman.
criminal cases, pending against said public officer. CAB: There can be no question as to the power and authority of
1991 LGC which repealed 1983 LGC: (1) reenacted in its Sec89 respondent Deputy Ombudsman Mojica to issue an order of
the legal provision of Sec41 of BP337 under which private preventive suspension against an official like Garcia – to prevent
respondent Bunao was charged and (2) penalizes the same act that official from using his office to intimidate or influence
previously penalized under the repealed law, such that the act witnesses or to tamper with records that might be vital to the
committed before the reenactment continuous to be a crime. prosecution of the case against him.
Hence, prosecution will proceed under the provisions of Section 89
in relation to Section 514 of R.A.7160. RE: Period for which an official may be preventively
suspended:
- must not exceed 6months under Sec24, RA6770
Garcia vs. Mojica (1999) CAB: Garcia was preventively suspended and ordered to cease
and desist from holding office for the entire period of 6months,
Facts: which is the maximum provided by law.
Garcia, as Cebu City mayor, signed a contract with F.E.Zuellig for
supply of asphalt to the city. Contract covers the period 1998-2001 RE: Determination of W/N the evidence of guilt is strong as to
which was to commence on September1998 when the first delivery warrant preventive suspension, and the period of such
should have been made by Zuellig. suspension:
March1999, news reports came out regarding alleged anomalous - rests and necessarily belongs to the Ombudsman, except that
purchase of asphalt by Cebu City, through the contract signed by he cannot extend the period of suspension beyond that
Garcia – which prompted Office of Ombudsman (Visayas) to provided by law.
conduct an inquiry. SC’s view: The strength of the evidence to warrant said
Preventive suspension order was issued by the Office of the suspension and the propriety of the length or period of suspension
Ombudsman. imposed on Garcia are properly raised in the petition under Rule65
Garcia assails validity of the order. where there may be grave abuse of discretion.

RE: Ombudsman's authority to conduct administrative 1. W/N the evidence is sufficiently strong to justify the
investigations: imposition of preventive suspension against Garcia.
- mandated by the Constitution under Art XI, Sec13(1) Held: Findings sufficiently justify it.
- RA6770 (Ombudsman Law) grants the statutory power to - Pertinent to note that the inquiry that preceded the filing of
conduct administrative investigations administrative case vs. Garcia was prompted by new reports
- Sec19 - to act on administrative complaints regarding the allegedly anomalous contract entered into by
- Sec21 - which names the officials subject to Garcia, on behalf of Cebu City, with Zuellig.
Ombudsman's disciplinary authority, among which are - In the memo to Mojica, Graft Investig. Off. Garciano
elective officials of the government recommended that Garcia be preventively suspended, based
CAB: Garcia is an elective official accused of grave misconduct on an initial investigation purportedly showing:
and dishonesty – it is clear from the law that the Office of the o Contract was designed to favor Zuellig
Ombudsman may conduct an administrative investigation into the o Amount quoted in contract was too expensive compared
acts complained of. to amount for which asphalt may be bought from local
suppliers like Shell and Petron – specially considering
RE: Ombudsman's authority to preventively suspend an
that it was fixed in dollars and payable in pesos 
official subject to its administrative investigation:
exposing city government to risks attendant to a
- Sec24, RA6770 provides for it –
fluctuating exchange rate
The Ombudsman or his Deputy may preventively suspend
o Interest of city under the contract is not protected by
any officer or employee under his authority pending an
adequate security
investigation, if in his judgment the evidence of guilt is strong,
- Findings were based on the contract, letters from Bitumex
and (a) the charge against such officer or employee involves
and Credit Lyonnais, letters from Shell and Petron on whether
dishonesty, oppression or grave misconduct or neglect in the
they can supply asphalt to Cebu City…
performance of duty; (b) the charges would warrant removal

The Apocryphal Maggots:


Rainier, Chrisgel, Corina, Geoffry, Grace and Sylvie Blanche
The Flibbertigibbet Worms:
Golda, Gladys and Melyjane
CA VE AT: By simply r ea ding this re vi ew er a t the end o f the sem este r wi ll (hopef ully) guar antee you r pas sing this cou rse. D rink mode rat ely .
Amusin S C A r y
gly
THE C2005 LOCAL GOVERNMENT REVIEWER - 167 -
2. W/N the imposition of the maximum period of 6 months is 2. Hagad vs. Gozo-Dadole has said that nothing in LGC indicate
warranted by the purpose and the circumstances in CAB
that it has repealed (express or implied) the pertinent
Held: NO.
provisions of Ombudsman Act – not being inconsistent.
SolGen’s statement for respondents during his oral argument show
Although it has been argued that 6month period under
that the evidence (documents, vouchers, purchase orders…) to
Ombudsman Law is much too repugnant to 60day period
prove guilt were obtained after Garcia has been suspended, and
under LGC, Vitug said that the two govern differently.
he claimed (even as an afterthought) that they strengthen the
evidence against Garcia.
Garcia still contends: The Hagad case did not settle the question
 If the purpose of the preventive suspension was to enable the of whether a local elective official may be preventively suspended
investigating authority to gather documents without intervention even before the issues could be joined. And he was not given a
from Garcia, then, from respondents’ submission – it can be chance to refute first the charges against him.
concluded that this purpose was already achieved during the Note: Under the LGC, preventive suspension may only be imposed
nearly month-long suspension of Garcia from June25-July19, after issues are joined.
1999. Held: It has been held in other cases that there could be
Granting that now the evidence vs. Garcia is already strong, even preventive suspension even before the charges against the official
without conceding that initially it was weak – it is clear that the are heard, or before the official is given an opportunity to prove his
maximum 6-month period is excessive and definitely longer than innocence.
necessary for Ombudsman to make its legitimate case vs. Garcia. Preventive suspension is merely a preliminary step in an
Conclusion: the period during which Garcia was already administrative investigation and is not in any way the final
preventively suspended, has been sufficient for the lawful purpose determination of the guilt of the official concerned.
of preventing him from hiding and destroying needed documents,
or harassing and preventing witnesses who wish to appear against Garcia continues on: Suspension order against him was issued
him. in violation of Sec26(2) of Ombudsman Law – that before an
inquiry may be converted into a full-blown administrative
DanGat: We’re upset – courts can overrule the discretion given investigation, official must be given 72hours to answer the charges
to the Disciplining Authority as regards sufficiency of evidence against him.  It was converted without him being given the
and the period of preventive suspension required number of hours to answer.
Held: Nope.
Conclusion reached without necessarily drawing a distinction
between the preventive suspension imposed by Ombudsman Law
1. Even though it appears that he was not given the requisite
72hours to submit a written answer to complaint against him,
and the Local Government Code.
this does not make invalid the preventive suspension order
Garcia would claim: LGC should apply to case of an elective local
issued against him. Again, It may be issued even before the
official.
charges against the official concerned is heard.
Note: Under LGC, preventive suspension may only be imposed
2. Moreover, respondents state that Garcia was given 10days to
only for a maximum period of 60days.
submit his counter-affidavit to the complaint filed by Tagaan.
Respondents’ argument: Administrative complaints commenced
This is in keeping with Sec5(a) of Rules of Procedure of
under Ombudsman Law are distinct from those initiated under
Office of the Ombudsman.
LGC.
- The shorter period (60days) of suspension under LGC is
Granting that Ombudsman Office may investigate, for purposes
intended to limit the period of suspension that may be
provided for by law, acts of Garcia committed prior to his present
imposed by a mayor, governor, or President, who may be
term of office; and that it may preventively suspend him for a
motivated by partisan political considerations
reasonable period, Can that office hold him administratively liable
- The Ombudsman, who can impose a longer period (6
for said acts?
months) is not likely to be similarly motivated because it is a
SC Note: It has repeatedly held in a number of cases that a
constitutional body.
reelected local official may not be held administratively
Held: Distinction between LGC and Ombudsman Law is valid but
accountable for misconduct committed during his prior term of
not decisive of whether there has been grave abuse of discretion in
office.
a specific case of preventive suspension.
1. Respondents may be correct in pointing out the reason for the  Rationale: When the electorate put him back into office, it is
shorter period imposable under the LGC (as based on the presumed that it did so with full knowledge of his life and character,
deliberations of the Senate on the Bill). Political color could including his past misconduct. If, armed with such knowledge, it
taint the exercise of the power to suspend local officials by the still reelects him, then such reelection is considered a condonation
mayor, governor or President’s office. In contrast, the of his past misdeeds.
Ombudsman, considering the constitutional origin of his Respondents argue however: In CAB, contract entered into by
Office, always ought to be insulated from the vagaries of Garcia with Zuellig was signed just 4days before date of elections
politics, as respondents would have the SC believe.  it was not made an issue during the election, and so the
electorate could not be said to have voted for Garcia with
knowledge of this particular aspect of his life and character.

The Apocryphal Maggots:


Rainier, Chrisgel, Corina, Geoffry, Grace and Sylvie Blanche
The Flibbertigibbet Worms:
Golda, Gladys and Melyjane
CA VE AT: By simply r ea ding this re vi ew er a t the end o f the sem este r wi ll (hopef ully) guar antee you r pas sing this cou rse. D rink mode rat ely .
Amusin S C A r y
gly
THE C2005 LOCAL GOVERNMENT REVIEWER - 168 -
Garcia contends: “The only conclusive determining factor” as payments were made even if the items were not yet delivered.
regards the people’s thinking on the matter is an election. State Auditors Cabreros and Quejada thereafter submitted a joint
SC held: Garcia is correct. affidavit and a supplemental joint affidavit wherein they disclosed
That the people voted for an official with knowledge of his other details such as the alleged ghost deliveries of asphalt.
character is presumed, precisely to eliminate the need to
determine, in factual terms, the extent of this knowledge. Such an Held:
undertaking is impossible. The COA Special Audit Report and the supporting affidavits
Rulings on the matter do not distinguish the precise timing or submitted by State Auditors constitute a valid complaint with the
period when the misconduct was committed, reckoned from the Office of the Ombudsman.
date of the official’s reelection, except that it must be prior to said For purposes of initiating a preliminary investigation before the
date. Office of the Ombudsman, a complaint "in any form or manner" is
Salalima case applies – that sound policy dictates it. To rule sufficient (Sec. 12 Art XI of Const.). In Almonte v. Vasquez, SC
otherwise would open the floodgates to exacerbating endless held that even unverified and anonymous letters may suffice to
partisan contests between the reelected official and his political start an investigation.
enemies, who may not stop to hound the former during his new In permitting the filing of complaints "in any form or manner," the
term with administrative cases for acts alleged to have been framers of the Constitution took into account the well-known
committed during his previous term. reticence of the people which keep them from complaining against
In CAB, Garcia cannot anymore be held administrative liable for an official wrongdoings.
act done during his previous term (his signing of contract with The Office of the Ombudsman is different from the other
Zuellig). investigatory and prosecutory agencies of the government
because those subject to its jurisdiction are public officials who,
Respondents continue to argue: Although signed during Garcia’s through official pressure and influence, can quash, delay or
prior term, it is to be made effective during his present term. dismiss investigations held against them.
SC held: NO.
Agreement was perfected on date contract was signed, during In CAB, the Deputy Ombudsman's Order requiring petitioner to
prior term. At that moment, Garcia already acceded to the terms of submit his counter-affidavit was accompanied by the COA Special
the contract, including stipulations now alleged to be prejudicial to Audit Report and the joint affidavit and supplemental joint affidavit
the city government. of State Auditors Cabreros and Quejada. Garcia did not deny
Thus, any culpability Garcia may have in signing the contract receiving the documents.
already became extant on the day the contract was signed. It
hardly matters that the deliveries under the contract are supposed Withdrawal of SPO Tagaan was not fatal to the case.
to have been made months later. Tagaan's report and affidavit still form part of the records of the
case. He could still be called by subpoena if necessary.
SUPER CONCLUSION: Tagaan was a nominal party, whose duty as special prosecutor
Garcia can no longer be held administratively liable for signing was to investigate the commission of crimes and file the
contract with Zuellig. corresponding complaint whenever warranted.
This should not prejudice the filing of any case other than The real complainant is the State
administrative against Garcia – this is not a total exoneration of
Garcia for whatever wrongdoing might have been committed in
signing the subject contract.
Hunting the Elusive G-Spot (Part3):
Garcia vs. Miro (2003) Finding Your G-Spot
The G-Spot is located along the upper/front wall of the vagina,
Facts: about two inches in, towards the stomach. Try some manual
Continuation of Garcia vs. Mojica…
exploration. Lie on your back with your knees bent and rest
Deputy Ombudsman required COA Director to conduct a special
your feet on the bed in front of you. Insert your middle finger
audit, and requested the City Administrator to submit documents
pertaining to the asphalt supply of the city and a copy of the into your vagina and gently stroke the front wall behind the
contract. pubic bone, about two inches up.You should feel a patch of skin
Special Prosecution Officer Tagaan, who conducted the inquiry, that has a different texture from the rest of your vaginal walls,
recommended that a criminal and an administrative complaint be slightly rough or "ruffled." Using a "comehither" motion, press
filed against Garcia and several others. Tagaan filed an affidavit into the center of this ruffled patch until you feel an area that is
with the Graft Investigation Office against Garcia and others for sensitive to pressure. That's your G-Spot, also known as your
violation of Section 3(g) of RA 3019. urethral sponge. It's on the other side of your vaginal wall,
The Office of the City Auditor filed its report which was prepared which is why you'll probably respond more to pressure than
by State Auditors Cabreros and Quejada, Jr. to Miro, concluding light stroking. Thearea is about the size of a pea, but can enlarge
that the transaction is highly questionable in view of the fact that to the size of a walnut when stimulated. You should also know

The Apocryphal Maggots:


Rainier, Chrisgel, Corina, Geoffry, Grace and Sylvie Blanche
The Flibbertigibbet Worms:
Golda, Gladys and Melyjane
CA VE AT: By simply r ea ding this re vi ew er a t the end o f the sem este r wi ll (hopef ully) guar antee you r pas sing this cou rse. D rink mode rat ely .
Amusin S C A r y
gly
THE C2005 LOCAL GOVERNMENT REVIEWER - 169 -
that many women feel like they need to urinate when this area (3) Direct the officer concerned to take appropriate action
is stimulated. This is due to the fact that, as it enlarges,it presses against a public officer or employee at fault or who neglect to
on the same nerves that signal a full bladder. Most women find, perform an act or discharge a duty required by law, and
recommend his removal, suspension, demotion, fine, censure,
however,that as stimulation is continued, this feeling goes away
or prosecution, and ensure compliance therewith; or enforce its
and is replaced by pleasurable sensations.
disciplinary authority as provided in Section 21 of this Act:
(continued…) Provided, That the refusal by any officer without just cause to
comply with an order of the Ombudsman to remove, suspend,
demote, fine, censure, or prosecute an officer or employee who
The Om bud sm an is at fault or who neglects to perform an act or discharge a duty
required by law shall be a ground for disciplinary action against
RA 6770 (1989) – Ombudsman Act of 1989 said officer;
(4) Direct the officer concerned, in any appropriate case,
Sec2. Declaration of Policy. — The State shall maintain honesty and subject to such limitations as it may provide in its rules of
and integrity in the public service and take positive and effective procedure, to furnish it with copies of documents relating to
measures against graft and corruption. contracts or transactions entered into by his office involving the
Public office is a public trust. Public officers and employees disbursement or use of public funds or properties, and report
must at all times be accountable to the people, serve them with any irregularity to the Commission on Audit for appropriate
utmost responsibility, integrity, loyalty, efficiency, act with action;
patriotism and justice and lead modest lives. (5) Request any government agency for assistance and
information necessary in the discharge of its responsibilities,
Sec13. Mandate. — The Ombudsman and his Deputies, as and to examine, if necessary, pertinent records and documents;
protectors of the people, shall act promptly on complaints filed in (6) Publicize matters covered by its investigation of the
any form or manner against officers or employees of the matters mentioned in paragraphs (1), (2), (3) and (4) hereof,
Government, or of any subdivision, agency or instrumentality when circumstances so warrant and with due prudence:
thereof, including government-owned or controlled corporations, Provided, That the Ombudsman under its rules and regulations
and enforce their administrative, civil and criminal liability in may determine what cases may not be made public: Provided,
every case where the evidence warrants in order to promote further, That any publicity issued by the Ombudsman shall be
efficient service by the Government to the people. casia balanced, fair and true;
(7) Determine the causes of inefficiency, red tape,
Sec14. Restrictions. — No writ of injunction shall be issued by mismanagement, fraud, and corruption in the Government, and
any court to delay an investigation being conducted by the make recommendations for their elimination and the observance
Ombudsman under this Act, unless there is a prima facie of high standards of ethics and efficiency;
evidence that the subject matter of the investigation is outside (8) Administer oaths, issue subpoena and subpoena
the jurisdiction of the Office of the Ombudsman. duces tecum, and take testimony in any investigation or inquiry,
No court shall hear any appeal or application for remedy against including the power to examine and have access to bank
the decision or findings of the Ombudsman, except the Supreme accounts and records;
Court, on pure question of law. (9) Punish for contempt in accordance with the Rules of
Court and under the same procedure and with the same
Sec15. Powers, Functions and Duties. — The Office of the penalties provided therein;
Ombudsman shall have the following powers, functions and (10) Delegate to the Deputies, or its investigators or
duties: representatives such authority or duty as shall ensure the
(1) Investigate and prosecute on its own or on complaint effective exercise or performance of the powers, functions, and
by any person, any act or omission of any public officer or duties herein or hereinafter provided;
employee, office or agency, when such act or omission appears (11) Investigate and initiate the proper action for the
to be illegal, unjust, improper or inefficient. It has primary recovery of ill-gotten and/or unexplained wealth amassed after
jurisdiction over cases cognizable by the Sandiganbayan and, in February 25, 1986 and the prosecution of the parties involved
the exercise of this primary jurisdiction, it may take over, at any therein.
stage, from any investigatory agency of Government, the The Ombudsman shall give priority to complaints filed against
investigation of such cases; high ranking government officials and/or those occupying
(2) Direct, upon complaint or at its own instance, any supervisory positions, complaints involving grave offenses as
officer or employee of the Government, or of any subdivision, well as complaints involving large sums of money and/or
agency or instrumentality thereof, as well as any government- properties.
owned or controlled corporations with original charter, to perform
and expedite any act or duty required by law, or to stop, prevent, Sec16. Applicability. — The provisions of this Act shall apply to all
and correct any abuse or impropriety in the performance of kinds of malfeasance, misfeasance, and non-feasance that have
duties; been committed by any officer or employee as mentioned in
Section 13 hereof, during his tenure of office.

The Apocryphal Maggots:


Rainier, Chrisgel, Corina, Geoffry, Grace and Sylvie Blanche
The Flibbertigibbet Worms:
Golda, Gladys and Melyjane
CA VE AT: By simply r ea ding this re vi ew er a t the end o f the sem este r wi ll (hopef ully) guar antee you r pas sing this cou rse. D rink mode rat ely .
Amusin S C A r y
gly
THE C2005 LOCAL GOVERNMENT REVIEWER - 170 -
(1) The complainant has an adequate remedy in another
Sec17. Immunities. — In all hearings, inquiries, and proceedings judicial or quasi-judicial body;
of the Ombudsman, including preliminary investigations of (2) The complaint pertains to a matter outside the
offenses, nor person subpoenaed to testify as a witness shall be jurisdiction of the Office of the Ombudsman;
excused from attending and testifying or from producing books, (3) The complaint is trivial, frivolous, vexatious or made in
papers, correspondence, memoranda and/or other records on bad faith;
the ground that the testimony or evidence, documentary or (4) The complainant has no sufficient personal interest in
otherwise, required of him, may tend to incriminate him or the subject matter of the grievance; or
subject him to prosecution: Provided, That no person shall be (5) The complaint was filed after one (1) year from the
prosecuted criminally for or on account of any matter concerning occurrence of the act or omission complained of.
which he is compelled, after having claimed the privilege against
self-incrimination, to testify and produce evidence, documentary Sec21. Official Subject to Disciplinary Authority; Exceptions.
or otherwise. — The Office of the Ombudsman shall have disciplinary
Under such terms and conditions as it may determine, taking authority over all elective and appointive officials of the
into account the pertinent provisions of the Rules of Court, the Government and its subdivisions, instrumentalities and
Ombudsman may grant immunity from criminal prosecution to agencies, including Members of the Cabinet, local government,
any person whose testimony or whose possession and government-owned or controlled corporations and their
production of documents or other evidence may be necessary to subsidiaries, except over officials who may be removed only by
determine the truth in any hearing, inquiry or proceeding being impeachment or over Members of Congress, and the Judiciary.
conducted by the Ombudsman or under its authority, in the
performance or in the furtherance of its constitutional functions Sec22. Investigatory Power. — The Office of the Ombudsman
and statutory objectives. The immunity granted under this and shall have the power to investigate any serious misconduct in
the immediately preceding paragraph shall not exempt the office allegedly committed by officials removable by
witness from criminal prosecution for perjury or false testimony impeachment, for the purpose of filing a verified complaint for
nor shall he be exempt from demotion or removal from office. impeachment, if warranted.
Any refusal to appear or testify pursuant to the foregoing In all cases of conspiracy between an officer or employee of the
provisions shall be subject to punishment for contempt and government and a private person, the Ombudsman and his
removal of the immunity from criminal prosecution. Deputies shall have jurisdiction to include such private person in
the investigation and proceed against such private person as
Sec18. Rules of Procedure. — the evidence may warrant. The officer or employee and the
(1) The Office of the Ombudsman shall promulgate its private person shall be tried jointly and shall be subject to the
rules of procedure for the effective exercise or performance of same penalties and liabilities.
its powers, functions, and duties.
(2) The rules of procedure shall include a provision Sec23. Formal Investigation. —
whereby the Rules of Court are made suppletory. (1) Administrative investigations conducted by the Office
(3) The rules shall take effect after fifteen (15) days of the Ombudsman shall be in accordance with its rules of
following the completion of their publication in the Official procedure and consistent with due process.
Gazette or in three (3) newspapers of general circulation in the (2) At its option, the Office of the Ombudsman may refer
Philippines, one of which is printed in the national language. certain complaints to the proper disciplinary authority for the
institution of appropriate administrative proceedings against
Sec19. Administrative Complaints. — The Ombudsman shall act erring public officers or employees, which shall be determined
on all complaints relating, but not limited to acts or omissions within the period prescribed in the civil service law. Any delay
which: without just cause in acting on any referral made by the Office of
(1) Are contrary to law or regulation; the Ombudsman shall be a ground for administrative action
(2) Are unreasonable, unfair, oppressive or against the officers or employees to whom such referrals are
discriminatory; addressed and shall constitute a graft offense punishable by a
(3) Are inconsistent with the general course of an fine of not exceeding Five thousand pesos (P5,000.00).
agency's functions, though in accordance with law; (3) In any investigation under this Act the Ombudsman
(4) Proceed from a mistake of law or an arbitrary may: (a) enter and inspect the premises of any office, agency,
ascertainment of facts; commission or tribunal; (b) examine and have access to any
(5) Are in the exercise of discretionary powers but for an book, record, file, document or paper; and (c) hold private
improper purpose; or hearings with both the complaining individual and the official
(6) Are otherwise irregular, immoral or devoid of concerned.
justification.
Sec24. Preventives Suspension. — The Ombudsman or his
Sec20. Exceptions. — The Office of the Ombudsman may not Deputy may preventively suspend any officer or employee under
conduct the necessary investigation of any administrative act or his authority pending an investigation, if in his judgment the
omission complained of if it believes that: evidence of guilt is strong, and (a) the charge against such

The Apocryphal Maggots:


Rainier, Chrisgel, Corina, Geoffry, Grace and Sylvie Blanche
The Flibbertigibbet Worms:
Golda, Gladys and Melyjane
CA VE AT: By simply r ea ding this re vi ew er a t the end o f the sem este r wi ll (hopef ully) guar antee you r pas sing this cou rse. D rink mode rat ely .
Amusin S C A r y
gly
THE C2005 LOCAL GOVERNMENT REVIEWER - 171 -
officer or employee involves dishonesty, oppression or grave issue such orders directing the officer, employee, office or
misconduct or neglect in the performance of duty; (b) the agency concerned to:
charges would warrant removal from the service; or (c) the (a) expedite the performance of duty;
respondent's continued stay in office may prejudice the case (b) cease or desist from the performance of a prejudicial
filed against him. act;
The preventive suspension shall continue until the case is (c) correct the omission;
terminated by the Office of the Ombudsman but not more than (d) explain fully the administrative act in question; or
six (6) months, without pay, except when the delay in the (e) take any other steps as may be necessary under the
disposition of the case by the Office of the Ombudsman is due circumstances to protect and preserve the rights of the
to the fault, negligence or petition of the respondent, in which complainant.
case the period of such delay shall not be counted in computing (4) Any delay or refusal to comply with the referral or
the period of suspension herein provided. directive of the Ombudsman or any of his Deputies, shall
constitute a ground for administrative disciplinary action against
Sec25. Penalties. — the officer or employee to whom it was addressed.
(1) In administrative proceedings under Presidential
Decree No. 807, the penalties and rules provided therein shall Sec27. Effectivity and Finality of Decisions. — (1) All
be applied. provisionary orders of the Office of the Ombudsman are
(2) In other administrative proceedings, the penalty immediately effective and executory.
ranging from suspension without pay for one (1) year to A motion for reconsideration of any order, directive or decision of
dismissal with forfeiture of benefits or a fine ranging from Five the Office of the Ombudsman must be filed within five (5) days
thousand pesos (P5,000.00) to twice the amount malversed, after receipt of written notice and shall be entertained only on
illegally taken or lost, or both at the discretion of the any of the following grounds:
Ombudsman, taking into consideration circumstances that (1) New evidence has been discovered which materially
mitigate or aggravate the liability of the officer or employee affects the order, directive or decision;
found guilty of the complaint or charges. (2) Errors of law or irregularities have been committed
prejudicial to the interest of the movant. The motion for
Sec26. Inquiries. — reconsideration shall be resolved within three (3) days from
(1) The Office of the Ombudsman shall inquire into acts filing: Provided, That only one motion for reconsideration shall
or omissions of a public officer, employee, office or agency be entertained.
which, from the reports or complaints it has received, the Findings of fact by the Officer of the Ombudsman when
Ombudsman or his Deputies consider to be: supported by substantial evidence are conclusive. Any order,
(a) contrary to law or regulation; directive or decision imposing the penalty of public censure or
(b) unreasonable, unfair, oppressive, irregular or reprimand, suspension of not more than one (1) month's salary
inconsistent with the general course of the operations shall be final and unappealable.
and functions of a public officer, employee, office or In all administrative disciplinary cases, orders, directives, or
agency; decisions of the Office of the Ombudsman may be appealed to
(c) an error in the application or interpretation of law, rules the Supreme Court by filing a petition for certiorari within ten
or regulations, or a gross or palpable error in the (10) days from receipt of the written notice of the order, directive
appreciation of facts; or decision or denial of the motion for reconsideration in
(d) based on improper motives or corrupt considerations; accordance with Rule 45 of the Rules of Court.
(e) unclear or inadequately explained when reasons The above rules may be amended or modified by the Office of
should have been revealed; or the Ombudsman as the interest of justice may require.
(f) inefficient performed or otherwise objectionable.
(2) The Officer of the Ombudsman shall receive Sec28. Investigation in Municipalities, Cities and Provinces. —
complaints from any source in whatever form concerning an The Office of the Ombudsman may establish offices in
official act or omission. It shall act on the complaint immediately municipalities, cities and provinces outside Metropolitan Manila,
and if it finds the same entirely baseless, it shall dismiss the under the immediate supervision of the Deputies for Luzon,
same and inform the complainant of such dismissal citing the Visayas and Mindanao, where necessary as determined by the
reasons therefor. If it finds a reasonable ground to investigate Ombudsman. The investigation of complaints may be assigned
further, it shall first furnish the respondent public officer or to the regional or sectoral deputy concerned or to a special
employee with a summary of the complaint and require him to investigator who shall proceed in accordance with the rules or
submit a written answer within seventy-two (72) hours from special instructions or directives of the Office of the
receipt thereof. If the answer is found satisfactory, it shall Ombudsman. Pending investigation the deputy or investigator
dismiss the case. may issue orders and provisional remedies which are
(3) When the complaint consists in delay or refusal to immediately executory subject to review by the Ombudsman.
perform a duty required by law, or when urgent action is Within three (3) days after concluding the investigation, the
necessary to protect or preserve the rights of the complainant, deputy or investigator shall transmit, together with the entire
the Office of the Ombudsman shall take steps or measures and records of the case, his report and conclusions to the Office of

The Apocryphal Maggots:


Rainier, Chrisgel, Corina, Geoffry, Grace and Sylvie Blanche
The Flibbertigibbet Worms:
Golda, Gladys and Melyjane
CA VE AT: By simply r ea ding this re vi ew er a t the end o f the sem este r wi ll (hopef ully) guar antee you r pas sing this cou rse. D rink mode rat ely .
Amusin S C A r y
gly
THE C2005 LOCAL GOVERNMENT REVIEWER - 172 -
the Ombudsman. Within five (5) days after receipt of said report, TO JUSTIFY …
the Ombudsman shall render the appropriate order, directive or Evidence of guilt must be It is enough that:
decision. strong and 1. there is reasonable
1. the charge ag. the ground to believe that
Sec29. Change of Unjust Laws. — If the Ombudsman believes officer or employee the respondent has
that a law or regulation is unfair or unjust, he shall recommend should involve committed the act or
to the President and to Congress the necessary changes therein dishonesty, acts complained of;
or the repeal thereof. oppression, grave 2. evidence of
misconduct or neglect culpability is strong
Sec30. Transmittal/Publication of Decision. — In every case in the performance of 3. gravity of the offense
where the Ombudsman has reached a decision, conclusion or duty; so warrants; or
recommendation adverse to a public official or agency, he shall 2. the charges should 4. the continuance in
transmit his decision, conclusion, recommendation or warrant removal from office of the
suggestion to the head of the department, agency or service; or respondent could
instrumentality, or of the province, city or municipality concerned 3. the continued stay in influence the
for such immediate action as may be necessary. When office of respondent witnesses or pose a
transmitting his adverse decision, conclusion or would prejudice the threat to the safety
recommendation, he shall, unless excused by the agency or case filed against him. and integrity of the
official affected, include the substance of any statement the records and other
public agency or official may have made to him by way of evidence
explaining past difficulties with or present rejection of the
Ombudsman's proposals. Not being in the nature of a penalty, a preventive suspension can
be decreed on an official under investigation after charges are
heard. Naturally such would occur prior to any finding of guilt.
Hagad vs. Gozo-Dadole (1993)
DanGat: Can one file administrative cases against elective
Facts: officials with different disciplining authorities simultaneously?
Criminal and administrative complaints were filed against This is possible. Cases do not categorically say that filing with
respondents (mayor, v-mayor and SP member) with the Deputy one excludes the filing with others.
Ombudsman after causing the alteration of Ordinance No. 018/92
by increasing the allocated appropriation therein.
Respondents prayed for dismissal of the complaint. Ombudsman
Garcia vs. Mojica (1999)
has no jurisdiction over the admin case filed against them since
Sec 63 LGC vests the power to investigate and impose admin.
Sanctions against local officials with the OP. Denied. Hagad vs. Gozo-Dadole has said that nothing in LGC indicate that
it has repealed (express or implied) the pertinent provisions of
Held: Ombudsman Act – not being inconsistent.
1. Ombudsman has jurisdiction over the case. Although it has been argued that 6month period under
There is nothing in the LGC to indicate that it has repealed, Ombudsman Law is much too repugnant to 60day period under
whether expressly or impliedly, the pertinent provisions of the LGC, Vitug said that the two govern differently.
Ombudsman Act. The 2 statutes on the specific matter in question
are not so inconsistent as to compel the court to uphold 1 and It is contended that the Hagad case did not settle the question of
strike down the other. whether a local elective official may be preventively suspended
Implied repeals are not favored and it is fundamental that the even before the issues could be joined. And he was not given a
legislature should be presumed to have known the existing laws on chance to refute first the charges against him. Note: Under the
the subject and not to have enacted conflicting statutes. LGC, preventive suspension may only be imposed after issues are
Moreover, Sections61 and 63 of the present LGC run almost joined.
parallel with the provisions then existing under the old code. The HOWEVER, It has been held in other cases that there could be
passage of 1991 LGC cannot be deemed to have effected a preventive suspension even before the charges against the official
change from what already prevailed, the modification being only in are heard, or before the official is given an opportunity to prove his
the substitution of the Sec. of the LG with the OP. innocence.
Preventive suspension is merely a preliminary step in an
2. The 6-month preventive suspension without pay under the administrative investigation and is not in any way the final
Ombudsman Act is not repugnant to the 60-day preventive determination of the guilt of the official concerned.
suspension in the LGC. The 2 provisions govern differently.

Preventive suspension Preventive suspension under Constantino vs. Desierto (1998)


under Ombudsman Act LGC

The Apocryphal Maggots:


Rainier, Chrisgel, Corina, Geoffry, Grace and Sylvie Blanche
The Flibbertigibbet Worms:
Golda, Gladys and Melyjane
CA VE AT: By simply r ea ding this re vi ew er a t the end o f the sem este r wi ll (hopef ully) guar antee you r pas sing this cou rse. D rink mode rat ely .
Amusin S C A r y
gly
THE C2005 LOCAL GOVERNMENT REVIEWER - 173 -
Facts:
Deputy Ombudsman Gervacio placed Mayor Constantino under 2. There is no denial of due process when they were not given
preventive suspension, being accused of violation of the Anti-Graft an opportunity to controvert the evidence before the order of
and Corrupt Practices Act because the terms of the lease preventive suspension was issued.
agreement and undertaking between the mayor for the municipality A preventive suspension can be decreed on an official under
and the company were contrary to Resolution passed by the investigation after charges are brought and even before the
Sangguniang Bayan. Motions and notice by Mayor were denied. charges are heard since the same is not in the nature of a penalty,
Information was filed against the mayor before Sandiganbayan. but merely a preliminary step in an administrative investigation.
Graft Investigation Officer Buena handed down a resolution finding CAB: The fact that the said order was issued 7 days after the
the mayor guilty and ordered his dismissal from service. complaint was filed did not constitute grave abuse of discretion.
The immediate issuance of such order is required in order to
Held: prevent the subject of the suspension from committing further
Review by the Ombudsman of the motions for inhibition and to irregularities.
reset hearing are no longer necessary where the authority of the Such prompt action is moreover in consonance with Sec. 15 of RA
Graft Investigation Officer to issue the order pending the 6770 which exhorts the Ombudsman to “…to give priority to
investigation of the administrative case is specifically conferred by complaints filed against high ranking government officials and/or
Sec28 of the Ombudsman Act. In fact, such order is immediately those occupying supervisory positions, complaints involving grave
executory. offenses as well as complaints involving large sums of money
Furthermore, the impugned order was actually reviewed by a and/or properties.”
superior officer. It was then recommended for approval by the
Deputy Ombudsman, and ultimately approved by Ombudsman 3. Conditions required for a preventive suspension have been
Desierto. The motions therefore received due attention and complied with.
st
consideration although resolved against him. 1 requisite: Evidence of guilt is strong. This rests upon the
determination of the disciplining authority who is given discretion to
HOWEVER, evidence against him is inadequate to warrant his decide when the evidence of guilt is strong.
dismissal from service on the specified grounds of grave 2nd requisite: Presence of any of the following circumstances –
misconduct, conduct prejudicial to the best interest of the service o The charge against such officer or employee involves
and gross neglect of duty. dishonesty, oppression or gross misconduct, or neglect in the
performance of duty;
o The charges would warrant removal from service; or
Castillo-Co vs. Barbers (1998) o The respondent’s continued stay in office may prejudice the
case filed against him.
Facts: CAB: all circumstances are present.
Congressman Cua filed a complaint before the Office of the It is even immaterial that no evidence has been adduced to prove
Ombudsman against Governor Castillo-Co and Provincial Engineer that petitioner may influence possible witnesses or may tamper
Ringor for violation of the Anti-Graft and Corrupt Practices Act and with public records. It is sufficient that there exists such possibility.
the RPC (fraud against the public treasury and malversation)
They were preventively suspended for a period of 6months based 4. Preventive suspension is not excessive.
on an order by Director Gonzales and approved by the Deputy Suspension for 6 mos. is within the limits prescribed by Sec. 24 of
Ombudsman. the Ombudsman Law. The length of the period of suspension
within such limits, like the evaluation of the strength of the
Held: evidence, lies in the discretion of the Ombudsman.
1. The Deputy Ombudsman has authority to place them under
preventive suspension.
Lapid vs. CA (2000)
There is nothing in RA7975 that remotely suggests that only the
Ombudsman, and not his Deputy, may sign an order preventively Facts:
suspending officials occupying positions classified as grade27 or The Ombudsman issued an Order preventively suspending
above. Governor Lapid et al for a period of 6months pursuant to Sec24,
In fact under Sec24 of the Ombudsman Law and Sec9, Rule III of RA6770 which was implemented by DILG.
the Rules of Procedure of the Office of the Ombudsman, there is Later on, the Ombudsman rendered a decision in the
no doubt that the Ombudsman or his Deputy may preventively administrative case finding Lapid administratively liable for
suspend an officer or employee, where appropriate, as indicated misconduct with a penalty of 1year suspension pursuant to
by the word “or” between the “Ombudsman” and “his Deputy”. It is Sec25(2) of RA6770.
a disjunctive term signifying disassociation and independence of
one thing from other things enumerated. Held:
The law does not require that only the Ombudsman himself may 1. The decision of the Office of the Ombudsman is not
sign the order of suspension. immediately executory pending appeal.

The Apocryphal Maggots:


Rainier, Chrisgel, Corina, Geoffry, Grace and Sylvie Blanche
The Flibbertigibbet Worms:
Golda, Gladys and Melyjane
CA VE AT: By simply r ea ding this re vi ew er a t the end o f the sem este r wi ll (hopef ully) guar antee you r pas sing this cou rse. D rink mode rat ely .
Amusin S C A r y
gly
THE C2005 LOCAL GOVERNMENT REVIEWER - 174 -
be applied suppletorily to the provisions of the Ombudsman Act as
It is clear under Sec27 of RA6770 and Rules of Procedure of the there is nothing in the Ombudsman Act which provides for such
Office of the Ombudsman that the punishment imposed upon suppletory application.
petitioner (suspension without pay for 1year) is not among those CAB: the acts attributed to Lapid could have been the subject of
listed as final and unappealable. administrative disciplinary proceedings before the Office of the
Sec. 27 states that all provisionary orders of the Office of the Pres. under the LGC or before the Office of the Ombudsman under
Ombudsman are immediately effective and executory; and that any the Ombudsman Act. Considering however, that petitioner was
order, directive or decision of the said Office imposing the penalty charged under the Ombudsman Act, it is this law alone which
of censure or reprimand or suspension of not more than 1 month’s should govern his case.
salary is final and unappealable.
As such, the legal maxim “inclusion unius est exclusion alterus”
finds application. The express mention of the things included
excludes those that are not included. The import of these Hunting the Elusive G-Spot (Part4):
statements is that all other decisions of the Office of the
Ombudsman which impose penalties that are not enumerated Doing the G-Spot Jiggly
in the said sec. 27 are not final, unappealable and immediately The G Spot is not a magical button,but rather, an area that some
executory.
women enjoy having stimulated. The following steps are
CAB: An appeal was timely filed which can stay the immediate
written for self-exploration, but can be modified for use with a
implementation of the decision – as supported by the Rules of
Procedure issued by the Ombudsman itself which states that “In all partner.
other cases, the decision shall become final after the expiration of • Relax. This should be fun, not a goal-oriented mission.
10 days from receipt thereof by the respondent, unless a MR or • Use a lubricant you like.
pet. for cert. shall have been filed by him as prescribed in Sec27 of
RA 6770.” • Masturbate in your usual way until you feel aroused.
• Insert one or two fingers into the vagina, crooking them up
2. As to the applicability of the case Fabian vs. Desierto toward the belly (12 o'clock position if you are lying on
The ruling in Fabian v. Desierto invalidated Sec. 27 of RA 6770 your back).
and any other provision of law implementing the Act only insofar as • Press firmly against the roof of the vagina about one-third
they provide for appeals in administrative disciplinary cases from
of the way in.
the Office of the Ombudsman to the SC. The only provision
affected by the Fabian ruling is the designation of the CA as the • You should feel a small ruffled lump that increases in size
proper forum and of Rule 43 as the proper mode of appeal. All with continued stimulation.
other matters included in said sec. 27, including the finality of • Start slow: insert your fingers gently and use soft, gentle
decisions, are not affected.
motions at first. When you feel the G-Spot area becoming
enlarged, use more pressure.
3. Rule43 of ROC which allegedly mandates immediate
execution of all decisions rendered by administrative and • Remember to keep paying attention to other parts of your
quasi-judicial agencies is not applicable. body: your breasts, your clitoris ... whatever feels good.
The fact that the Ombudsman Act gives the parties the right to • As your body begins to respond positively and you start
appeal from its decisions should generally carry with it the stay of
feeling pleasure, use more pressure and more rapid
these decisions pending appeal. Otherwise, the essential nature of
these judgments as being appealable would be rendered nugatory. motions.
There is no general legal principle that mandates that all decisions • As you continue to stimulate the G-Spot, you may feel your
of such agencies are immediately executory. vagina clench and bear down, the signal that orgasm is
imminent. Apply more pressure to the urethral sponge,
The decisions of the Civil Service Commission under the Admin. stroking and manipulating the area around the urethral
Code, and the Office of the Pres. under the LGC, which opening.
respondents cite, are immediately executory even pending appeal
because the pertinent laws under which the decisions were • You may feel the urge to urinate. Don't fight it! You are
rendered expressly mandate them to be so. Similarly, the provision probably getting ready to ejaculate. Relax, trust your body
in the 1987 Admin. Code mandating execution pending review and your partner, and see what happens.
applies specifically to administrative decisions of the CSC involving (continued…)
members of the Civil Service.
Sec. 12 of Rule 43 should therefore be interpreted as mandating
that the appeal will not stay the award, judgment, final order or The Cou rt s
resolution unless the law directs otherwise.
There is no basis in law for the proposition that the provisions of
the Admin. Code and the LGC on execution pending review should

The Apocryphal Maggots:


Rainier, Chrisgel, Corina, Geoffry, Grace and Sylvie Blanche
The Flibbertigibbet Worms:
Golda, Gladys and Melyjane
CA VE AT: By simply r ea ding this re vi ew er a t the end o f the sem este r wi ll (hopef ully) guar antee you r pas sing this cou rse. D rink mode rat ely .
Amusin S C A r y
gly
THE C2005 LOCAL GOVERNMENT REVIEWER - 175 -
DanGat Notes: Sec13, RA3019 does not provide for the length Barangay Chairman Juan and other barangay kagawads were
of the suspension. criminally charged before RTC QC for violating the Omnibus
It will be unconstitutional if the law be applied strictly where the Election Code by unlawfully using their political campaigns
official can be suspended for the whole duration of the case. properties belonging to the barangay.
Possible Remedy: RTC issued an order directing their immediate suspension from
officer for 60days (authority under Sec13 of RA3019).
- Discretion given to the courts as to the length of the
suspension
Held:
- Apply other laws (regardless of the nature of the case) 1. RTCs have jurisdiction over violations of the Election Code.
Example: It is evident in Sec32 of BP129 (as amended by RA7691) that the
For elective officials – apply LGC and impose 60days… (as in jurisdiction of first-level courts (MTCs) does not cover those
Rios case) criminal cases which by specific provision of law are cognizable by
For non-elective officials – apply Civil Service or Admin Code RTCs.
and impose 90days… (as in Segovia case) Petitioners were charged with violating Sec261(o) of the Omnibus
Election Code, and under Sec268 of said Code, RTCs havee
exclusive jurisdiction to try and decide any criminal action or
Bunye vs. Escareal (1993) proceedings for violation of the Code, except those relating to the
offense of failure to register or failure to vote.
Facts: Pursuant to Sec268 of the Omnibus Election Code, election
Petitioners Municipal Mayor, Vice Mayor and Councilors questions offenses fall within the exception.
the resolution suspending them from office for 90 days pending
their trial for violation of Sec3(3) of the Anti-Graft and Corrupt 2. The preventive suspension order against petitioners is proper.
Practices Act. Sec13, RA3019 as amended by BP195 now covers offenses of
public officers falling under “Crimes Committed by Public Officers”
Held: Suspension of petitioners was proper. in the RPC, and with any other form of fraud involving government
Sec13 of RA3019 unequivocally provides that the accused public funds or property.
official “shall be suspended from office” while the criminal Cases against petitioners involve violations of the Election Code
prosecution is pending in court. but it must be emphasized that they are charged with the
unauthorized and unlawful use of government property in their
Under Gonzaga case, such suspension is mandatory. It is not custody which is akin to that committed by public officers as laid
violative of the Constitution as it is not a penalty. It is mandatory as down in RPC.
soon as the validity of the information is determined. Certainly, their acts constitute fraud against the government, thus
the case is covered by Sec13 of RA3019.
There is no merit in the contention that their admission of the acts
constituting the offense charged against them eliminates Under Sec13 of RA3019 and as enunciated in Socrates vs.
apprehension that they might tamper with the records. Sandiganbayan, the suspension of a public officer is mandatory
It is not for the petitioners to say that their admissions are all the after the determination of the validity of the information in a pre-
evidence that he prosecution will need to hold up its case against suspension hearing conducted for the purpose.
them CAB: There was no pre-suspension hearing held to determine the
The prosecution must be given the opportunity to gather and validity of the information that had been filed against them.
prepare the facts for trial under conditions which would ensure non However, the numerous pleadings filed for and against them have
intervention and non-interference for 90 days from petitioner’s achieved the goal of this procedure.
camp Right to due process is satisfied not just by an oral hearing but by
the filing and the consideration by the court of the parties’
The fear that the government will be paralyzed by their suspension pleadings, memoranda and other position papers.
is remote:
There are 8 councilors left who can meet as the Sangguniang
Bayan. Llorente, Jr. vs. Sandiganbayan (2000)
The President or his alter ego, the Secretary of Interior and Local
Government will know how to deal with the problem of filling up the Facts:
temporarily vacant positions in accordance with the provisions of Two informations were filed before the Sandiganbayan, charging
the LGC. Municipal Mayor Llorento for violating §§3 (e)(f) of RA3019.
Then Congress enacted RA7975:
"Sec. 4. Jurisdiction. - The Sandiganbayan shall exercise
Juan vs. People of the Philippines (2000) original jurisdiction on all cases involving:
"a. Violations of Republic Act No. 3019, as amended,
Facts: otherwise known as the Anti-Graft and Corrupt Practices Act xxx
where one or more of the principal accused are officials

The Apocryphal Maggots:


Rainier, Chrisgel, Corina, Geoffry, Grace and Sylvie Blanche
The Flibbertigibbet Worms:
Golda, Gladys and Melyjane
CA VE AT: By simply r ea ding this re vi ew er a t the end o f the sem este r wi ll (hopef ully) guar antee you r pas sing this cou rse. D rink mode rat ely .
Amusin S C A r y
gly
THE C2005 LOCAL GOVERNMENT REVIEWER - 176 -
occupying the following positions in the government, whether in The court has maintained a consistent policy of non-interference in
a permanent, acting or interim capacity, at the time of the the determination of the Ombudsman regarding the existence of
commission of the offense: probable cause, EXCEPT when there is grave abuse of discretion
"(1) Officials of the executive branch occupying the in such exercise by the Ombudsman.
positions of regional director and higher, otherwise classified as This rule is based not only upon respect for the investigatory and
grade 27 and higher, of the Compensation and Position prosecutory powers granted to the Ombudsman by the
Classification Act of 1989 (Republic Act No. 6758) xxx Constitution; it is also grounded on practicality. Otherwise,
"In cases where none of the principal accused are occupying innumerable petitions seeking dismissal of investigatory
positions corresponding to salary grade "27" or higher, as proceedings conducted by the Ombudsman will hamper the
prescribed in the said Republic Act No. 6758 xxx exclusive functions of said office and the courts.
jurisdiction thereof shall be vested in the proper Regional Trial
Court, Metropolitan Trial Court, Municipal Trial Court, and 2. Sandiganbayan has jurisdiction over the criminal cases
Municipal Circuit Trial Court, as the case may be, pursuant to against the Mayor and the Barangay Captain.
their respective jurisdictions as provided in Batas Pambansa Jurisprudence has already settled this question. Municipal mayors
Blg. 129.” fall under the exclusive jurisdiction of the Sandiganbayan.
Llorente moved to dismiss or transfer the cases to RTC on the Neither can Barangay Captain Esquivel claim that since he is
ground that RA7975 divested the Sandiganbayan of its jurisdiction not a municipal mayor, he is not within the jurisdiction of the
over criminal cases against municipal mayors who receive salary Snadiganbayan. It is only in cases when "NONE of the
less than that corresponding to Grade27. accused are occupying positions corresponding to salary grade
27 or higher " that exclusive jurisdiction will be vested in the
Held: Sandiganbayan has jurisdiction over the cases. proper court.
It is the official’s grade that determines his salary and not the other
way around. There is no merit to the contention that the salary
received by a public officer dictates his salary grade. Rios vs. Sandiganbayan, 2nd Div. (1997)
An official’s grade is not a matter of proof, but a matter of law
which the court must take judicial notice [of]. Facts:
The municipal mayor has a salary grade of 27 according to: An information was filed with the Sandiganbayan against Mayor
- §444(d) of the LGC  that municipal mayor shall receive a Rios for alleged unauthorized disposition of confiscated lumber, in
min monthly compensation corresponding to SG27 as violation of the Anti-Graft and Corrupt Practices Act.
prescribed under RA6758 and the IRR issued pursuant Sandiganbayan granted Office of the Special Prosecutor’s motion
thereto to suspend Mayor Rios (for 90days).
- 1989 & 1997 versions of the Index of Occupational Services,
Position Titles & Salary Grades Held:
It is subject to the jurisdiction of the Sandiganbayan. 1. Sandiganbayan did not commit Grave Abuse of Discretion
when it ruled that the facts charged in the information
constituted a violation of RA3019 (Anti-Graft Law)
Esquivel vs. Ombudsman (2002) The SC in a resolution (in the case Rios v Sandiganbayan
and the People of the Philippines) already passed upon this
issue when it held that the act of disposing confiscated
Facts:
lumber without prior authority from the DENR and the
Complaint-affidavits were filed with PNP against Mayor Esquivel et
Sandiganbayan constituted a violation of RA 3019. Hence,
al. After initial investigation, PNP-CIDG forwarded the records to
there was probable cause
the Office of the Deputy Ombudsman.
Any act or omission not in consonance with the prescribed
Deputy Ombudsman conducted preliminary investigation and later
norms of conduct inflicts injury to the Government for it is a
issued a resolution recommending that both Mayor Esquival and
disturbance of law and order.
his brother Barangay Captain Esquivel be indicted for the crime of
The assertion that there was no undue injury is based on the
less serious physical injuries and Mayor Esquivel alone for grave
wrong assumption that the lumber belonged to the municipality.
threats.
The lumber belongs to the National Government.
Separate informations were filed with Sandiganbayan.
2. Also, No Grave Abuse of Discretion when Sandiganbayan
Held:
provided for the suspension of Mayor Rios, but only for
1. No grave abuse of discretion on the part of the Ombudsman
60days and not 90days.
in direting the filing of the informations against petitioners.
As the information filed against Rios is valid, there can be no
The Ombudsman is empowered to determine whether there exists
impediment to the application of Section 13 of R.A. No. 3019
reasonable ground to believe that a crime has been committed and
which makes it mandatory for the Sandiganbayan to suspend any
that the accused is probably guilty thereof and, thereafter, file the
public officer who has been validly charged with graft and
corresponding information with the appropriate courts.
corrupt practices.

The Apocryphal Maggots:


Rainier, Chrisgel, Corina, Geoffry, Grace and Sylvie Blanche
The Flibbertigibbet Worms:
Golda, Gladys and Melyjane
CA VE AT: By simply r ea ding this re vi ew er a t the end o f the sem este r wi ll (hopef ully) guar antee you r pas sing this cou rse. D rink mode rat ely .
Amusin S C A r y
gly
THE C2005 LOCAL GOVERNMENT REVIEWER - 177 -
However, it is well settled that preventive suspension should 2. WON the acts for which he’s charged constitute a violation of RA
not extend beyond 60 days. 3019 or title 7, Book II of RPC
3. WON the information against him can be quashed under any of
DanGat Notes :Sec63 of LGC provides that preventive the grounds in Sec 2 Rule 117 ROC
suspension can be for 90days if several administrative cases
are filed against an elective official but only within a single year When the statute is clear and explicit, there is hardly any room for
on the same ground or grounds existing and known at the time court ratiocination of the law. RA 3019 unequivocally mandates the
of the first suspension. suspension of a public official from office pending a criminal
prosecution against him. Such preventive suspension is
mandatory, and there are no ‘ifs’ and ‘butts’ about it.
Segovia vs. Sandiganbayan (1999)
Removal
Facts:
Petitioners Segovia et al were criminally charged in Aguinaldo vs. Santos (1992)
Sandiganbayan with infringement of Sec3(e) of RA3019 for
“causing undue injury to any party including the Gov’t, or giving Issue: W/N his re-election to Governor position has rendered the
any party any umwarranted benefits, advantage or preference in administrative case pending before SC moot and academic.
the discharge of his official, administrative, or judicial functions Held: YES
through manifest partiality, evident bad faith or gross or RULE: A public official can not be removed for administrative
negligence.” misconduct committed during a prior term, since his re-election to
The People filed a motion to suspend the petitioners office operates as a condonation of the officer's previous
Sandiganbayan issued a resolution ordering the suspension of misconduct to the extent of cutting off the right to remove him
petitioners under Sec 1313 RA 3019 for 90 days. therefor. It is assumed that the electorate did this with knowledge
of his life and character.
Held: It is mandatory to place public officers who stand accused Such rule finds no application to criminal cases pending against
before the court to be placed under preventive suspension petitioner for acts he may have committed during the failed coup.
pursuant to Sec13 of RA3019.
It has been consistently held that it is mandatory for the court to The 1987 Constitution has not repealed the power of DLG Sec to
place under preventive suspension a public officer accused before suspend or remove local government officials as alter ego of the
it. Imposition of suspension, however, is not automatic or self- President.
operative. The Sec’s power to remove is anchored on the Constitution and a
Pre-condition: existence of valid information determined at a pre- statutory grant from the legislature:
suspension hearing - Constitutional basis: in Articles VII (17) and X (4) – which vest
Purpose of pre-suspension hearing: to determine the validity of the in the President the power of control over all executive
information and furnish the court w/ basis to either suspend departments, bureaus and offices and the power of general
accused and proceed with trial on merits or refuse suspension and supervision over local governments, and by the doctrine that
dismiss case. the acts of the department head are presumptively the acts of
Once a proper determination of validity of the information has been the President unless expressly rejected by him
made, it becomes the ministerial duty of the court to issue the - The statutory grant in BP337 has constitutional roots - having
order of preventive suspension not exceeding 90 days in been enacted by the Batas Pambansa pursuant to Art XI,
consonance w/ Civil Service Decree/Admin Code. Sec2 of 1973 Constitution which states that the National
Assembly shall... provide for the qualifications, election and
DOCTRINE: Under Sec 13, RA 3019, the suspension of a public removal... of local government officials
officer is mandatory after a determination has been made of the - a similar provision is found in Art X, Sec3 of 1987 Constitution
validity of the information in a pre-suspension hearing conducted that the Congress shall... provide for the qualifications,
for that purpose. election, appointment, and removal... and all other matters
The Court’s discretion lies only during the pre-suspension hearing relating to the organization and
where it is required to ascertain operation of the local units.
1. WON the accused has been accorded due preliminary Inasmuch as the power and authority of the legislature to enact a
investigation prior to filing of information local government code, which provides for the manner of removal
of local government officials, is found in the 1973 and 1987
Constitutions, then it cannot be said that BP337 was repealed by
13 the effectivity of the present Consitution.
SEC. 13. Suspension and Loss of Benefits. – Any incumbent public officer
against whom any criminal prosecution under a valid information under this
Moreover, the Court had stated in the case Bagabuyo vs. Davide
Act or under Title 7, Book II of the Revised Penal Code or for any offense
involving fraud upon government or public funds or property whether as a that BP337 remained in force despite the effectivity of the present
simple or as a complex offense in whatever stage of execution and mode of Constitution, until such time as the proposed 1991 LGC is
participation , is pending in court shall be suspended from office.

The Apocryphal Maggots:


Rainier, Chrisgel, Corina, Geoffry, Grace and Sylvie Blanche
The Flibbertigibbet Worms:
Golda, Gladys and Melyjane
CA VE AT: By simply r ea ding this re vi ew er a t the end o f the sem este r wi ll (hopef ully) guar antee you r pas sing this cou rse. D rink mode rat ely .
Amusin S C A r y
gly
THE C2005 LOCAL GOVERNMENT REVIEWER - 178 -
approved. The power of DLG Sec to remove local elective That remedies other than dismissal can be imposed by the
government officials is found in Sec60 and 61 of BP337. Disciplining Authority, and if the remedy is removal, one must
go to the courts?
What happens to the power of the President under the
Pablico vs. Villpando (2002) Constitution (general supervision over local government units)
to remove and discipline local officials as held in the cases of
Facts: Ganzon vs. CA and Aguinaldo vs. Mojica?
An administrative complaint against Mayor Villapando was filed What would be the procedure to ask for the removal of the local
with the Sangguniang Panlalawigan of Palawan for abuse of official with the courts?
authority and culpable violation of Constitution - Mayor allegedly
entered into a consultancy agreement with Orlando Tiape, a losing
mayoralty candidate which amounted to an appointment in a Recall – Local Government Code
government position w/in the prohibited 1 yr period under Art. IX-B
Sec. 6 of Constitution. Sec69. By Whom Exercised. — The power of recall for loss of
Sangguniang Panlalawigan found Mayor guilty and imposed the confidence shall be exercised by the registered voters of a local
penalty of dismissal from service. Affirmed by the Office of the government unit to which the local elective official subject to
President – so Vice Mayor Pablico became Mayor. such recall belongs.
CA declared the decision rendered as void.
Sec70. Initiation of the Recall Process. —
Held: Local legislative bodies and/or the Office of the President (a) Recall may be initiated by a preparatory recall
cannot validly impose the penalty of dismissal or removal from assembly or by the registered voters of the local government
service on erring local elective officials. unit to which the local elective official subject to such recall
belongs.
It is clear from Sec. 60 of LGCode that: xxx An elective local official (b) There shall be a preparatory recall assembly in every
may be removed from office on the grounds enumerated above by province, city, district, and municipality which shall be composed
order of the proper court. of the following:
In the case of Salalima v Guingona, the Office of the President is (1) Provincial level. — All mayors, vice-mayors, and
without any power to remove elected officials since such power is sanggunian members of the municipalities and
exclusively vested in the proper courts as expressly provided in component cities;
Sec. 60. (2) City level. — All punong barangay and sanggunian
barangay members in the city;
Under Art 24(b) IRR of LGC, it is provided that an elective official (3) Legislative District level. — In case where sangguniang
may be removed by order of proper court or the disciplining panlalawigan members are elected by district, all
committee (SB, SP and OP) whichever first acquires jurisdiction to elective municipal officials in the district; and in cases
the exclusion of the other. where sangguniang panlungsod members are elected by
district, all elective barangay officials in the district; and
HOWEVER, it was held in Salalima that this grant to the (4) Municipal level. — All punong barangay and
disciplining authority clearly beyond the power of the oversight sangguniang barangay members in the municipality.
committee that prepared the IRR as no IRR may alter, amend or (c) A majority of all the preparatory recall assembly
contravene the LGCode. members may convene in session in a public place and initiate a
Even Senator Pimentel expressed doubt as to the validity of this recall proceedings against any elective official in the local
rule. government unit concerned. Recall of provincial, city, or
There is very clear legislative intent to make the power of removal municipal officials shall be validly initiated through a resolution
a judicial prerogative as seen in the Senate deliberations. adopted by a majority of all the members of the preparatory
recall assembly concerned during its session called for the
The law on suspension or removal of public officials must be purpose.
strictly construed since what is involved is not just an ordinary (d) Recall of any elective provincial, city, municipal, or
public official but one chosen by people through their right of barangay official may also be validly initiated upon petition of at
suffrage. If there is unrestricted authority to suspend/remove public least twenty-five percent (25%) of the total number of registered
officials, there will be injustice and harm to public interest. voters in the local government unit concerned during the
election in which the local official sought to be recalled was
elected.
(1) A written petition for recall duly signed before the
election registrar or his representative, and in the
presence of a representative of the petitioner and a
representative of the official sought to be recalled and,
DanGat Notes: So what does this ruling imply?
and in a public place in the province, city, municipality, or
barangay, as the case may be, shall be filed with the

The Apocryphal Maggots:


Rainier, Chrisgel, Corina, Geoffry, Grace and Sylvie Blanche
The Flibbertigibbet Worms:
Golda, Gladys and Melyjane
CA VE AT: By simply r ea ding this re vi ew er a t the end o f the sem este r wi ll (hopef ully) guar antee you r pas sing this cou rse. D rink mode rat ely .
Amusin S C A r y
gly
THE C2005 LOCAL GOVERNMENT REVIEWER - 179 -
COMELEC through its office in the local government unit has been held to be indispensable for the proper administration
concerned. The COMELEC or its duly authorized of public affairs. It is frequently described as a fundamental right
representative shall cause the publication of the petition of the people in a representative democracy.
in a public and conspicuous place for a period of not less
than ten (10) days nor more than twenty (20) days, for II. History of Recall in the Philippines
the purpose of verifying the authenticity and It made its maiden appearance in the 1973 Constitution as
genuineness of the petition and the required percentage mandated in Sec2, ArtXI entitled Local Government where the
of voters. Batasang Pambansa shall enact a LGC xxx with an effective
(2) Upon the lapse of the aforesaid period, the COMELEC system of recall xxx
or its duly authorized representative shall announce the Batasang Pambansa enacted BP337 “LGC of 1983” where
acceptance of candidates to the position and thereafter Sec54(Chapter3) provided only one mode of initiating the recall
prepare the list of candidates which shall include the elections of local elective officials – by petition of at least 25% of
name of the official sought to be recalled. the total number of registered voters in the LGU concerned.
No instance in legal history where the power of recall was
Sec71. Election on Recall. — Upon the filing of a valid resolution exercised by the people.
or petition for recall with the appropriate local office of the Filipinos more than exercised their right of recall for they
COMELEC, the Commission or its duly authorized resorted to revolution and booted out of office the highest
representative shall set the date of the election on recall, which elective officials of the land. This led to its firm institutionalization
shall not be later than thirty (30) days after the filing of the in the 1987Constitution.
resolution or petition for recall in the case of the barangay, city, ArtXIII expressly recognized the Role and Rights of People’s
or municipal officials. and forty-five (45) days in the case of Organization, particularly in Secs15 and 16.
provincial officials. The official or officials sought to be recalled Sec3 of ArtX also reiterated mandate for Congress to enact a
shall automatically be considered as duly registered candidate LGC which “shall provide for a more responsive and
or candidates to the pertinent positions and, like other accountable local government structure instituted through a
candidates, shall be entitled to be voted upon. system of decentralization with effective mechanisms of recall,
initiative and referendum xxx.”
Sec72. Effectivity of Recall. — The recall of an elective local Congress enacted RA7160 (LGC of 1991) where it provided for
official shall be effective only upon the election and proclamation a second mode of initiating the recall process through a
of a successor in the person of the candidate receiving the preparatory recall assembly which in the provincial level is
highest number of votes cast during the election on recall. composed of all mayors, vice-mayors and sanggunian members
Should the official sought to be recalled receive the highest of the municipalities and component cities. See Secs69-74,
number of votes, confidence in him is thereby affirmed, and he LGC.
shall continue in office.
The legislative history of the present recall provisions will reveal
Sec73. Prohibition from Resignation. — The elective local that the idea of empowering a preparatory recall assembly to
official sought to be recalled shall not be allowed to resign while initiate the recall from office of local elective officials originated
the recall process is in progress. from House of Reps and not the Senate.
It was adopted for 2 principal reasons:
Sec74. Limitations on Recall. — (1) to diminish the difficulty of initiating recall thru the direct
(a) Any elective local official may be the subject of a action of the people; and
recall election only once during his term of office for loss of (2) to cut down on its expenses
confidence. They took note of undesirable fact that mechanism of initiating
(b) No recall shall take place within one (1) year from the recall by direct action of electorate was utilized only once in
date of the official's assumption to office or one (1) year Angeles City, Pampanga, but even this failed – Direct action
immediately preceding a regular local election. being too cumbersome, too expensive and almost impossible to
implement so the second more thru a PRA was added, and
Sec75. Expenses Incident to Recall Elections. — All expenses brushed aside the argument that the 2 nd mode may cause
incident to recall elections shall be borne by the COMELEC. For instability in the LGUs due to its imagined ease.
this purpose, there shall be included in the annual General Garcia vs. COMELEC (1993)
Appropriations Act a contingency fund at the disposal of the
COMELEC for the conduct of recall elections.
Evardone vs. COMELEC (1991)
I. Concept of Recall
It is a mode of removal of a public officer by the people before
Facts:
the end of his term of office.
Evardone was elected mayor during the 1988 local elections.
People’s prerogative to remove a public officer is an incident of
Apelado et al then filed a petition for the recall of Mayor Evardone
their sovereign power and in the absence of constitutional
with the office of the local Election Registrar. Pursuant to the rules
restraint, the power is implied in all governmental operations. It
under Resolution2272 (embodies rules and regulations on the

The Apocryphal Maggots:


Rainier, Chrisgel, Corina, Geoffry, Grace and Sylvie Blanche
The Flibbertigibbet Worms:
Golda, Gladys and Melyjane
CA VE AT: By simply r ea ding this re vi ew er a t the end o f the sem este r wi ll (hopef ully) guar antee you r pas sing this cou rse. D rink mode rat ely .
Amusin S C A r y
gly
THE C2005 LOCAL GOVERNMENT REVIEWER - 180 -
recall of elective provincial, city, and municipal officials), official’s assumption of office or 1year immediately preceding a
COMELEC issued Resolution90-0557 approving recommendation regular local election.
to hold signing of petition for recall on July14. Constitution has mandated a synchronized national and local
SC issued TRO on July12 which was received only on July15 by election on May 1992. To hold an election on recall approximately
the field agent – a day after completion of signing process. 7months before the regular local election will be violative of BP337.
COMELEC nullified signing process.

Held: Garcia vs. COMELEC (1993)


1. Resolution2272 is constitutional contrary to assertion that the
1987 Constitution has repealed BP337 and there was no LGC Facts:
at the time as basis for the promulgation of said resolution. July1993, some mayors, vice-mayors and members of the
ART XVIII, Section 3 of the 1987 Constitution provides that all Sangguniang Bayan of 12 municipalities of Bataan province met
existing laws not inconsistent with the Constitution shall remain and constituted themselves into a Preparatory Recall Assembly
operative until amended, repealed, or revoked. On the other hand, (PRA) in the Bagac town plaza to initiate the recall election of
the Local Government Code of 1991 will take effect only on Governor Garcia (elected governor of Bataan in May, 1992
January 1, 1992. Hence BP Blg. 337 is still the law applicable to elections) for “loss of confidence”.
the present case. COMELEC scheduled recall elections.
In the Constitutional Commission proceedings, the effectivity of BP Governor Garcia asserts the unconstitutionality of Sec70, LGC.
337 was expressly recognized prior to the enactment of a new SC Resolution:
Local Government Code. The requirement of notice is mandatory for it is
Chapter3 of BP337 provides for recall of local elective officials. indispensable in determining the collective wisdom of the
Sec59 expressly authorizes COMELEC to conduct and supervise members of PRA. Its non-observance is fatal to the validity
the process of and election on recall and in the exercise of such of the resolution to recall Garcia as Governor of Bataan.
powers, promulgate necessary rules and regulations. The due process clause of the Consti requiring notice as an
element of fairness is inviolable and should always be
Election Code contains no special provision on the manner of considered as part and parcel of every law in case of its
conducting elections of the recall of local officials. Any such silence.
election shall be conducted in the manner and under the rules on Need for notice to all members of PRA is also imperative for
special elections, unless otherwise provided by law or rule of the these members represent the different sectors of the
COMELEC. electorate.
Thus, pursuant to the rule-making power vested in COMELEC, it Resolution to recall should articulate the majority will of the
promulgated Resolution No. 2272 in 1990. members of the assembly but the majority will can be
genuinely determined only after all the members have been
2. The TRO issued by SC did not render nugatory the signing given a fair opportunity to express the will of their
process of the petition for recall. constituents
SC had issued a TRO on July12 but signing of petition for recall In accord with the SC Resolution, Notice of Session was again
took place just the same on scheduled date through no fault of sent to the members of PRAC.
COMELEC and Apelado et al. Signing process was undertaken by Garcia asserts that Sec70, LGC is unconstitutional.
constituents of Sulat Municipality and its Election Registrar in good
faith and without knowledge of the TRO earlier issued. Held: Sec70 of LGC is constitutional.
As attested by Election Registrar, about 34% signed the petition for The presumption of validity rests on the respect due to the wisdom,
recall, and there is no turning back the clock (Paredes vs. Exec integrity, and the patriotism of the legislative, by which the law is
Sec.) passed, and the Chief Executive, by whom the law is approved.
WON the electorate of Sulat Municipality has lost confidence in the Garcia et al do not point to any constitutional provision that will
incumbent mayor is a political question – belonging to the realm of sustain their contention – for surely, there is nothing in there that
politics where only the people who are the judge. will remotely suggest that the people have the “sole and exclusive
“Loss of confidence is the formal withdrawal by an electorate of right to decide on whether to initiate a recall proceeding.”
their trust in a person’s ability to discharge his office previously 1. Consti did not provide for any mode of initiating recall
bestowed on him by the same electorate.” elections.
CAB: Constituents have made a judgment and their will to recall 2. It did not prohibit the adoption of multiple modes of initiating
Evardone (as mayor) has already been ascertained and must be recall elections.
afforded the highest respect.
The signing process is valid and has legal effect. 3. Mandate given by Sec3 of ArtX of Consti is for Congress to
“enact a LGC which shall provide for a more responsive and
HOWEVER, recall is no longer possible because of the limitation accountable local government structure through a system of
provided in Sec55(2) of BP33 – no recall within 2years from date of decentralization with effective mechanisms of recall, initiative,
and referendum xxx”

The Apocryphal Maggots:


Rainier, Chrisgel, Corina, Geoffry, Grace and Sylvie Blanche
The Flibbertigibbet Worms:
Golda, Gladys and Melyjane
CA VE AT: By simply r ea ding this re vi ew er a t the end o f the sem este r wi ll (hopef ully) guar antee you r pas sing this cou rse. D rink mode rat ely .
Amusin S C A r y
gly
THE C2005 LOCAL GOVERNMENT REVIEWER - 181 -
- Congress was clearly given the power to choose the component cities are made members of the PRA at the
provincial level
effective mechanisms of recall as its discernment
- its membership is not apportioned to political parties –
dictates
no significance is given to the political affiliation of its
- Power given was to select which among the means and
members
methods of initiating recall elections are effective to carry
out the judgment of the electorate, and it was not 2. PRA at the provincial level includes all the elected officials in
straightjacketed to one particular mechanism of initiating the province concerned
recall elections. - considering their number, the greater probability is that no
- Consti requires only that the mechanisms chosen (one one political party can control its majority
3. Sec69, LGC provides that the only ground to recall is loss of
or many) be effective.
confidence of the people
- Congress deemed it wise to enact the alternative mode
- The members of PRAC are there not in representation of
to supplement the former mode by direct action, and the
their political parties but as reps of the peoples.
Court cannot supplant this judgment by Congress in
- By necessary implication, loss of confidence cannot be
respect of the principle of separation of powers.
premised on mere differences in political party affiliation
- Choice may be erroneous but the remedy against a bad
- Consti even encourages multi-party system to nurture
law is to seek its amendment or repeal by the legislative.
the democratic system
Initiation by the PRAC is also initiation by the people, albeit done  Fear that a PRA may be dominated by a political party and that
it may use its power to initiate the recall of officials of opposite
indirectly through their representatives  this act by the people
political persuasions is not a ground to strike down the law as
through their elected representatives is not constitutionally
unconstitutional.
impermissible as seen in the task of drafting the Constitution which
is delegated to their representatives (either by constitutional  Moreover, law instituted safeguards to assure that the initiation
convention or as a congressional constituent assembly). of the recall process by a PRA will not be corrupted by extraneous
Initiation of recall process is a lesser act and there is no rhyme or influences. Its diverse and distinct composition guarantees that all
reason why it cannot be entrusted to and exercised by the elected the sectors of the electorate province shall be heard. Following are
representatives of the people. required for the validity of resolution.
- notice to all members is a condition sine qua non to the
PRA resolution of recall is not the recall itself. validity of its proceedings
The PRA resolution merely starts the process – only a part of the
process, and not the whole.
- Law also requires a qualified majority of all the PRA members
This is self-evident because a PRA resolution of recall that is not to convene in session and in a public place
submitted to the COMELEC for validation will not recall its subject - Also, the recall resolution by majority must be adopted during
officials. its session called for the purpose
Likewise, a PRA resolution of recall that is rejected by the people  Furthermore, it cannot be claimed that the PRA members voted
in the election called for the purpose bears no effect whatsoever. along narrow political lines.
The initiatory resolution merely sets the stage for the official - Neither COMELEC nor SC made a judicial inquiry as to the
concerned to appear before the tribunal of the people so he can reason that led the members of the said recall assembly to
justify why he should be allowed to continue in office. cast a vote against Garcia
Before the people render their sovereign judgment, the official - Pimentel in his book stressed that the substantive content of
concerned remains in office but his right to continue in office is a vote of lack of confidence is beyond any inquiry – a political
subject to question. question (as held in Evardone vs COMELEC)
 This is clear in Sec72, LGC which explicitly states that “the
recall of an elective local official shall be effective only upon the The proposal will still be passed upon by the sovereign electorate
election and proclamation of a successor in the person of the of Bataan – yet to be expressed. It is premature to conclude that
candidate receiving the highest number of votes cast during the the will has been subverted.
election on recall.” If electorate re-elects him, the proposal to recall is rejected. If they
do not, then he has lost the confidence of the people which he
The fear expressed is that the members of PRAC may inject once enjoyed
political color in their decision as they may initiate recall
proceedings only against their political opponents especially those Conclusion: The alternative mode of initiating recall proceedings
belonging to the minority. thru a PRA is an innovative attempt by Congress to remove
Careful reading of the law will show that it does not give an impediments to the effective exercise by the people of their
asymmetrical treatment to locally elected officials belonging to sovereign power to check the performance of their elected officials.
the political minority. The power to determine this mode was specifically given to
1. The politically neutral composition of the PRA under Sec70(b) Congress and is not proscribed by the Constitution.
where all mayors, v-m, sangg members of munic and

The Apocryphal Maggots:


Rainier, Chrisgel, Corina, Geoffry, Grace and Sylvie Blanche
The Flibbertigibbet Worms:
Golda, Gladys and Melyjane
CA VE AT: By simply r ea ding this re vi ew er a t the end o f the sem este r wi ll (hopef ully) guar antee you r pas sing this cou rse. D rink mode rat ely .
Amusin S C A r y
gly
THE C2005 LOCAL GOVERNMENT REVIEWER - 182 -
Paras vs. COMELEC (1996) official sought to be recalled will be contested and be filled by the
electorate.
Facts: However, recall is no longer possible in CAB because of the
Petition for recall of Paras as Punong Barangay (elected last 1994 limitation under Sec74(b) – the next regular election involving the
regular barangay elections) was filed by the registered voters of barangay office concerned is barely 7months away (scheduled on
the barangay. COMELEC approved the petition and scheduled the May1997).
petition signing on October14, 1995 and set the recall election on
Nov13, 1995. CONCURRING: Davide
At least 29.30% of registered voters signed the petition (above the SK election is not a regular local election for purposes of recall
required 25%). under Sec74, LGC.
Paras opposed so recall election was deferred by COMELEC to The term “regular local election” must be confined to the regular
Dec16, 1995. election of local elective officials15, as distinguished from the
COMELEC rescheduled recall election on Jan13, 1996. regular election of national officials (President, VP, Senators and
Congressmen).
Held: There can still be a recall election even with the 4months The officials enumerated under footnote are the only local elective
that separate the recall election from the upcoming SK elections. officials deemed recognized by Sec2(2) of ArtIX-C of Constitution
(COMELEC’s power and exclusive original jurisdiction over all
Evident intent of Sec74 of LGC14 is to subject an elective local contests relating to elections, returns, and qualifications of all
official to recall election once during his term of office. elective regional, provincial, and city officials, and appellate
Paragraph (b) construed together with paragraph (a) merely jurisdiction over all contests involving elective municipal officials
designates the period when such elective official may be subject of decided by RTCs of general jurisdiction, or involving elective
a recall election, that is, during the 2nd year of his term of office. barangay officials decided by trial courts of limited jurisdiction).
Thus, subscribing to Paras’ interpretation of the phrase regular A regular election (local or national) can only refer to an election
local election to include the SK election will unduly circumscribe participated in by those who possess the right of suffrage, are not
the novel provision of the LGC on recall, a mode of removal of otherwise disqualified by law, and who are registered voters.
public officers by initiation of the people before the end of his term. One of the requirements for exercise of suffrage under Sec1, ArtV
 If the SK election were to be deemed within the purview of the of Consti is that the person must be at least 18years of age, and
phrase “regular local election” (to be held every 3years from one requisite before he can vote is that he be a registered voter
May1996), then no recall election can be conducted rendering pursuant to the rules on registration prescribed in the Omnibus
inutile the recall provision of the LGC. Election Code (Sec113-118)
Under the law, SK includes the youth with ages ranging from 15-21
It is a basic precept of stat con that a statute should be interpreted (Sec424, LGC). Accordingly, they include many who are not
in harmony with the Constitution. qualified to vote in a regular election – those from ages 15 to less
Interpretation of Sec74(b), LGC should not be in conflict with than 18. In no manner then may SK elections be considered a
Consti mandate of Sec3, ArtX to “enact a LGC which shall provide regular election.
for a more responsive and accountable local gov structure SK is nothing more than a youth organization. Although fully
instituted through a system of decentralization with effective recognized in LGC and vested with certain powers and functions,
mechanisms of recall, initiative, and referendum xxx” its elective officials have not attained the status of local elective
officials.
Rrecall election is potentially disruptive of the normal working of Angobung vs. COMELEC (1997)
the LGU necessitating additional expenses, hence the prohibition
against the conduct of recall election 1year immediately preceding Facts:
the regular local election. De Alban filed with the Local Election Registrar a Petition for Recall
The proscription is due to the proximity of the next regular election against Angobung (elected Mayor in 1995), who received a copy of
for the office of the local elective official concerned. this petition. Subsequently said petition was forwarded to the
Electorate could choose the official’s replacement in the said Regional Office and then to the main office of COMELEC for
election who certainly has a longer tenure in office than a approval.
successor elected through a recall election. Acting on the petition, Deputy Executive Director for Operations
Joson submitted to the COMELEC En Banc, a Memorandum
Therefore, it would be more in keeping with the intent of the recall recommending approval of the petition for recall (note: recall
provision of the Code to construe regular local election as one petition was signed only by 1 registered voter-respondent De
referring to an election where the office held by the local elective Alban) and its signing by other qualified voters in order to garner at
least 25% of the total number of registered voters as required by
14
Sec74: Limitations on Recall – Section 69(d) of 1991 LGC. COMELEC en banc issued the
(a) Any elective official may be the subject of a recall election only once during his
15
term of office for loss of confidence. (Prov. Gov, Vice-Gov, Mayors, Vice-Mayors, members of Sanggunians, punong
(b) No recall shall take place within 1year from the date of the official’s barangays and members of sangguniang barangays, and elective regional
assumption to office or 1year immediately preceding a regular local election. officials of autonomous regions)

The Apocryphal Maggots:


Rainier, Chrisgel, Corina, Geoffry, Grace and Sylvie Blanche
The Flibbertigibbet Worms:
Golda, Gladys and Melyjane
CA VE AT: By simply r ea ding this re vi ew er a t the end o f the sem este r wi ll (hopef ully) guar antee you r pas sing this cou rse. D rink mode rat ely .
Amusin S C A r y
gly
THE C2005 LOCAL GOVERNMENT REVIEWER - 183 -
assailed resolution approving recall petition and set further signing
of registered voters. Held:
1. COMELEC had conducted an investigation on the notices
Held: contrary to Solicitor General’s claim.
1. COMELEC Resolution is not violative of the 1year ban for The matter of validity of notices to the members of the Preparatory
being scheduled within 1year from 1997 Barangay Elections. Recall Assembly was sufficiently considered by the respondent
In construing the meaning of the term, "regular local election" in Commission.
Section 74 of the Local Government Code of 1991 which provides In response to petitioner's request for a technical examination of
that "no recall shall take place within one (1) year . . . immediately the recall documents, the COMELEC directed its Election Records
preceding a regular local election," SC ruled in PARAS v and Statistics Department (ERSD) to resolve the matter of notices
COMELEC, that for the time bar to apply, the approaching regular sent to the Preparatory Recall Assembly members. The ERSD in
local election must be one where the position of the official to be turn performed its task and reported its findings to the COMELEC.
recalled, is to be actually contested and filled by the electorate. Needless to state, the issue of propriety of the notices sent to the
PRA members is factual in nature, and the determination of the
2. The COMELEC Resolution approving the Petition for Recall same is therefore a function of the COMELEC. In the absence of
signed by just one person is invalid for failure to comply with patent error, or serious inconsistencies in the findings, the Court
the statutory 25% minimum. should not disturb the same. The factual findings of the
In Sanchez and Evardone, the COMELEC-prescribed procedure of COMELEC, based on its own assessments and duly supported by
(1) allowing the recall petition to be filed by at least one person or gathered evidence, are conclusive upon the court, more so, in the
by less than 25% of the total number of registered voters and then absence of a substantiated attack on the validity of the same.
(2) inviting voters to sign said petition on a date set for that Moreover, to order the COMELEC to repeat the process of
purpose, was never put to issue. determining the notices' propriety would be sanctioning a recycling
Section 69 (d) of 1991 LGC is plain and unequivocal as to what of administrative functions, entailing added cost and waste of
initiates recall proceedings: only a petition of at least 25% of the effort.
total number of registered voters, may validly initiate recall
proceedings. This is understandable, since the signing of the 2. It is not the Liga ng mga Barangay which initiated the recall
petition is statutorily required to be undertaken "before the election and convened as the PRA.
registrar or his representative, and in the presence of a The Liga ng mga Barangay is undoubtedly an entity distinct from
representative of the official sought to be recalled, and in a public the Preparatory Recall Assembly. It just so happens that the
place in the . . . municipality . . ." Hence, while the initiatory recall personalities representing the barangays in the Liga are the very
petition may not yet contain the signatures of at least 25% of the members of the Preparatory Recall Assembly, the majority of
total number of registered voters, the petition must contain the whom met and voted in favor of the resolution calling for the recall
names of at least 25% of the total number of registered voters in of Mayor Malonzo, after deliberation reported, in the record in
whose behalf only one person may sign the petition in the accordance with the existing law.
meantime. Thus, the Punong Barangays and Sangguniang Barangay
Our legislators did not peg the voter requirement at 25% out of members conveyed and voted as members of the Preparatory
caprice or in a vacuum. While recall was intended to be an Recall Assembly of the City of Caloocan, and not as members of
effective and speedy remedy to remove an official who is not giving the Liga ng mga Barangay. The recall proceedings, therefore,
satisfaction to the electorate regardless of whether or not he is cannot be denied merit on this ground.
discharging his full duty to the best of his ability and as his
conscience dictates, it is a power granted to the people who, in The charges of graft and corruption, violence and irregularities,
concert, desire to change their leaders for reasons only they, as a before and during the session of the preparatory recall assembly
collective, can justify. are largely uncorroborated, and cannot override the substantiated
findings of the respondent COMELEC.

Jariol vs. COMELEC (1997)


Malonzo vs. COMELEC (1997)
Facts:
Facts: COMELEC issued Resolution No 2879 which adopted the calendar
PRA Resolution expressing loss of confidence in Mayor Malonzo of activities for the recall election of the mayor, vice-mayor and 6
and calling for initiation of recall proceedings against him was filed members of the Sangguniang Bayan of Basilisa.
with COMELEC. Petitioners filed present action assailing the COMELEC
Malonzo filed a Petition with the respondent Commission alleging, Resolution. They claim that the recall election is scheduled within
principally, that the recall process was deficient in form and 1year immediately preceding a regular election of barangay
substance, and therefore, illegally initiated. officials.
The COMELEC found the petition devoid of merit and declared the
recall proceedings to be in order. Held: COMELEC did not commit Grave Abuse of Discretion.

The Apocryphal Maggots:


Rainier, Chrisgel, Corina, Geoffry, Grace and Sylvie Blanche
The Flibbertigibbet Worms:
Golda, Gladys and Melyjane
CA VE AT: By simply r ea ding this re vi ew er a t the end o f the sem este r wi ll (hopef ully) guar antee you r pas sing this cou rse. D rink mode rat ely .
Amusin S C A r y
gly
THE C2005 LOCAL GOVERNMENT REVIEWER - 184 -
a reasonable basis for judging the performance of an elective
Petitioners failed to exhaust administrative remedies. local official.
Petitioners should have 1st moved for reconsideration before filing As long as the election is held outside the 1 yr period, the
the special civil action for certiorari under Rule 65 ROC. preliminary proceedings to initiate recall can be held even
COMELEC performed a purely administrative function when it before said period ends.
promulgated the resolution. A party aggrieved thereby must not c. To construe that the term “recall” includes the convening of
merely initiate the prescribed administrative procedure to obtain the PRA would be to unduly restrict the constitutional right of
relief, but must also pursue it to its appropriate conclusion before free speech and assembly of its members. The people
seeking judicial intervention. This is to give the administrative cannot be asked to decide on the performance of their
agency the opportunity to decide the matter by itself correctly and officials on the day of the election itself.
prevent unnecessary and premature resort to the court. As the recall election is set on April 15, there is no bar to its being
held on that date.
COMELEC cannot be said to have acted with GAD when it issued
the decision relying on the Report of its Municipal Election Officer. Justice Puno’s dissent –
1. purpose of the 1yr. period in par (b) is to provide the local
The scheduled barangay election on May 12 1997 is not the official concerned a period of repose during which his
regular election contemplated in Sec 74(b) of the LGC whose attention should not be distracted by any impediment.
conduct is the basis for computing the 1-yr. prohibited period. Majority – The law cannot provide for a period of honeymoon or
“Regular local election” means the election where the office moratorium in politics.
held by the local elective official sought to be recalled could
be contested and be filled by the electorate. 2. judgments of PRAs are not as politically unassailable as
Election on May 12, 1997 is the election for barangay officials. recalls initiated directly by the people.
The 1 year prohibitive period should be counted from the date of Majority – it is no disparagement of the PRA that in the ensuing
election for mayor, v-mayor and SB members which are the election the local official whose recall is sought is actually
positions held by the petitioners. reelected.
- the question here is not whether recalls initiated by 25% of the
voters are better.
Claudio vs. COMELEC; PRA vs. COMELEC (2000)
2. The phrase “regular local election” in Sec74(b) does not
FACTS: include the election period for that regular election.
May 1999 - Chairs of several barangays in Pasay gathered to The law is unambiguous that no recall shall take place within 1 yr.
discuss the possibility of filing a petition for recall against Mayor immediately preceding a regular local election.
Claudio for loss of confidence. Had Congress intended this limitation to refer to the campaign
Ad hoc committee was formed for the purpose of convening the period, it could have expressly said so.
PRA. Advincula was designated as chair. To uphold Petitioner’s interpretation would be to severely limit and
May 29 – members of the PRA adopted Resolution No. 01 which decrease the period during which a recall election may be held
provides for the initiation of recall proceedings against Mayor from 9 months and 15 days to 8 months.
Claudio.
July 2 – petition for recall filed with COMELEC. It was opposed. 3. The Recall Resolution was signed by a majority of the PRA
COMELEC granted petition for recall. and was duly verified.
Claim is being raised for the 1 st time. Although the word
Held: attendance appears at the top of the page, it is apparent that it was
1. The word “recall” in Sec74(b) of LGC does not include the written by mistake because it was crossed out by 2 parallel lines.
convening of the PRA and the filing of the recall resolution. It is absurd to believe that the 74 members of the PRA who signed
The term “recall” as used in Sec 74 LGC refers to the election itself the recall resolution signified their attendance twice.
by means of which voters decide whether they should retain their
local official or elect his replacement. J. PUNO dissent:
a. Since the power vested on the electorate is not the power to To allow early recall initiative is to encourage divisive, expensive,
initiate recall proceedings but the power to elect an official wasteful politics.
into office, the limitations in Sec 74 cannot be deemed to The reasoning of the majority is based on the misleading
apply to the entire recall proceedings. perception that the only participation of the people in recall is on
There is therefore no legal limit on the no. of times such election day when they cast their votes. But their role is not so
PRAs may be held. These are merely preliminary steps for limited as they participate in the initiation of the recall process.
the purpose of initiating a recall. It is not the recall itself. Initiation by the PRA is also initiation by the people albeit done
If these preliminary steps do not produce a decision by the indirectly through their representatives.
electorate, then the prohibition in Sec 74 does not apply. It does not follow that during the 1-yr waiting period, the people’s
b. The purpose of the limitation that no recall shall take place freedom of speech and assembly are suspended. These rights are
within 1 yr. from the date of assumption of office is to provide in no way restricted during said period.

The Apocryphal Maggots:


Rainier, Chrisgel, Corina, Geoffry, Grace and Sylvie Blanche
The Flibbertigibbet Worms:
Golda, Gladys and Melyjane
CA VE AT: By simply r ea ding this re vi ew er a t the end o f the sem este r wi ll (hopef ully) guar antee you r pas sing this cou rse. D rink mode rat ely .
Amusin S C A r y
gly
THE C2005 LOCAL GOVERNMENT REVIEWER - 185 -
The limitations in Sec 74 should be strictly followed considering the
short 3-yr term of office of local officials. The majority decision Even if the PRA were to reconvene to adopt another resolution for
failed to recognize the need for stability of a public office. the recall of Navarro as mayor, the same would still not prosper in
view of Sec. 74 (b): No recall shall take place within 1 year from
J. KAPUNAN dissent and separate opinion: the date of the official’s assumption to office or 1 year immediately
Recall is a process which begins once the PRA makes its 1 st preceding a regular local election.
affirmative act towards the recall of the elective local official. The There is no more allowable time within which to hold recall
Garcia case cited by the majority even describes recall as a elections. Navarro assumed office on Oct. 1999. One year after
process. her assumption of office as mayor will be Oct. 2000 which is
Since our form of govt. is a representative democracy, it cannot be already within the 1-year prohibited period immediately preceding
claimed that the initiation of the recall process by the PRA is not an the next regular election in May 2001.
initiation by the people.
Senator Pimentel, the main author of the LGC, used the phrase DanGat Notes: If absence of mayor is only by virtue of a
“move to recall” in his book which connotes a progressive course temporary vacancy, resolution of PRA may still be valid since the
of action or a step-by-step process. Acting Mayor would still be Vice-Mayor after resumption of
An exercise of the right to peaceably assemble and exchange office of the Mayor.
views about the governance of local officials does not violate the
proscription in Sec 74

Afiado vs. COMELEC (2000)

Facts:
Election and proclamation of Miranda as Mayor of Santiago City Hunting the Elusive G-Spot (Part5):
was annulled so duly-elected Vice-Mayor Navarro became the new
Mayor by virtue of the law of succession. Many women find G-Spot stimulation easier and more
PRA was constituted. It then passed and adopted PRA Resolution pleasurable in positions other than on their backs. Try rolling
No.1 for the recall of Vice-Mayor Navarro. over on your stomach or getting on all fours. Women don't
usually enjoy penetration until they are somewhat aroused.
Held: An elective official who became city mayor by legal Engage in whatever foreplay you find arousing: touching,
succession cannot be the subject of a recall election by virtue of a kissing, stroking, oral sex, talking ... whatever works for you. G-
PRA Resolution adopted when the elective official was still vice- Spot stimulation should come at the middle or the end of your
mayor. sexual play, not at the beginning. Use lubrication when you're
The assumption by legal succession of Navarro as the new
ready for digital penetration, even if your body is already
mayor is a supervening event which rendered the recall
proceeding against her moot and academic. producing natural lubrication, which can often run dry at an
The person subject of the recall process is a specific elective inopportune moment. You might also consider purchasing one
official in relation to her specific office. The said resolution is of the many sex toys designed specifically for G-Spot
replete with statements, which leave no doubt that the purpose of stimulation.
the assembly was to recall Navarro as VM for her official acts as That's all there is to it. No magic. Some women can have an
VM. orgasm from G-Spot stimulation alone. Some say it increases the
o Title suggests that the recall is intended for the incumbent strength of their orgasms or allows them to be multi-orgasmic.
VM. A few even say that stimulation leads to ejaculation. Others
o The 3rd para. Recounted “the official acts of VM Navarro that actively dislike stimulation of the area. Experiment and see
brought forth the loss of confidence in her capacity and what feels good to you or your partner. Most of all, have fun.
fitness to discharge the duties and to perform the functions of And don't forget -- if you enjoy G-Spot stimulation, be sure to
her public office.” teach your partner how to find it and what to do with it! (end)
o Because of such acts, the assembly “RESOLVED to invoke
the rescission of the electoral mandate of the incumbent VM.”
The intent of the PRA as expressed in the resolution is to remove Human Resources Development – LGC
Navarro as VM for they already lost their confidence in her by
reason of her official acts as such.
Sec76. Organizational Structure and Staffing Pattern. — Every
To recall then, Navarro when she is already the incumbent
local government unit shall design and implement its own
mayor is to deviate from the expressed will of the PRA. She
organizational structure and staffing pattern taking into
had already vacated the office of VM. Having thus succeeded to
consideration its service requirements and financial capability,
the position of mayor, Navarro was placed beyond the reach
subject to the minimum standards and guidelines prescribed by
of the effects of the PRA resolution.
the Civil Service Commission.

The Apocryphal Maggots:


Rainier, Chrisgel, Corina, Geoffry, Grace and Sylvie Blanche
The Flibbertigibbet Worms:
Golda, Gladys and Melyjane
CA VE AT: By simply r ea ding this re vi ew er a t the end o f the sem este r wi ll (hopef ully) guar antee you r pas sing this cou rse. D rink mode rat ely .
Amusin S C A r y
gly
THE C2005 LOCAL GOVERNMENT REVIEWER - 186 -
increase shall have expired: Provided, further, That the increase
Sec77. Responsibility for Human Resources and in compensation of the appointive officials and employees shall
Development. — The chief executive of every local government take effect as provided in the ordinance authorizing such
unit shall be responsible for human resources and development increase: Provided, however, That said increases shall not
in his unit and shall take all personnel actions in accordance exceed the limitations on budgetary allocations for personal
with the Constitutional provisions on civil service, pertinent laws, services provided under Title Five, Book II of this Code:
and rules and regulations thereon, including such policies, Provided, finally, That such compensation may be based upon
guidelines and standards as the Civil Service Commission may the pertinent provisions of R.A. No 6758, otherwise known as
establish: Provided, That the local chief executive may employ the "Compensation and Position Classification Act of 1989".
emergency or casual employees or laborers paid on a daily The punong barangay, the sangguniang barangay member, the
wage or piecework basis and hired through job orders for local sangguniang kabataan chairman, the barangay treasurer, and
projects authorized by the sanggunian concerned, without need the barangay secretary shall be entitled to such compensation,
of approval or attestation by the Civil Service Commission: allowances, emoluments, and such other privileges as provided
Provided, further, That the period of employment of emergency under Title One Book III of this Code.
or casual laborers as provided in this Section shall not exceed Elective local officials shall be entitled to the same leave
six (6) months. privileges as those enjoyed by appointive local officials,
The Joint Commission on Local Government Personnel including the cumulation and commutation thereof.
Administration organized pursuant to Presidential Decree
Numbered Eleven Hundred thirty-six (P.D. No. 1136) is hereby Sec82. Resignation of Elective Local Officials. —
abolished and its personnel, records, equipment and other (a) Resignations by elective local officials shall be deemed
assets transferred to the appropriate office in the Civil Service effective only upon acceptance by the following authorities:
Commission. (1) The President, in the case of governors, vice-governors,
and mayors and vice-mayors of highly urbanized cities
Sec78. Civil Service Law, Rules and Regulations, and Other and independent component cities;
Related Issuances. — All matters pertinent to human resources (2) The governor, in the case of municipal mayors,
and development in local government units shall be governed by municipal vice-mayors, city mayors and city vice-mayors
the civil service law and such rules and regulations and other of component cities;
issuances promulgated pursuant thereto, unless otherwise (3) The sanggunian concerned, in the case of sanggunian
specified in this Code. members; and
(4) The city or municipal mayor, in the case of barangay
Sec79. Limitation to Appointments. — No person shall be officials.
appointed in the career service of the local government if he is (b) Copies of the resignation letters of elective local
related within the fourth civil degree of consanguinity or affinity officials, together with the action taken by the aforesaid
to the appointing or recommending authority. authorities, shall be furnished the Department of the Interior and
Local Government.
Sec80. Public Notice of Vacancy; Personnel Selection Board. (c) The resignation shall be deemed accepted if not acted
(a) Whenever a local executive decides to fill a vacant upon by the authority concerned within fifteen (15) days from
career position, there shall be posted notices of the vacancy in receipt thereof.
at least three (3) conspicuous public places in the local (d) Irrevocable resignations by sanggunian members
government unit concerned for a period of not less than fifteen shall be deemed accepted upon presentation before an open
(15) days. session of the sanggunian concerned and duly entered in its
(b) There shall be established in every province, city or records: Provided, however, That this subsection does not apply
municipality a personnel selection board to assist the local chief to sanggunian members who are subject to recall elections or to
executive in the judicious and objective selection or personnel cases where existing laws prescribed the manner of acting upon
for employment as well as for promotion, and in the formulation such resignations.
of such policies as would contribute to employee welfare.
(c) The personnel selection board shall be headed by the Sec83. Grievance Procedure. — In every local government unit,
local chief executive, and its members shall be determined by the local chief executive shall establish a procedure to inquire
resolution of the sanggunian concerned. A representative of the into, act upon, resolve or settle complaints and grievances
Civil Service Commission, if any, and the personnel officer of the presented by local government employees.
local government unit concerned shall be ex officio members of
the board. Sec84. Administrative Discipline. — Investigation and
adjudication of administrative complaints against appointive
Sec81. Compensation of Local Officials and Employees. — local officials and employees as well as their suspension and
The compensation of local officials and personnel shall be removal shall be in accordance with the civil service law and
determined by the sanggunian concerned: Provided, That the rules and other pertinent laws. The results of such administrative
increase in compensation of elective local officials shall take investigations shall be reported to the Civil Service Commission.
effect only after the terms of office of those approving such

The Apocryphal Maggots:


Rainier, Chrisgel, Corina, Geoffry, Grace and Sylvie Blanche
The Flibbertigibbet Worms:
Golda, Gladys and Melyjane
CA VE AT: By simply r ea ding this re vi ew er a t the end o f the sem este r wi ll (hopef ully) guar antee you r pas sing this cou rse. D rink mode rat ely .
Amusin S C A r y
gly
THE C2005 LOCAL GOVERNMENT REVIEWER - 187 -
Sec85. Preventive Suspension of Appointive Local Officials thing of value is to be transferred, directly or indirectly,
and Employees. — out of the resources of the local government unit to such
(a) The local chief executives may preventively suspend for a person or firm;
period not exceeding sixty (60) days and subordinate official or (2) Hold such interests in any cockpit or other games
employee under his authority pending investigation if the charge licensed by a local government unit;
against such official or employee involves dishonesty, (3) Purchase any real estate or other property forfeited in
oppression or grave misconduct or neglect in the performance favor of such local government unit for unpaid taxes or
of duty, or if there is reason to believe that the respondent is assessment, or by virtue of a legal process at the
guilty of the charges which would warrant his removal from the instance of the said local government unit;
service. (4) Be a surety for any person contracting or doing business
(b) Upon expiration of the preventive suspension, the with the local government unit for which a surety is
suspended official or employee shall be automatically reinstated required; and
in office without prejudice to the continuation of the (5) Possess or use any public property of the local
administrative proceedings against him until its termination. If government unit for private purposes.
the delay in the proceedings of the case is due to the fault, (b) All other prohibitions governing the conduct of
neglect or request of the respondent, the time of the delay shall national public officers relating to prohibited business and
not be counted in computing the period of suspension herein pecuniary interest so provided for under Republic Act Numbered
provided. Sixty-seven thirteen (R.A. No. 6713) otherwise known as the
"Code of Conduct and Ethical Standards for Public Officials and
Sec86. Administrative Investigation. — In any local government Employees" and other laws shall also be applicable to local
unit, administrative investigation may be conducted by a person government officials and employees.
or a committee duly authorized by the local chief executive. Said
person or committee shall conduct hearings on the cases Sec90. Practice of Profession. —
brought against appointive local officials and employees and (a) All governors, city and municipal mayors are
submit their findings and recommendations to the local chief prohibited from practicing their profession or engaging in any
executive concerned within fifteen (15) days from the conclusion occupation other than the exercise of their functions as local
of the hearings. The administrative cases herein mentioned shall chief executives.
be decided within ninety (90) days from the time the respondent (b) Sanggunian members may practice their professions,
is formally notified of the charges. engage in any occupation, or teach in schools except during
session hours: Provided, That sanggunian members who are
Sec87. Disciplinary Jurisdiction. — Except as otherwise also members of the Bar shall not:
provided by law, the local chief executive may impose the (1) Appear as counsel before any court in any civil case
penalty of removal from service, demotion in rank, suspension wherein a local government unit or any office, agency, or
for not more than one (1) year without pay, fine in an amount not instrumentality of the government is the adverse party;
exceeding six (6) months salary, or reprimand and otherwise (2) Appear as counsel in any criminal case wherein an
discipline subordinate officials and employees under his officer or employee of the national or local government is
jurisdiction. If the penalty imposed is suspension without pay for accused of an offense committed in relation to his office.
not more than thirty (30) days, his decision shall be final. If the (3) Collect any fee for their appearance in administrative
penalty imposed is heavier than suspension of thirty (30) days, proceedings involving the local government unit of which
the decision shall be appealable to the Civil Service he is an official; and
Commission, which shall decide the appeal within thirty (30) (4) Use property and personnel of the government except
days from receipt thereof. when the sanggunian member concerned is defending
the interest of the government.
Sec88. Execution Pending Appeal. — An appeal shall not (c) Doctors of medicine may practice their profession
prevent the execution of a decision of removal or suspension of even during official hours of work only on occasions of
a respondent-appellant. In case the respondent-appellant is emergency: Provided, That the officials concerned do not derive
exonerated, he shall be reinstated to his position with all the monetary compensation therefrom.
rights and privileges appurtenant thereto from the time he had
been deprived thereof. Sec91. Statement of Assets and Liabilities. — (a) Officials and
employees of local government units shall file sworn statements
Sec89. Prohibited Business and Pecuniary Interest. — of assets, liabilities and net worth, lists of relatives within the
(a) It shall be unlawful for any local government official or fourth civil degree of consanguinity or affinity in government
employee, directly or indirectly, to: service, financial and business interests, and personnel data
(1) Engage in any business transaction with the local sheets as required by law.
government unit in which he is an official or employee or
over which he has the power of supervision, or with any Sec92. Oath of Office. — (a) All elective and appointive local
of its authorized boards, officials, agents, or attorneys, officials and employees shall, upon assumption to office,
whereby money is to be paid, or property or any other subscribe to an oath or affirmation of office in the prescribed

The Apocryphal Maggots:


Rainier, Chrisgel, Corina, Geoffry, Grace and Sylvie Blanche
The Flibbertigibbet Worms:
Golda, Gladys and Melyjane
CA VE AT: By simply r ea ding this re vi ew er a t the end o f the sem este r wi ll (hopef ully) guar antee you r pas sing this cou rse. D rink mode rat ely .
Amusin S C A r y
gly
THE C2005 LOCAL GOVERNMENT REVIEWER - 188 -
form. The oath or affirmation of office shall be filed with the office emergency or crisis or when the travel involves the use of public
of the local chief executive concerned. A copy of the oath or funds, permission from the Office of the President shall be
affirmation of office of all elective and appointive local officials secured.
and employees shall be preserved in the individual personal (d) Field officers of national agencies or offices assigned
records file under the custody of the personnel office, division, or in provinces, cities, and municipalities shall not leave their
section of the local government unit concerned. official stations without giving prior written notice to the local
chief executive concerned. Such notice shall state the duration
Sec93. Partisan Political Activity. — No local official or employee of travel and the name of the officer whom he shall designate to
in the career civil service shall engage directly or indirectly in act for and in his behalf during his absence.
any partisan political activity or take part in any election,
initiative, referendum, plebiscite, or recall, except to vote, nor Sec97. Annual Report. — On or before March 31 of each year,
shall he use his official authority or influence to cause the every local chief executive shall submit an annual report to the
performance of any political activity by any person or body. He sanggunian concerned on the socio-economic, political and
may, however, express his views on current issues, or mention peace and order conditions, and other matters concerning the
the names of certain candidates for public office whom he local government unit, which shall cover the immediately
supports. Elective local officials may take part in partisan preceding calendar year. A copy of the report shall be forwarded
political and electoral activities, but it shall be unlawful for them to the Department of the Interior and Local Government.
to solicit contributions from their subordinates or subject these Component cities and municipalities shall likewise provide the
subordinates to any of the prohibited acts under the Omnibus sangguniang panlalawigan copies of their respective annual
Election Code. reports.

Sec94. Appointment of Elective and Appointive Local


Officials; Candidates Who Lost in an Election. — (a) No RA 6713 (1989) – Code of Conduct and Ethical Standards
elective or appointive local official shall be eligible for for Public Officials and Employees
appointment or designation in any capacity to any public office
or position during his tenure. Sec2. Declaration of Policies. — It is the policy of the State to
Unless otherwise allowed by law or by the primary functions of promote a high standard of ethics in public service. Public
his position, no elective or appointive local official shall hold any officials and employees shall at all times be accountable to the
other office or employment in the government or any people and shall discharge their duties with utmost
subdivision, agency or instrumentality thereof, including responsibility, integrity, competence, and loyalty, act with
government-owned or controlled corporations or their patriotism and justice, lead modest lives, and uphold public
subsidiaries. interest over personal interest.
Sec95. Additional or Double Compensation. — No elective or Sec3. Definition of Terms. — As used in this Act, the term:
appointive local official or employee shall receive additional, (a) "Government" includes the National Government, the
double, or indirect compensation, unless specifically authorized local governments, and all other instrumentalities, agencies or
by law, nor accept without the consent of Congress, any branches of the Republic of the Philippines including
present, emoluments, office, or title of any kind from any foreign government-owned or controlled corporations, and their
government. Pensions or gratuities shall not be considered as subsidiaries.
additional, double, or indirect compensation. (b) "Public Officials" includes elective and appointive
officials and employees, permanent or temporary, whether in the
Sec96. Permission to Leave Station. — career or non-career service, including military and police
(a) Provincial, city, municipal, and barangay appointive personnel, whether or not they receive compensation,
officials going on official travel shall apply and secure written regardless of amount.
permission from their respective local chief executives before (c) "Gift" refers to a thing or a right to dispose of
departure. The application shall specify the reasons for such gratuitously, or any act or liberality, in favor of another who
travel, and the permission shall be given or withheld based on accepts it, and shall include a simulated sale or an ostensibly
considerations of public interest, financial capability of the local onerous disposition thereof. It shall not include an unsolicited
government unit concerned and urgency of the travel. gift of nominal or insignificant value not given in anticipation of,
Should the local chief executive concerned fall to act upon such or in exchange for, a favor from a public official or employee.
application within four (4) working days from receipt thereof, it (d) "Receiving any gift" includes the act of accepting
shall be deemed approved. directly or indirectly, a gift from a person other than a member of
(b) Mayors of component cities and municipalities shall his family or relative as defined in this Act, even on the occasion
secure the permission of the governor concerned for any travel of a family celebration or national festivity like Christmas, if the
outside the province. value of the gift is neither nominal nor insignificant, or the gift is
(c) Local government officials traveling abroad shall notify given in anticipation of, or in exchange for, a favor.
their respective sanggunian: Provided, That when the period of
travel extends to more than three (3) months, during periods of

The Apocryphal Maggots:


Rainier, Chrisgel, Corina, Geoffry, Grace and Sylvie Blanche
The Flibbertigibbet Worms:
Golda, Gladys and Melyjane
CA VE AT: By simply r ea ding this re vi ew er a t the end o f the sem este r wi ll (hopef ully) guar antee you r pas sing this cou rse. D rink mode rat ely .
Amusin S C A r y
gly
THE C2005 LOCAL GOVERNMENT REVIEWER - 189 -
(e) "Loan" covers both simple loan and commodatum as members of their personal staff whose terms are
well as guarantees, financing arrangements or accommodations coterminous with theirs.
intended to ensure its approval. (d) Political neutrality. — Public officials and employees
(f) "Substantial stockholder" means any person who shall provide service to everyone without unfair
owns, directly or indirectly, shares of stock sufficient to elect a discrimination and regardless of party affiliation or
director of a corporation. This term shall also apply to the parties preference.
to a voting trust. (e) Responsiveness to the public. — Public officials and
(g) "Family of public officials or employees" means their employees shall extend prompt, courteous, and
spouses and unmarried children under eighteen (18) years of adequate service to the public. Unless otherwise
age. provided by law or when required by the public interest,
(h) "Person" includes natural and juridical persons unless public officials and employees shall provide information
the context indicates otherwise. of their policies and procedures in clear and
(i) "Conflict of interest" arises when a public official or understandable language, ensure openness of
employee is a member of a board, an officer, or a substantial information, public consultations and hearings whenever
stockholder of a private corporation or owner or has a appropriate, encourage suggestions, simplify and
substantial interest in a business, and the interest of such systematize policy, rules and procedures, avoid red tape
corporation or business, or his rights or duties therein, may be and develop an understanding and appreciation of the
opposed to or affected by the faithful performance of official socio-economic conditions prevailing in the country,
duty. especially in the depressed rural and urban areas.
(j) "Divestment" is the transfer of title or disposal of (f) Nationalism and patriotism. — Public officials and
interest in property by voluntarily, completely and actually employees shall at all times be loyal to the Republic and
depriving or dispossessing oneself of his right or title to it in to the Filipino people, promote the use of locally
favor of a person or persons other than his spouse and relatives produced goods, resources and technology and
as defined in this Act. encourage appreciation and pride of country and people.
(k) "Relatives" refers to any and all persons related to a They shall endeavor to maintain and defend Philippine
public official or employee within the fourth civil degree of sovereignty against foreign intrusion.
consanguinity or affinity, including bilas, inso and balae. (g) Commitment to democracy. — Public officials and
employees shall commit themselves to the democratic
Sec4. Norms of Conduct of Public Officials and Employees. — way of life and values, maintain the principle of public
(A) Every public official and employee shall observe the accountability, and manifest by deeds the supremacy of
following as standards of personal conduct in the discharge and civilian authority over the military. They shall at all times
execution of official duties: uphold the Constitution and put loyalty to country above
(a) Commitment to public interest. — Public officials and loyalty to persons or party.
employees shall always uphold the public interest over (h) Simple living. — Public officials and employees and their
and above personal interest. All government resources families shall lead modest lives appropriate to their
and powers of their respective offices must be employed positions and income. They shall not indulge in
and used efficiently, effectively, honestly and extravagant or ostentatious display of wealth in any
economically, particularly to avoid wastage in public form.
funds and revenues. (B) The Civil Service Commission shall adopt positive
(b) Professionalism. — Public officials and employees shall measures to promote (1) observance of these standards
perform and discharge their duties with the highest including the dissemination of information programs and
degree of excellence, professionalism, intelligence and workshops authorizing merit increases beyond regular
skill. They shall enter public service with utmost devotion progression steps, to a limited number of employees recognized
and dedication to duty. They shall endeavor to by their office colleagues to be outstanding in their observance
discourage wrong perceptions of their roles as of ethical standards; and (2) continuing research and
dispensers or peddlers of undue patronage. experimentation on measures which provide positive motivation
(c) Justness and sincerity. — Public officials and employees to public officials and employees in raising the general level of
shall remain true to the people at all times. They must observance of these standards.
act with justness and sincerity and shall not discriminate
against anyone, especially the poor and the Sec5. Duties of Public Officials and Employees. — In the
underprivileged. They shall at all times respect the rights performance of their duties, all public officials and employees
of others, and shall refrain from doing acts contrary to are under obligation to:
law, good morals, good customs, public policy, public (a) Act promptly on letters and requests. — All public
order, public safety and public interest. They shall not officials and employees shall, within fifteen (15) working days
dispense or extend undue favors on account of their from receipt thereof, respond to letters, telegrams or other
office to their relatives whether by consanguinity or means of communications sent by the public. The reply must
affinity except with respect to appointments of such contain the action taken on the request.
relatives to positions considered strictly confidential or as

The Apocryphal Maggots:


Rainier, Chrisgel, Corina, Geoffry, Grace and Sylvie Blanche
The Flibbertigibbet Worms:
Golda, Gladys and Melyjane
CA VE AT: By simply r ea ding this re vi ew er a t the end o f the sem este r wi ll (hopef ully) guar antee you r pas sing this cou rse. D rink mode rat ely .
Amusin S C A r y
gly
THE C2005 LOCAL GOVERNMENT REVIEWER - 190 -
(b) Submit annual performance reports. — All heads or constitute prohibited acts and transactions of any public official
other responsible officers of offices and agencies of the and employee and are hereby declared to be unlawful:
government and of government-owned or controlled (a) Financial and material interest. — Public officials and
corporations shall, within forty-five (45) working days from the employees shall not, directly or indirectly, have any financial or
end of the year, render a performance report of the agency or material interest in any transaction requiring the approval of their
office or corporation concerned. Such report shall be open and office.
available to the public within regular office hours. (b) Outside employment and other activities related
(c) Process documents and papers expeditiously. — All thereto. — Public officials and employees during their
official papers and documents must be processed and incumbency shall not:
completed within a reasonable time from the preparation thereof (1) Own, control, manage or accept employment as officer,
and must contain, as far as practicable, not more than three (3) employee, consultant, counsel, broker, agent, trustee or
signatories therein. In the absence of duly authorized nominee in any private enterprise regulated, supervised
signatories, the official next-in-rank or officer in charge shall sign or licensed by their office unless expressly allowed by
for and in their behalf. law;
(d) Act immediately on the public's personal transactions. (2) Engage in the private practice of their profession unless
— All public officials and employees must attend to anyone who authorized by the Constitution or law, provided, that such
wants to avail himself of the services of their offices and must, at practice will not conflict or tend to conflict with their
all times, act promptly and expeditiously. official functions; or
(e) Make documents accessible to the public. — All public (3) Recommend any person to any position in a private
documents must be made accessible to, and readily available enterprise which has a regular or pending official
for inspection by, the public within reasonable working hours. transaction with their office.
These prohibitions shall continue to apply for a period of one (1)
Sec6. System of Incentives and Rewards. — A system of annual year after resignation, retirement, or separation from public
incentives and rewards is hereby established in order to office, except in the case of subparagraph (b) (2) above, but the
motivate and inspire public servants to uphold the highest professional concerned cannot practice his profession in
standards of ethics. For this purpose, a Committee on Awards to connection with any matter before the office he used to be with,
Outstanding Public Officials and Employees is hereby created in which case the one-year prohibition shall likewise apply.
composed of the following: the Ombudsman and Chairman of (c) Disclosure and/or misuse of confidential information.
the Civil Service Commission as Co-Chairmen, and the —
Chairman of the Commission on Audit, and two government Public officials and employees shall not use or divulge,
employees to be appointed by the President, as members. confidential or classified information officially known to them by
It shall be the task of this Committee to conduct a periodic, reason of their office and not made available to the public,
continuing review of the performance of public officials and either:
employees, in all the branches and agencies of Government (1) To further their private interests, or give undue
and establish a system of annual incentives and rewards to the advantage to anyone; or
end that due recognition is given to public officials and (2) To prejudice the public interest.
employees of outstanding merit on the basis of the standards (d) Solicitation or acceptance of gifts. — Public officials
set forth in this Act. and employees shall not solicit or accept, directly or indirectly,
The conferment of awards shall take into account, among other any gift, gratuity, favor, entertainment, loan or anything of
things, the following: the years of service and the quality and monetary value from any person in the course of their official
consistency of performance, the obscurity of the position, the duties or in connection with any operation being regulated by, or
level of salary, the unique and exemplary quality of a certain any transaction which may be affected by the functions of their
achievement, and the risks or temptations inherent in the work. office.
Incentives and rewards to government officials and employees As to gifts or grants from foreign governments, the Congress
of the year to be announced in public ceremonies honoring them consents to:
may take the form of bonuses, citations, directorships in (i) The acceptance and retention by a public official or
government-owned or controlled corporations, local and foreign employee of a gift of nominal value tendered and received as a
scholarship grants, paid vacations and the like. They shall souvenir or mark of courtesy;
likewise be automatically promoted to the next higher position (ii) The acceptance by a public official or employee of a
with the commensurate salary suitable to their qualifications. In gift in the nature of a scholarship or fellowship grant or medical
case there is no next higher position or it is not vacant, said treatment; or
position shall be included in the budget of the office in the next (iii) The acceptance by a public official or employee of
General Appropriations Act. The Committee on Awards shall travel grants or expenses for travel taking place entirely outside
adopt its own rules to govern the conduct of its activities. the Philippine (such as allowances, transportation, food, and
lodging) of more than nominal value if such acceptance is
Sec7. Prohibited Acts and Transactions. — In addition to acts appropriate or consistent with the interests of the Philippines,
and omissions of public officials and employees now prescribed and permitted by the head of office, branch or agency to which
in the Constitution and existing laws, the following shall he belongs.

The Apocryphal Maggots:


Rainier, Chrisgel, Corina, Geoffry, Grace and Sylvie Blanche
The Flibbertigibbet Worms:
Golda, Gladys and Melyjane
CA VE AT: By simply r ea ding this re vi ew er a t the end o f the sem este r wi ll (hopef ully) guar antee you r pas sing this cou rse. D rink mode rat ely .
Amusin S C A r y
gly
THE C2005 LOCAL GOVERNMENT REVIEWER - 191 -
The Ombudsman shall prescribe such regulations as may be (4) Officers of the armed forces from the rank of colonel
necessary to carry out the purpose of this subsection, including or naval captain, with the Office of the President, and those
pertinent reporting and disclosure requirements. below said ranks, with the Deputy Ombudsman in their
Nothing in this Act shall be construed to restrict or prohibit any respective regions; and
educational, scientific or cultural exchange programs subject to (5) All other public officials and employees, defined in
national security requirements. Republic Act No. 3019, as amended, with the Civil Service
Commission.
Sec8. Statements and Disclosure. — Public officials and (B) Identification and disclosure of relatives. — It shall be
employees have an obligation to accomplish and submit the duty of every public official or employee to identify and
declarations under oath of, and the public has the right to know, disclose, to the best of his knowledge and information, his
their assets, liabilities, net worth and financial and business relatives in the Government in the form, manner and frequency
interests including those of their spouses and of unmarried prescribed by the Civil Service Commission.
children under eighteen (18) years of age living in their (C) Accessibility of documents. —
households. (1) Any and all statements filed under this Act, shall be
(A) Statements of Assets and Liabilities and Financial made available for inspection at reasonable hours.
Disclosure. — All public officials and employees, except those (2) Such statements shall be made available for copying
who serve in an honorary capacity, laborers and casual or or reproduction after ten (10) working days from the time they
temporary workers, shall file under oath their Statement of are filed as required by law.
Assets, Liabilities and Net Worth and a Disclosure of Business (3) Any person requesting a copy of a statement shall be
Interests and Financial Connections and those of their spouses required to pay a reasonable fee to cover the cost of
and unmarried children under eighteen (18) years of age living reproduction and mailing of such statement, as well as the cost
in their households. of certification.
The two documents shall contain information on the following: (4) Any statement filed under this Act shall be available to
(a) real property, its improvements, acquisition costs, the public for a period of ten (10) years after receipt of the
assessed value and current fair market value; statement. After such period, the statement may be destroyed
(b) personal property and acquisition cost; unless needed in an ongoing investigation.
(c) all other assets such as investments, cash on hand or (D) Prohibited acts. — It shall be unlawful for any person
in banks, stocks, bonds, and the like; to obtain or use any statement filed under this Act for:
(d) liabilities, and; (a) any purpose contrary to morals or public policy; or
(e) all business interests and financial connections. (b) any commercial purpose other than by news and
The documents must be filed: communications media for dissemination to the general public.
(a) within thirty (30) days after assumption of office;
(b) on or before April 30, of every year thereafter; and Sec9. Divestment. — A public official or employee shall avoid
(c) within thirty (30) days after separation from the conflicts of interest at all times. When a conflict of interest
service. arises, he shall resign from his position in any private business
All public officials and employees required under this section to enterprise within thirty (30) days from his assumption of office
file the aforestated documents shall also execute, within thirty and/or divest himself of his shareholdings or interest within sixty
(30) days from the date of their assumption of office, the (60) days from such assumption.
necessary authority in favor of the Ombudsman to obtain from The same rule shall apply where the public official or employee
all appropriate government agencies, including the Bureau of is a partner in a partnership.
Internal Revenue, such documents as may show their assets, The requirement of divestment shall not apply to those who
liabilities, net worth, and also their business interests and serve the Government in an honorary capacity nor to laborers
financial connections in previous years, including, if possible, and casual or temporary workers.
the year when they first assumed any office in the Government.
Husband and wife who are both public officials or employees Sec10. Review and Compliance Procedure. —
may file the required statements jointly or separately. (a) The designated Committees of both Houses of the
The Statements of Assets, Liabilities and Net Worth and the Congress shall establish procedures for the review of
Disclosure of Business Interests and Financial Connections statements to determine whether said statements which have
shall be filed by: been submitted on time, are complete, and are in proper form.
(1) Constitutional and national elective officials, with the In the event a determination is made that a statement is not so
national office of the Ombudsman; filed, the appropriate Committee shall so inform the reporting
(2) Senators and Congressmen, with the Secretaries of individual and direct him to take the necessary corrective action.
the Senate and the House of Representatives, respectively; (b) In order to carry out their responsibilities under this
Justices, with the Clerk of Court of the Supreme Court; Judges, Act, the designated Committees of both Houses of Congress
with the Court Administrator; and all national executive officials shall have the power within their respective jurisdictions, to
with the Office of the President. render any opinion interpreting this Act, in writing, to persons
(3) Regional and local officials and employees, with the covered by this Act, subject in each instance to the approval by
Deputy Ombudsman in their respective regions;

The Apocryphal Maggots:


Rainier, Chrisgel, Corina, Geoffry, Grace and Sylvie Blanche
The Flibbertigibbet Worms:
Golda, Gladys and Melyjane
CA VE AT: By simply r ea ding this re vi ew er a t the end o f the sem este r wi ll (hopef ully) guar antee you r pas sing this cou rse. D rink mode rat ely .
Amusin S C A r y
gly
THE C2005 LOCAL GOVERNMENT REVIEWER - 192 -
affirmative vote of the majority of the particular House provisions of this Act, including guidelines for individuals who
concerned. render free voluntary service to the Government. The
The individual to whom an opinion is rendered, and any other Ombudsman shall likewise take steps to protect citizens who
individual involved in a similar factual situation, and who, after denounce acts or omissions of public officials and employees
issuance of the opinion acts in good faith in accordance with it which are in violation of this Act.
shall not be subject to any sanction provided in this Act.
(c) The heads of other offices shall perform the duties
stated in subsections (a) and (b) hereof insofar as their See also… Local Government Code
respective offices are concerned, subject to the approval of the
Secretary of Justice, in the case of the Executive Department
DanGat Notes:
and the Chief Justice of the Supreme Court, in the case of the
Judicial Department. Appointing Authority – generally, the Local Chief Executive
-- The Secretary of Finance for appointive positions connected
Sec11. Penalties. — with finance (e.g. The Treasurer and the Assistant Treasurer)
(a) Any public official or employee, regardless of whether who shall be appointed by the Sec from a list of at least 3
or not he holds office or employment in a casual, temporary, ranking, eligible recommendees of the governor or mayor, as
holdover, permanent or regular capacity, committing any the case may be, subject to civil service law, rules and
violation of this Act shall be punished with a fine not exceeding regulations
the equivalent of six (6) months' salary or suspension not
exceeding one (1) year, or removal depending on the gravity of TITLE V: Appointive Local Officials Common to All
the offense after due notice and hearing by the appropriate body Municipalities, Cities and Provinces
or agency. If the violation is punishable by a heavier penalty
under another law, he shall be prosecuted under the latter
statute. Violations of Sections 7, 8 or 9 of this Act shall be
1. Secretary to the Sanggunian (Sec469) – appointment of
one is mandatory for LGU
punishable with imprisonment not exceeding five (5) years, or a
fine not exceeding five thousand pesos (P5,000), or both, and,
in the discretion of the court of competent jurisdiction, 2. The Treasurer (Sec470) – appointment of one is mandatory
disqualification to hold public office. for LGU
(b) Any violation hereof proven in a proper administrative - appointed by the Sec. of Finance from a list of at least 3
proceeding shall be sufficient cause for removal or dismissal of ranking, eligible recommendees of the governor or
a public official or employee, even if no criminal prosecution is mayor, as the case may be, subject to civil service law,
instituted against him. rules and regulations
(c) Private individuals who participate in conspiracy as - under the administrative supervision of the governor or
co-principals, accomplices or accessories, with public officials or mayor, as the case may be, to whom he shall report
employees, in violation of this Act, shall be subject to the same regularly on the tax collection efforts in the LGU
penal liabilities as the public officials or employees and shall be
tried jointly with them.
(d) The official or employee concerned may bring an
3. Assistant Treasurer (Sec471) – appointed by the Sec from a
list of at least 3 ranking, eligible recommendees of the
action against any person who obtains or uses a report for any
governor or mayor, as the case may be, subject to civil
purpose prohibited by Section 8 (D) of this Act. The Court in
service law, rules and regulations
which such action is brought may assess against such person a
penalty in any amount not to exceed twenty-five thousand pesos
(P25,000). If another sanction hereunder or under any other law 4. The Assessor (Sec472) – appointment of one is mandatory
is heavier, the latter shall apply. for LGU

Sec12. Promulgation of Rules and Regulations,


Administration and Enforcement of this Act. — The Civil
5. Assistant Assessor (Sec473) – appointment of one shall be
optional for LGU
Service Commission shall have the primary responsibility for the
administration and enforcement of this Act. It shall transmit all
cases for prosecution arising from violations of this Act to the 6. The Accountant (Sec474) – appointment of one is
proper authorities for appropriate action: Provided, however, mandatory for LGU
That it may institute such administrative actions and disciplinary
measures as may be warranted in accordance with law. Nothing
in this provision shall be construed as a deprivation of the right
7. The Budget Officer (Sec475) – appointment of one is
mandatory for LGU
of each House of Congress to discipline its Members for
disorderly behavior.
The Civil Service Commission is hereby authorized to 8. The Planning and Development Coordinator (Sec476) –
promulgate rules and regulations necessary to carry out the appointment of one is mandatory for LGU

The Apocryphal Maggots:


Rainier, Chrisgel, Corina, Geoffry, Grace and Sylvie Blanche
The Flibbertigibbet Worms:
Golda, Gladys and Melyjane
CA VE AT: By simply r ea ding this re vi ew er a t the end o f the sem este r wi ll (hopef ully) guar antee you r pas sing this cou rse. D rink mode rat ely .
Amusin S C A r y
gly
THE C2005 LOCAL GOVERNMENT REVIEWER - 193 -
22. The General Services Officer (Sec490) – appointment of
9. The Engineer (Sec477) – appointment of one is mandatory one is mandatory for LGU
for LGU
Macalincag vs. Chang (1992)
10. The Health Officer (Sec478) – appointment of one is
mandatory for LGU Facts:
Acting Sec. of Finance Macalinag issued an Order of Preventive
Suspension against Municipal Treasurer Chang for dishonesty,
11. The Civil Registrar (Sec479) – appointment of one is neglect of duty and acts prejudicial to the best interest of the
mandatory for LGU service.
Macalinag was ordered by RTC to permanently desist from
12. The Administrator (Sec480) – appointment of one is enforcing preventive suspension order ruling that until an acting
mandatory for provincial and city governments, and optional municipal treasurer is appointed to replace Chang, the order is
for municipal government incomplete and cannot be said to have taken effect.
- term is coterminous with that of the appointing authority
Held:
1. Sec. of Finance has jurisdiction to issue an order of
13. The Legal Officer (Sec481) – appointment of one is preventive suspension against the acting municipal treasurer
mandatory for provincial and city governments, and optional of Makati.
for municipal government Under Sec4116 of the Civil Service Law, the designation of the
- term is coterminous with that of the appointing authority replacement is not a requirement to give effect to the
preventive suspension.
14. The Agriculturist (Sec482) – appointment of one is Sec156 of BP337 (old LGC) even provides for the automatic
mandatory for provincial government, and optional for city and assumption of the assistant municipal treasurer or next in
municipal governments rank officer in case of suspension of the municipal treasurer.
The Order of Preventive Suspension of Chang became effective
upon his receipt thereof, which is presumed when he filed a
15. The Social Welfare and Development Officer (Sec483) – complaint in the trial court preventing the implementation of such
appointment of one is mandatory for provincial and city order. Otherwise stated, the designation of the OIC to replace
governments, and optional for municipal government Chang is immaterial to the effectivity of the latter's suspension. A
contrary view would render nugatory the very purpose of
16. The Environment and Natural Resources Officer (Sec484) preventive suspension.
– appointment of one is optional for LGUs
2. The power to discipline is specifically vested under the Civil
Service Law in heads of departments, agencies and
17. The Architect (Sec485) – appointment of one is optional for instrumentalities, provinces and chartered cities who have
LGUs original jurisdiction to investigate and decide on matters
involving disciplinary action. They are the proper disciplining
18. The Information Officer (Sec486) – appointment of one is authority referred to in Sec41 of the Civil Service Law.
optional for LGUs The Office of the Municipal Treasurer is under the Department
- term is coterminous with that of the appointing authority of Finance. Hence, the Sec. of Finance is the proper
disciplining authority to issue the preventive suspension
order. Acting Sec. of Finance, Macalincag, acted within his
19. The Cooperatives Officer (Sec487) – appointment of one is jurisdiction in issuing the order.
optional for LGUs Even assuming that the power to appoint includes the power to
discipline, acting Sec. Macalincag as Sec. of Finance is an alter
20. The Population Officer (Sec488) – appointment of one is ego of the President and therefore, it is within his authority, as an
optional for LGU: Provided, however, That provinces and alter ego, to preventively suspend Chang.
cities which have existing population offices shall continue to
maintain such offices for a period of 5years from date of
LGC’s effectivity, after which said offices shall become
optional 16
Preventive Suspension. — The proper disciplining authority may preventively
suspend any subordinate officer or ee under his authority pending an
21. The Veterinarian (Sec489) – appointment of one is investigation, if the charge against such officer or ee involves dishonesty,
mandatory for LGU oppression or grave misconduct, or neglect in the performance of duty, or if there
are reasons to believe that the respondent is guilty of charges which would
warrant his removal from the service.

The Apocryphal Maggots:


Rainier, Chrisgel, Corina, Geoffry, Grace and Sylvie Blanche
The Flibbertigibbet Worms:
Golda, Gladys and Melyjane
CA VE AT: By simply r ea ding this re vi ew er a t the end o f the sem este r wi ll (hopef ully) guar antee you r pas sing this cou rse. D rink mode rat ely .
Amusin S C A r y
gly
THE C2005 LOCAL GOVERNMENT REVIEWER - 194 -
Debulgado vs. Civil Service Commission (1994) o purpose is to take out of the discretion of the appointing and
recommending authority the matter for appointment of a
Facts: relative (for objectivity)
Incumbent Mayor Debulgado appointed his wife as General
Services Officer (head of the Office of General Services). Argument of vested right misconceived the nature of the action
CSC issued a resolution recalling approval issued by Director taken by the Commission.
Escobia to the wife, and disapproving the promotion of the wife on It was not an imposition of an administrative disciplinary measure.
the ground that it violated the statutory prohibition against nepotic Commission’s function is only to examine the conformity of the
appointments. appointment with law.
Contended that nepotic appointments is applicable only to original Wife was afforded an opportunity to be heard when she filed a
appointments and not to promotional appointments. motion for reconsideration and challenged the disapproval by the
Commission.
Held:
1. The prohibition against nepotic appointments applies to all Action of Commission was valid because the appointment violated
appointments (original or promotional). the provisions of the law. It was null and void as being contra
Basis: Sec59, Book5 of the Revised Administrative Code, and the legem. A void appointment cannot give rise to security of tenure.
Implementing Rules where prohibition was case in comprehensive
and unqualified terms.
It covers ALL APPOINTMENTS without seeking to make any Dimaandal vs. COA (1998)
distinction between differing kinds or types of appointments
It covers all appointments to the national, provincial., city, and Facts:
municipal governments, as well as any branch or instrumentality Dimaandal, designated Acting Assistant Provincial Treasurer for
thereof and all government-owned or controlled corporations Administration by Governor Mayo, filed a claim for difference in
Section 59 contains a list of exceptions which includes only: (a salary and RATA of the positions Assistant Provincial Treasurer
closed list) – persons employed in a confidential capacity, and Supply Officer III (his original position).
teachers, physicians, and members of the AFP Provincial Auditor disallowed the claim.

The implementing rules provide that: “All original appointments and Held: Dimaandal is not entitled to the difference in salary between
personnel actions shall be in accordance with these Rules and with his regular position and the higher position to which he is
other regulations and standards that may be promulgated by the designated.
Commission.” 1. Only the authorized appointing officer can authorize the
Under the abovequoted provisions, both the original appointment payment of compensation.
and all subsequent personnel actions (promotion, transfer, Sec. 471 of the Local Government Code and Sec. 2077 of the
reinstatement, reemployment) must comply with the Rules Revised Administrative Code do not authorize the Provincial
including the prohibition against nepotism Governor to appoint nor even designate a person in cases of
To the extent that all personnel actions after the original temporary absence or disability or a vacancy in a provincial office.
appointment require the issuance of new appointment to another That power resides in the President of the Philippines, or the
position, it is believed that such appointment must also comply with Secretary of Finance who has the power to appoint an Assistant
and prohibition against nepotism Treasurer from a list of 3 ranking eligible recommendees of the
governor or mayor.
Policy reason: civil service appointment should be based on merit Thus, Dimmandal’s designation as Assistant Provincial Treasurer
and fitness by Governor Mayo was defective and conferred no right on the part
Laurel vs. CSC: designated employees are also covered by the of Dimaandal to claim the difference in salaries and allowances
prohibition attached to the position occupied by him.
If not covered by the term appointment “then the prohibition on
nepotism would be meaningless and toothless. Any appointing 2. What was extended to Dimaandal by the Governor was
authority may circumvent it by merely designating, and not merely a designation, not an appointment
appointing, a relative within the prohibited degree to a vacant While an appointment is the selection by the proper authority of an
position in the career service.” individual who is to exercise the powers and functions of a given
office, designation merely connotes an imposition of additional
Question in the CAB is her promotional appointment duties, usually by law, upon a person already in the public service
While the application of the prohibition imposed restrictions on the by virtue of an earlier appointment. (Santiago v COA)
petitioner, Court says: Designation is mere imposition of new or additional duties on the
o prohibition applied without regard to the actual merits of the officer or employee to be performed by him in a special manner. It
porposed appointee and does not entail payment of additional benefits or grant upon the
o to the good intentions of the appointing authority person so designated the right to claim the salary attached to the
o prohibition not intended to penalize faithful service position. (COA Decision dated 1995 & Opinion of the Office of the

The Apocryphal Maggots:


Rainier, Chrisgel, Corina, Geoffry, Grace and Sylvie Blanche
The Flibbertigibbet Worms:
Golda, Gladys and Melyjane
CA VE AT: By simply r ea ding this re vi ew er a t the end o f the sem este r wi ll (hopef ully) guar antee you r pas sing this cou rse. D rink mode rat ely .
Amusin S C A r y
gly
THE C2005 LOCAL GOVERNMENT REVIEWER - 195 -
Legal Affairs of CSC; construction given by an agency should be This view ignores the clear-cut difference in the wording bet. the
given great weight and respect) two ¶s of §7 Art. IX-B, w/c distinction was purposely sought by the
drafters of the Constitution.
3. No violation of constitutional right against deprivation of a. ¶217 allows the holding of multiple offices by an appointive
property without due process of law and non-impairment of
official when allowed by law or by the primary functions of his
obligation of contracts clause in the Constitution
position
The right to the salary of an Assistant Provincial Treasurer is based
on the assumption that the appointment or designation thereof was b. ¶118 is more stringent. It does not provide any exception to the
made in accordance with law. rule against appointment or designation of an elective official
Considering that Dimaandal’s designation was without color of to other government posts except as particularly recognized
authority, the right to the salary or an allowance due form said in the Constitution itself, such as:
office never existed. In the absence of such right, there can be no  President, as head of the Economic and Planning
violation of any constitutional right no an impairment of the Agency
obligation of the contracts clause.  VP, who may be appointed Cabinet member
 Congressman, who may be designated ex officio
4. He was not a de facto officer as to entitle him to receive member of the Judicial & Bar Council
salary for services actually rendered The exemption allowed to appointive officials in ¶2 can’t be
A de facto officer is defined as one who derives his appointment extended to elective officials who are governed by ¶1.
form one having colorable authority to appoint, if the office is an
appointive office, and whose appointment is valid on its face. Color That the SBMA posts are merely ex officio to the position of Mayor
of authority in this context means authority derived from an of Olongapo City, hence, an excepted circumstance  citing Civil
appointment, however irregular or informal, so that the incumbent Liberties v Exec Sec where the Court held that the prohibition in
is not a mere volunteer. §13 Art. VII of the Constitution doesn’t apply to additional duties &
functions required by the primary functions of the official
concerned, who are to perform them in an ex officio capacity… is
Flores vs. Drilon (1993) also untenable
Congress did not contemplate making the SBMA posts as ex
Facts: officio or automatically attached to the Office of the Mayor of
Mayor Gordon of Olongapo City was appointed Chairman/CEO of Olongapo City w/o need of appointment.
the SBMA pursuant to §13 ¶d of RA7277 “Bases Conversion & 1) The phrase “shall be appointed” shows the intent to makes
Development Act of 1992”: the posts appointive.
(d) Chairman/Administrator. - The President shall 2) In the Senate deliberations, Sen. Saguisag suggested that
appoint a professional manager as administrator of the Subic they make the post ex officio so as not to contravene §7 ¶1
Authority with a compensation to be determined by the Board Art.IX-B Constitution, but Congress decided to have the
subject to the approval of the Secretary of Budget, who shall controversy resolved by the courts instead.
be the ex officio chairman of the Board and who shall serve
as the chief executive officer of the Subic Authority: Provided, That the proviso is NOT a legislative encroachment on the
however, That for the first year of its operations from the appointing authority of the President…
effectivity of this Act, the mayor of the City of Olongapo shall The power of appointment necessarily carries the discretion of
be appointed as the chairman and chief executive officer of whom to appoint. When Congress clothes the President with the
the Subic Authority. power to appoint an officer, it cannot at the same time limit the
Petitioners filed an original petition before the SC, challenging the choice of the President to only one candidate, even on the pretext
constitutionality of the proviso above. of prescribing qualifications of the officer (as in this case, where
the qualifications prescribed can only be met by one individual).
Held: The proviso violates the constitutional proscription against Such enactment eliminates the discretion of the appointing power
appointment or designation of elective officials to other government [and encroaches upon his power of appointment].
posts.
Purpose of the proscription: It is an affirmation that a public office SOLUTION: Since the ineligibility of an elective official for
is a full-time job. A public officer should be precluded from appointment remains all throughout his tenure or during his
dissipating his efforts…among too many positions of responsibility, incumbency, Gordon may resign first from his elective post to cast
which may result in inefficiency. off the constitutionally-attached disqualification.

That §94 LGC permits the appointment of a local elective official to 17


Unless otherwise allowed by law or by the primary functions of his position, no
another post if so allowed by law or by the primary functions of his appointive official shall hold any other office or employment in the Government or
office is untenable. any subdivision, agency or instrumentality thereof, including GOCCs or their
No legislative act can prevail over the Constitution subsidiaries.
18
No elective official shall be eligible for appointment or designation in any
capacity to any public office or position during his tenure.

The Apocryphal Maggots:


Rainier, Chrisgel, Corina, Geoffry, Grace and Sylvie Blanche
The Flibbertigibbet Worms:
Golda, Gladys and Melyjane
CA VE AT: By simply r ea ding this re vi ew er a t the end o f the sem este r wi ll (hopef ully) guar antee you r pas sing this cou rse. D rink mode rat ely .
Amusin S C A r y
gly
THE C2005 LOCAL GOVERNMENT REVIEWER - 196 -
a. intent to abandon: failure to perform functions as
CONCLUSION: Sangguniang Bayan member; failure to collect remuneration
- Gordon’s appointment pursuant to an unconstitutional of a Sangguniang Bayan member; failure to object to the
legislative act is null & void. appointment of Aquino as replacement; failure to initiate any
- He however remains Mayor of Olongapo City. act to reassume his old post
- His acts as SBMA Chair/CEO are not necessarily null and b. overt acts: letter of resignation; assumption of office as
Sangguniang Panlalawigan member; receipt of remuneration
void. He may be considered a de facto officer, whose acts will
for said post; faithful discharge of duties of said post
hold valid in so far as they involve the interests of the public
and 3rd persons.
- Also, all emoluments received by Gordon pursuant to his SC notes of the term "designated"  when one is called to
appointment may be retained by him. discharge duties IN ADDITION to his regular responsibilities (ex:
SC Justice designated as member of the House of
Representatives Electoral Tribunal).
In CAB, Antonio was not also designated to simultaneously
Sangguniang Bayan of San Andres, Catanduanes vs. CA discharge functions of both positions.
(1998)

Facts: Mathay vs. Civil Service Commission (1999)


Antonio was elected as Baranggay Captain in 1989, and later
elected as president of the Association of Barangay Councils
(ABC). Result: pursuant to 1983 LGC, he was appointed by the Facts:
President as member of the Sangguniang Bayan QC Mayor appointed Tabernilla as Electrical Engineer V of the City
Because of the reorganization found to be necessary by DILG, Government, pursuant to qualifications provided in city ordinance.
DILG Sec appointed Antonio as temporary member of the The appointment was approved by the CSC Regional Field Officer.
Sangguniang Panlalawigan. The appointment was disputed with the CSC, claiming thatit was
So Antonio resigned as member of the Sangguinang Bayan. issued in violation of CSC Memorandum Circular No 42 which
Pursuant to the 1983 LGC, the Vice President of the ABC was the lays down the qualifications of an Engineer V (no degree in
one appointed to the Sangguniang Bayan. Engineering).
SC ruled that appointment of Antonio as sectoral representative to CSC revoked the appointment
the Sangguniang Panlalawigan was void.
Antonio wrote the members of the Sangguniang Bayan advising Held: CSC has jurisdiction to reverse the appointment of the City
them of his resumption of his original position. Mayor.
Sangguniang Bayan issued Resolution No. 6, declaring Antonio The CSC has the power to take action on all appointments and
had no legal basis to resume office. other personnel actions and that such power includes the authority
to recall an appointment initially approved in disregard of
Held: There was no valid resignation on the part of Antonio but applicable provisions of Civil Service laws and regulations
there was an abandonment in office. (Debulgado vs. CSC)
Administrative Code of 1997: CSC has the power to:
As to Resignation – Definition of Resignation: act of giving up or a. hear and decide administrative cases including contested
the act of an officer by which he declines his office and renounces appointments and
the further right to use it b. review the decisions and actions of its agencies
To constitute a complete resignation, there must be:
a. an intention to relinquish a part of the term The case before the CSC was not a disciplinary case. (citing
b. an act of relinquishment Debulgado case again). The CSC, in approving or disapproving
c. an acceptance by the proper authority appointments,
The last requisite is not present. There was no showing that the a. only examines the conformity of the appointment with
letter given to the DILG was received, much less acted upon by applicable provisions of law and
said Department. b. WON the appointee possess the minimum qualifications and
none of the disqualifications.
As to Abandonment of Office - There was a voluntary
abandonment of office by Antonio. According to Omnibus Rules Implementing BookV of E.O. No. 292
Definition of Abandonment: voluntary relinquishment of an office by as well as Uniform Rules of Procedure in the Conduct of
the holder, with the intention of terminating his possession and Administrative Investigations: Tabernilla will be reverted to his
control thereof. former position.
Essential elements of Abandonment:
a. intent to abandon
b. an overt act by which the intention is to be carried into effect Mathay vs. CA (1999)
In CAB: both elements are present:
Facts:

The Apocryphal Maggots:


Rainier, Chrisgel, Corina, Geoffry, Grace and Sylvie Blanche
The Flibbertigibbet Worms:
Golda, Gladys and Melyjane
CA VE AT: By simply r ea ding this re vi ew er a t the end o f the sem este r wi ll (hopef ully) guar antee you r pas sing this cou rse. D rink mode rat ely .
Amusin S C A r y
gly
THE C2005 LOCAL GOVERNMENT REVIEWER - 197 -
Former QC Mayor Brigido Simon appointed private respondents to
positions in Civil Service Units (CSU) of QC pursuant to PD51. Every action must be prosecuted/ defended in the name of the real
However, DOJ rendered Opinion No. 33 stating that PD51 was party in interest.
never in force or effect because of non-publication in OG and thus IN CAB, the person adversely affected is not CSC but the private
cannot be the basis of CSU appointments. Hence, CSC issued MC respondent Labajo who did not opt to appeal. The CSC had
No.30 directing the revocation of all appointments made under dangerously departed from its role as adjudicator and became an
PD51 w/in one year from the issuance of the MC. advocate.
Ordinance NC-140 was enacted which established the Department Jurisprudence has held that CSC has the right to bring an appeal
of Public Order and Safety (DPOS). Sec3 provided for the only as the aggrieved party affected by a ruling which may
absorption of CSU personnel into the DPOS. Mayor Simon offered seriously prejudice the civil service system.
contractual appointments to the private respondents from June 5,
1991-June 30, 1992.
Subsequent Mayor Mathay did not renew the appointments after De Rama vs. CA (2001)
their expiry.
Private respondents appealed to CSC which issued resolutions Facts:
holding that the reappointment of private respondent to the DPOS After losing in reelection, former Mayor Evelyn Abeja made 14
was automatic pursuant to the Ordinance NC-140 and ordering appointment w/in 27 days, the last one just 3 days before she
their reinstatements. bowed out of the service.
Upon assumption of the new mayor of Pagbilao Quezon, Conrado
Held: de Rama, he wrote to CSC seeking the recall of the 14 appointees
1. CSC has no authority to direct Mayor Mathay to reinstate on the ground that these are midnight appointments prohibited by
respondents in DPOS. Art VII Sec 1519 of 1987 Constitution.
BP337 applies and not the LGC of 1991.
Sec3 of Ordinance NC-140 is invalid for being inconsistent with Held:
BP337. The ordinance refers to personnel and not to positions. In 1. Making appointments during the last days of tenure by a local
effect, the Sanggunian or City council is dictating who shall occupy elective official is not prohibited.
the DPOS positions. Art VII Sec 15 of 1987 Constitution or the so-called constitutional
BP 337 provides that the power to appoint rests exclusively with prohibition on midnight appointment applies only to the President
the local chief executive and thus cannot be usurped by the city or Acting president. There is no law prohibiting local elective
council or sanggunian through the simple expedient of enacting officials from making midnight appointments.
ordinances that provide for the absorption of specific persons to
certain provisions. 2. The appointments were made not in violation of the rules in
the issuance of appointments
BP 337 expressly enumerated the separately and clearly Records reveal that:
delineated the powers of the mayor and the city council. The power - the 14 employees were duly appointed following 2 meetings
to appoint is vested in the local chief executive. On the other hand, of the Personnel Election Board.
the power of the city council is limited to creating, consolidating - No showing that 14 employees were not qualified for the
and reorganizing city officers and positions supported by local positions they were appointed to
funds. - Their appointment was duly attested to by the Head of CSC
Had congress intended to grant the power to appoint to both the field office in Lucena City
city council and the local chief executive, it would have said so.
3. The mayor cannot unilaterally revoke such appointments
2. CSC has no authority to direct that an appointment of a made.
specific individual be made. It only attest to W/N person The appointees had already assumed their positions even before
chosen to fill the position is eligible or not. the new mayor assumed office. Consequently, their appointments
By ordering the Mayor to reinstate private respondents, it took effect immediately and cannot be unilaterally revoked or
substituted its own judgment. recalled by the new mayor.
CSC does not have the authority to direct that an appointment of a Upon the issuance of an appointment and the appointee’s
specific individual be made. Once the CSC attests WON the assumption of the position in civil service, he acquires a right which
person chosen to fill the position is eligible or not, its role in the cannot be taken away either by revocation of the appointment or
appointment process necessarily ends. by removal except for cause and with previous notice and hearing.
IN CAB, Ordinance refers to CSU, the identities of which cannot
be mistaken. CSC resolution calls for reinstatement of the named
individuals. Thus, there being no issue as to who are to sit in the 19
newly created DPOS, there is therefore no room left for the Sec. 15. Two months immediately before the next presidential elections and up
exercise of discretion. to the end of his term, a president or acting president shall not make
CSC in effect technically issued new appointments which usurped appointments, except temporary appointments to executive positions when
the appointing power of the local chief executive. continued vacancies therein will prejudice public service or endanger public
safety.

The Apocryphal Maggots:


Rainier, Chrisgel, Corina, Geoffry, Grace and Sylvie Blanche
The Flibbertigibbet Worms:
Golda, Gladys and Melyjane
CA VE AT: By simply r ea ding this re vi ew er a t the end o f the sem este r wi ll (hopef ully) guar antee you r pas sing this cou rse. D rink mode rat ely .
Amusin S C A r y
gly
THE C2005 LOCAL GOVERNMENT REVIEWER - 198 -
It is well settled that the person assuming a position in the civil
service under a completed appointed acquires a legal and not just Held: CSC and CA erred. Under the circumstances of the case,
an equitable right. Lameyra should be given a full opportunity to prove his contention
that he was illegally terminated which was not fully done in CAB.
WHO IS AUTHORIZED TO RECALL AN APPOINTMENT
INITIALLY REVOKED? I. Accordingly, the first opportunity that Lameyra had to contest the
 It is the CSC that is authorized to recall an appointment initially sufficiency of the evidence to support his dismissal was when he
approved but only when appointment and approval are proven to filed his motion for recon from the CSC Resolution.
be in disregard of applicable provisions of the CSC laws and The three sworn statements which were annexes to the motion
regulations directly controverted Vicencio’s certification that he was AWOL 
BASIS: Rule V Sec 9 of the Omnibus Implementing Regulations of cannot be considered new evidence belatedly submitted (as
the Revised Administrative Code specifically provides that an claimed by Mayor Pangilinan) because there was no notice and
appointment accepted by the appointee cannot be withdrawn or hearing when he was dropped from the rolls.
revoked by the appointing authority and shall remain in force and
effect until disapproved by the Commission. II. One of the affiants is ViceMayor Fernandez (whose acts as a
public official are entitled to presumption of regularity in the
Grounds for Recall (Under Sec 20 Rule VI) performance of duty).
a. Non-compliance w/procedure or criteria provided in the It would be in compliance with due process requirements to have
agency’s Merit promotion Plan given due consideration to his sworn statement in view of the
b. Failure to pass through the agency’s selection/promotion prevailing circumstances in CAB, and also in consonance with
board Mayor Pangilinan’s own theory that Lameyra was afforded his right
c. Violation of the existing collective agreement between to be heard when he filed his motion for recon in CSC.
management and employees relative to promotion or
d. Violation of other existing civil service law rules and III. Findings of fact by administrative agency will be respected by
regulation. SC if supported by substantial evidence.
However, certification of personnel officer that Lameyra did not
DISSENT: Mendoza report for work does not appear to constitute substantial evidence
Art VII Sec 15 of 1987 Consti. is simply an application of the a in light of Lameyra’s submission:
broader principle that after the appointing authority has lost the (1) that personnel precisely prevented him from signing logbook
elections, his is the duty of a prudent caretaker of the office and (2) he has been replaced by one Leynes
therefore he should not fill positions in the gov’t unless required by (3) he has been asked to submit his resignation which he refused
the imperatives of public service. to do.
This rule binds all including mayors who are who are vested with  Lameyra should then be given a last full opportunity to prove his
power of appointment, and it flows from the principle that a public contention that the termination of his services was illegal.
office is a public trust.
From Aytona case: Midnight appointments are bad because they
offend principles of fairness justice and righteousness. They Makati City Government vs. CSC (2002)
cannot now be less bad because they were made at the local level,
by mayors and other local executives. Facts:
The appointments must be few and so spaced as to afford some Galzote, employed as clerk in the Dept. of Eng and Public Works
assurance of deliberate action and careful consideration of the of Makati City, was arrested without warrant and detained allegedly
need for the appointment and the appointee’s qualification. for kidnapping for ransom with physical injuries, then subjected to
inquest proceedings, then a criminal case was eventually docketed
with RTC Pasig. Incarcerated in the Rizal Provincial Jail, she could
Lameyra vs. Pangilinan (2000) not report for work – so she was suspended from office by City
Government starting Sept9, 1991 until the final disposition of her
Facts: case. She was eventually dropped from the rolls of employees.
Lameyra was appointed as janitor/messenger in the Municipal Hall 3years later, she was acquitted of the crime charged for failure of
of Famy, Laguna on Feb2, 1988 under temporary status. His the prosecution to prove any act establishing her complicity in the
appointment was made permanent on Jan1, 1989 by then Mayor crime. She was released.
Acomular (who was defeated by Mayor Pangilinan in the last CSC Resolution 960153 – ordered reinstatement with backwages
election). from Oct19, 1994 until actual resumption of duty.
Lameyra received a letter from Mayor Pangilinan informing him
that he is dropped from the roll of employees pursuant to Memo Held:
Circular No.12, Series of 1994 of the CSC due to (1) She cannot be faulted for failing to file an application for LOA under
Insubordination, and (2) AWOL. Sec52 and 63 of CSC Rules prior to her detention as application
CSC dismissed Lameyra’s appeal – that there is sufficient ground for leave and obtain approval thereof.
to support action of the municipality in dropping him from service

The Apocryphal Maggots:


Rainier, Chrisgel, Corina, Geoffry, Grace and Sylvie Blanche
The Flibbertigibbet Worms:
Golda, Gladys and Melyjane
CA VE AT: By simply r ea ding this re vi ew er a t the end o f the sem este r wi ll (hopef ully) guar antee you r pas sing this cou rse. D rink mode rat ely .
Amusin S C A r y
gly
THE C2005 LOCAL GOVERNMENT REVIEWER - 199 -
Records clearly show that she had been advised 3days after her Rule of automatic LOA clearly falls within the constitutionally
arrest (Sept9, 1991), that Makati City Gov. had placed her under delegated power of CSC and is reasonable under the
suspension until the final disposition of her criminal case. circumstances to address absences from work which are not
The arrangement bound the City Government to allow her to return attributable to the concerned gov’t employee.
to work after the termination of her case (if acquitted of charge)
She had no intention to go AWOL (leaving or abandoning without If it were already written in CSC rules or an existing provision, no
justifiable reason and without notifying employer) from her post reason for CAB to ensue and be disputed.
since she had a valid reason for failing to report for work  being If such were already in place, CSC would have no basis for
detained without bail interpreting its rules since all it had to do was to implement them.
Had she been told that it was still necessary to file an application What CSC interpreted in CAB were Sec20, 35 and other related
for leave despite the assurance from the City Government, she provisions of CSC rules on requirement of an approved LOA.
would have filed it - In Sec20 – it is enough that the employer be informed of the
Also clear from the records is her ordeal in jail beginning on Sept6, absent employee’s illness, which information is effective
1991 and ending after her acquittal  leaving her no time to attend substitute for a prior leave application illness is not the only
to the formality of filing a leave application. instance of force majeure where there are other events
beyond the control of employee which may force him to be
Later memorandum dropping her from the rolls for absence cannot absent – it is in these cases that CSC sought to interpret
be valid being violative of due process. them as contemplating an automatic LOA
City Gov should have required her to file an application by
 In CAB, she has sufficiently informed City Gov of her
informing her of such requirement  the subsequent memo predicament so no prior leave application needed
dropping her should have been sent to her at the jail where she - Being in an automatic LOA, exercise of authority under Sec35
had been detained and where she could have received it to drop employees from rolls is tempered with
City Gov had actual and official knowledge of her incarceration as CSC also has power to allow exemptions from prior filing of leave
obvious from admissions in the petition  obliged it to send notice applications – logically flows from task of CSC to regulate civil
to where she was detained service in country as ordained by COnsti and mandated by Admin
Code.
CSC holding that she was on automatic leave of absence during CSC Rules even do not limit powers of CSC in this regard to cases
the period of detention is SUSTAINED. of illness only.
CSC is the constitutionally mandated central personnel agency of Reasonableness as a standard, CSC can determine in exercise of
Gov. its constitutional mandate.
Admin Code of 1987 further empowers CSC to “prescribe rules CSC’s reasonable implementation of its own rules in the specific
and regulations for carrying into effect provisions of Civil Service and actual case of Galzote is advocated by the court as was done
Law and other pertinent laws”  and for matters converning LOAs, in the case UP vs. CSC where UP was exempted from CSC rules
Admin Code specifically vests the CSC to ordain as regards it. because of academic freedom with the Consti as basis.
- pursuant thereto – it promulgated Resolution 91-1631
devoting Rule XVI to LOAs which have been further amended She cannot be faulted for failure to assume her work.
- Sec20 (or 52) and 35 (or 63) require an approved leave of Clearly, she reported for work as soon as she was free to do so,
absence to avoid being an AWOL but was unfortunately turned back by City Gov.
o cannot be interpreted as exclusive and referring only to In locking her out from her job, City Gov illegally deprived her of
1mode of securing the approval of LOA which would her opportunity to work and must be held liable.
require an employee to apply for it (formalities) before
exceeding 30days in order to avoid being dropped from DISSENTING: Panganiban
the rolls I. Galzote was AWOL
o there is the CSC recognized rule of automatic leave of No Automatic Leave of Absence
absence under specified circumstances as CSC stated 1. current Civil Service Law and Rules do not contain any
in its assailed Resolution citing a similar case – Cenon provision on automatic leave of absence – and not disputed.
Vargas, CSC Resolution 94-2795 and 95-5559 2. Sec35 in providing for absence without approved leave
 CSC was only interpreting its own rules on LOA and not a makes no distinction or qualification and gives no regard for
statutory provision in coming up with this uniform rule the reason for the absence
CSC has the power to interpret its own rules and any phrase - It simply states that an employee who fails to report
contained in them with its interpretation significantly becoming part continuously for at least 30days without an approved
of the rules themselves. leave is considered AWOL, and dropped form service.
To be consistent with established principle stated, CSC ruling will 3. Sec20 states that approval of LOA is “contingent on needs of
not be rejected. Rejecting the ruling on an automatic LOA solely for the service”
want of a provision expressly and specifically allowing it would - clearly, approval for any reason other than illness cannot
erroneously repudiate the difference between the agency’s own be presumed
understanding of its rules and its interpretation of a statute. - it is granted only after evaluation of the “needs of the
service”

The Apocryphal Maggots:


Rainier, Chrisgel, Corina, Geoffry, Grace and Sylvie Blanche
The Flibbertigibbet Worms:
Golda, Gladys and Melyjane
CA VE AT: By simply r ea ding this re vi ew er a t the end o f the sem este r wi ll (hopef ully) guar antee you r pas sing this cou rse. D rink mode rat ely .
Amusin S C A r y
gly
THE C2005 LOCAL GOVERNMENT REVIEWER - 200 -
- to secure such approval, necessary for one to file an SO… letter delivered to her house and informing her of her AWOL
application for it before exceeding 30days of absence in status is due notice to her.
order to avoid being dropped from the rolls Also, failure to give notice does not prevent the dropping of
- No specific or clear-cut provision allowing an automatic employee concerned from government service – not being
leave of absence, such rule forecloses possibility of such jurisdictional in nature as held in Quezon vs. Borromeo.
leave, even on the ground of incarceration – where the
person is not prevented from communicating with
outside world. Pastor vs. City of Pasig (2002)
CAB: Galzote did not file an application for any type of leave – no
approved leave. Facts:
There was nothing to stop her from writing City Gov, informing it of Pastor was appointed Budget Officer of Pasig with her
her plight and applying for leave and show interest in her job – appointment having been confirmed by the Department of Budget
necessary to enable government to take appropriate measures to and Management.
ensure smooth delivery of service to the public. Mayor Eusebio issued a memo relieving Pastor from her position
and she was reassigned to the Office of the Administrator.
4. EO292 is clear that the only LOA that a civil service employee Pastor filed a complaint with CSC and claimed for reinstatement.
is entitled to is that which may be provided by law, rules and CSC ruled in favor of Pastor. Mayor Eusebio complied by
regulations designation Pastor as head of Pasig City Hall Annex.
- None provide for automatic leaves of absence CSC once again ruled for Pastor.
- The law also considers the grant subservient to “interest CA held that the reassignment was a valid exercise of the
of service” – an interpretation that would promote and "extraordinary powers of the respondent City Government. It
enhance government service should be upheld over the pointed out that the reassignment to the Office of the Municipal
ponencia’s interpretation Administrator was only "temporary in nature" and that, in
CAB: It was duty of Galzote to appraise the government of her designating petitioner as head of the City Hall Annex, the city
inability. Pursuant to Law and Rules, she should be dropped. government had substantially complied with the first CSC
Resolution.
Interpretation Refers Only to Extant Laws and Rules
Ponencia: CSC is granted sufficient residual authority via its Held: CA erred. Pastor’s reassignment amounts to a constructive
power of “interpreting” its own rules, to allow automatic LOAs. removal without cause from the service.
 NO. Undisputed that CSC Law and Rules do not provide for an It has been held that a reassignment that is indefinite and results in
automatic leave. Construction/interpretation is resorted to only in a reduction in rank, status, and salary is in effect a constructive
case of doubt on how to understand an existing legal provision. removal from the service. Pastor’s reassignment to different offices
CAB: There is no room for doubt, the law and rules clearly do not in the local government of Pasig City is indefinite. Pastor has been
provide for automatic LOAs. There is nothing to interpret. on virtual floating assignments which cannot but amount to a
CSC has not power to interpret an inexistent rule, especially is diminution of her rank, hence impermissible under the law.
such interpretation takes away the provision of Sec60, EO292 For all intents and purposes, her reassignment, lasting nearly ten
which mandates that LOAs shall be allowed only in the interest of years now, is a removal without cause as Budget Officer of the City
the service  that approval cannot be presumed but may be of Pasig. Indeed, her duties in her new assignment as head of the
granted only after considering the exigencies of government Pasig City Hall Annex show the "more than temporary" nature of
service. her reassignment.
That she has suffered a diminution in her rank is also evident. As
II. She was afforded Due Process head of the Pasig City Hall Annex, Pastor's budget proposals f will
Sec35 required due notice for dropping of employee on AWOL. be subject to review by the City Budget Officer. Moreover, the
She was sent a letter informing her that she had been reported position of City Budget Officer is created by statute, while that of
AWOL for over a year – reason for being dropped from rolls  the head of the Pasig City Hall Annex is created by mere
delivered to her house. She did not respond nor contest the letter. ordinance.
CA assumed City Gov knew of her predicament and her
location
 Records show total absence of support for such assumption. Garcia vs. Pajaro (2002)
Undisputed that she had not filed any application for LOA – she
had not record of the reason for her continued absence. Facts:
 City Gov is a public corporation – it would be unreasonable and Garcia, an employee at the Dagupan City Treasurer’s Office, was
unjustly burdensome for it to know the whereabouts of Galzote, ordered suspended by City Treasurer because of a Formal Charge
when she herself did not bother to inform it of her situation (being against him.
one of thousand employees) During his suspension, Garcia still reported for work not honoring
Relying on its records to ascertain her address would be more in the suspension order.
accord with reason and exigencies of public service.

The Apocryphal Maggots:


Rainier, Chrisgel, Corina, Geoffry, Grace and Sylvie Blanche
The Flibbertigibbet Worms:
Golda, Gladys and Melyjane
CA VE AT: By simply r ea ding this re vi ew er a t the end o f the sem este r wi ll (hopef ully) guar antee you r pas sing this cou rse. D rink mode rat ely .
Amusin S C A r y
gly
THE C2005 LOCAL GOVERNMENT REVIEWER - 201 -
Held: The City Treasurer has authority to discipline petitioner who Masturbation is fun, healthy and safe. In fact, it's the safest form
is an employee at the Treasurer’s Office. of sex you can practice. You can't become pregnant or contract
Under the present LGC, appointive officers and employees of an STD from masturbating, nor will your hands cheat on you,
LGUs are covered by the Civil Service Law, and those lie about how good it was, or spend all your money. If you're
promulgated pursuant thereto. Also, the investigation and the
one of the few who hasn't touched themselves, give it a try.
adjudication of administrative complaints against appointive local
officials and employees, as well as their suspension and removal, What can it hurt? At the worst, you won't care for it much. At
shall be in accordance with the Civil Service Law and rules and best, you'll discover an exciting new way to relieve stress,
other pertinent laws. celebrate your body, relieve sexual tension and give yourself a
BookV of 1987 Administrative Code is the primary law governing whole lotta pleasure.
appointive officials and employees in the government and it Even if you've been masturbating for years, a little variety never
enumerates the grounds for disciplining them. hurt anyone. Have you tried using different kinds of lube when
you masturbate? Most men have tried some form of lubricant,
The power to discipline is specifically granted by Sec47 of the but surprisingly few women have. Try a couple of different
1987 Administrative Code to heads of departments, agencies and brands and types... a little moisture adds a lot to the experience.
instrumentalities, provinces and cities. What about sex toys? Many women have tried vibrators, dildos
On the other hand, the power to commence administrative and clitoral stimulators, but few men have tried sheaths, penis
proceedings against a subordinate officer or employee is granted
pumps or cyberskin vaginas--all of which can be a lot more fun
by Sec34 of the Omnibus Rules Implementing BookV of Admin
Code to the secretary of a department, head of office of equivalent than your bare hands. Whether you're a newbie or a pro at self
rank, head of LGU, chief of an agency, the regional director or a love, relax, take your pants off, and try some of these tricks for a
person with a sworn written complaint. zestier solo sex experience.
(continued…)
Further, the city treasurer may institute, motu propio, disciplinary
proceedings against a subordinate officer or employee. Local Q&A with Dr. Sandor Gardos
Administrative Regulations No.2-85 authorized the Secretary of
Finance, the Regional Director, and head of a local treasury or an Question
assessment office to start administrative disciplinary action against I love my boyfriend very much but he is just not very good in
officers or employees subordinate to them. bed. Although he tries, he is frequently "barking up the wrong
tree" with where he touches me -- if you know what I mean. I
Sec455(b-1-x) of 1991LGC stating that the city mayor “may cause don't really want to hurt his feelings by telling him he is doing
to be instituted administrative or judicial proceedings against any
things wrong, but I also want to be satisfied. Any suggestions?
official or employee of the city” is not incongruent with the
provisions of the 1987 Administrative Code which authorizes heads
Answer
of agencies to discipline subordinate employees.
There is in fact no provision in the 1991LGC expressly rescinding While I am all for open, detailed conversations, I agree that
the authority of the DOF to exercise disciplinary authority over its yelling out "stop that" or "not there" in bed is probably not the
employees. Likewise, there is nothing that prohibits the city ideal approach. Generally, it's best to focus on the positive when
treasurer from filing a complaint against his subordinate. communicating with a partner about sexual matters. For
example, instead of saying, "Don't do that!" when he is missing
Power to discipline evidently includes the power to investigate. the mark, try saying, "Oh yes! Just like THAT!" when he hits the
spot. You can also lean over and whisper in his ear, "You know
what would really turn me on?" then suggest what you'd like!
HOW-TO GUIDE: Masturbation by Tamar Love Another option is to masturbate in front of your partner. You
can make it into a little erotic game, taking turns, or doing it at
Masturbation used to be an utterly taboo subject. We were told the same time. You might even learn something more about
touching ourselves was dirty and wrong, that if we did it we'd what he likes.
go crazy or grow hair on our palms. If these lies were spread to If you don't think you can bring yourself to say anything or
keep us from pleasuring ourselves, it worked: in the mid-20th masturbate in front of him, the best approach is to take his hand
century, only 50% of men and far fewer women owned up to gently and place it where you want it. Show him where you like
masturbating. The women's movement, better health education to be touched, how hard, how fast, etc. While I appreciate that
and increased levels of body consciousness have done wonders you don't want to hurt his feelings, most men report that they
for masturbation's bad rap. Now, according to the Kinsey like it when women give them more clear feedback about
Institute, 92% of males and 62% of females report having what they like. However you approach it, you will certainly be
masturbated. In fact, single females report that masturbation is doing both of you a service!
their most important sexual outlet. We must be growing more
enlightened as a culture!

The Apocryphal Maggots:


Rainier, Chrisgel, Corina, Geoffry, Grace and Sylvie Blanche
The Flibbertigibbet Worms:
Golda, Gladys and Melyjane
CA VE AT: By simply r ea ding this re vi ew er a t the end o f the sem este r wi ll (hopef ully) guar antee you r pas sing this cou rse. D rink mode rat ely .
Amusin S C A r y
gly
THE C2005 LOCAL GOVERNMENT REVIEWER - 202 -
Practice of Profession & Related LGC Provisions sangguniang bayan members, subject to civil service law, rules
and regulations. The sangguniang bayan shall act on the
Sec31. Submission of Municipal Questions to the Provincial appointment within fifteen (15) days from the date of its
Legal Officer or Prosecutor. — In the absence of a municipal submission; otherwise, the same shall be deemed confirmed.
legal officer, the municipal government may secure the opinion (e) Elective and appointive municipal officials shall
of the provincial legal officer, and in the absence of the latter, receive such compensation, allowances and other emoluments
that of the provincial prosecutor on any legal question affecting as may be determined by law or ordinance, subject to the
the municipality. budgetary limitations on personal services as prescribed in Title
Five, Book Two of this Code: Provided, That no increase in
Sec90. Practice of Profession. — compensation of the mayor, vice-mayor, and sangguniang
(a) All governors, city and municipal mayors are bayan members shall take effect until after the expiration of the
prohibited from practicing their profession or engaging in any full term of all the elective local officials approving such
occupation other than the exercise of their functions as local increase.
chief executives.
(b) Sanggunian members may practice their professions, Sec481. Qualifications, Terms, Powers and Duties. —
engage in any occupation, or teach in schools except during (a) No person shall be appointed legal officer unless he is
session hours: Provided, That sanggunian members who are a citizen of the Philippines, a resident of the local government
also members of the Bar shall not: concerned, of good moral character, and a member of the
(1) Appear as counsel before any court in any civil case Philippine Bar. He must have practiced his profession for at
wherein a local government unit or any office, agency, or least five (5) years in the case of the provincial and city legal
instrumentality of the government is the adverse party; officer, and three (3) years in the case of the municipal legal
(2) Appear as counsel in any criminal case wherein an officer.
officer or employee of the national or local government is The term of the legal officer shall be coterminous with that of his
accused of an offense committed in relation to his office. appointing authority.
(3) Collect any fee for their appearance in administrative The appointment of legal officer shall be mandatory for the
proceedings involving the local government unit of which provincial and city governments and optional for the municipal
he is an official; and government.
(4) Use property and personnel of the government except (b) The legal officer, the chief legal counsel of the local
when the sanggunian member concerned is defending government unit, shall take charge of the office of legal services
the interest of the government. and shall:
(c) Doctors of medicine may practice their profession (1) Formulate measures for the consideration of the
even during official hours of work only on occasions of sanggunian and provide legal assistance and support to
emergency: Provided, That the officials concerned do not derive the governor or mayor, as the case may be, in carrying
monetary compensation therefrom. out the delivery of basic services and provisions of
adequate facilities as provided for under Section 17 of
Sec443. Officials of the Municipal Government. — this Code;
(a) There shall be in each municipality a municipal mayor, (2) Develop plans and strategies and upon approval thereof
a municipal vice-mayor, sangguniang bayan members, a by the governor or mayor, as the case may be,
secretary to the sangguniang bayan, a municipal treasurer, a implement the same, particularly those which have to do
municipal assessor, a municipal accountant, a municipal budget with programs and projects related to legal services
officer, a municipal planning and development coordinator, a which the governor or mayor is empowered to implement
municipal engineer/building official, a municipal health officer and which the sanggunian is empowered to provide for
and a municipal civil registrar. under this Code;
(b) In addition thereto, the mayor may appoint a (3) In addition to the foregoing duties and functions, the
municipal administrator, a municipal legal officer, a municipal legal officer shall:
agriculturist, a municipal environment and natural resources (i) Represent the local government unit in all civil
officer, a municipal social welfare and development officer, a actions and special proceedings wherein the local
municipal architect, and a municipal information officer. government unit or any official thereof, in his official
(c) The sangguniang bayan may: capacity, is a party: Provided, That, in actions or
(1) Maintain existing offices not mentioned in subsections proceedings where a component city or municipality is a
(a) and (b) hereof; party adverse to the provincial government or to another
(2) Create such other offices as may be necessary to carry component city or municipality, a special legal officer
out the purposes of the municipal government; or may be employed to represent the adverse party;
(3) Consolidate the functions of any office with those of (ii) When required by the governor, mayor or
another in the interest of efficiency and economy. sanggunian, draft ordinances, contracts, bonds, leases
(d) Unless otherwise provided herein, heads of and other instruments, involving any interest of the local
departments and offices shall be appointed by the municipal government unit and provide comments and
mayor with the concurrence of the majority of all the recommendations on any instrument already drawn;

The Apocryphal Maggots:


Rainier, Chrisgel, Corina, Geoffry, Grace and Sylvie Blanche
The Flibbertigibbet Worms:
Golda, Gladys and Melyjane
CA VE AT: By simply r ea ding this re vi ew er a t the end o f the sem este r wi ll (hopef ully) guar antee you r pas sing this cou rse. D rink mode rat ely .
Amusin S C A r y
gly
THE C2005 LOCAL GOVERNMENT REVIEWER - 203 -
(iii) Render his opinion in writing on any question Judgment then against City Eng. Divinagracia would be a
of law when requested to do so by the governor, mayor judgment against the City Govt.
or sanggunian; By serving as counsel for the complaining employees and assisting
(iv) Investigate or cause to be investigated any them to prosecute their claims against the City Eng., Javellana
local official or employee for administrative neglect or violated Memo Circ 74-58 which prohibits govt. officials from
misconduct in office, and recommend appropriate action engaging in the private practice of his profession, if such practice
to the governor, mayor or sanggunian, as the case may would represent interests adverse to the government.
be;
(v) Investigate or cause to be investigated any 2. Memorandum Circular 90-81 and Sec90 of LGC do not
person, firm or corporation holding any franchise or violate ArtVIII, Sec5 of Constitution.
exercising any public privilege for failure to comply with MC 90-81 and the LGC simply prescribe rules of conduct for public
any term or condition in the grant of such franchise or officials to avoid conflicts of interest between the discharge of their
privilege, and recommending appropriate action to the public officials and the private practice of their profession, in those
governor, mayor or sanggunian, as the case may be; instances where the law allows it.
(vi) When directed by the governor, mayor, or
sanggunian, initiate and prosecute in the interest of the 3. Sec90 of LGC does not discriminate against lawyers and
local government unit concerned any civil action on any doctors.
bond, lease or other contract upon any breach or It applies to all provincial and municipal officials in the profession
violation thereof; and or engaged in any occupation.
(vii) Review and submit recommendations on If there are some prohibitions that apply particularly to lawyers, it is
ordinances approved and execute orders issued by because of all professions, the practice of law is more likely than
component units; others to relate to, or affect, the area of public service.
(3) Recommend measures to the sanggunian and advise
the governor or mayor as the case may be on all other
matters related to upholding the rule of law; Ramos vs. CA (1981)
(4) Be in the frontline of protecting human rights and
prosecuting any violations thereof, particularly those Facts:
which occur during and in the aftermath of man-made or Municipality of Hagonoy filed a suit to recover its 74-ha fishpond
natural disasters or calamities; and through the law firm of Cruz Durian and Academia
(5) Exercise such other powers and perform such other Par. 19 of complaint alleged that the municipality had obligated
duties and functions as may be prescribed by law or itself to pay the law firm not less than 20% of the amount to be
ordinance. recovered as attorney’s fees.
Provincial Fiscal and municipal attorney entered their appearance
Javellana vs. DILG (1992) as counsel for municipality with the manifestation that private
counsel would be under their supervision and control.
Facts: Lessee and sublessee of fishpond moved to dismiss the Cruz law
City Engineer Divinagracia filed an Admin. Case against Atty. firm from serving as counsel of the municipality.
Javellana (elected City Councilor) for: TC denied motion. Found that Cruz, the head of the law firm,
- continuously engaging in the practice of law without securing volunteered to act as counsel due to his desire to serve his native
the authorization of the Regional Director as required by DILG town. CA affirmed.
Memo Circular No.80-38 in relation to DLG Memo Circular
No. 74-58, and Held: Cruz law firm cannot serve as counsel of municipality.
- for being counsel for Javiero and Catapang, who filed a case To allow them to serve as counsel of the municipality is a
against him for illegal dismissal, putting him in public ridicule. transgression of:
During pendency of said admin case, Javellana requested the DLG 1. Sec 1683 of the Revised Admin Code which provides that
for a permit to continue his practice of law. Request granted.
“the provincial fiscal shall represent the province and any
Memo Circular 90-81 was issued setting forth the guidelines for the
municipality or mun. district thereof in any court, except (1)in
practice of professions by local elective officials.
cases where original jurisdiction is vested in the SC or (2)in
1991LGC was enacted.
cases where the municipality or the municipal district in
question is a party adverse to the provincial govt. or to some
Held:
other municipality or municipal district in the same province.”
1. Javellana violated Memorandum Circular 74-58 prohibiting
government officials from engaging in private practice if it 2. Sec 3 Local Autonomy Act which provides that “the municipal
would represent interests adverse to the government. attorney as head of the legal division or office of a
The illegal dismissal case filed by Javiero and Catapang against municipality” shall act as legal counsel of the municipality xxx
Divinagracia is in effect a complaint against the City Govt., their 3. settled jurisprudence where it was held that the municipality’s
real employer, of which Javellana is a councilman. authority to employ a private lawyer is expressly limited only

The Apocryphal Maggots:


Rainier, Chrisgel, Corina, Geoffry, Grace and Sylvie Blanche
The Flibbertigibbet Worms:
Golda, Gladys and Melyjane
CA VE AT: By simply r ea ding this re vi ew er a t the end o f the sem este r wi ll (hopef ully) guar antee you r pas sing this cou rse. D rink mode rat ely .
Amusin S C A r y
gly
THE C2005 LOCAL GOVERNMENT REVIEWER - 204 -
to situations where the provincial fiscal is disqualified t o situations where the provincial fiscal is disqualified to
represent it. represent it.
The municipality should not be burdened with the expenses of For the exception to apply, the fact that the provincial fiscal was
hiring a private lawyer. Lawmakers assumed that the interests of disqualified to handle the municipality's case must appear on
the municipality would be best protected if a government lawyer record. In the CAB, there is nothing to show that the provincial
handles its litigation. fiscal is disqualified to act as counsel for the municipality on
appeal, hence the appearance of private counsel is without
The fact that the municipal attorney and the fiscal are supposed to authority of law.
collaborate with a private law firm does not legalize the latter’s
representation of the municipality. While a private prosecutor is 2. The fiscal's refusal to represent the municipality is not a
allowed in criminal cases, an analogous arrangement is not
legal justification for employing the services of private
allowed in civil cases wherein a municipality is the plaintiff
counsel. Unlike a practising lawyer who has the right to
decline employment, a fiscal cannot refuse to perform his
Section 1683 of the Revised Admin Code is clear in providing that
functions on grounds not provided for by law without violating
only the provincial fiscal and the municipal attorney can represent
his oath of office.
a municipality in its lawsuits. That provision is mandatory.

J. BARREDO separate opinion: 3. Instead of engaging the services of a special attorney, the
A municipality or province should be allowed to seek the help of municipal council should request the Sec. of Justice to
competent counsel if it feels its case is of such importance that the appoint an acting provincial fiscal in place of the
services of the fiscal and the municipal attorney would be provincial fiscal who has declined to handle and
inadequate and the assistance of private counsel is offered gratis. prosecute its case in court, pursuant to Sec. 1679 of the
RAC.
DanGat Notess: Rule on Representation –
4. It is also significant that the lack of authority of Mendiola was
Civil Cases – Legal Officer
even raised by the municipality itself in its comment and
 Mandatory for Province and City opposition to said counsel's motion for execution of his lien,
 Optional for Municipality (so Prosecutor represents which was filed with the court by the office of the Provincial
based on case law ) Prosecutor of Rizal in behalf of said municipality.
Criminal Cases – Prosecutor; Private Prosecutor allowed
Administrative Cases – Legal officer 5. The contention of Mendiola that PPC cannot raise for the first
LGC and Case Law applied – Provincial Prosecutor still time on appeal his lack of authority to represent the
represents municipality subject to certain exceptions municipality is untenable. The legality of his representation
can be questioned at any stage of the proceedings.

6. Even assuming that the representation of the municipality by


Pililla vs. CA (1994)
Mendiola was duly authorized, said authority is deemed to
have been revoked by the municipality when the latter,
Facts: through the mayor and without said counsel's participation,
In connection with the execution of the judgment in favor of the entered into a compromise agreement with PPC with regard
municipality of Pililla, Atty. Felix Mendiola filed a motion in behalf of to the execution of the judgment in its favor and thereafter
the municipality with the RTC for the examination of the PPC’s filed personally with the court 2 pleadings constitutive of a
gross sales for certain years for the purpose of computing the tax "Satisfaction of Judgment" and a "Release and Quitclaim."
on business imposed under the Local Tax Code. A client, by appearing personally and presenting a motion by
PPC filed a motion questioning Mendiola’s authority to represent himself, is considered to have impliedly dismissed his lawyer.
the municipality. Herein counsel cannot pretend to be authorized to continue
representing the municipality since the latter is entitled to dispense
Held: Atty. Mendiola has no authority to file a petition in behalf and with his services at any time.
in the name of the municipality. The client has also an undoubted right to compromise a suit
Private attys. cannot represent a province or municipality in without the intervention of his lawyer. Even the lawyers' right to
lawsuits. fees from their clients may not be invoked by the lawyers
1. Under Sec. 1683 of the Revised Administrative Code, themselves as a ground for disapproving the approval of a
complemented by Sec. 3 of RA 2264 (Local Autonomy Law), compromise agreement.
only the provincial fiscal and the municipal attorney can
represent a province or municipality in their lawsuits. The 7. That it is impossible that municipality will file a similar petition
provision is mandatory. The municipality's authority to considering that the mayor who controls its legislative body
employ a private lawyer is expressly limited only to will not take the initiative is conjectural and without factual
basis.

The Apocryphal Maggots:


Rainier, Chrisgel, Corina, Geoffry, Grace and Sylvie Blanche
The Flibbertigibbet Worms:
Golda, Gladys and Melyjane
CA VE AT: By simply r ea ding this re vi ew er a t the end o f the sem este r wi ll (hopef ully) guar antee you r pas sing this cou rse. D rink mode rat ely .
Amusin S C A r y
gly
THE C2005 LOCAL GOVERNMENT REVIEWER - 205 -
There is presently a manifestation and motion pending with RTC The order of the court stated that Romanillos "entered his
filed by the mayor for withdrawal of Satisfaction and Quitclaim appearance as collaborating counsel of the provincial prosecutor
previously filed. and the provincial attorney." This collaboration is contrary to
law.
Private lawyers may not represent municipalities on their own.
Ramos vs. CA (1997) Neither may they do so even in collaboration with authorized
government lawyers. This is anchored on the principle that
Facts: only accountable public officers may act for and in behalf of
Petitioners and the Baliuag Market Vendors Association filed a public entities and that public funds should not be expended
petition before the RTC for the declaration of nullity of Municipal to hire private lawyers.
Ordinances Nos91 and 7 and the contract of lease over a
commercial arcade to be constructed in the municipality Petitioners cannot be held in estoppel for questioning the legality of
At the pre-trial conference, Atty. Roberto Romanillos appeared, the appearance of Romanillos, notwithstanding that they
manifesting that he was counsel for the municipality. He also filed a questioned the witnesses of the municipality during the hearing of
motion to dissolve injunction and other pleadings. its motion to dissolve the preliminary injunction.
Provincial Attorney Regalado appeared as collaborating counsel of The legality of the representation of an unauthorized counsel
Romanillos. The Provincial Fiscal did not appear. It was may be raised at any stage of the proceedings.
Romanillos who submitted the Reply to petitioners' Opposition to
the motion to dissolve injunction. Romanillos also submitted a 2. Nevertheless, the adoption by the Provincial Attorney of the
written formal offer of evidence for the municipality. proceedings participated in by Atty. Romanillos has validated
Petitioners questioned the personality of Romanillos to appear as the proceedings.
counsel for the municipality. The petitioners also sought to In sum, although a municipality may not hire a private lawyer
disqualify Romanillos from appearing as counsel for the to represent it in litigations, in the interest of substantial
municipality and to declare null and void the proceedings justice however, a municipality may adopt the work already
participated in by Romanillos. performed in good faith by such private lawyer, which work is
Meanwhile, Romanillos and Regalado filed a joint motion stating beneficial to it
that Romanillos was withdrawing as counsel for the municipality o provided that no injustice is thereby heaped on the
and that Regalado, as his collaborating counsel is adopting the adverse party and
entire proceedings participated in by Romanillos. o provided further that no compensation in any guise is
paid therefor by said municipality to the private lawyer.
Held: Unless so expressly adopted, the private lawyer's work
1. A private lawyer cannot represent the municipality in CAB. cannot bind the municipality.
Under Sec. 1683 of the Revised Administrative Code, private It does not appear that the adoption of proceedings participated in
attorneys cannot represent a province or municipality in or undertaken by Romanillos when he was private counsel for the
lawsuits. municipality would have resulted in any substantial prejudice to
The provincial fiscal's functions as legal officer and adviser for the petitioners' interest.
civil cases of a province and corollarily, of the municipalities To declare the said proceedings null and void — notwithstanding
thereof, were subsequently transferred to the provincial attorney. the formal adoption thereof by Regalado as Provincial Attorney —
and to require trial anew to cover the same subject matter, to hear
Only the provincial fiscal, provincial attorney, and municipal the same witnesses and to admit the same evidence adduced by
attorney should represent a municipality in its lawsuits. Only the same parties cannot enhance the promotion of justice.
in exceptional instances may a private attorney be hired by a
municipality to represent it in lawsuits. The ff. are the exceptional 3. A motion to withdraw the appearance of an unauthorized
instances: lawyer is a non-adversarial motion that need not comply with
a. If and when original jurisdiction of a case involving the Sec14, Rule15 as to notice to the adverse party.
municipality is vested in the SC
b. When the municipality is a party adverse to the provincial
government or to some other municipality in the same Alinsug vs. RTC (1993)
province
c. When, in a case involving the municipality, he, or his wife, or Facts:
child, is pecuniarily involved, as heir, legatee, creditor or Alinsug, received a permanent appointment as Clerk III in the
otherwise. Office of the Municipal Planning and Development Coordinator.
None of the exceptions is present in this case. It may be said She was later detailed to the office of the mayor.
that Romanillos appeared for the municipality inasmuch as he was Mayor Ponsica issued Office Order No. 31suspending Zonsayda
already counsel of Kristi Corporation which was sued with the for one month and one day commencing for "a simple misconduct
municipality in this same case. which can also be categorized as an act of insubordination."
Alinsug petitioned against the Mayor and the municipal treasurer.

The Apocryphal Maggots:


Rainier, Chrisgel, Corina, Geoffry, Grace and Sylvie Blanche
The Flibbertigibbet Worms:
Golda, Gladys and Melyjane
CA VE AT: By simply r ea ding this re vi ew er a t the end o f the sem este r wi ll (hopef ully) guar antee you r pas sing this cou rse. D rink mode rat ely .
Amusin S C A r y
gly
THE C2005 LOCAL GOVERNMENT REVIEWER - 206 -
Mayor Ponsica and the municipal treasurer filed an answer through Private respondents then filed a petition for mandamus, prohibition
private practitioner Samuel SM Lezama, and filed a counterclaim. and injunction with the court of appeals with the prayer, among
Petitioner opposed – since they were sued in their official other things, that their notice to appeal be given due course and
capacities, "not including their private capacities," they should have the trial court be prohibited from enforcing the partial execution of
been represented by either the municipal legal officer or the its judgment. Appellate court rendered a decision favorable to
provincial legal officer or prosecutor as provided for by Sec. 481 private respondents.
(b) [i] and [3] of the Local Government Code.
Held: A private counsel may represent municipal officials sued in
Held: A private counsel may represent municipal officials being their official capacities depending on the nature of the action and
sued in their official capacities. the relief sought.
Section 481, Article 11 of Title V of the Code specifies that a legal In a line of cases, the SC held that only when the provincial fiscal
officer can: is disqualified may the municipal council be authorized to hire the
(i) Represent the local government unit in all civil actions services of a special attorney.
and special proceedings wherein the local government unit or The following are the instances when the provincial public
any official thereof, in his official capacity, is a party: Provided, prosecutor is disqualified from representing a particular
that in actions or proceedings where a component city or municipality:
municipality is a party adverse to the provincial government or to 1. When the jurisdiction of a case involving the municipality lies
another component city or municipality, a special legal officer with the Supreme Court
may be employed to represent the adverse party; 2. When the municipality is a party adverse to the provincial
It appears that the law allows a private counsel to be hired by a government or to some other municipality in the same
municipality only when the municipality is an adverse party in a province
case involving the provincial government or another municipality or 3. When in a case involving the municipality, the provincial
city within the province prosecutor, his spouse, his child is involved as a creditor,
legatee, or otherwise.
Court sanctions the representation by private counsel when the
official charged is found to have exceeded his authority and is In resolving whether a local government official may secure the
therefore personally liable for the effects thereof services of a private counsel in an action filed against him in his
official capacity, the nature of the action and the relief sought are to
Where rigid adherence to the law on representation of local be considered.
officials in court actions could deprive a party of his right to redress Instances when representation by private counsel of a public
for a valid grievance, the hiring of a private counsel would be official sued in his official capacity was allowed:
proper. 1. Where the complaint contained other allegations and a prayer
In Albuera v. Torres: a provincial governor sued in his official for moral damages, which if due from the defendants must be
capacity may engage the services of private counsel when "the satisfied by them in their private capacity.
complaint contains other allegations and a prayer for moral 2. Where rigid adherence to the law on representation would
damages, which, if due from the defendants, must be satisfied by deprive a part of his right to redress for a valid grievance
them in their private capacity."
In view of the damages sought which, if granted, could result in
Look at the nature of the action and the relief sought: personal liability, respondents could not be deemed to have been
CAB: admittedly suit was file against respondents as public improperly represented by private counsel. Hence, CA did not err
officials, BUT when it recognized the right of respondents to be represented by
- Certain allegations were beyond official duties of the officers: private counsel.
harassment and political vendetta
- petition then claims moral and exemplary damages, as well
as litigation expenses
- personal liability of the officials concerned

Mancenido vs. CA (2000) Natividad vs. Felix (1994)

Facts: Facts:
Mancenido filed an action for mandamus and damages with the Natividad was Municipal Mayor of Ramos, Tarlac. An information
RTC against the Provincial Board, the school board, the provincial for the murder of Severino Aquino at the Ramos Police Station was
governor, provincial treasurer and provincial auditor to pay the filed against Mayor Natividad and one Boy Llerina before the sala
teachers’ claim for unpaid salary increases. of Judge Felix of RTC. A warrant for his arrest was issued.
The RTC rendered a decision ordering the Provincial School Board Upon motion, Judge Felix recalled the warrant & remanded the
to appropriate amount as unpaid salary increases and to satisfy case for further prelim investigation.
the claim.

The Apocryphal Maggots:


Rainier, Chrisgel, Corina, Geoffry, Grace and Sylvie Blanche
The Flibbertigibbet Worms:
Golda, Gladys and Melyjane
CA VE AT: By simply r ea ding this re vi ew er a t the end o f the sem este r wi ll (hopef ully) guar antee you r pas sing this cou rse. D rink mode rat ely .
Amusin S C A r y
gly
THE C2005 LOCAL GOVERNMENT REVIEWER - 207 -
The panel of prosecutors submitted an amended information with a
resolution that probable cause exists.  Judge Felix admitted the Masturbation (Part 2)
amended information and again directed the issuance of a warrant
of arrest against Natividad. Masturbation Tips for Girls
Natividad moved to remand the case for prelim investigation with
motion to quash the warrant, contending that the Ombudsman and
If you're masturbating for the first time--or even if you're a
not the Provincial Prosecutor had jurisdiction to conduct the prelim
seasoned pro--take a few moments to relax, heighten your
investigation and that the proper court w/c had jurisdiction over the
senses and explore your body. Dim the lights, turn on some soft,
case was the Sandiganbayan  denied.
sensual music, light a few candles and burn some incense. You
Held: Ombudsman has no jurisdiction to conduct a preliminary may feel a little silly making all these preparations just to have
investigation over the case. sex with yourself, but try it at least once and see if you like it.
The Ombudsman’s primary jurisdiction is dependent on the cases Many women are surprised at their bodies' responses to a
cognizable by the Sandiganbayan, hence, legislation relative to the sensual environment.
jurisdiction of the Sandiganbayan must be traced.
§4 (a)(2) of the latest law on the Sandiganbayan, PD 1861, Once the room is ready, remove your clothes and recline on the
requires that the ff. requisites concur for a case to fall under the bed or sofa. Make sure you're completely comfortable; try
jurisdiction of the Sandigan: propping your elbows on a few pillows. Then begin to explore
- the offense committed by the public officer must be in relation your body. Stroke your breasts, belly and thighs. Feel your skin
to his office; and
raise into goosebumps as you become aroused by your own
- the penalty prescribed must be higher than prision
touch. Don't leave an inch of skin unexplored: search out and
correccional (or imprisonment for 6yrs) or a fine of P6K
CAB: The 1st requirement is absent. touch all the places on your body you don't consider to be
"sexual," such as the backs of your knees, your underarms, the
PD1861 should be taken into consideration in the application & crevice between your vulva and inner thighs. You'll soon find
interpretation of Art. XI §§12-13 of the 1987 Consti and the out what turns you on.
Ombudsman Act of 1989 because PD1861 is in pari materia20 to
Art. XI and the Ombudsman Act. When you're completely comfortable with your body, move
A statute must be interpreted, not only to be consistent w/ itself, but your hands between your legs. If this is difficult or
also to harmonize w/ other laws on the same subject matter, as to uncomfortable for you, start slow. Get out a hand mirror and a
form a complete, coherent and intelligible system. flashlight and look at yourself. Many women have never done
this before and have no idea what they look like. In order to feel
The offense was NOT committed in the performance of the completely comfortable masturbating, you need to understand
mayor’s responsibility to maintain peace & order. (The victim was
your anatomy. Pull your labia apart and examine your clitoris
allegedly a robbery & NPA suspect, under investigation at the time
he was killed at the police station). and vaginal opening. Try stroking your clitoris and watch what
 The alleged act does not fall under any of the functions of happens. As you become aroused, blood will flow to your
municipal mayor under §444 of the LGC. Neither may it be implied clitoris, enlarging it and turning it a deep shade of red--much
therefrom. like a man's erect penis. The sensitive skin below your clitoris,
surrounding your vaginal opening, may become puckered, like
Assuming that the act was in the performance of petitioner’s official the goosebumps on your arms and legs. Don't worry! It's a
functions, thus satisfying the 1st requirement, still the Ombudsman perfectly natural response to arousal.
has only primary jurisdiction over cases cognizable by the (continued…)
Sandiganbayan, not EXCLUSIVE original jurisdiction. His authority
is a shared or concurrent authority with similarly authorized
agencies of the govt in respect of the offense charged.
 §15(1) of RA6770 (Ombudsman Act of 1989) uses the word The Loc al Boa rd s
“may” in regard to the Ombudsman’s assumption of its primary
jurisdiction over cases cognizable by the Sandiganbayan. It follows
Local School Boards
that the Ombudsman’s powers are but directory in nature.
Sec98. Creation, Composition, and Compensation. —
(a) There shall be established in every province, city, or
municipality a provincial, city, or municipal school board,
respectively.
20
(b) The composition of local school boards shall be as follows:
Statutes are in pari materia when they relate to the same person or (1) The provincial school board shall be composed of the
thing or to the same class of persons or things, or object, or cover the governor and the division superintendent of schools as
same specific or particular subject matter.

The Apocryphal Maggots:


Rainier, Chrisgel, Corina, Geoffry, Grace and Sylvie Blanche
The Flibbertigibbet Worms:
Golda, Gladys and Melyjane
CA VE AT: By simply r ea ding this re vi ew er a t the end o f the sem este r wi ll (hopef ully) guar antee you r pas sing this cou rse. D rink mode rat ely .
Amusin S C A r y
gly
THE C2005 LOCAL GOVERNMENT REVIEWER - 208 -
co-chairman; the chairman of the education committee of revenue as this Code and other laws or ordinances may
of the sangguniang panlalawigan, the provincial provide;
treasurer, the representative of the "pederasyon ng mga (b) Authorize the provincial, city or municipal treasurer, as the
sangguniang kabataan" in the sangguniang case may be, to disburse funds from the Special Education
panlalawigan, the duly elected president of the provincial Fund pursuant to the budget prepared and in accordance with
federation of parents-teachers associations, the duly existing rules and regulations;
elected representative of the teachers' organizations in (c) Serve as an advisory committee to the sanggunian
the province, and the duly elected representative of the concerned on educational matters such as, but not limited to,
non-academic personnel of public schools in the the necessity for and the uses of local appropriations for
province, as members; educational purposes; and
(2) The city school board shall be composed of the city mayor (d) Recommend changes in the names of public schools within
and the city superintendent of schools as co-chairmen; the territorial jurisdiction of the local government unit for
the chairman of the education committee of the enactment by the sanggunian concerned.
sangguniang panlungsod, the city treasurer, the The Department of Education, Culture and Sports shall consult
representative of the "pederasyon ng mga sangguniang the local school board on the appointment of division
kabataan" in the sangguniang panlungsod, the duly superintendents, district supervisors, school principals, and
elected president of the city federation of parents- other school officials.
teachers associations, the duly elected representative of
the teachers' organizations in the city, and the duly
elected representative of the non-academic personnel of Sec100. Meetings and Quorum; Budget. —
public schools in the city, as members; and (a) The local school board shall meet at least once a month or
(3) The municipal school board shall be composed of the as often as may be necessary.
municipal mayor and the district supervisor of schools as (b) Any of the co-chairmen may call a meeting. A majority of all
co-chairmen; the chairman of the education committee its members shall constitute a quorum. However, when both co-
of the sangguniang bayan, the municipal treasurer, the chairmen are present in a meeting, the local chief executive
representative of the "pederasyon ng mga sangguniang concerned, as a matter of protocol, shall be given preference to
kabataan" in the sangguniang bayan, the duly elected preside over the meeting. The division superintendent, city
president of the municipal federation of parent-teacher superintendent or district supervisor, as the case may be, shall
associations, the duly elected representative of the prepare the budget of the school board concerned. Such budget
teachers' organizations in the municipality, and the duly shall be supported by programs, projects, and activities of the
elected representative of the non-academic personnel of school board for the ensuing fiscal year. The affirmative vote of
public schools in the municipality, as members. the majority of all the members shall be necessary to approve
(c) In the event that a province or city has two (2) or more the budget.
school superintendents, and in the event that a municipality has (c) The annual school board budget shall give priority to the
two (2) or more district supervisors, the co-chairman of the local following:
school board shall be determined as follows: (1) Construction, repair, and maintenance of school buildings
(1) The Department of Education, Culture and Sports shall and other facilities of public elementary and secondary
designate the co-chairman for the provincial and city schools;
school boards; and (2) Establishment and maintenance of extension classes where
(2) The division superintendent of schools shall designate the necessary; and
district supervisor who shall serve as co-chairman of the (3) Sports activities at the division, district, municipal, and
municipal school board. barangay levels.
(d) The performance of the duties and responsibilities of the
abovementioned officials in their respective local school boards Sec101. Compensation and Remuneration. — The co-chairmen
shall not be delegated. and members of the provincial, city or municipal school board
shall perform their duties as such without compensation or
Sec99. Functions of Local School Boards. — The provincial, remuneration. Members thereof who are not government
city or municipal school board shall: officials or employees shall be entitled to necessary traveling
(a) Determine, in accordance with the criteria set by the expenses and allowances chargeable against the funds of the
Department of Education, Culture and Sports, the annual local school board concerned, subject to existing accounting
supplementary budgetary needs for the operation and and auditing rules and regulations.
maintenance of public schools within the province, city, or
municipality, as the case may be, and the supplementary local Cebu Province COA v. Province of Cebu (2001)
cost of meeting such as needs, which shall be reflected in the
form of an annual school board budget corresponding to its The salaries and personnel-related benefits of the public school
share of the proceeds of the special levy on real property teachers in connection with the establishment and maintenance of
constituting the Special Education Fund and such other sources extension classes may be charged to the SEF. The deliberations
on the LGC clearly demonstrated that the legislature intended the

The Apocryphal Maggots:


Rainier, Chrisgel, Corina, Geoffry, Grace and Sylvie Blanche
The Flibbertigibbet Worms:
Golda, Gladys and Melyjane
CA VE AT: By simply r ea ding this re vi ew er a t the end o f the sem este r wi ll (hopef ully) guar antee you r pas sing this cou rse. D rink mode rat ely .
Amusin S C A r y
gly
THE C2005 LOCAL GOVERNMENT REVIEWER - 209 -
SEF to answer for the compensation of teachers handling
extension classes.
• The appointment issued by Ramos in favor of Malaya as
Schools Division Superintendent did not specify her station.
Even under the Doctrine of Necessary Implication, the allocation It was Sec Gloria who assigned respondent to the Division of
of the SEF for the establishment and maintenance of extension Camarines Sur, and Osea to the division of Iriga City.
classes logically implies the hiring of teachers who should, as a • Hence, these designations partook of a nature of
matter of course, be compensated for their services. reassignments. These do not need prior consultation with the
local school board.
The operation and maintenance of public schools are lodged • Appointment: the selection, by the authority vested with the
principally with the DECS. This is why only salaries of public
power, of an individual who is to exercise the functions of a
school teachers appointed for extension classes pertain to the
given office.
supplementary budget of the local school boards. So, not every
Reassignment: merely a movement of an employee
kind of personnel-related benefits may be charged to the SEF.
from one organizational unit to another in the same
department or agency which does not involve a reduction in
But scholarship grants are not among the projects chargeable to
rank, status or salary and does not require the issuance of an
the SEF. This item was omitted intentionally from the LGC. The
appointment.
Doctrine of Necessary Implication cannot be applied, since
scholarship grants are neither necessary nor indispensable to the
operation and maintenance of public schools. Such grants should From class notes:
be charged instead to the General Funds of the province. Transfer of appointment by virtue of memorandum circular has
become an accepted practice. But sir doubts if this is correct.

Osea v. Malaya (2002)


Local Health Boards
Osea prays for the recall of respondent's appointment. She avers
that Sec102. Creation and Composition. — (a) There shall be
established a local health board in every province, city, or
1. she was appointed as Officer-in-charge, Assistant Schools
municipality. The composition of the local health boards shall be
Division Superintendent of Camarines Sur by DECS Sec. as follows:
Gloria, upon endorsement of the Provincial School Board of (1) The provincial health board shall be headed by the governor
Camarines Sur as chairman, the provincial health officer as vice-chairman, and
2. despite the recommendation of Gloria, Pres. Ramos instead the chairman of the committee on health of the sangguniang
appointed Malaya to the position of Schools Division panlalawigan, a representative from the private sector or non-
Superintendent of Camarines Sur governmental organizations involved in health services, and a
3. Malaya's appointment was made without prior consultation representative of the Department of Health in the province, as
with the Provincial School Board, in violation of Section 99 of members;
the LGC. (2) The city health board shall be headed by the city mayor as
CSC found that Ramos appointed respondent from OIC Schools chairman, the city health officer as vice-chairman, and the
Division Superintendent of Iriga City to Schools Division chairman of the committee on health of the sangguniang
Superintendent without any specific Division. Subsequently, Gloria panlungsod, a representative from the private sector or non-
designated respondent as Schools Division Superintendent of governmental organizations involved in health services, and a
Camarines Sur and petitioner as Schools Division representative of the Department of Health in the city, as
Superintendent of Iriga City. members; and
(3) The municipal health board shall be headed by the municipal
Held: Osea’s petition has no merit. mayor as chairman, the municipal health officer as vice-
• Sec. 99 applies to appointments made by the DECS, because chairman, and the chairman of the committee on health of the
when the LGC was enacted, Schools Division sangguniang bayan, a representative from the private sector or
Superintendents were appointed by the DECS to specific non-governmental organizations involved in health services, and
division or location. In 1994, the Career Executive Service a representative of the Department of Health in the municipality,
Board issued Memorandum Circular 21 placing the positions as members.
of Schools Division Superintendent within the career (b) The functions of the local health board shall be:
executive service. Consequently, the power to appoint (1) To propose to the sanggunian concerned, in accordance with
persons to career executive service positions was standards and criteria set by the Department of Health, annual
transferred from the DECS to the President. The appointment budgetary allocations for the operation and maintenance of
may not be specific as to location. The perogative to health facilities and services within the municipality, city or
designate the appointees to their particular stations was province, as the case may be;
vested in the DECS. (2) To serve as an advisory committee to the sanggunian
concerned on health matters such as, but not limited to, the

The Apocryphal Maggots:


Rainier, Chrisgel, Corina, Geoffry, Grace and Sylvie Blanche
The Flibbertigibbet Worms:
Golda, Gladys and Melyjane
CA VE AT: By simply r ea ding this re vi ew er a t the end o f the sem este r wi ll (hopef ully) guar antee you r pas sing this cou rse. D rink mode rat ely .
Amusin S C A r y
gly
THE C2005 LOCAL GOVERNMENT REVIEWER - 210 -
necessity for, and application of local appropriations for public than one fourth (¼) of the members of the fully
health purposes; and organized council;
(3) Consistent with the technical and administrative standards of (3) A representative of the congressman.
the Department of Health, create committees which shall advise
local health agencies on matters such as, but not limited to, (b) The city or municipal development council shall be
personnel selection and promotion, bids and awards, grievance headed by the mayor and shall be composed of the following
and complaints, personnel discipline, budget review, operations members:
review and similar functions.
(1) All punong barangays in the city or municipality;
(2) The chairman of the committee on appropriations of the
Sec103. Meetings and Quorum. — sangguniang panlungsod or sangguniang bayan
(a) The board shall meet at least once a month or as may be concerned;
necessary. (3) The congressman or his representative; and
(b) A majority of the members of the board shall constitute a (4) Representatives of non-governmental organizations
quorum, but the chairman or the vice- chairman must be present operating in the city or municipality, as the case may be,
during meetings where budgetary proposals are being prepared who shall constitute not less than one-fourth (¼) of the
or considered. The affirmative vote of all the majority of the members of the fully organized council.
members shall be necessary to approve such proposals.
(c) The provincial development council shall be headed by
Sec104. Compensation and Remuneration. — The chairman, the governor and shall be composed of the following members:
vice-chairman, and members of the provincial, city or municipal (1) All mayors of component cities and municipalities;
health board shall perform their duties as such without (2) The chairman of the committee on appropriations of the
compensation or remuneration. Members thereof who are not sangguniang panlalawigan;
government officials or employees shall be entitled to necessary (3) The congressman or his representative; and
traveling expenses and allowances chargeable against the (4) Representatives of non-governmental organizations
funds of the local health board concerned, subject to existing operating in the province, who shall constitute not less
accounting and auditing rules and regulations. than one-fourth (¼) of the members of the fully
organized council.
Sec105. Direct National Supervision and Control by the
Secretary of Health. — In cases of epidemics, pestilence, and (d) The local development councils may call upon any local
other widespread public health dangers, the Secretary of Health official concerned or any official of national agencies or offices in
may, upon the direction of the President and in consultation with the local government unit to assist in the formulation of their
the local government unit concerned, temporarily assume direct respective development plans and public investment programs.
supervision and control over health operations in any local
government unit for the duration of the emergency, but in no Sec108. Representation of Non-governmental Organizations.
case exceeding a cumulative period of six (6) months. With the — Within a period of sixty (60) days from the start of
concurrence of the government unit concerned, the period for organization of local development councils, the non-
such direct national control and supervision may be further governmental organizations shall choose from among
extended. themselves their representatives to said councils. The local
sanggunian concerned shall accredit non-governmental
Local Development Councils organizations subject to such criteria as may be provided by law.

Sec106. Local Development Councils. — (a) Each local Sec109. Functions of Local Development Councils. —
government unit shall have a comprehensive multi-sectoral (a) The provincial, city, and municipal development councils
development plan to be initiated by its development council and shall exercise the following functions:
approved by its sanggunian. For this purpose, the development
council at the provincial, city, municipal, or barangay level, shall (1) Formulate long-term, medium-term, and annual socio-
assist the corresponding sanggunian in setting the direction of economic development plans and policies;
economic and social development, and coordinating (2) Formulate the medium-term and annual public investment
development efforts within its territorial jurisdiction. programs;
(3) Appraise and prioritize socio-economic development
Sec107. Composition of Local Development Councils. — The programs and projects;
composition of the local development council shall be as follows: (4) Formulate local investment incentives to promote the inflow
and direction of private investment capital;
Barangay Local Development Councils: (5) Coordinate, monitor, and evaluate the implementation of
(1) Members of the sangguniang barangay; development programs and projects; and
(2) Representatives of non-governmental organizations (6) Perform such other functions as may be provided by law or
operating in the barangay who shall constitute not less component authority.

The Apocryphal Maggots:


Rainier, Chrisgel, Corina, Geoffry, Grace and Sylvie Blanche
The Flibbertigibbet Worms:
Golda, Gladys and Melyjane
CA VE AT: By simply r ea ding this re vi ew er a t the end o f the sem este r wi ll (hopef ully) guar antee you r pas sing this cou rse. D rink mode rat ely .
Amusin S C A r y
gly
THE C2005 LOCAL GOVERNMENT REVIEWER - 211 -
required in the discharge of its functions. The local development
(b) The barangay development council shall exercise the council may avail of the services of any non-governmental
following functions: organization or educational or research institution for this
(1) Mobilize people's participation in local development efforts; purpose.
(2) Prepare barangay development plans based on local The secretariats of the provincial, city, and municipal
requirements; development councils shall be headed by their respective
(3) Monitor and evaluate the implementation of national or local planning and development coordinators. The secretariat of the
programs and projects; and barangay development council shall be headed by the barangay
(4) Perform such other functions as may be provided by law or secretary who shall be assisted by the city or municipal planning
competent authority. and development coordinator concerned.

Sec110. Meetings and Quorum. — The local development Sec114. Relation of Local Development Councils to the
council shall meet at least once every six (6) months or as often Sanggunian and the Regional Development Council. —
as may be necessary. (a) The policies, programs, and projects proposed by local
development councils shall be submitted to the sanggunian
Sec111. Executive Committee. — The local development council concerned for appropriate action.
shall create an executive committee to represent it and act in its (b) The approved development plans of provinces, highly-
behalf when it is not in session. The composition of the urbanized cities, and independent component cities shall be
executive committee shall be as follows: submitted to the regional development council, which shall be
(1) The executive committee of the provincial development integrated into the regional development plan for submission to
council shall be composed of the governor as chairman, the the National Economic and Development Authority, in
representative of component city and municipal mayors to be accordance with existing laws.
chosen from among themselves, the chairman of the committee
on appropriations of the sangguniang panlalawigan, the Sec115. Budget Information. — The Department of Budget and
president of the provincial league of barangays, and a Management shall furnish the various local development
representative of non-governmental organizations that are councils information on financial resources and budgetary
represented in the council, as members; allocations applicable to their respective jurisdictions to guide
(2) The executive committee of the city or municipal them in their planning functions
development council shall be composed of the mayor as
chairman, the chairman of the committee on appropriations of
the sangguniang panlalawigan, the president of the city or RA 7368 (1992): An Act to Launch and Promote
municipal league of barangays, and a representative of non- Countrywide Industrialization Through The
governmental organizations that are represented in the council, Establishment Of Manufacturing, Processing And Related
as members; and aisa dc Industries By Providing Financial Assistance To
(3) The executive committee of the barangay development Enterprises In Every Town And City Not Exceeding P30
council shall be composed of the punong barangay as Million And P40 Million, Respectively, Authorizing The
chairman, a representative of the sangguniang barangay to be Appropriation Therefor In The Amount Of P1 Billion Per
chosen from among its members, and a representative of non- Year For The Next Five Years As Initial Capital, And For
governmental organizations that are represented in the council, Other Purposes
as members.
(b) The executive committee shall exercise the following powers Sec6. Executive Committee of Local Development Council as
and functions: the Local Countrywide Industrialization Board. — The
(1) Ensure that the decision of the council are faithfully carried Executive Committee of the municipal, city and provincial
out and implemented; development councils shall serve as the Local Countrywide
(2) Act on matters requiring immediate attention or action by the Industrialization Board (LCIB) which shall be directed towards
council; the development of technology and skills, assist all enterprises
(3) Formulate policies, plans, and programs based on the in the utilization of indigenous raw materials for livelihood
general principles laid down by the council; and programs and in the delivery of credit organizations and
(4) Act on other matters that may be authorized by the council. marketing services.
In addition to the functions of the local development councils as
Sec112. Sectoral or Functional Committees. — The local mandated in the Local Government Code of 1991 (Republic Act
development councils may form sectoral or functional No. 7160), they shall perform the following functions:
committees to assist them in the performance of their functions. a) receive all applications for countrywide industrialization
projects within its area of jurisdiction; evaluate and approve or
Sec113. Secretariat. — There is hereby constituted for each local disapprove all project applications within thirty (30) days from
development council a secretariat which shall be responsible for receipt thereof; and endorse projects to the Countrywide
providing technical support, documentation of proceedings, Industrialization Office (CIO) for funding;
preparation of reports and such other assistance as may be

The Apocryphal Maggots:


Rainier, Chrisgel, Corina, Geoffry, Grace and Sylvie Blanche
The Flibbertigibbet Worms:
Golda, Gladys and Melyjane
CA VE AT: By simply r ea ding this re vi ew er a t the end o f the sem este r wi ll (hopef ully) guar antee you r pas sing this cou rse. D rink mode rat ely .
Amusin S C A r y
gly
THE C2005 LOCAL GOVERNMENT REVIEWER - 212 -
b) in coordination with the conduit banks, monitor the status of Towards this end, the State shall endeavor to establish special
approved projects, help enterprises obtain marketing, technical economic zones in suitable and selected areas in the country
training and other forms of non-financial assistance as they may where enterprises will be given incentives to create an
require from agencies of the National and Local Governments; environment conducive to business. This shall be the means to
c) review the industrialization plan for the province, city or attract local and foreign investors, general employment
municipality as prepared by the municipal, city or provincial opportunities, and encourage the regional dispersal of
development officer if the funding required for the proposed industries.
project is to be sourced from CIF;
d) disseminate pertinent information; and Sec3. Creation of the Zamboanga City Special Economic
e) submit quarterly reports to the CIO on the amounts of Zone. — In accordance with the foregoing policy and subject to
financial assistance received by the countrywide industrialization the concurrence of the city government of Zamboanga affected
projects within its jurisdiction, and a status report on each by the zone, there is hereby established a special economic
countrywide industrialization project. Copy of the report shall be zone in the City of Zamboanga to be known as the Zamboanga
furnished the Sangguniang Bayan or Sangguniang Panlungsod City Special Economic Zone, hereinafter referred to as the
concerned. ZAMBOECOZONE. The specific metes and bounds of the
ZAMBOECOZONE shall be more particularly defined in a
presidential proclamation that shall be issued for this purpose.
Local Peace and Order Council
Sec4. Governing Principles. — The Zamboanga City Special
Sec116. Organization. — There is hereby established in every Economic Zone shall be managed and operated under the
province, city and municipality a local peace and order council, following principles:
pursuant to (E.O. No. 309), as amended, Series of 1988. The (a) Within the framework and limitations of the Constitution and
local peace and order councils shall have the same composition the applicable provisions of the Local Government Code, the
and functions as those prescribed by said executive order. ZAMBOECOZONE shall be developed into a decentralized, self-
reliant and self-sustaining agro-industrial, commercial, financial,
investment and tourist center and free port with suitable
retirement and residential areas. The ZAMBOECOZONE shall
Autonomous Special Economic Zones
be provided with transportation, telecommunications, and other
facilities needed to attract legitimate and productive foreign
Sec117. Establishment of Autonomous Special Economic investments, generate linkage industries and employment
Zones. — The establishment by law of autonomous special opportunities for the people of Zamboanga City and its
economic zones in selected areas of the country shall be neighboring towns and cities.
subject to concurrence by the local government units included (b) The ZAMBOECOZONE may establish mutually beneficial
therein. economic relations with other entities within the country, subject
to the administrative guidance of the Department of Foreign
Affairs and/or Department of Trade and Industry with foreign
RA 7916 (1995) – An Act Providing For The Legal entities or enterprises.
Framework And Mechanisms For The Creation, Operation, (c) Foreign citizens and companies owned by non-Filipinos in
Administration, And Coordination Of Special Economic whatever proportion may set up enterprises in the
Zones In The Philippines, Creating For This Purpose, The ZAMBOECOZONE, either by themselves or in joint venture with
Philippine Economic Zone Authority (Peza), And For Other Filipinos in any sector of industry, international trade and
Purposes commerce within the ZAMBOECOZONE.
(d) The ZAMBOECOZONE shall be managed and operated as a
Sec44. Relationship with the Local Government Units. — separate customs territory to ensure and facilitate the free flow,
Except as herein provided, the local government units entry and movement of machinery and other goods. It shall be
comprising the ECOZONE shall retain their basic autonomy and vested with the authority to issue certificates of origin for
identity. The cities shall be governed by their respective products manufactured or processed in the ZAMBOECOZONE
charters and the municipalities shall operate and function in in accordance with prevailing rules of origin, and the pertinent
accordance with RA7160, otherwise known as the Local regulations of the duly recognized national bodies tasked to
Government Code of 1991. oversee all other ECOZONES in the country.
(e) Business establishments within the ZAMBOECOZONE shall
RA 7903 (1995) – The Zamboanga City Special Economic be entitled to the existing fiscal incentives as provided for under
Zone Act of 1995 Presidential Decree No. 66, the law creating the Export
xxx Processing Zone Authority, or those provided under Book VI of
Sec2. Declaration of Policy. — It is hereby declared the policy of Executive Order No. 226, otherwise known as the Omnibus
the State to encourage and promote the attainment of a sound Investment Code of 1987, and such incentives, benefits or
and balanced industrial, economic and social development in privileges presently enjoyed by business establishments
the country through the promotion of private enterprises.

The Apocryphal Maggots:


Rainier, Chrisgel, Corina, Geoffry, Grace and Sylvie Blanche
The Flibbertigibbet Worms:
Golda, Gladys and Melyjane
CA VE AT: By simply r ea ding this re vi ew er a t the end o f the sem este r wi ll (hopef ully) guar antee you r pas sing this cou rse. D rink mode rat ely .
Amusin S C A r y
gly
THE C2005 LOCAL GOVERNMENT REVIEWER - 213 -
operating within the Subic special economic zone pursuant to foreign borrowings, foreign investments, establishment and
Republic Act No. 7227. acd operation of local and foreign banks, foreign currency deposit
(f) Any provisions of existing laws, rules or regulations to the units, offshore banking units and other financial institutions
contrary notwithstanding, no taxes, local and national, shall be under the supervision of the BSP.
imposed on business establishments operating within the
ZAMBOECOZONE. In lieu of paying taxes, said business Sec5. Creation of the Zamboanga City Special Economic Zone
establishments shall pay and remit to the national government Authority. — Subject to the concurrence of the local
two percent (2%) of their gross income. In addition, they shall government units that will be affected by the creation of the
remit to the city government of Zamboanga three percent (3%) ZAMBOECOZONE, there is hereby created a body corporate to
of their gross income, to be allocated as follows: be known as the Zamboanga City Special Economic Zone
(1) 2% to the City of Zamboanga; and Authority, hereinafter referred to as the ZAMBOECOZONE
(2) 1% to the barangay special development fund, which is Authority, which shall manage and operate the
hereby created, for the development and improvement of ZAMBOECOZONE, in accordance with this Act. It shall be
the barangays within the City of Zamboanga. organized within one hundred eighty (180) days after the
(g) Except as otherwise provided herein, the LGU/s embraced effectivity of this Act.
within the ZAMBOECOZONE shall retain and maintain their
basic autonomy and identity. Zamboanga City shall operate and Sec6. Principal Office. — The ZAMBOECOZONE Authority shall
function in accordance with RA No. 7160, otherwise known as maintain its principal office in the City of Zamboanga, but it may
the Local Government Code of 1991. establish branches and agencies within the Philippines and
(h) nAny foreign investor who establishes a business enterprise abroad as may be necessary for the proper conduct of its
within the ZAMBOECOZONE and who maintains capital business.
investment of not less than US$150,000 shall be granted, along
with his or her spouse, dependents, and unmarried children Sec7. Powers and Functions of the ZAMBOECOZONE
below 21 years of age, a permanent resident status within the Authority. — The ZAMBOECOZONE Authority shall have the
ZAMBOECOZONE. following functions:
Such foreign investor and his or her spouse, dependents and (a) To operate, administer, and manage the ZAMBOECOZONE
unmarried children below the age of 21 years, shall have according to the principles and provisions set forth in this Act;
freedom of ingress and egress to and from the (b) To recommend to the President the issuance of a
ZAMBOECOZONE without any need of any special proclamation to fix and delimit the site of the ZAMBOECOZONE;
authorization from the Bureau of Immigration and Deportation. (c) To register, regulate and supervise the enterprises in the
Likewise, the Zamboanga City Special Economic Zone Authority ZAMBOECOZONE in an efficient and decentralized manner;
shall issue working visas renewable every 2 years to foreign (d) To regulate and undertake the establishment, operation and
executives and foreign technicians with highly specialized skills maintenance of utilities, other services and infrastructure in the
which no Filipino possesses, as certified by the Department of ZAMBOECOZONE such as heat, light and power, water supply,
Labor and Employment. telecommunications, transport, toll roads and bridges, port
The names of the foreigners granted permanent resident status services, etc., and to fix reasonable and competitive rates, fares,
and working visas by the Zamboanga City Special Economic charges and prices therefor;
Zone Authority shall be reported to the Bureau of Immigration (e) To construct, acquire, own, lease, operate and maintain on
and Deportation within 30 days from such grant. its own or through others by virtue of contracts, franchises,
The foregoing is without prejudice to a foreigner acquiring licenses, or permits under any of the schemes allowed in
permanent resident status in the Philippines in accordance with Republic Act No. 6957 (the build-operate-transfer law), or in joint
applicable immigration, retirement, and other related laws. venture with the private sector, any or all of the public utilities
(i) The provisions of any law to the contrary notwithstanding, any and infrastructure required or needed in the ZAMBOECOZONE
foreigner, partnership, corporation, or any other business in coordination with appropriate national and local government
association not created and existing under the laws of the authorities and in conformity with applicable laws thereon;
Republic of the Philippines, engaged in the business of retailing (f) To operate on its own, either directly or through a subsidy
goods and merchandise, shall be permitted to engage in the entity, or license to other tourism related activities, including
business in such retail trade within the ZAMBOECOZONE after games, amusements and recreational and sports facilities;
securing license for that purpose from the Zamboanga City (g) Within the limitation provided by law, to raise or borrow
Special Economic Zone Authority: Provided, That only foreign adequate and necessary funds from local or foreign sources to
nationals engaged in medium- and large-scale retail trade may finance its projects and programs under this Act, and for that
be permitted to engage in such business within the purpose to issue bonds, promissory notes, and other forms of
ZAMBOECOZONE. The determination of the medium- and securities, and to secure the same by a guarantee, pledge,
large-scale retail trade operation shall be the responsibility of mortgage, deed of trust, or an assignment of all part of its
the ZAMBOECOZONE Authority. property or assets;
(j) Existing banking laws and Bangko Sentral ng Pilipinas (BSP) (h) To provide security for the ZAMBOECOZONE in coordination
rules and regulations shall apply on foreign exchange and other with national and local governments. For this purpose, the
current account transactions (trade and non-trade), local and ZAMBOECOZONE Authority may establish and maintain its own

The Apocryphal Maggots:


Rainier, Chrisgel, Corina, Geoffry, Grace and Sylvie Blanche
The Flibbertigibbet Worms:
Golda, Gladys and Melyjane
CA VE AT: By simply r ea ding this re vi ew er a t the end o f the sem este r wi ll (hopef ully) guar antee you r pas sing this cou rse. D rink mode rat ely .
Amusin S C A r y
gly
THE C2005 LOCAL GOVERNMENT REVIEWER - 214 -
security force and firefighting capability or hire others to provide The city's congressional representative, the mayor of the City of
the same; Zamboanga and the representative of the city council shall
(i) To protect, preserve, maintain and develop the virgin forests, serve as ex officio voting members of the Board.
beaches, coral and functional units or offices within the The chairman and the members of the Board, except the ex
ZAMBOECOZONE. officio members, shall be appointed by the President of the
(j) To create, operate and/or contract to operate such agencies Philippines to serve for a term of six (6) years, unless sooner
and functional units or offices of the ZAMBOECOZONE removed for cause or dies or resigns voluntarily. In case of
Authority as it may deem necessary; death, resignation or removal for cause, the replacement shall
(k) To adopt, alter and use a corporate seal; make contracts, serve only the unexpired portion of the term.
leases, own or otherwise dispose of personal or real property; Except for the representatives of the business and investment
sue and be sued; and otherwise carry out its functions and sectors no person shall be appointed by the President of the
duties as provided for in this Act; and Philippines as member of the Board unless he is a Filipino
(l) To issue rules and regulations consistent with the provisions citizen, of good moral character and of recognized competence
of this Act as may be necessary to implement and accomplish in some relevant fields of business, banking, shipping, business
the purposes, objectives, and policies provided therein. or labor management, port operations, engineering or law.
Members of the Board shall receive a reasonable per diem
Sec8. Non-profit Character of the ZAMBOECOZONE Authority. which shall be fixed by the President of the Philippines once
— The ZAMBOECOZONE Authority shall be non-profit and shall every six (6) years for every Board meeting: Provided, however,
devote the use of its returns from capital investments, as well as That the total per diem collected each month shall not exceed
excess revenues from its operations, for the development, the equivalent per diems for four (4) meetings. Unless and until
improvement, and maintenance and other related expenditures the President of the Philippines has fixed a higher per diem for
of the ZAMBOECOZONE Authority to pay its indebtedness and the members of the Board, such per diem shall not be more
obligations and in furtherance and effective implementation of than Ten thousand pesos (P10,000.00) for every Board meeting.
the policy provided in this Act. In consonance with this, the
ZAMBOECOZONE Authority is hereby declared exempt from Sec10. Powers and Duties of the Chairman-Administrator. —
the payment of all taxes, duties, fees, imposts, charges, costs The chairman-administrator shall have the following powers and
and service fees in any court or administrative proceedings in duties:
which it may be a party. (a) To direct and manage the affairs of the Authority in
The foregoing exemptions may however be entirely or partially accordance with the policies of the Board;
lifted by the President of the Philippines upon the (b) To establish the internal organization of the Authority under
recommendation of the Secretary of Finance, not earlier than 5 such conditions that the Board may prescribe;
years from the effectivity of this Act, if the President shall find the (c) To submit an annual budget and necessary supplemental
Authority to be self-sustaining and financially capable by then to budget to the Board for its approval;
pay such taxes, customs duties, fees and other charges after (d) To submit within thirty (30) days after the close of each fiscal
providing for debt service requirements of the year an annual report to the Board and such other reports as
ZAMBOECOZONE Authority and of its projected capital and may be required; and
operating expenditures. (e) To perform such other duties as may be assigned to him by
the Board.
Sec9. Board of Directors of the ZAMBOECOZONE Authority.
— The powers of the ZAMBOECOZONE Authority shall be Sec11. Promotion of Industrial Peace. — One representative
vested in and exercised by a Board of Directors, hereinafter each from the Department of Labor and Employment (DOLE),
referred to as the Board, which shall be composed of the labor sector, cultural minorities, business and industry sectors
following: shall formulate a mechanism under a social pact for the
(a) A chairman who shall, at the same time, be the administrator enhancement and preservation of industrial peace in the City of
of the ZAMBOECOZONE Authority; Zamboanga within thirty (30) days after the effectivity of this Act.
(b) A vice-chairman who shall come from the national agency
tasked to coordinate and monitor special economic zones and Sec12. Capitalization. — The Zamboanga City Special Economic
the like in the country; Zone Authority shall have an authorized capital stock of two
(c) 6 members consisting of: billion (2,000,000,000) no par shares with a minimum issue
(1) The city's congressional representative; value of Ten pesos (P10.00) each. The national government
(2) The mayor of the City of Zamboanga; shall initially subscribe and fully pay three hundred million
(3) 1 representative of the city council; (300,000,000) shares of such capital stock. The initial amount
(4) 1 representative from domestic investors in the necessary to subscribe and pay for the shares of stock shall be
ZAMBOECOZONE; included in the General Appropriations Act of the year following
(5) 1 representative from foreign investors in the its enactment into law and thereafter. The Board of Directors of
ZAMBOECOZONE; and the ZAMBOECOZONE Authority may, from time to time and with
(6) 1 representative from the labor sector chosen from the the written concurrence of the Secretary of Finance, increase
workers in the ZAMBOECOZONE. the issue value of the shares representing the capital stock of

The Apocryphal Maggots:


Rainier, Chrisgel, Corina, Geoffry, Grace and Sylvie Blanche
The Flibbertigibbet Worms:
Golda, Gladys and Melyjane
CA VE AT: By simply r ea ding this re vi ew er a t the end o f the sem este r wi ll (hopef ully) guar antee you r pas sing this cou rse. D rink mode rat ely .
Amusin S C A r y
gly
THE C2005 LOCAL GOVERNMENT REVIEWER - 215 -
the ZAMBOECOZONE Authority. The Board of Directors of the
• Try tracing the alphabet on your clitoris with your index
ZAMBOECOZONE Authority, with the written concurrence of the
Secretary of Finance, may sell shares representing not more finger. This technique may not bring you to orgasm, but it
than forty per centum (40%) of the capital stock of the might clue you in to some sensations otherwise
ZAMBOECOZONE Authority to the general public with such undiscovered.
annual dividend policy as the Board and the Secretary of • Use one hand to separate and hold apart your labia, fully
Finance may determine. The national government shall in no exposing your clitoris. Dip the index finger from your
case own less than sixty per centum (60%) of the total issued other hand in some lubricant--from your body or from a
and outstanding capital stock of the ZAMBOECOZONE bottle--and gently tap your clitoris. Vary speed and
Authority.
intensity and be patient: as you tap harder and longer, a
Sec13. Supervision and Coordination of Development Plans. wonderful sensation will build until you feel like you're
— For purposes of policy direction and coordination, the going to explode.
ZAMBOECOZONE Authority shall be under the direct control • When you're aroused and lubricated, gently thrust one or
and supervision of the Office of the President, in the meantime two fingers in and out of your vagina. Many women who
that the agency tasked with the coordination of special believe they can only achieve orgasm through direct
economic zones is not yet in place. clitoral stimulation are surprised to find that penetration
can also bring them to a rousing climax.
Sec14. Relationship with the Local Government Units. — In
case of any conflict between the ZAMBOECOZONE Authority • While stimulating your clitoris with one hand, try
and the City of Zamboanga on matters affecting the thrusting a few fingers into your vagina with your other
ZAMBOECOZONE other than defense and security matters, the hand. The combination of clitoral and vaginal stimulation
decision of the ZAMBOECOZONE Authority shall prevail. can be very exciting.
• Try any of the above techniques while lying on your belly.
Sec15. Auditing. — The Commission on Audit shall appoint a
representative who shall be a full time auditor of the Some women prefer the slight difference in stimulation
ZAMBOECOZONE Authority and assign such number of and the gentle "humping" motion they can make while
personnel as may be necessary to assist said representative in enjoying this position.
the performance of his/her duties. The salaries and emoluments • Try stimulating your G-Spot: lie on your back with your
of the assigned auditor and personnel shall be in accordance knees raised and insert your middle finger into your
with pertinent laws, rules and regulations. vagina in an inverted "come hither" motion. About two
inches into your vagina, on the wall closest to your
Masturbation: Tips for Girls (Part 3): stomach, you'll feel a spongy, slightly raised area. That's
your G-Spot, also known as your urethral sponge. Because
As you become more comfortable and aroused, set aside the the sensitive portion of this area is actually on the other
mirror and flashlight and try to completely relax. Continue to side of several layers of skin, it will respond best to
stroke your clitoris, mons pubis and vaginal opening. pressure, not stroking. Try pressing your G-Spot with
Concentrate on the areas that feel the best. Once you are fully varying intensities. If you don't like it, just stop doing it. G-
aroused, you'll most likely become wet with vaginal lubrication. Spot stimulation isn't for everyone. However, if you do like
Try inserting a finger or two and see how that feels. Some it, try simultaneously stroking your clitoris with your other
women enjoy penetration when masturbating, some don't. hand. Rumor has it that orgasms produced by G-Spot
Neither way is better. That's one of the joys of self-love--you can stimulation can be very intense.
engage in only the stimulation that you enjoy, nothing more and
nothing less. You're in it for yourself alone. Try different types One rule to remember--regardless of the technique you prefer--
of stimulation and see what feels best. Or try a few of the is to be nice to yourself. Masturbation is for you; you are the
following "advanced" moves: only person you need to worry about pleasing. Do what feels
• With your thumb and forefinger on either side of your good for you. Don't worry about having an orgasm. With the
clitoris, gently roll it between your fingers. Begin with a multitude of nerve endings in your vulva, you will enjoy the
slow, gentle roll and then gradually accelerate the stimulation regardless of whether or not you come. Revel in the
movement until you find the speed that works for you. sensations you produce in your body.
• Place two fingers directly on your clitoris and move them
in a circular motion. Try varying speed and intensity. Once you become more comfortable and experienced with
masturbation, you might want to try some props. Get a vibrator
• "Draw" a circle around your clitoris with your middle
or dildo and some lubricant. Try a beginner anal plug while
finger. Women who find direct clitoral pressure too intense masturbating. Experiment with different sensations: feathers,
will enjoy this move. silk, warming oil. Get a few erotic novels, magazines or videos.

The Apocryphal Maggots:


Rainier, Chrisgel, Corina, Geoffry, Grace and Sylvie Blanche
The Flibbertigibbet Worms:
Golda, Gladys and Melyjane
CA VE AT: By simply r ea ding this re vi ew er a t the end o f the sem este r wi ll (hopef ully) guar antee you r pas sing this cou rse. D rink mode rat ely .
Amusin S C A r y
gly
THE C2005 LOCAL GOVERNMENT REVIEWER - 216 -
Try masturbating in the shower, either with a waterproof sex (a) Sangguniang panlungsod or sangguniang bayan for disputes
toy, or with the "massage" setting on your handheld shower involving two (2) or more barangays in the same city or
head. Some women also enjoy lying on their backs in the municipality, as the case may be;
(b) Sangguniang panlalawigan, for those involving two (2) or
bathtub and having water from the faucet drip or stream onto
more municipalities within the same province;
their clitorises. For more suggestions, buy a copy of "Sex for
(c) Jointly, to the sanggunians of provinces concerned, for those
One" by Betty Dodson, or "For Yourself" by Lonnie Barbach. involving component cities or municipalities of different
Both are written by women and filled with wonderful ideas for provinces; or
increasing your self-pleasure. (d) Jointly, to the respective sanggunians, for those involving a
(continued…) component city or municipality and a highly-urbanized city; or
two (2) or more highly-urbanized cities.

Art17. Procedures for Settling Boundary Disputes. — The


Other Pr ovis ions App lic ab le to Lo ca l following procedures shall govern the settlement of boundary
Governmen t Uni ts disputes:
(a) Filing of petition — The sanggunian concerned may initiate
Settlement of Boundary Disputes action by filing a petition, in the form of a resolution, with the
sanggunian having jurisdiction over the dispute.
(b) Contents of petition — The petition shall state the grounds,
Sec118. Jurisdictional Responsibility for Settlement of
reasons or justifications therefor.
Boundary Dispute. — Boundary disputes between and among
(c) Documents attached to petition — The petition shall be
local government units shall, as much as possible, be settled
accompanied by:
amicably. To this end:
(1) Duly authenticated copy of the law or statute creating the
(a) Boundary disputes involving two (2) or more barangays in
LGU or any other document showing proof of creation of
the same city or municipality shall be referred for settlement to
the LGU;
the sangguniang panlungsod or sangguniang bayan concerned.
(2) Provincial, city, municipal, or barangay map, as the case may
(b) Boundary disputes involving two (2) or more municipalities
be, duly certified by the LMB;
within the same province shall be referred for settlement to the
(3) Technical description of the boundaries of the LGUs
sangguniang panlalawigan concerned.
concerned;
(c) Boundary disputes involving municipalities or component
(4) Written certification of the provincial, city, or municipal
cities of different provinces shall be jointly referred for settlement
assessor, as the case may be, as to territorial jurisdiction
to the sanggunians of the province concerned.
over the disputed area according to records in custody;
(d) Boundary disputes involving a component city or municipality
(5) Written declarations or sworn statements of the people
on the one hand and a highly urbanized city on the other, or two
residing in the disputed area; and
(2) or more highly urbanized cities, shall be jointly referred for
(6) Such other documents or information as may be required by
settlement to the respective sanggunians of the parties.
the sanggunian hearing the dispute.
(e) In the event the sanggunian fails to effect an amicable
(d) Answer of adverse party — Upon receipt by the sanggunian
settlement within sixty (60) days from the date the dispute was
concerned of the petition together with the required documents,
referred thereto, it shall issue a certification to that effect.
the LGU or LGUs complained against shall be furnished copies
Thereafter, the dispute shall be formally tried by the sanggunian
thereof and shall be given fifteen (15) working days within which
concerned which shall decide the issue within sixty (60) days
to file their answers.
from the date of the certification referred to above.
(e) Hearing — Within five (5) working days after receipt of the
answer of the adverse party, the sanggunian shall hear the case
Sec119. Appeal. — Within the time and manner prescribed by the
and allow the parties concerned to present their respective
Rules of Court, any party may elevate the decision of the
evidences.
sanggunian concerned to the proper Regional Trial Court having
(f) Joint hearing — When two or more sanggunians jointly hear
jurisdiction over the area in dispute. The Regional Trial Court
a case, they may sit en banc or designate their respective
shall decide the appeal within one (1) year from the filing
representatives. Where representatives are designated, there
thereof. Pending final resolution of the disputed area prior to the
shall be an equal number of representatives from each
dispute shall be maintained and continued for all legal purposes.
sanggunian. They shall elect from among themselves a
presiding officer and a secretary. In case of disagreement,
Implementing Rules and Regulations
selection shall be by drawing lot.
Art15. Definition and Policy. — There is a boundary dispute
(g) Failure to settle — In the event the sanggunian fails to
when a portion or the whole of the territorial area of an LGU is
amicably settle the dispute within sixty (60) days from the date
claimed by two or more LGUs. Boundary disputes between or
such dispute was referred thereto, it shall issue a certification to
among LGUs shall, as much as possible, be settled amicably.
that effect and copies thereof shall be furnished the parties
concerned.
Art16. Jurisdictional Responsibility. — Boundary disputes shall
be referred for settlement to the following:

The Apocryphal Maggots:


Rainier, Chrisgel, Corina, Geoffry, Grace and Sylvie Blanche
The Flibbertigibbet Worms:
Golda, Gladys and Melyjane
CA VE AT: By simply r ea ding this re vi ew er a t the end o f the sem este r wi ll (hopef ully) guar antee you r pas sing this cou rse. D rink mode rat ely .
Amusin S C A r y
gly
THE C2005 LOCAL GOVERNMENT REVIEWER - 217 -
(h) Decision — Within sixty (60) days from the date the
certification was issued, the dispute shall be formally tried and Held: The 2 officials should be cited for contempt, and no
decided by the sanggunian concerned. Copies of the decision plebiscite is required. RA 5480 does not purport to have amended
shall, within fifteen (15) days from the promulgation thereof, be the Revised Administrative Code. RA 5480 enumerates the
furnished the parties concerned, DILG, local assessor, component bgys. up to the boundary of CN and Quezon as defined
COMELEC, NSO, and other NGAs concerned. in the RAC. That boundary has been defined in the 1922 EB
(i) Appeal — Within the time and manner prescribed by the Decision which in turn has been ordered enforced in the SC
Rules of Court, any party may elevate the decision of the decision. The enumeration of bgys. in RA 5480 is not intended to
sanggunian concerned to the proper Regional Trial Court having delimit the territorial jurisdiction of Sta. Elena.
jurisdiction over the dispute by filing therewith the appropriate
pleading, stating among others, the nature of the dispute, the The 1922 EB decision did not alter, redefine or amend the
decision of the sanggunian concerned and the reasons for provincial boundary line between Quezon and CN. All that the
appealing therefrom. The Regional Trial Court shall decide the Chief of the Executive Bureau did was implement RAC. Hence no
case within one (1) year from the filing thereof. Decisions on need for a plebiscite.
boundary disputes promulgated jointly by two (2) or more
sangguniang panlalawigans shall be heard by the Regional Trial Municipality of Kanaga v. Madrona (2003)
Court of the province which first took cognizance of the dispute.
A boundary dispute arose between the Kanaga Municipality and
Art18. Maintenance of Status Quo. — Pending final resolution of Ormoc City. Ormoc City filed before the RTC a complaint to settle
the dispute, the status of the affected area prior to the dispute the dispute. Kanaga filed MTD alleging that the RTC had no
shall be maintained and continued for all purposes. jurisdiction over the subject matter of the claim.
Art19. Official Custodian. — The DILG shall be the official Held: The procedure in Sec. 118 is not applicable to the CAB since
custodian of copies of all documents on boundary disputes of the provision applies to a situation in which a component city or a
LGUs. municipality seeks to settle a boundary dispute with a highly
urbanized city, not with an independent component city. Ormoc is
not a highly urbanized city, but an independent component city.
Pasig v. COMELEC (1999)
Since Sec. 118 is not applicable, general rules governing
Held: The plebiscites scheduled should be suspended or jurisdiction should then be used. BP 129 (Judiciary Reorganization
cancelled in view of the pending boundary dispute. The boundary Act) as amended is the applicable law where Sec. 19 (6) provides
dispute presents a prejudicial question which must first be that RTCs exercise exclusive original jurisdiction in all cases
decided before plebiscites for the creation of the proposed not within the exclusive jurisdiction of any court, tribunal,
barangays may be held. The boundary dispute has a bearing on person or body exercising judicial or quasi-judicial functions.
the creation of the proposed bgys., because a requisite for the
creation of a bgy. is for its territorial jurisdiction to be properly Since no law provides for exclusive jurisdiction of any court/
identified. agency over settlement of boundary disputes between a
municipality and an independent component city of the same
The territorial boundaries of an LGU must be clear for they define province, the RTC did not abuse its discretion in denying the MTD.
the limits of territorial jurisdiction. An LGU can exercise powers of
government only within the limits of its territorial jurisdiction.
Beyond these, its acts are ultra vires. Local Initiative and Referendum – LGC

The fact that the plebiscite has already been held does not mean Sec120. Local Initiative Defined. — Local initiative is the legal
the case has become moot and academic, considering that the process whereby the registered voters of a local government
legality of the plebiscite itself is challenged. unit may directly propose, enact, or amend any ordinance.

Province of Camarines Norte v. Province of Quezon (2001) Sec121. Who May Exercise. — The power of local initiative and
referendum may be exercised by all registered voters of the
The SC had already decided the long-drawn boundary dispute provinces, cities, municipalities, and barangays.
between Camarines Norte and Quezon, but Quezon Gov.
Rodriguez and Mayor Lim refused to comply. They made an Sec122. Procedure in Local Initiative. —
argument that Sec. 1 of RA 5480 creating the municipality of Sta. (a) Not less than one thousand (1,000) registered voters in case
Elena provides the latest definition of the boundary between of provinces and cities, one hundred (100) in case of
Quezon and CN and nowhere in the said RA can be found 9 bgys. municipalities, and fifty (50) in case of barangays, may file a
To include the 9 bgys. in Sta. Elena would violate the RA, Sec. 10 petition with the sanggunian concerned proposing the adoption,
of Art. X of the Consti. And Sec. 10 of the LGC, all of which require enactment, repeal, or amendment of an ordinance.
a plebiscite in cases of substantial alteration of boundaries.

The Apocryphal Maggots:


Rainier, Chrisgel, Corina, Geoffry, Grace and Sylvie Blanche
The Flibbertigibbet Worms:
Golda, Gladys and Melyjane
CA VE AT: By simply r ea ding this re vi ew er a t the end o f the sem este r wi ll (hopef ully) guar antee you r pas sing this cou rse. D rink mode rat ely .
Amusin S C A r y
gly
THE C2005 LOCAL GOVERNMENT REVIEWER - 218 -
(b) If no favorable action thereon is taken by the sanggunian or amended by the sanggunian concerned within six (6) months
concerned within thirty (30) days from its presentation, the from the date of the approval thereof, and may be amended,
proponents, through their duly authorized and registered modified or repealed by the sanggunian within three (3) years
representatives, may invoke their power of initiative, giving thereafter by a vote of three-fourths (3/4) of all its members:
notice thereof to the sanggunian concerned. Provided, That in case of barangays, the period shall be
(c) The proposition shall be numbered serially starting from eighteen (18) months after the approval thereof.
Roman numeral I. The COMELEC or its designated
representative shall extend assistance in the formulation of the Sec126. Local Referendum Defined. — Local referendum is the
proposition. legal process whereby the registered voters of the local
(d) Two (2) or more propositions may be submitted in an government units may approve, amend or reject any ordinance
initiative. enacted by the sanggunian.
(e) Proponents shall have ninety (90) days in case of provinces The local referendum shall be held under the control and
and cities, sixty (60) days in case of municipalities, and thirty direction of the COMELEC within sixty (60) days in case of
(30) days in case of barangays, from notice mentioned in provinces and cities, forty-five (45) days in case of municipalities
subsection (b) hereof to collect the required number of and thirty (30) days in case of barangays.
signatures. (f) The petition shall be signed before the election The COMELEC shall certify and proclaim the results of the said
registrar. or his designated representatives, in the presence of a referendum.
representative of the proponent, and a representative of the
sanggunian concerned in a public place in the local government Sec127. Authority of Courts. — Nothing in this Chapter shall
unit, as the case may be. Stations for collecting signatures may prevent or preclude the proper courts from declaring null and
be established in as many places as may be warranted. void any proposition approved pursuant to this Chapter for
(g) Upon the lapse of the period herein provided, the violation of the Constitution or want of capacity of the
COMELEC, through its office in the local government unit sanggunian concerned to enact the said measure.
concerned, shall certify as to whether or not the required
number of signatures has been obtained. Failure to obtain the
required number defeats the proposition.
(h) If the required number of signatures is obtained, the
COMELEC shall then set a date for the initiative during which RA 6735 – The Initiative and Referendum Act
the proposition shall be submitted to the registered voters in the
local government unit concerned for their approval within sixty I. — General Provisions
(60) days from the date of certification by the COMELEC, as Sec2. Statement of Policy. — The power of the people under a
provided in subsection (g) hereof, in case of provinces and system of initiative and referendum to directly propose, enact,
cities, forty-five (45) days in case of municipalities, and thirty approve or reject, in whole or in part, the Constitution, laws,
(30) days in case of barangays. The initiative shall then be held ordinances, or resolutions passed by any legislative body upon
on the date set, after which the results thereof shall be certified compliance with the requirements of this Act is hereby affirmed,
and proclaimed by the COMELEC. recognized and guaranteed.
Sec123. Effectivity of Local Propositions. — If the proposition is Sec3. Definition of Terms. — For purposes of this Act, the
approved by a majority of the votes cast, it shall take effect following terms shall mean:
fifteen (15) days after certification by the COMELEC as if (a) "Initiative" is the power of the people to propose
affirmative action thereon had been made by the sanggunian amendments to the Constitution or to propose and enact
and local chief executive concerned. If it fails to obtain said legislations through an election called for the purpose.
number of votes, the proposition is considered defeated. There are three (3) systems of initiative, namely:
a.1 Initiative on the Constitution which refers to a petition
Sec124. Limitations on Local Initiative. — proposing amendments to the Constitution;
(a) The power of local initiative shall not be exercised more than a.2. Initiative on statutes which refers to a petition proposing to
once a year. enact a national legislation; and
(b) Initiative shall extend only to subjects or matters which are a.3. Initiative on local legislation which refers to a petition
within the legal powers of the sanggunian to enact. proposing to enact a regional, provincial, city, municipal,
(c) If at any time before the initiative is held, the sanggunian or barangay law, resolution or ordinance.
concerned adopts in toto the proposition presented and the local (b) "Indirect initiative" is exercise of initiative by the people
chief executive approves the same, the initiative shall be through a proposition sent to Congress or the local legislative
cancelled. However, those against such action may, if they so body for action.
desire, apply for initiative in the manner herein provided. (c) "Referendum" is the power of the electorate to approve or
reject a legislation through an election called for the purpose. It
Sec125. Limitations upon Sanggunians. — Any proposition or may be of two classes, namely:
ordinance approved through the system of initiative and
referendum as herein provided shall not be repealed, modified

The Apocryphal Maggots:


Rainier, Chrisgel, Corina, Geoffry, Grace and Sylvie Blanche
The Flibbertigibbet Worms:
Golda, Gladys and Melyjane
CA VE AT: By simply r ea ding this re vi ew er a t the end o f the sem este r wi ll (hopef ully) guar antee you r pas sing this cou rse. D rink mode rat ely .
Amusin S C A r y
gly
THE C2005 LOCAL GOVERNMENT REVIEWER - 219 -
c.1. Referendum on statutes which refers to a petition to however, That if the province or city is composed only of one (1)
approve or reject an act or law, or part thereof, passed legislative district, then at least each municipality in a province
by Congress; and or each barangay in a city should be represented by at least
c.2. Referendum on local law which refers to a petition to three per centum (3%) of the registered voters therein.
approve or reject a law, resolution or ordinance enacted (e) A referendum of initiative on an ordinance passed in a
by regional assemblies and local legislative bodies. municipality shall be deemed validly initiated if the petition
(d) "Proposition" is the measure proposed by the voters. therefor is signed by at least ten per centum (10%) of the
(e) "Plebiscite" is the electoral process by which an initiative on registered voters in the municipality, of which every barangay is
the Constitution is approved or rejected by the people. represented by at least three per centum (3%) of the registered
(f) "Petition" is the written instrument containing the proposition voters therein.
and the required number of signatories. It shall be in a form to (f) A referendum or initiative on a barangay resolution or
be determined by and submitted to the Commission on ordinance is deemed validly initiated if signed by at least ten per
Elections, hereinafter referred to as the Commission. centum (10%) of the registered voters in said barangay.
(g) "Local government units" refers to provinces, cities,
municipalities and barangays. Sec6. Special Registration. — The Commission on Election shall
(h) "Local legislative bodies" refers to the Sangguniang set a special registration day at least three (3) weeks before a
Panlalawigan, Sangguniang Panlungsod, Sangguniang Bayan, scheduled initiative or referendum.
and Sangguniang Nayon.
(i) "Local executives" refers to the Provincial Governors, City or Sec7. Verification of Signatures. — The Election Registrar shall
Municipal Mayors and Punong Barangay, as the case may be. verify the signatures on the basis of the registry list of voters,
voters' affidavits and voters identification cards used in the
Sec4. Who may exercise. — The power of initiative and immediately preceding election.
referendum may be exercised by all registered voters of the
country, autonomous regions, provinces, cities, municipalities III. — Local Initiative and Referendum
and barangays. Sec13. Procedure in Local Initiative. —
(a) Not less than two thousand (2,000) registered voters in case
Sec5. Requirements. — of autonomous regions, one thousand (1,000) in case of
(a) To exercise the power of initiative or referendum, at provinces and cities, one hundred (100) in case of
least ten per centum (10%) of the total number of the registered municipalities, and fifty (50) in case of barangays, may file a
voters, of which every legislative district is represented by at petition with the Regional Assembly or local legislative body,
least three per centum (3%) of the registered voters thereof, respectively, proposing the adoption, enactment, repeal, or
shall sign a petition for the purpose and register the same with amendment, of any law, ordinance or resolution.
the Commission. (b) If no favorable action thereon is made by local legislative
(b) A petition for an initiative on the 1987 Constitution body within (30) days from its presentation, the proponents
must have at least twelve per centum (12%) of the total number through their duly authorized and registered representative may
of registered voters as signatories, of which every legislative invoke their power of initiative, giving notice thereof to the local
district must be represented by at least three per centum (3%) of legislative body concerned.
the registered voters therein. Initiative on the Constitution may (c) The proposition shall be numbered serially starting from one
be exercised only after five (5) years from the ratification of the (1). The Secretary of Local Government or his designated
1987 Constitution and only once every five (5) years thereafter. representative shall extend assistance in the formulation of the
(c) The petition shall state the following: proposition.
c.1. contents or text of the proposed law sought to be (d) Two or more propositions may be submitted in an initiative.
enacted, approved or rejected, amended or repealed, as (e) Proponents shall have one hundred twenty (120) days in
the case may be; case of autonomous regions, ninety (90) days in case of
c.2. the proposition; provinces and cities, sixty (60) days in case of municipalities,
c.3. the reason or reasons therefor; and thirty (30) days in case of barangays, from notice mentioned
c.4. that it is not one of the exceptions provided herein; in subsection (b) hereof to collect the required number of
c.5. signatures of the petitioners or registered voters; and signatures.
c.6. an abstract or summary in not more than one hundred (f) The petition shall be signed before the Election Registrar, or
(100) words which shall be legibly written or printed at his designated representative, in the presence of a
the top of every page of the petition. representative of the proponent, and a representative of the
(d) A referendum or initiative affecting a law, resolution or regional assemblies and local legislative bodies concerned in a
ordinance passed by the legislative assembly of an autonomous public place in the autonomous region or local government unit,
region, province or city is deemed validly initiated if the petition as the case may be. Signature stations may be established in as
thereof is signed by at least ten per centum (10%) of the many places as may be warranted.
registered voters in the province or city, of which every (g) Upon the lapse of the period herein provided, the
legislative district must be represented by at least three per Commission on Elections, through its office in the local
centum (3%) of the registered voters therein; Provided, government unit concerned shall certify as to whether or not the

The Apocryphal Maggots:


Rainier, Chrisgel, Corina, Geoffry, Grace and Sylvie Blanche
The Flibbertigibbet Worms:
Golda, Gladys and Melyjane
CA VE AT: By simply r ea ding this re vi ew er a t the end o f the sem este r wi ll (hopef ully) guar antee you r pas sing this cou rse. D rink mode rat ely .
Amusin S C A r y
gly
THE C2005 LOCAL GOVERNMENT REVIEWER - 220 -
required number of signatures has been obtained. Failure to Constitution or want of capacity of the local legislative body to
obtain the required number is a defeat of the proposition. cd enact the said measure.
(h) If the required number of the signatures is obtained, the
Commission shall then set a date for the initiative at which the
proposition shall be submitted to the registered voters in the Garcia v. COMELEC (1994)
local government unit concerned for their approval within ninety
(90) days from the date of certification by the Commission, as The Sangguniang Bayan of Morong, Bataan in its Resolution 10
provided in subsection (g) hereof, in case of autonomous agreed to the inclusion of the municipality as part of the Subic
regions, sixty (60) days in case of the provinces and cities, forty- Special Economic Zone. When the municipality did not take action
five (45) days in case of municipalities, and thirty (30) days in on the petition of Garcia and others to annul the resolution, the
case of barangays. The initiative shall then be held on the date latter resorted to their power of initiative under the LGC and started
set, after which the results thereof shall be certified and soliciting the signatures. The COMELEC denied the pet. for local
proclaimed by the Commission on Elections. initiative.
Sec14. Effectivity of Local Propositions. — If the proposition is Held: Resolution 10 is the proper subject of an initiative. The
approved by a majority of the votes cast, it shall take effect Constitution clearly includes not only ordinances but resolutions as
fifteen (15) days after certification by the Commission as if appropriate subjects of a local initiative. An act includes a
affirmative action thereon had been made by the local legislative resolution (Black’s Law Dictionary). In enacting RA 6735,
body and local executive concerned. If it fails to obtain said Congress implemented the constitutional command to include acts
number of votes, the proposition is considered defeated. (resolutions) as appropriate subjects of initiative.
Sec15. Limitations on Local Initiatives. — 1991 LGC did not change the scope of coverage of local initiative
(a) The power of local initiative shall not be exercised more than as limiting the coverage to ordinances alone. Sec. 120 merely
once a year. defines the concept of local initiative as the legal process whereby
(b) Initiative shall extend only to subjects or matters which are registered voters of a LGU may directly propose, enact or amend
within the legal powers of the local legislative bodies to enact. any ordinance. It does not deal with subjects or matters that can be
(c) If at any time before the initiative is held, the local legislative taken up in a local initiative.
body shall adopt in toto the proposition presented, the initiative
shall be cancelled. However, those against such action may, if Sec. 124 (which deals with local initiative subjects or matters)
they so desire, apply for initiative in the manner herein provided. clearly does not limit its application to ordinances, but to all
“subjects or matters which are within the legal powers of the
Sec16. Limitations Upon Local Legislative Bodies. — Any Sanggunians to enact” which undoubtedly includes resolutions.
proposition or ordinance or resolution approved through the
system of initiative and referendum as herein provided shall not Sec. 125 providing for limitations upon Sanggunians supports the
be repealed, modified or amended, by the local legislative body interpretation, where inclusion of the word proposition is
concerned within six (6) months from the date therefrom, and inconsistent with respondents’ thesis that only ordinances can be
may be amended, modified or repealed by the local legislative the subject of local initiatives.
body within three (3) years thereafter by a vote of three-fourths
(3/4) of all its members: Provided, however, that in case of Distinction between a resolution and ordinance:
barangays, the period shall be one (1) year after the expiration Resolution: used whenever the legislature wishes to express an
of the first six (6) months. opinion which is to have only a temporary effect
Ordinance: intended to permanently direct and control matters
Sec17. Local Referendum. — Notwithstanding the provisions of applying to persons or things in general
Section 4 hereof, any local legislative body may submit to the
registered voters of autonomous region, provinces, cities, The subject matter of the resolution in CAB does not merely
municipalities and barangays for the approval or rejection, any temporarily affect the people of Morong for it directs a permanent
ordinance or resolution duly enacted or approved. rule of conduct or government. Its inclusion as part of SSEZ has
Said referendum shall be held under the control and direction of far reaching implications in the governance of its people. It is not
the Commission within sixty (60) days in case of provinces and material that the decision of the municipality came in the form of a
cities, forty-five (45) days in case of municipalities and thirty (30) resolution for what matters is its enduring effect on the welfare of
days in case of barangays. the people of Morong.
The Commission shall certify and proclaim the results of the
said referendum.
From class notes:
Can people file an initiative to rescind a contract entered into by
Sec18. Authority of Courts. — Nothing in this Act shall prevent or
an LGU? It depends.
preclude the proper courts from declaring null and void any
“within the competence of the sanggunian” -> if contract needs
proposition approved pursuant to this Act for violation of the
approval of the sanggu, may be coverd by initiative; but if purely
executive function, maybe not.

The Apocryphal Maggots:


Rainier, Chrisgel, Corina, Geoffry, Grace and Sylvie Blanche
The Flibbertigibbet Worms:
Golda, Gladys and Melyjane
CA VE AT: By simply r ea ding this re vi ew er a t the end o f the sem este r wi ll (hopef ully) guar antee you r pas sing this cou rse. D rink mode rat ely .
Amusin S C A r y
gly
THE C2005 LOCAL GOVERNMENT REVIEWER - 221 -
Look at whether the sanggunian has a role to play.
If your partner is disturbed by your masturbation practices,
including technique or frequency, gently encourage him or her
SBMA v. COMELEC (1996) to leave it alone. Masturbation is not a replacement for sex. Just
because you are masturbating does not necessarily mean that
Continuation of Garcia case… your partner is failing to meet your sexual needs. People
COMELEC promulgated Resolution 2848 providing for the rules masturbate for all kinds of reasons! If there is a relationship
and guidelines to govern the conduct of the referendum proposing problem, talk it out, buy a communication book, or see a couples
to annul or repeal Resolution 10 of the SB of Morong. therapist. Otherwise, it's perfectly healthy, normal and positive
for both partners to masturbate. Finally, encourage your partner
Held: The COMELEC committed grave abuse of discretion in to masturbate with you; this can be a huge turn-on for both of
promulgating Resolution 2848. The process started by private you.
respondents was an INITIATIVE but COMELEC made
preparations for a referendum only.
Most men masturbate by wrapping their fingers around their
Differences between an initiative and referendum: erect penis and stroking it up and down until they ejaculate.
Initiative Referendum Sounds boring, doesn't it? Centuries of practice assure us
- entirely the work of the - begun and consented to by otherwise, but you can still add a little variety. While male
electorate the law-making body. masturbation tends to be focused on the penis, many men really
- process of law-making by the - drawn up or enacted by a enjoy stimulating other areas, such as the testicles, anus,
people themselves without the legislative body. prostate, inner thighs or nipples. If you haven't tried stroking
participation and against the one of these areas while you masturbate, give it a go. You might
wishes of their elected reps. really like it! You can also try one of the following techniques:
- process and voting more - voters simply write either “yes” - Try a variance of your "regular" program by using your
complex or “no” in the ballot non-dominant hand.
There is a need for COMELEC to supervise an initiative more
closely. Its authority extending not only to the counting and - Lube up and use both hands at once in a pumping motion
canvassing of votes but also to seeing to it that the matter or act along your shaft.
submitted to the people is in the proper form and language so it - Encircle your penis with your thumb and forefinger
may easily be understood and voted by the electorate. positioned in a ring, and stroke it up and down your shaft.
When you get to the top, close the ring, then squeeze your
way in as you slide back down to the bottom of your shaft.
Masturbation Tips for Guys (Part 4): - With one hand, stroke your penis from top to the bottom.
When you reach the base, release it. Meanwhile do the
Sadly, even after decades of women's lib, men are encouraged to same thing with your other hand, over and over again,
be more open and expressive with their sexuality than women alternating hands. Develop a rhythm!
are. The downside is that fewer women feel comfortable - Place your hands on either side of your shaft and start
discussing or engaging in masturbation; the upside is that most spinning it like it's a stick of wood you're trying to start a
men are really, really good at masturbating and need little fire with. Proceed gently, you want gentle friction, not
encouragement to do so. burning flames!
- "Force" your penis into your closed fist as though you are
At MyPleasure, we firmly believe (and sexuality experts will penetrating it. Repeat the motion with your other hand,
concur) that there is really no such thing as "too much" interchanging hands at the end of each stroke.
masturbation. If it feels good, do it once a day, five times a day, - Use your open palm to swirl around the head of your
twice a week or semiannually. Only you can decide how much penis, the way your tongue would lick an ice cream cone.
self-stimulation is right for you. However, if you are worried - Turn the head of his penis like a you're trying to open a
that you masturbate too much, you may want to reflect on your door knob coated with grease. Now try turning the other
motives. Are you engaging in solo sex for healthy reasons, such way. Repeat.
as pleasure, stress release, celebration or sexual release? Do you (continued…)
enjoying it when you masturbate, or are you doing it to avoid
something? If you are able to eat, sleep, work or engage in any
other activities you enjoy without constantly thinking about or
engaging in masturbation, then you're perfectly fine. Masturbate
all you want! Otherwise, back off for awhile, see a therapist and
get to know yourself a little better.

The Apocryphal Maggots:


Rainier, Chrisgel, Corina, Geoffry, Grace and Sylvie Blanche
The Flibbertigibbet Worms:
Golda, Gladys and Melyjane
CA VE AT: By simply r ea ding this re vi ew er a t the end o f the sem este r wi ll (hopef ully) guar antee you r pas sing this cou rse. D rink mode rat ely .
Amusin S C A r y
gly
THE C2005 LOCAL GOVERNMENT REVIEWER - 222 -
Book II: LOCAL TAXA TION AND Republic Act No. 7160, otherwise known as the Local
Government Code of 1991.
FI SCAL MA TTE RS "In addition to the internal revenue allotment as provided for in
LOC AL GOVER NMEN T T AX ATI ON the preceding paragraph, fifty percent (50%) of the national
taxes collected under Sections 100, 102, 112, 113, and 114 of
this Code in excess of the increase in collections for the
1987 Constitution, Article X immediately preceding year shall be distributed as follows: (a)
Twenty percent (20%) shall accrue to the city or municipality
Sec3. The Congress shall enact a local government code which where such taxes are collected and shall be allocated in
shall provide for a more responsive and accountable local accordance with Section 150 of Republic Act No. 7160,
government structure instituted through a system of otherwise known as the Local Government Code of 1991; and
decentralization with effective mechanisms of recall, initiative, (b) Eighty percent (80%) shall accrue to the National
and referendum, allocate among the different local government Government."
units their powers, responsibilities, and resources, and provide
for the qualifications, election, appointment and removal, term,
salaries, powers and functions and duties of local officials, and RA 7716 (1994) – An Act Restructuring The Value Added Tax
all other matters relating to the organization and operation of (Vat) System, Widening Its Tax Based And Enhancing Its
local units. Administration And For These Purposes Amending And
Repealing The Relevant Provisions Of The National Internal
Sec5. Each local government unit shall have the power to create Revenue Code, As Amended, And For Other Purposes
its own sources of revenues and to levy taxes, fees, and
charges subject to such guidelines and limitations as the
Sec11. Section 115 of the National Internal Revenue Code, as
Congress may provide, consistent with the basic policy of local
amended, is hereby further amended to read as follows:
autonomy. Such taxes, fees, and charges shall accrue
"Sec. 115. Percentage tax on carriers and keepers of garages.
exclusively to the local governments.
— Keepers of garages, and common carriers by land, air or
water for the transport of passengers, except owners of bancas,
Sec6. Local government units shall have a just share, as
and owners of animal-drawn two-wheeled vehicles, shall pay a
determined by law, in the national taxes which shall be
tax equivalent to three per centum (3%) of their quarterly gross
automatically released to them.
receipts.
"The gross receipts of common carriers derived from their
Sec7. Local governments shall be entitled to an equitable share in
incoming and outgoing freight shall not be subjected to the local
the proceeds of the utilization and development of the national
taxes imposed under Republic Act No. 7160, otherwise known
wealth within their respective areas, in the manner provided by
as the Local Government Code of 1991.
law, including sharing the same with the inhabitants by way of
"In computing the percentage tax provided in this Section, the
direct benefits.
following shall be considered the minimum quarterly gross
receipts in each particular case:
"Jeepney for hire —
See also Secs. 128-196 1. Manila and other cities P2,400.00
(Note: I didn’t include these provisions because they were not 2. Provincial 1,200.00
assigned in the course outline and they are long, but you may "Public utility bus —
want to take a look at them anyway. – Corina ) Not exceeding 30 passengers P3,600.00
Exceeding 30 but not exceeding
50 passengers 6,000.00
RA 7643 (1992) – An Act To Empower The Commissioner Of Exceeding 50 passengers 7,200.00
Internal Revenue To Require The Payment Of The Value- "Taxis —
Added Tax Every Month And To Allow Local Government 1. Manila and other cities P3,600.00
Units To Share In Vat Revenue, Amending For This Purpose 2. Provincial 2,400.00
Certain Sections Of The National Internal Revenue Code Car for hire (w/ chauffeur) 3,000.00
Car for hire (w/one chauffeur) 1,800.00"
Sec2. Sec. 282 of the National Internal Revenue Code, as
amended, is hereby further amended to read as follows: Sec17. Effectivity of the Imposition of VAT on Certain Goods,
"Sec. 282. Disposition of national internal revenue. — Properties and Services. — The value-added tax shall be levied
National internal revenue collected and not applied as assessed and collected on the following, two (2) years after the
hereinabove provided or otherwise specially disposed of by law effectivity of this Act:
shall accrue to the National Treasury and shall be available for (a) Services performed in the exercise of profession or calling
the general purposes of the Government, with the exception of subject to the professional tax under the Local Government
the amounts set apart by way of allotment as provided for under Code or Republic Act No. 7160, and professional services
performed by registered general professional partnerships;

The Apocryphal Maggots:


Rainier, Chrisgel, Corina, Geoffry, Grace and Sylvie Blanche
The Flibbertigibbet Worms:
Golda, Gladys and Melyjane
CA VE AT: By simply r ea ding this re vi ew er a t the end o f the sem este r wi ll (hopef ully) guar antee you r pas sing this cou rse. D rink mode rat ely .
Amusin S C A r y
gly
THE C2005 LOCAL GOVERNMENT REVIEWER - 223 -
actors, actresses, talents, singers and emcees; radio and their transport of goods or cargoes; services of franchise
television broadcasters, choreographers; musical, radio, movie, grantees of telephone and telegraph, radio and television
television and stage directors; and professional athletes; broadcasting and all other franchise grantees except those
(b) Services rendered by banks, non-bank financial under Section 117 of this Code; services of banks, non-bank
intermediaries, finance companies and other financial financial intermediaries and finance companies; and non-life
intermediaries not performing quasi-banking functions; insurance companies (except their crop insurances) including
(c) Freight services rendered by international cargo vessels; and surety, fidelity, indemnity and bonding companies; and similar
(d) The lease or use of sports facilities and equipment by services regardless of whether or not the performance thereof
amateur players, as provided under Republic Act No. 6847, calls for the exercise or use of the physical or mental faculties.
except sports facilities and equipment which are exclusively or The phrase 'sale or exchange of services' shall likewise include:
mainly for the private use of shareholders or members of the "(1) The lease or the use of or the right or privilege to use
club or organization which owns or operates such sports any copyright, patent, design or model, plan, secret formula or
facilities and equipment. process, goodwill, trademark, trade brand or other like property
Prior to their inclusion in the coverage of the value-added tax, or right;
the above services shall continue to pay the applicable tax "(2) The lease or the use of, or the right to use of any
prescribed under the present provisions of the National Internal industrial, commercial or scientific equipment;
Revenue Code, as amended. "(3) The supply of scientific, technical, industrial or
However, when public interest so requires, the President, taking commercial knowledge or information;
into account the impact on the prices of goods and services, "(4) The supply of any assistance that is ancillary and
may, upon the recommendation of the Secretary of Finance, subsidiary to and is furnished as a means of enabling the
exclude any of the above services from the coverage of the application or enjoyment of any such property, or right as is
value-added tax: Provided, however, That in the event of the mentioned in subparagraph (2) or any such knowledge or
exclusion of any of the above services the existing applicable information as is mentioned in subparagraph (3);
tax under the provisions of the National Internal Revenue Code, "(5) The supply of services by a nonresident person or his
as amended, shall continue to be paid on the services so employee in connection with the use of property or rights
excluded. belonging to, or the installation or operation of any brand,
machinery, or other apparatus purchased from such nonresident
person;
RA 8241 (1996) – An Act Amending Republic Act No, 7716, "(6) The supply of technical advice, assistance or services
Otherwise Known As The Expanded Value-Added Tax Law rendered in connection with technical management or
And Other Pertinent Provisions Of The National Internal administration of any scientific, industrial or commercial
Revenue Code As Amended undertaking, venture, project or scheme;
"(7) The lease of motion picture films, films, tapes and
Sec1. Section 3 of Republic Act No. 7716 is hereby amended to discs and;
read as follows: "(8) The lease or the use of or the right to use radio,
"Sec. 3. Section 102 of the National Internal Revenue Code, television, satellite transmission and cable television time.
as amended, is hereby further amended to read as follows: "Lease of properties shall be subject to the tax herein imposed
"Sec. 102. Value-added tax on sale of services and use or lease irrespective of the place where the contract of lease or licensing
of properties. — (a) Rate and base of tax. — There shall be agreement was executed if the property is leased or used in the
levied assessed and collected, a value-added tax equivalent to Philippines.
ten percent (10%) of gross receipts derived from the sale or "The term 'gross receipts' means the total amount of money or
exchange of services, including the use or lease of properties. its equivalent representing the contract price, compensation,
"The phrase 'sale or exchange of services' means the service fee, rental or royalty, including the amount charged for
performance of all kinds of services in the Philippines for others materials supplied with the services and deposits and advanced
for a fee, remuneration or consideration, including those payments actually or constructively received during the taxable
performed or rendered by construction and service contractors; quarter for the services performed or to be performed for
stock, real estate, commercial, customs and immigration another person, excluding value-added tax.
brokers; lessors of property, whether personal or real; "(b) Transactions subject to zero percent (0%) rate. —
warehousing services; lessors or distributors of cinematographic The following services performed in the Philippines by VAT-
films; persons engaged in milling, processing, manufacturing or registered persons shall be subject to zero percent (0%) rate;
repacking goods for others; proprietors, operators or keepers of "(1) Processing, manufacturing or repacking goods for
hotels, motels, resthouses, pension houses, inns, resorts; other persons doing business outside the Philippines which
proprietors or operators of restaurants, refreshment parlors, goods are subsequently exported, where the services are paid
cafes and other eating places, including clubs and caterers; for the in acceptable foreign currency and accounted for in
dealers in securities; lending investors; transportation accordance with the rules and regulations of the Bangko Sentral
contractors on their transport of goods or cargoes, including ng Pilipinas (BSP);
persons who transport goods or cargoes for hire and other "(2) Services other than those mentioned in the preceding
domestic common carriers by land, air, and water relative to subparagraph, the consideration for which is paid for in

The Apocryphal Maggots:


Rainier, Chrisgel, Corina, Geoffry, Grace and Sylvie Blanche
The Flibbertigibbet Worms:
Golda, Gladys and Melyjane
CA VE AT: By simply r ea ding this re vi ew er a t the end o f the sem este r wi ll (hopef ully) guar antee you r pas sing this cou rse. D rink mode rat ely .
Amusin S C A r y
gly
THE C2005 LOCAL GOVERNMENT REVIEWER - 224 -
acceptable foreign currency and accounted for in accordance "(h) Importation of personal and household effects
with the rules and regulations of the Bangko Sentral ng Pilipinas belonging to the residents of the Philippines returning from
(BSP); abroad and nonresident citizens coming to resettle in the
"(3) Services rendered to persons or entities whose Philippines: Provided, That such goods are exempt from
exemption under laws or international agreements to which the customs duties under the Tariff and Customs Code in the
Philippines is a signatory effectively subjects the supply of such Philippines;
services to zero percent (0%) rate; "(i) Importation of professional instruments and
"(4) Services rendered to vessels engaged exclusively in implements, wearing apparel, domestic animals, and personal
international shipping; and household effects (except any vehicle, vessel, aircraft,
"(5) Services performed by subcontractors and/or machinery, other goods for use in the manufacture and
contractors in processing, converting, or manufacturing goods merchandise of any kind in commercial quantity) belonging to
for an enterprise whose export sales exceed seventy percent persons coming to settle in the Philippines, for their own use
(70%) of total annual production. and not for sale, barter or exchange, accompanying such
"(c) Determination of the Tax. — The tax shall be persons, or arriving within ninety (90) days before or after their
computed by multiplying the total amount indicated in the official arrival, upon the production of evidence satisfactory to the
receipt by 1/11." Commissioner of Internal Revenue; that such persons are
actually coming to settle in the Philippines and that the change
SEC2. Section 4 of Republic Act No. 7716 is hereby amended to of residence is bona fide;
read as follows: "(j) Services subject to percentage tax under Title V;
"Sec. 4. Section 103 of the National Internal Revenue Code, as "(k) Services by agricultural contract growers and milling
amended, is hereby further amended to read as follows: for others of palay into rice, corn into grits and sugar case into
"Sec. 103. Exempt Transactions. — The following shall be raw sugar;
exempt from the value-added tax: "(l) Medical, dental, hospital and veterinary services
"(a) Sale of nonfood agricultural products; marine and subject to the provisions of Section 17 of Republic Act No. 7716,
forest products in their original state by the primary producer or as amended;
the owner of the land where the same are produced; "(m) Educational services rendered by private educational
"(b) Sale of cotton and cotton seeds in their original state institutions, duly accredited by the Department of Education,
and copra; Culture and Sports (DECS) and the Commission on Higher
"(c) Sale or importation of agricultural and marine food Education (CHED), and those rendered by government
products in their original state, livestock and poultry of a kind, educational institutions;
generally used as, or yielding or producing foods for human "(n) Sale by the artist himself of his works or art, literary
consumption; and breeding stock and genetic materials therefor. works, musical compositions and similar creations, or his
"Products classified under this paragraph and paragraph (a) services performed for the production of such works;
shall be considered in their original state even if they have "(o) Services rendered by individuals pursuant to an
undergone the simple processes of preparation or preservation employer-employee relationship;
for the market, such as freezing, drying, salting, broiling, "(p) Services rendered by regional or area headquarters
roasting, smoking or stripping. Polished and/or husked rice, corn established in the Philippines by multinational corporations
grits, raw cane sugar and molasses, and ordinary salt shall be which act as supervisory, communications and coordinating
considered in their original stated; centers for their affiliates, subsidiaries or branches in the Asia-
"(d) Sale or importation of fertilizers; seeds, seedlings and Pacific Region and do not earn or derive income from the
fingerlings; fish, prawn, livestock and poultry feeds, including Philippines;
ingredients, whether locally produced or imported, used in the "(q) Transactions which are exempt under international
manufacture of finished feeds (except specialty feeds for race agreements to which the Philippines is a signatory or under
horses, fighting cocks, aquarium fish, zoo animals and other special laws, except those under Presidential Decree Nos. 66,
animals generally considered as pets); 529, and 1590;
"(e) Sale or importation of coal and natural gas, in "(r) Sales by agricultural cooperatives duly registered with
whatever form or state, and petroleum products (Except the Cooperative Development Authority to their members as well
lubricating oil processed gas, grease, wax, and petrolatum) as sale or their produce, whether in its original state or
subject to excise tax imposed under Title VI; processed form, to non-members; their importation of direct
"(f) Sale or importation of raw materials to be used by the farm inputs, machineries and equipment, including spare parts
buyer or importer himself in the manufacture of petroleum thereof, to be used directly and exclusively in the production
products subject to excise tax, except lubricating oil, processed and/or processing of their produce;
gas, grease, wax, and petrolatum; "(s) Sales by electric cooperatives duly registered with the
"(g) Importation of passenger and/or cargo vessel of more Cooperative Development Authority or National Electrification
than five thousand tons, whether coastwise or ocean-going, Administration, relative to the generation and distribution of
including engine and spare parts of said vessel to be used by electricity as well as their importation of machineries and
the importer himself as operator thereof; equipment, including spare parts, which shall be directly used in
the generation and distribution of electricity;

The Apocryphal Maggots:


Rainier, Chrisgel, Corina, Geoffry, Grace and Sylvie Blanche
The Flibbertigibbet Worms:
Golda, Gladys and Melyjane
CA VE AT: By simply r ea ding this re vi ew er a t the end o f the sem este r wi ll (hopef ully) guar antee you r pas sing this cou rse. D rink mode rat ely .
Amusin S C A r y
gly
THE C2005 LOCAL GOVERNMENT REVIEWER - 225 -
"(t) Gross receipts from lending activities by credit or liable to value-added tax or any person who elects to be a VAT-
multi-purpose cooperatives duly registered with the Cooperative registered person shall, subject to the filing of an inventory as
Development Authority whose lending operation is limited to prescribed by regulations, be allowed input tax on his beginning
their members; inventory of goods, materials and supplies equivalent to eight
"(u) Sales by non-agricultural, non-electric and non-credit percent (8%) of the value of such inventory or the actual value-
cooperatives duly registered with the Cooperative Development added tax paid on such goods, materials and supplies,
Authority: Provided, That the share capital contribution of each whichever is higher, which shall be creditable against the output
member does not exceed Fifteen thousand pesos (P15,000.00) tax.
and regardless of the aggregate capital and net surplus ratably "(b) Presumptive input tax credits. — (1) Persons or firms
distributed among the members; engaged in the processing of sardines, mackerel, and milk, and
"(v) Export sales by persons who are not VAT-registered; in manufacturing refined sugar and cooking oil, shall be allowed
"(w) Sale of real properties not primarily held for sale to a presumptive input tax, creditable against the output tax,
customers or held for lease in the ordinary course of trade or equivalent to one and one-half percent (1.5%) of the gross value
business or realty property utilized for low-cost and socialized in money of their purchases of primary agricultural products
housing as defined by Republic Act No. 7279, otherwise known which are used as inputs to their production.
as the Urban Development and Housing Act of 1992, and other "As used in this paragraph (b), the term 'processing' shall mean
related laws, house and lot and other residential dwellings pasteurization, canning and activities which through physical or
valued at One million pesos (P1,000,000.00) and below: chemical process alter the exterior texture or form or inner
Provided, That not later than January 31st of the calendar year substance of a product in such manner as to prepare it for
subsequent to the effectivity of this Act and each calendar year special use to which it could not have been put in its original
thereafter, the amount of One million pesos (P1,000,000.00) form or condition.
shall be adjusted to its present value using the consumer price "(2) Public works contractors shall be allowed a
index, as published by the National Statistics Office (NSO); presumptive input tax equivalent to one and one-half percent
"(x) Lease of a residential unit with a monthly rental not (1.5%) of the contract price with respect to government
exceeding Eight thousand pesos (P8,000.00): Provided, That contracts only in lieu of actual input taxes therefrom."
not later than January 31st of the calendar year subsequent to
the effectivity of this Act and each calendar year thereafter, the Sec4. Section 7 of Republic Act No. 7716 is hereby amended to
amount of Eight thousand pesos (P8,000.00) shall be adjusted read as follows:
to its present value using the consumer price index, as "Sec. 7. Section 107 of the National Internal Revenue Code, as
published by the National Statistics Office (NSO); amended, is hereby further amended to read as follows:
"(y) Sale, importation, printing or publication of books and "Sec. 107. Registration of value-added taxpayers. — (a) In
any newspaper, magazine, review, or bulletin which appears at General. — Any person subject to a value-added tax under
regular intervals with fixed prices for subscription and sale and Sections 100 and 102 of this Code shall register with the
which is not devoted principally to the publication of paid appropriate Revenue District Officer and pay an annual
advertisements; registration fee in the amount of One thousand pesos
"(z) Sale or lease of goods or properties or the (P1,000.00) for every separate or distinct establishment or place
performance of services other than the transactions mentioned of business and every year thereafter on or before the last day
in the preceding paragraphs, the gross annual sales and/or of January. Any person just commencing a business subject to
receipts do not exceed the amount of Five hundred fifty the value-added tax must pay the fee before engaging therein.
thousand pesos (P550,000.00): Provided, That not later than "A Person who maintains a head or main office and branches in
January 31st of the calendar year subsequent to the effectivity different places shall register with the Revenue District Office
of this Act and each calendar year thereafter, the amount of Five which has jurisdiction over the place wherein the main or head
hundred fifty thousand pesos (P550,000.00) shall be adjusted to office is located. However, the fee shall be paid to the Revenue
its present value using the consumer price index, as published district Officer, collection agent, authorized treasurer of the
by the National Statistics Office (NSO); municipality where each place of business or branch is situated.
"The foregoing exemptions to the contrary notwithstanding any "(b) Persons commencing business. — Any person who
person whose sale of goods or properties or services which are expects to realize gross sales or receipts subject to valued-
otherwise not subject to VAT, but who issues a VAT invoice or added tax in excess of the amount prescribed under Section
receipt therefor shall, in addition to his liability to other 103(z) of this Code for the next 12-month period from the
applicable percentage tax, if any, be liable to the tax imposed in commencement of the business shall, within thirty (30) days
Section 100 or 102 without the benefit of input tax credit, and before the start of the said business register with the Revenue
such tax shall not also be recognized as input tax credit to the District Officer who has jurisdiction over his principal place of
purchaser under Section 104, all of this Code." business and shall pay the annual registration fee prescribed in
the preceding paragraph.
Sec3. Section 105 of the National Internal Revenue Code, as "(c ) Persons becoming liable to the value-added tax. — Any
amended, is hereby further amended to read as follows: person whose gross sales or receipts in any 12-month period
"Sec. 105. Transitional/Presumptive Input Tax Credits. — (a) exceeds the amount prescribed under Section 103(z) of this
Transitional Input Tax Credits. — A person who becomes Code for exemption from the value-added tax shall register and

The Apocryphal Maggots:


Rainier, Chrisgel, Corina, Geoffry, Grace and Sylvie Blanche
The Flibbertigibbet Worms:
Golda, Gladys and Melyjane
CA VE AT: By simply r ea ding this re vi ew er a t the end o f the sem este r wi ll (hopef ully) guar antee you r pas sing this cou rse. D rink mode rat ely .
Amusin S C A r y
gly
THE C2005 LOCAL GOVERNMENT REVIEWER - 226 -
pay the annual registration fee prescribed in paragraph (a) of Sec6. Section 10 of Republic Act No. 7716 is hereby amended to
this section within thirty (30) days after the end of the last month read as follows:
of that period, and shall be liable to the valued-added tax "Sec. 10. Section 112 of the National Internal Revenue Code, as
commencing from the first day of the month following his amended, is hereby further amended to read as follows:
registration. "Sec. 112. Tax on persons exempt from value-added tax
"(d) Optional registration of exempt person. — Any person (VAT). — Any person whose sales or receipts are exempt under
whose transactions are exempt from value-added tax under Section 103(z) of this Code from the payment of value-added
Section 103(z) of this Code, Section 103(a), (b), (c ), and (d) of tax and who is not a VAT-registered person shall pay a tax
this Code with respect to export sales only, and Section 103(j) equivalent to three percent (3%) of his gross quarterly sales or
with respect to service of franchise grantees of radio and/or receipts: Provided, That cooperatives shall be exempt from the
television broadcasting as defined under Section 117 of this three percent (3%) gross receipt tax herein imposed."
Code, may apply for registration as a VAT-registered person not
later than ten (10) days before the beginning of the taxable Sec7. Section 11 of Republic Act No. 7716 is hereby amended to
quarter and shall pay the annual registration fee prescribed in read as follows:
subparagraph (a) of this section. 'Sec. 11. Section 115 of the National Internal Revenue Code, as
"In any case, the Commissioner may, for administrative reason, amended, is hereby further amended to read as follows:
deny any application for registration. "Sec. 115. Percentage tax on domestic carriers and keepers
"For purposes of this Title, any person registered in accordance of garages. — Cars for rent or hire driven by the lessee,
with the provisions of this section shall be referred to as a 'VAT- transportation contractors, including persons who transport
registered person.' Each VAT-registered person shall be passenger for hire, and other domestic carriers by land, air or
assigned only one taxpayer's identification number. water, for the transport of passengers, except owners of bancas,
"(e) Cancellation of registration. — The registration of any and owners of animal-drawn two-wheeled vehicle, and keepers
person who ceases to be liable to the valued-added tax shall be of garages shall pay a tax equivalent to three per centum (3%)
cancelled by the Commissioner upon filing of an application for of their quarterly gross receipts.
cancellation of registration. Any person who opted to be "The gross receipts of common carriers derived from their
registered under paragraph (d) of this section may, under incoming and outgoing freight shall not be subjected to the local
regulation of the Secretary of Finance, apply for cancellation of taxes imposed under Republic Act No. 7160, otherwise known
such registration." as the Local Government Code of 1991.
"In computing the percentage tax provided in this section, the
Sec5. Section 9 of Republic Act No. 7716 is hereby amended to following shall be considered the minimum quarterly gross
read as follows: receipts in each particular case:
"Sec. 9. Section 110(c) of the National Internal Revenue Code, "Jeepney for hire —
is hereby further amended to read as follows: "1. Manila and other cities P2,400.00
"(c) Withholding of Creditable Value-Added Tax. — The "2. Provincial 1,200.00
government or any of its political subdivisions, instrumentalities "Public utility bus —
or agencies, including government-owned or controlled "Not exceeding 30 passengers P3,600.00
corporations (GOCCs) shall, before making payment on account "Exceeding 30 but not exceeding
of each purchase of goods from sellers and services rendered 50 passengers 6,000.00
by contractors which are subject to the value-added tax "Exceeding 50 passengers 7,200.00
imposed in Section 100 and 102 of this Code, deduct and "Taxis —
withhold the value-added tax due at the rate of three percent "1. Manila and other cities P3,600.00
(3%) of the gross payment for the purchase of goods and six "2. Provincial 3,400.00
percent (6%) on gross receipts for services rendered by "Car for hire (w/chauffeur) 3,000.00
contractors on every sale or installment payment which shall be "Car for hire (w/o chauffeur) 1,800.00"
creditable against the valued-added tax liability of the seller or
contractor: Provided, however, That in the case of government Sec8. A new section is hereby added after Section 115 of the
public works contractors, the withholding rate shall be eight and National Internal Revenue Code, as amended, to read as
one-half percent (8.5%): Provided, further, That the payment for follows:
lease or use of properties or property rights to nonresident "Sec. 115-A. Percentage tax on international carriers. —
owners shall be subject to ten percent (10%) withholding tax at "a) International air carriers doing business in the
the time of payment. For this purpose, the payor of person in Philippines shall pay a tax of three per centum (3%) of their
control of the payment shall be considered as the withholding quarterly gross receipts.
agent. "(b) International shipping carriers doing business in the
'The value-added tax withheld under this section shall be Philippines shall pay a tax equivalent to three percent (3%) of
remitted with ten (10) days following the end of the month the their quarterly gross receipts."
withholding was made.'
Sec9. Section 12 of Republic Act No. 7716 is hereby amended to
read as follows:

The Apocryphal Maggots:


Rainier, Chrisgel, Corina, Geoffry, Grace and Sylvie Blanche
The Flibbertigibbet Worms:
Golda, Gladys and Melyjane
CA VE AT: By simply r ea ding this re vi ew er a t the end o f the sem este r wi ll (hopef ully) guar antee you r pas sing this cou rse. D rink mode rat ely .
Amusin S C A r y
gly
THE C2005 LOCAL GOVERNMENT REVIEWER - 227 -
"Sec. 12. Section 117 of the National Internal Revenue Code, as Sec11. Section 17 of Republic Act No. 7716 is hereby amended to
amended, is hereby further amended to read as follows: read as follows:
"Sec. 117. Tax on franchise. — Any provision of general or "Sec. 17. Effectivity of the Imposition of VAT on Certain
special law to the contrary, notwithstanding, there shall be Goods, Properties and Services. — The value-added tax shall
levied, assessed and collected in respect to all franchises on be levied, assessed and collected on the following transactions,
radio and/or television broadcasting companies whose annual starting January 1, 1998:
gross receipts of the preceding year does not exceed Ten million "(a) Services performed in the exercise of profession or
pesos (P10,000,000.00), subject to Section 107(d) of this Code, calling subject to the professional tax under the Local
a tax of two three percent (3%) and on electric, gas and water Government Code of Republic Act No. 7160, and professional
utilities, a tax of two percent (2%) on the gross receipts derived services performed by registered general professional
from the business covered by the law granting the franchise: partnerships; actors, actresses, talents, singers and emcees;
Provided, however, That radio and television broadcasting radio and television broadcasters, choreographers; musical,
companies referred to in this section, shall have an option to be radio, movie, television and stage directors; and professional
registered as a value-added tax payer and pay the tax due athletes;
thereon: Provided, further, That once the option is exercised, it "(b) Services rendered by banks, non-bank financial
shall not be revoked. intermediaries, finance companies and other financial
"The grantee shall file the return with, and pay the tax due intermediaries, not performing quasi-banking functions; and
thereon to, the Commissioner of Internal Revenue or his duly "(c ) The lease or use of sports facilities and equipment by
authorized representative in accordance with the provisions of amateur players, as provided under Republic Act No. 6847,
Section 125 of this Code and the return shall be subject to audit except sports facilities and equipment which are exclusively or
by the Bureau of Internal Revenue, any provision of any existing mainly for the private use of shareholders or members of the
law to the contrary notwithstanding." club or organization which owns or operates such sports
facilities and equipment.
Sec10. Section 15 of Republic Act No. 7716 is hereby amended to "Prior to their inclusion in the coverage of the valued-added tax
read as follows: the above services shall continue to pay the applicable tax
"Sec. 15. Section 237 of the National Internal Revenue Code, as prescribed under the present provisions of the National Internal
amended, is hereby further amended to read as follows: Revenue Code, as amended.
"Sec. 237. Registration of Name or Style with the Revenue "However, when public interest so requires, the Congress of the
District Officer or Collection Agent. — Every person, other Republic of the Philippines, taking into account the impact on
than persons required to be registered under the provisions of prices of goods and services, may, exclude any of the above
Section 107 engaged in any business shall, on or before the services from the coverage of the value-added tax: Provided,
commencement of his business shall, on or before the however, That in the event of the exclusion of any of the above
commencement of his business, or whenever he transfers to services the existing applicable tax under the provisions of the
another revenue district, register with the Revenue District National Internal Revenue Code, as amended, shall continue to
Officer concerned within 10 days from the commencement of be paid on the service so excluded."
business or transfer and shall pay the annual registration fee in xxx
the amount of Five hundred pesos (P500.00) for every separate
or distinct establishment or place of business and every year Sec13. Repealing Clause. — The provisions of Republic Act No.
thereafter on or before the last day of January. The fee shall be 6938, otherwise known as the Cooperative Code of the
paid to the Revenue District Officer, collection agent, authorized Philippines, authorizing the exemption of cooperatives from the
treasurer of the municipality where each place of business or sales or value-added tax are hereby repealed. All other laws,
branch is situated. In cities or municipalities where no Revenue orders, issuances, rules and regulations or parts thereof which
District Officer is stationed, such person shall register and pay are not consistent with this Act are hereby repealed amended or
the fee prescribed herein with the collection agent. The modified accordingly.
registration shall contain his name or style, place of where such
business is carried on, and such other information as may be
required by the Commissioner in the form prescribed therefor. In
the case of a firm, the names and residences of the various National Internal Revenue Code (1997)
persons constituting the same shall also be registered. The
Commissioner, after taking into consideration the volume of Sec117. Percentage Tax on Domestic Carriers and Keepers of
sales, financial condition and other relevant factors, may require Garages. — Cars for rent or hire driven by the lessee,
the registrant to guarantee the payment of his taxes by way of transportation contractors, including persons who transport
advance payment, or the posting or filing of a security, passengers for hire, and other domestic carriers by land, air or
guarantee or collateral acceptable to the Commissioner: water, for the transport of passengers, except owners of bancas
Provided, however, That cooperatives shall not pay the and owners of animal-drawn two wheeled vehicle, and keepers
registration fee imposed herein.' of garages shall pay a tax equivalent to three percent (3%) of
their quarterly gross receipts.

The Apocryphal Maggots:


Rainier, Chrisgel, Corina, Geoffry, Grace and Sylvie Blanche
The Flibbertigibbet Worms:
Golda, Gladys and Melyjane
CA VE AT: By simply r ea ding this re vi ew er a t the end o f the sem este r wi ll (hopef ully) guar antee you r pas sing this cou rse. D rink mode rat ely .
Amusin S C A r y
gly
THE C2005 LOCAL GOVERNMENT REVIEWER - 228 -
"The gross receipts of common carriers derived from their the utilization and development of the national wealth within
incoming and outgoing freight shall not be subjected to the local their territorial jurisdiction.
taxes imposed under Republic Act No. 7160, otherwise known
as the Local Government Code of 1991. "(B) Share of the Local Governments from Any Government
"In computing the percentage tax provided in this Section, the Agency or Government-owned or -Controlled Corporation.
following shall be considered the minimum quarterly gross — Local government units shall have a share, based on the
receipts in each particular case: preceding fiscal year, from the proceeds derived by any
"Jeepney for hire — government agency or government-owned or -controlled
"1. Manila and other cities P2,400 corporation engaged in the utilization and development of the
"2. Provincial 1,200 national wealth based on the following formula, whichever will
"Public utility bus — produce a higher share for the local government unit:
"Not exceeding 30 passengers P3,600 "(1) One Percent (1%) of the gross sales or
"Exceeding 30 but not exceeding 50 passengers receipts of the preceding calendar year; or
6,000 "(2) Forty percent (40%) of the excise taxes on
"Exceeding 50 passengers 7,200 mineral products, royalties, and such other taxes, fees or
"Taxis — charges, including related surcharges, interests or fines
"1. Manila and other cities 3,600 the government agency or government-owned or
"2. Provincial 2,400 -controlled corporation would have paid if it were not
"Car for hire (with chauffeur) 3,000 otherwise exempt.
"Car for hire (without chauffeur) 1,800 "(C) Allocation of Shares. — The share in the preceding
Section shall be distributed in the following manner:
Sec283. Disposition of National Internal Revenue. — National "(1) Where the natural resources are located in the
internal revenue collected and not applied as hereinabove province:
provided or otherwise specially disposed of by law shall accrue "(a) Province — twenty percent (20%);
to the National Treasury and shall be available for the general "(b) Component city/municipality — forty-five
purposes of the Government, with the exception of the amounts percent (45%); and
set apart by way of allotment as provided for under Republic Act "(c) Barangay — thirty-five percent (35%).
No. 7160, otherwise known as the Local Government Code of "Provided, however, That where the natural resources
1991. are located in two (2) or more provinces, or in two (2) or
"In addition to the internal revenue allotment as provided for in more component cities or municipalities or in two (2) or
the preceding paragraph, fifty percent (50%) of the national more barangays, their respective shares shall be
taxes collected under Sections 106, 108 and 116 of this Code in computed on the basis of: (1) Population — seventy
excess of the increase in collections for the immediately percent (70%); and (2) Land area — thirty percent
preceding year shall be distributed as follows: (30%).
"(a) Twenty percent (20%) shall accrue to the city or "(2) Where the natural resources are located in a
municipality where such taxes are collected and shall be highly urbanized or independent component city:
allocated in accordance with Section 150 of Republic Act No. "(a) City — sixty-five percent (65%); and
7160, otherwise known as the Local Government Code of 1991; "(b) Barangay — thirty-five percent (35%).
and "Provided, however, That where the natural resources
"(b) Eighty percent (80%) shall accrue to the National are located in two (2) or more cities, the allocation of
Government. shares shall be based on the formula on population and
land area as specified in subsection (C)(1) hereof.
Sec287. Shares of Local Government Units in the Proceeds
from the Development and Utilization of the National
Wealth. — Local government units shall have an equitable RA 8245 (1996) – An Act Appropriating The Sum Of
share in the proceeds derived from the utilization and P14,455,000,000 For The Increased Share In 1997 Of LGUs
development of the national wealth, within their respective In The National Internal Revenue Taxes And For Other
areas, including sharing the same with the inhabitants by way of Purposes
direct benefits.
Sec1. Appropriation, Allotment and Release of Funds. — The
"(A) Amount of Share of Local Government Units. — Local amount of Fourteen billion four hundred fifty-five million pesos
government units shall, in addition to the internal revenue (14,455,000,000) is hereby appropriated for the increased share
allotment, have a share of forty percent (40%) of the gross in 1997 of local government units in the internal revenue taxes
collection derived by the national government from the pursuant to Section 284 of Republic Act No. 7160, the Local
preceding fiscal year from excise taxes on mineral products, Government Code of 1991, and shall be apportioned among the
royalties, and such other taxes, fees or charges, including local government units in accordance with the formula
related surcharges, interests or fines, and from its share in any prescribed in Section 285 thereof, after deducting the actual
co-production, joint venture or production sharing agreement in cost of devolution and the cost of city-funded hospitals as of

The Apocryphal Maggots:


Rainier, Chrisgel, Corina, Geoffry, Grace and Sylvie Blanche
The Flibbertigibbet Worms:
Golda, Gladys and Melyjane
CA VE AT: By simply r ea ding this re vi ew er a t the end o f the sem este r wi ll (hopef ully) guar antee you r pas sing this cou rse. D rink mode rat ely .
Amusin S C A r y
gly
THE C2005 LOCAL GOVERNMENT REVIEWER - 229 -
December 31, 1992. Said internal revenue allotment shall be "Over P30,000 but not over P70,000 P2,500+15%
released directly by the Department of Budget and Management of the excess over P30,000
to the local government units concerned. "Over P70,000 but not over P140,000 P8,500+20%
of the excess over P70,000
Sec2. Use of Funds. — The use of funds herein appropriated "Over P140,000 but not over P250,000 P22,500+25% of the
shall be in accordance with the provisions of the Local excess over P140,000
Government Code of 1991, Republic Act No. 7160. "Over P250,000 but not over P500,000 P50,000+30% of the
excess over P250,000
"Over P500,000
RA 7942 (1995) – An Act Instituting A New System Of P125,000+34% of the excess over P500,000 in 1998.
Mineral Resources Exploration, Development, Utilization, "Provided, That effective January 1, 1999, the top marginal rate
And Conservation shall be thirty-three percent (33%) and effective January 1,
2000, the said rate shall be thirty-two percent (32%).
Sec82. Allocation of Government Share. — The Government "For married individuals, the husband and wife, subject to the
share as referred to in the preceding sections shall be shared provision of Section 51(D) hereof, shall compute separately their
and allocated in accordance with Sections 290 and 292 of individual income tax based on their respective total taxable
Republic Act No. 7160 otherwise known as the Local income: Provided, That if any income cannot be definitely
Government Code of 1991. In case the development and attributed to or identified as income exclusively earned or
utilization of mineral resources is undertaken by a government- realized by either of the spouses, the same shall be divided
owned or -controlled corporation, the sharing and allocation equally between the spouses for the purpose of determining
shall be in accordance with Sections 291 and 292 of the said their respective taxable income.
Code. "(B) Rate of Tax on Certain Passive Income: —
"(1) Interests, Royalties, Prizes, and Other Winnings. — A
final tax at the rate of twenty percent (20%) is hereby
imposed upon the amount of interest from any currency
RA8424 amending Sec24 of RA7916 - An Act Amending The
bank deposit and yield or any other monetary benefit
National Internal Revenue Code, As Amended, And For
from deposit substitutes and from trust funds and similar
Other Purposes
arrangements; royalties, except on books, as well as
other literary works and musical compositions, which
Sec24. Income Tax Rates. — shall be imposed a final tax of ten percent (10%); prizes
"(A) Rates of Income Tax on Individual Citizen and (except prizes amounting to Ten thousand pesos
Individual Resident Alien of the Philippines. — (P10,000) or less which shall be subject to tax under
"(1) An income tax is hereby imposed: Subsection (A) of Section 24; and other winnings (except
"(a) On the taxable income defined in Section 31 Philippine Charity Sweepstakes and Lotto winnings),
of this Code, other than income subject to tax under derived from sources within the Philippines: Provided,
Subsections (B), (C) and (D) of this Section, derived for however, That interest income received by an individual
each taxable year from all sources within and without the taxpayer (except a nonresident individual) from a
Philippines by every individual citizen of the Philippines depository bank under the expanded foreign currency
residing therein; deposit system shall be subject to a final income tax at
"(b) On the taxable income defined in Section 31 the rate of seven and one-half percent (7 ½%) of such
of this Code, other than income subject to tax under interest income: Provided, further, That interest income
Subsections (B), (C) and (D) of this Section, derived for from long-term deposit or investment in the form of
each taxable year from all sources within the Philippines savings, common or individual trust funds, deposit
by an individual citizen of the Philippines who is residing substitutes, investment management accounts and other
outside of the Philippines including overseas contract investments evidenced by certificates in such form
workers referred to in Subsection (C) of Section 23 prescribed by the Bangko Sentral ng Pilipinas (BSP)
hereof; and shall be exempt from the tax imposed under this
"(c) On the taxable income defined in Section 31 Subsection: Provided, finally, That should the holder of
of this Code, other than income subject to tax under the certificate pre-terminate the deposit or investment
Subsections (B), (C) and (D) of this Section, derived for before the fifth (5th) year, a final tax shall be imposed on
each taxable year from all sources within the Philippines the entire income and shall be deducted and withheld by
by an individual alien who is a resident of the the depository bank from the proceeds of the long-term
Philippines. deposit or investment certificate based on the remaining
"The tax shall be computed in accordance with and at the rates maturity thereof:
established in the following schedule: "Four (4) years to less than five (5) years — 5%;
"Not over P10,000 5% "Three (3) years to less than four (4) years — 12%; and
"Over P10,000 but not over P30,000 P500+10% of "Less than three (3) years — 20%.
the excess over P10,000

The Apocryphal Maggots:


Rainier, Chrisgel, Corina, Geoffry, Grace and Sylvie Blanche
The Flibbertigibbet Worms:
Golda, Gladys and Melyjane
CA VE AT: By simply r ea ding this re vi ew er a t the end o f the sem este r wi ll (hopef ully) guar antee you r pas sing this cou rse. D rink mode rat ely .
Amusin S C A r y
gly
THE C2005 LOCAL GOVERNMENT REVIEWER - 230 -
"(2) Cash and/or Property Dividends. — A final tax at the the historical cost or adjusted basis of the real property
following rates shall be imposed upon the cash and/or sold or disposed shall be carried over to the new
property dividends actually or constructively received by principal residence built or acquired: Provided, further,
an individual from a domestic corporation or from a joint That the Commissioner shall have been duly notified by
stock company, insurance or mutual fund companies the taxpayer within thirty (30) days from the date of sale
and regional operating headquarters of multinational or disposition through a prescribed return of his intention
companies, or on the share of an individual in the to avail of the tax exemption herein mentioned:
distributable net income after tax of a partnership Provided, still further, That the said tax exemption can
(except a general professional partnership) of which he only be availed of once every ten (10) years: Provided,
is a partner, or on the share of an individual in the net finally, That if there is no full utilization of the proceeds of
income after tax of an association, a joint account, or a sale or disposition, the portion of the gain presumed to
joint venture or consortium taxable as a corporation of have been realized from the sale or disposition shall be
which he is a member or co-venturer: subject to capital gains tax. For this purpose, the gross
"Six percent (6%) beginning January 1, 1998; selling price or fair market value at the time of sale,
"Eight percent (8%) beginning January 1, 1999; whichever is higher, shall be multiplied by a fraction
"Ten percent (10%) beginning January 1, 2000. which the unutilized amount bears to the gross selling
"Provided, however, That the tax on dividends shall price in order to determine the taxable portion and the
apply only on income earned on or after January 1, tax prescribed under paragraph (1) of this Subsection
1998. Income forming part of retained earnings as of shall be imposed thereon.
December 31, 1997 shall not, even if declared or
distributed on or after January 1, 1998, be subject to this
tax. cdtai RA 8975 (2000) – An Act To Ensure The Expeditious
"(C) Capital Gains from Sale of Shares of Stock not Implementation And Completion Of Government
Traded in the Stock Exchange. — The provisions of Section Infrastructure Projects By Prohibiting Lower Courts From
39(B) notwithstanding, a final tax at the rates prescribed below Issuing Temporary Restraining Orders, Preliminary
is hereby imposed upon the net capital gains realized during the Injunctions Or Preliminary Mandatory Injunctions,
taxable year from the sale, barter, exchange or other disposition Providing Penalties For Violations Thereof, And For Other
of shares of stock in a domestic corporation, except shares sold, Purposes
or disposed of through the stock exchange.
"Not over P100,000 5% Sec7. Issuance of Permits. — Upon payment in cash of the
"On any amount in excess of P100,000 10% necessary fees levied under Republic Act No. 7160, as
"(D) Capital Gains from Sale of Real Property. — amended, otherwise known as the Local Government Code of
"(1) In General. — The provisions of Section 39(B) 1991, the governor of the province or mayor of a highly-
notwithstanding, a final tax of six percent (6%) based on urbanized city shall immediately issue the necessary permit to
the gross selling price or current fair market value as extract sand, gravel and other quarry resources needed in
determined in accordance with Section 6(E) of this government projects. The issuance of said permit shall consider
Code, whichever is higher, is hereby imposed upon environmental laws, land use ordinances and the pertinent
capital gains presumed to have been realized from the provisions of the Local Government Code relating to
sale, exchange, or other disposition of real property environment.
located in the Philippines, classified as capital assets,
including pacto de retro sales and other forms of
conditional sales, by individuals, including estates and
RA 9165 (2002) - An Act Instituting The Comprehensive
trusts: Provided, That the tax liability, if any, on gains
Dangerous Drugs Act Of 2002, Repealing Republic Act No.
from sales or other dispositions of real property to the
6425, Otherwise Known As The Dangerous Drugs Act Of
government or any of its political subdivisions or
1972, As Amended, Providing Funds Therefor, And For
agencies or to government-owned or -controlled
Other Purposes
corporations shall be determined either under Section
24(A) or under this Subsection, at the option of the
taxpayer; Sec51. Local Government Units' Assistance. — Local
"(2) Exception. — The provisions of paragraph (1) of this government units shall appropriate a substantial portion of their
Subsection to the contrary notwithstanding, capital gains respective annual budgets to assist in or enhance the
presumed to have been realized from the sale or enforcement of this Act giving priority to preventive or
disposition of their principal residence by natural educational programs and the rehabilitation or treatment of drug
persons, the proceeds of which is fully utilized in dependents.
acquiring or constructing a new principal residence
within eighteen (18) calendar months from the date of From class notes:
sale or disposition, shall be exempt from the capital Where do LGUs get their money?
gains tax imposed under this Subsection: Provided, That  portion of the internal revenue collected (IRA)

The Apocryphal Maggots:


Rainier, Chrisgel, Corina, Geoffry, Grace and Sylvie Blanche
The Flibbertigibbet Worms:
Golda, Gladys and Melyjane
CA VE AT: By simply r ea ding this re vi ew er a t the end o f the sem este r wi ll (hopef ully) guar antee you r pas sing this cou rse. D rink mode rat ely .
Amusin S C A r y
gly
THE C2005 LOCAL GOVERNMENT REVIEWER - 231 -
 power to levy taxes City of Olongapo v. Stallholders of the East Bajac – Bajac
 just share in the utilization of natural resources Public Market of Olongapo City (2000)
 also generate own source
2/3 of income of LGUs come from IRA Respondents questioned the validity of an ordinance fixing the
monthly rental fees for stall in the public market. They alleged
among others, that the rates were unjust, excessive, oppressive,
Basco v. PAGCOR (1991) confiscatory and not equitable. The Sec. of Justice upheld its
validity and refrained from taking action when MR was filed.
Respondents then field an actin for declaration of nullity of the
Manila, being a mere municipal corporation, has no inherent right ordinance with the RTC. RTC, without trial, rendered a decision
to impose taxes. Congress has the power of control over local sustaining the validity of the ordinance.
governments. If Congress can grant Manila the power to tax
certain matters, it can also provide for exemptions or even take Held: The RTC’s function is not limited to determining whether
back the power. there is evidence in the administrative record substantial enough to
support the findings therein. As determined by the allegations of
LGUs also have no power to tax instrumentalities of the National the complaint or pet., the pet. filed with the RTC cannot be limited
Government. PAGCOR, being such, should be and actually is to reviewing evidence adduced before the Sec. of Justice. The
exempt from local taxes. Otherwise, mere creatures of the State Memorandum and the accompanying letter amounted to an
can defeat national policies thru extermination of what local abdication by the Sec. of his jurisdiction over the appeal.
authorities may perceive to be undesirable activities or enterprise
using the power to tax as a tool for regulation.
From class notes:
When can a tax law be considered oppressive?
From class notes:
 wide area of discretion
LGU no inherent power to tax? Not exactly true, because there
is constitutional basis (but can’t say that it is inherent)  factual issue that needs trial – thresh it out at the court
BASCO IS NO LONGER GOOD LAW!!! level

Estanislao v. Costales (1991)


Figuerres v. CA (1999)
An ordinance of Zamboanga City imposed a P.01 tax per liter of
Figuerres received a notice of assessment from the municipal softdrinks produced, manufactured and/or bottled within the
treasurer, which was based on a number of ordinances. Petitioner territorial jurisdiction of Z City.
thus bought a prohibition suit on the ground that the ordinances
were invalid for having been adopted without public hearings and Held: The ordinance is ultra vires and is not within the authority of
prior publication or posting. the city to impose. The authority of the city is limited to the
imposition of a percentage tax on the gross sales or receipts of
Held: Petitioner failed to exhaust administrative remedies. The said product. The tax being imposed under said ordinance is
legal questions raised by petitioner require proof of facts for their based on the output or production and not on the gross sales or
resolution. The action before the CA was thus premature. receipts.

It is the Sec. of Justice who can take cognizance of a case The ordinance cannot be considered valid on the ground that the
involving the constitutionality or legality of tax ordinances where, Minister of Finance failed to take action within the prescribed
as in this case, there are factual issues involved. period. There is no authority under the Local Tax Code for the
conclusion that since the minister failed to act or otherwise
There need be no fear that compliance with the rule on exhaustion suspend the effectivity of the tax ordinance within 120 days from
will unduly delay resort to courts. Sec. 187 requires the Sec. to receipt of a copy thereof, said ordinance is valid and remains in
render a decision within 60 days from the date of receipt of the force. By the Sec.’s failure to act, it does not follow as a legal
appeal, after which the aggrieved party may file appropriate consequence that an otherwise invalid ordinance is thereby
proceedings with a court. validated.

The presented period for review is only directory and the Sec. may
From class notes: still review the ordinance and act accordingly even after the lapse
no need to go to DOJ Sec. on pure questions of law of the said period provided he acts within a reasonable time.
when there is a factual issue – go to DOJ SEC. Same goes for the provincial and city treasurers.
Sir: but one might still be able to go to court.
From class notes:
Period now is 60 days (not 120 days).
You can only tax as you are told to tax.

The Apocryphal Maggots:


Rainier, Chrisgel, Corina, Geoffry, Grace and Sylvie Blanche
The Flibbertigibbet Worms:
Golda, Gladys and Melyjane
CA VE AT: By simply r ea ding this re vi ew er a t the end o f the sem este r wi ll (hopef ully) guar antee you r pas sing this cou rse. D rink mode rat ely .
Amusin S C A r y
gly
THE C2005 LOCAL GOVERNMENT REVIEWER - 232 -

First Philippine Industrial Corp. v CA (1998)


Floro Cement v. Gorospe (1991)
First Philippine Industrial Corporation is a grantee of a pipeline
As the power of taxation is a high prerogative of sovereignty, the concession to contract, install and operate pipelines. The City
general rule is that any claim for exemption from the tax statute Treasurer required it to pay a local tax based on its gross receipts
should be strictly construed against the taxpayer. He who claims for the fiscal year 1993 pursuant to Sec. 143 of the LGC: “the
an exemption must be able to point out some provision of law municipality may impose taxes on ‘contractors and other
creating the right; it cannot be allowed to exist upon a mere independent contractors’ in accordance with a prescribed
implication or inference. schedule.

The manufacture and export of cement does not fall under the Held: FPIC falls under the definition of a common carrier under the
exemption provision for it is not a mineral product. It is not cement CC. It is engaged in the business of transporting or carrying foods,
that is mined, only the mineral products composing the finished i.e. petroleum products, for hire as a public employment. It
product. undertakes to carry for all persons indifferently, that is, to all
persons who choose to employ its services, and transports the
goods by land and for compensation.
Bulacan v. CA (1998)
Since FPIC is a common carrier, it is exempt from the business tax
The provincial ordinance of Bulacan provides that a tax of 10% of as provided in Sec. 133. The legislative intent in excluding from
the fair market value will be imposed per cubic meter of stones, the taxing power of the LGU the imposition of business tax against
sand and other quarry resources extracted from public lands. The common carriers is to prevent a duplication of the so-called
provincial treasurer assessed Republic Cement taxes for extracting “common carrier’s tax.”
materials from several parcels of private land.
Now FPIC is already paying 3% common carrier’s tax on its gross
Held: The province had no authority to impose taxes on materials sales/earnings under the NIRC. To tax petitioner again on its gross
extracted from private lands. Section 138 of the LGC expressly receipts in its transportation of petroleum business would defeat
prohibits the province from imposing taxes on stones, etc extracted the purpose of the LGC.
from private lands. Section 133 of the LGC also prohibits the
province from levying excise taxes like the one imposed in the
CAB. Palma Development Corp. v. Municipality of Malangas
(2003)
NIRC levies a tax on all quarry resources, regardless of origin,
whether extracted from public or private lands. The province was A service fee imposed on vehicles using municipal roads leading to
only expressly allowed to levy such tax on extractions from public the wharf is valid. By express language of Secs 153 and 155 of
lands thru the LGC provision. The court therefore did not unjustly LGC, LGU’s, through their SP/SB, may prescribe the terms and
deprive them of the power to create sources of revenue, their conditions for the imposition of toll fees or charges for the use of
assessment of tax against Republic being ultra vires. any public road, pier or wharf funded and constructed by them.

The legality of the ordinance was never questioned by the CA. But wharfage labeled as a service fee “for police surveillance on all
What the CA questioned was the assessment and not the goods” is not valid. Section 133(e) of LGC prohibits the imposition,
ordinance itself. A reading of the ordinance, which is a practically a in the guise of wharfage, of fees -- as well as all other taxes or
reproduction of Section 138 of the LGC limits the province’s rights charges in any form whatsoever -- on goods or merchandise. It is
to public land extractions. Bulacan may not invoke the Regalian therefore irrelevant if the fees imposed are actually for police
doctrine to extend coverage of their ordinance, for taxes, being surveillance on the goods, because any other form of imposition on
burdens, are not to be presumed beyond what the applicable goods passing through the territorial jurisdiction of the municipality
statute expressly and clearly declares. is clearly prohibited by Section 133(e).

From class notes: Wharfage: a fee assessed against the cargo of a vessel engaged
Taxing power of national government vis-à-vis taxing power of in foreign or domestic trade based on quantity, weight, or measure
LGU – are they mutually exclusive? Yes. received and/or discharged by vessel.
Gen. Rule: If something is already taxed by the national
government, it can no longer be taxed by the LGU. From class notes:
Exception: Unless allowed by LGC. Prohibition limited to goods, not use of the road
In the case above, LGU is allowed to tax, but the LGC provides
that the LGU may tax only PUBLIC lands.
Amusement Taxes

The Apocryphal Maggots:


Rainier, Chrisgel, Corina, Geoffry, Grace and Sylvie Blanche
The Flibbertigibbet Worms:
Golda, Gladys and Melyjane
CA VE AT: By simply r ea ding this re vi ew er a t the end o f the sem este r wi ll (hopef ully) guar antee you r pas sing this cou rse. D rink mode rat ely .
Amusin S C A r y
gly
THE C2005 LOCAL GOVERNMENT REVIEWER - 233 -
factories, project offices, plants, and plantations in the pursuit of
Sec140. Amusement Tax. — their business:
(a) The province may levy an amusement tax to be collected (1) Thirty percent (30%) of all sales recorded in the principal
from the proprietors, lessees, or operators of theaters, cinemas, office shall be taxable by the city or municipality where
concert halls, circuses, boxing stadia, and other places of the principal office is located; and
amusement at a rate of not more than thirty percent (30%) of the (2) Seventy percent (70%) of all sales recorded in the principal
gross receipts from admission fees. office shall be taxable by the city or municipality where
(b) In the case of theaters or cinemas, the tax shall first be the factory, project office, plant, or plantation is located.
deducted and withheld by their proprietors, lessees, or operators (c) In case of a plantation located at a place other than the place
and paid to the provincial treasurer before the gross receipts are where the factory is located, said seventy percent (70%)
divided between said proprietors, lessees, or operators and the mentioned in subparagraph (b) of subsection (2) above shall be
distributors of the cinematographic films. divided as follows:
(c) The holding of operas, concerts, dramas, recitals, painting (1) Sixty percent (60%) to the city or municipality where the
and art exhibitions, flower shows, musical programs, literary and factory is located; and
oratorical presentations, except pop, rock, or similar concerts (2) Forty percent (40%) to the city or municipality where the
shall be exempt from the payment of the tax hereon imposed. plantation is located.
(d) The sangguniang panlalawigan may prescribe the time, (d) In cases where a manufacturer, assembler, producer,
manner, terms and conditions for the payment of tax. In case of exporter or contractor has two (2) or more factories, project
fraud or failure to pay the tax, the sangguniang panlalawigan offices, plants, or plantations located in different localities, the
may impose such surcharges, interest and penalties as it may seventy percent (70%) sales allocation mentioned in
deem appropriate. subparagraph (b) of subsection (2) above shall be prorated
(e) The proceeds from the amusement tax shall be shared among the localities where the factories, project offices, plants,
equally by the province and the municipality where such and plantations are located in proportion to their respective
amusement places are located. volumes of production during the period for which the tax is due.
(e) The foregoing sales allocation shall be applied irrespective of
Sec141. Annual Fixed Tax For Every Delivery Truck or Van of whether or not sales are made in the locality where the factory,
Manufacturers or Producers, Wholesalers of, Dealers, or project office, plant, or plantation is located.
Retailers in, Certain Products. —
(a) The province may levy an annual fixed tax for every truck,
van or any vehicle used by manufacturers, producers, RA 9167 (2002) – An Act Creating The Film Development
wholesalers, dealers or retailers in the delivery or distribution of Council Of The Philippines, Defining Its Powers And
distilled spirits, fermented liquors, soft drinks, cigars and Functions, Appropriating Funds Therefor, And For Other
cigarettes, and other products as may be determined by the Purposes
sangguniang panlalawigan, to sales outlets, or consumers,
whether directly or indirectly, within the province in an amount Sec13. Privileges of Graded Films. — Films which have obtained
not exceeding Five hundred pesos (P500.00). an "A" or "B" grading from the Council pursuant to Sections 11
(b) The manufacturers, producers, wholesalers, dealers and and 12 of this Act shall be entitled to the following privileges:
retailers referred to in the immediately foregoing paragraph shall a. Amusement tax reward. — A grade "A" or "B" film shall entitle
be exempt from the tax on peddlers prescribed elsewhere in this its producer to an incentive equivalent to the amusement tax
Code. imposed and collected on the graded films by cities and
municipalities in Metro Manila and other highly urbanized and
Sec150. Situs of the Tax. — independent component cities in the Philippines pursuant to
(a) For purposes of collection of the taxes under Section 143 of Sections 140 and 151 of Republic Act No. 7160 at the following
this Code, manufacturers, assemblers, repackers, brewers, rates:
distillers, rectifiers and compounders of liquor, distilled spirits 1. For grade "A" films — 100% of the amusement tax collected
and wines, millers, producers, exporters, wholesalers, on such films; and
distributors, dealers, contractors, banks and other financial 2. For grade "B" films — 65% of the amusement tax collected on
institutions, and other businesses, maintaining or operating such films. The remaining thirty-five (35%) shall accrue
branch or sales outlet elsewhere shall record the sale in the to the funds of the Council.
branch or sales outlet making the sale or transaction, and the
tax thereon shall accrue and shall be paid to the municipality Sec14. Amusement Tax Deduction and Remittances. — All
where such branch or sales outlet is located. In cases where revenue from the amusement tax on the graded film which may
there is no such branch or sales outlet in the city or municipality otherwise accrue to the cities and municipalities in Metropolitan
where the sale or transaction is made, the sale shall be duly Manila and highly urbanized and independent component cities
recorded in the principal office and the taxes due shall accrue in the Philippines pursuant to Section 140 of Republic Act No.
and shall be paid to such city or municipality. 7160 during the period the graded film is exhibited, shall be
(b) The following sales allocation shall apply to manufacturers, deducted and withheld by the proprietors, operators or lessees
assemblers, contractors, producers, and exporters with of theatres or cinemas and remitted within thirty (30) days from

The Apocryphal Maggots:


Rainier, Chrisgel, Corina, Geoffry, Grace and Sylvie Blanche
The Flibbertigibbet Worms:
Golda, Gladys and Melyjane
CA VE AT: By simply r ea ding this re vi ew er a t the end o f the sem este r wi ll (hopef ully) guar antee you r pas sing this cou rse. D rink mode rat ely .
Amusin S C A r y
gly
THE C2005 LOCAL GOVERNMENT REVIEWER - 234 -
the termination of the exhibition to the Council which shall Amusement taxes – There shall be collected from the
reward the corresponding amusement tax to the producers of proprietor, lessee, or operator of …professional basketball
the graded film within fifteen (15) days from receipt thereof. games …a tax equivalent to (4) Fifteen percentum [of their
Proprietors, operators and lessees of theaters or cinemas who gross receipts] …in lieu of all other percentage taxes of
fail to remit the amusement tax proceeds within the prescribed whatever nature and description.
period shall be liable to a surcharge equivalent to five percent Under the principle of ejusdem generis, the phrase “other places of
(5%) of the amount due for each month of delinquency which amusement” in §13 must pertain to artistic forms of entertainment,
shall be paid to the Council. which is the common characteristic of the enumeration “theaters,
cinematographs, concert halls and circuses” prior to the phrase.
Sec15. Enforcement. — For purposes of implementing the Professional basketball does not pertain to the arts, but caters to
previous Section, the Council may: sports and gaming. Therefore, it is not covered by the general
1. Impose administrative fines and penalties of not more than phrase.
One hundred thousand pesos (P100,000.00) for the violation of
any provision of this Act and/or its implementing rules and From class notes:
regulations issued by the Council; This is still good law, although the LGC now includes “boxing
2. Cause or initiate the criminal or administrative prosecution of stadia”.
violators of this Act and its implementing rules and regulations. The LGU can still tax advertisements by PBA.
For this purpose, the Council is hereby given the power to issue
subpoena duces tecum and enlist the assistance or services of
the Department of Justice or the Office of the Solicitor General;
Franchise Taxes
3. Cause the closure of any theater or cinema that fails or
unreasonably refuses to remit the tax collected on a graded film
for a period not exceeding fifteen (15) days after which such Sec137. Franchise Tax. — Notwithstanding any exemption
period shall be automatically lifted without prejudice to the right granted by any law or other special law, the province may
of the Council to cause or initiate criminal or administrative impose a tax on businesses enjoying a franchise, at the rate not
prosecution against the erring theater or cinema owners exceeding fifty percent (50%) of one percent (1%) of the gross
pursuant to paragraph 2 of this section; annual receipts for the preceding calendar year based on the
4. Call upon representatives of the different government incoming receipt, or realized, within its territorial jurisdiction.
agencies and other various associations in the movie industry to In the case of a newly started business, the tax shall not exceed
help ensure compliance with the provisions of this Act and its one-twentieth (1/20) of one percent (1%) of the capital
implementing rules and regulations. investment. In the succeeding calendar year, regardless of when
For this purpose, the Council may constitute Local Regulatory the business started to operate, the tax shall be based on the
Council or Councils in the cities and municipalities throughout gross receipts for the preceding calendar year, or any fraction
the country composed of representatives from the government thereon, as provided herein.
and from the private sector as may be appropriate to implement
the purposes and objectives of this Act and its implementing Sec193. Withdrawal of Tax Exemption Privileges. — Unless
rules and regulations and they shall serve without compensation otherwise provided in this Code, tax exemptions or incentives
and only for such period of time as the Chairperson shall granted to, or presently enjoyed by all persons, whether natural
determine; or juridical, including government-owned or controlled
5. To deputize any law enforcement agency and instrumentality corporations, except local water districts, cooperatives duly
of the government for assistance in the implementation and registered under R.A. No. 6938, non-stock and non-profit
enforcement of its functions and orders. hospitals and educational institutions, are hereby withdrawn
upon the effectivity of this Code.

Sec534. Repealing Clause. —


(a) Batas Pambansa Blg. 337, otherwise known as the Local
Government Code, Executive Order No. 112 (1987), and
Executive Order No. 319 (1988) are hereby repealed.
(b) Presidential Decree Nos. 684, 1191, 1508 and such other
decrees, orders, instructions, memoranda and issuances related
PBA v. CA (2000)
to or concerning the barangay are hereby repealed.
(c) The provisions of Sections 2, 3, and 4 of Republic Act No.
PBA claims that the power to collect amusement taxes has been 1939 regarding hospital fund; Section 3, a (3) and b (2) of
transferred from the national to the local governments. Republic Act No. 5447 regarding the Special Education Fund;
Presidential Decree No. 144 as amended by Presidential
Held: The authority to tax professional basketball games is NOT Decree Nos. 559 and 1741; Presidential Decree No. 231 as
INCLUDED in §13 of the Local Tax Code, the same is expressly amended; Presidential Decree No. 436 as amended by
embraced in §44 of PD1959, w/c provides: Presidential Decree No. 558; and Presidential Decree Nos. 381,

The Apocryphal Maggots:


Rainier, Chrisgel, Corina, Geoffry, Grace and Sylvie Blanche
The Flibbertigibbet Worms:
Golda, Gladys and Melyjane
CA VE AT: By simply r ea ding this re vi ew er a t the end o f the sem este r wi ll (hopef ully) guar antee you r pas sing this cou rse. D rink mode rat ely .
Amusin S C A r y
gly
THE C2005 LOCAL GOVERNMENT REVIEWER - 235 -
436, 464, 477, 526, 632, 752, and 1136 are hereby repealed
and rendered of no force and effect. National Power Corp. v. Cabanatuan City (2003)
(d) Presidential Decree No. 1594 is hereby repealed insofar as it
governs locally-funded projects. NPC is a GOCC tasked to undertake the development of
(e) The following provisions are hereby repealed or amended hydroelectric generations of power and the production of electricity
insofar as they are inconsistent with the provisions of this Code: and the transmission of electric power on a nationwide basis.
Sections 2, 16 and 29 of Presidential Decree No. 704; Section
12 of Presidential Decree No. 87, as amended; Section 52, 53, Held: NPC is still liable to pay a franchise tax.
66, 67, 68, 69, 70, 71, 72, 73, and 74 of Presidential Decree No. General Rule: LGUs cannot impose taxes, fees or charges of any
463, as amended; and Section 16 of Presidential Decree No. kind on the National Government, its agencies and
972, as amended, and instrumentalities.
(f) All general and special laws, acts, city charters, decrees, Exception: when specific provisions of the LGC authorize the
executive orders, proclamations and administrative regulations, LGUs to impose taxes, fees or charges on the aforementioned
or part or parts thereof which are inconsistent with any of the entities. (Sec 133, LGC)
provisions of this Code are hereby repealed or modified
accordingly. Under the present LGC, the doctrine in Basco vs. PAGCOR no
longer applies. In enacting the LGC, Congress exercised its
prerogative to tax instrumentalities and agencies of government as
MERALCO v. Province of Laguna (1999) it sees fit.

Meralco can be taxed by the province. The LGC explicitly NPC fulfilled the requisites to be subject to Franchise tax. A
authorizes provincial governments, notwithstanding “any franchise tax is "a tax on the privilege of transacting business in
exemptions granted by any law or other special law…(to) impose a the state and exercising corporate franchises granted by the state."
tax on businesses enjoying a franchise.” Indicative of the
legislative intent to vest broad tax powers to LGU's, the LGC has Primary Franchise: relates to the right to exist as a corporation,
effectively withdrawn tax exemptions or incentives enjoyed by by virtue of duly approved articles of incorporation, or a charter
certain entities. In addition, the LGC also contains a general pursuant to a special law creating the corporation.
repealing clause in Section 534.
Secondary Franchise: refers to the right or privileges conferred
The phrase "in lieu of all taxes" in a franchise have to give way to upon an existing corporation such as the right to use the streets of
the peremptory language of the LGC specifically providing for the a municipality to lay pipes of tracks, erect poles or string wires. It is
withdrawal of exemptions from local taxation. within this context that the phrase "tax on businesses enjoying a
franchise" in section 137 of the LGC should be interpreted and
While the SC has frequently referred to tax exemptions contained understood. To determine whether NPC is covered by the
in special franchises as being in the nature of contracts and a part franchise tax in question, the following requisites should concur:
of the inducement for carrying on the franchise, these exemptions
are far from being strictly contractual in nature. A franchise • that petitioner has a "franchise" in the sense of a secondary
partakes the nature of a grant which is beyond the purview on the or special franchise;
non-impairment clause of the Constitution. What is covered by the • that it is exercising its rights or privileges under this franchise
term "contractual tax exemption" and under the purview of non- within the territory of the city gov’t.
impairment of contracts: those agreed to by taxing authority in NPC satisfies both requisites.
contracts such as government bonds / debentures, lawfully
entered into by the parties under enabling laws in which the As to the contention that it should be excluded because its stocks
government waives its governmental immunity from suit. are wholly owned by the National Government, a franchise tax is
imposed based not on the ownership but on the exercise by the
corporation of a privilege to do business. The taxable entity is the
PLDT v. City of Davao (2001) corporation which exercises the franchise, and not the individual
stockholders.
PLDT is liable for the local franchise tax. Section 137 does not
state that it covers future exemptions. The grant of taxing powers By virtue of its charter, NPC was created as a separate and distinct
to LGU's in the consti and the LGC does not affect the power of entity from the National Government. It can sue and be sued under
Congress to grant exemptions in the future. The only legal effect of its own name and can exercise all the powers of a corporation
the constitutional grant to local governemnts: in interpreting under the Corporation Code. The ownership by the National
statutory provisions on municipal taxing powers, doubts should be Government of its entire capital stock does not necessarily imply
resolved in favor of municipal corporations. that petitioner is not engaged in business.

NPC generates power and sells electricity in bulk. Certainly, these


activities do not partake of the sovereign functions of the

The Apocryphal Maggots:


Rainier, Chrisgel, Corina, Geoffry, Grace and Sylvie Blanche
The Flibbertigibbet Worms:
Golda, Gladys and Melyjane
CA VE AT: By simply r ea ding this re vi ew er a t the end o f the sem este r wi ll (hopef ully) guar antee you r pas sing this cou rse. D rink mode rat ely .
Amusin S C A r y
gly
THE C2005 LOCAL GOVERNMENT REVIEWER - 236 -
government. They are purely private and commercial undertakings, (k) "Economic Life" is the estimated period over which it
albeit imbued with public interest. is anticipated that a machinery or equipment may be profitably
utilized;
(l) "Fair Market Value" is the price at which a property
Real Pr op erty Taxa ti on may be sold by a seller who is not compelled to sell and bought
by a buyer who is not compelled to buy;
CHAPTER I: General Provisions (m) "Improvement" is a valuable addition to a property or
Sec197. Scope. — This Title shall govern the administration, an amelioration in its condition, amounting to more than a mere
appraisal, assessment, levy and collection of real property tax. repair or replacement of parts involving capital expenditures and
labor, which is intended to enhance its value, beauty or utility or
Sec198. Fundamental Principles. — The appraisal, assessment, to adapt it for new or further purposes;
levy and collection of real property tax shall be guided by the (n) "Industrial Land" is land devoted principally to
following fundamental principles: industrial activity as capital investment and is not classified as
(a) Real property shall be appraised at its current and fair agricultural, commercial, timber, mineral or residential land;
market value; (o) "Machinery" embraces machines, equipment,
(b) Real property shall be classified for assessment mechanical contrivances, instruments, appliances or apparatus
purposes on the basis of its actual use; which may or may not be attached, permanently or temporarily,
(c) Real property shall be assessed on the basis of a to the real property. It includes the physical facilities for
uniform classification within each local government unit; production, the installations and appurtenant service facilities,
(d) The appraisal, assessment, levy and collection of real those which are mobile, self-powered or self-propelled, and
property tax shall not be let to any private person; and those not permanently attached to the real property which are
(e) The appraisal and assessment of real property shall actually, directly, and exclusively used to meet the needs of the
be equitable. particular industry, business or activity and which by their very
nature and purpose are designed for, or necessary to its
Sec199. Definitions. — When used in this Title: manufacturing, mining, logging, commercial, industrial or
(a) "Acquisition Cost" for newly-acquired machinery not agricultural purposes:
yet depreciated and appraised within the year of its purchase, (p) "Mineral Lands" are lands in which minerals, metallic
refers to the actual cost of the machinery to its present owner, or non-metallic, exist in sufficient quantity or grade to justify the
plus the cost of transportation, handling, and installation at the necessary expenditures to extract and utilize such materials;
present site; (q) "Reassessment" is the assigning of new assessed
(b) "Actual Use" refers to the purpose for which the values to property, particularly real estate, as the result of a
property is principally or predominantly utilized by the person in general, partial, or individual reappraisal of the property;
possession thereof; (r) "Remaining Economic Life" is the period of time
(c) "Ad Valorem Tax" is a levy on real property expressed in years from the date of appraisal to the date when
determined on the basis of a fixed proportion of the value of the the machinery becomes valueless;
property; (s) "Remaining Value" is the value corresponding to the
(d) "Agricultural Land" is land devoted principally to the remaining useful life of the machinery;
planting of trees, raising of crops, livestock and poultry, dairying, (t) "Replacement or Reproduction Cost" is the cost that
salt making, inland fishing and similar aquacultural activities, would be incurred on the basis of current prices, in acquiring an
and other agricultural activities, and is not classified as mineral, equally desirable substitute property, or the cost of reproducing
timber, residential, commercial or industrial land; a new replica of the property on the basis of current prices with
(e) "Appraisal" is the act or process of determining the the same or closely similar material; and
value of property as of a specified date for a specific purpose; (u) "Residential Land" is land principally devoted to
(f) "Assessment" is the act or process of determining the habitation.
value of a property, or proportion thereof subject to tax, including
the discovery, listing, classification, and appraisal of properties; Sec200. Administration of the Real Property Tax. — The
(g) "Assessment Level" is the percentage applied to the provinces and cities, including the municipalities within the
fair market value to determine the taxable value of the property; Metropolitan Manila Area, shall be primarily responsible for the
(h) "Assessed Value" is the fair market value of the real proper, efficient and effective administration of the real property
property multiplied by the assessment level. It is synonymous to tax.
taxable value;
(i) "Commercial Land" is land devoted principally for the CHAPTER II: Appraisal and Assessment of Real Property
object of profit and is not classified as agricultural, industrial, Sec201. Appraisal of Real Property. — All real property, whether
mineral, timber, or residential land; taxable or exempt, shall be appraised at the current and fair
(j) "Depreciated Value" is the value remaining after market value prevailing in the locality where the property is
deducting depreciation from the acquisition cost; situated. The Department of Finance shall promulgate the
necessary rules and regulations for the classification, appraisal,

The Apocryphal Maggots:


Rainier, Chrisgel, Corina, Geoffry, Grace and Sylvie Blanche
The Flibbertigibbet Worms:
Golda, Gladys and Melyjane
CA VE AT: By simply r ea ding this re vi ew er a t the end o f the sem este r wi ll (hopef ully) guar antee you r pas sing this cou rse. D rink mode rat ely .
Amusin S C A r y
gly
THE C2005 LOCAL GOVERNMENT REVIEWER - 237 -
and assessment of real property pursuant to the provisions of proportionately for all obligations imposed by this Title and the
this Code. payment of the real property tax with respect to the undivided
property.
Sec202. Declaration of real Property by the Owner or (c) The real property of a corporation, partnership, or
Administrator. — It shall be the duty of all persons, natural or association shall be listed, valued and assessed in the same
juridical, owning or administering real property, including the manner as that of an individual.
improvements therein, within a city or municipality, or their duly (d) Real property owned by the Republic of the
authorized representative, to prepare, or cause to be prepared, Philippines, its instrumentalities and political subdivisions, the
and file with the provincial, city or municipal assessor, a sworn beneficial use of which has been granted, for consideration or
statement declaring the true value of their property, whether otherwise, to a taxable person, shall be listed, valued and
previously declared or undeclared, taxable or exempt, which assessed in the name of the possessor, grantee or of the public
shall be the current and fair market value of the property, as entity if such property has been acquired or held for resale or
determined by the declarant. Such declaration shall contain a lease.
description of the property sufficient in detail to enable the
assessor or his deputy to identify the same for assessment Sec206. Proof of Exemption of Real Property from Taxation. —
purposes. The sworn declaration of real property herein referred Every person by or for whom real property is declared, who shall
to shall be filed with the assessor concerned once every three claim tax exemption for such property under this Title shall file
(3) years during the period from January first (1st) to June with the provincial, city or municipal assessor within thirty (30)
thirtieth (30th) commencing with the calendar year 1992. days from the date of the declaration of real property sufficient
documentary evidence in support of such claim including
Sec203. Duty of Person Acquiring Real Property or Making corporate charters, title of ownership, articles of incorporation,
Improvement Thereon. — It shall also be the duty of any by-laws, contracts, affidavits, certifications and mortgage deeds,
person, or his authorized representative, acquiring at any time and similar documents.
real property in any municipality or city or making any If the required evidence is not submitted within the period herein
improvement on real property, to prepare, or cause to be prescribed, the property shall be listed as taxable in the
prepared, and file with the provincial, city or municipal assessor, assessment roll. However, if the property shall be proven to be
a sworn statement declaring the true value of subject property, tax exempt, the same shall be dropped from the assessment
within sixty (60) days after the acquisition of such property or roll.
upon completion or occupancy of the improvement, whichever
comes earlier. Sec207. Real Property Identification System. — All declarations
of real property made under the provisions of this Title shall be
Sec204. Declaration of Real Property by the Assessor. — kept and filed under a uniform classification system to be
When any person, natural or juridical, by whom real property is established by the provincial, city or municipal assessor.
required to be declared under Section 202 hereof, refuses or
fails for any reason to make such declaration within the time Sec208. Notification of Transfer of Real Property Ownership.
prescribed, the provincial, city or municipal assessor shall — Any person who shall transfer real property ownership to
himself declare the property in the name of the defaulting owner, another shall notify the provincial, city or municipal assessor
if known, or against an unknown owner, as the case may be, concerned within sixty (60) days from the date of such transfer.
and shall assess the property for taxation in accordance with the The notification shall include the mode of transfer, the
provision of this Title. No oath shall be required of a declaration description of the property alienated, the name and address of
thus made by the provincial, city or municipal assessor. the transferee.

Sec205. Listing of Real Property in the Assessment Rolls. — Sec209. Duty of Registrar of Deeds to Appraise Assessor of
(a) In every province and city, including the municipalities Real Property Listed in Registry. —
within the Metropolitan Manila Area, there shall be prepared and (a) To ascertain whether or not any real property entered
maintained by the provincial, city or municipal assessor an in the Registry of Property has escaped discovery and listing for
assessment roll wherein shall be listed all real property, whether the purpose of taxation, the Registrar of Deeds shall prepare
taxable or exempt, located within the territorial jurisdiction of the and submit to the provincial, city or municipal assessor, within
local government unit concerned. Real property shall be listed, six (6) months from the date of effectivity of this Code and every
valued and assessed in the name of the owner or administrator, year thereafter, an abstract of his registry, which shall include
or anyone having legal interest in the property. brief but sufficient description of the real properties entered
(b) The undivided real property of a deceased person therein, their present owners, and the dates of their most recent
may be listed, valued and assessed in the name of the estate or transfer or alienation accompanied by copies of corresponding
of the heirs and devisees without designating them individually; deeds of sale, donation, or partition or other forms of alienation.
and undivided real property other than that owned by a (b) It shall also be the duty of the Registrar of Deeds to
deceased may be listed, valued and assessed in the name of require every person who shall present for registration a
one or more co-owners: Provided, however, That such heir, document of transfer, alienation, or encumbrance of real
devisee, or co-owner shall be liable severally and property to accompany the same with a certificate to the effect

The Apocryphal Maggots:


Rainier, Chrisgel, Corina, Geoffry, Grace and Sylvie Blanche
The Flibbertigibbet Worms:
Golda, Gladys and Melyjane
CA VE AT: By simply r ea ding this re vi ew er a t the end o f the sem este r wi ll (hopef ully) guar antee you r pas sing this cou rse. D rink mode rat ely .
Amusin S C A r y
gly
THE C2005 LOCAL GOVERNMENT REVIEWER - 238 -
that the real property subject of the transfer, alienation, or
encumbrance, as the case may be, has been fully paid of all real Sec215. Classes of Real Property for Assessment Purposes.
property taxes due thereon. Failure to provide such certificate — For purposes of assessment, real property shall be classified
shall be a valid cause for the Registrar of Deeds to refuse the as residential, agricultural, commercial, industrial, mineral,
registration of the document. timberland or special.
The city or municipality within the Metropolitan Manila Area,
Sec210. Duty of Official Issuing Building Permit or Certificate through their respective sanggunian, shall have the power to
of Registration of Machinery to Transmit Copy to Assessor. classify lands as residential, agricultural, commercial, industrial,
— Any public official or employee who may now or hereafter be mineral, timberland, or special in accordance with their zoning
required by law or regulation to issue to any person a permit for ordinances.
the construction, addition, repair, or renovation of a building, or
permanent improvement on land, or a certificate of registration Sec216. Special Classes of Real Property. — All lands,
for any machinery, including machines, mechanical buildings, and other improvements thereon actually, directly and
contrivances, and apparatus attached or affixed on land or to exclusively used for hospitals, cultural, or scientific purposes,
another real property, shall transmit a copy of such permit or and those owned and used by local water districts, and
certificate within thirty (30) days of its issuance, to the assessor government-owned or controlled corporations rendering
of the province, city or municipality where the property is essential public services in the supply and distribution of water
situated. and/or generation and transmission of electric power shall be
classified as special.
Sec211. Duty of Geodetic Engineers to Furnish Copy of Plans
to Assessor. — It shall be the duty of all geodetic engineers, Sec217. Actual Use of Real Property as Basis for Assessment.
public or private, to furnish free of charge to the assessor of the — Real property shall be classified, valued and assessed on the
province, city or municipality where the land is located with a basis of its actual use regardless of where located, whoever
white or blue print copy of each of all approved original or owns it, and whoever uses it.
subdivision plans or maps of surveys executed by them within
thirty (30) days from receipt of such plans from the Lands Sec218. Assessment Levels. — The assessment levels to be
Management Bureau, the Land Registration Authority, or the applied to the fair market value of real property to determine its
Housing and Land Use Regulatory Board, as the case may be. assessed value shall be fixed by ordinances of the sangguniang
panlalawigan, sangguniang panlungsod or sangguniang bayan
Sec212. Preparation of Schedule of Fair Market Values. — of a municipality within the Metropolitan Manila Area, at the
Before any general revision of property assessment is made rates not exceeding the following:
pursuant to the provisions of this Title, there shall be prepared a (a) On Lands:
schedule of fair market values by the provincial, city and CLASS ASSESSMENT LEVELS
municipal assessor of the municipalities within the Metropolitan Residential 20%
Manila Area for the different classes of real property situated in Agricultural 40%
their respective local government units for enactment by Commercial 50%
ordinance of the sanggunian concerned. The schedule of fair Industrial 50%
market values shall be published in a newspaper of general Mineral 50%
circulation in the province, city or municipality concerned or in Timberland 20%
the absence thereof, shall be posted in the provincial capitol, (b) On Buildings and Other Structures:
city or municipal hall and in two other conspicuous public places (1) Residential
therein. Fair market Value
Over Not Over Assessment Levels
Sec213. Authority of Assessor to Take Evidence. — For the
purpose of obtaining information on which to base the market P175,000.00 0%
value of any real property, the assessor of the province, city or P175,000.00 300,000.00 10%
municipality or his deputy may summon the owners of the 300,000.00 500,000.00 20%
properties to be affected or persons having legal interest therein 500,000.00 750,000.00 25%
and witnesses, administer oaths, and take deposition 750,000.00 1,000,000.00 30%
concerning the property, its ownership, amount, nature, and 1,000,000.00 2,000,000.00 35%
value. 2,000,000.00 5,000,000.00 40%
5,000,000.00 10,000,000.00 50%
Sec214. Amendment of Schedule of Fair Market Values. — The 10,000,000.00 60%
provincial, city or municipal assessor may recommend to the (2) Agricultural
sanggunian concerned amendments to correct errors in Fair Market Value
valuation in the schedule of fair market values. The sanggunian Over Not Over Assessment Levels
concerned shall, by ordinance, act upon the recommendation
within ninety (90) days from receipt thereof. P300,000.00 25%

The Apocryphal Maggots:


Rainier, Chrisgel, Corina, Geoffry, Grace and Sylvie Blanche
The Flibbertigibbet Worms:
Golda, Gladys and Melyjane
CA VE AT: By simply r ea ding this re vi ew er a t the end o f the sem este r wi ll (hopef ully) guar antee you r pas sing this cou rse. D rink mode rat ely .
Amusin S C A r y
gly
THE C2005 LOCAL GOVERNMENT REVIEWER - 239 -
P300,000.00 500,000.00 30% city or municipal assessor or his duly authorized deputy shall, in
500,000.00 750,000.00 35% accordance with the provisions of this Chapter, make a
750,000.00 1,000,000.00 40% classification, appraisal and assessment or taxpayer's valuation
1,000,000.00 2,000,000.00 45% thereon: Provided, however, That the assessment of real
2,000,000.00 50% property shall not be increased oftener than once every three (3)
(3) Commercial / Industrial years except in case of new improvements substantially
Fair Market Value increasing the value of said property or of any change in its
Over Not Over Assessment Levels actual use.

P300,000.00 30% Sec221. Date of Effectivity of Assessment or Reassessment.


P300,000.00 500,000.00 35% — All assessments or reassessments made after the first (1st)
500,000.00 750,000.00 40% day of January of any year shall take effect on the first (1st) day
750,000.00 1,000,000.00 50% of January of the succeeding year: Provided, however, That the
1,000,000.00 2,000,000.00 60% reassessment of real property due to its partial or total
2,000,000.00 5,000,000.00 70% destruction, or to a major change in its actual use, or to any
5,000,000.00 10,000,000.00 75% great and sudden inflation or deflation of real property values, or
10,000,000.00 80% to the gross illegality of the assessment when made or to any
(4) Timberland other abnormal cause, shall be made within ninety (90) days
Fair Market Value from the date any such cause or causes occurred, and shall
Over Not Over Assessment Levels take effect at the beginning of the quarter next following the
P300,000.00 45% reassessment.
P300,000.00 500,000.00 50%
500,000.00 750,000.00 55% Sec222. Assessment of Property Subject to Back Taxes. —
750,000.00 1,000,000.00 60% Real property declared for the first time shall be assessed for
5,000,000.00 2,000,000.00 65% taxes for the period during which it would have been liable but in
2,000,000.00 70% no case of more than ten (10) years prior to the date of initial
(c) On Machineries assessment: Provided, however, That such taxes shall be
Class Assessment Levels computed on the basis of the applicable schedule of values in
Agricultural 40% force during the corresponding period.
Residential 50% If such taxes are paid on or before the end of the quarter
Commercial 80% following the date the notice of assessment was received by the
Industrial 80% owner or his representative, no interest for delinquency shall be
(d) On Special Classes: The assessment levels for all imposed thereon; otherwise, such taxes shall be subject to an
lands buildings, machineries and other improvements; interest at the rate of two percent (2%) per month or a fraction
Actual Use Assessment Level thereof from the date of the receipt of the assessment until such
Cultural 15% taxes are fully paid.
Scientific 15%
Hospital 15% Sec223. Notification of New or Revised Assessment. — When
Local water districts 10% real property is assessed for the first time or when an existing
Government-owned or assessment is increased or decreased, the provincial, city or
controlled corporations municipal assessor shall within thirty (30) days give written
engaged in the supply notice of such new or revised assessment to the person in
and distribution of water whose name the property is declared. The notice may be
and/or generation and delivered personally or by registered mail or through the
transmission of assistance of the punong barangay to the last known address of
electric power 10% the person to be served.
Sec224. Appraisal and Assessment of Machinery. —
Sec219. General Revision of Assessment and Property (a) The fair market value of a brand-new machinery shall
Classification. — The provincial, city or municipal assessor be the acquisition cost. In all other cases, the fair market value
shall undertake a general revision of real property assessments shall be determined by dividing the remaining economic life of
within two (2) years after the effectivity of this Code and every the machinery by its estimated economic life and multiplied by
three (3) years thereafter. the replacement or reproduction cost.
(b) If the machinery is imported, the acquisition cost
Sec220. Valuation of Real Property. — In cases where (a) real includes freight, insurance, bank and other charges, brokerage,
property is declared and listed for taxation purposes for the first arrastre and handling, duties and taxes, plus charges at the
time; (b) there is an ongoing general revision of property present site. The cost in foreign currency of imported machinery
classification and assessment; or (c) a request is made by the shall be converted to peso cost on the basis of foreign currency
person in whose name the property is declared, the provincial, exchange rates as fixed by the Central Bank.

The Apocryphal Maggots:


Rainier, Chrisgel, Corina, Geoffry, Grace and Sylvie Blanche
The Flibbertigibbet Worms:
Golda, Gladys and Melyjane
CA VE AT: By simply r ea ding this re vi ew er a t the end o f the sem este r wi ll (hopef ully) guar antee you r pas sing this cou rse. D rink mode rat ely .
Amusin S C A r y
gly
THE C2005 LOCAL GOVERNMENT REVIEWER - 240 -
enable the Board in their respective localities to operate
Sec225. Depreciation Allowance for Machinery. — For effectively.
purposes of assessment, a depreciation allowance shall be Sec229. Action by the Local Board of Assessment Appeals. —
made for machinery at a rate not exceeding five percent (5%) of (a) The Board shall decide the appeal within one hundred
its original cost or its replacement or reproduction cost, as the twenty (120) days from the date of receipt of such appeal. The
case may be, for each year of use: Provided, however, That the Board, after hearing, shall render its decision based on
remaining value for all kinds of machinery shall be fixed at not substantial evidence or such relevant evidence on record as a
less than twenty percent (20%) of such original, replacement, or reasonable mind might accept as adequate to support the
reproduction cost for so long as the machinery is useful and in conclusion.
operation. (b) In the exercise of its appellate jurisdiction, the Board
shall have the power to summon witnesses, administer oaths,
CHAPTER III: Assessment Appeals conduct ocular inspection, take depositions, and issue
Sec226. Local Board of Assessment Appeals. — Any owner or subpoena and subpoena duces tecum. The proceedings of the
person having legal interest in the property who is not satisfied Board shall be conducted solely for the purpose of ascertaining
with the action of the provincial, city or municipal assessor in the the facts without necessarily adhering to technical rules
assessment of his property may, within sixty (60) days from the applicable in judicial proceedings.
date of receipt of the written notice of assessment, appeal to the (c) The secretary of the Board shall furnish the owner of
Board of Assessment Appeals of the provincial or city by filing a the property or the person having legal interest therein and the
petition under oath in the form prescribed for the purpose, provincial or city assessor with a copy of the decision of the
together with copies of the tax declarations and such affidavits Board. In case the provincial or city assessor concurs in the
or documents submitted in support of the appeal. revision or the assessment, it shall be his duty to notify the
owner of the property or the person having legal interest therein
Sec227. Organization, Powers, Duties, and Functions of the of such fact using the form prescribed for the purpose. The
Local Board of Assessment Appeals. — owner of the property or the person having legal interest therein
(a) The Board of Assessment Appeals of the province or or the assessor who is not satisfied with the decision of the
city shall be composed of the Registrar of Deeds, as Chairman, Board, may, within thirty (30) days after receipt of the decision of
the provincial or city prosecutor and the provincial, or city said Board, appeal to the Central Board of Assessment Appeals,
engineer as members, who shall serve as such in an ex officio as herein provided. The decision of the Central Board shall be
capacity without additional compensation. final and executory.
(b) The chairman of the Board shall have the power to
designate any employee of the province or city to serve as Sec230. Central Board of Assessment Appeals. — The Central
secretary to the Board also without additional compensation. Board of Assessment Appeals shall be composed of a chairman,
(c) The chairman and members of the Board of and two (2) members to be appointed by the President, who
Assessment Appeals of the province or city shall assume their shall serve for a term of seven (7) years, without reappointment.
respective positions without need of further appointment or Of those first appointed, the chairman shall hold office for seven
special designations immediately upon effectivity of this Code. (7) years, one member for five (5) years, and the other member
They shall take oath or affirmation of office in the prescribed for three (3) years. Appointment to any vacancy shall be only for
form. the unexpired portion of the term of the predecessor. In no case
(d) In provinces and cities without a provincial or city shall any member be appointed or designated in a temporary or
engineer, the district engineer shall serve as member of the acting capacity. The chairman and the members of the Board
Board. In the absence of the Registrar of Deeds, or the shall be Filipino citizens, at least forty (40) years old at the time
provincial or city prosecutor, or the provincial or city engineer, or of their appointment, and members of the Bar or Certified Public
the district engineer, the persons performing their duties, Accountants for at least ten (10) years immediately preceding
whether in an acting capacity or as a duly designated officer-in- their appointment. The chairman of the Board of Assessment
charge, shall automatically become the chairman or member, Appeals shall have the salary grade equivalent to the rank of
respectively, of the said Board, as the case may be. Director III under the Salary Standardization Law exclusive of
allowances and other emoluments. The members of the Board
Sec228. Meetings and Expenses of the Local Board of shall have the salary grade equivalent to the rank of Director II
Assessment Appeals. under the Salary Standardization Law exclusive of allowances
(a) The Board of Assessment Appeals of the province or city and other emoluments. The Board shall have appellate
shall meet once a month and as often as may be necessary for jurisdiction over all assessment cases decided by the Local
the prompt disposition of appealed cases. No member of the Board of Assessment Appeals.
Board shall be entitled to per diems or traveling expenses for his There shall be Hearing Officers to be appointed by the Central
attendance in Board meetings, except when conducting an Board of Assessment Appeals pursuant to civil service laws,
ocular inspection in connection with a case under appeal. rules and regulations, one each for Luzon, Visayas and
(b) All expenses of the Board shall be charged against the Mindanao, who shall hold office in Manila, Cebu City and
general fund of the province or city, as the case may be. The Cagayan de Oro City, respectively, and who shall serve for a
sanggunian concerned shall appropriate the necessary funds to term of six (6) years, without reappointment until their

The Apocryphal Maggots:


Rainier, Chrisgel, Corina, Geoffry, Grace and Sylvie Blanche
The Flibbertigibbet Worms:
Golda, Gladys and Melyjane
CA VE AT: By simply r ea ding this re vi ew er a t the end o f the sem este r wi ll (hopef ully) guar antee you r pas sing this cou rse. D rink mode rat ely .
Amusin S C A r y
gly
THE C2005 LOCAL GOVERNMENT REVIEWER - 241 -
successors have been appointed and qualified. The Hearing Except as provided herein, any exemption from payment of real
Officers shall have the same qualifications as that of the Judges property tax previously granted to, or presently enjoyed by, all
of the Municipal Trial Courts. persons, whether natural or juridical, including all government-
The Central Board Assessment Appeals, in the performance of owned or controlled corporations are hereby withdrawn upon the
its powers and duties, may establish and organize staffs, offices, effectivity of this Code.
units, prescribe the titles, functions and duties of their members
and adopt its own rules and regulations. CHAPTER V: Special Levies on Real Property
Unless otherwise provided by law, the annual appropriations for Sec235. Additional Levy on Real Property for the Special
the Central Board of Assessment Appeals shall be included in Education Fund. — A province or city, or a municipality within
the budget of the Department of Finance in the corresponding the Metropolitan Manila Area, may levy and collect an annual
General Appropriations Act. tax of one percent (1%) on the assessed value of real property
which shall be in addition to the basic real property tax. The
Sec231. Effect of Appeal on the Payment of Real Property Tax. proceeds thereof shall exclusively accrue to the Special
— Appeal on assessments of real property made under the Education Fund (SEF).
provisions of this Code shall, in no case, suspend the collection
of the corresponding realty taxes on the property involved as Sec236. Additional Ad Valorem Tax on Idle Lands. — A
assessed by the provincial or city assessor, without prejudice to province or city, or a municipality within the Metropolitan Manila
subsequent adjustment depending upon the final outcome of the Area, may levy an annual tax on idle lands at the rate not
appeal. exceeding five percent (5%) of the assessed value of the
property which shall be in addition to the basic real property tax.
CHAPTER IV: Imposition of Real Property Tax
Sec232. Power to Levy Real Property Tax. — A province or city Sec237. Idle Lands, Coverage. — For purposes of real property
or a municipality within the Metropolitan Manila Area my levy an taxation, idle lands shall include the following:
annual ad valorem tax on real property such as land, building, (a) Agricultural lands, more than one (1) hectare in area,
machinery, and other improvement not hereinafter specifically suitable for cultivation, dairying, inland fishery, and other
exempted. agricultural uses, one-half (1/2) of which remain uncultivated or
unimproved by the owner of the property or person having legal
Sec233. Rates of Levy. — A province or city or a municipality interest therein. Agricultural lands planted to permanent or
within the Metropolitan Manila Area shall fix a uniform rate of perennial crops with at least fifty (50) trees to a hectare shall not
basic real property tax applicable to their respective localities as be considered idle lands. Lands actually used for grazing
follows: purposes shall likewise not be considered idle lands.
(a) In the case of a province, at the rate not exceeding (b) Lands, other than agricultural, located in a city or
one percent (1%) of the assessed value of real property; and municipality, more than one thousand (1,000) square meters in
(b) In the case of a city or a municipality within the area one-half (1/2) of which remain unutilized or unimproved by
Metropolitan Manila Area, at the rate not exceeding two percent the owner of the property or person having legal interest therein.
(2%) of the assessed value of real property. Regardless of land area, this Section shall likewise apply to
residential lots in subdivisions duly approved by proper
Sec234. Exemptions from Real Property Tax. — The following authorities, the ownership of which has been transferred to
are exempted from payment of the real property tax: individual owners, who shall be liable for the additional tax:
(a) Real property owned by the Republic of the Provided, however, That individual lots of such subdivisions, the
Philippines or any of its political subdivisions except when the ownership of which has not been transferred to the buyer shall
beneficial use thereof has been granted, for consideration or be considered as part of the subdivision, and shall be subject to
otherwise, to a taxable person; the additional tax payable by subdivision owner or operator.
(b) Charitable institutions, churches, parsonages or
convents appurtenant thereto, mosques, non-profit or religious Sec238. Idle Lands Exempt from Tax. — A province or city or a
cemeteries and all lands, buildings, and improvements actually, municipality within the Metropolitan Manila Area may exempt
directly, and exclusively used for religious, charitable or idle lands from the additional levy by reason of force majeure,
educational purposes; civil disturbance, natural calamity or any cause or circumstance
(c) All machineries and equipment that are actually, which physically or legally prevents the owner of the property or
directly and exclusively used by local water districts and person having legal interest therein from improving, utilizing or
government owned or controlled corporations engaged in the cultivating the same.
supply and distribution of water and/or generation and
transmission of electric power; Sec239. Listing of Idle Lands by the Assessor. — The
(d) All real property owned by duly registered provincial, city or municipal assessor shall make and keep an
cooperatives as provided for under R.A. No. 6938; and updated record of all idle lands located within his area of
(e) Machinery and equipment used for pollution control jurisdiction. For purposes of collection, the provincial, city or
and environmental protection. municipal assessor shall furnish a copy thereof to the provincial
or city treasurer who shall notify, on the basis of such record, the

The Apocryphal Maggots:


Rainier, Chrisgel, Corina, Geoffry, Grace and Sylvie Blanche
The Flibbertigibbet Worms:
Golda, Gladys and Melyjane
CA VE AT: By simply r ea ding this re vi ew er a t the end o f the sem este r wi ll (hopef ully) guar antee you r pas sing this cou rse. D rink mode rat ely .
Amusin S C A r y
gly
THE C2005 LOCAL GOVERNMENT REVIEWER - 242 -
owner of the property or person having legal interest therein of
the imposition of the additional tax. Sec245. Accrual of Special Levy. — The special levy shall
accrue on the first day of the quarter next following the effectivity
Sec240. Special Levy by Local Government Units. — A of the ordinance imposing such levy.
province, city or municipality may impose a special levy on the
lands comprised within its territorial jurisdiction specially CHAPTER VI: Collection of Real Property Tax
benefited by public works projects or improvements funded by Sec246. Date of Accrual of Tax. — The real property tax for any
the local government unit concerned: Provided, however, That year shall accrue on the first day of January and from that date it
the special levy shall not exceed sixty percent (60%) of the shall constitute a lien on the property which shall be superior to
actual cost of such projects and improvements, including the any other lien, mortgage, or encumbrance of any kind
costs of acquiring land and such other real property in whatsoever, and shall be extinguished only upon the payment of
connection therewith: Provided, further, That the special levy the delinquent tax.
shall not apply to lands exempt from basic real property tax and
the remainder of the land portions of which have been donated Sec247. Collection of Tax. — The collection of the real property
to the local government unit concerned for the construction of tax with interest thereon and related expenses, and the
such projects or improvements. enforcement of the remedies provided for in this Title or any
applicable laws, shall be the responsibility of the city or
Sec241. Ordinance Imposing a Special Levy. — A tax ordinance municipal treasurer concerned.
imposing a special levy shall describe with reasonable accuracy The city or municipal treasurer may deputize the barangay
the nature, extent, and location of the public works projects or treasurer to collect all taxes on real property located in the
improvements to be undertaken, state the estimated cost barangay: Provided, That the barangay treasurer is properly
thereof, specify the metes and bounds by monuments and lines bonded for the purpose: Provided, further, That the premium on
and the number of annual installments for the payment of the the bond shall be paid by the city or municipal government
special levy which in no case shall be less than five (5) nor more concerned.
than ten (10) years. The sanggunian concerned shall not be
obliged, in the apportionment and computation of the special Sec248. Assessor to Furnish Local Treasurer with
levy, to establish a uniform percentage of all lands subject to the Assessment Roll. — The provincial, city or municipal assessor
payment of the tax for the entire district, but it may fix different shall prepare and submit to the treasurer of the local
rates for different parts or sections thereof, depending on government unit, on or before the thirty-first (31st) day of
whether such land is more or less benefited by proposed work. December each year, an assessment roll containing a list of all
persons whose real properties have been newly assessed or
Sec242. Publication of Proposed Ordinance Imposing a reassessed and the values of such properties.
Special Levy. — Before the enactment of an ordinance
imposing a special levy, the sanggunian concerned shall Sec249. Notice of Time for Collection of Tax. — The city or
conduct a public hearing thereon; notify in writing the owners of municipal treasurer shall, on or before the thirty-first (31st) day
the real property to be affected or the persons having legal of January each year, in the case of the basic real property tax
interest therein as to the date and place thereof and afford the and the additional tax for the Special Education Fund (SEF) or
latter the opportunity to express their positions or objections any other date to be prescribed by the sanggunian concerned in
relative to the proposed ordinance. the case of any other tax levied under this title, post the notice of
the dates when the tax may be paid without interest at a
Sec243. Fixing the Amount of Special Levy. — The special levy conspicuous and publicly accessible place at the city or
authorized herein shall be apportioned, computed, and municipal hall. Said notice shall likewise be published in a
assessed according to the assessed valuation of the lands newspaper of general circulation in the locality once a week for
affected as shown by the books of the assessor concerned, or two (2) consecutive weeks.
its current assessed value as fixed by said assessor if the
property does not appear of record in his books. Upon the Sec250. Payment of Real Property Taxes in Installments. —
effectivity of the ordinance imposing special levy, the assessor The owner of the real property or the person having legal
concerned shall forthwith proceed to determine the annual interest therein may pay the basic real property tax and the
amount of special levy assessed against each parcel of land additional tax for Special Education Fund (SEF) due thereon
comprised within the area especially benefited and shall send to without interest in four (4) equal installments; the first installment
each landowner a written notice thereof by mail, personal to be due and payable on or before March Thirty-first (31st); the
service or publication in appropriate cases. second installment, on or before June Thirty (30); the third
installment, on or before September Thirty (30); and the last
Sec244. Taxpayer's Remedies Against Special Levy. — Any installment on or before December Thirty-first (31st), except the
owner of real property affected by a special levy or any person special levy the payment of which shall be governed by
having a legal interest therein may, upon receipt of the written ordinance of the sanggunian concerned.
notice of assessment of the special levy, avail of the remedies
provided for in Chapter 3, Title Two, Book II of this Code.

The Apocryphal Maggots:


Rainier, Chrisgel, Corina, Geoffry, Grace and Sylvie Blanche
The Flibbertigibbet Worms:
Golda, Gladys and Melyjane
CA VE AT: By simply r ea ding this re vi ew er a t the end o f the sem este r wi ll (hopef ully) guar antee you r pas sing this cou rse. D rink mode rat ely .
Amusin S C A r y
gly
THE C2005 LOCAL GOVERNMENT REVIEWER - 243 -
The date for the payment of any other tax imposed under this newspaper of general circulation in the province, city, or
Title without interest shall be prescribed by the sanggunian municipality.
concerned. (b) Such notice shall specify the date upon which the tax
Payments of real property taxes shall first be applied to prior became delinquent and shall state that personal property may
years delinquencies, interests, and penalties, if any, and only be distrained to effect payment. It shall likewise state that any
after said delinquencies are settled may tax payments be time before the distraint of personal property, payment of the tax
credited for the current period. with surcharges, interests and penalties may be made in
accordance with the next following Section, and unless the tax,
Sec251. Tax Discount for Advanced Prompt Payment. — If the surcharges and penalties are paid before the expiration of the
basic real property tax and the additional tax accruing to the year for which the tax is due except when the notice of
Special Education Fund (SEF) are paid in advance in assessment or special levy is contested administratively or
accordance with the prescribed schedule of payment as judicially pursuant to the provisions of Chapter 3, Title II, Book II
provided under Section 250, the sanggunian concerned may of this Code, the delinquent real property will be sold at public
grant a discount not exceeding twenty percent (20%) of the auction, and the title to the property will be vested in the
annual tax due. purchaser, subject, however, to the right of the delinquent owner
of the property or any person having legal interest therein to
Sec252. Payment Under Protest. — redeem the property within one (1) year from the date of sale.
(a) No protest shall be entertained unless the taxpayer
first pays the tax. There shall be annotated on the tax receipts Sec255. Interests on Unpaid Real Property Tax. — In case of
the words "paid under protest". The protest in writing must be failure to pay the basic real property tax or any other tax levied
filed within thirty (30) days from payment of the tax to the under this Title upon the expiration of the periods as provided in
provincial, city treasurer or municipal treasurer, in the case of a Section 250, or when due, as the case may be, shall subject the
municipality within Metropolitan Manila Area, who shall decide taxpayer to the payment of interest at the rate of two percent
the protest within sixty (60) days from receipt. (2%) per month on the unpaid amount or a fraction thereof, until
(b) The tax or a portion thereof paid under protest, shall the delinquent tax shall have been fully paid: Provided, however,
be held in trust by the treasurer concerned. That in no case shall the total interest on the unpaid tax or
(c) In the event that the protest is finally decided in favor portion thereof exceed thirty-six (36) months.
of the taxpayer, the amount or portion of the tax protested shall
be refunded to the protestant, or applied as tax credit against his Sec256. Remedies For The Collection Of Real Property Tax. —
existing or future tax liability. For the collection of the basic real property tax and any other tax
(d) In the event that the protest is denied or upon the levied under this Title, the local government unit concerned may
lapse of the sixty day period prescribed in subparagraph (a), the avail of the remedies by administrative action thru levy on real
taxpayer may avail of the remedies as provided for in Chapter 3, property or by judicial action.
Title II, Book II of this Code.
Sec257. Local Governments Lien. — The basic real property tax
Sec253. Repayment of Excessive Collections. — When an and any other tax levied under this Title constitutes a lien on the
assessment of basic real property tax, or any other tax levied property subject to tax, superior to all liens, charges or
under this Title, is found to be illegal or erroneous and the tax is encumbrances in favor of any person, irrespective of the owner
accordingly reduced or adjusted, the taxpayer may file a written or possessor thereof, enforceable by administrative or judicial
claim for refund or credit for taxes and interests with the action, and may only be extinguished upon payment of the tax
provincial or city treasurer within two (2) years from the date the and the related interests and expenses.
taxpayer is entitled to such reduction or adjustment.
The provincial or city treasurer shall decide the claim for tax Sec258. Levy on Real Property. — After the expiration of the time
refund or credit within sixty (60) days from receipt thereof. In required to pay the basic real property tax or any other tax
case the claim for tax refund or credit is denied, the taxpayer levied under this Title, real property subject to such tax may be
may avail of the remedies as provided in Chapter 3, Title II, levied upon through the issuance of a warrant on or before, or
Book II of this Code. simultaneously with, the institution of the civil action for the
collection of the delinquent tax. The provincial or city treasurer,
Sec254. Notice of Delinquency in the Payment of the Real or a treasurer of a municipality within the Metropolitan Manila
Property Tax. — Area, as the case may be, when issuing a warrant of levy shall
(a) When the real property tax or any other tax imposed prepare a duly authenticated certificate showing the name of the
under this Title becomes delinquent, the provincial, city or delinquent owner of the property or person having legal interest
municipal treasurer shall immediately cause a notice of the therein, the description of the property, the amount of the tax
delinquency to be posted at the main hall and in a publicly due and the interest thereon. The warrant shall operate with the
accessible and conspicuous place in each barangay of the local force of a legal execution throughout the province, city or a
government unit concerned. The notice of delinquency shall also municipality, within the Metropolitan Manila Area. The warrant
be published once a week for two (2) consecutive weeks, in a shall be mailed to or served upon the delinquent owner of the
real property or person having legal interest therein, or in case

The Apocryphal Maggots:


Rainier, Chrisgel, Corina, Geoffry, Grace and Sylvie Blanche
The Flibbertigibbet Worms:
Golda, Gladys and Melyjane
CA VE AT: By simply r ea ding this re vi ew er a t the end o f the sem este r wi ll (hopef ully) guar antee you r pas sing this cou rse. D rink mode rat ely .
Amusin S C A r y
gly
THE C2005 LOCAL GOVERNMENT REVIEWER - 244 -
he is out of the country or cannot be located, the administrator remedies provided for in this Title, including the expenses of
or occupant of the property. At the same time, written notice of advertisement and sale.
the levy with the attached warrant shall be mailed to or served
upon the assessor and the Registrar of Deeds of the province, Sec261. Redemption of Property Sold. — Within one (1) year
city or municipality within the Metropolitan Manila Area where from the date of sale, the owner of the delinquent real property
the property is located, who shall annotate the levy on the tax or person having legal interest therein, or his representative,
declaration and certificate of title of the property, respectively. shall have the right to redeem the property upon payment to the
The levying officer shall submit a report on the levy to the local treasurer of the amount of the delinquent tax, including the
sanggunian concerned within ten (10) days after receipt of the interest due thereon, and the expenses of sale from the date of
warrant by the owner of the property or person having legal delinquency to the date of sale, plus interest of not more than
interest therein. two percent (2%) per month on the purchase price from the date
of sale to the date of redemption. Such payment shall invalidate
Sec259. Penalty for Failure to Issue and Execute Warrant. — the certificate of sale issued to the purchaser and the owner of
Without prejudice to criminal prosecution under the Revised the delinquent real property or person having legal interest
Penal Code and other applicable laws, any local treasurer or his therein shall be entitled to a certificate of redemption which shall
deputy who fails to issue or execute the warrant of levy within be issued by the local treasurer or his deputy.
one (1) year from the time the tax becomes delinquent or within From the date of sale until the expiration of the period of
thirty (30) days from the date of the issuance thereof, or who is redemption, the delinquent real property shall remain in
found guilty of abusing the exercise thereof in an administrative possession of the owner or person having legal interest therein
or judicial proceeding shall be dismissed from the service who shall be entitled to the income and other fruits thereof.
The local treasurer or his deputy, upon receipt from the
Sec260. Advertisement and Sale. — Within thirty (30) days after purchaser of the certificate of sale, shall forthwith return to the
service of the warrant of levy, the local treasurer shall proceed to latter the entire amount paid by him plus interest of not more
publicly advertise for sale or auction the property or a usable than two percent (2%) per month. Thereafter, the property shall
portion thereof as may be necessary to satisfy the tax be free from lien of such delinquent tax, interest due thereon
delinquency and expenses of sale. The advertisement shall be and expenses of sale.
effected by posting a notice at the main entrance of the
provincial, city or municipal building, and in a publicly accessible Sec262. Final Deed to Purchaser. — In case the owner or person
and conspicuous place in the barangay where the real property having legal interest fails to redeem the delinquent property as
is located, and by publication once a week for two (2) weeks in a provided herein, the local treasurer shall execute a deed
newspaper of general circulation in the province, city or conveying to the purchaser said property, free from lien of the
municipality where the property is located. The advertisement delinquent tax, interest due thereon and expenses of sale. The
shall specify the amount of the delinquent tax, the interest due deed shall briefly state the proceedings upon which the validity
thereon and expenses of sale, the date and place of sale, the of the sale rests.
name of the owner of the real property or person having legal
interest therein, and a description of the property to be sold. At Sec263. Purchase of Property By the Local Government Units
any time before the date fixed for the sale, the owner of the real for Want of Bidder. — In case there is no bidder for the real
property or person havi legal interest therein may stay the property advertised for sale as provided herein, the real property
proceedings by paying the delinquent tax, the interest due tax and the related interest and costs of sale the local treasurer
thereon and the expenses of sale. The sale shall be held either conducting the sale shall purchase the property in behalf of the
at the main entrance of the provincial, city or municipal building, local government unit concerned to satisfy the claim and within
or on the property to be sold, or at any other place as specified two (2) days thereafter shall make a report of his proceedings
in the notice of the sale. which shall be reflected upon the records of his office. It shall be
Within thirty (30) days after the sale, the local treasurer or his the duty of the Registrar of Deeds concerned upon registration
deputy shall make a report of the sale to the sanggunian with his office of any such declaration of forfeiture to transfer the
concerned, and which shall form part of his records. The local title of the forfeited property to the local government unit
treasurer shall likewise prepare and deliver to the purchaser a concerned without the necessity of an order from a competent
certificate of sale which shall contain the name of the purchaser, court.
a description of the property sold, the amount of the delinquent Within one (1) year from the date of such forfeiture, the taxpayer
tax, the interest due thereon, the expenses of sale and a brief or any of his representative, may redeem the property by paying
description of the proceedings: Provided, however, That to the local treasurer the full amount of the real property tax and
proceeds of the sale in excess of the delinquent tax, the interest the related interest and the costs of sale. If the property is not
due thereon, and the expenses of sale shall be remitted to the redeemed as provided herein, the ownership thereof shall be
owner of the real property or person having legal interest vested on the local government unit concerned.
therein.
The local treasurer may, by ordinance duly approved, advance Sec264. Resale of Real Estate Taken for Taxes, Fees, or
an amount sufficient to defray the costs of collection thru the Charges. — The sanggunian concerned may, by ordinance duly
approved, and upon notice of not less than twenty (20) days,

The Apocryphal Maggots:


Rainier, Chrisgel, Corina, Geoffry, Grace and Sylvie Blanche
The Flibbertigibbet Worms:
Golda, Gladys and Melyjane
CA VE AT: By simply r ea ding this re vi ew er a t the end o f the sem este r wi ll (hopef ully) guar antee you r pas sing this cou rse. D rink mode rat ely .
Amusin S C A r y
gly
THE C2005 LOCAL GOVERNMENT REVIEWER - 245 -
sell and dispose of the real property acquired under the of such period. In case of fraud or intent to evade payment of
preceding section at public auction. The proceeds of the sale the tax, such action may be instituted for the collection of the
shall accrue to the general fund of the local government unit same within ten (10) years from the discovery of such fraud or
concerned. intent to evade payment.
The period of prescription within which to collect shall be
Sec265. Further Distraint or Levy. — Levy may be repeated if suspended for the time during which:
necessary until the full amount due, including all expenses, is (1) The local treasurer is legally prevented from collecting
collected. the tax;
(2) The owner of the property or the person having legal
Sec266. Collection of Real Property Tax Through the Courts. interest therein requests for reinvestigation and executes a
— The local government unit concerned may enforce the waiver in writing before the expiration of the period within which
collection of the basic real property tax or any other tax levied to collect; and
under this Title by civil action in any court of competent (3) The owner of the property or the person having legal
jurisdiction. The civil action shall be filed by the local treasurer interest therein is out of the country or otherwise cannot be
within the period prescribed in Section 270 of this Code. located.

Sec267. Action Assailing Validity of Tax Sale. — No court shall CHAPTER VII: Disposition of Proceeds
entertain any action assailing the validity or any sale at public Sec271. Distribution of Proceeds. — The proceeds of the basic
auction of real property or rights therein under this Title until the real property tax, including interest thereon, and proceeds from
taxpayer shall have deposited with the court the amount for the use, lease or disposition, sale or redemption of property
which the real property was sold, together with interest of two acquired at a public auction in accordance with the provisions of
percent (2%) per month from the date of sale to the time of the this Title by the province or city or a municipality within the
institution of the action. The amount so deposited shall be paid Metropolitan Manila Area shall be distributed as follows:
to the purchaser at the auction sale if the deed is declared (a) In the case of provinces:
invalid but it shall be returned to the depositor if the action fails. (1) Province — Thirty-five percent (35%) shall accrue to
Neither shall any court declare a sale at public auction invalid by the general fund;
reason or irregularities or informalities in the proceedings unless (2) Municipality — Forty percent (40%) to the general
the substantive rights of the delinquent owner of the real fund of the municipality where the property is located; and
property or the person having legal interest therein have been (3) Barangay — Twenty-five percent (25%) shall accrue
impaired. to the barangay where the property is located.
(b) In the case of cities:
Sec268. Payment of Delinquent Taxes on Property Subject of (1) City — Seventy percent (70%) shall accrue to the
Controversy. — In any action involving the ownership or general fund of the city; and
possession of, or succession to, real property, the court may, (2) Thirty percent (30%) shall be distributed among the
motu propio or upon representation of the provincial, city, or component barangays of the cities where the property is located
municipal treasurer or his deputy, award such ownership, in the following manner:
possession, or succession to any party to the action upon (i) Fifty percent (50%) shall accrue to the barangay
payment to the court of the taxes with interest due on the where the property is located;
property and all other costs that may have accrued, subject to (ii) Fifty percent (50%) shall accrue equally to all
the final outcome of the action. component barangays of the city; and
(c) In the case of a municipality within the Metropolitan
Sec269. Treasurer to Certify Delinquencies Remaining Manila Area:
Uncollected. — The provincial, city or municipal treasurer or (1) Metropolitan Manila Authority — Thirty-five percent
their deputies shall prepare a certified list of all real property tax (35%) shall accrue to the general fund of the authority;
delinquencies which remained uncollected or unpaid for at least (2) Municipality — Thirty-five percent (35% shall accrue
one (1) year in his jurisdiction, and a statement of the reason or to the general fund of the municipality where the property is
reasons for such non-collection or non-payment, and shall located;
submit the same to the sanggunian concerned on or before (3) Barangays — Thirty percent (30%) shall be distributed
December thirty-first (31st) of the year immediately succeeding among the component barangays of the municipality where the
the year in which the delinquencies were incurred, with a property is located in the following manner:
request for assistance in the enforcement of the remedies for (i) Fifty percent (50%) shall accrue to the barangay
collection provided herein. where the property is located;
(ii) Fifty percent (50%) shall accrue equally to all
Sec270. Periods Within Which To Collect Real Property Taxes. component barangays of the municipality.
— The basic real property tax and any other tax levied under (d) The share of each barangay shall be released,
this Title shall be collected within five (5) years from the date without need of any further action, directly to the barangay
they become due. No action for the collection of the tax, whether treasurer on a quarterly basis within five (5) days after the end
administrative or judicial, shall be instituted after the expiration

The Apocryphal Maggots:


Rainier, Chrisgel, Corina, Geoffry, Grace and Sylvie Blanche
The Flibbertigibbet Worms:
Golda, Gladys and Melyjane
CA VE AT: By simply r ea ding this re vi ew er a t the end o f the sem este r wi ll (hopef ully) guar antee you r pas sing this cou rse. D rink mode rat ely .
Amusin S C A r y
gly
THE C2005 LOCAL GOVERNMENT REVIEWER - 246 -
of each quarter and shall not be subject to any lien or holdback Sec278. Duty of Registrar of Deeds and Notaries Public to
for whatever purpose. Assist the Provincial, City or Municipal Assessor. — It shall
be the duty of the Registrar of Deeds and notaries public to
Sec272. Application of Proceeds of the Additional One furnish the provincial, city or municipal assessor with copies of
Percent SEF Tax. — The proceeds from the additional one all contracts selling, transferring, or otherwise conveying,
percent (1%) tax on real property accruing to the Special leasing, or mortgaging real property received by, or
Education Fund (SEF) shall be automatically released to the acknowledged before them.
local school boards: Provided, That, in case of provinces, the
proceeds shall be divided equally between the provincial and Sec279. Insurance Companies to Furnish Information. —
municipal school boards: Provided, however, That the proceeds Insurance companies are hereby required to furnish the
shall be allocated for the operation and maintenance of public provincial, city or municipal assessor copies of any contract or
schools, construction and repair of school buildings, facilities policy insurance on buildings, structures, and improvements
and equipment, educational research, purchase of books and insured by them or such other documents which may be
periodicals, and sports development as determined and necessary for the proper assessment thereof.
approved by the Local School Board.
Sec280. Fees in Court Actions. — All court actions, criminal or
Sec273. Proceeds of the Tax on Idle Lands. — The proceeds of civil, instituted at the instance of the provincial, city or municipal
the additional real property tax on idle lands shall accrue to the treasurer or assessor under the provisions of this Code, shall be
respective general fund of the province or city where the land is exempt from the payment of court and sheriff's fees.
located. In the case of a municipality within the Metropolitan
Manila Area, the proceeds shall accrue equally to the Sec281. Fees in Registration of Papers or Documents on Sale
Metropolitan Manila Authority and the municipality where the of Delinquent Real Property to Province, City or
land is located. Municipality. — All certificates, documents, and papers
covering the sale of delinquent property to the province, city or
Sec274. Proceeds of the Special Levy. — The proceeds of the municipality, if registered in the Registry of Property, shall be
special levy on lands benefited by public works, projects and exempt from the documentary stamp tax and registration fees.
other improvements shall accrue to the general fund of the local
government unit which financed such public works, projects or Sec282. Real Property Assessment Notices or Owner's Copies
other improvements. of Tax Declarations to be Exempt from Postal Charges or
Fees. — All real property assessment notices or owner's copies
CHAPTER VIII: Special Provisions of tax declaration sent through the mails by the assessor shall
Sec275. General Assessment Revision; Expenses Incident be exempt from the payment of postal charges or fees.
Thereto. — The sanggunian of provinces, cities and
municipalities within the Metropolitan Manila Area shall provide Sec283. Sale and Forfeiture Before Effectivity of Code. — Tax
the necessary appropriations to defray the expenses incident to delinquencies incurred, and sales and forfeitures of delinquent
the general revision of real property assessment. real property effected, before the effectivity of this Code shall be
All expenses incident to a general revision of real property governed by the provisions of applicable laws then in force
assessment shall, by ordinance of the sangguniang
panlalawigan, be apportioned between the province and the Benguet Corp. v. Central Board of Assessment Appeals
municipality on the basis of the taxable area of the municipality (1992)
concerned.
Realty taxes are national taxes collected by LGUs. While LGU’s
Sec276. Condonation or Reduction of Real Property Tax and are charged with fixing the rate of real property taxes, it does not
Interest. — In case of a general failure of crops or substantial necessarily follow from that authority the determination of whether
decrease in the price of agricultural or agribased products, or or not to impose the tax. In fact, LGU’s have no alternative but to
calamity in any province, city or municipality, the sanggunian collect taxes as mandated in Sec. 38 of the Real Property Tax
concerned, by ordinance passed prior to the first (1st) day of Code. It is thus clear that it is the national government, expressing
January of any year and upon recommendation of the Local itself through the legislative branch, that levies the real property
Disaster Coordinating Council, may condone or reduce, wholly tax.
or partially, the taxes and interest thereon for the succeeding
year or years in the city or municipality affected by the calamity. Consequently, when LGU’s are required to fix the rates, they are
merely constituted as agents of the national government in the
Sec277. Condonation or Reduction of Tax by the President of enforcement of the Real Property Tax Code. The delegation of
the Philippines. — The President of the Philippines may, when taxing power is not even involved here because the national
public interest so requires, condone or reduce the real property government has already imposed realty tax in Sec. 38 leaving only
tax and interest for any year in any province or city or a the enforcement to be done LGU’s.
municipality within the Metropolitan Manila Area.
From class notes:

The Apocryphal Maggots:


Rainier, Chrisgel, Corina, Geoffry, Grace and Sylvie Blanche
The Flibbertigibbet Worms:
Golda, Gladys and Melyjane
CA VE AT: By simply r ea ding this re vi ew er a t the end o f the sem este r wi ll (hopef ully) guar antee you r pas sing this cou rse. D rink mode rat ely .
Amusin S C A r y
gly
THE C2005 LOCAL GOVERNMENT REVIEWER - 247 -
Real property tax is a national tax. – still good law? Yes. taxpayers with regard to questions on the legality of a tax
ordinance.

Ordinance 7905 affects the resulting tax imposed on the market


National Development Corp. v. Cebu City (1992)
values of real properties specified in Ordinance 7894. This has
rendered the petition moot and academic for failure to amend
NDC is exempt from the payment of real estate taxes on the cause of action. The facts upon which the petition was anchored
land. no longer exist because the tax rates in Ordinance 7894 have
been amended, or impliedly repealed by Ordinance 7905.
To come within the ambit of the exemption, it is important to
establish that the property is owned by the government or by its Ordinance 7905 is favourable to the taxpayers when it specifically
unincorporated agency. Once government ownership is states that the reduced assessment levels shall be applied
determined, the nature of the use of the property, whether for retroactively. The reduced assessment levels resulted to decrease
proprietary or sovereign purposes, becomes immaterial. in taxes. To that extent, the ordinance is likewise, a social
legislation intended to soften the impact of the tremendous
However in CAB, what appears to have been ceded to NDC is increase in the value of the real properties subject to tax. In
merely the administration of the property while the government enacting this ordinance, the due process of law was considered by
retains ownership of what has been declared reserved for the City of Manila so that the increase in realty tax will not amount
warehousing purposes under the proclamation. to the confiscation of the property.
As reserved land (public land that has been withheld and kept
From class notes:
back from sale or disposition), it remains absolute property of the
Difference between 226 and 252:
government, because the government does not part with its title by
226 – refers to assessment; there is no tax yet
reserving them, but simply gives notice to all that it desires them
252 – tax is excessive; later in the process, one protests the tax
for a certain purpose. As its title remains with the Republic, the
Lopez should have amended complaint
reserved land is covered by the tax exemption provision.

NDC is not exempt from the payment of real estate taxes on


the warehouse. Callanta v. Office of the Ombudsman (1998)

A different rule applies because “the exemption of public property A general revision of assessment was conducted by the Cebu City
from taxation does not extend to improvements on the public lands Assessor’s Office. Notices of assessment together with the new
made by preemptioners, homesteaders and other claimants at tax declarations were sent to the property owners. Thereafter,
their own expense, and these are taxable by the state…(CJS)”. Callanta et. Al., without the authority of the Local Board of
Consequently, warehouse constructed on the reserved land by Assessment Appeals, reassessed the values of certain properties,
NDC should properly be assessed real estate tax as such resulting in the reduction of assessed values of the properties.
improvement does not appear to belong to the Republic. Criminal and administrative charges were then filed against
Callanta and co.
From class notes:
May LGU impose franchise tax on all GOCCs? Held: The aggrieved owners should have brought their appeals
Yes, unless the gov’t provides for a law giving exemption. before the LBAA. Despite the advice to this effect contained in their
respective notices of assessment, the owners chose to bring their
requests for a review/readjustment before the city assessor, a
remedy not sanctioned by the law.
Lopez v. City of Manila (1999)
If this practice was allowed, a tendency to overvalue initially and
Manila Ordinance 7894 taxed the land owned by Lopez to an thereafter to reduce the increases upon “request” of the property
increased 580% value, and as to the improvement, by 250%. So owner would occur. To avoid this dubious, suspicious, bribable and
Lopez filed an action in court to annul the ordinance. compromising situation, the law itself specifically provided an
appellate body — the LBAA — before which property owners may
Later, Ordinance 7905 took effect – reducing by 50% the seek relief.
assessment levels for computation of tax due, amending those
provided by of Ordinance 7894. It also provides that the While PD 464 requires the local assessor to certify to the finance
amendment will have retroactive effect. The new ordinance secretary that the general revision has been finished, such
reduced the tax increase on Lopez’s properties. certification is, however, not the operative act for the effectivity of
the new assessments.
Held: There was failure to exhaust administrative remedies.
Sec187, 226 and 252 of LGC provides for the remedies to the An assessment fixes and determines the tax liability of a taxpayer.
As soon as it is served, an obligation arises on the part of the

The Apocryphal Maggots:


Rainier, Chrisgel, Corina, Geoffry, Grace and Sylvie Blanche
The Flibbertigibbet Worms:
Golda, Gladys and Melyjane
CA VE AT: By simply r ea ding this re vi ew er a t the end o f the sem este r wi ll (hopef ully) guar antee you r pas sing this cou rse. D rink mode rat ely .
Amusin S C A r y
gly
THE C2005 LOCAL GOVERNMENT REVIEWER - 248 -
taxpayer concerned to pay the amount assessed and demanded.
Thus, with respect to real property taxes, the obligation to pay Gen. Rule: The taxing powers of LGUs cannot extend to the levy
arises on the first day of January of the year following the of “taxes, fees and charges of any kind on the National
assessment. Corollarily, on the same date, the right of the local Government, its agencies and instrumentalities, and LGUs.” (Sec.
government to collect said taxes also arises. And where the 133)
taxpayer fails to question such assessment within the reglementary
period provided by law, the local government’s right becomes However, provinces, cities and municipalities in the Metropolitan
absolute upon the expiration of such period with respect to that Mla. Area may impose the real property tax except on “real
taxpayer’s property. In the CAB, there is thus injury or prejudice to property owned by the Republic of the Philippines or any of its
the city gov’t. political subdivisions (Sec. 232), except when the beneficial use
thereof has been granted, for consideration or otherwise, to a
Ty v. Trampe (1995) taxable person.” (Sec. 234)

Assessor sent a notice of assessment respecting certain real As to tax exemptions or incentives granted to or presently enjoyed
properties of petitioners. Later, petitioners filed a petition to by natural or judicial persons, including GOCCS,
declare null and void the new tax assessments and to enjoin Gen. Rule: They are withdrawn upon the effectivity of the LGC,
collection of real estate taxes based on said assessments. except those granted to local water districts, cooperatives duly
registered under RA 6938, non-stock and non-profit hospitals and
Held: PD 921 and RA 7160 are not co-extensive and mutually educational institutions, and unless otherwise provided in the LGC.
inclusive in their scope and purpose. While RA 7160 covers almost
all governmental functions delegated to LGUs all over the country, “Unless otherwise provided in the LGC” could refer to Sec. 234,
PD 921 embraces only the Metropolitan Manila area and is limited which enumerates the properties exempt from real property tax.
to the administration of financial services therein, especially the
assessment and collection of real estate (and some other local) But the last para. of Sec. 234 further qualifies the retention of the
taxes. exemption insofar as real property taxes are concerned by limiting
the retention only to those enumerated herein; all others not
By reading together and harmonizing the 2 provisions of the 2 included in the enumeration lost the privilege upon the effectivity of
laws, we arrive at the ff. steps in the preparation of the schedule of the LGC.
market values:
But even as to real property owned by the Republic or any of its
• The assessor in each municipality or city in the Metropolitan political subdivisions covered by item (a) of the first paragraph of
Section 234, the exemption is withdrawn if the beneficial use of
Mla. Area shall prepare his/her proposed schedule of values
such property has been granted to a taxable person for
(Sec. 212, RA 7160).
consideration or otherwise.
• The Local Treasury and Assessment District shall meet (Sec.
9, PD 921). In this meeting, the different assessors shall MCIAA is a GOCC. It necessarily follows that its exemption from
compare their individual assessments, discuss and thereafter real property tax granted it in its Charter has been withdrawn.
jointly agree and produce a schedule of values for their
district. As to MCIAA’s contention that it is an instrumentality of the gov’t, it
• The schedule jointly agreed upon by the assessors shall then fails to consider the fact that the legislature used the phrase
be published in a newspaper of general circulation and "National Government, its agencies and instrumentalities" in
submitted to the sanggunian concerned for enactment by Section 133(o), but only the phrase "Republic of the Philippines or
ordinance (Sec. 212, RA 7160). any of its political subdivisions" in Section 234(a). “Republic of the
Hence, the schedule of values prepared solely by the municipal Philippines” is a broader term.
assessor is illegal and void.
It is clear that Congress did not wish to expand the scope of the
MCIAA v. Marcos (1996) exemption in Section 234(a) to include real property owned by
other instrumentalities or agencies of the government including
MCIAA is mandated to control, manage and supervise the Mactan GOCCs.
International Airport and other airports in Cebu. City Treasurer
demanded payment for realty taxes on lands belonging to MCIAA. Also, the parcels of land in this case do not belong to the Republic
Petitioner claimed in its favor the provision in its charter which whose beneficial use has been granted to MCIAA. This "transfer"
exempts it from payment of realty taxes. It also claimed that it is is actually an absolute conveyance of the ownership thereof
an instrumentality of the government performing governmental because the petitioner's authorized capital stock consists of "the
functions, citing Sec. 133 of LGC. value of such real estate owned and/or administered by the
airports." Hence, the petitioner is now the owner of the land and
Held: Reading together Secs. 133, 232 and 234 of the LGC, we the exception in Sec. 234(c) of the LGC is inapplicable.
conclude that:

The Apocryphal Maggots:


Rainier, Chrisgel, Corina, Geoffry, Grace and Sylvie Blanche
The Flibbertigibbet Worms:
Golda, Gladys and Melyjane
CA VE AT: By simply r ea ding this re vi ew er a t the end o f the sem este r wi ll (hopef ully) guar antee you r pas sing this cou rse. D rink mode rat ely .
Amusin S C A r y
gly
THE C2005 LOCAL GOVERNMENT REVIEWER - 249 -
From class notes: Unlike public roads which are open for use by everyone, the LRT is
Paranaque can levy on or auction off NAIA properties (including accessible only to those who pay the required fare. The LRTA
the airport runway) for the payment of its tax liabilities, applying does not exist solely for public service, and the LRT carriageways
the Mactan case. and terminal stations are not exclusively for public use.

Although LRTA is a public utility, it is nonetheless profit-earning. It


actually uses those carriageways and terminal stations in its public
NAPOCOR V. Lanao del Sur(1996)
utility business and earns money therefrom.
Considering the entire chain of events, it is clear that petitioner's Even granting that the national government indeed owns the
tax exemptions for the period in question had effectively been carriageways and terminal stations, the exemption would not apply
preserved intact by virtue of their restoration through FIRB because their beneficial use has been granted to petitioner, a
resolutions. taxable entity.
The Real Property Tax Code, PD 464, as amended, expressly
exempts them from such tax. The properties in question comprise
Philreca v. Secretary (2003)
the site of the entire Hydroelectric Power Plant Complex, which
supplies relatively cheap electricity to Mindanao.
Sec. 193 and 234 of the LGC, giving tax exemptions to coops
These are government properties, wholly owned by petitioner and under RA 6938, are constitutional and not violative of equal
devoted directly and solely for public service and utilized in the protection. Substantial distinctions exist between coops under PD
implementation of the state policy of bringing about the total 269 and those under RA 6938:
electrification of the country at the least cost to the public.
Coops under PD Coops under RA
It can be noted, from the relevant laws, that petitioner's non-profit 269 6938
character has been maintained throughout its existence, and that Capital Nowhere in PD 269 Members must
petitioner is mandated to devote all its returns from capital contribution by does it require make equitable
investment and excess revenues from operations to its expansion. members coops to make contributions to the
On account thereof, and to enable petitioner to pay its equitable capital required.
indebtedness and obligations and in furtherance of the state policy contributions to
on electrification and power generation, petitioner has always been capital.
exempted from taxes. Extent of gov’t PD 269 is replete Under the Act, the
control over with provisions State shall maintain
The assessment and levy on as well as the sale of the properties coops which grant the the policy of
of petitioner were null and void for having been made in violation of NEA, upon the noninterference in
PD 938 and the Real Property Tax Code. happening of the management
certain events, the and operation of the
The power to sell at public auction is premised on the real property power to control cooperatives
tax or any portion thereof first becoming delinquent. The properties and take over the (principle of
in this case being exempt from payment of realty taxes, no such management and subsidiarity).
delinquency was possible to begin with. operations of coops Coops envisioned to
registered under it. be self-sufficient
LRTA v. Central Board of Assessment Appeals (2000) and independent
organizations with
minimal government
Though the creation of the LRTA was impelled by public service its
intervention or
operation partakes of ordinary business. LRTA is clothed with
regulation.
corporate status and corporate powers in the furtherance of its
proprietary objectives. Given that it is engaged in a service-
The intention of the law is to broaden the tax base of LGUs to
oriented commercial endeavor, its carriageways and terminal
assure them of substantial sources of revenue.
stations are patrimonial property subject to tax, notwithstanding its
claim of being a GOCC.
Violations of tax ordinances
While LRTA’s carriageways and terminal stations are anchored at
certain points on public roads, these structures do not form part of Sec516. Penalties for Violation of Tax Ordinances. — The
such roads, since the former have been constructed over the latter sanggunian of a local government unit is authorized to prescribe
in such a way that the flow of vehicular traffic would not be fines or other penalties for violation of tax ordinances but in no
impeded. case shall such fines be less than One thousand pesos
(P1,000.00) nor more than Five thousand pesos (P5,000.00),

The Apocryphal Maggots:


Rainier, Chrisgel, Corina, Geoffry, Grace and Sylvie Blanche
The Flibbertigibbet Worms:
Golda, Gladys and Melyjane
CA VE AT: By simply r ea ding this re vi ew er a t the end o f the sem este r wi ll (hopef ully) guar antee you r pas sing this cou rse. D rink mode rat ely .
Amusin S C A r y
gly
THE C2005 LOCAL GOVERNMENT REVIEWER - 250 -
nor shall imprisonment be less than one (1) month nor more Sec284. Allotment of Internal Revenue Taxes. — Local
than six (6) months. Such fine or other penalty, or both, shall be government units shall have a share in the national internal
imposed at the discretion of the court. The sangguniang revenue taxes based on the collection of the third fiscal year
barangay may prescribe a fine of not less than One hundred preceding the current fiscal year as follows:
pesos (P100.00) nor more than One thousand pesos (a) On the first year of the effectivity of this Code, thirty
(P1,000.00). percent (30%);
(b) On the second year, thirty-five percent (35%); and
Sec517. Omission of Property from Assessment or Tax Rolls (c) On the third year and thereafter, forty percent (40%).
by Officers and Other Acts. — Any officer charged with the Provided, That in the event that the national government incurs
duty of assessing real property who willfully fails to assess, or an unmanageable public sector deficit, the President of the
who intentionally omits from the assessment or tax roll any real Philippines is hereby authorized, upon the recommendation of
property which he knows to be taxable, or who willfully or Secretary of Finance, Secretary of Interior and Local
negligently under assesses any real property, or who Government and Secretary of Budget and Management, and
intentionally violates or fails to perform any duty imposed upon subject to consultation with the presiding officers of both Houses
him by law relating to the assessment of taxable real property of Congress and the presidents of the "liga", to make the
shall, upon conviction, be punished by a fine of not less than necessary adjustments in the internal revenue allotment of local
One thousand pesos (P1,000.00) nor more than Five thousand government units but in no case shall the allotment be less than
pesos (P5,000.00), or by imprisonment of not less than one (1) thirty percent (30%) of the collection of national internal revenue
month nor more than six (6) months, or both such fine and taxes of the third fiscal year preceding the current fiscal year:
imprisonment, at the discretion of the court. Provided, further, That in the first year of the effectivity of this
The same penalty shall be imposed upon any officer charged Code, the local government units shall, in addition to the thirty
with the duty of collecting the tax due on real property who percent (30%) internal revenue allotment which shall include the
willfully or negligently fails to collect the tax and institute the cost of devolved functions for essential public services, be
necessary proceedings for the collection of the same. entitled to receive the amount equivalent to the cost of devolved
Any other officer required by this Code to perform acts relating personal services.
to the administration of the real property tax or to assist the
assessor or treasurer in such administration, who willfully fails to Sec285. Allocation to Local Government Units. — The share of
discharge such duties shall, upon conviction be punished by a local government units in the internal revenue allotment shall be
fine of not less than Five hundred pesos (P500.00) nor more collected in the following manner:
than Five thousand pesos (P5,000.00) or imprisonment of not (a) Provinces — Twenty-three percent (23%);
less than one (1) month nor more than six (6) months, or both (b) Cities — Twenty-three percent (23%);
such fine and imprisonment, at the discretion of the court. (c) Municipalities — Thirty-four percent (34%); and
(d) Barangays — Twenty percent (20%)
Sec518. Government Agents Delaying Assessment of Real Provided, however, That the share of each province, city, and
Property and Assessment Appeals. — Any government municipality shall be determined on the basis of the following
official who intentionally and deliberately delays the assessment formula:
of real property or the filing of any appeal against its (a) Population — Fifty percent (50%);
assessment shall, upon conviction, be punished by a fine of not (b) Land Area — Twenty-five percent (25%); and
less than Five hundred pesos (P500.00) nor more than Five (c) Equal sharing — Twenty-five percent (25%)
thousand pesos (P5,000.00), or by imprisonment of not less Provided, further, That the share of each barangay with a
than one (1) month nor more than six (6) months, or both such population of not less than one hundred (100) inhabitants shall
fine and imprisonment, at the discretion of the court. not be less than Eighty thousand (P80,000.00) per annum
chargeable against the twenty percent (20%) share of the
Sec519. Failure to Dispose of Delinquent Real Property at barangay from the internal revenue allotment, and the balance
Public Auction. — The local treasurer concerned who fails to to be allocated on the basis of the following formula:
dispose of delinquent real property at public auction in (a) On the first year of the effectivity of this Code:
compliance with the pertinent provisions of this Code, and any (1) Population — Forty percent (40%); and
other local government official whose acts hinder the prompt (2) Equal sharing — Sixty percent (60%)
disposition of delinquent real property at public auction shall, (b) On the second year:
upon conviction, be subject to a fine of not less than One (1) Population — Fifty percent (50%); and
thousand pesos (P1,000.00) nor more than Five thousand (2) Equal sharing — Fifty percent (50%)
pesos (P5,000.00), or imprisonment of not less than one (1) (c) On the third year and thereafter:
month nor more than six (6) months, or both such fine and (1) Population — Sixty percent (60%); and
imprisonment, at the discretion of the court. (2) Equal sharing — Forty percent (40%).
Provided, finally, That the financial requirements of barangays
Internal Revenue Allotment created by local government units after the effectivity of this
Code shall be the responsibility of the local government unit
concerned.

The Apocryphal Maggots:


Rainier, Chrisgel, Corina, Geoffry, Grace and Sylvie Blanche
The Flibbertigibbet Worms:
Golda, Gladys and Melyjane
CA VE AT: By simply r ea ding this re vi ew er a t the end o f the sem este r wi ll (hopef ully) guar antee you r pas sing this cou rse. D rink mode rat ely .
Amusin S C A r y
gly
THE C2005 LOCAL GOVERNMENT REVIEWER - 251 -
If Congress provides less than 40% as IRA in its general
appropriations act, this is not allowed as an amendment to the
Sec286. Automatic Release of Shares. — LGC.
(a) The share of each local government unit shall be Otherwise, Congress can amend the 40% IRA of the LGC by
released, without need of any further action, directly to the enacting laws.
provincial, city, municipal or barangay treasurer, as the case
may be, on a quarterly basis within five (5) days after the end of
each quarter, and which shall not be subject to any lien or
holdback that may be imposed by the national government for
whatever purpose.
Pimentel v. Aguirre (2000)
(b) Nothing in this Chapter shall be understood to
diminish the share of local government units under existing
laws. Pres. Ramos issued AO 372 entitled “Adoption of Economy
Measures for FY 1998”.
Sec287. Local Development Projects. — Each local government 4. Section 1 directed all government departments and agencies,
unit shall appropriate in its annual budget no less than twenty including LGUs, to reduce total expenditures for the year by
percent (20%) of its annual internal revenue allotment for at least 25%.
development projects. Copies of the development plans of local
government units shall be furnished the Department of Interior
5. Section 4 provided that the amount equivalent to 10% of the
and Local Government. internal revenue allotment to LGUs shall be withheld pending
the assessment and evaluation by the Development Budget
Sec288. Rules and Regulations. — The Secretary of Finance, in Coordinating Committee of the emerging fiscal situation.
consultation with the Secretary of Budget and Management, shall
promulgate the necessary rules and regulations for a simplified Held: Sec. 1 is valid. While the wordings of Sec. 1 have a rather
disbursement scheme designed for the speedy and effective commanding tone, and while the requirements of the LGC (Sec.
enforcement of the provisions of this Chapter. 284) have not been satisfied, the directive to implement measures
that will reduce total expenditures by 25% is merely advisory in
Alvarez v. Guingona character, and does not constitute a mandatory or binding order
1996 that interferes with local autonomy. All concerned could do well to
Santiago has met the minimum average annual income required to heed this advisory. It is understood, however, that no legal sanction
be a component city. IRAs form part of the income of LGUs. They may be imposed upon LGUs and their officials who do not follow
are items of income because they form part of the gross accretion such advice.
of the funds of the LGU. They regularly and automatically accrue to
the local treasury without need of any further action on the part of Sec. 4 is not valid as it encroaches on local fiscal autonomy. A
the LGU. basic feature of local fiscal autonomy is the automatic release of
the shares of the LGUs in the national revenue. This is mandated
Funds generated from local taxes, IRAs, and national wealth by the Constitution and the LGC. Although what is provided for in
utilization proceeds accrue to the general fund of the local Section 4 is merely temporary (pending assessment & evaluation
government and are used to finance its operations subject to by DBCC), it is equivalent to a holdback, which means “something
specified modes of spending. IRAs are considered items of held back or withheld, often temporarily.” Hence, the temporary
income, since income is defined in the LGC to be all revenues and nature of the retention by the national government does not matter.
receipts collected or received forming the gross accretions of funds Any retention is prohibited.
of the LGU.

Sec450(c) also provides that “the average annual income shall Share of LGUs in National Wealth
include the income accruing to the general fund, exclusive of
special funds, transfers, and non-recurring income.” IRA’s are Sec289. Share in the Proceeds from the Development and
regular, recurring item of income. It is not a special fund or Utilization of the National Wealth. — Local government units
transfer, since IRAs have a technical definition and meaning all its shall have an equitable share in the proceeds derived from the
own as used in LGC that unequivocally makes it distinct from utilization and development of the national wealth within their
special funds or transfers referred to when LGC speaks of “funding respective areas, including sharing the same with the
support from the national government, its instrumentalities, and inhabitants by way of direct benefits.
GOCCs.”
Sec290. Amount of Share of Local Government Units. — Local
government units shall, in addition to the internal revenue
From class notes: allotment, have a share of forty percent (40%) of the gross
collection derived by the national government from the
preceding fiscal year from mining taxes, royalties, forestry and

The Apocryphal Maggots:


Rainier, Chrisgel, Corina, Geoffry, Grace and Sylvie Blanche
The Flibbertigibbet Worms:
Golda, Gladys and Melyjane
CA VE AT: By simply r ea ding this re vi ew er a t the end o f the sem este r wi ll (hopef ully) guar antee you r pas sing this cou rse. D rink mode rat ely .
Amusin S C A r y
gly
THE C2005 LOCAL GOVERNMENT REVIEWER - 252 -
fishery charges, and such other taxes, fees, or charges, this chapter shall be appropriated by their respective
including related surcharges, interests, or fines, and from its sanggunian to finance local government and livelihood projects:
share in any co-production, joint venture or production sharing Provided, however, That at least eighty percent (80%) of the
agreement in the utilization and development of the national proceeds derived from the development and utilization of
wealth within their territorial jurisdiction. hydrothermal. geothermal, and other sources of energy shall be
applied solely to lower the cost of electricity in the local
Sec291. Share of the Local Governments from any government unit where such a source of energy is located.
Government Agency or Owned or Controlled Corporation.
— Local government units shall have a share based on the
preceding fiscal year from the proceeds derived by any RA 7076 – An Act Creating A People's Small-Scale Mining
government agency or government-owned or controlled Program And For Other Purposes
corporation engaged in the utilization and development of the
national wealth based on the following formula whichever will Sec19. Government Share and Allotment. — The revenue
produce a higher share for the local government unit: to be derived by the Government from the operation of the
(a) One percent (1%) of the gross sales or receipts of the mining program herein established shall be subject to the
preceding calendar year; or sharing provided in the Local Government Code.
(b) Forty percent (40%) of the mining taxes, royalties,
forestry and fishery charges and such other taxes, fees or
charges, including related surcharges, interests, or fines the
government agency or government owned or controlled
Credit Financing
corporation would have paid if it were not otherwise exempt.

Sec292. Allocation of Shares. — The share in the preceding Sec295. Scope. — This Title shall govern the power of local
Section shall be distributed in the following manner: government units to create indebtedness and to enter into credit
(a) Where the natural resources are located in the and other financial transactions.
province:
(1) Province — Twenty percent (20%); Sec296. General Policy. —
(2) Component City/Municipality — Forty-five percent (a) It shall be the basic policy that any local government
(45%); and unit may create indebtedness, and avail of credit facilities to
(3) Barangay — Thirty-five percent (35%) finance local infrastructure and other socio-economic
Provided, however, That where the natural resources are development projects in accordance with the approved local
located in two (2) or more provinces, or in two (2) or more development plan and public investment program.
component cities or municipalities or in two (2) or more (b) A local government unit may avail of credit lines from
barangays, their respective shares shall be computed on the government or private banks and lending institutions for the
basis of: purpose of stabilizing local finances.
(1) Population — Seventy percent (70%); and
(2) Land area — Thirty percent (30%) Sec297. Loans, Credits, and Other Forms of Indebtedness of
(b) Where the natural resources are located in a highly Local Government Units. —
urbanized or independent component city: (a) A local government unit may contract loans, credits,
(1) City — Sixty-five percent (65%); and and other forms of indebtedness with any government or
(2) Barangay — Thirty-five percent (35%) domestic private bank and other lending institutions to finance
Provided, however, That where the natural resources are the construction, installation, improvement, expansion,
located in such two (2) or more cities, the allocation of shares operation, or maintenance of public facilities, infrastructure
shall be based on the formula on population and land area as facilities, housing projects, the acquisition of real property, and
specified in paragraph (a) of this Section. the implementation of other capital investment projects, subject
to such terms and conditions as may be agreed upon by the
Sec293. Remittance of the Share of Local Government Units. local government unit and the lender. The proceeds from such
— The share of local government units from the utilization and transactions shall accrue directly to the local government unit
development of national wealth shall be remitted in accordance concerned.
with Section 286 of this Code: Provided, however, That in the (b) A local government unit may likewise secure from any
case of any government agency or government-owned or government bank and lending institution short, medium and
controlled corporation engaged in the utilization and long-term loans and advances against security of real estate or
development of the national wealth, such share shall be directly other acceptable assets for the establishment, development, or
remitted to the provincial, city, municipal or barangay treasurer expansion of agricultural, industrial, commercial, house
concerned within five (5) days after the end of each quarter. financing projects, livelihood projects, and other economic
enterprises.
Sec294. Development and Livelihood Projects. — The (c) Government financial and other lending institutions
proceeds from the share of local government units pursuant to are hereby authorized to grant loans, credits, and other forms of

The Apocryphal Maggots:


Rainier, Chrisgel, Corina, Geoffry, Grace and Sylvie Blanche
The Flibbertigibbet Worms:
Golda, Gladys and Melyjane
CA VE AT: By simply r ea ding this re vi ew er a t the end o f the sem este r wi ll (hopef ully) guar antee you r pas sing this cou rse. D rink mode rat ely .
Amusin S C A r y
gly
THE C2005 LOCAL GOVERNMENT REVIEWER - 253 -
indebtedness out of their loanable funds to local government Sec302. Financing, Construction, Maintenance, Operation,
units for purposes specified above. and Management of Infrastructure Projects by the Private
Sector. —
Sec298. Deferred-Payment and other Financial Schemes. — (a) Local government units may enter into contracts with
Provincial, city and municipal governments may likewise acquire any duly prequalified individual contractor, for the financing,
property, plant, machinery, equipment, and such necessary construction, operation, and maintenance of any financially
accessories under a supplier's credit, deferred payment plan, or viable infrastructure facilities, under the build-operate-transfer
either financial scheme. agreement, subject to the applicable provisions of Republic Act
Numbered Sixty-nine hundred fifty-seven (R.A. No. 6957)
Sec299. Bonds and Other Long-Term Securities. — Subject to authorizing the financing, construction, operation and
the rules and regulations of the Central Bank and the Securities maintenance of infrastructure projects by the private sector and
and Exchange Commission, provinces, cities, and municipalities the rules and regulations issued thereunder and such terms and
are hereby authorized to issue bonds, debentures, securities, conditions provided in this Section.
collaterals, notes and other obligations to finance self- (b) Local government units shall include in their
liquidating, income-producing development or livelihood projects respective local development plans and public investment
pursuant to the priorities established in the approved local programs priority projects that may be financed, constructed,
development plan or the public investment program. The operated and maintained by the private sector under this
sanggunian concerned shall, through an ordinance approved by Section. It shall be the duty of the local government unit
a majority of all its members, declare and state the terms and concerned to disclose to the public all projects eligible for
conditions of the bonds and the purpose for which the proposed financing under this Section, including official notification of duly
indebtedness is to be incurred. registered contractors and publications in newspapers of
general or local circulation and in conspicuous and accessible
Sec300. Inter-Local Government Loans, Grants, and public places. Local projects under the build-operate-and-
Subsidies. — Provinces, cities, and municipalities may, upon transfer agreement shall be confirmed by the local development
approval of the majority of all members of the sanggunian councils.
concerned and in amounts not exceeding their surplus funds, (c) Projects implemented under this Section shall be
extend loans, grants, or subsidies to other local government subject to the following terms and conditions:
units under such terms and conditions as may be agreed upon (1) The provincial, city or municipal engineer, as the case
by the contracting parties. may be, upon formal request in writing by the local chief
Local government units may, upon approval of their respective executive, shall prepare the plans and specifications for
sanggunian, jointly or severally contract loans, credits, and other the proposed projects, which shall be submitted to the
forms of indebtedness for purposes mutually beneficial to them. sanggunian for approval.
(2) Upon approval by the sanggunian of the project plans
Sec301. Loans from Funds Secured by the National and specifications, the provincial, city, or municipal
Government from Foreign Sources. — engineer shall, as the case may be, cause to be
(a) The President, or his duly authorized representative, published once every week, for two (2) consecutive
may, through any government financial or other lending weeks in at least one (1) local newspaper which is
institution, relend to any province, city, municipality, or barangay, circulated in the region, province, city or municipality in
the proceeds of loans contracted with foreign financial which the project is to be implemented, a notice inviting
institutions or other international funding agencies for the all duly qualified contractors to participate in a public
purpose of financing the construction, installation, improvement, bidding for the projects so approved. The conduct of
expansion, operation, or maintenance of public utilities and public bidding and award of contracts for local
facilities, infrastructure facilities, or housing projects, the government projects under this Section shall be in
acquisition of real property, and the implementation of other accordance with this Code and other applicable laws,
capital investment projects, subject to such terms and conditions rules and regulations.
as may be agreed upon by the President and the local In the case of a build-operate-and-transfer agreement,
government unit. The proceeds from such loans shall accrue the contract shall be awarded to the lowest complying
directly to the local government concerned. bidder whose offer is deemed most advantageous to the
(b) The President may likewise authorize the relending to local government and based on the present value of its
local government units the proceeds of grants secured from proposed tolls, fees, rentals, and charges over a fixed
foreign sources, subject to the provisions of existing laws and term for the facility to be constructed, operated, and
the applicable grant agreements. (c) Repayment or amortization maintained according to the prescribed minimum design
of loans including accrued interest thereon, may be financed and performance standards, plans, and specifications.
partly from the income of the projects or services and from the For this purpose, the winning contractor shall be
regular income of the local government unit, which must be automatically granted by the local government unit
provided for and appropriated regularly in its annual budget until concerned the franchise to operate and maintain the
the loan and the interest thereon shall have been fully paid. facility, including the collection of tolls, fees, rentals, and
charges in accordance with subsection (c-4) hereof.

The Apocryphal Maggots:


Rainier, Chrisgel, Corina, Geoffry, Grace and Sylvie Blanche
The Flibbertigibbet Worms:
Golda, Gladys and Melyjane
CA VE AT: By simply r ea ding this re vi ew er a t the end o f the sem este r wi ll (hopef ully) guar antee you r pas sing this cou rse. D rink mode rat ely .
Amusin S C A r y
gly
THE C2005 LOCAL GOVERNMENT REVIEWER - 254 -
In the case of a build-operate-and-transfer agreement, Provided, That failure to provide the appropriations herein
the contract shall be awarded to the lowest complying required shall render their annual budgets inoperative.
bidder based on the present value of its proposed
schedule of amortization payments for the facility to be
constructed according to the prescribed minimum design Chavez v. Public Estates Authority (2002)
and performance standards, plans, and specifications.
(3) Any contractor who shall undertake the prosecution of Sec. 302 of the LGC does not constitute legislative authority to sell
any project under this Section shall post the required reclaimed lands to private corporations. The BOT Law recognizes
bonds to protect the interest of the province, city, or the constitutional ban (“…subject to constitutional requirements
municipality, in such amounts as may be fixed by the …”). Although §302 doesn’t contain a proviso similar to that of the
sanggunian concerned and the provincial, city or BOT law, the constitutional restrictions on land ownership
municipal engineer shall not, as the case may be, allow automatically apply even though not expressly mentioned in the
any contractor to initiate the prosecution of projects LGC.
under this Section unless such contractor presents proof Thus, to avoid direct collision of provs of the LGC and the BOT law
or evidence that he has posted the required bond. with the provs of the Consti: If the contractor or developer (in the
(4) The contractor shall be entitled to a reasonable return of reclamation of lands) is –
its investment in accordance with its bid proposal as
accepted by the local government unit concerned. 1. a corporate entity  it can only be paid with leaseholds on
In the case of a build-operate-and-transfer agreement, portions of the reclaimed land
the repayment shall be made by authorizing the 2. an individual  s/he can be paid portions of the reclaimed
contractor to charge and collect reasonable tolls, fees, land, not exceeding 12 hectares of non-agricultural lands.
rentals, and charges for the use of the project facility not
exceeding those proposed in the bid and incorporated in
the contract: Provided, That the local government unit
Local Fiscal Administration
concerned shall, based on reasonableness and equity,
approve the tolls, fees, rentals and charges: Provided,
further, That the imposition and collection of tolls, fees, Sec304. Scope. — This Title shall govern the conduct and
rentals and charges shall be for a fixed period as management of financial affairs, transactions, and operations of
proposed in the bid and incorporated in the contract provinces, cities, municipalities, and barangays.
which shall in no case exceed fifty (50) years: Provided,
finally, That during the lifetime of the contract, the Sec305. Fundamental Principles. — The financial affairs,
contractor shall undertake the necessary maintenance transactions, and operations of local government units shall be
and repair of the facility in accordance with standards governed by the following fundamental principles:
prescribed in the bidding documents and in the contract. (a) No money shall be paid out of the local treasury
In the case of a build-operate-and-transfer agreement, except in pursuance of an appropriations ordinance or law;
the repayment shall be made through amortization (b) Local government funds and monies shall be spent
payments in accordance with the schedule proposed in solely for public purposes;
the bid and incorporated in the contract. (c) Local revenue is generated only from sources
In case of land reclamation or construction of industrial expressly authorized by law or ordinance, and collection thereof
estates, the repayment plan may consist of the grant of a shall at all times be acknowledged properly;
portion or percentage of the reclaimed land or the (d) All monies officially received by a local government
industrial estate constructed. officer in any capacity or on any occasion shall be accounted for
(5) Every infrastructure project undertaken under this as local funds, unless otherwise provided by law;
Section shall be constructed, operated, and maintained (e) Trust funds in the local treasury shall not be paid out
by the contractor under the technical supervision of the except in fulfillment of the purpose for which the trust was
local government unit and in accordance with the plans, created or the funds received;
specifications, standards, and costs approved by it. (f) Every officer of the local government unit whose
(d) The provincial, city, or municipal legal officer shall, as duties permit or require the possession or custody of local funds
the case may be, review the contracts executed pursuant to this shall be properly bonded, and such officer shall be accountable
Section to determine their legality, validity, enforceability and and responsible for said funds and for the safekeeping thereof in
correctness of form. conformity with the provisions of law;
(g) Local governments shall formulate sound financial
Sec303. Remedies and Sanctions. — Local government unit plans, and local budgets shall be based on functions, activities,
shall appropriate in their respective annual budgets such and projects, in terms of expected results;
amounts as are sufficient to pay the loans and other (h) Local budget plans and goals shall, as far as
indebtedness incurred or redeem or retire bonds, debentures, practicable, be harmonized with national development plans,
securities, notes and other obligations issued under this Title: goals, and strategies in order to optimize the utilization of
resources and to avoid duplication in the use of fiscal and
physical resources;

The Apocryphal Maggots:


Rainier, Chrisgel, Corina, Geoffry, Grace and Sylvie Blanche
The Flibbertigibbet Worms:
Golda, Gladys and Melyjane
CA VE AT: By simply r ea ding this re vi ew er a t the end o f the sem este r wi ll (hopef ully) guar antee you r pas sing this cou rse. D rink mode rat ely .
Amusin S C A r y
gly
THE C2005 LOCAL GOVERNMENT REVIEWER - 255 -
(i) Local budgets shall operationalize approved local (k) "Personal Services" refers to appropriations for the
development plans; payment of salaries, wages and other compensation of
(j) Local government units shall ensure that their permanent, temporary, contractual, and casual employees of the
respective budgets incorporate the requirements of their local government unit;
component units and provide for equitable allocation of (l) "Receipts" refers to income realized from operations
resources among these component units; and activities of the local government or are received by it in the
(k) National planning shall be based on local planning to exercise of its corporate functions, consisting of charges for
ensure that the needs and aspirations of the people as services rendered, conveniences furnished, or the price of a
articulated by the local government units in their respective local commodity sold, as well as loans, contributions or aids from
development plans are considered in the formulation of budgets other entities, except provisional advances for budgetary
of national line agencies or offices; purposes; and
(l) Fiscal responsibility shall be shared by all those (m) "Revenue" refers to income derived from the regular
exercising authority over the financial affairs, transactions, and system of taxation enforced under authority of law or ordinance,
operations of the local government units; and and, as such, accrue more or less regularly every year.
(m) The local government unit shall endeavor to have a
balanced budget in each fiscal year of operation. CHAPTER II: Local and Other Special Funds
Art I: Receipts, Safekeeping Article and Disposition of Local
Sec306. Definitions. — When used in this Title, the term — Funds
(a) "Annual Budget" refers to a financial plan embodying Sec307. Remittance of Government Monies to the Local
the estimates of income and expenditures for one (1) fiscal year; Treasury. — Officers of local government authorized to receive
(b) "Appropriation" refers to an authorization made by and collect monies arising from taxes, revenues, or receipts of
ordinance, directing the payment of goods and services from any kind shall remit the full amount received and collected to the
local government funds under specified conditions or for specific treasury of such local government unit which shall be credited to
purposes; the particular account or accounts to which the monies in
(c) "Budget Document" refers to the instrument used by question properly belong.
the local chief executive to present a comprehensive financial
plan to the sanggunian concerned; Sec308. Local Funds. — Every local government unit shall
(d) "Capital Outlays" refers to appropriations for the maintain a General Fund which shall be used to account for
purchase of goods and services, the benefits of which extend such monies and resources as may be received by and
beyond the fiscal year and which add to the assets of the local disbursed from the local treasury. The General Fund shall
government unit concerned, including investments in public consist of monies and resources of the local government which
utilities such as public markets and slaughterhouses; are available for the payment of expenditures, obligations or
(e) "Continuing Appropriation" refers to an appropriation purposes not specifically declared by law as accruing and
available to support obligations for a specified purpose or chargeable to, or payable from, any other fund.
projects, such as those for the construction of physical
structures or for the acquisition of real property or equipment, Sec309. Special Funds. — There shall be maintained in every
even when these obligations are incurred beyond the budget provincial, city, or municipal treasury the following special funds:
year; (a) Special Education Fund (SEF) shall consist of the
(f) "Current Operating Expenditures" refers to respective shares of provinces, cities, municipalities and
appropriations for the purchase of goods and services for the barangays in the proceeds of the additional tax on real property
conduct of normal local government operations within the fiscal to be appropriated for purposes prescribed in Section 272 of this
year, including goods and services that will be used or Code; and
consumed during the budget year; (b) Trust Funds shall consist of private and public monies
(g) "Expected Results" refers to the services, products, or which have officially come into the possession of the local
benefits that shall accrue to the public, estimated in terms of government or of a local government official as trustee, agent or
performance measures or physical targets; administrator, or which have been received as a guaranty for the
(h) "Fund" refers to a sum of money, or other assets fulfillment of some obligation. A trust fund shall only be used for
convertible to cash, set aside for the purpose of carrying out the specific purpose for which it was created or for which it
specific activities or attaining certain objectives in accordance came into the possession of the local government unit.
with special regulations, restrictions, or limitations, and
constitutes as independent fiscal and accounting entity; Sec310. Separation of Books and Depository Accounts. —
(i) "Income" refers to all revenues and receipts collected Local accountants and treasurers shall maintain separate books
or received forming the gross accretions of funds of the local and depository accounts, respectively, for each fund in their
government unit; custody or administration under such rules and regulations as
(j) "Obligations" refers to an amount committed to be the Commission on Audit may prescribe.
paid by the local government unit for any lawful act made by an
accountable officer for and in behalf of the local unit concerned; Sec311. Depository Accounts. — Local treasurers shall maintain
depository accounts in the name of their respective local

The Apocryphal Maggots:


Rainier, Chrisgel, Corina, Geoffry, Grace and Sylvie Blanche
The Flibbertigibbet Worms:
Golda, Gladys and Melyjane
CA VE AT: By simply r ea ding this re vi ew er a t the end o f the sem este r wi ll (hopef ully) guar antee you r pas sing this cou rse. D rink mode rat ely .
Amusin S C A r y
gly
THE C2005 LOCAL GOVERNMENT REVIEWER - 256 -
government units with banks, preferably government-owned, (iii) The estimates of income for the ensuing fiscal
located in or nearest to their respective areas of jurisdiction. year from ordinances and laws existing at the time the
Earnings of each depository account shall accrue exclusively proposed budget is transmitted, together with other
thereto. proposals;
(iv) The estimated expenditures necessary to
Sec312. Separation of Personal Money from Public Funds. — carry out the functions, projects, and activities of the
Local treasurers and other accountable officers shall keep local government unit for the ensuing fiscal year;
monies separate and distinct from local public funds in their (v) All essential facts regarding the bonded and
custody and shall not make profit out of public money or other long-term obligations and indebtedness of the local
otherwise apply the same to any use not authorized by law or government unit, if any;
ordinance. (vi) Summary statement of all statutory and
contractual obligations due; and
Art II: Special Accounts (vii) Such other financial statements and data as
Sec313. Special Accounts to be Maintained in the General are deemed necessary or desirable in order to disclose
Fund. — Local government units shall maintain special in all practicable detail the financial condition of the local
accounts in the general fund for the following: government unit.
(a) Public utilities and other economic enterprises;
(b) Loans, interests, bond issues, and other contributions Sec315. Submission of Detailed Statements of Income and
for specific purposes; and Expenditures. — (a) On or before the fifteenth (15th) day of
(c) Development projects funded from the share of the July of each year, local treasurers shall submit to their
local government unit concerned in the internal revenue respective local chief executives a certified statement, covering
allotment and such other special accounts which may be the income and expenditures of the preceding fiscal year, the
created by law or ordinance. actual income and expenditures of the first two (2) quarters of
Receipts, transfers, and expenditures involving the foregoing the current year, and the estimated income and expenditures for
special accounts shall be properly taken up thereunder. the last two (2) quarters of the current year.
Profits or income derived the operation of public utilities and
other economic enterprises, after deduction for the cost of Sec316. Local Finance Committee. — There is hereby created in
improvement, repair and other related expenses of the public every province, city or municipality a local finance committee to
utility or economic enterprise concerned, shall first be applied for be composed of the local planning and development officer, the
the return of the advances or loans made therefor. Any excess local budget officer, and the local treasurer. It shall exercise the
shall form part of the general fund of the local government unit following functions:
concerned. (a) Determine the income reasonably projected as
collectible for the ensuing fiscal year;
CHAPTER III: Budgeting (b) Recommend the appropriate tax and other revenue
Art I: Local Government Budgets measures or borrowings which may be appropriate to support
Sec314. Form and Content. — the budget;
(a) Local government budgets shall primarily consists of (c) Recommend to the local chief executive concerned
two (2) parts: the level of the annual expenditures and the ceilings of spending
(1) The estimates of income; and for economic, social, and general services based on the
(2) The total appropriations covering the current operating approved local development plans;
expenditures and capital outlays. (d) Recommend to the local chief executive concerned
(b) The budget document shall contain: the proper allocation of expenditures for each development
(1) A budget message of the local chief executive setting activity between current operating expenditures and capital
forth in brief the significance of the executive budget, outlays;
particularly in relation to the approved local development (e) Recommend to the local chief executive concerned
plan; the amount to be allocated for capital outlay under each
(2) A brief summary of the functions, projects, and activities development activity or infrastructure project;
to be accomplished in pursuit of the goals and objectives (f) Assist the sangguniang panlalawigan in the review
of the local government unit for the ensuing fiscal year, and evaluation of budget of component cities and municipalities
specifically the delivery of basic services or facilities in the case of provincial finance committee, the barangay
enumerated under Section 17 of this Code; budgets in the case of city or municipal finance committee, and
(3) Summary of financial statements setting forth: recommend the appropriate action thereon;
(i) The actual income and expenditures during (g) Assist the sanggunian concerned in the analysis and
the immediately preceding year; review of annual regular and supplemental budgets of the
(ii) The actual income and expenditures of the respective local government unit to determine compliance with
first two (2) quarters and the estimates of income and statutory and administrative requirements; and
expenditures for the last two (2) quarters of the current
fiscal year;

The Apocryphal Maggots:


Rainier, Chrisgel, Corina, Geoffry, Grace and Sylvie Blanche
The Flibbertigibbet Worms:
Golda, Gladys and Melyjane
CA VE AT: By simply r ea ding this re vi ew er a t the end o f the sem este r wi ll (hopef ully) guar antee you r pas sing this cou rse. D rink mode rat ely .
Amusin S C A r y
gly
THE C2005 LOCAL GOVERNMENT REVIEWER - 257 -
(h) Conduct semi-annual review and general examination The local chief executive shall submit the said executive budget
of cost and accomplishments against performance standards to the sanggunian concerned not later than the sixteenth (16th)
applied in undertaking development projects. of October of the current fiscal year. Failure to submit such
A copy of this report shall be furnished the local chief executive budget on the date prescribed herein shall subject the local chief
and the sanggunian concerned, and shall be posted in executive to such criminal and administrative penalties as
conspicuous and publicly accessible places in the provinces, provided for under this Code and other applicable laws.
cities, municipalities and barangays.
Sec319. Legislative Authorization of the Budget. — On or
Sec317. Submission of Budget Proposals by Heads or before the end of the current fiscal year, the sanggunian
Departments or Offices. — concerned shall, through an ordinance, the annual budget of the
(a) Each head of department or office shall submit a local government unit for the ensuing fiscal year on the basis of
budget proposal for his department or office to the local chief the estimates of income and expenditures submitted by the local
executive on or before the fifteenth (15th) of July of each year: chief executive.
Provided, That the budget proposal of each department of office
shall be categorized under either economic, social or general Sec320. Effectivity of Budgets. — The ordinance enacting the
services: Provided, further, That each service shall be covered annual budget shall take effect at the beginning of the ensuing
by the budget of at least one (1) department or office of the local calendar year. An ordinance enacting a supplemental budget,
government unit concerned. however, shall take effect upon its approval or on the date fixed
The said budget proposal shall be prepared in accordance with therein.
such policy and program guidelines as the local chief executive The responsibility for the execution of the annual and
concerned may issue in conformity with the local development supplemental budgets and the accountability therefor shall be
plan, the budgetary ceilings prescribed by the local finance vested primarily in the local chief executive concerned.
committee, and the general requirements prescribed in this Title.
(b) Budget proposals of departments or offices shall be Sec321. Changes in the Annual Budget. — All budgetary
divided into two (2) primary categories, namely: the current proposals shall be included and considered in the budget
operating expenditures and the capital outlays. Such budget preparation process. After the local chief executive concerned
proposals shall contain the following information: shall have submitted the executive budget to the sanggunian, no
(1) Objectives, functions, and projects showing the general ordinance providing for a supplemental budget shall be enacted,
character and relative importance of the work to be except when supported by funds actually available as certified
accomplished or the services to be rendered, and the by the local treasurer or by new revenue sources.
cost thereof; A supplemental budget may also be enacted in times of public
(2) Organizational charts and staffing patterns indicating the calamity by way of budgetary realignment to set aside
list of plantilla positions with their corresponding salaries, appropriations for the purchase of supplies and materials or the
and proposals for reclassification of positions and salary payment of services which are exceptionally urgent or absolutely
changes, as well as the creation of new positions with indispensable to prevent imminent danger to, or loss of, life or
their proposed salary grade, duly supported by proper property, in the jurisdiction of the local government unit or in
justification; other areas declared by the President in a state of calamity.
(3) Brief description of the functions, projects and activities Such ordinance shall clearly indicate the sources of funds
for the ensuing fiscal year, expected results for each available for appropriations, as certified under oath by the local
function, project and activity, and the nature of work to treasurer and local accountant and attested by the local chief
be performed, including the objects of expenditures for executive, and the various items of appropriations affected and
each function, project and activity; the reasons for the change.
(4) Relation of the work and financial proposals to approved
local development plans; Sec322. Reversion of Unexpended Balances of
(5) Estimated current operating expenditures and capital Appropriations, Continuing Appropriations. — Unexpended
outlays with comparative data for the last two (2) balances of appropriations authorized in the annual
preceding, current, and ensuing fiscal years; and appropriations ordinance shall revert to the unappropriated
(6) Accomplishment reports for the last two (2) preceding surplus of the general fund at the end of the fiscal year and shall
and current fiscal years. not thereafter be available for the expenditure except by
subsequent enactment. However, appropriations for capital
Sec318. Preparation of the Budget by the Local Chief outlays shall continue and remain valid until fully spent, reverted
Executive. — Upon receipt of the statements of income and or the project is completed. Reversions of continuing
expenditures from the treasurer, the budget proposals of the appropriations shall not be allowed unless obligations therefor
heads of departments and offices, and the estimates of income have been fully paid or otherwise settled.
and budgetary ceilings from the local finance committee, the The balances of continuing appropriations shall be reviewed as
local chief executive shall prepare the executive budget for the part of the annual budget preparation and the sanggunian
ensuing fiscal year in accordance with the provisions of this concerned may approve, upon recommendation of the local
Title. chief executive, the reversion of funds no longer needed in

The Apocryphal Maggots:


Rainier, Chrisgel, Corina, Geoffry, Grace and Sylvie Blanche
The Flibbertigibbet Worms:
Golda, Gladys and Melyjane
CA VE AT: By simply r ea ding this re vi ew er a t the end o f the sem este r wi ll (hopef ully) guar antee you r pas sing this cou rse. D rink mode rat ely .
Amusin S C A r y
gly
THE C2005 LOCAL GOVERNMENT REVIEWER - 258 -
connection with the activities funded by said continuing (a) The total appropriations, whether annual or
appropriations subject to the provisions of this Section. supplemental, for personal services of a local government unit
for one (1) fiscal year shall not exceed forty-five percent (45%)
Sec323. Failure to Enact the Annual Appropriations. — In case in the case of first to third class provinces, cities and
the sanggunian concerned fails to pass the ordinance municipalities, and fifty-five percent (55%) in the case of fourth
authorizing the annual appropriations at the beginning of the class or lower, of the total annual income from regular sources
ensuing fiscal year, it shall continue to hold sessions, without realized in the next preceding fiscal year. The appropriations for
additional remuneration for its members, until such ordinance is salaries, wages, representation and transportation allowances of
approved, and no other business may be taken up during such officials and employees of the public utilities and economic
sessions. If the sanggunian still fails to enact such ordinance enterprises owned, operated, and maintained by the local
after ninety (90) days from the beginning of the fiscal year, the government unit concerned shall not be included in the annual
ordinance authorizing the appropriations of the preceding year budget or in the computation of the maximum amount for
shall be deemed reenacted and shall remain in force and effect personal services. The appropriations for the personal services
until the ordinance authorizing the proposed appropriations is of such economic enterprises shall be charged to their
passed by the sanggunian concerned. However, only the annual respective budgets;
appropriations for salaries and wages of existing positions, (b) No official or employee shall be entitled to a salary
statutory and contractual obligations, and essential operating rate higher than the maximum fixed for his position or other
expenses authorized in the annual and supplemental budgets positions of equivalent rank by applicable laws or rules and
for the preceding year shall be deemed reenacted and regulations issued thereunder;
disbursement of funds shall be in accordance therewith. (c) No local fund shall be appropriated to increase or
In the implementation of such reenacted ordinance, the local adjust salaries or wages of officials and employees of the
treasurer concerned shall exclude from the estimates of income national government, except as may be expressly authorized by
for the preceding fiscal year those realized from nonrecurring law;
sources, like national aids, proceeds from loans, sale of assets, (d) In cases of abolition of positions and the creation of
prior year adjustments, and other analogous sources of income. new ones resulting from the abolition of existing positions in the
No ordinance authorizing supplemental appropriations shall be career service, such abolition or creation shall be made in
passed in place of the annual appropriations. accordance with pertinent provisions of this code and the civil
In case the revised income estimates be less than the service law, rules and regulations;
aggregate reenacted appropriations, the local treasurer (e) Positions in the official plantilla for career positions
concerned shall accordingly advise the sanggunian concerned which are occupied by incumbents holding permanent
which shall, within ten (10) days from the receipt of such advice, appointments shall be covered by adequate appropriations;
make the necessary adjustments or reductions. The revised (f) No changes in designation or nomenclature of
appropriations authorized by the sanggunian concerned shall positions resulting in a promotion or demotion in rank or
then be the basis for disbursements. increase or decrease in compensation shall be allowed, except
when the position is actually vacant, and the filling of such
Sec324. Budgetary Requirements. — The budgets of local positions shall be strictly made in accordance with the civil
government units for any fiscal year shall comply with the service law, rules and regulations;
following requirements: (g) The creation of new positions and salary increases or
(a) The aggregate amount appropriated shall not exceed adjustments shall in no case be made retroactive; and
the estimates of income; (h) The annual appropriations for discretionary purposes
(b) Full provision shall be made for all statutory and of the local chief executive shall not exceed two percent (2%) of
contractual obligations of the local government unit concerned: the actual receipts derived from basic real property tax in the
Provided, however, That the amount of appropriations for debt next preceding calendar year. Discretionary funds shall be
servicing shall not exceed twenty percent (20%) of the regular disbursed only for public purposes to be supported by
income of the local government unit concerned; appropriate vouchers and subject to such guidelines as may be
(c) In the case of provinces, cities, and municipalities, aid prescribed by law. No amount shall be appropriated for the
to component barangays shall be provided in amounts of not same purpose except as authorized under this Section.
less than One thousand pesos (P1,000.00) per barangay; and
(d) Five percent (5%) of the estimated revenue from Sec326. Review of Appropriation Ordinances of Provinces,
regular sources shall be set aside as an annual lump sum Highly-Urbanized Cities, Independent Component Cities,
appropriation for unforeseen expenditures arising from the and Municipalities within the Metropolitan Manila Area. —
occurrence of calamities: Provided, however, That such The Department of Budget and Management shall review
appropriation shall be used only in the area, or a portion thereof, ordinances authorizing the annual or supplemental
of the local government unit or other areas declared by the appropriations of provinces, highly-urbanized cities, independent
President in a state of calamity. component cities, and municipalities within the Metropolitan
Manila Area in accordance with the immediately succeeding
Sec325. General Limitations. — The use of the provincial, city, Section.
and municipal funds shall be subject to the following limitations:

The Apocryphal Maggots:


Rainier, Chrisgel, Corina, Geoffry, Grace and Sylvie Blanche
The Flibbertigibbet Worms:
Golda, Gladys and Melyjane
CA VE AT: By simply r ea ding this re vi ew er a t the end o f the sem este r wi ll (hopef ully) guar antee you r pas sing this cou rse. D rink mode rat ely .
Amusin S C A r y
gly
THE C2005 LOCAL GOVERNMENT REVIEWER - 259 -
Sec327. Review of Appropriation Ordinances of Component shall prepare the barangay budget for the ensuing fiscal year in
Cities and Municipalities. — The sangguniang panlalawigan the manner and within the period prescribed in this Title and
shall review the ordinance authorizing annual or supplemental submit the annual barangay budget to the sangguniang
appropriations of component cities and municipalities in the barangay for legislative enactment.
same manner and within the same period prescribed for the (b) The total annual appropriations for personal services
review of other ordinances. of a barangay for one (1) fiscal year shall not exceed fifty-five
If within ninety (90) days from receipt of copies of such percent (55%) of the total annual income actually realized from
ordinance, the sangguniang panlalawigan takes no action local sources during the next preceding fiscal year.
thereon, the same shall be deemed to have been reviewed in (c) The barangay budget shall likewise be subject to the
accordance with law and shall continue to be in full force and same budgetary requirements and limitations hereinabove
effect. If within the same period, the sangguniang panlalawigan prescribed.
shall have ascertained that the ordinance authorizing annual or
supplemental appropriations has not complied with the Sec332. Effectivity of Barangay Budgets. — The ordinance
requirements set forth in this Title, the sangguniang enacting the annual budget shall take effect at the beginning of
panlalawigan shall, within the ninety-day period hereinabove the ensuing calendar year. An ordinance enacting a
prescribed declare such ordinance inoperative in its entirety or supplemental budget, however, shall take effect upon its
in part. Items of appropriation contrary to limitations prescribed approval or on the date fixed therein.
in this Title or in excess of the amounts prescribed herein shall The responsibility for the execution of the annual and
be disallowed or reduced accordingly. supplemental budgets and the accountability therefor shall be
The sangguniang panlalawigan shall within the same period vested primarily in the punong barangay concerned.
advise the sangguniang panlungsod or sangguniang bayan
concerned through the local chief executive of any action on the Sec333. Review of the Barangay Budget. —
ordinance under review. Upon receipt of such advice, the city or (a) Within ten (10) days from its approval, copies of the
municipal treasurer concerned shall not make further barangay ordinance authorizing the annual appropriations shall
disbursements of funds from any of the items of appropriation be furnished the sangguniang panlungsod or the sangguniang
declared inoperative, disallowed or reduced. bayan, as the case may be, through the city or municipal budget
officer. The sanggunian concerned shall have the power to
Sec328. Duration of Appropriation. — Appropriations for review such ordinance in order to ensure that the provisions of
ordinary administrative purposes not duly obligated shall this Title are complied with. If within sixty (60) days after the
terminate with the fiscal year and all unexpended balances receipt of the ordinance, the sanggunian concerned takes no
thereof shall be automatically reverted on the thirty-first (31st) action thereon, the same shall continue to be in full force and
day of December of each year to the general fund of the local effect. If within the same period, the sanggunian concerned shall
government unit. have ascertained that the ordinance contains appropriations in
excess of the estimates of the income duly certified as
ARTICLE II: Barangay Budgets collectible, or that the same has not complied with the budgetary
Sec329. Barangay Funds. — Unless otherwise provided in this requirements set forth in this Title, the said ordinance shall be
Title, all the income of the barangay from whatever source shall declared inoperative in its entirety or in part. Items of
accrue to its general fund and shall, at the option of the appropriation contrary to, or in excess of, any of the general
barangay concerned, be kept as trust fund in the custody of the limitations or the maximum amount prescribed in this Title shall
city or municipal treasurer or be deposited in a bank, preferably be disallowed or reduced accordingly.
government-owned, situated in or nearest to its area of (b) Within the period hereinabove fixed, the sangguniang
jurisdiction. Such funds shall be disbursed in accordance with panlungsod or sangguniang bayan concerned shall return the
the provisions of this Title. Ten percent (10%) of the general fund barangay ordinance, through the city or municipal budget officer,
of the barangay shall be set aside for the sangguniang to the punong barangay with the advice of action thereon for
kabataan. proper adjustments, in which event, the barangay shall operate
on the ordinance authorizing annual appropriations of the
Sec330. Submission of Detailed Statements of Income and preceding fiscal year until such time that the new ordinance
Expenditures for the Barangay Budgets. — On or before the authorizing annual appropriations shall have met the objections
fifteenth (15th) day of September of each year, the barangay raised. Upon receipt of such advice, the barangay treasurer or
treasurer shall submit to the punong barangay a statement the city or municipal treasurer who has custody of the funds
covering the estimates of income and expenditures for the shall not make further disbursement from any item of
ensuing fiscal year, based on a certified statement issued by the appropriation declared inoperative, disallowed, or reduced.
city or municipal treasurer covering the estimates of income
from local sources for the barangay concerned. Sec334. Barangay Financial Procedures. —
(a) The barangay treasurer shall collect all taxes, fees,
Sec331. Preparation of the Barangay Budget. — and other charges due and contributions accruing to the
(a) Upon receipt of the statement of income and barangay for which he shall issue official receipts, and shall
expenditures from the barangay treasurer, the punong barangay deposit all collections with the city or municipal treasury or in the

The Apocryphal Maggots:


Rainier, Chrisgel, Corina, Geoffry, Grace and Sylvie Blanche
The Flibbertigibbet Worms:
Golda, Gladys and Melyjane
CA VE AT: By simply r ea ding this re vi ew er a t the end o f the sem este r wi ll (hopef ully) guar antee you r pas sing this cou rse. D rink mode rat ely .
Amusin S C A r y
gly
THE C2005 LOCAL GOVERNMENT REVIEWER - 260 -
depository account maintained in the name of the barangay with the first collections of the immediately succeeding fiscal
within five (5) days after receipt thereof. He may collect real year accruing to such local fund.
property taxes and such other taxes as may be imposed by a
province, city or municipality that are due in his barangay only Sec338. Prohibitions Against Advance Payments. — No money
after being deputized by the local treasurer concerned for the shall be paid on account of any contract under which no
purpose. services have been rendered or goods delivered.
(b) The barangay treasurer may be authorized by the
sangguniang barangay to make direct purchases amounting to Sec339. Cash Advances. — No cash advance shall be granted to
not more than One thousand pesos (P1,000.00) at any time for any local official or employee, elective or appointive, unless
the ordinary and essential needs of the barangay. The petty made in accordance with the rules and regulations as the
cash that the barangay treasurer may be authorized to hold for Commission on Audit may prescribe.
the purpose shall not exceed twenty percent (20%) of the funds
available and to the credit of the barangay treasury. Sec340. Persons Accountable for Local Government Funds. —
(c) The financial records of the barangay shall be kept in Any officer of the local government unit whose duty permits or
the office of the city or municipal accountant in simplified requires the possession or custody of local government funds
manner as prescribed by the Commission on Audit. shall be accountable and responsible for the safekeeping
Representatives of the Commission on Audit shall audit such thereof in conformity with the provisions of this Title. Other local
accounts annually or as often as may be necessary and make a officers who, though not accountable by the nature of their
report of the audit to the sangguniang panlungsod or duties, may likewise be similarly held accountable and
sangguniang bayan, as the case may be. The Commission on responsible for local government funds through their
Audit shall prescribe and put into effect simplified procedures for participation in the use or application thereof.
barangay finances within six (6) months following the effectivity
of this Code. Sec341. Prohibitions Against Pecuniary Interest. — Without
prejudice to criminal prosecution under applicable laws, any
CHAPTER IV: Expenditures, Disbursements, Accounting and local treasurer, accountant, budget officer, or other accountable
Accountability local officer having any pecuniary interest, direct or indirect, in
Sec335. Prohibitions Against Expenditures for Religious or any contract, work or other business of the local government
Private Purposes. — No public money or property shall be unit of which he is an accountable officer shall be
appropriated or applied for religious or private purposes. administratively liable therefor.

Sec336. Use of Appropriated Funds and Savings. — Funds Sec342. Liability for Acts Done Upon Direction of Superior
shall be available exclusively for the specific purpose for which Officer, or Upon Participation of Other Department Heads or
they have been appropriated. No ordinance shall be passed Officers of Equivalent Rank. — Unless he registers his
authorizing any transfer of appropriations from one item to objection in writing, the local treasurer, accountant, budget
another. However, the local chief executive or the presiding officer, or other accountable officer shall not be relieved of
officer of the sanggunian concerned may, by ordinance, be liability for illegal or improper use or application or deposit of
authorized to augment any item in the approved annual budget government funds or property by reason of his having acted
for their respective offices from savings in other items within the upon the direction of a superior officer, elective or appointive, or
same expense class of their respective appropriations. upon participation of other department heads or officers of
equivalent rank. The superior officer directing, or the department
Sec337. Restriction Upon Limit of Disbursements. — head participating in such illegal or improper use or application
Disbursements in accordance with appropriations in the or deposit of government funds or property, shall be jointly and
approved annual budget may be made from any local fund in severally liable with the local treasurer, accountant, budget
the custody of the treasurer, but the total disbursements from officer, or other accountable officer for the sum or property so
any local fund shall in no case exceed fifty percent (50%) of the illegally or improperly used, applied or deposited.
uncollected estimated revenue accruing to such local fund in
addition to the actual collections: Provided, however, That no Sec343. Prohibition Against Expenses for Reception and
cash overdraft in any local fund shall be incurred at the end of Entertainment. — No money shall be appropriated, used, or
the fiscal year. paid for entertainment or reception except to the extent of the
In case of emergency arising from a typhoon, earthquake, or representation allowances authorized by law or for the reception
any other calamity, the sanggunian concerned may authorize of visiting dignitaries of foreign governments or foreign missions,
the local treasurer to continue making disbursements from any or when expressly authorized by the President in specific cases.
local fund in his possession in excess of the limitations herein
provided, but only for such purposes and amounts included in Sec344. Certification, and Approval of, Vouchers. — No money
the approved annual budgets. shall be disbursed unless the local budget officer certifies to the
Any overdraft which may be incurred at the end of the year in existence of appropriation that has been legally made for the
any local fund by virtue of the provisions hereof shall be covered purpose, the local accountant has obligated said appropriation,
and the local treasurer certifies to the availability of funds for the

The Apocryphal Maggots:


Rainier, Chrisgel, Corina, Geoffry, Grace and Sylvie Blanche
The Flibbertigibbet Worms:
Golda, Gladys and Melyjane
CA VE AT: By simply r ea ding this re vi ew er a t the end o f the sem este r wi ll (hopef ully) guar antee you r pas sing this cou rse. D rink mode rat ely .
Amusin S C A r y
gly
THE C2005 LOCAL GOVERNMENT REVIEWER - 261 -
purpose. Vouchers and payrolls shall be certified to and officer next-in-rank in the local treasury service, unless the said
approved by the head of the department or office who has officer is likewise under investigation, the office of the treasurer
administrative control of the fund concerned, as to validity, and its contents, and close and render his accounts on the date
propriety, and legality of the claim involved. Except in cases of of turnover. In case the accountable officer next in rank is under
disbursements involving regularly recurring administrative investigation, the auditor shall take full possession of the office
expenses such as payrolls for regular or permanent employees, and its contents, close and render his accounts on the date of
expenses for light, water, telephone and telegraph services, taking possession, and temporarily continue the public business
remittances to government creditor agencies such as GSIS, of such office until such time that the local treasurer is restored
SSS, LDP, DBP, National Printing Office, Procurement Service or a successor has been duly designated. The local treasurer or
of the DBM and others, approval of the disbursement voucher accountable officer found with such shortage shall be
by the local chief executive himself shall be required whenever automatically suspended from office.
local funds are disbursed.
In cases of special or trust funds, disbursements shall be Sec349. Accounting for Revenues. — Estimated revenues which
approved by the administrator of the fund. remain unrealized at the close of the fiscal year shall not be
In case of temporary absence or incapacity of the department booked or credited to the unappropriated surplus or any other
head or chief of office, the officer next-in-rank shall automatically account.
perform his function and he shall be fully responsible therefor.
Sec350. Accounting for Obligations. — All lawful expenditures
Sec345. Officials Authorized to Draw Checks in Settlement of and obligations incurred during a fiscal year shall be taken up in
Obligations. — Checks in obligations shall be drawn by the the accounts of that year.
local treasurer and countersigned by the local administrator.
In case of temporary absence or incapacity of the foregoing Sec351. General Liability for Unlawful Expenditures. —
officials, these duties shall devolve upon their immediate Expenditures of funds or use of property in violation of this Title
assistants. and other laws shall be a personal liability of the official or
employee responsible therefor.
Sec346. Disbursements of Local Funds and Statement of
Accounts. — Disbursements shall be made in accordance with Sec352. Posting of the Summary of Income and Expenditures.
the ordinance authorizing the annual or supplemental — Local treasurers, accountants, budget officers, and other
appropriations without the prior approval of the sanggunian accountable officers shall, within thirty (30) days from the end of
concerned. Within thirty (30) days after the close of each month, the fiscal year, post in at least three (3) publicly accessible and
the local accountant shall furnish the sanggunian with such conspicuous places in the local government unit a summary of
financial statements as may be prescribed by the Commission all revenues collected and funds received including the
on Audit. In the case of the year-end statement of accounts, the appropriations and disbursements of such funds during the
period shall be sixty (60) days after the thirty-first (31st) of preceding fiscal year.
December.
Sec353. The Official Fiscal Year. — The official fiscal year of
Sec347. Rendition of Accounts. — Local treasurers, accountants local government units shall be the period beginning with the
and other local accountable officers shall render their accounts first day of January and ending with the thirty-first day of
within such time, in such form, style, and content and under December of the same year.
such regulations as the Commission on Audit may prescribe.
Provincial, city, and municipal auditors shall certify the balances Sec354. Administrative Issuances; Budget Operations Manual.
arising in the accounts settled by them to the Chairman of the — The Secretary of Budget and Management jointly with the
Commission on Audit and to the local treasurer, accountant, and Chairman of the Commission on Audit shall, within one (1) year
other accountable officers. Copies of the certification shall be from the effectivity of this Code, promulgate a Budget
prepared and furnished other local officers who may be held Operations Manual for local government units to improve and
jointly and severally liable for any loss or illegal, improper or systematize methods, techniques, and procedures employed in
unauthorized use or misappropriation of local funds or property. budget preparation, authorization, execution, and accountability.

Sec348. Auditorial Visitation. — The books, accounts, papers,


and cash of local treasurer, accountant, budget officer, or other Malonzo v. Zamora (1999)
accountable officers shall at all times be open for inspection of
the Commission on Audit or its duly authorized representative. The Sangguniang Panlungsod passed Ordinance 246 which
In case an examination of the accounts of a local treasurer increased the appropriated amount to be used for the expropriation
discloses a shortage in cash which should be on hand, it shall of the Maysilo Estate. The city govt. failed to conclude a voluntary
be the duty of the examining officer to seize the office and its sale. A suit for eminent domain was filed against CLT.
contents, notify the Commission on Audit, the local chief
executive concerned, and the local accountant. Thereupon, the Caloocan Legal Officer informed Mayor through a letter-
examining officer shall immediately turn over to the accountable memorandum of a pending interpleader case involving the land.

The Apocryphal Maggots:


Rainier, Chrisgel, Corina, Geoffry, Grace and Sylvie Blanche
The Flibbertigibbet Worms:
Golda, Gladys and Melyjane
CA VE AT: By simply r ea ding this re vi ew er a t the end o f the sem este r wi ll (hopef ully) guar antee you r pas sing this cou rse. D rink mode rat ely .
Amusin S C A r y
gly
THE C2005 LOCAL GOVERNMENT REVIEWER - 262 -
He recommended that pending the final determination and "In case of fire or conflagration, the calamity fund shall be
resolution of the same, the expropriation of the land be cancelled utilized only for relief operations.
and/or abandoned. "The local development council shall more monitor the use and
disbursement of the calamity fund."
Vice Mayor wrote Mayor requesting the immediate repair and
renovation of the offices of the incoming councilors and the hiring
of add’l personnel Property and Supply Management in the Local Government
Units
Mayor endorsed letter to the City Treasurer who manifested
through a memorandum that since the expropriation of the land is Sec355. Scope. — This Title shall govern the procurement, care,
discontinued, the appropriation for expropriation of 50 M can be utilization, custody, and disposal of supplies, as defined herein,
reverted for use in a supplemental budget. by local government units and the other aspects of supply
management at the local levels.
Mayor endorsed Supplemental Budget to Sanggunian which then
passed Ordinance 254. An admin complaint was filed against city Sec356. General Rule in Procurement or Disposal. — Except
officials before OP. as otherwise provided herein, acquisition of supplies by local
government units shall be through competitive public bidding.
Held: The petitioners are not guilty of misconduct due to the Supplies which have become unserviceable or no longer
alleged irregularity of Ordinance 254. OP’s reliance on SEC 322 needed shall be sold, whenever applicable, at public auction,
LGC is misplaced as the CAB is not a question of WON the subject to applicable rules and regulations.
appropriations fall under the def. of capital outlay or continuing
appropriations. Issue is WON pets. are liable in regard to the Sec357. Definition of Terms. — When used in this Title, the term
ordinance which realigned part of the 50 M which was —
denominated in a general manner as “Expropriation Properties” (a) "Lowest Complying and Responsible Bid" refers to the
and classified as “Current Operating Expenditures” in the 1998 proposal of one who offers the lowest price, meets all the
Annual Budget of Caloocan. Clearly these two amounts are technical specifications and requirements of the supplies
distinct. desired and, as a dealer in the line of supplies involved,
maintains a regular establishment, and has complied
5O M was appropriated NOT for the purpose of purchasing lot 26 consistently with previous commitments;
but for expenses incidental to expropriation. Amount appropriated (b) "Suitable Substitute" refers to that kind of article which
under Ord 246 still subsists and is not lumped with other funds to would serve substantially the same purpose or produce
arrive at the 50 M allocated in the 1998 budget. substantially the same results as the brand, type, or make of
article originally desired or requisitioned;
The 50M can be realigned. Realignments are not allowed only if (c) "Supplies" includes everything, except real property,
cont. appropriations or capital outlays are involved. The 50 M was which may be needed in the transaction of public business or in
classified as a current operating expenditure by the local council. the pursuit of any undertaking, project, or activity, whether in the
It cannot be said then that Ord.0254 was enacted without funds nature of equipment, furniture, stationary materials for
actually available. construction or personal property of any sort, including non-
personal or contractual services such as the repair and
maintenance of equipment and furniture, as well as trucking,
hauling, janitorial, security, and related services; and
RA 8185 (1996) - An Act Amending Section 324 (D) Of LGC (d) "Terms and Conditions" refer to other requirements
not affecting the technical specifications and requirements of the
Sec1. Section 324(d) of Republic Act No. 7160 is hereby amended required supplies desired such as bonding, terms of delivery
to read as follows: and payment, and related preferences.
"(d) Five percent (5%) of the estimated revenue from
regular sources shall be set aside as annual lump sum Sec358. Requirement of Requisition. — Any order for supplies
appropriations for relief, rehabilitation, reconstruction and other shall be filled by the provincial or city general services officer or
works or services in connection with calamities which may occur the municipal or barangay treasurer concerned, as the case
during the budget year. Provided, however, That such fund shall may be, for any office or department of a local government unit
be used only in the area, or a portion thereof, of the local only upon written requisition as hereinafter provided.
government unit or other areas affected by a disaster or
calamity, as determined and declared by the local sanggunian Sec359. Officers Having Authority to Draw Requisitions. —
concerned. Requisitions shall be prepared by the head of office or
"Calamity shall be defined as a state of extreme distress or department needing the supplies, who shall certify as to their
misfortune, produced by some adverse circumstance or event or necessity for official use and specify the project or activity where
any great misfortune or cause or loss or misery caused by the supplies are to be used.
natural forces.

The Apocryphal Maggots:


Rainier, Chrisgel, Corina, Geoffry, Grace and Sylvie Blanche
The Flibbertigibbet Worms:
Golda, Gladys and Melyjane
CA VE AT: By simply r ea ding this re vi ew er a t the end o f the sem este r wi ll (hopef ully) guar antee you r pas sing this cou rse. D rink mode rat ely .
Amusin S C A r y
gly
THE C2005 LOCAL GOVERNMENT REVIEWER - 263 -
Sec360. Certification by the Local Budget Officer, Accountant, The Committee on Awards shall be composed of the local chief
and Treasurer. — Every requisition must be accompanied by a executive as chairman, the local treasurer, the local accountant,
certificate signed by the local budget officer, the local the local budget officer, the local general services officer, and
accountant, and the local treasurer showing that an the head of office or department for whose use the supplies are
appropriation therefor exists, the estimated amount of such being procured, as members. In case a head of office or
expenditure has been obligated, and the funds are available for department would sit in a dual capacity, a member of the
the purpose, respectively. sanggunian elected from among its members shall sit as a
member. The committee on awards at the barangay level shall
Sec361. Approval of Requisitions. — Approval of the requisition be the sangguniang barangay. No national official shall sit as a
by the head of office or department concerned who has member of the committee on awards.
administrative control of the appropriation against which the The results of the bidding shall be made public by conspicuously
proposed expenditure is chargeable is deemed sufficient, except posting the same in the provincial capitol or city, municipal, or
in case of requisition r supplies to be carried in stock which shall barangay hall.
be approved by the local chief executive concerned: Provided,
That such supplies are listed or included in the annual Sec365. Rule on Awards. — Awards in the procurement of
procurement plan and the maximum quantity thereof does not supplies shall be given to the lowest complying and responsible
exceed the estimated consumption corresponding to a bid which meets all the terms and conditions of the contract or
programmed three-month period: Provided, further, That nothing undertaking.
herein contained shall be held as authorizing the purchase of
furniture and equipment for stock purposes. Sec366. Procurement Without Public Bidding. —
Procurement of supplies may be made without the benefit of
Sec362. Call for Bids. — When procurement is to be made by public bidding under any of the following modes:
local government units, the provincial or city general services (a) Personal canvass of responsible merchants;
officer or the municipal or barangay treasurer shall call bids for (b) Emergency purchase;
open public competition. The call for bids shall show the (c) Negotiated purchase;
complete specifications and technical descriptions of the (d) Direct purchase from manufacturers or exclusive
required supplies and shall embody all terms and conditions of distributors; and
participation and award, terms of delivery and payment, and all (e) Purchase from other government entities.
other covenants affecting the transaction. In all calls for bids, the
right to waive any defect in the tender as well as the right to Sec367. Procurement through Personal Canvass. — Upon
accept the bid most advantageous to the government shall be approval by the Committee on Awards, procurement of supplies
reserved. In no case, however, shall failure to meet the may be effected after personal canvass of at least three (3)
specifications or technical requirements of the supplies desired responsible suppliers in the locality by a committee of three (3)
be awarded. composed of the local services officer or the municipal or
barangay treasurer, as the case may be, the local accountant,
Sec363. Publication of Call for Bids. — The call for bids shall be and the head of office or department for whose use the supplies
given the widest publicity possible, sending, by mail or are being procured. The award shall be decided by the
otherwise, any known prospective participant in the locality, of Committee on Awards.
copies of the call and by posting copies of the same in at least Purchases under this Section shall not exceed the amounts
three (3) publicly accessible and conspicuous places in the specified hereunder for all items in any one (1) month for each
provincial capitol or city, municipal, or barangay hall, as the case local government unit:
may be. Provinces and Cities and Municipalities within the Metropolitan
The notice of the bidding may likewise be published in a Manila Area:
newspaper of general circulation in the territorial jurisdiction of First and Second Class — One hundred fifty thousand pesos
the local government unit concerned when the provincial or city (P150,000.00)
general services officer or the municipal or barangay treasurer, Third and Fourth Class — One hundred thousand pesos
as the case may be, deems it necessary in order to obtain the (P100,000.00)
lowest responsible and complying bid. Fifth and Sixth Class — Fifty thousand pesos
The opening of bids shall only be made in the presence of the (P50,000.00)
provincial or city auditor or his duly authorized representative Municipalities:
who shall initial and secure copies of the bids and certify the First Class — Sixty thousand pesos (P60,000.00)
abstract of the bidding. Second and Third Class — Forty thousand pesos
(P40,000.00)
Sec364. The Committee on Awards. — There shall be in Fourth Class and Below — Twenty thousand pesos
every province, city or municipality a committee on awards to (P20,000.00)
decide the winning bids and questions of awards on
procurement and disposal of property. Sec368. Emergency Purchase. — In cases of emergency where
the need for the supplies is exceptionally urgent or absolutely

The Apocryphal Maggots:


Rainier, Chrisgel, Corina, Geoffry, Grace and Sylvie Blanche
The Flibbertigibbet Worms:
Golda, Gladys and Melyjane
CA VE AT: By simply r ea ding this re vi ew er a t the end o f the sem este r wi ll (hopef ully) guar antee you r pas sing this cou rse. D rink mode rat ely .
Amusin S C A r y
gly
THE C2005 LOCAL GOVERNMENT REVIEWER - 264 -
indispensable and only to prevent imminent danger to, or loss (5) A certification of the provincial or city general services of
of, life or property, local government units may, through the local the municipal or barangay treasurer, as the case may
chief executive concerned, make emergency purchases or place be, to the effect that the price paid or contracted for was
repair orders, regardless of amount, without public bidding. the lowest at the time of procurement;
Delivery of purchase orders or utilization of repair orders (6) A certification to the effect that the price paid or
pursuant to this Section shall be made within ten (10) days after contracted for was the lowest at the time of procurement;
placement of the same. Immediately after the emergency and
purchase or repair order is made, the chief of office or (7) A certification of the local budget officer as to the
department making the emergency purchase or repair order existence of appropriations for the purpose, the local
shall draw a regular requisition to cover the same which shall accountant as to the obligation of the amount involved,
contain the following: and the local treasurer as to the availability of funds.
(a) A complete description of the supplies acquired or the (b) In case of repeat orders for regular supplies,
work done or to be performed; procurement may be made by negotiated purchase: Provided,
(b) By whom furnished or executed; That the repeat order is made within three (3) months from the
(c) Date of placing the order and the date and time of last procurement of the same item: Provided, further, That the
delivery or execution; same terms and conditions of sale are obtained for the said
(d) The unit price and the total contract price; repeat order.
(e) A brief and concise explanation of the circumstances
why procurement was of such urgency that the same could not Sec370. Procurement from Duly Licensed Manufacturer. —
be done through the regular course without involving danger to, Procurement may be made directly from duly licensed
or loss of, life or property; manufacturers in cases of supplies of Philippine manufacture or
(f) A certification of the provincial or city general services origin and in case there are two (2) or more manufacturers shall
or the municipal or barangay treasurer, as the case may be, to be conducted to obtain the lowest price for the quality of the said
the effect that the price paid or contracted for was the lowest at supplies.
the time of procurement; and
(g) A certification of the local budget officer as to the Sec371. Procurement from Exclusive Philippine Agents or
existence of appropriations for the purpose, the local accountant Distributors. — Procurement may, in the case of supplies of
as to the obligation of the amount involved, and the local foreign origin, preferably be made directly from the exclusive or
treasurer as to the availability of funds. The goods or services reputable Philippine distributors or agents, subject to the
procured under this Section must be utilized or availed of within following conditions:
fifteen (15) days from the date of delivery or availability. (a) That the Philippine distributor has no subdealers
Without prejudice to criminal prosecution under applicable laws, selling at lower prices; and
the local chief executive, the head of department, or the chief of (b) That no suitable substitutes or substantially the same
office making the procurement shall be administratively liable for quality are available at lower prices.
any violation of this Section and shall be a ground for
suspension or dismissal from service. Sec372. Procurement from Government Entities. —
Procurement may be made directly from the government entities
Sec369. Negotiated Purchase. — producing the required supplies, including units or agencies of
(a) In cases where public biddings have failed for two (2) foreign governments with which the Philippines maintains
consecutive times and no suppliers have qualified to participate diplomatic relations. In the latter case, prior authority from the
or win in the biddings, local government units may, through the Office of the President shall be required.
local chief executive concerned, undertake the procurement of
supplies by negotiated purchase, regardless of amount, without Sec373. Annual Procurement Program. —
public bidding: Provided, however, That the contract covering (a) On or before the fifteenth (15th) day of July each year,
the negotiated purchase shall be approved by the sanggunian the local chief executive shall prepare an annual procurement
concerned. Delivery of purchase orders or utilization of repair program for the ensuing fiscal year which shall contain an
orders pursuant to this Section shall be made within seven (7) itemized list of the estimated quantity of supplies needed for
days after placement of the same. Immediately after the such year, a complete description thereof as to kind, quality,
negotiated purchase or repair order is made, the local chief estimated cost, and balance on hand: Provided, however, That
executive concerned shall draw a regular requisition to cover the the total estimated cost of the approved annual procurement
same which shall contain the following: program shall not exceed the total appropriations authorized for
(1) A complete description of the supplies acquired or the the acquisition of supplies. The local government units may
work done or to be performed; augment the supplies and equipment provided by the Supreme
(2) By whom furnished or executed; Court to the lower courts located in their respective jurisdictions.
(3) Date of placing the order and the date and time of (b) Except in emergency cases or where urgent
delivery or execution; indispensable needs could not have been reasonably
(4) The unit price and the total contract price; anticipated, no purchase of supplies shall be made unless
included in. or covered by, the approved procurement program.

The Apocryphal Maggots:


Rainier, Chrisgel, Corina, Geoffry, Grace and Sylvie Blanche
The Flibbertigibbet Worms:
Golda, Gladys and Melyjane
CA VE AT: By simply r ea ding this re vi ew er a t the end o f the sem este r wi ll (hopef ully) guar antee you r pas sing this cou rse. D rink mode rat ely .
Amusin S C A r y
gly
THE C2005 LOCAL GOVERNMENT REVIEWER - 265 -
(c) The conversion of excess cash into supplies stock is (b) Unless he registers his objection in writing, an
hereby prohibited except to the extent of the kind and quantity accountable person shall not be relieved from liability by reason
specified in the approved annual procurement plan. A violation of his having acted under the direction of a superior officer in
of this Section shall be a ground for suspension or dismissal of using property with which he is chargeable; but the officer
any political or employee responsible therefor. directing any illegal, unauthorized or improper use of property
shall first be required to answer therefor.
Sec374. Establishment of an Archival System. — Every local (c) In cases of loss, damage, or deterioration of
government unit shall provide for the establishment of archival government property arising from, or attributable to, negligence
system to ensure the safety and protection of all government in security, the head of the security agency shall be held liable
property, public documents or records such as records of births, therefor.
marriages, property inventory, land assessments, land
ownership, tax payments, tax accounts, and business permits, Sec378. Credit for Loss Occurring in Transit or Due to
and such other records or documents of public interest in the Casualty. — When a loss of government property occurs while
various departments and offices of the provincial, city, or the same is in transit or is caused by fire, theft, force majeure, or
municipal government concerned. other casualty, the officer accountable therefor or having
custody thereof shall immediately notify the provincial or city
Sec375. Primary and Secondary Accountability for auditor concerned within thirty (30) days from the date the loss
Government Property. — occurred or for such longer period as the provincial, city or
(a) Each head of department or office of a province, city, municipal auditor, as the case may be, may in the particular
municipality or barangay shall be primarily accountable for all case allow, and he shall present his application for relief, with
government property assigned or issued to his department or the available evidence in support thereof. An officer who fails to
office. The person or persons entrusted with the possession or comply with this requirement shall not be relieved of liability or
custody of government property under the accountability of any allowed credit for any such loss in the settlement of his
head of department or office shall be immediately accountable accounts.
to such officer. A provincial, city or municipal auditor shall not allow credit for
(b) The head of a department or office primarily these losses unless so expressly authorized by the Chairman of
accountable for government property may require any person in the Commission on Audit, to the exercised only if the loss is not
possession of the property or having custody and control thereof in excess of fifty thousand pesos (P50,000.00). In any case
under him to keep such records and make reports as may be when the allowance of credit is not within the competence of the
necessary for his own information and protection. provincial, city or municipal auditor, the application and
(c) Buildings and other physical structures shall be under evidence, with the recommendation of the auditor concerned,
the accountability and responsibility of the provincial or city shall be forwarded to the Chairman of the Commission on Audit
general services officer or the municipal mayor or punong for his appropriate action.
barangay, as the case may be. (d) Every officer primarily
accountable for government property shall keep a complete Sec379. Property Disposal. — When property of any local
record of all properties under his charge and render his government unit has become unserviceable for any cause or is
accounts therefor semiannually to the provincial or city general no longer needed, it shall upon application of the officer
services officer or the municipal mayor or punong barangay, as accountable therefor, be inspected and appraised by the
the case may be. provincial, city or municipal auditor, as the case may be, or his
duly authorized representative or that of the Commission on
Sec376. Responsibility for Proper Use and Care of Audit and, if found valueless or unusable, shall be destroyed in
Government Property. — The person in actual physical the presence of the inspecting officer.
possession of government property or entrusted with its custody If found valuable, the same shall be sold at public auction to the
and control shall be responsible for its proper use and care and highest bidder under the supervision of the committee on
shall exercise due diligence in the utilization and safekeeping awards and in the presence of the provincial, city or municipal
thereof. auditor or his duly authorized representative. Notice of the public
auction shall be posted in at least three (3) publicly accessible
Sec377. Measure of Liability of Persons Accountable for and conspicuous places, and if the acquisition cost exceeds
Government Property. — One hundred thousand pesos (P100,000.00) in the case of
(a) The person immediately accountable for government provinces and cities, and Fifty thousand pesos (P50,000.00) in
property shall be liable for its money value in case of the illegal, the case of municipalities, notice of auction shall be published at
improper or unauthorized use or misapplication thereof, by least two (2) times within a reasonable period in a newspaper of
himself or any other person for whose acts he may be general circulation in the locality.
responsible, and he shall be liable for all loss, damage, or
deterioration occasioned by negligence in the keeping or use of Sec380. Negotiated Sale of Property. — Property no longer
property unless it is proved that he has exercised due diligence needed may also be disposed of at a private sale at such price
and care in the utilization and safekeeping thereof. as may be determined by the committee on awards, subject to
the approval of the Commission on Audit or its duly authorized

The Apocryphal Maggots:


Rainier, Chrisgel, Corina, Geoffry, Grace and Sylvie Blanche
The Flibbertigibbet Worms:
Golda, Gladys and Melyjane
CA VE AT: By simply r ea ding this re vi ew er a t the end o f the sem este r wi ll (hopef ully) guar antee you r pas sing this cou rse. D rink mode rat ely .
Amusin S C A r y
gly
THE C2005 LOCAL GOVERNMENT REVIEWER - 266 -
representative when the acquisition or transfer cost of the law which requires that the said valuation is a condition sine qua
property exceeds Fifty thousand pesos (P50,000.00) in the case non for the validity of a donation.
of provinces and cities, and Twenty-five thousand pesos
(P25,000.00) in the case of municipalities and barangays. There being a perfected contract, it cannot be revoked without the
In case of real property, the disposal shall be subject to the consent of the other party.
approval of the Commission on Audit regardless of the value or
cost involved.

Sec381. Transfer Without Cost. — Property which has become


unserviceable or is no longer needed may be transferred without
cost to another office, agency, subdivision or instrumentality of
the national government or another local government unit at an
appraised valuation determined by the local committee on
awards. Such transfer shall be subject to the approval of the
sanggunian concerned making the transfer and by the head of
the office, agency, subdivision, instrumentality or local
government unit receiving the property.

Sec382. Tax Exemption Privileges of Local Government Units.


— Local government units shall be exempt from the payment of
duties and taxes for the importation of heavy equipment or
machineries which shall be used for the construction, Masturbation: Tips for Guys (Part 5)
improvement, repair, and maintenance of roads, bridges and
other infrastructure projects, as well as garbage trucks, fire - Stroke only your shaft, ignoring the head, which will swell
trucks, and other similar equipment: Provided, however, That and turn red. When it's bright red and rock hard, stimulate
such equipment or machineries shall not be disposed of, either
it gently with your fingers.
by public auction or negotiated sale as hereinabove provided,
within five (5) years from the importation thereof. In case the - Lightly and slowly run a finger up the under side of your
machinery or equipment is sold within the five-year period, the cock, gently pinching the sensitive vein underneath.
purchasers or recipients shall be considered the importers - Here's a complicated move from The Society for Human
thereof, and shall be liable for duties and taxes computed on the Sexuality: Take the penis in both hands, fingers lightly
book value of such importation. touching the sides of the shaft. In order to visualize the
position, think of yourself holding a clarinet. Now flick the
Sec383. Implementing Rules and Regulations. — The penis back and forth between your two hands by holding
Chairman of the Commission on Audit shall promulgate the rules on to the loose skin of the shaft. Shuttling it back and forth
and regulations necessary to effectively implement the in this manner may not seem incredibly thrilling at first,
provisions of this Title, including requirements as to testing, but pretty soon, as it builds up momentum, it will drive
inspection, and standardization of supply and property. you out of your mind. Orgasms encountered via this
method are sometimes messy, but always memorable.
GSIS v. Tarlac (2003)
If you've mastered these techniques and would like to try
The Sangguniang Panlalawigan passed a resolution that something different, consider purchasing an erection ring,
- authorized and approved the conversion of the athletic which will delay and enhance your orgasm. You might also
field into a government center, and the experiment with sex toys for men, such as penis pumps, virtual
- segregation and donation of portions of said land to sheaths, masturbators and extenders. Try different kinds of
different government agencies for the purpose of lubrication, such as oil, water or silicone-based lubes. Start a
constructing or relocating their office buildings. collection of adult videos, magazines and other "inspirational"
materials. Try masturbating with a pillow or other soft
GSIS availed of the offer and proceeded to issue a MOA whereby:
substance. Your possibilities are really limitless.
- Province of Tarlac donated the lot to the GSIS
- GSIS, in return, donated 2M to the province as financial Partner Masturbation
assistance.
Once you and your partner have reached a level of intimacy that
Held: Sec. 381 does not expressly prohibit or declare void such
allows both of you to feel comfortable discussing solo sex, you
transfers if an appraised valuation from the local committee on
might enjoy practicing masturbation together, either on
awards is not first acquired. There is no express provision in the
yourselves (mutual masturbation) or on one another (also

The Apocryphal Maggots:


Rainier, Chrisgel, Corina, Geoffry, Grace and Sylvie Blanche
The Flibbertigibbet Worms:
Golda, Gladys and Melyjane
CA VE AT: By simply r ea ding this re vi ew er a t the end o f the sem este r wi ll (hopef ully) guar antee you r pas sing this cou rse. D rink mode rat ely .
Amusin S C A r y
gly
THE C2005 LOCAL GOVERNMENT REVIEWER - 267 -
known as genital massage). Before you begin, discuss your
upcoming experience. Find out if your partner would prefer to Barangay Officials and Offices
masturbate in front of you, or just massage your genitals. punong barangay,
Always respect his or her wishes. There's plenty of time to try (7) sangguniang barangay members,
different things later on in your relationship. When you're both sangguniang kabataan chairman,
on the same wavelength about what you want, jump in and get  barangay secretary,
started. Make an event of the experience: create a sensual, barangay treasurer.
appealing environment in your bedroom by lighting candles, lupong tagapamayapa.
playing sensual music and showering together first. Relax on
Persons in Authority (For purposes of the Revised Penal Code)
the bed and try any of the techniques described above. Don't
focus on climaxing or giving your partner an orgasm. Just enjoy  the punong barangay
 sangguniang barangay members
the experience and concentrate on finding the strokes that work
best for both of you.  members of the lupong tagapamayapa
 other barangay officials and members who may be
designated by law or ordinance and charged with the
Whatever your preferences, remember to vary your
maintenance of public order, protection and security of life
masturbation techniques. While it's great to know how to bring and property, or the maintenance of a desirable and
yourself to orgasm in 30 seconds or less, you're going to have a balanced environment
much more enjoyable experience if you romance yourself a little  any barangay member who comes to the aid of persons in
bit first. Take the time to have fun ... don't be "goal oriented." authority, shall be deemed agents of persons in authority.
And do try a different stroke now and then. Just because one
stimulation works well for you, doesn't mean something else The Punong Barangay - Chief Executive
won't feel great too! (end) shall exercise such powers and perform such duties and
functions, as provided by this Code and other laws.
 FOR efficient, effective and economical governance, the
purpose of which is the general welfare of the barangay
Par t I II: Local Go vernment Units and its inhabitants pursuant to Section 16 of this Code,
the punong barangay shall:
The Barangay (Sec384-439) (1) Enforce all laws and ordinances which are applicable
within the barangay;
Barangay —basic political unit, serves as (2) Negotiate, enter into, and sign contracts for and in behalf
 the primary planning and implementing unit of government of the barangay, upon authorization of the sangguniang
policies, plans, programs, projects, and activities in the community, barangay;
forum wherein the collective views of the people may be (3) Maintain public order in the barangay and, in pursuance
expressed, crystallized and considered, and where disputes may thereof, assist the city or municipal mayor and the
be amicably settled. sanggunian members in the performance of their duties
and functions;
Manner of Creation. — may be created, divided, merged, (4) Call and preside over the sessions of the sangguniang
abolished, or its boundary substantially altered, by law or by an barangay and the barangay assembly, and vote only to
ordinance of the sangguniang panlalawigan or panlungsod, break a tie;
subject to approval by a majority of the votes cast in a plebiscite to (5) Upon approval by a majority of all the members of the
be conducted by the COMELEC in the local government unit or sangguniang barangay, appoint or replace the barangay
units directly affected within such period of time as may be treasurer, the barangay secretary, and other appointive
determined by the law or ordinance creating said barangay. In the barangay officials;
case of the creation of barangays by the sangguniang (6) Organize and lead an emergency group whenever the
panlalawigan, the recommendation of the sangguniang bayan same may be necessary for the maintenance of peace
concerned shall be necessary. and order or on occasions of emergency or calamity
within the barangay;
Requisites for Creation. — (7) In coordination with the barangay development council,
 a contiguous territory EXCEPT if it comprises two (2) or prepare the annual executive and supplemental budgets
more islands. of the barangay;
a population of at least 2,000 inhabitants (5,000 in cities (8) Approve vouchers relating to the disbursement of
and municipalities within Metro Manila and other metropolitan barangay funds;
political subdivisions or in highly urbanized cities (9) Enforce laws and regulations relating to pollution control
Provided, That the creation thereof shall not reduce the and protection of the environment;
population of the original barangay or barangays to less than the
minimum requirement prescribed herein.

The Apocryphal Maggots:


Rainier, Chrisgel, Corina, Geoffry, Grace and Sylvie Blanche
The Flibbertigibbet Worms:
Golda, Gladys and Melyjane
CA VE AT: By simply r ea ding this re vi ew er a t the end o f the sem este r wi ll (hopef ully) guar antee you r pas sing this cou rse. D rink mode rat ely .
Amusin S C A r y
gly
THE C2005 LOCAL GOVERNMENT REVIEWER - 268 -
(10) Administer the operation of the katarungang Appointive Barangay Officials:
pambarangay in accordance with the provisions of this Barangay Secretary and Barangay Treasurer
Code;  appointed by the punong barangay with the
(11) Exercise general supervision over the activities of the concurrence of the majority of all the sangguniang
sangguniang kabataan; barangay members. The appointment of the barangay
(12) Ensure the delivery of basic services as mandated under secretary shall not be subject to attestation by the Civil Service
Section 17 of this Code; Commission.
(13) Conduct an annual palarong barangay which shall  shall be of legal age, a qualified voter and an actual
feature traditional sports and disciplines included in resident of the barangay concerned.
national and international games, in coordination with  not a sangguniang barangay member, a gov’t
the Department of Education, Culture and Sports; employee, or a relative of the punong barangay within the fourth
(14) Promote the general welfare of the barangay; and civil degree of consanguinity of affinity.
(15) Exercise such other powers and perform such other  barangay treasurer shall be bonded in an amount
duties and functions as may be prescribed by law or to be determined by the sangguniang barangay but not
ordinance. exceeding Ten thousand pesos
In the performance of his peace and order functions. the
punong barangay shall be entitled to possess and carry the Barangay Assembly
necessary firearm within his territorial jurisdiction, subject to  composed of all persons who are actual residents of the
appropriate rules and regulations. barangay for at least six (6) months, fifteen (15) years of age or
over, citizens of the Philippines, and duly registered in the list of
The Sangguniang Barangay barangay assembly members.
 the legislative body of the barangay, shall be composed of  meet at least twice a year to hear and discuss the semestral
the punong barangay as presiding officer, and the 7 regular report of the sangguniang barangay concerning its activities and
sangguniang barangay members elected at large and finances as well as problems affecting the barangay upon call of
sangguniang kabataan chairman, as members. the punong barangay or of at least four (4) members of the
sangguniang barangay, or upon written petition of at least five
As legislative body, enact ordinances as may be necessary to percent (5%) of the assembly members.
discharge the responsibilities conferred upon it by law or 1 wk prior written notice to meeting needed except on matters
ordinance and to promote the general welfare of the inhabitants involving public safety or security, in which case notice within a
therein; by law or ordinance. reasonable time shall be sufficient
Other Duties of Sangguniang Barangay Members Powers of the Barangay Assembly.
Assist the punong barangay in the discharge of his duties and (a) Initiate legislative processes by recommending to the
functions; sangguniang barangay the adoption of measures for the welfare
Act as peace officers in the maintenance of public order and of the barangay and the city or municipality concerned;
safety; and (b) Decide on the adoption of initiative as a legal process
Perform such other duties and functions as the punong whereby the registered voters of the barangay may directly
barangay may delegate. propose, enact, or amend any ordinance; and
(c) Hear and pass upon the semestral report of the
Benefits of Barangay Officials. — sangguniang barangay concerning its activities and finances.
honoraria, allowances, and such other emoluments as may be
authorized by law or barangay
Christmas bonus of at least One thousand pesos (P1,000.00) RA 8441 (1997) - An Act Increasing The Cash Gift To Five
insurance coverage Thousand Pesos (P5,000.00), Amending For The Purpose
free medical care including subsistence, medicines, and Certain Sections Of RA6686, And For Other Purposes
medical attendance in any gov’t hospital or institution
exempted during their incumbency from paying tuition and Sec4. Section 4 of Republic Act No. 6686 is hereby amended to
matriculation fees for their legitimate dependent children attending read as follows:
state colleges or universities. "SEC. 4. All officials and employees of local government units
 appropriate civil service eligibility on the basis of the number may receive the same benefits as are provided under Sections 1
of years and 2 of this Act chargeable against their respective local funds.
 Elective barangay officials shall have preference in For CY 1997, local government units (municipalities, cities and
appointments to any government position or GOCC’s provinces) may realign their budgets to give priority to the
 members of the barangay tanod brigades, (not more than 20) funding requirements under this Act and any deficiency may be
in each barangay, shall be granted insurance or other benefits booked as accounts payable to be paid on a first priority basis in
during their incumbency succeeding years."

The Apocryphal Maggots:


Rainier, Chrisgel, Corina, Geoffry, Grace and Sylvie Blanche
The Flibbertigibbet Worms:
Golda, Gladys and Melyjane
CA VE AT: By simply r ea ding this re vi ew er a t the end o f the sem este r wi ll (hopef ully) guar antee you r pas sing this cou rse. D rink mode rat ely .
Amusin S C A r y
gly
THE C2005 LOCAL GOVERNMENT REVIEWER - 269 -
competent authority, is charged with the maintenance of public
RA 9178 (2002) - Barangay Micro Business Enterprises order and the protection and security of life and property.
(BMBEs) Act of 2002."
Under RPC, Direct assault may be committed by any person/s
Declaration of Policy. — It is hereby declared to be the policy of who without a public uprising shall attack, employ force, or
the State to hasten the country's economic development by seriously intimidate or resist any person in authority or any of
encouraging the formation and growth of barangay micro his agents while engaged in the performance of his official
business enterprises which effectively serve as seedbeds of duties, or on occasion of such performance
Filipino entrepreneurial talents, and integrating those in the
informal sector with the mainstream economy, through the NOTES:
rationalization of bureaucratic restrictions, the active intervention SC emphasized on the fact that the tanod was on his way home.
of the government specially in the local level, and the granting of
incentives and benefits to generate much-needed employment SIR: Since tanod is a person (agent) in authority, surrender to
and alleviate poverty. him is equivalent to a mitigating circumstance.

***Barangay Micro Business Enterprise (BMBE) - refers to any


business entity or enterprise engaged in the production, David vs. COMELEC (1997)
processing or manufacturing of products or commodities,
including agro-processing, trading and services, whose total Under Sec. 2 of RA 6653, the term of office of barangay officials
assets including those arising from loans but exclusive of the shall be for 5 years. Sec. 43 of RA 7160 reduced the term of office
land on which the particular business entity's office, plant and of all local elective officials to 3 years.
equipment are situated, shall not be more than Three Million
Pesos (P3,000,000.00). RA 7160 was enacted later than RA 6679. In case of an
irreconcilable conflict between 2 laws of different vintages, the later
REGISTRATION AND OPERATION OF BMBEs law repeals an earlier one, because it is the later legislative will.
The Office of the Treasurer of each city or municipality shall
register the BMBEs and issue a Certificate of Authority to enable The Constitution did not expressly prohibit Congress from fixing
the BMBE to avail of the benefits under this Act. Any such any term of office for barangay officials. It merely left the
application shall be processed within fifteen (15) working days determination of such term to the lawmaking body, without any
upon submission of complete documents. Otherwise, the specific limitation or prohibition, thereby leaving to the lawmakers
BMBEs shall be deemed registered. full discretion to fix such term in accordance with the exigencies of
The Certificate of Authority shall be effective for a period of 2 public service.
years, renewable for a period of two (2) years for every renewal.
NOTE: Sec 2 of RA 9164 provided that the term of office of
Who are Eligible to Register. — Any person, natural or juridical, barangay officials shall be for 3 years and that no barangay
or cooperative, or association, having the qualifications as officials shall serve for more than 3 consecutive terms reckoning
defined hereof (see *** above) may apply for registration as from the 1994 barangay elections.
BMBE.

INCENTIVES AND BENEFITS


Exemption from Taxes and Fees. Alquizola vs. Ocol (1999)
Exemption from the Coverage of the Minimum Wage Law.
Credit Delivery — LBP, DBP, the Small Business Guarantee Under Sec 389 LGCode, upon the approval by a majority of all the
and Finance Corporation (SBGFC), and the People's Credit and members of the SB, the punong brgy can appoint and replace the
Finance Corporation (PCFC) shall set up a special credit brgy treasurer, secretary and other appointive brgy oficicials.
window that will service the financing needs of BMBEs
registered under this Act consistent with the Bangko Sentral ng The term replace would obviously embrace not only the
Pilipinas (BSP) policies, rules and regulations. appointment of the replacement but also the prior removal of or
Technology Transfer, Production and Management Training, vacation by, the official currently occupying the appointive position
and Marketing Assistance. — P300Millionfrom PAGCOR concerned.

Since there is no other provision in the LGCode that treats of the


power of Punong Brgy to remove the brgy appointive officials and
People vs. Recto (2001) the duration of their term of office has not been fixed by LGCode,
it is a sound and useful rule to consider the power of removal as
An agent or person in authority is any person who by direct being an incident to the power of appointment.
provision of law or by election, or by appointment by

The Apocryphal Maggots:


Rainier, Chrisgel, Corina, Geoffry, Grace and Sylvie Blanche
The Flibbertigibbet Worms:
Golda, Gladys and Melyjane
CA VE AT: By simply r ea ding this re vi ew er a t the end o f the sem este r wi ll (hopef ully) guar antee you r pas sing this cou rse. D rink mode rat ely .
Amusin S C A r y
gly
THE C2005 LOCAL GOVERNMENT REVIEWER - 270 -
Verily, the power of appointment is to be exercised conjointly by qualified person who shall hold office only for the unexpired
the punong brgy and a majority of all the members of the SB. portion of the term.
W/out such conjoint action, neither an appointment nor a
replacement can be effectual. Sec402. Functions of the Lupon. — The lupon shall:
(a) Exercise administrative supervision over the
To hold otherwise would create an absurd situation of the SB conciliation panels provided herein;
members refusing to give their approval to the replacements (b) Meet regularly once a month to provide a forum for
selected by the punong brgy who has unilaterally terminated the exchange of ideas among its members and the public on
services of the incumbents. matters relevant to the amicable settlement of disputes, and to
enable various conciliation panel members to share with one
another their observations and experiences in effecting speedy
Katarungang Pambarangay – LGC resolution of disputes; and
(c) Exercise such other powers and perform such other
Sec399. Lupong Tagapamayapa. — duties and functions as may be prescribed by law or ordinance.
(a) There is hereby created in each barangay a lupong
tagapamayapa, hereinafter referred to as the lupon, composed Sec403. Secretary of the Lupon. — The barangay secretary
of the punong barangay, as chairman and ten (10) to twenty (20) shall concurrently serve as the secretary of the lupon. He shall
members. The lupon shall be constituted every three (3) years in record the results of mediation proceedings before the punong
the manner provided herein. barangay and shall submit a report thereon to the proper city or
(b) Any person actually residing or working, in the municipal courts. He shall also receive and keep the records of
barangay, not otherwise expressly disqualified by law, and proceedings submitted to him by the various conciliation panels.
possessing integrity, impartiality, independence of mind, sense
of fairness, and reputation for probity, may be appointed a Sec404. Pangkat ng Tagapagkasundo. —
member of the lupon. (a) There shall be constituted for each dispute brought
(c) A notice to constitute the lupon, which shall include before the lupon a conciliation panel to be known as the pangkat
the names of proposed members who have expressed their ng tagapagkasundo, hereinafter referred to as the pangkat,
willingness to serve, shall be prepared by the punong barangay consisting of three (3) members who shall be chosen by the
within the first fifteen (15) days from the start of his term of parties to the dispute from the list of members of the lupon.
office. Such notice shall be posted in three (3) conspicuous Should the parties fail to agree on the pangkat membership, the
places in the barangay continuously for a period of not less than same shall be determined by lots drawn by the lupon chairman.
three (3) weeks; (b) The three (3) members constituting the pangkat shall
(d) The punong barangay, taking into consideration any elect from among themselves the chairman and the secretary.
opposition to the proposed appointment or any The secretary shall prepare the minutes of the pangkat
recommendations for appointment as may have been made proceedings and submit a copy duly attested to by the chairman
within the period of posting, shall within ten (10) days thereafter, to the lupon secretary and to the proper city or municipal court.
appoint as members those whom he determines to be suitable He shall issue and cause to be served notices to the parties
therefor. Appointments shall be in writing, signed by the punong concerned.
barangay, and attested to by the barangay secretary. The lupon secretary shall issue certified true copies of any
(e) The list of appointed members shall be posted in public record in his custody that is not by law otherwise declared
three (3) conspicuous places in the barangay for the entire confidential.
duration of their term of office; and
(f) In barangays where majority of the inhabitants are Sec405. Vacancies in the Pangkat. — Any vacancy in the
members of indigenous cultural communities, local systems of pangkat shall be chosen by the parties to the dispute from
settling disputes through their councils of datus or elders shall among the other lupon members. Should the parties fail to
be recognized without prejudice to the applicable provisions of agree on a common choice, the vacancy shall be filled by lot to
this Code. be drawn by the lupon chairman.

Sec400. Oath and Term of Office. — Upon appointment, each Sec406. Character of Office and Service of Lupon
lupon member shall take an oath of office before the punong Members. —
barangay. He shall hold office until a new lupon is constituted on (a) The lupon members, while in the performance of their
the third year following his appointment unless sooner official duties or on the occasion thereof, shall be deemed as
terminated by resignation, transfer of residence or place of work, persons in authority, as defined in the Revised Penal Code.
or withdrawal of appointment by the punong barangay with the (b) The lupon or pangkat members shall serve without
concurrence of the majority of all the members of the lupon. compensation, except as provided for in Section 393 and
without prejudice to incentives as provided for in this Section
Sec401. Vacancies. — Should a vacancy occur in the lupon and in Book IV of this Code. The Department of the Interior and
for any cause, the punong barangay shall immediately appoint a Local Government shall provide for a system of granting
economic or other incentives to the lupon or pangkat members

The Apocryphal Maggots:


Rainier, Chrisgel, Corina, Geoffry, Grace and Sylvie Blanche
The Flibbertigibbet Worms:
Golda, Gladys and Melyjane
CA VE AT: By simply r ea ding this re vi ew er a t the end o f the sem este r wi ll (hopef ully) guar antee you r pas sing this cou rse. D rink mode rat ely .
Amusin S C A r y
gly
THE C2005 LOCAL GOVERNMENT REVIEWER - 271 -
who adequately demonstrate the ability to judiciously and Objections to venue shall be raised in the mediation
expeditiously resolve cases referred to them. While in the proceedings before the punong barangay; otherwise, the same
performance of their duties, the lupon or pangkat members, shall be deemed waived. Any legal question which may confront
whether in public or private employment, shall be deemed to be the punong barangay in resolving objections to venue herein
on official time, and shall not suffer from any diminution in referred to may be submitted to the Secretary of Justice, or his
compensation or allowance from said employment by reason duly designated representative, whose ruling thereon shall be
thereof. binding.

Sec407. Legal Advice on Matters Involving Questions of Sec410. Procedure for Amicable Settlement. —
Law. — The provincial, city legal officer or prosecutor or the (a) Who may initiate proceeding — Upon payment of the
municipal legal officer shall render legal advice on matters appropriate filing fee, any individual who has a cause of action
involving questions of law to the punong barangay or any lupon against another individual involving any matter within the
or pangkat member whenever necessary in the exercise of his authority of the lupon may complain, orally or in writing, to the
functions in the administration of the katarungang pambarangay. lupon chairman of the barangay.
(b) Mediation by lupon chairman — Upon receipt of the
Sec408. Subject Matter for Amicable Settlement; Exception complaint, the lupon chairman shall within the next working day
Thereto. — The lupon of each barangay shall have authority to summon the respondent(s), with notice to the complainant(s) for
bring together the parties actually residing in the same city or them and their witnesses to appear before him for a mediation
municipality for amicable settlement of all disputes except: of their conflicting interests. If he fails in his mediation effort
(a) Where one party is the government, or any within fifteen (15) days from the first meeting of the parties
subdivision or instrumentality thereof; before him, he shall forthwith set a date for the constitution of
(b) Where one party is a public officer or employee, and the pangkat in accordance with the provisions of this Chapter.
the dispute relates to the performance of his official functions; (c) Suspension of prescriptive period of offenses — While
(c) Offenses punishable by imprisonment exceeding one the dispute is under mediation, conciliation, or arbitration, the
(1) year or a fine exceeding Five thousand pesos (P5,000.00); prescriptive periods for offenses and cause of action under
(d) Offenses where there is no private offended party; existing laws shall be interrupted upon filing the complaint with
(e) Where the dispute involves real properties located in the punong barangay. The prescriptive periods shall resume
different cities or municipalities unless the parties thereto agree upon receipt by the complainant of the complainant or the
to submit their differences to amicable settlement by an certificate of repudiation or of the certification to file action
appropriate lupon; issued by the lupon or pangkat secretary: Provided, however,
(f) Disputes involving parties who actually reside in That such interruption shall not exceed sixty (60) days from the
barangays of different cities or municipalities, except where such filing of the complaint with the punong barangay.
barangay units adjoin each other and the parties thereto agree (d) Issuance of summons; hearing; grounds for
to submit their differences to amicable settlement by an disqualification — The pangkat shall convene not later than
appropriate lupon; three (3) days from its constitution, on the day and hour set by
(g) Such other classes of disputes which the President the lupon chairman, to hear both parties and their witnesses,
may determine in the interest of Justice or upon the simplify issues, and explore all possibilities for amicable
recommendation of the Secretary of Justice. settlement. For this purpose, the pangkat may issue summons
The court in which non-criminal cases not falling within the for the personal appearance of parties and witnesses before it.
authority of the lupon under this Code are filed may, at any time In the event that a party moves to disqualify any member of the
before trial motu propio refer the case to the lupon concerned pangkat by reason of relationship, bias, interest, or any other
for amicable settlement. similar grounds discovered after the constitution of the pangkat,
the matter shall be resolved by the affirmative vote of the
Sec409. Venue. — majority of the pangkat whose decision shall be final. Should
(a) Disputes between persons actually residing in the disqualification be decided upon, the resulting vacancy shall be
same barangay shall be brought for amicable settlement before filled as herein provided for.
the lupon of said barangay. (e) Period to arrive at a settlement — The pangkat shall
(b) Those involving actual residents of different arrive at a settlement or resolution of the dispute within fifteen
barangays within the same city or municipality shall be brought (15) days from the day it convenes in accordance with this
in the barangay where the respondent or any of the respondents section. This period shall, at the discretion of the pangkat, be
actually resides, at the election of the complaint. extendible for another period which shall not exceed fifteen (15)
(c) All disputes involving real property or any interest days, except in clearly meritorious cases.
therein shall be brought in the barangay where the real property
or the larger portion thereof is situated. Sec411. Form of settlement. — All amicable settlements shall
(d) Those arising at the workplace where the contending be in writing, in a language or dialect known to the parties,
parties are employed or at the institution where such parties are signed by them, and attested to by the lupon chairman or the
enrolled for study, shall be brought in the barangay where such pangkat chairman, as the case may be. When the parties to the
workplace or institution is located.

The Apocryphal Maggots:


Rainier, Chrisgel, Corina, Geoffry, Grace and Sylvie Blanche
The Flibbertigibbet Worms:
Golda, Gladys and Melyjane
CA VE AT: By simply r ea ding this re vi ew er a t the end o f the sem este r wi ll (hopef ully) guar antee you r pas sing this cou rse. D rink mode rat ely .
Amusin S C A r y
gly
THE C2005 LOCAL GOVERNMENT REVIEWER - 272 -
dispute do not use the same language or dialect, the settlement have the force and effect of a final judgment of a court upon the
shall be written in the language known to them. expiration of ten (10) days from the date thereof, unless
repudiation of the settlement has been made or a petition to
Sec412. Conciliation. — nullify the award has been filed before the proper city or
(a) Pre-condition to Filing of Complaint in Court. — No municipal court.
complaint, petition, action, or proceeding involving any matter However, this provision shall not apply to court cases settled by
within the authority of the lupon shall be filed or instituted the lupon under the last paragraph of Section 408 of this Code,
directly in court or any other government office for adjudication, in which case the compromise or the pangkat chairman shall be
unless there has been a confrontation between the parties submitted to the court and upon approval thereof, have the force
before the lupon chairman or the pangkat, and that no and effect of a judgment of said court.
conciliation or settlement has been reached as certified by the
lupon secretary or pangkat secretary as attested to by the lupon Sec417. Execution. — The amicable settlement or arbitration
or pangkat chairman or unless the settlement has been award may be enforced by execution by the lupon within six (6)
repudiated by the parties thereto. months from the date of the settlement. After the lapse of such
(b) Where Parties May Go Directly to Court. — The time, the settlement may be enforced by action in the
parties may go directly to court in the following instances: appropriate city or municipal court.
(1) Where the accused is under detention;
(2) Where a person has otherwise been deprived of Sec418. Repudiation. — Any party to the dispute may, within
personal liberty calling for habeas corpus proceedings; ten (10) days from the date of the settlement, repudiate the
(3) Where actions are coupled with provisional remedies same by filing with the lupon chairman a statement to that effect
such as preliminary injunction, attachment, delivery of sworn to before him, where the consent is vitiated by fraud,
personal property and support pendente lite; and violence, or intimidation. Such repudiation shall be sufficient
(4) Where the action may otherwise be barred by the statute basis for the issuance of the certification for filing a complaint as
of limitations. hereinabove provided.
(c) Conciliation among members of indigenous cultural
communities. — The customs and traditions of indigenous Sec419. Transmittal of Settlement and Arbitration. — Award
cultural communities shall be applied in settling disputes to the Court. — The secretary of the lupon shall transmit the
between members of the cultural communities. settlement or the arbitration award to the appropriate city or
municipal court within five (5) days from the date of the award or
Sec413. Arbitration. — from the lapse of the ten-day period repudiating the settlement
(a) The parties may, at any stage of the proceedings, and shall furnish copies thereof to each of the parties to the
agree in writing that they shall abide by the arbitration award of settlement and the lupon chairman.
the lupon chairman or the pangkat. Such agreement to arbitrate
may be repudiated within five (5) days from the date thereof for Sec420. Power to Administer Oaths. — The punong
the same grounds and in accordance with the procedure barangay, as chairman of the lupong tagapamayapa, and the
hereinafter prescribed. The arbitration award shall be made after members of the pangkat are hereby authorized to administer
the lapse of the period for repudiation and within ten (10) days oaths in connection with any matter relating to all proceedings in
thereafter. the implementation of the katarungang pambarangay.
(b) The arbitration award shall be in writing in a language
or dialect known to the parties. When the parties to the dispute Sec421. Administration; Rules and Regulations. — The city
do not use the same language or dialect, the award shall be or municipal mayor, as the case may be, shall see to the
written in the language or dialect known to them. efficient and effective implementation and administration of the
katarungang pambarangay. The Secretary of Justice shall
Sec414. Proceedings Open to the Public; Exception. — All promulgate the rules and regulations necessary to implement
proceedings for settlement shall be public and informal: this Chapter.
Provided, however, That the lupon chairman or the pangkat
chairman, as the case may be, may motu proprio or upon Sec422. Appropriations. — Such amount as may be
request of a party, exclude the public from the proceedings in necessary for the effective implementation of the katarungang
the interest of privacy, decency, or public morals. pambarangay shall be provided for in the annual budget of the
city or municipality concerned.
Sec415. Appearance of Parties in Person. — In all
katarungang pambarangay proceedings, the parties must
appear in person without the assistance of counsel or Supreme Court Administrative Circular No. 14-93
representative, except for minors and incompetents who may be
assisted by their next-of-kin who are not lawyers. Guidelines on the Katarungang Pambarangay conciliation
procedure to All RTC’s, MTC’s, MCTC’s MeTC’s
Sec416. Effect of Amicable Settlement and Arbitration
Award. — The amicable settlement and arbitration award shall

The Apocryphal Maggots:


Rainier, Chrisgel, Corina, Geoffry, Grace and Sylvie Blanche
The Flibbertigibbet Worms:
Golda, Gladys and Melyjane
CA VE AT: By simply r ea ding this re vi ew er a t the end o f the sem este r wi ll (hopef ully) guar antee you r pas sing this cou rse. D rink mode rat ely .
Amusin S C A r y
gly
THE C2005 LOCAL GOVERNMENT REVIEWER - 273 -
I. All disputes are subject to Barangay conciliation pursuant to the Secretary of Justice, the certification for filing a complaint in court
Revised Katarungang Pambarangay Law and prior recourse or any government office shall be issued by Barangay authorities
thereto is a pre-condition before filing a complaint in court or any only upon compliance with the following requirements:
government offices, except in the following disputes: 1. Issued by the Lupon Secretary and attested by the
1. Where one party is the government, or any Lupon Chairman (Punong Barangay), certifying that a
subdivision or instrumentality thereof; confrontation of the parties has taken place and that a conciliation
2. Where one party is a public officer or employee, and or settlement has been reached, but the same has been
the dispute relates to the performance of his official functions; subsequently repudiated (Sec. 412, Revised Katarungang
3. Where the dispute involves real properties located in Pambarangay Law; Sec. 2[h], Rule III, Katarungang Pambarangay
different cities and municipalities, unless the parties thereto agree Rules);
to submit their difference to amicable settlement by an appropriate 2. Issued by the Pangkat Secretary and attested by the
Lupon; Pangkat Chairman, certifying that:
4. Any complaint by or against corporations, partnerships a. a confrontation of the parties took place but no
or juridical entities, since only individuals shall be parties to conciliation/settlement has been reached (Sec. 4[f], Rule
Barangay conciliation proceedings either as complainants or III, Katarungang Pambarangay Rules; or
respondents (Sec. 1, Rule VI, Katarungang Pambarangay Rules); b. that no personal confrontation took place before the
5. Disputes involving parties who actually reside in Pangkat through no fault of the complainant (Sec. 4[f],
barangays of different cities or municipalities, except where such Rule III, Katarungang Pambarangay Rules).
barangay units adjoin each other and the parties thereto agree to 3. Issued by the Punong Barangay, as requested by the
submit their differences to amicable settlement by an appropriate proper party on the ground of failure of settlement where the
Lupon; dispute involves members of the same indigenous cultural
6. Offenses for which the law prescribes a maximum community, which shall be settled in accordance with the customs
penalty of imprisonment exceeding one (1) year or a fine over five and traditions of that particular cultural community, or where one or
thousand pesos (P5,000.00); more of the parties to the aforesaid dispute belong to the minority
7. Offenses where there is no private offended party; and the parties mutually agreed to submit their dispute to the
8. Disputes where urgent legal action is necessary to indigenous system of amicable settlement, and there has been no
prevent injustice from being committed or further continued, settlement as certified by the datu or tribal leader or elder to the
specifically the following: Punong Barangay of the place of settlement (Secs. 1, 4, & 5, Rule
a. Criminal cases where accused is under police custody IX, Katarungang Pambarangay Rules); and
or detention (See Sec. 412 (b)(1), Revised Katarungang 4. If mediation or conciliation efforts before the Punong
Pambarangay Law); Barangay proved unsuccessful, there having been no agreement
b. Petitions for habeas corpus by a person illegally to arbitrate (Sec. 410 [b], Revised Rule Katarungang
deprived of his rightful custody over another or a person Pambarangay Lay; Sec. 1, c, (1), Rule III, Katarungang
illegally deprived of his liberty or one acting in his behalf; Pambarangay Rules), or where the respondent fails to appear at
c. Actions coupled with provisional remedies such as the mediation proceeding before the Punong Barangay (3rd par.
preliminary injunction, attachment, delivery of personal Sec. 8, a, Rule VI, Katarungang Pambarangay Rules), the Punong
property and support during the pendency of the action; Barangay shall not cause the issuance of this stage of a
and certification to file action, because it is now mandatory for him to
d. Actions which may be barred by the Statute of constitute the Pangkat before whom mediation, conciliation, or
Limitations. arbitration proceedings shall be held.
9. Any class of disputes which the President may
determine in the interest of justice or upon the recommendation of III. All complaints and/or informations filed or raffled to your
the Secretary of Justice; sala/branch of the Regional Trial Court, Metropolitan Trial Court or
10. Where the dispute arises from the Comprehensive Municipal Trial Court shall be carefully read and scrutinized to
Agrarian Reform Law (CARL) (Secs. 46 & 47, R.A. 6657); determine if there has been compliance with prior Barangay
11. Labor disputes or controversies arising from conciliation procedure under the Revised Katarungang
employer-employee relations (Montoya vs. Escayo, et al., 171 Pambarangay Law and its Implementing Rules and Regulations,
SCRA 442; Art. 226, Labor Code, as amended, which grants as a pre-condition to judicial action, particularly whether the
original and exclusive jurisdiction over conciliation and mediation of certification to file action attached to the records of the case
disputes, grievances or problems to certain offices of the comply with the requirements hereinabove enumerated in par. II;
Department of Labor and Employment);
12. Actions to annul judgment upon a compromise, IV. A case filed in court without compliance with prior Barangay
which may be filed directly in court (See Sanchez vs. Tupaz, 158 conciliation which is a pre-condition for formal adjudication (Sec.
SCRA 459). 412[a] of the Revised Katarungang Pambarangay Law) may be
dismissed upon motion of defendant/s, not for lack of jurisdiction of
II. Under the provisions of R.A. 7160 on Katarungang the court but for failure to state a cause of action or prematurity
Pambarangay conciliation, as implemented by the Katarungang (Royales vs. IAC, 127 SCRA 470; Gonzales vs. CA, 151 SCRA
Pambarangay Rules and Regulations promulgated by the 289), or the court may suspend proceedings upon petition of any

The Apocryphal Maggots:


Rainier, Chrisgel, Corina, Geoffry, Grace and Sylvie Blanche
The Flibbertigibbet Worms:
Golda, Gladys and Melyjane
CA VE AT: By simply r ea ding this re vi ew er a t the end o f the sem este r wi ll (hopef ully) guar antee you r pas sing this cou rse. D rink mode rat ely .
Amusin S C A r y
gly
THE C2005 LOCAL GOVERNMENT REVIEWER - 274 -
party under Sec. 1, Rule 21 of the Rules of Court; and refer the Uy vs. Contreras (1994)
case motu propio to the appropriate Barangay authority, applying
by analogy Sec. 408[g], 2nd par., of the Revised Katarungang Facts:
Pambarangay Law which reads as follows: Atayde et al filed a complaint w/ the brgy ca[pt for alleged injuries
"The Court in which non-criminal cases not falling within the inflicted by Uy. Confrontation was scheduled by the Brgy Capt but
authority of the Lupon under this Code are filed may at any time only Uy appeared so it was reset. Thereafter, Atayde filed 2 infos
before trial, motu proprio refer the case to the Lupon concerned for for slight physical injuries. Uy filed MTD for non-compliance w/
amicable settlement. requirement in PD 1508 on prior referral to the Lupong
Tagapamayapa. Contreras denied MTD on the ground that the
accused has already waived the right to a reconciliation
Morata vs. Go (1983) proceedings.

Facts: Issue: WON there was compliance w/ PD 1508


Go filed in the CFI a complaint against Morata for recovery of a Held:
sum of money plus damages. Morata filed MTD on the ground of Yes. Katarungang pambarangay was originally governed by
failure to comply w/ PD1508 as well as absence of certification by PD1508 but was expressly repealed by the revision in the 1991
the Lupon or Pangkat Secretary. Judge denied MTD holding that LGC. Sec of Justice promulgated Rules on KP to implement
PD1508 applies only to cases falling under JD of inferior courts. revised law – with 3 new significant features:
Since the amount involved falls under JD of CFI, such law does nt a. necessary broadening of the jurisdiction of the lupon – authority
apply. in criminal offenses increased up to those offenses punishable by
imprisonment not exceeding 1year or fine not exceeding 5,000P
Issue: WON PD 1508 applies to actions not only by MTC’s but (from PD1508’s 30days and 200P)
also by CFI’s (Now RTC) b. appropriate civil and criminal cases arising from incidents
Held: occurring in workplaces or institutions of learning shall be brought
Under the Katarungang Pambarangay Law, the Lupon has in barangay where workplace is located (Sec409(d), LGC) –
authority to settle amicably all types of disputes involving c. 60day period for suspension of prescriptive periods – to
parties who actually reside in the same city or municipality. maximize effectiveness of mediation, conciliation, or arbitration
The law makes no distinction whatsoever with respect to the process and to afford parties sufficient time to cool off and face
classes of civil disputes that should be compromised at the each other with less emotionalism and more objectivity.
barangay level. To say that the authority of the Lupon is limited to
cases exclusively cognizable by the inferior courts is to lose sight
of its objective which is to ease congestion of dockets and worse, it Diu vs. CA (1995)
would make the law a self-defeating one. (as parties can bloat up
claim to make it go beyond JD of inferior courts) Facts:
Pagba purchased on credit various articles of merchandise
from Diu's store but failed to pay despite repeated demands.
Mendova vs. Afable (2002) DIU brought the matter before the Barangay Chairman and the
latter set the case for hearing, When the case was again set for
Facts: hearing, the parties appeared but they failed to reach an
Mendova had filed a complaint for slight physical injuries against amicable settlement. Accordingly, the barangay chairman
Palada with the office of the Brgy Chairman but the parties issued a Certification to File Action. DIU then filed their
failed to reach an amicable settlement as certified by the brgy complaint for a sum of money before the MTC but it was
chairman. Mendova then filed with MCTC the complaint for dismissed. RTC reversed. CA reversed RTC holding that there
slight physical injuries. Afable dismissed ruling that the action had been no compliance w/ PD 1508.
has already prescribed (without considering that the filing of Issue: WON there was substantial compliance w/ 1508
the case with the barangay tolled the prescriptive period. Held:
Mendova filed an administrative complaint against MCTC judge, There was substantial compliance with the law. It is noteworthy
Afable, for ignorance of the law. that under Section 412 of the Local Government Code
aforequoted, the confrontation before the lupon chairman OR the
Issue: WON filing w/ the brgy tolls the prescriptive period pangkat is sufficient compliance with the pre-condition for filing the
Held: case in court. IN CAB, while no pangkat was constituted, it is not
Yes. LGC provides that the filing with the barangay interrupts denied that the parties met at the office of the barangay chairman
the prescriptive period, but it starts to run again upon receipt by for possible settlement. The efforts of the barangay chairman,
the complainant of the Certification to File Action issued by however, proved futile as no agreement was reached.
the Pangkat Secretary.
NOTES:

The Apocryphal Maggots:


Rainier, Chrisgel, Corina, Geoffry, Grace and Sylvie Blanche
The Flibbertigibbet Worms:
Golda, Gladys and Melyjane
CA VE AT: By simply r ea ding this re vi ew er a t the end o f the sem este r wi ll (hopef ully) guar antee you r pas sing this cou rse. D rink mode rat ely .
Amusin S C A r y
gly
THE C2005 LOCAL GOVERNMENT REVIEWER - 275 -
SIR: But the SC Circular require us to go to Pangkat, therfor Looking at Secs. 2 and 3 of the LGC, it is clear that the Lupon shall
MANDATORY. have no jurisdiction over disputes where the parties are not actual
residents of the same city or municipality, except where the
Sec 410- is construed as mandatory barangays in which they actually reside adjoin each other.
V
Sec 412- not meant to say that you can go to court straight after It is clear even from the Katarungang Pambarangay Rules (Rule
brgy VI) that recourse to barangay conciliation proceedings is not
necessary where the parties do not reside in the same municipality
or city or in adjoining barangays.
Boleyley vs. Villanueva (1999)
Heirs of Vinzons vs. CA (1999)
Facts:
Boleyley filed with the RTC-Baguio, a complaint against Surla for
collection of a sum of money. Surla filed MTD on the ground that Facts:
petitioner did not comply with the Revised Katarungan There are 3 ejectment cases filed. The first 2 cases underwent
Pambarangay Law requiring as a condition for the filing of a barangay conciliation proceedings. The 3rd case did not. MTC and
complaint in court referral of the matter to the barangay lupon RTC ordered lessee to vacate premises and pay rent. CA reversed
chairman or the pangkat, for conciliation or settlement. citing failure to Law (PD 1508); and lack of evidence of prior
demand to vacate before instituting the complaint.
Boleyley opposes MTD on the ground that private respondent was
not a resident of Baguio City so that the dispute involving the Issue1: WON there was brgy conciliation proceedings
parties was not within the authority of the lupon to bring together Held:
for conciliation or settlement. Yes, the case was correctly dismissed the for failure of the
plaintiffs, the petitioners herein, to avail of the barangay
Issue: WON the case should’ve been referred to brgy conciliation conciliation process under PD 1508, preliminary to judicial
Held: recourse. There is no clear showing that it was brought before the
No. In the complaint filed by petitioner with the RTC, it is obvious Barangay Lupon or Pangkat of Barangay Daet, Camarines Norte,
that the parties do not reside in the same city or municipality, and where the parties reside and the property subject of the case is
hence, the dispute is excepted from the requirement of referral to situated, as there is no barangay certification to file action attached
the barangay lupon or pangkat for conciliation or settlement prior to to the complaint.
filing with the court.
Issue2: WON the previous brgy conciliation proceedings are
The allegation of defendant's actual residence would have been applicable to the present case
ideal to determine venue, which is plaintiff's choice of either his Held:
place of residence or that of the defendant or any of the principal No. The 3rd case for unlawful detainer should have been coursed
defendants. "In procedural law, however, specifically for purposes first to the barangay court. Petitioners cannot rely on the barangay
of venue it has been held that the residence of a person is his conciliation proceedings held in the other cases and consider the
personal, actual or physical habitation or his actual residence or same as compliance with the law.
place of abode, which may not necessarily be his legal residence
or domicile provided he resides therein with continuity and
consistency. Nevertheless, the complaint clearly implies that the
parties do not reside in the same city or municipality. NOTES:
Sir: Evidence that conciliation proceedings have failed does not
have to be presented as long as the certification to file action
Vercide vs. Judge Priscilla T. Hernandez (2000) from barangay is attached to the pleadings.

Q: can there be conciliation after filing of case in court?


Facts:
A: yes, court will dismiss the case w/out prejudice.
Without referral to the Lupong Tagapamayapa, Vercide (Dipolog
City) filed a complaint for recovery of possession of a piece of land
located in Misamis Oriental against Galleros (Misamis Or). Judge
Hernandez ordered the dismissal of the case for lack of said Idolor vs. CA (2001)
referral
Facts:
Held: To secure a loan Idolor executed in favor of Gumersindo de
Where parties do not reside in the same city or municipality or in Guzman a Deed of Real Estate Mortgage with right of extrajudicial
adjoining barangays, there is no requirement for them to submit foreclosure upon failure to redeem the mortgage. De Guzman filed
their dispute involving real property to the Lupong Tagapamayapa.

The Apocryphal Maggots:


Rainier, Chrisgel, Corina, Geoffry, Grace and Sylvie Blanche
The Flibbertigibbet Worms:
Golda, Gladys and Melyjane
CA VE AT: By simply r ea ding this re vi ew er a t the end o f the sem este r wi ll (hopef ully) guar antee you r pas sing this cou rse. D rink mode rat ely .
Amusin S C A r y
gly
THE C2005 LOCAL GOVERNMENT REVIEWER - 276 -
a complaint against Idolor before the Office of the Bgy. Capt. which (a) There shall be in every barangay a sangguniang
resulted in a “Kasunduang Pag-aayos”, kabataan to be composed of a chairman, seven (7) members, a
secretary, and a treasurer.
Held: (b) A sangguniang kabataan official who, during his term
The compromise agreement has the force and effect of a final of office, shall have passed the age of twenty-one (21) years
judgment. The Kasanduan merely gave life to the mortgage shall be allowed to serve the remaining portion of the term for
contract, which was then more than 2 years overdue. Evidently, it which he was elected.
was executed to facilitate easy compliance by mortgagor with her
mortgage obligation. It is not incompatible and can stand together Sec424. Katipunan ng Kabataan. — The katipunan ng
with the mortgage contract. kabataan shall be composed of all citizens of the Philippines
actually residing in the barangay for at least six (6) months, who
Q&A are fifteen (15) but not more than twenty-one (21) years of age,
and who are duly registered in the list of the sangguniang
Question kabataan or in the official barangay list in the custody of the
My husband and I occasionally use our camcorder to record our barangay secretary.
lovemaking sessions. We both enjoy viewing them later, and
Sec425. Meetings of the Katipunan ng Kabataan. — The
now have a small "greatest hits" collection. Recently, I started
katipunan ng kabataan shall meet at least once every three (3)
wondering about it. Is there anything we should be worried months, or at the call of the chairman of the sangguniang
about, or can any harm come from doing this? kabataan or upon written petition of at least one-twentieth (1/20)
of its members, to decide on important issues affecting the youth
Answer of the barangay.
Since the advent of affordable camcorders (and now digital
recorders), couples have enjoyed making movies of themselves Sec426. Powers and Functions of the Sangguniang
engaged in various behaviors, including sex. It certainly can be a Kabataan. — The sangguniang kabataan shall:
great turn-on, sort of like making your own personal porn flick. (a) Promulgate resolutions necessary to carry out the
There is no harm in videotaping yourself and your husband objectives of the youth in the barangay in accordance with the
engaging in sexual activities. Of course, some couples are applicable provisions of this Code;
surprised by what they see. Few of us look as good as the porn (b) Initiate programs designed to enhance the social,
political, economic, cultural, intellectual, moral, spiritual, and
stars who are carefully made-up and filmed under perfect
physical development of the members;
lighting. Plus, most individuals cannot keep up with the sexual
(c) Hold fund-raising activities, the proceeds of which
athletics of these professionals. Just keep a good sense of humor, shall be tax-exempt and shall accrue to the general fund of the
and remember that comparing yourself negatively would be sangguniang kabataan: Provided, however, That in the
akin to making a home movie of your family playing basketball appropriation thereof, the specific purpose for which such
and then being embarrassed that you can't shoot as well as activity has been held shall be first satisfied;
Michael Jordan. (d) Create such bodies or committees as it may deem
The biggest thing you should be worried about is what happens necessary to effectively carry out its programs and activities;
to these tapes. It is not unheard of for people to accidentally (e) Submit annual and end-of-term reports to the
return the wrong video to Blockbusters, or for kids looking for a sangguniang barangay on their projects and activities for the
blank tape to stumble upon mommy and daddy swinging from survival and development of the youth in the barangay;
the chandelier. I highly suggest that you lock up your tapes in a (f) Consult and coordinate with all youth organizations in
the barangay for policy formulation and program
secure place and make sure they are well labeled and kept track
implementation;
of. You also might want to make a deal with your spouse that
(g) Coordinate with the appropriate national agency for
under no circumstances will either of you ever make a copy or the implementation of youth development projects and programs
do anything with these tapes without the other's permission. at the national level;
But aside from these safeguards, enjoy yourselves and just hope (h) Exercise such other powers and perform such other
that you don't have a nasty divorce and find your face (or much duties and functions as the sangguniang barangay may
worse) plastered all over the Internet. determine or delegate; and
(i) Exercise such other powers and perform such other
duties and functions as may be prescribed by law or ordinance.
Sangguniang Kabataan
Sec427. Meetings of the Sangguniang Kabataan. — The
sangguniang kabataan shall meet regularly once a month on the
CHAPTER VIII: Sangguniang Kabataan
date, time, and place to be fixed by the said sanggunian.
Sec423. Creation and Election. —
Special meetings may be called by the sangguniang kabataan
chairman or any three (3) of its members by giving written notice

The Apocryphal Maggots:


Rainier, Chrisgel, Corina, Geoffry, Grace and Sylvie Blanche
The Flibbertigibbet Worms:
Golda, Gladys and Melyjane
CA VE AT: By simply r ea ding this re vi ew er a t the end o f the sem este r wi ll (hopef ully) guar antee you r pas sing this cou rse. D rink mode rat ely .
Amusin S C A r y
gly
THE C2005 LOCAL GOVERNMENT REVIEWER - 277 -
to all members of the date, time, place and agenda of the coordination with the barangay secretary and the COMELEC;
meeting at least one (1) day in advance. Notices of regular or and
special meetings shall be furnished the punong barangay and (d) Perform such other duties and discharge such other
the sangguniang barangay. functions as the chairman of the sangguniang kabataan may
A majority of the members of the sangguniang kabataan shall prescribe or direct.
constitute a quorum.
Sec433. Sangguniang Kabataan Treasurer. — The
Sec428. Qualifications. — An elective official of the sangguniang kabataan treasurer shall:
sangguniang kabataan must be a citizen of the Philippines, a (a) Take custody of all sangguniang kabataan property
qualified voter of the katipunan ng kabataan, a resident of the and funds not otherwise deposited with the city or municipal
barangay for at least one (1) year immediately prior to election, treasurer;
at least fifteen (15) years but not more than twenty- one (21) (b) Collect and receive contributions, monies, materials,
years of age on the day of his election, able to read and write and all other sources intended for the sangguniang kabataan
Filipino, English, or the local dialect, and must not have been and katipunan ng kabataan;
convicted of any crime involving moral turpitude. (c) Disburse funds in accordance with an approved
budget of the sangguniang kabataan;
Sec429. Term of Office. — The sangguniang kabataan (d) Certify to the availability of funds whenever
chairman and members shall hold office for a period of three (3) necessary;
years, unless sooner removed for cause as provided by law, (e) Submit to the sangguniang kabataan and to the
permanently incapacitated, die or resign from office. sangguniang barangay certified and detailed statements of
actual income and expenditures at the end of every month; and
Sec430. Sangguniang Kabataan Chairman. — The (f) Perform such other duties and discharge such other
registered voters of the katipunan ng kabataan shall elect the functions as the chairman of the sangguniang kabataan may
chairman of the sangguniang kabataan who shall automatically direct.
serve as an ex officio member of the sangguniang barangay
upon his assumption to office. As such, he shall exercise the Sec434. Privileges of Sangguniang Kabataan Officials. —
same powers, discharge the same duties and functions, and The sangguniang kabataan chairman shall have the same
enjoy the same privileges as the regular sangguniang barangay privileges enjoyed by other sangguniang barangay officials
members, and shall be the chairman of the committee on youth under this Code subject to such requirements and limitations
and sports development in the said sanggunian. provided herein. During their incumbency, sangguniang
kabataan officials shall be exempt from payment of tuition and
Sec431. Powers and Duties of the Sangguniang Kabataan matriculation fees while enrolled in public tertiary schools,
Chairman. — In addition to the duties which may be assigned including state colleges and universities. The national
to him by the sangguniang barangay, the sangguniang kabataan government shall reimburse said college or university the
chairman shall: amount of the tuition and matriculation fees: Provided, That, to
(a) Call and preside over all meetings of the katipunan ng qualify for the privilege, the said officials shall enroll in a state
kabataan and the sangguniang kabataan; college or university within or nearest their area of jurisdiction.
(b) Implement policies, programs, and projects within his
jurisdiction in coordination with the sangguniang barangay; Sec435. Succession and Filling of Vacancies. —
(c) Exercise general supervision over the affairs and (a) In case a sangguniang kabataan chairman refuses to
activities of the sangguniang kabataan and the official conduct assume office, fails to qualify, is convicted of a felony, voluntarily
of its members, and such other officers of the sangguniang resigns, dies, is permanently incapacitated, is removed from
kabataan within his jurisdiction; office, or has been absent without leave for more than three (3)
(d) With the concurrence of the sangguniang kabataan, consecutive months, the sangguniang kabataan member who
appoint from among the members of the sangguniang kabataan, obtained the next highest number of votes in the election
the secretary and treasurer and such other officers as may be immediately preceding shall assume the office of the chairman
deemed necessary; and for the unexpired portion of the term, and shall discharge the
(e) Exercise such other powers and perform such other powers and duties, and enjoy the rights and privileges
duties and functions as may be prescribed by law or ordinance. appurtenant to the office. In case the said member refuses to
assume the position or fails to qualify, the sangguniang member
Sec432. Sangguniang Kabataan Secretary. — The obtaining the next highest number of votes shall assume the
sangguniang kabataan secretary shall: position of the chairman for the unexpired portion of the term.
(a) Keep all records of the katipunan ng kabataan and (b) Where two (2) or more sangguniang kabataan
sangguniang kabataan; members obtained the same next highest number of votes, the
(b) Prepare and keep the minutes of all meetings of the other sangguniang kabataan members shall conduct an election
katipunan ng kabataan and sangguniang kabataan; to choose the successor to the chairman from among the said
(c) Prepare all forms necessary for the conduct of members.
registrations, elections, initiatives, referenda, or plebiscites, in

The Apocryphal Maggots:


Rainier, Chrisgel, Corina, Geoffry, Grace and Sylvie Blanche
The Flibbertigibbet Worms:
Golda, Gladys and Melyjane
CA VE AT: By simply r ea ding this re vi ew er a t the end o f the sem este r wi ll (hopef ully) guar antee you r pas sing this cou rse. D rink mode rat ely .
Amusin S C A r y
gly
THE C2005 LOCAL GOVERNMENT REVIEWER - 278 -
(c) After the vacancy shall have been filled, the (b) The vice-president of the pederasyon whose
sangguniang kabataan chairman shall call a special election to president has been elected as president of a higher pederasyon
complete the membership of said sanggunian. Such shall serve as ex-officio member of the sanggunian concerned
sangguniang kabataan member shall hold office for the without need of further appointment.
unexpired portion of the term of the vacant seat. cd i (c) The pederasyon president or vice-president, as the
(d) In case of suspension of the sangguniang kabataan case may be, shall be the chairman of the committee on youth
chairman, the successor, as determined in subsections (a) and and sports development of the sanggunian concerned.
(b) of this Section shall assume the position during the period of
such suspension. CHAPTER X: Linggo ng Kabataan
Sec439. Observance of Linggo ng Kabataan. —
CHAPTER IX: Pederasyon ng mga Sangguniang Kabataan (a) Every barangay, municipality, city and province shall,
Sec436. Pederasyon ng mga Kabataan. — in coordination with the pederasyon ng mga sangguniang
(a) There shall be an organization of all the pederasyon kabataan at all levels, conduct an annual activity to be known as
ng mga sangguniang kabataan to be known as follows: the Linggo ng Kabataan on such date as shall be determined by
(1) in municipalities pambayang pederasyon ng mga the Office of the President.
sangguniang kabataan; (b) The observance of the Linggo ng Kabataan shall
(2) in cities, panlungsod na pederasyon ng mga include the election of the counterparts of all local elective and
sangguniang kabataan; appointive officials, as well as heads of national offices or
(3) in provinces, panlalawigang pederasyon ng mga agencies stationed or assigned in the territorial jurisdiction of the
kabataan; local government unit, among in-school and community youth
(4) in special metropolitan political subdivisions, residing in the local government unit concerned from ages
pangmetropolitan pederasyon ng mga sangguniang thirteen (13) to seventeen (17). During said week, they shall
kabataan; and hold office as boy and girl officials and shall perform such duties
(5) on the national level pambansang pederasyon ng mga and conduct such activities as may be provided in the ordinance
sangguniang kabataan. enacted pursuant to this Chapter
(b) The pederasyon ng mga sangguniang kabataan shall,
at all levels, elect from among themselves the president, vice-
president and such other officers as may be necessary and shall RA 7808 (1994) – An Act Resetting The Elections Of SK
be organized in the following manner: Officials To The First Monday Of May 1996, And Every (3) Years
(1) The panlungsod and pambayang pederasyon shall be Thereafter, Amending For The Purpose Section 532(A) Of
composed of the sangguniang kabataan chairmen of RA7160 LGC of 1991
barangays in the city or municipality, respectively;
(2) The panlalawigang pederasyon shall be composed of Sec1. Section 532(a) of Republic Act No. 7160, otherwise
presidents of the panlungsod and pambayang known as the Local Government Code of 1991, is hereby
pederasyon; amended to read as follows: b
(3) The pangmetropolitang pederasyon shall be composed "Sec. 532. Elections for the Sangguniang Kabataan. —
of presidents of the panlungsod and pambayan "(a) The first elections for the sangguniang kabataan to be
pederasyon; conducted under this Code shall be held thirty (30) days after
(c) The elected presidents of the pederasyon at the the next local elections: Provided, That, the regular elections for
provincial, highly urbanized city, and metropolitan political the sangguniang kabataan shall be held on the first Monday of
subdivision levels shall constitute the pambansang katipunan ng May 1996: Provided, further, That the succeeding regular
mga sangguniang kabataan. elections for the sangguniang kabataan shall be held every
three (3) years thereafter: Provided, finally, That the national,
Sec437. Constitution and By-Laws. — The term of office, special metropolitan, provincial, city and municipal federations of
manner of election, removal and suspension of the officers of the sangguniang kabataan shall conduct the election of their
the pederasyon ng mga sangguniang kabataan at all levels shall respective officers thirty (30) days after the May 1996
be governed by the constitution and by-laws of the pederasyon sangguniang kabataan elections on dates to be scheduled by
in conformity with the provisions of this Code and national the Commission on Elections.
policies on youth. "The conduct of the sangguniang kabataan elections shall be
under the supervision of the Commission on Elections.
Sec438. Membership in the Sanggunian. — "The Omnibus Election Code shall govern the elections of the
(a) A sangguniang kabataan chairman shall, upon sangguniang kabataan."
certification of his election by the COMELEC and during his
tenure of office is elected as pederasyon president, serve as an Sec2. All laws, decrees, orders or administrative rules and
ex-officio member of the sangguniang panlalawigan, regulations or any part thereof which may be inconsistent with
sangguniang panlungsod, and sangguniang bayan, as the case the provisions of this Act are hereby repealed, amended or
may be, without need of further appointment. modified accordingly.

The Apocryphal Maggots:


Rainier, Chrisgel, Corina, Geoffry, Grace and Sylvie Blanche
The Flibbertigibbet Worms:
Golda, Gladys and Melyjane
CA VE AT: By simply r ea ding this re vi ew er a t the end o f the sem este r wi ll (hopef ully) guar antee you r pas sing this cou rse. D rink mode rat ely .
Amusin S C A r y
gly
THE C2005 LOCAL GOVERNMENT REVIEWER - 279 -
Sec3. This Act shall take effect immediately after its (m) To register, establish and/or facilitate and help in the
publication in at least two (2) national newspapers of general establishment of the youth organizations and youth-serving
circulation. organizations;
(n) To participate in international youth fora, symposia
and organizations such as the International Youth Forum, Asian
RA 8044 (1995) – An Act Creating The National Youth Youth Council, ASEAN Youth Forum, United Nations
Commission, Establishing A National Comprehensive And Commission for International Youth Year (IYY) and other similar
Coordinated Program On Youth Development, Appropriating bodies;
Funds Therefor, And For Other Purposes (o) To provide training and a national secretariat for the
Sangguniang Kabataan National Federation pursuant to R.A No.
Sec10. Functions of the Commission. — The Commission 7160, otherwise known as the Local Government Code;
shall have the following functions: (p) To submit an annual report on the implementation of
(a) To formulate and initiate the national policy or policies this Act to the President and to Congress; and
on youth; (q) To perform such other functions as may be necessary
(b) To plan, implement, and oversee a national integrated to effectively and efficiently carry out the provisions of this Act.
youth promotion and development program;
(c) To establish a consultative mechanism which shall
provide a forum for continuing dialogue between the RA 9164 (2002) – An Act Providing For Synchronized Barangay
government and the youth sector on the proper planning and And SK Elections, Amending RA7160, As Amended, "LGC Of
evaluation of policies, programs and projects affecting the youth, 1991", And For Other Purposes
convening for the purpose, representatives of all youth
organizations and institutions, including the sangguniang Sec1. Date of Election. — There shall be synchronized barangay
kabataan from barangay, municipal, city, provincial and national and sangguniang kabataan elections which shall be held on July
levels; 15, 2002. Subsequent synchronized barangay and sangguniang
(d) To assist and coordinate with governmental and kabataan elections shall be held on the last Monday of October
nongovernmental organizations or institutions in the and every three (3) years thereafter.
implementation of all laws, policies, programs and projects
relative to youth promotion and development; Sec2. Term of Office. — The term of office of all barangay and
(e) To seek or request the assistance and support of any sangguniang kabataan officials after the effectivity of this Act
government agency, office or instrumentality including shall be three (3) years.
government-owned or -controlled corporations, local No barangay elective official shall serve for more than three (3)
government units as well as nongovernmental organizations or consecutive terms in the same position: Provided, however, that
institutions in pursuance of its policies, programs and projects; the term of office shall be reckoned from the 1994 barangay
(f) To conduct scientific interdisciplinary and policy- elections. Voluntary renunciation of office for any length of time
oriented researches and studies on youth-related matters, as shall not be considered as an interruption in the continuity of
well as trainings, seminars and workshops that will enhance the service for the full term for which the elective official was
skills and leadership potentials of the youth, instilling in them elected.
nationalism and patriotism, with particular emphasis on Filipino
culture and values; Sec3. Registration. — For purposes of the July 15, 2002
(g) To establish and maintain linkages with international synchronized barangay and sangguniang kabataan elections
youth and youth-serving organizations or institutions and provided under this Act, a special registration of voters for the
counterpart agencies of foreign governments in order to sangguniang kabataan shall be fixed by the Commission on
facilitate and ensure the participation of Filipino youth in elections (COMELEC). Subsequent registration of barangay and
international functions and affairs; sangguniang kabataan voters shall be governed by RA8189.
(h) To administer youth exchange programs as well as
monitor and coordinate all foreign-sponsored youth programs Sec4. Assumption of Office. — The term of office of the
and projects such as the Ship for Southeast Asia Youth Program barangay and sangguniang kabataan officials elected under this
and other similar exchanges and goodwill missions; Act shall commence on August 15, 2002. The term of office of
(i) To establish such organizational structures including the barangay and sangguniang kabataan officials elected in
regional offices, as may be required to effectively carry out its subsequent elections shall commence at noon of November 30,
functions; next following their election.
(j) To conduct promotion and fund-raising campaigns in
accordance with existing laws; Sec5. Hold Over. — All incumbent barangay officials and
(k) To allocate resources for the implementation of youth sangguniang kabataan officials shall remain in office unless
programs and projects; sooner removed or suspended for cause until their successors
(l) To extend and provide support or assistance to shall have been elected and qualified. The provisions of the
deserving youth and youth organizations including scholarship Omnibus Election Code relative to the failure of elections and
grants; special elections are hereby reiterated in this Act.

The Apocryphal Maggots:


Rainier, Chrisgel, Corina, Geoffry, Grace and Sylvie Blanche
The Flibbertigibbet Worms:
Golda, Gladys and Melyjane
CA VE AT: By simply r ea ding this re vi ew er a t the end o f the sem este r wi ll (hopef ully) guar antee you r pas sing this cou rse. D rink mode rat ely .
Amusin S C A r y
gly
THE C2005 LOCAL GOVERNMENT REVIEWER - 280 -
framework of detailed and comprehensive rules embodied in Res
Sec6. Section 424 of Republic Act No. 7160, otherwise known as 2499. What was left to the DILG was the enforcement of rules.
the Local Government Code of 1991, is hereby amended to
read as follows: Issue2: WON the Sec of DILG can exempt an LGU from holding
"SECTION 424. Katipunan ng Kabataan. — The katipunan elections for SK officers on Dec’92.
ng kabataan shall be composed of Filipino citizens actually Held:
residing in the barangay for at least six (6) months, who are Yes. The authority to supervise the conduct of election necessarily
fifteen (15) but less than eighteen (18) years of age on the day includes the authority to determine which KB would not be included
of the election, and who are duly registered in the list of the in the ’92 elections by virtue of Sec 532(d) of LGC. This authority
sangguniang kabataan or in the official barangay list in the was nothing more than ascertainment of fact of WON LGU had an
custody of the barangay secretary." SK election from 1988 up to the effectivity of LGC.

Sec7. Section 428 of Republic Act No. 7160, otherwise known as NOTES:
the Local Government Code of 1991, is hereby amended to Q: is this good law? Is SK elections under DILG?
read as follows: A: No. RA 9164 or Omnibus Election Code is applicable. Hence,
"Section 428. Qualifications. — An elective official of the SK elections is under COMELEC.
sangguniang kabataan must be a Filipino citizen, a qualified
voter of the katipunan ng kabataan, a resident of the barangay
for at least one (1) year immediately prior to election, at least
Associated Labor Unions vs. Letrondo (1994)
fifteen (15) years but less than eighteen (18) years of age on the
day of the election, able to read and write Filipino, English, or
the local dialect, and must not have been convicted of any crime The election for members of the SK may properly be considered
involving moral turpitude." “local election” w/in the meaning of the CBA and the day on which
it was to be held a holiday. It was a nonworking holiday as
Sec9. Applicability of Other election Laws. — The Omnibus announced in the media. Proclamation No. 118 of President
Election Code and other existing election laws, as far as Ramos declared the day as "a special day throughout the country
practicable, shall apply to barangay and sangguniang kabataan on the occasion of the Sangguniang Kabataan Elections". A
elections. "special day" is a "special holiday," as provided by the
Administrative Code of 1987. The fact that only those between 15
and 21 take part in the election for members of the SK does not
make such election any less a regular local election.
Alunan III vs. Mirasol (1997)
The phrase "general election" means regular local and national
elections. On the other hand, the term "general elections" means,
Facts: in the context of SK elections, the regular elections for members of
COMELEC issued Resolution 2499, providing guidelines for the the SK, as distinguished from the special elections for such
holding of the general elections for the SK on September 30, 1992. officers.
placing the SK elections under the direct control and supervision of
the DILG, with the technical assistance of the COMELEC.
Miguel vs. CA (1994)
Mirasol questioned the decision of the RTC nullifying an order of
the DILG w/c cancelled the general elections for the SK on Dec ’92
in Manila on the ground that the elections previously held on May Under Art 210 (g)(3) of the IRR, the election of the 1 st set of officers
’90 served the purpose of the first SK elections under LGC 1991. of the national and local chapters of the Liga cannot be held unless
a constitution and by-laws for the Liga is first adopted and ratified
Issue1: WON COMELEC can delegate the control and supervision by the barangay national assembly. This was in several opinions
of the SK elections to DILG issued by the DILG. These pertinent issues of the DILG are in the
Held: nature of executive construction and are entitled to a great weight
YES. Resolution 2499, wherein COMELEC placed the SK and respect by the court.
elections under the direct control and supervision of the DILG, did
not contravene the provision in the Consti. that the COMELEC
shall have the power to "enforce and administer all laws and Marquez vs. COMELEC (1999)
regulations relative to the conduct of an election, plebiscite,
initiative, referendum, and recall." Elections for SK officers are not Facts:
subject to the supervision of the COMELEC in the same way that Marquez & Santos ran for the position for SK Chairman. Marquez
contests involving elections of SK officials do not fall within the won. Santos filed an election protest before the MTC, on the
jurisdiction of the COMELEC. The DILG has since been in charge ground that Marquez was disqualified by age to be SK Chairman.
of SK then (KB) elections. There is no abdication of COMELEC Marquez filed a MTD. He argued that the MTC had no jurisdiction
functions because DILG supervision was to be exercised w/in the MTC denied MTD. election protest.

The Apocryphal Maggots:


Rainier, Chrisgel, Corina, Geoffry, Grace and Sylvie Blanche
The Flibbertigibbet Worms:
Golda, Gladys and Melyjane
CA VE AT: By simply r ea ding this re vi ew er a t the end o f the sem este r wi ll (hopef ully) guar antee you r pas sing this cou rse. D rink mode rat ely .
Amusin S C A r y
gly
THE C2005 LOCAL GOVERNMENT REVIEWER - 281 -
Issue: WON the MTC had jurisdiction over a disqualification case Facts:
of an SK Chairman, filed after the elections Lallave won over Zarate as SK chairman. Zarate lodged an
Held: election protest before the MTC which annulled the proclamation of
ANY contest relating to the election of members of the SK Lallave, holding eight of the 46 ballots of Lallave were declared
(including the chairman) – whether pertaining to their eligibility or marked.
the manner of election – is cognizable by the MTCs. Lallave appealed to the Comelec EN BANC, and Comelec
reversed MTC ruling
COMELEC promulgated Resolution No. 2824, §49 of which
provided that the MTCs shall have original jurisdiction over all Held:
election protect cases, whose decision shall be final. Comelec en banc has no jurisdiction. The appeal must have been
referred to a Division of the Comelec. Referring the appeal to the
§6 of COMELEC Res. No. 2824, which provides: “…Cases Comelec en banc transgressed Section 3, Article IX-C of the
involving the eligibility or qualification of candidates [of SK] shall be Constitution which provides that “The COMELEC may sit en banc
decided by the city/municipal Election officer (EO) whose decision or in division and shall promulgate its rules of procedure in order
shall be final”, applies only to the proceedings BEFORE the to expedite disposition of election cases, including pre-
election. proclamation controversies. All such election cases shall be heard
and decided in division, provided that motions for reconsideration
Hence the use of the word “candidates” in §6 and the phrase of decisions shall be decided by the COMELEC En Banc.
“winning candidates” in §49. The distinction is based on the
principle that it is the proclamation which marks off the jurisdiction
of the courts from the jurisdiction of the election officials. Baytan vs. COMELEC (2003)

NOTES: Facts:
Sir: If unhappy, go to COMELEC. Regardless of type of office? During the registration period for the May 1998 elections,
Yes! petitioners Baytan registered themselves twice – one in Precinct
No. 83-A and the other in Precinct No. 129-A. The Election Officer
(of Cavite City) forwarded copies of petitioners’ voters’ registration
records to the Provincial Election Supervisor for evaluation.
Eventually, the Law Department endorsed the case back to the
Garvida vs. Sales (1997) Provincial Election Supervisor, who then recommended the filing of
an information for double registration against petitioners. The
Facts: Comelec en banc affirmed the recommendation of the Provincial
Garvida was born on 6/11/74. On the day she registered as a voter Eleciton Supervisor. MFR denied.
for SK elections, she was 21 yrs and 9 mos old. When she
assumed office on 6/1/96 she was 10 days away from turning 22. Issue: WON THE COMELEC EN BANC’S ASSUMPTION OF
ORIGINAL JURISDICTION OVER THE CASE VIOLATEDTHE
Issue: WON petitioner is qualified to be a voter and a member of CONSTITUTION
the SK
Held: Held:
Garvida is qualified to be a member and to vote but not as a No. Comelec exercises both administrative and quasi-judicial
candidate for election. LGC: distinction between the maximum age function. As regards Comelec’s administrative powers, the
of a member in the Katipunan ng Kabataan and the maximum age Constitution does not prescribe how they should be exercised, i.e.
of an elective SK official. Section 424 of the Code sets a member's whether en banc or in division. The Constitution merely vests the
maximum age at 21 years only. On the other hand, Section 428 Comelec’s administrative powers in the “COMELEC while
provides that the maximum age of an elective SK official is 21 providing that the Comelec may sit en banc or in division. As
years old "on the day of his election." regards its quasi-judicial, Comelec is mandated to hear and decide
The phrase "not more than 21 years of age" means not over 21 cases first by division, and then, upon MFR, by the Comelec en
years, not beyond 21 years. It means 21 365-day cycles. It does banc.
not mean 21 years and one or some days or a fraction of a year
because that would be more than 21 365-day cycles. "Not more Clearly, Comelec en banc can act directly on matters falling within
than 21 years old" is not equivalent to "less than 22 years old," its administrative powers. The prosecution of election law violators
contrary to petitioner's claims. The law does not state that the involves the exercises of the Comelec’s ADMINISTRATIVE
candidate be less than 22 years on election day. POWERS. There is no constitutional requirement that the filing of
the criminal information be first decided by any of the divisions of
the Comelec.
Zarate vs. COMELEC (1999)
Montesclaros vs. COMELEC (2002)

The Apocryphal Maggots:


Rainier, Chrisgel, Corina, Geoffry, Grace and Sylvie Blanche
The Flibbertigibbet Worms:
Golda, Gladys and Melyjane
CA VE AT: By simply r ea ding this re vi ew er a t the end o f the sem este r wi ll (hopef ully) guar antee you r pas sing this cou rse. D rink mode rat ely .
Amusin S C A r y
gly
THE C2005 LOCAL GOVERNMENT REVIEWER - 282 -
1. petition for annulment of permanent list of voters
Facts: 2. petitions for accreditation of citizen’s arms
PD684 established Kabataang Barangay (SK now) which was
composed of all barangay residents less than 18yrs old without II. Election Matters
any specified minimum age, to provide its members opportunity to 1. Recall elections
express their views and opinions on issues of transcendental 2 Initiative
importance. 3. Pleadings
Dec4, 1992 – 1st SK elections. It was Reset. 4. Other contentious matters in exercise of administrative,
RA9164 providing that voters and candidates for SK elections must supervisory, and regulatory powers of the Constitution (not falling
be “at least 15 but less than 18years of age on day of election”, in the above enumeration)
and a synchronized SK and Barangay elections on July15, 2002.
May6,2002 SK elections and May13 Barangay elections were not III. Motu Proprio action of the Commission in disqualifying
held as scheduled. candidates.
COMELEC promulgated Resolution4846 for conduct of
July15,2002 synchronized elections.
Municipality – LGC
Held:
Court has no power to dictate to Congress the object or subject of CHAPTER I: Role and Creation of the Municipality
bills to be enacted into law – it would destroy the system of checks Sec440. Role of the Municipality. — The municipality,
and balances in the Constitution. With the passage of RA9164, the consisting of a group of barangays, serves primarily as a
right originally conferred to those 15-21 has been limited to those general purpose government for the coordination and delivery of
who on the date of SK elections are 15-18. The new law restricted basic, regular and direct services and effective governance of
the membership – they no longer fall within the classification and the inhabitants within its territorial jurisdiction.
have ceased to be SK members.
Sec441. Manner of Creation. — A municipality may be
SK membership is not a property right protected by the created, divided, merged, abolished, or its boundary
Constitution because it is a mere statutory right conferred by law. substantially altered only by an Act of Congress and subject to
Congress has the power to prescribe the qualifications for SK the approval by a majority of the votes cast in a plebiscite to be
membership. It may amend at any time the law to change or even conducted by the COMELEC in the local government unit or
withdraw the statutory right. State policy to encourage the youth’s units directly affected. Except as may otherwise be provided in
involvement in public affairs refers to those who belong to the class the said Act, the plebiscite shall be held within one hundred
of people defined as the youth – and Congress has defined and twenty (120) days from the date of its effectivity.
limited it.
Sec442. Requisites for Creation. —
RA9164 is Constitutional! (a) A municipality may be created if it has an average
annual income, as certified by the provincial treasurer, of at least
Two million five hundred thousand pesos (P2,500,000.00) for
See COMELEC Resolution No. 00-0046 (2000) the last two (2) consecutive years based on the 1991 constant
prices; a population of at least twenty-five thousand (25,000)
Thus, after the above rulings, it is clear that the limited instances inhabitants as certified by the National Statistics Office; and a
where the SC approved the assumption of jurisdiction by the contiguous territory of at least fifty (50) square kilometers as
COMELEC En Banc as follows: certified by the Lands Management Bureau: Provided, That the
creation thereof shall not reduce the land area, population or
I. Special Action Cases income of the original municipality or municipalities at the time of
1. Petitions for postponement or suspension of elections. said creation to less than the minimum requirements prescribed
2. Petitions to declare failure of election or to annul election herein.
(b) The territorial jurisdiction of a newly-created
II. Special cases municipality shall be properly identified by metes and bounds.
1. petition to declare illegal the composition of proceedings of local The requirement on land area shall not apply where the
canvassers municipality proposed to be created is composed of one (1) or
2. petitions to correct manifest errors more islands. The territory need not be contiguous if it
comprises two (2) or more islands.
Further, these are petitions which require actions of the (c) The average annual income shall include the income
commission en banc on the basis of the constitutional mandate to accruing to the general fund of the municipality concerned,
enforce and administer all laws relative to the conduct of elections, exclusive of special funds, transfers and non-recurring income.
plebescites and initiative to wit: (d) Municipalities existing as of the date of the effectivity
of this Code shall continue to exist and operate as such. Existing
I. Special proceedings cases municipal districts organized pursuant to presidential issuances

The Apocryphal Maggots:


Rainier, Chrisgel, Corina, Geoffry, Grace and Sylvie Blanche
The Flibbertigibbet Worms:
Golda, Gladys and Melyjane
CA VE AT: By simply r ea ding this re vi ew er a t the end o f the sem este r wi ll (hopef ully) guar antee you r pas sing this cou rse. D rink mode rat ely .
Amusin S C A r y
gly
THE C2005 LOCAL GOVERNMENT REVIEWER - 283 -
or executive orders and which have their respective set of (1) Exercise general supervision and control over all
elective municipal officials holding office at the time of the programs, projects, services, and activities of the
effectivity of this Code shall henceforth be considered as regular municipal government, and in this connection, shall:
municipalities. (i) Determine the guidelines of municipal policies
and be responsible to the sangguniang bayan for the
CHAPTER II: Municipal Officials in General program of government;
Sec443. Officials of the Municipal Government. — (ii) Direct the formulation of the municipal
(a) There shall be in each municipality a municipal mayor, development plan, with the assistance of the municipal
a municipal vice-mayor, sangguniang bayan members, a development council, and upon approval thereof by the
secretary to the sangguniang bayan, a municipal treasurer, a sangguniang bayan, implement the same;
municipal assessor, a municipal accountant, a municipal budget (iii) At the opening of the regular session of the
officer, a municipal planning and development coordinator, a sangguniang bayan for every calendar year and, as may
municipal engineer/building official, a municipal health officer be deemed necessary, present the program of
and a municipal civil registrar. government and propose policies and projects for the
(b) In addition thereto, the mayor may appoint a consideration of the sangguniang bayan as the general
municipal administrator, a municipal legal officer, a municipal welfare of the inhabitants and the needs of the municipal
agriculturist, a municipal environment and natural resources government may require;
officer, a municipal social welfare and development officer, a (iv) Initiate and propose legislative measures to
municipal architect, and a municipal information officer. the sangguniang bayan and, from time to time as the
(c) The sangguniang bayan may: situation may require, provide such information and data
(1) Maintain existing offices not mentioned in subsections needed or requested by said sanggunian in the
(a) and (b) hereof; performance of its legislative functions;
(2) Create such other offices as may be necessary to carry (v) Appoint all officials and employees whose
out the purposes of the municipal government; or salaries and wages are wholly or mainly paid out of
(3) Consolidate the functions of any office with those of municipal funds and whose appointments are not
another in the interest of efficiency and economy. otherwise provided for in this Code, as well as those he
(d) Unless otherwise provided herein, heads of may be authorized by law to appoint;
departments and offices shall be appointed by the municipal (vi) Upon authorization by the sangguniang bayan,
mayor with the concurrence of the majority of all the represent the municipality in all its business transactions
sangguniang bayan members, subject to civil service law, rules and sign on its behalf all bonds, contracts, and
and regulations. The sangguniang bayan shall act on the obligations, and such other documents made pursuant to
appointment within fifteen (15) days from the date of its law or ordinance;
submission; otherwise, the same shall be deemed confirmed. (vii) Carry out such emergency measures as may
(e) Elective and appointive municipal officials shall be necessary during and in the aftermath of man-made
receive such compensation, allowances and other emoluments and natural disasters and calamities;
as may be determined by law or ordinance, subject to the (viii) Determine, according to law or ordinance, the
budgetary limitations on personal services as prescribed in Title time, manner and place of payment of salaries or wages
Five, Book Two of this Code: Provided, That no increase in of the officials and employees of the municipality;
compensation of the mayor, vice-mayor, and sangguniang (ix) Allocate and assign office space to municipal
bayan members shall take effect until after the expiration of the and other officials and employees who, by law or
full term of all the elective local officials approving such ordinance, are entitled to such space in the municipal
increase. hall and other buildings owned or leased by the
municipal government;
CHAPTER III: Officials and Offices Common to All (x) Ensure that all executive officials and
Municipalities employees of the municipality faithfully discharge their
ARTICLE I: The Municipal Mayor duties and functions as provided by law and this Code,
Sec444. The Chief Executive: Powers, Duties, Functions and cause to be instituted administrative or judicial
and Compensation. — proceedings against any official or employee of the
(a) The municipal mayor, as the chief executive of the municipality who may have committed as offense in the
municipal government, shall exercise such powers and performs performance of his official duties;
such duties and functions as provided by this Code and other (xi) Examine the books, records and other
laws. documents of all offices, officials, agents or employees
(b) For efficient, effective and economical governance the of the municipality and in aid of his executive powers
purpose of which is the general welfare of the municipality and and authority, require all national officials and employees
its inhabitants pursuant to Section 16 of this Code, the municipal stationed in or assigned to the municipality to make
mayor shall: available to him such books, records, and other
documents in their custody, except those classified by
law as confidential;

The Apocryphal Maggots:


Rainier, Chrisgel, Corina, Geoffry, Grace and Sylvie Blanche
The Flibbertigibbet Worms:
Golda, Gladys and Melyjane
CA VE AT: By simply r ea ding this re vi ew er a t the end o f the sem este r wi ll (hopef ully) guar antee you r pas sing this cou rse. D rink mode rat ely .
Amusin S C A r y
gly
THE C2005 LOCAL GOVERNMENT REVIEWER - 284 -
(xii) Furnish copies of executive orders issued by respective metropolitan council chairmen and to the
him to the provincial governor within seventy-two (72) Office of the President;
hours after their issuance: Provided, That municipalities (2) Enforce all laws and ordinances relative to the
of Metropolitan Manila Area and that of any metropolitan governance of the municipality and the exercise of its
political subdivision shall furnish copies of said executive corporate powers provided for under Section 22 of this
orders to the metropolitan authority council chairman Code implement all approved policies, programs,
and to the Office of the President; projects, services and activities of the municipality and,
(xiii) Visit component barangays of the municipality in addition to the foregoing, shall:
at least once every six (6) months to deepen his (i) Ensure that the acts of the municipality's
understanding of problems and conditions therein, listen component barangays and of its officials and employees
and give appropriate counsel to local officials and are within the scope of their prescribed powers,
inhabitants, inform the component barangay officials and functions, duties and responsibilities;
inhabitants of general laws and ordinances which (ii) Call conventions, conferences, seminars or
especially concern them, and otherwise conduct visits meetings of any elective and appointive officials of the
and inspections to the end that the governance of the municipality, including provincial officials and national
municipality will improve the quality of life of the officials and employees stationed in or assigned to the
inhabitants; municipality at such time and place and on such subject
(xiv) Act on leave applications of officials and as he may deem important for the promotion of the
employees appointed by him and the commutation of the general welfare of the local government unit and its
monetary value of leave credits according to law; inhabitants;
(xv) Authorize official trips outside of the (iii) Issue such executive orders as are necessary
municipality of municipal officials and employees for a for the proper enforcement and execution of laws and
period not exceeding thirty (30) days; ordinances;
(xvi) Call upon any national official or employee (iv) Be entitled to carry the necessary firearm
stationed in or assigned to the municipality to advise him within his territorial jurisdiction;
on matters affecting the municipality and to make (v) Act as the deputized representative of the
recommendations thereon, or to coordinate in the National Police Commission, formulate the peace and
formulation and implementation of plans, programs and order plan of the municipality and upon its approval
projects, and when appropriate, initiate an administrative implement the same and exercise general and
or judicial action against a national government official or operational control and supervision over the local police
employee who may have committed an offense in the in the municipality in accordance with R.A. No 6975;
performance of his official duties while stationed in or (vi) Call upon the appropriate law enforcement
assigned to the local government unit concerned; agencies to suppress disorder, riot, lawless violence,
(xvii) Subject to availability of funds, authorize rebellion or sedition or to apprehend violators of the law
payment of medical care, necessary transportation, when public interest so requires and the municipal police
subsistence, hospital or medical fees of municipal forces are inadequate to cope with the situation or the
officials and employees who are injured while in the violators;
performance of their official duties and functions; (3) Initiate and maximize the generation of resources and
(xviii) Solemnize marriages, any provision of law to revenues, and apply the same to the implementation of
the contrary notwithstanding; development plans, program objectives and priorities as
(xix) Conduct a palarong bayan, in coordination provided for under Section 18 of this Code, particularly
with the Department of Education, Culture and Sports, those resources and revenues programmed for gro-
as an annual activity which shall feature traditional industrial development and country-wide growth and
sports and disciplines included in national and progress, and relative thereto, shall:
international games; and (i) Require each head of an office or department
(xx) Submit to the provincial governor the following to prepare and submit an estimate of appropriations for
reports: an annual report containing a summary of all the ensuing calendar year, in accordance with the
matters pertaining to the management, administration budget preparation process under Title Five, Book II of
and development of the municipality and all information this Code;
and data relative to its political, social and economic (ii) Prepare and submit to the sanggunian for
conditions; and supplemental reports when unexpected approval the executive and supplemental budgets of the
events and situations arise at any time during the year, municipality for the ensuing calendar year in the manner
particularly when man-made or natural disasters or provided for under Title Five, Book II of this Code;
calamities affect the general welfare of the municipality, (iii) Ensure that all taxes and other revenues of
province, region or country. mayors of municipalities of the municipality are collected and that municipal funds
the Metropolitan Manila Area and other metropolitan are applied in accordance with law or ordinance to the
political subdivisions shall submit said reports to their payment of expenses and settlement of obligations of
the municipality;

The Apocryphal Maggots:


Rainier, Chrisgel, Corina, Geoffry, Grace and Sylvie Blanche
The Flibbertigibbet Worms:
Golda, Gladys and Melyjane
CA VE AT: By simply r ea ding this re vi ew er a t the end o f the sem este r wi ll (hopef ully) guar antee you r pas sing this cou rse. D rink mode rat ely .
Amusin S C A r y
gly
THE C2005 LOCAL GOVERNMENT REVIEWER - 285 -
(iv) Issue licenses and permits and suspend or (1) Be the presiding officer of the sangguniang bayan and
revoke the same for any violation of the conditions upon sign all warrants drawn on the municipal treasury for all
which said licenses or permits had been issued, expenditures appropriated for the operation of the
pursuant to law or ordinance; sangguniang bayan;
(v) Issue permits, without need of approval (2) Subject to civil service law, rules and regulations,
therefor from any national agency, for the holding of appoint all officials and employees of the sangguniang
activities for any charitable or welfare purpose, excluding bayan, except those whose manner of appointment is
prohibited games of chance or shows contrary to law, specifically provided in this Code;
public policy and public morals; (3) Assume the office of the municipal mayor for the
(vi) Require owners of illegally constructed unexpired term of the latter in the event of permanent
houses, buildings or other structures to obtain the vacancy as provided for in Section 44, Book I of this
necessary permit, subject to such fines and penalties as Code;
may be imposed by law or ordinance, or to make (4) Exercise the powers and perform the duties and
necessary changes in the construction of the same when functions of the municipal mayor in cases of temporary
said construction violates any law or ordinance, or to vacancy as provided for in Section 46, Book I of this
order the demolition or removal of said house, building Code; and
or structure within the period prescribed by law or (5) Exercise such other powers and perform such other
ordinance; duties and functions as may be prescribed by law or
(vii) Adopt adequate measures to safeguard and ordinance.
conserve land, mineral, marine, forest, and other (b) The vice-mayor shall receive a monthly compensation
resources of the municipality; provide efficient and corresponding to Salary Grade twenty five (25) as prescribed
effective property and supply management in the under R.A. No. 6758 and the implementing guidelines issued
municipality; and protect the funds, credits, rights and pursuant thereto.
other properties of the municipality; and
(viii) Institute or cause to be instituted ARTICLE III: The Sangguniang Bayan
administrative or judicial proceedings for violation of Sec446. Composition. —
ordinances in the collection of taxes, fees or charges, (a) The sangguniang bayan, the legislative body of the
and for the recovery of funds and property; and cause municipality, shall be composed of the municipal vice mayor as
the municipality to be defended against all suits to the presiding officer, the regular sanggunian members, the
ensure that its interests, resources and rights shall be president of the municipal chapter of the liga ng mga barangay,
adequately protected; the president of the pambayang pederasyon ng mga
(4) Ensure the delivery of basic services and the provision sangguniang kabataan, and the sectoral representatives, as
of adequate facilities as provided for under Section 17 of members.
this Code and, in addition thereto, shall: (b) In addition thereto, there shall be three (3) sectoral
(i) Ensure that the construction and repair of representatives: one (1) from the women; and as shall be
roads and highways funded by the national government determined by the sanggunian concerned within ninety (90)
shall be, as far as practicable, carried out in a spatially days prior to the holding of local elections, one (1) from the
contiguous manner and in coordination with the agricultural or industrial workers, and one (1) from other sectors,
construction and repair of the roads and bridges of the including the urban poor, indigenous cultural communities, or
municipality and the province; and disabled persons.
(ii) Coordinate the implementation of technical (c) The regular members of the sangguniang bayan and
services rendered by national and provincial offices, the sectoral representatives shall be elected in the manner as
including public works and infrastructure programs in the may be provided for by law.
municipality; and
(5) Exercise such other powers and perform such other Sec447. Powers, Duties, Functions and Compensation. —
duties and functions as may be prescribed by law or (a) The sangguniang bayan, as the legislative body of the
ordinance. municipality, shall enact ordinances, approve resolutions and
(c) During his incumbency, the municipal mayor shall hold appropriate funds for the general welfare of the municipality and
office in the municipal hall. its inhabitants pursuant to Section 16 of this Code and in the
(d) The municipal mayor shall receive a minimum monthly proper exercise of the corporate powers of the municipality as
compensation corresponding to Salary Grade twenty-seven (27) provided for under Section 22 of this Code, and shall:
as prescribed under R.A. No. 6758 and the implementing (1) Approve ordinances and pass resolutions necessary for
guidelines issued pursuant thereto. an efficient and effective municipal government, and in
this connection shall:
ARTICLE II: The Vice Mayor (i) Review all ordinances approved by the
Sec445. Powers, Duties and Compensation. — sangguniang barangay and executive orders issued by
(a) The vice-mayor shall: the punong barangay to determine whether these are

The Apocryphal Maggots:


Rainier, Chrisgel, Corina, Geoffry, Grace and Sylvie Blanche
The Flibbertigibbet Worms:
Golda, Gladys and Melyjane
CA VE AT: By simply r ea ding this re vi ew er a t the end o f the sem este r wi ll (hopef ully) guar antee you r pas sing this cou rse. D rink mode rat ely .
Amusin S C A r y
gly
THE C2005 LOCAL GOVERNMENT REVIEWER - 286 -
within the scope of the prescribed powers of the such other records and documents of public interest in
sanggunian and of the punong barangay; the offices and departments of the municipal
(ii) Maintain peace and order by enacting government;
measures to prevent and suppress lawlessness, (xi) When the finances of the municipal
disorder, riot, violence, rebellion or sedition and impose government allow, provide for additional allowances and
penalties for the violation of said ordinances; other benefits to judges, prosecutors, public elementary
(iii) Approve ordinances imposing a fine not and high school teachers, and other national
exceeding Two thousand five hundred pesos government officials stationed in or assigned to the
(P2,500.00) or an imprisonment for a period not municipality;
exceeding six (6) months, or both in the discretion of the (xii) Provide for legal assistance to barangay
court, for the violation of a municipal ordinance; officials who, in the performance of their official duties or
(iv) Adopt measures to protect the inhabitants of on the occasion thereof, have to initiate judicial
the municipality from the harmful effects of man-made or proceedings or defend themselves against legal action;
natural disasters and calamities and to provide relief and
services and assistance for victims during and in the (xii) Provide for group insurance or additional
aftermath of said disasters or calamities and their return insurance coverage for barangay officials, including
to productive livelihood following said events; members of barangay tanod brigades and other service
(v) Enact ordinances intended to prevent, units, with public or private insurance companies, when
suppress and impose appropriate penalties for habitual the finances of the municipal government allow said
drunkenness in public places, vagrancy, mendicancy, coverage.
prostitution, establishment and maintenance of houses (2) Generate and maximize the use of resources and
of ill repute, gambling and other prohibited games of revenues for the development plans, program objectives
chance, fraudulent devices and ways to obtain money or and priorities of the municipality as provided for under
property, drug addiction, maintenance of drug dens, drug Section 18 of this Code with particular attention to agro-
pushing, juvenile delinquency, the printing, distribution or industrial development and countryside growth and
exhibition of obscene or pornographic materials or progress, and relative thereto, shall:
publications, and such other activities inimical to the (i) Approve the annual and supplemental budgets
welfare and morals of the inhabitants of the municipality; of the municipal government and appropriate funds for
(vi) Protect the environment and impose specific programs, projects, services and activities of the
appropriate penalties for acts which endanger the municipality, or for other purposes not contrary to law, in
environment, such as dynamite fishing and other forms order to promote the general welfare of the municipality
of destructive fishing, illegal logging and smuggling of and its inhabitants;
logs, smuggling of natural resources products and of (ii) Subject to the provisions of Book II of this
endangered species of flora and fauna, slash and burn Code and applicable laws and upon the majority vote of
farming, and such other activities which result in all the members of the sangguniang bayan, enact
pollution, acceleration of eutrophication of rivers and ordinances levying taxes, fees and charges, prescribing
lakes, or of ecological imbalance; the rates thereof for general and specific purposes, and
(vii) Subject to the provisions of this Code and granting tax exemptions, incentives or reliefs;
pertinent laws, determine the powers and duties of (iii) Subject to the provisions of Book II of this
officials and employees of the municipality; Code and upon the majority vote of all the members of
(viii) Determine the positions and salaries, wages, the sangguniang bayan, authorize the municipal mayor
allowances and other emoluments and benefits of to negotiate and contract loans and other forms of
officials and employees paid wholly or mainly from indebtedness;
municipal funds and provide for expenditures necessary (iv) Subject to the provisions of Book II of this
for the proper conduct of programs. projects, services, Code and applicable laws and upon the majority vote of
and activities of the municipal government; all the members of the sangguniang bayan, enact
(ix) Authorize the payment of compensation to a ordinances authorizing the floating of bonds or other
qualified person not in the government service who fills instruments of indebtedness, for the purpose of raising
up a temporary vacancy or grant honorarium to any funds to finance development projects;
qualified official or employee designated to fill a (v) Appropriate funds for the construction and
temporary vacancy in a concurrent capacity, at the rate maintenance or the rental of buildings for the use of the
authorized by law; municipality and, upon the majority vote of all the
(x) Provide a mechanism and the appropriate members of the sangguniang bayan, authorize the
funds therefor, to ensure the safety and protection of all municipal mayor to lease to private parties such public
municipal government property, public documents, or buildings held in a proprietary capacity, subject to
records such as those relating to property inventory, land existing laws, rules and regulations;
ownership, records of births, marriages, deaths,
assessments, taxation, accounts, business permits, and

The Apocryphal Maggots:


Rainier, Chrisgel, Corina, Geoffry, Grace and Sylvie Blanche
The Flibbertigibbet Worms:
Golda, Gladys and Melyjane
CA VE AT: By simply r ea ding this re vi ew er a t the end o f the sem este r wi ll (hopef ully) guar antee you r pas sing this cou rse. D rink mode rat ely .
Amusin S C A r y
gly
THE C2005 LOCAL GOVERNMENT REVIEWER - 287 -
(vi) Prescribe reasonable limits and restraints on (ii) Regulate any business, occupation, or
the use of property within the jurisdiction of the practice of profession or calling which does not require
municipality: government examination within the municipality and the
(vii) Adopt a comprehensive land use plan for the conditions under which the license for said business or
municipality: Provided, That the formulation, adoption, or practice of profession may be issued or revoked;
modification of said plan shall be in coordination with the (iii) Prescribe the terms and conditions under
approved provincial comprehensive land use plan; which public utilities owned by the municipality shall be
(viii) Reclassify land within the jurisdiction of the operated by the municipal government or leased to
municipality, subject to the pertinent provisions of this private persons or entities, preferably cooperatives; cd
Code; (iv) Regulate the display of and fix the license fees
(ix) Enact integrated zoning ordinances in for signs, signboards, or billboards at the place or places
consonance with the approved comprehensive land use where the profession or business advertised thereby is,
plan, subject to existing laws, rules and regulations; in whole or in part, conducted;
established fire limits or zones, particularly in populous (v) Any law to the contrary notwithstanding,
centers; and regulate the construction, repair or authorize and license the establishment, operation, and
modification of buildings within said fire limits or zones in maintenance of cockpits, and regulate cockfighting and
accordance with the provisions of this Code; commercial breeding of gamecocks: Provided, That
(x) Subject to national law, process and approve existing rights should not be prejudiced;
subdivision plans for residential, commercial, or (vi) Subject to the guidelines prescribed by the
industrial purposes and other development purposes, Department of Transportation and Communications,
and collect processing fees and other charges the regulate the operation of tricycles and grant franchises
proceeds of which shall accrue entirely to the for the operation thereof within the territorial jurisdiction
municipality: Provided, however, That, where approval of the municipality;
by a national agency or office is required, said approval (vii) Upon approval by a majority vote of all the
shall not be withheld for more than thirty (30) days from members of the sangguniang bayan, grant a franchise to
receipt of the application. Failure to act on the any person, partnership, corporation, or cooperative to
application within the period stated above shall be establish, construct, operate and maintain ferries,
deemed as approval thereof; wharves, markets or slaughterhouses, or such other
(xi) Subject to the provisions of Book II of this similar activities within the municipality as may be
Code, grant the exclusive privilege of constructing fish allowed by applicable laws: Provided, That, cooperatives
corrals or fish pens, or the taking or catching of bangus shall be given preference in the grant of such a
fry, prawn fry or kawag-kawag of fry of any species or franchise.
fish within the municipal waters; (4) Regulate activities relative to the use of land, buildings
(xii) With the concurrence of at least two-thirds and structures within the municipality in order to promote
(2/3) of all the members of the sangguniang bayan, grant the general welfare and for said purpose shall:
tax exemptions, incentives or reliefs to entities engaged (i) Declare, prevent or abate any nuisance;
in community growth-inducing industries, subject to the (ii) Require that buildings and the premises
provisions of Chapter 5, Title I, Book II of this Code. thereof and any land within the municipality be kept and
(xiii) Grant loans or provide grants to other local maintained in a sanitary condition; impose penalties for
government units or to national, provincial and municipal any violation thereof, or upon failure to comply with said
charitable, benevolent or educational institutions: requirement, have the work done and require the owner,
Provided, That said institutions are operated and administrator or tenant concerned to pay the expenses
maintained within the municipality; of the same; or require the filling up of any land or
(xiv) Regulate the numbering of residential, premises to a grade necessary for proper sanitation;
commercial and other buildings; and (iii) Regulate the disposal of clinical and other
(xv) Regulate the inspection, weighing and wastes from hospitals, clinics and other similar
measuring of articles of commerce. establishments;
(3) Subject to the provisions of Book II of this Code, grant (iv) Regulate the establishment, operation and
franchises, enact ordinances authorizing the issuance of maintenance of cafes, restaurants, beerhouses, hotels,
permits or licenses, or enact ordinances levying taxes, motels, inns, pension houses, lodging houses, and other
fees and charges upon such conditions and for such similar establishments, including tourist guides and
purposes intended to promote the general welfare of the transports;
inhabitants of the municipality, and pursuant to this (v) Regulate the sale, giving away or dispensing
legislative authority shall: of any intoxicating malt, vino, mixed or fermented liquors
(i) Fix and impose reasonable fees and charges at any retail outlet;
for all services rendered by the municipal government to (vi) Regulate the establishment and provide for
private persons or entities; the inspection of steam boilers or any heating device in

The Apocryphal Maggots:


Rainier, Chrisgel, Corina, Geoffry, Grace and Sylvie Blanche
The Flibbertigibbet Worms:
Golda, Gladys and Melyjane
CA VE AT: By simply r ea ding this re vi ew er a t the end o f the sem este r wi ll (hopef ully) guar antee you r pas sing this cou rse. D rink mode rat ely .
Amusin S C A r y
gly
THE C2005 LOCAL GOVERNMENT REVIEWER - 288 -
buildings and the storage of inflammable and highly welfare, authorize the removal of encroachments and
combustible materials within the municipality; illegal constructions in public places;
(vii) Regulate the establishment, operation, and (vii) Subject to existing laws, provide for the
maintenance of entertainment or amusement facilities, establishment, operation, maintenance, and repair of an
including theatrical performances, circuses, billiards efficient waterworks system to supply water for the
pools, public dancing schools, public dance halls, sauna inhabitants; regulate the construction, maintenance,
baths, massage parlors, and other places of repair and use of hydrants, pumps, cisterns and
entertainment or amusement; regulate such other events reservoirs; protect the purity and quantity of the water
or activities for amusement or entertainment, particularly supply of the municipality and, for this purpose, extend
those which tend to disturb the community or annoy the the coverage of appropriate ordinances over all territory
inhabitants, or require the suspension or suppression of within the drainage area of said water supply and within
the same; or, prohibit certain forms of amusement or one hundred (100) meters of the reservoir, conduit,
entertainment in order to protect the social and moral canal, aqueduct, pumping station, or watershed used in
welfare of the community; connection with the water service; and regulate the
(viii) Provide for the impounding of stray animals; consumption, use or wastage of water;
regulate the keeping of animals in homes or as part of a (viii) Regulate the drilling and excavation of the
business, and the slaughter, sale or disposition of the ground for laying of water, gas, sewer, and other pipes
same; and adopt measures to prevent and penalize and the construction, repair and maintenance of public
cruelty to animals; and drains, sewers, cesspools, tunnels and similar
(ix) Regulate the establishment, operation, and structures; regulate the placing of poles and the use of
maintenance of funeral parlors and the burial or crosswalks, curbs, and gutters; adopt measures to
cremation of the dead, subject to existing laws, rules and ensure public safety against open canals, manholes, live
regulations. wires and other similar hazards to life and property; and
(5) Approve ordinances which shall ensure the efficient and regulate the construction and use of private water
effective delivery of the basic services and facilities as closets, privies and other similar structures in buildings
provided for under Section 17 of this Code, and in and homes;
addition to said services and facilities, shall: (ix) Regulate the placing, stringing, attaching,
(i) Provide for the establishment, maintenance, installing, repair and construction of all gas mains,
protection, and conservation of communal forests and electric, telegraph and telephone wires, conduits, meters
watersheds, tree parks, greenbelts, mangroves, and and other apparatus; and, provide for the correction,
other similar forest development projects; condemnation or removal of the same when found to be
(ii) Establish markets, slaughterhouses or animal dangerous, defective or otherwise hazardous to the
corrals and authorize the operation thereof, and regulate welfare of the inhabitants;
the construction and operation of private markets, (x) Subject to the availability of funds and to
talipapas or other similar buildings and structures; existing laws, rules and regulations, establish and
(iii) Authorize the establishment, maintenance and provide for the operation of vocational and technical
operation of ferries, wharves, and other structures, and schools and similar post-secondary institutions and, with
marine and seashore or offshore activities intended to the approval of the Department of Education. Culture
accelerate productivity; and Sports, fix and collect reasonable fees and other
(iv) Regulate the preparation and sale of meat, school charges on said institutions, subject to existing
poultry, fish, vegetables, fruits, fresh dairy products, and laws on tuition fees;
other foodstuffs for public consumption; (xi) Establish a scholarship fund for poor but
(v) Regulate the use of streets, avenues, alleys, deserving students residing within the municipality in
sidewalks, bridges, parks and other public places and schools located within its jurisdiction;
approve the construction, improvement, repair and (xii) Approve measures and adopt quarantine
maintenance of the same; establish bus and vehicle regulations to prevent the introduction and spread of
stops and terminals or regulate the use of the same by diseases;
privately-owned vehicles which serve the public; regulate (xiii) Provide for an efficient and effective system of
garages and the operation of conveyances for hire; solid waste and garbage collection disposal and prohibit
designate stands to be occupied by public vehicles when littering and the placing or throwing of garbage, refuse
not in use; regulate the putting up of signs, signposts, and other filth and wastes;
awnings and awning posts on the streets; provide for the (xiv) Provide for the care of paupers, the aged, the
lighting, cleaning and sprinkling of streets and public sick, persons of unsound mind, disabled persons,
places; abandoned minors, juvenile delinquents, drug
(vi) Regulate traffic on all streets and bridges, dependents, abused children and other needy and
prohibit the putting up of encroachments or obstacles disadvantaged persons, particularly children and youth
thereon, and, when necessary in the interest of public below eighteen (18) years of age and, subject to
availability of funds, establish and provide for the

The Apocryphal Maggots:


Rainier, Chrisgel, Corina, Geoffry, Grace and Sylvie Blanche
The Flibbertigibbet Worms:
Golda, Gladys and Melyjane
CA VE AT: By simply r ea ding this re vi ew er a t the end o f the sem este r wi ll (hopef ully) guar antee you r pas sing this cou rse. D rink mode rat ely .
Amusin S C A r y
gly
THE C2005 LOCAL GOVERNMENT REVIEWER - 289 -
operation of centers and facilities for said needy and The City – LGC
disadvantaged persons;
(xv) Establish and provide for the maintenance and CHAPTER I: Role and Creation of the City
improvement of jails and detention centers, institute Sec448. Role of the City. — The city, consisting of more
sound jail management programs, and appropriate funds urbanized and developed barangays. serves as a general
for the subsistence of detainees and convicted prisoners purpose government for the coordination and delivery of basic,
in the municipality; regular, and direct services and effective governance of the
(xvi) Establish a municipal council whose purpose inhabitants within its territorial jurisdiction.
is the promotion of culture and the arts, coordinate with
government agencies and non-governmental Sec449. Manner of Creation. — A city may be created,
organizations and, subject to the availability of funds, divided, merged, abolished, or its boundary substantially altered,
appropriate funds for the support and development of only by an Act of Congress, and subject to approval by a
the same; and majority of the votes cast in a plebiscite to be conducted by the
(xvi) Establish a municipal council for the orderly COMELEC in the local government unit or units directly affected.
which shall formulate policies and adopt measures Except as may otherwise be provided in such Act. the plebiscite
mutually beneficial to the elderly and to the community; shall be held within one hundred twenty (120) days from the
provide incentives for non-governmental agencies and date of its effectivity.
entities and, subject to the availability of funds,
appropriate funds to support programs and projects for Sec450. Requisites for Creation. —
the benefit of the elderly; and (a) A municipality or a cluster of barangays may be
(6) Exercise such other powers and perform such other converted into a component city if it has an average annual
duties and functions as may be prescribed by law or income, as certified by the Department of Finance, of at least
ordinance. Twenty million (P20,000,000.00) for the last two (2) consecutive
(b) The members of the sangguniang bayan shall receive years based on 1991 constant prices, and if it has either of the
a minimum monthly compensation corresponding to Salary following requisites:
Grade twenty-four (24) as prescribed under R.A. No. 6758 and (i) a contiguous territory of at least one hundred (100)
the implementing guidelines issued pursuant thereto: Provided, square kilometers, as certified by the Lands
That, in municipalities in Metropolitan Manila Area and other Management Bureau; or
metropolitan political subdivisions, members of the sangguniang (ii) a population of not less than one hundred fifty thousand
bayan shall receive a minimum monthly compensation (150,000) inhabitants, as certified by the National
corresponding to Salary grade twenty-five (25). Statistics Office:
Provided, That, the creation thereof shall not reduce the land
Olivarez vs. Sandiganbayan (1995) area, population, and income of the original unit or units at the
time of said creation to less than the minimum requirements
Facts: prescribed herein.
Parañaque Mayor Olivarez was charged by the Baclaran Credit (b) The territorial jurisdiction of a newly-created city shall
Coop Inc (BCCI) w/ a violation of the anti-graft and corrupt be properly identified by metes and bounds. The requirement on
practices law, for unreasonably refusing to issue a mayor’s permit land area shall not apply where the city proposed to be created
despite their application and the SB resolution authorizing the is composed of one (1) or more islands. The territory need not
BCCI to set up a night manufacturer’s fair during the Christmas be contiguous if it comprises two (2) or more islands.
Fiesta Celebration. One of the grounds in the MTD filed by (c) The average annual income shall include the income
petitioner was that he had no authority to act on the letter- accruing to the general fund, exclusive of specific funds,
application of BCCI. transfers, and non-recurring income.

Issue: WON the mayor has the authority to issue permits Sec451. Cities, Classified. — A city may either be component
Held: or highly urbanized: Provided, however, That the criteria
Yes, as mayor, he has authority over the officials referred to, and established in this Code shall not affect the classification and
he could take appropriate action on the letter-application even corporate status of existing cities.
though it did not strictly follow the normal procedure. He could refer Independent component cities are those component cities
it to the licensing department. He is expressly authorized and has whose charters prohibit their voters from voting for provincial
the power to issue permits and licenses for the holding of activities elective officials. Independent component cities shall be
for any charitable or welfare purpose (Sec444(b)(3)(iv and v) of independent of the province.
LGC). He cannot feign total lack of authority to act on letter-
application of BCCI. Sec452. Highly Urbanized Cities. —
(a) Cities with a minimum population of two hundred
thousand (200,000) inhabitants as certified by the National
Statistics Office, and within the latest annual income of at least
Fifty Million Pesos (P50,000,000.00) based on 1991 constant

The Apocryphal Maggots:


Rainier, Chrisgel, Corina, Geoffry, Grace and Sylvie Blanche
The Flibbertigibbet Worms:
Golda, Gladys and Melyjane
CA VE AT: By simply r ea ding this re vi ew er a t the end o f the sem este r wi ll (hopef ully) guar antee you r pas sing this cou rse. D rink mode rat ely .
Amusin S C A r y
gly
THE C2005 LOCAL GOVERNMENT REVIEWER - 290 -
prices, as certified by the city treasurer, shall be classified as appointment within fifteen (15) days from the date of its
highly urbanized cities. submission, otherwise the same shall be deemed confirmed.
(b) Cities which do not meet above requirements shall be (e) Elective and appointive city officials shall receive such
considered component cities of the province in which they are compensation, allowances, and other emoluments as may be
geographically located. If a component city is located within the determined by law or ordinance, subject to the budgetary
boundaries of two (2) or more provinces, such city shall be limitations on personal services prescribed under Title Five,
considered a component of the province of which it used to be a Book II of this Code: Provided, That, no increase in
municipality. compensation of the mayor, vice-mayor and sangguniang
(c) Qualified voters of highly urbanized cities shall remain panlungsod members shall take effect until after the expiration
excluded from voting for elective provincial officials. of the full term of the said local officials approving such increase.
Unless otherwise provided in the Constitution or this Code,
qualified voters of independent component cities shall be CHAPTER III: Officials and Offices Common to All Cities
governed by their respective charters, as amended, on the ARTICLE I: The City Mayor
participation of voters in provincial elections. Sec455. Chief Executive; Powers, Duties and
Qualified voters of cities who acquired the right to vote for Compensation. — (a) The city mayor, as chief executive
elective provincial officials prior to the classification of said cities of the city government, shall exercise such powers and perform
as highly-urbanized after the ratification of the Constitution and such duties and functions as provided by this Code and other
before the effectivity of this Code, shall continue to exercise laws.
such right. (b) For efficient, effective and economical governance the
purpose of which is the general welfare of the city and its
Sec453. Duty to Declare Highly Urbanized Status. — It shall inhabitants pursuant to Section 16 of this Code, the city mayor
be the duty of the President to declare a city as highly urbanized shall:
within thirty (30) days after it shall have met the minimum (1) Exercise general supervision and control over all
requirements prescribed in the immediately preceding section, programs, projects, services, and activities of the city
upon proper application therefor and ratification in a plebiscite government. and in this connection, shall:
by the qualified voters therein. (i) Determine the guidelines of city policies and
be responsible to the sangguniang panlungsod for the
CHAPTER II: City Officials in General program of government;
Sec454. Officials of the City Government. — (ii) Direct the formulation of the city development
(a) There shall be in each city a mayor, a vice-mayor, plan, with the assistance of the city development council,
sangguniang panlungsod members, a secretary to the and upon approval thereof by the sangguniang
sangguniang panlungsod, a city treasurer, a city assessor, a city panlungsod, implement the same;
accountant, a city budget officer, a city planning and (iii) Present the program of government and
development coordinator, a city engineer, a city health officer, a propose policies and projects for the consideration of the
city civil registrar, a city administrator, a city legal officer, a city sangguniang panlungsod at the opening of the regular
veterinarian, a city social welfare and development officer, and a session of the sangguniang panlungsod every calendar
city general services officer. year and as often as may be deemed necessary as the
(b) In addition thereto, the city mayor may appoint a city general welfare of the inhabitants and the needs of the
architect, a city information officer, a city agriculturist, a city city government may require;
population officer, a city environment and natural resources (iv) Initiate and propose legislative measures to
officer, and a city cooperatives officer. the sangguniang panlungsod and as often as may be
The appointment of a city population officer shall be optional in deemed necessary, provide such information and data
the city: Provided, however, That cities which have existing needed or requested by said sanggunian in the
population offices shall continue to maintain such offices for a performance of its legislative functions;
period of five (5) years from the date of the effectivity of this (v) Appoint all officials and employees whose
Code, after which said offices shall become optional. salaries and wages are wholly or mainly paid out of city
(c) The sangguniang panlungsod may: funds and whose appointments are not otherwise
(1) Maintain existing offices not mentioned in subsections provided for in this Code, as well as those he may be
(a) and (b) hereof; authorized by law to appoint;
(2) Create such other offices as may be necessary to carry (vi) Represent the city in all its business
out the purposes of the city government; or transactions and sign in its behalf all bonds, contracts,
(3) Consolidate the functions of any office with those of and obligations, and such other documents upon
another in the interest of efficiency and economy. authority of the sangguniang panlungsod or pursuant to
(d) Unless otherwise provided herein, heads of law or ordinance;
departments and offices shall be appointed by the city mayor (vii) Carry out such emergency measures as may
with the concurrence of the majority of all the sangguniang be necessary during and in the aftermath of man-made
panlungsod members, subject to civil service law, rules and and natural disasters and calamities;
regulations. The sangguniang panlungsod shall act on the

The Apocryphal Maggots:


Rainier, Chrisgel, Corina, Geoffry, Grace and Sylvie Blanche
The Flibbertigibbet Worms:
Golda, Gladys and Melyjane
CA VE AT: By simply r ea ding this re vi ew er a t the end o f the sem este r wi ll (hopef ully) guar antee you r pas sing this cou rse. D rink mode rat ely .
Amusin S C A r y
gly
THE C2005 LOCAL GOVERNMENT REVIEWER - 291 -
(viii) Determine the time, manner and place of medical fees of city officials and employees who are
payment of salaries or wages of the officials and injured while in the performance of their duties and
employees of the city, in accordance with law or functions, subject to availability of funds;
ordinance; (xviii) Solemnize marriage, any provision of law to
(ix) Allocate and assign office space to city and the contrary notwithstanding;
other officials and employees who, by law or ordinance, (xix) Conduct an annual palarong panlungsod,
are entitled to such space in the city hall and other which shall feature traditional sports and disciplines
buildings owned or leased by the city government; included in national and international games, in
(x) Ensure that all executive officials and coordination with the Department of Education, Culture
employees of the city faithfully discharge their duties and and Sports; and
functions as provided by law and this Code, and cause (xx) Submit to the provincial governor, in case of
to be instituted administrative or judicial proceedings component cities; to the Office of the President, in the
against any official or employee of the city who may case of highly-urbanized cities; to their respective
have committed an offense in the performance of his metropolitan authority council chairmen and to the Office
official duties; of the President, in case of cities of the Metropolitan
(xi) Examine the books, records and other Manila Area and other metropolitan political subdivisions,
documents of all offices, officials, agents or employees the following reports: an annual report containing a
of the city and, in aid of his executive powers and summary of all matters pertinent to the management,
authority, require all national officials and employees administration and development of the city and all
stationed in or assigned to the city to make available to information and data relative to its political, social and
him such books, records, and other documents in their economic conditions; and supplemental reports when
custody, except those classified by law as confidential; unexpected events and situations arise at any time
(xii) Furnish copies of executive orders issued by during the year, particularly when man-made or natural
him, to the provincial governor in the case of component disasters or calamities affect the general welfare of the
city mayors, to the Office of the President in the case of city, province, region or country;
highly-urbanized city mayors and to their respective (2) Enforce all laws and ordinances relative to the
metropolitan council chairmen in the case of mayors of governance of the city and in the exercise of the
cities in the Metropolitan Manila Area and other appropriate corporate powers provided for under Section
metropolitan political subdivisions, within seventy-two 22 of this Code, implement all approved policies,
(72) hours after their issuances; programs, projects, services and activities of the city
(xiii) Visit component barangays of the city at least and, in addition to the foregoing, shall:
once every six (6) months to deepen his understanding (i) Ensure that the acts of the city's component
of problems and conditions, listen and give appropriate barangays and of its officials and employees are within
counsel to, local officials and inhabitants, inform the the scope of their prescribed powers, duties and
component barangay officials and inhabitants of general functions;
laws and ordinances which especially concern them, and (ii) Call conventions, conferences, seminars, or
otherwise conduct visits and inspections to ensure that meetings of any elective and appointive officials of the
the governance of the city will improve the quality of life city, including provincial officials and national officials
of the inhabitants; and employees stationed in or assigned to the city, at
(xiv) Act on leave applications of officials and such time and place and on such subject as he may
employees appointed by him and the commutation of the deem important for the promotion of the general welfare
monetary value of their leave credits in accordance with of the local government unit and its inhabitants;
law; (ii) Issue such executive orders for the faithful and
(xv) Authorize official trips of city officials and appropriate enforcement and execution of laws and
employees outside of the city for a period not exceeding ordinances;
thirty (30) days; (iv) Be entitled to carry the necessary firearm
(xvi) Call upon any national official or employee within his territorial jurisdiction;
stationed in or assigned to the city to advise him on (v) Act as the deputized representative of the
matters affecting the city and to make recommendations National Police Commission, formulate the peace and
thereon; coordinate with said official or employee in the order plan of the city and upon its approval, implement
formulation and implementation of plans, programs and the same; and as such exercise general and operational
projects; and, when appropriate, initiate an control and supervision over the local police forces in the
administrative or judicial action against a national city, in accordance with R.A. No. 6975;
government official or employee who may have (vi) Call upon the appropriate law enforcement
committed an offense in the performance of his official agencies to suppress disorder, riot, lawless violence,
duties while stationed in or assigned to the city; rebellion or sedition, or to apprehend violators of the law
(xvii) Authorize payment for medical care, when public interest so requires and the city police
necessary transportation, subsistence, hospital or

The Apocryphal Maggots:


Rainier, Chrisgel, Corina, Geoffry, Grace and Sylvie Blanche
The Flibbertigibbet Worms:
Golda, Gladys and Melyjane
CA VE AT: By simply r ea ding this re vi ew er a t the end o f the sem este r wi ll (hopef ully) guar antee you r pas sing this cou rse. D rink mode rat ely .
Amusin S C A r y
gly
THE C2005 LOCAL GOVERNMENT REVIEWER - 292 -
forces are inadequate to cope with the situations or the shall be, as far as practicable, carried out in a spatially
violators; contiguous manner and in coordination with the
(3) Initiate and maximize the generation of resources and construction and repair of the roads and bridges of the
revenues, and apply the same to the implementation of city, and in the case of component cities, of the city and
development plans, program objectives and priorities as of the province; and
provided for under Section 18 of this Code, particularly (ii) Coordinate the implementation of technical
those resources and revenues programmed for agro- services, including public works and infrastructure
industrial development and countryside growth and programs, rendered by national offices in the case of
progress and, relative thereto, shall: highly urbanized and independent component cities, and
(i) Require each head of an office or department by national and provincial offices in the case of
to prepare and submit an estimate of appropriations for component cities; and
the ensuing calendar year, in accordance with the (5) Exercise such other powers and perform such other
budget preparations process under Title Five, Book II of duties and functions as may be prescribed by law or
this Code; ordinance.
(ii) Prepare and submit to the sanggunian for (c) During his incumbency, the city mayor shall hold office
approval the executive and supplemental budgets of the in the city hall.
city for the ensuing calendar year in the manner (d) The city mayor shall receive a minimum monthly
provided for under Title Five, Book II of this Code; compensation corresponding to Salary Grade Thirty (30) as
(iii) Ensure that all taxes and other revenues of prescribed under R.A. No. 6758 and the implementing
the city are collected, and that city funds are applied to guidelines issued pursuant thereto.
the payment of expenses and settlement of obligations
of the city, in accordance with law or ordinance; ARTICLE II: The City Vice-Mayor
(iv) Issue licenses and permits and suspend or Sec456. Powers, Duties and Compensation. —
revoke the same for any violation of the conditions upon (a) The city vice-mayor shall:
which said licenses or permits had been issued, (1) Be the presiding officer of the sangguniang panlungsod
pursuant to law or ordinance; and sign all warrants drawn on the city treasury for all
(v) Issue permits, without need of approval expenditures appropriated for the operation of the
therefor from any national agency, for the holding of sangguniang panlungsod;
activities for any charitable or welfare purpose, excluding (2) Subject to civil service law, rules and regulations,
prohibited games of chance or shows contrary to law, appoint all officials and employees of the sangguniang
public policy and public morals; panlungsod, except those whose manner of appointment
(vi) Require owners of illegally constructed is specifically provided in this Code;
houses, buildings or other structures to obtain the (3) Assume the office of the city mayor for the unexpired
necessary permit, subject to such fines and penalties as term of the latter in the event of permanent vacancy as
may be imposed by law or ordinance, or to make provided for in Section 44, Book I of this Code;
necessary changes in the construction of the same when (4) Exercise the powers and perform the duties and
said construction violates any law or ordinance, or to functions of the city mayor in cases of temporary
order the demolition or removal of said house, building vacancy as provided for in Section 46, Book I of this
or structure within the period prescribed by law or Code; and
ordinance; (5) Exercise such other powers and perform such other
(vii) Adopt adequate measures to safeguard and duties and functions as may be prescribed by law or
conserve land, mineral, marine, forest, and other ordinance.
resources of the city; provide efficient and effective (b) The city vice-mayor shall receive a monthly
property and supply management in the city; and protect compensation corresponding to Salary Grade twenty eight (28)
the funds, credits, rights and other properties of the city; for a highly urbanized city and Salary Grade twenty-six (26) for a
and component city, as prescribed under R.A. No. 6758 and the
(viii) Institute or cause to be instituted implementing guidelines issued pursuant thereto.
administrative or judicial proceedings for violation of
ordinances in the collection of taxes, fees or charges, ARTICLE III: The Sangguniang Panlungsod
and for the recovery of funds and property; and cause Sec457. Composition —
the city to be defended against all suits to ensure that its (a) The sangguniang panlungsod, the legislative body of
interests, resources and rights shall be adequately the city, shall be composed of the city vice-mayor as presiding
protected; officer, the regular sanggunian members, the president of the
(4) Ensure the delivery of basic services and the provision city chapter of the liga ng mga barangay, the president of the
of adequate facilities as provided for under Section 17 of panlungsod na pederasyon ng mga sangguniang kabataan, and
this Code and, in addition thereto, shall: the sectoral representatives, as members.
(i) Ensure that the construction and repair of (b) In addition thereto, there shall be three (3) sectoral
roads and highways funded by the national government representatives: one (1) from the women; and as shall be

The Apocryphal Maggots:


Rainier, Chrisgel, Corina, Geoffry, Grace and Sylvie Blanche
The Flibbertigibbet Worms:
Golda, Gladys and Melyjane
CA VE AT: By simply r ea ding this re vi ew er a t the end o f the sem este r wi ll (hopef ully) guar antee you r pas sing this cou rse. D rink mode rat ely .
Amusin S C A r y
gly
THE C2005 LOCAL GOVERNMENT REVIEWER - 293 -
determined by the sanggunian concerned within ninety (90) pollution, acceleration of eutrophication of rivers and
days prior to the holding of the local elections, one (1) from lakes, or of ecological imbalance;
agricultural or industrial workers; and one (1) from the other (vii) Subject to the provisions of this Code and
sectors, including the urban poor, indigenous cultural pertinent laws, determine the powers and duties of
communities, or disabled persons. officials and employees of the city;
(c) The regular members of the sangguniang panlungsod (viii) Determine the positions and the salaries,
and the sectoral representatives shall be elected in the manner wages, allowances and other emoluments and benefits
as may be provided for by law. of officials and employees paid wholly or mainly from city
funds and provide for expenditures necessary for the
Sec458. Powers, Duties, Functions and Compensation. — proper conduct of programs, projects, services, and
(a) The sangguniang panlungsod, as the legislative body activities of the city government;
of the city, shall enact ordinances, approve resolutions and (ix) Authorize the payment of compensation to a
appropriate funds for the general welfare of the city and its qualified person not in the government service who fills
inhabitants pursuant to Section 16 of this Code and in the up a temporary vacancy or grant honorarium to any
proper exercise of the corporate powers of the city as provided qualified official or employee designated to fill a
for under Section 22 of this Code, and shall: temporary vacancy in a concurrent capacity, at the rate
(1) Approve ordinances and pass resolutions necessary for authorized by law;
an efficient and effective city government, and in this (x) Provide a mechanism and the appropriate
connection, shall: funds therefor, to ensure the safety and protection of all
(i) Review all ordinances approved by the city government property, public documents, or records
sangguniang barangay and executive orders issued by such as those relating to property inventory, land
the punong barangay to determine whether these are ownership, records of births, marriages, deaths,
within the scope of the prescribed powers of the assessments, taxation, accounts, business permits, and
sanggunian and of the punong barangay; such other records and documents of public interest in
(ii) Maintain peace and order by enacting the offices and departments of the city government;
measures to prevent and suppress lawlessness, (xi) When the finances of the city government
disorder, riot, violence, rebellion or sedition and impose allow, provide for additional allowances and other
penalties for violation of said ordinances; benefits to judges, prosecutors, public elementary and
(iii) Approve ordinances imposing a fine not high school teachers, and other national government
exceeding Five thousand pesos (P5,000.00) or an officials stationed in or assigned to the city;
imprisonment for a period not exceeding one (1) year, or (xii) Provide legal assistance to barangay officials
both in the discretion of the court, for the violation of a who, in the performance of their official duties or on the
city ordinance; occasion thereof, have to initiate judicial proceedings or
(iv) Adopt measures to protect the inhabitants of defend themselves against legal action; and
the city from the harmful effects of man-made or natural (xiii) Provide for group insurance or additional
disasters and calamities, and to provide relief services insurance coverage for all barangay officials, including
and assistance for victims during and in the aftermath of members of barangay tanod brigades and other service
said disasters or calamities and their return to productive units, with public or private insurance companies, when
livelihood following said events; the finances of the city government allow said coverage;
(v) Enact ordinances intended to prevent, (2) Generate and maximize the use of resources and
suppress and impose appropriate penalties for habitual revenues for the development plans, program objectives
drunkenness in public places, vagrancy, mendicancy, and priorities of the city as provided for under Section 18
prostitution, establishment and maintenance of houses of this Code, with particular attention to agro-industrial
of ill repute, gambling and other prohibited games of development and city-wide growth and progress, and
chance, fraudulent devices and ways to obtain money or relative thereto, shall:
property, drug addiction, maintenance of drug dens, drug (i) Approve the annual and supplemental budgets
pushing, juvenile delinquency, the printing, distribution or of the city government and appropriate funds for specific
exhibition of obscene or pornographic materials or programs, projects, services and activities of the city, or
publications, and such other activities inimical to the for other purposes not contrary to law, in order to
welfare and morals of the inhabitants of the city. promote the general welfare of the city and its
(vi) Protect the environment and impose inhabitants;
appropriate penalties for acts which endanger the (ii) Subject to the provisions of Book II of this
environment, such as dynamite fishing and other forms Code and applicable laws and upon the majority vote of
of destructive fishing, illegal logging and smuggling of all the members of the sangguniang panlungsod, enact
logs, smuggling of natural resources products and of ordinances levying taxes, fees and charges, prescribing
endangered species of flora and fauna, slash and burn the rates thereof for general and specific purposes, and
farming, and such other activities which result in granting tax exemptions, incentives or reliefs;

The Apocryphal Maggots:


Rainier, Chrisgel, Corina, Geoffry, Grace and Sylvie Blanche
The Flibbertigibbet Worms:
Golda, Gladys and Melyjane
CA VE AT: By simply r ea ding this re vi ew er a t the end o f the sem este r wi ll (hopef ully) guar antee you r pas sing this cou rse. D rink mode rat ely .
Amusin S C A r y
gly
THE C2005 LOCAL GOVERNMENT REVIEWER - 294 -
(iii) Subject to the provisions of Book II of this charitable, benevolent or educational institutions:
Code and upon the majority vote of all the members of Provided, That, said institutions are operated and
the sangguniang panlungsod, authorize the city mayor to maintained within the city;
negotiate and contract loans and other forms of (xiv) Regulate the numbering of residential,
indebtedness; commercial and other buildings; and
(iv) Subject to the provisions of Book II of this (xv) Regulate the inspection, weighing and
Code and applicable laws and upon the majority vote of measuring of articles of commerce.
all the members of the sangguniang panlungsod, enact (3) Subject to the provisions of Book II of this Code, enact
ordinances authorizing the floating of bonds or other ordinances granting franchises and authorizing the
instruments of indebtedness, for the purpose of raising issuance of permits or licenses, upon such conditions
funds to finance development projects; and for such purposes intended to promote the general
(v) Appropriate funds for the construction and welfare of the inhabitants of the city and pursuant to this
maintenance or the rental of buildings for the use of the legislative authority shall:
city; and, upon the majority vote of all the members of (i) Fix and impose reasonable fees and charges
the sangguniang panlungsod, authorize the city mayor to for all services rendered by the city government to
lease to private parties such public buildings held in a private persons or entities;
proprietary capacity, subject to existing laws, rules and (ii) Regulate or fix license fees for any business
regulations; or practice of profession within the city and the
(vi) Prescribe reasonable limits and restraints on conditions under which the license for said business or
the use of property within the jurisdiction of the city; practice of profession may be revoked and enact
(vii) Adopt a comprehensive land use plan for the ordinances levying taxes thereon;
city: Provided, That in the case of component cities, the (iii) Provide for and set the terms and conditions
formulation, adoption or modification of said plan shall under which public utilities owned by the city shall be
be in coordination with the approved provincial operated by the city government, and prescribe the
comprehensive land use plan; conditions under which the same may be leased to
(viii) Reclassify land within the jurisdiction of the private persons or entities, preferably cooperatives; cd i
city, subject to the pertinent provisions of this Code; (iv) Regulate the display of and fix the license fees
(ix) Enact integrated zoning ordinances in for signs, signboards, or billboards at the place or places
consonance with the approved comprehensive land use where the profession or business advertised thereby is,
plan, subject to existing laws, rules and regulations; in whole or in part, conducted;
establish fire limits or zones, particularly in populous (v) Any law to the contrary notwithstanding,
centers; and regulate the construction, repair or authorize and license the establishment, operation, and
modification of buildings within said fire limits or zones in maintenance of cockpits, and regulate cockfighting and
accordance with the provisions of the Fire Code; commercial breeding of gamecocks: Provided, That
(x) Subject to national law, process and approve existing rights should not be prejudiced;
subdivision plans for residential, commercial, or (vi) Subject to the guidelines prescribed by the
industrial purposes and other development purposes, Department of Transportation and Communications,
and to collect processing fees and other charges, the regulate the operation of tricycles and grant franchises
proceeds of which shall accrue entirely to the city: for the operation thereof within the territorial jurisdiction
Provided, however, That where approval of a national of the city;
agency or office is required, said approval shall not be (vii) Upon approval by a majority vote of all the
withheld for more than thirty (30) days from receipt of the members of the sangguniang panlungsod: grant a
application. Failure to act on the application within the franchise to any person, partnership, corporation, or
period stated above shall be deemed as approval cooperative to do business within the city; establish,
thereof; construct, operate and maintain ferries, wharves,
(xi) Subject to the provisions of Book II of this markets or slaughterhouses; or undertake such other
Code, grant the exclusive privilege of constructing fish activities within the city as may be allowed by existing
corrals or fish pens, or the taking or catching of bangus laws: Provided, That, cooperatives shall be given
fry, prawn fry or kawag-kawag, or fry of any species or preference in the grant of such a franchise.
fish within the city waters; (4) Regulate activities relative to the use of land, buildings
(xii) With the concurrence of at least two-thirds and structures within the city in order to promote the
(2/3) of all the members of the sangguniang panlungsod, general welfare and for said purpose shall:
grant tax exemptions, incentives or reliefs to entities (i) Declare, prevent or abate any nuisance;
engaged in community growth-inducing industries, (ii) Require that buildings and the premises
subject to the provisions of Chapter 5, Title I, Book II of thereof and any land within the city be kept and
this Code; maintained in a sanitary condition; impose penalties for
(xiii) Grant loans or provide grants to other local any violation thereof; or, upon failure to comply with said
government units or to national, provincial, and city requirement, have the work done at the expense of the

The Apocryphal Maggots:


Rainier, Chrisgel, Corina, Geoffry, Grace and Sylvie Blanche
The Flibbertigibbet Worms:
Golda, Gladys and Melyjane
CA VE AT: By simply r ea ding this re vi ew er a t the end o f the sem este r wi ll (hopef ully) guar antee you r pas sing this cou rse. D rink mode rat ely .
Amusin S C A r y
gly
THE C2005 LOCAL GOVERNMENT REVIEWER - 295 -
owner, administrator or tenant concerned; or require the (iv) Regulate the preparation and sale of meat,
filling up of any land or premises to a grade necessary poultry, fish, vegetables, fruits, fresh dairy products, and
for proper sanitation; other foodstuffs for public consumption;
(iii) Regulate the disposal of clinical and other (v) Regulate the use of streets, avenues, alleys,
wastes from hospitals, clinics and other similar sidewalks, bridges, parks and other public places and
establishments; approve the construction, improvement repair and
(iv) Regulate the establishment, operation and maintenance of the same; establish bus and vehicle
maintenance of cafes, restaurants, beerhouses, hotels, stops and terminals or regulate the use of the same by
motels, inns, pension houses, lodging houses, and other privately-owned vehicles which serve the public; regulate
similar establishments, including tourist guides and garages and the operation of conveyances for hire;
transports; designate stands to be occupied by public vehicles when
(v) Regulate the sale, giving away or dispensing not in use; regulate the putting up of signs, signposts,
of any intoxicating malt, vino, mixed or fermented liquors awnings and awning posts on the streets; and provide
at any retail outlet; for the lighting, cleaning and sprinkling of streets; and
(vi) Regulate the establishment and provide for public places;
the inspection of steam boilers or any heating device in (vi) Regulate traffic on all streets and bridges;
buildings and the storage of inflammable and highly prohibit encroachments or obstacles thereon, and when
combustible materials within the city; necessary in the interest of public welfare, authorize the
(vii) Regulate the establishment, operation, and removal or encroachments and illegal constructions in
maintenance of any entertainment or amusement public places;
facilities, including theatrical performances, circuses, (vii) Subject to existing laws, establish and provide
billiard pools, public dancing schools, public dance halls, for the maintenance, repair and operation of an efficient
sauna baths, massage parlors, and other places for waterworks system to supply water for the inhabitants
entertainment or amusement; regulate such other events and to purify the source of the water supply; regulate the
or activities for amusement or entertainment, particularly construction, maintenance, repair and use of hydrants,
those which tend to disturb the community or annoy the pumps, cisterns and reservoirs; protect the purity and
inhabitants, or require the suspension or suppression of quantity of the water supply of the city and, for this
the same; or, prohibit certain forms of amusement or purpose, extend the coverage of appropriate ordinances
entertainment in order to protect the social and moral over all territory within the drainage area of said water
welfare of the community; supply and within one hundred (100) meters of the
(viii) Provide for the impounding of stray animals; reservoir, conduit, canal, aqueduct, pumping station, or
regulate the keeping of animals in homes or as part of a watershed used in connection with the water service;
business, and the slaughter, sale or disposition of the and regulate the consumption, use or wastage of water
same; and adopt measures to prevent and penalize and fix and collect charges therefor;
cruelty to animals; and (viii) Regulate the drilling and excavation of the
(ix) Regulate the establishment, operation and ground for the laying of water, gas, sewer, and other
maintenance of funeral parlors and the burial or pipes and the construction, repair and maintenance of
cremation of the dead, subject to existing laws, rules and public drains, sewers, cesspools, tunnels and similar
regulations. structures; regulate the placing of poles and the use of
(5) Approve ordinances which shall ensure the efficient and crosswalks, curbs, and gutters; adopt measures to
effective delivery of the basic services and facilities as ensure public safety against open canals, manholes. live
provided for under Section 17 of this Code, and in wires and other similar hazards to life and property; and
addition to said services and facilities, shall: regulate the construction and use of private water
(i) Provide for the establishment, maintenance, closets, privies and other similar structures in buildings
protection, and conservation of communal forests and and homes;
watersheds, tree parks, greenbelts, mangroves, and (ix) Regulate the placing, stringing, attaching,
other similar forest development projects; installing, repair and construction of all gas mains,
(ii) Establish markets, slaughterhouses or animal electric, telegraph and telephone wires, conduits, meters
corrals and authorize the operation thereof by the city and other apparatus; and provide for the correction,
government; and regulate the construction and operation condemnation or removal of the same when found to be
of private markets, talipapas or other similar buildings dangerous, defective, or otherwise hazardous to the
and structures; welfare of the inhabitants;
(iii) Authorize the establishment, maintenance and (x) Subject to the availability of funds and to
operation by the city government of ferries, wharves, and existing laws, rules and regulations, establish and
other structures intended to accelerate productivity provide for the operation of vocational and technical
related to marine and seashore or offshore activities; schools and similar post-secondary institutions and, with
the approval of the Department of Education, Culture
and Sports and subject to existing law on tuition fees, fix

The Apocryphal Maggots:


Rainier, Chrisgel, Corina, Geoffry, Grace and Sylvie Blanche
The Flibbertigibbet Worms:
Golda, Gladys and Melyjane
CA VE AT: By simply r ea ding this re vi ew er a t the end o f the sem este r wi ll (hopef ully) guar antee you r pas sing this cou rse. D rink mode rat ely .
Amusin S C A r y
gly
THE C2005 LOCAL GOVERNMENT REVIEWER - 296 -
and collect reasonable tuition fees and other school Sec1. Section 450 of Republic Act No. 7160, otherwise
charges in educational institutions supported by the city known as the Local Government Code of 1991, is hereby
government; amended to read as follows:
(xi) Establish a scholarship fund for the poor but "Section 450. Requisites for Creation. —
deserving students in schools located within its (a) A municipality or a cluster of barangays may be
jurisdiction or for students residing within the city; converted into a component city if it has a locally generated
(xii) Approve measures and adopt quarantine average annual income, as certified by the Department of
regulations to prevent the introduction and spread of Finance, of at least One hundred million pesos
diseases; (P100,000,000.00) for the last two (2) consecutive years based
(xiii) Provide for an efficient and effective system of on 2000 constant prices, and if it has either of the following
solid waste and garbage collection and disposal; prohibit requisites:
littering and the placing or throwing of garbage, refuse (i) a contiguous territory of at least one hundred (100)
and other filth and wastes; square kilometers, as certified by the Land Management
(xiv) Provide for the care of disabled persons, Bureau; or
paupers, the aged, the sick, persons of unsound mind, (ii) a population of not less than one hundred fifty thousand
abandoned minors, juvenile delinquents, drug (150,000) inhabitants, as certified by the National
dependents, abused children and other needy and Statistics Office.
disadvantaged persons, particularly children and youth The creation thereof shall not reduce the land area,
below eighteen (18) years of age; and subject to population and income of the original unit or units at the time of
availability of funds, establish and provide for the said creation to less than the minimum requirements prescribed
operation of centers and facilities for said needy and herein.
disadvantaged persons; (b) The territorial jurisdiction of a newly-created city shall
(xv) Establish and provide for the maintenance and be properly identified by metes and bounds. The requirement on
improvement of jails and detention centers, institute a land area shall not apply where the city proposed to be created
sound jail management program, and appropriate funds is composed of one (1) or more islands. The territory need not
for the subsistence of detainees and convicted prisoners be contiguous if it comprises two (2) or more islands.
in the city; (c) The average annual income shall include the income
(xvi) Establish a city council whose purpose is the accruing to the general fund, exclusive of special funds,
promotion of culture and the arts, coordinate with transfers, and non-recurring income."
government agencies and non-governmental
organizations and, subject to the availability of funds,
appropriate funds for the support and development of Gordon vs. Verdiano II (1988)
the same; and
(xvii) Establish a city council for the elderly which Facts:
shall formulate policies and adopt measures mutually Yambao operated 2 drugstores in Olongapo . One was temporarily
beneficial to the elderly and to the community; provide closed down by FDA after a test buy operation. After receiving the
incentives for non-governmental agencies and entities same report, Mayor Gordon subsequently revoked the mayor’s
and, subject to the availability of funds, appropriate permit for rampant violations of pharmacy and dangerous drugs
funds to support programs and projects for the benefit of law. He ordered permanent closure. FDA lifted the temporary
the elderly; and closure and Yambao sought reissuance of permit. Yambao then
(6) Exercise such other powers and perform such other sought the permission from FDA to exchange location of 2
duties and functions as may be prescribed by law or drugstores for business preference. FDA granted but Mayor
ordinance. disapproved and suspended permit of 2nd drugstore.
(b) The members of the sangguniang panlungsod of
component cities shall receive a minimum monthly Issue: Who has the power to grant and revoke licenses for
compensation corresponding to Salary Grade twenty-five (25) operation of drug stores in the city
and members of the sangguniang panlungsod of highly- Held:
urbanized cities shall receive a minimum monthly compensation Mayor has no authority to revoke a business permit for the
corresponding to Salary Grade twenty-seven (27), as prescribed violation of the Pharmacy Law or Dangerous Drugs Act. It however
under R.A. 6758 and the implementing guidelines issued has authority to approve or disapprove the exchange of locations
pursuant thereto. requested by Yambao.
An application to establish a drug store in Olongapo must be filed
RA9009 (2001) – An Act Amending Section 450 Of RA7160 w/ the office of the Mayor and must show that the applicant has
LGC Of 1991, By Increasing The Average Annual Income complied with the existing ordinances in health and sanitation,
Requirement For A Municipality Or Cluster Of Barangays To Be location or zoning, fire or building and other local requirements. On
Converted Into A Component City the other hand, the authorization to operate issued by the FDA is a
condition precedent to the grant of a mayor's permit to the drug
store seeking to operate within the limits of the city. This

The Apocryphal Maggots:


Rainier, Chrisgel, Corina, Geoffry, Grace and Sylvie Blanche
The Flibbertigibbet Worms:
Golda, Gladys and Melyjane
CA VE AT: By simply r ea ding this re vi ew er a t the end o f the sem este r wi ll (hopef ully) guar antee you r pas sing this cou rse. D rink mode rat ely .
Amusin S C A r y
gly
THE C2005 LOCAL GOVERNMENT REVIEWER - 297 -
requirement is imperative. Hence, a permit issued by the mayor to No.A license or permit is not in the nature of a contract but a
a drug store not previously cleared with and licensed by the said special privilege, thus estoppel cannot apply in this case. The fact
agency will be a nullity. that petitioner acquiesced in the special conditions imposed by the
However, the issuance of the Mayor’s permit is not mandatory City Mayor in subject business permit does not preclude it from
once the FDA has licensed the operation of the applicant challenging the said imposition, which is ultra vires or beyond the
drugstore. The applicant still has to comply w/ the local ambit of authority of respondent City Mayor. Ultra vires acts or acts
requirements of the city. Should there be no compliance w/ the which are clearly beyond the scope of one's authority are null and
local requirements, the mayor in the exercise of his own authority void and cannot be given any effect. The doctrine of estoppel
under the charter may refuse to grant the permit sought. The cannot operate to give effect to an act which is otherwise null and
power to approve a license includes by implication, even if not void or ultra vires.
expressly granted, the power to revoke it.
Thus, if the FDA grants a license upon its finding that the applicant The issuance of business licenses and permits by a municipality or
drug store has complied with the requirements of the general laws city is essentially regulatory in nature. The authority, which
and the implementing administrative rules and regulations, it is devolved upon local government units to issue or grant such
only for their violation that the FDA may revoke the said license. licenses or permits, is essentially in the exercise of the police
Necessarily, the city mayor may only revoke the permits issued for power of the state within the contemplation of the general welfare
violation of the local requirements imposed. clause of the Local Government Code.

NOTES:
Since LGU is not bound to allow operations even after the Lim vs. CA (2002)
proper agency had given its license, LGU’s can make it more
diffcicult for this business to operate. Facts:
Policemen under Mayor Lim’s instructions inspected and
Q: Can Mayor require compliance w/ all the national laws and investigated Bistro Pigalle’s license as well as the work permits
regulations before issuing the permit? (ie present in the permit) and health certificates of its staff causing the stoppage of work in
A: Yes. The presence of this clause does not invalidate the Bistro’s night club and restaurant operations. Lim also refused to
permit. But the finding of violation must be made by the nat’l accept Bistro’s application for a business license, as well as the
agency and not the mayor. work permit applications of Bistro’s staff, for the year 1993.Bistro
filed before the trial court a petition against Manila Mayor Lim..

Acebedo Optical Company vs. CA (2000) Held:


The law expressly provides for such authority. Section 11 (l), Article
Facts: II of the Revised Charter of the City of Manila and Section 455 (3)
Acebedo applied with the Office of the Mayor of Iligan City for a (iv) of the LGC is clear that the power of the mayor to issue
business permit and was issued subject to the certain conditions. business licenses and permits necessarily includes the corollary
SOPI lodged a complaint against the Acebedo before the Office of power to suspend, revoke or even refuse to issue the same.
the City Mayor, alleging that Acebedo had violated the conditions However, the power to suspend or revoke these licenses and
set forth in its business permit. permits is expressly premised on the violation of the conditions of
these permits and licenses. Similarly, the power to refuse to issue
Issue1: What is a business permit? such licenses and permits is premised on non-compliance with the
Held: prerequisites for the issuance of such licenses and permits. The
The grant of a license or permit to do business is usually granted mayor must observe due process in exercising these powers,
by the local authorities which authorizes the person, natural or which means that the mayor must give the applicant or licensee
otherwise, to engage in business or some form of commercial notice and opportunity to be heard.
activity. Whereas, the issuance of a license to engage in the
practice of a particular profession is issued by the Board or True, the mayor has the power to inspect and investigate private
Commission tasked to regulate the particular profession. It is the commercial establishments for any violation of the conditions of
grant of authority to a natural person to engage in the practice or their licenses and permits. However, the mayor has no power to
exercise of his or her profession. order a police raid on these establishments in the guise of
inspecting or investigating these commercial establishments
ISSUE 2: WON the business permit issued by the city Mayor is
a contract entered into by Iligan City in the exercise of its
proprietary functions, such that although petitioner agreed to Mathay Jr. vs. Felt Foods, Inc. (1999)
such conditions, it cannot be held in estoppel since ultra vires
acts cannot be given effect. Facts:
Mathay issued a temporary business permit valid until Dec 31,
HELD: 1997 in favor of respondent to operate its business, Club Giorgio.
Sometime in November, there were police reports that respondent

The Apocryphal Maggots:


Rainier, Chrisgel, Corina, Geoffry, Grace and Sylvie Blanche
The Flibbertigibbet Worms:
Golda, Gladys and Melyjane
CA VE AT: By simply r ea ding this re vi ew er a t the end o f the sem este r wi ll (hopef ully) guar antee you r pas sing this cou rse. D rink mode rat ely .
Amusin S C A r y
gly
THE C2005 LOCAL GOVERNMENT REVIEWER - 298 -
violated terms of its permit. Petitioner ordered the closure of the City of Quezon vs. Lexber, Inc. (2001)
establishment and ordered the city legal dept to investigate the
matter. Due notice was sent to respondent but it failed to reply. Facts:
Upon finding that respondent violated permit, petitioner cancelled A Tri-Partitie MOA was drawn between QC, represented by its
permit. Respondent went to RTC to annul the closure order. RTC mayor Brigido Simon, Jr., Lexber, and the Municipality of Antipolo,
issued injunction. CA affirmed. whereby a parcel of land in Antipolo was to be used as a garbage
dumping site by QC and other M. Mla. cities or municipalities, for a
Issue: WON cause of action has been mooted due to expiration of 5-year period from Jan. 1991 to Dec. 1995. A 2nd contract was
respondent’s permit entered into where Lexber shall provide maintenance services in
Held: the form of manpower, equipment and engineering operations for
Respondent’s permit has already expired. It is a widely accepted the dumpsite
rule that courts will not assume jurisdiction over a case when it has In view of the idle state of the dumpsite for more than year, Lexber
been rendered moot by a supervening event such as the expiration also sought clarification from QC regarding its intention on the
of a contract. This forecloses the complainant’s right to demand dumpsite project.
specific performance under the terms of expired contract from the QC, through Mayor Mathay, denied any liability under the contract
defendant. Any loss or damage suffered by the complainant from on the ground that the same was invalid and unenforceable.
the alleged unlawful act under the terms and during the existence
of the expired contract may be remedied by a claim for damages Issue: WON the contract is void ab initio
and not by a writ of injunction to enjoin the effects of the expiration Held:
of the contract. No. PD 1445 does not provide that the absence of an appropriation
law ipso facto makes a contract entered into by an LGU null and
NOTES: void Sec. 84 specifically provides: “Revenue funds shall not be
Q: What if police visited and saw pirated cds or prostitutes? Can paid out of any public treasury or depository except in pursuance
they shut down place or make arrests? of an appropriation law or other specific statutory authority.”
A: Yes. Can shut down and arrest so long as they are given Consequently, public funds may be disbursed not only pursuant to
chance to defend themselves. If there’s a violation of nat’l law, an appropriation law, but also in pursuance of other specific
can shut down. statutory authority.

Q: Are raids allowed? (outside this case) Now, LGC of 1991 prohibits the city mayor from entering into
A: Yes, raids are not conducted by police only but jointly w/ gov’t contracts for the public welfare, unless and until there is prior
agency. Except if there’s an ordinance prohibiting raids. authority from the city council.

Negros Oriental II Electric Cooperative Inc. vs. Sangguniang Dadole vs. COA (2002)
Panglungsod ng Dumaguete (1987)
Facts:
Issue: WON the Sangguniang Panglungsod has the authority to In 1986, RTC and MTC judges of Mandaue city started receiving
issue subpoenas and punish non-members for legislative contempt monthly allowances of P1,260 each through the yearly
Held: appropriation ordinance enacted by the Sangguniang Panlungsod
The Sangguniang panglungsod has no authority to issue of the city. (DBM) issued the Local Budget Circular 55 which
subpoenas and punish non-members for legislative contempt. The provided that: such additional allowances in the form of
contempt power is sui generis and local legislative bodies cannot honorarium at rates not exceeding P1,000 in provinces and cities
correctly claim to possess it for the same reasons that the national and P700 in municipalities may be granted, effective immediately
legislature does. The power attaches not to the discharge of
legislative functions per se but to the character of the legislature as Issue: WON LBC 55 of the DBM is void (for going beyond the
one of the 3 independent and coordinate branches of the gov’t. supervisory powers of the Pres. and for not having been published)
The same thing cannot be said of local legislative bodies which are Held:
mere creatures of law. The power to subpoena witnesses and the YES, the Pres. or any of his or her alter egos cannot interfere in
power to punish non-members for contempt may not also be local affairs as long as the concerned LGU acts within the
implied in the delegation of legislative power as such partake of a parameters of the law and the Constitution. Any directive therefore
judicial nature. by the Pres. or any of his or her alter egos seeking to alter the
wisdom of a law-conforming judgment on local affairs of a LGU is a
NOTE: patent nullity because it violates the principle of local autonomy
Still good law. LGC did not grant it power to issue subpoena or and separation of powers of the executive and legislative
contempt power. departments in governing municipal corporations.

Sec. 458 of RA 7160, the law that supposedly serves as the legal
basis of LBC 55, allows the grant of additional allowances “when

The Apocryphal Maggots:


Rainier, Chrisgel, Corina, Geoffry, Grace and Sylvie Blanche
The Flibbertigibbet Worms:
Golda, Gladys and Melyjane
CA VE AT: By simply r ea ding this re vi ew er a t the end o f the sem este r wi ll (hopef ully) guar antee you r pas sing this cou rse. D rink mode rat ely .
Amusin S C A r y
gly
THE C2005 LOCAL GOVERNMENT REVIEWER - 299 -
the finances of the city government allow.” The said provision days from the date of effectivity of said Act, unless otherwise
does not authorize setting a definite maximum limit to the provided therein.
additional allowances granted to judges. Thus, we need not
belabor the point that the finances of a city government may allow Sec461. Requisites for Creation. —
the grant of additional allowances higher than P1,000 if the (a) A province may be created if it has an average annual
revenues of the said city government exceed its annual income, as certified by the Department of Finance, of not less
expenditures. than Twenty million pesos (P20,000,000.00) based on 1991
constant prices and either of the following requisites:
LBC 55 is also void on account of its lack of publication in violation (i) a contiguous territory of at least two thousand (2,000)
of the SC ruling in Tanada v. Tuvera. square kilometers, as certified by the Lands
Management Bureau; or
Issue2: WON the yearly appropriation ordinance enacted by (ii) a population of not less than two hundred fifty thousand
Mandaue City that provides for additional allowances to judges (250,000) inhabitants as certified by the National
contravenes the annual appropriation laws enacted by Cong. Statistics Office:
Held: Provided, That, the creation thereof shall not reduce the
No. COA failed to prove that Mandaue City used the IRA to spend land area, population, and income of the original unit or units at
for the additional allowances of the judges. Moreover, the DBM the time of said creation to less than the minimum requirements
neither conducted a formal review nor ordered a disapproval of prescribed herein.
Mandaue City’s appropriation ordinances, in accordance with the (b) The territory need not be contiguous if it comprise two
procedure outlined in Secs. 326 and 327 of RA 7160: (2) or more islands or is separated by a chartered city or cities
Sec. 326. Review of Appropriation Ordinances of Provinces, which do not contribute to the income of the province.
Highly Urbanized Cities, Independent Component Cities and (c) The average annual income shall include the income
Municipalities within the Metropolitan Mla. Area. in accordance accruing to the general fund, exclusive of special funds, trust
with the immediately succeeding sec. funds, transfers and non-recurring income.

Sec. 327. Review of Appropriation Ordinances of Component Sec462. Existing Sub-Provinces. — Existing sub-provinces
Cities and Municipalities. – The sangguniang panlalawigan shall are hereby converted into regular provinces upon approval by a
review the ordinance authorizing annual or supplemental majority of the votes cast in a plebiscite to be held in the said
appropriations of component cities and municipalities in the same subprovinces and the original provinces directly affected. The
manner and within the same period prescribed for the review of plebiscite shall be conducted by the COMELEC simultaneously
other ordinances. with the national elections following the effectivity of this Code.
If within 90 days from receipt of copies of such ordinance, the SP The new legislative districts created as a result of such
takes no action thereon, the same shall be deemed to have been conversion shall continue to be represented in Congress by the
reviewed in accordance with law and shall continue to be in full duly-elected representatives of the original districts out of which
force and effect. city. said new provinces or districts were created until their own
representatives shall have been elected in the next regular
NOTES: congressional elections and qualified.
Is this good idea? LGU gives allowance to judges? Does this The incumbent elected officials of the said subprovinces
impair integrity of court? converted into regular provinces shall continue to hold office
until June 30, 1992. Any vacancy occurring in the offices
occupied by said incumbent elected officials, or resulting from
expiration of their terms of office in case of a negative vote in the
The Province – LGC
plebiscite results, shall be filled by appointment by the
President. The appointees shall hold office until their successors
CHAPTER I: Role and Creation of the Province shall have been elected in the regular local elections following
Sec459. Role of the Province. — The province, composed of the plebiscite mentioned herein and qualified. After effectivity of
cluster of municipalities, or municipalities and component cities, such conversion, the President shall fill up the position of
and as a political and corporate unit of government, serves as governor of the newly-created province through appointment if
dynamic mechanism for developmental processes and effective none has yet been appointed to the same as hereinbefore
governance of local government units within its territorial provided, and shall also appoint a vice-governor and the other
jurisdiction. members of the sangguniang panlalawigan, all of whom shall
likewise hold office until their successors shall have been
Sec460. Manner of Creation. — A province may be created, elected in the next regular local elections and qualified.
divided, merged, abolished, or its boundary substantially altered, All qualified appointive officials and employees in the career
only by an Act of Congress and subject to approval by a majority service of the said subprovinces at the time of their conversion
of the votes cast in a plebiscite to be conducted by the into regular provinces shall continue in office in accordance with
COMELEC in the local government unit or units directly affected. civil service law, rules and regulations.
The plebiscite shall be held within one hundred twenty (120)

The Apocryphal Maggots:


Rainier, Chrisgel, Corina, Geoffry, Grace and Sylvie Blanche
The Flibbertigibbet Worms:
Golda, Gladys and Melyjane
CA VE AT: By simply r ea ding this re vi ew er a t the end o f the sem este r wi ll (hopef ully) guar antee you r pas sing this cou rse. D rink mode rat ely .
Amusin S C A r y
gly
THE C2005 LOCAL GOVERNMENT REVIEWER - 300 -
CHAPTER II: Provincial Officials in General (a) The provincial governor, as the chief executive of the
Sec463. Officials of the Provincial Government. — provincial government, shall exercise such powers and perform
(a) There shall be in each province a governor, a vice- such duties and functions as provided by this Code and other
governor, members of the sangguniang panlalawigan, a laws.
secretary to the sangguniang panlalawigan, a provincial (b) For efficient, effective and economical governance the
treasurer, a provincial assessor, a provincial accountant, a purpose of which is the general welfare of the province and its
provincial engineer, a provincial budget officer, a provincial inhabitants pursuant to Section 16 of this Code, the provincial
planning and development coordinator, a provincial legal officer, governor shall:
a provincial administrator, a provincial health officer, a provincial (1) Exercise general supervision and control over all
social welfare and development officer, a provincial general programs, projects, services, and activities of the
services officer, a provincial agriculturist, and a provincial provincial government, and in this connection, shall:
veterinarian. (i) Determine the guidelines of provincial policies
(b) In addition thereto, the governor may appoint a and be responsible to the sangguniang panlalawigan for
provincial population officer, a provincial natural resources and the program of government;
environment officer, a provincial cooperative officer, a provincial (ii) Direct the formulation of the provincial
architect, and a provincial information officer. development plan, with the assistance of the provincial
The appointment of a provincial population officer shall be development council, and upon approval thereof by the
optional in the province: Provided, however, That provinces sangguniang panlalawigan, implement the same;
which have existing population offices shall continue to maintain (iii) Present the program of government and
such offices for a period of five (5) years from the date of the propose policies and projects for the consideration of the
effectivity of this Code, after which said offices shall become sangguniang panlalawigan at the opening of the regular
optional. session of the sangguniang panlalawigan every calendar
(c) The sangguniang panlalawigan may: year and as after as may be deemed necessary as the
(1) Maintain existing offices not mentioned in subsections general welfare of the inhabitants and the needs of the
(a) and (b) hereof; provincial government may require;
(2) Create such other offices as may be necessary to carry (iv) Initiate and propose legislative measures to
out the purposes of the provincial government; or the sangguniang panlalawigan and as often as may be
(3) Consolidate the functions of any office with those of deemed necessary, provide such information and data
another in the interest of efficiency and economy; needed or requested by said sanggunian in the
(d) Unless otherwise provided herein, heads of performance of its legislative functions;
departments and offices shall be appointed by the governor with (v) Appoint all officials and employees whose
the concurrence of the majority of all the sangguniang salaries and wages are wholly or mainly paid out of
panlalawigan members, subject to civil service law, rules and provincial funds and whose appointments are not
regulations. The sangguniang panlalawigan shall act on the otherwise provided for in this Code, as well as those he
appointment with fifteen (15) days from the date of its may be authorized by law to appoint;
submission; otherwise the same shall be deemed confirmed; (vi) Represent the province in all its business
(e) Elective and appointive provincial officials shall transactions and sign in its behalf all bonds, contracts,
receive such compensation, allowances, and other emoluments and obligations, and such other documents upon
as may be determined by law or ordinance, subject to the authority of the sangguniang panlalawigan or pursuant to
budgetary limitations on personal services prescribed under Title law or ordinance;
Five, Book II of this Code: Provided, That, no increase in (vii) Carry out such emergency measures as may
compensation shall take effect until after the expiration of the full be necessary during and in the aftermath of man-made
term of all the elective officials approving such increase. and natural disasters and calamities;
(viii) Determine the time, manner and place of
Sec464. Residence and Office. — During the incumbency of payment of salaries or wages of the officials and
the governor, he shall have his official residence in the capital of employees of the province, in accordance with law or
the province. All elective and appointive provincial officials shall ordinance;
hold office in the provincial capital: Provided, That, upon (ix) Allocate and assign office space to provincial
resolution of the sangguniang panlalawigan, elective and and other officials and employees who, by law or
appointive provincial officials may hold office in any component ordinance, are entitled to such space in the provincial
city or municipality within the province for a period of not more capitol and other buildings owned or leased by the
than seven (7) days for any given month. provincial government;
(x) Ensure that all executive officials and
CHAPTER III: Officials and Offices Common to All Provinces employees of the province faithfully discharge their
ARTICLE I: The Provincial Governor duties and functions as provided by law and this Code,
Sec465. The Chief Executive: Powers, Duties, Functions, and cause to be instituted administrative or judicial
and Compensation. — proceedings against any official or employee of the

The Apocryphal Maggots:


Rainier, Chrisgel, Corina, Geoffry, Grace and Sylvie Blanche
The Flibbertigibbet Worms:
Golda, Gladys and Melyjane
CA VE AT: By simply r ea ding this re vi ew er a t the end o f the sem este r wi ll (hopef ully) guar antee you r pas sing this cou rse. D rink mode rat ely .
Amusin S C A r y
gly
THE C2005 LOCAL GOVERNMENT REVIEWER - 301 -
province who may have committed an offense in the economic conditions; and supplemental reports when
performance of his official duties; unexpected events and situations arise at any time
(xi) Examine the books, records and other during the year, particularly when man-made or natural
documents of all offices, officials, agents or employees disasters or calamities affect the general welfare of the
of the province and, in aid of his executive powers and province, region or country;
authority, require all national officials and employees (2) Enforce all laws and ordinances relative to the
stationed in the province to make available to him such governance of the province and the exercise of the
books, records, and other documents in their custody, appropriate corporate powers provided for under Section
except those classified by law as confidential; 22 of this Code, implement all approved policies,
(xii) Furnish copies of executive orders issued by programs, projects, services and activities of the
him to the Office of the President within seventy-two (72) province and, in addition to the foregoing, shall:
hours after their issuance; (i) Ensure that the acts of the component cities
(xiii) Visit component cities and municipalities of the and municipalities of the province and of its officials and
province at least once every six (6) months to deepen employees are within the scope of their prescribed
his understanding of problems and conditions, listen and powers, duties and functions;
give appropriate counsel to local officials and (ii) Call conventions, conferences, seminars, or
inhabitants, inform the officials and inhabitants of meetings of any elective and appointive officials of the
component cities and municipalities of general laws and province and its component cities and municipalities,
ordinances which especially concern them, and including national officials and employees stationed in or
otherwise conduct visits and inspections to ensure that assigned to the province, at such time and place and on
the governance of the province will improve the quality of such subject as he may deem important for the
life of the inhabitants; promotion of the general welfare of the province and its
(xiv) Act on leave applications of officials and inhabitants;
employees appointed by him and the commutation of the (iii) Issue such executive orders for the faithful and
monetary value of leave credits in accordance with law; appropriate enforcement and execution of laws and
(xv) Authorize official trips of provincial officials and ordinances;
employees outside of the province for a period not (iv) Be entitled to carry the necessary firearm
exceeding thirty (30) days; within his territorial jurisdiction;
(xvi) Call upon any national official or employee (v) In coordination with the mayors of component
stationed in or assigned to the province to advise him on cities and municipalities and the National Police
matters affecting the province and to make Commission, formulate the peace and order plan of the
recommendations thereon; coordinate with said official province and upon its approval, implement the same in
or employee in the formulation and implementation of accordance with R.A. No. 6975;
plans, programs and projects; and when appropriate, (vi) Call upon the appropriate national law
initiate an administrative or judicial action against a enforcement agencies to suppress disorder, riot, lawless
national government official or employee who may have violence, rebellion or sedition or to apprehend violators
committed an offense in the performance of his official of the law when public interest so requires and the police
duties while stationed in or assigned to the province; forces of the component city or municipality where the
(xvii) Authorize payment for medical care, disorder or violation is happening are inadequate to cope
necessary transportation, subsistence, hospital or with the situation or the violators;
medical fees of provincial officials and employees who (3) Initiate and maximize the generation of resources and
are injured while in the performance of their official revenues, and apply the same to the implementation of
duties and functions, subject to availability of funds; development plans, program objectives and priorities as
(xviii) Represent the province in inter-provincial or provided for under Section 18 of this Code, particularly
regional sports councils or committees, and coordinate those resources and revenues programmed for agro-
the efforts of component cities or municipalities in the industrial development and country-wide growth and
regional or national palaro or sports development progress and, relative thereto, shall:
activities; (i) Require each head of an office or department
(xix) Conduct an annual palarong panlalawigan, to prepare and submit an estimate of appropriations for
which shall feature traditional sports and disciplines the ensuing calendar year, in accordance with the
included in national and international games in budget preparation process under Title Five, Book II of
coordination with the Department of Education, Culture this Code;
and Sports; and (ii) Prepare and submit to the sanggunian for
(xx) Submit to the Office of the President the approval the executive and supplemental budgets of the
following reports: an annual report containing a summary province for the ensuing calendar year in the manner
of all matters pertinent to the management, provided for under Title Five, Book II of this Code;
administration and development of the province and all (iii) Ensure that all taxes and other revenues of
information and data relative to its political, social and the province are collected, and that provincial funds are

The Apocryphal Maggots:


Rainier, Chrisgel, Corina, Geoffry, Grace and Sylvie Blanche
The Flibbertigibbet Worms:
Golda, Gladys and Melyjane
CA VE AT: By simply r ea ding this re vi ew er a t the end o f the sem este r wi ll (hopef ully) guar antee you r pas sing this cou rse. D rink mode rat ely .
Amusin S C A r y
gly
THE C2005 LOCAL GOVERNMENT REVIEWER - 302 -
applied to the payment of expenses and settlement of (4) Exercise the powers and perform the duties and
obligations of the province, in accordance with law or functions of the governor in cases of temporary vacancy
ordinance; as provided for in Section 46, Book I of this Code; and
(iv) Issue licenses and permits and suspend or (5) Exercise such other powers and perform such other
revoke the same for any violation of the conditions upon duties and functions as may be prescribed by law or
which said licenses or permits had been issued, ordinance.
pursuant to law or ordinance; (b) The vice-governor shall receive a monthly
(v) Adopt adequate measures to safeguard and compensation corresponding to Salary Grade twenty-eight (28)
conserve land, mineral, marine, forest and other as prescribed under R.A. No. 6758 and the implementing
resources of the province, in coordination with the guidelines issued pursuant thereto.
mayors of component cities and municipalities; provide
efficient and effective property and supply management ARTICLE III: The Sangguniang Panlalawigan
in the province; and protect the funds, credits, rights, and Sec467. Composition. —
other properties of the province; and (a) The sangguniang panlalawigan, the legislative body of
(vi) Institute or cause to be instituted administrative or the province, shall be composed of the provincial vice-governor
judicial proceedings for violation of ordinances in the collection of as presiding officer, the regular sanggunian members, the
taxes, fees or charges, and for the recovery of funds and president of the provincial chapter of the liga ng mga barangay,
property, and cause the province to be defended against all suits the president of the panlalawigang pederasyon ng mga
to ensure that its interests, resources and rights shall be sangguniang kabataan, the president of the provincial federation
adequately protected. of sanggunian members of municipalities and component cities
(4) Ensure the delivery of basic services and the provision and the sectoral representatives, as members.
of adequate facilities as provided for under Section 17 of (b) In addition thereto, there shall be three (3) sectoral
this Code, and in addition thereto, shall: representatives: one (1) from the women; and as shall be
(i) Ensure that the construction and repair of determined by the sanggunian concerned within ninety (90)
roads and highways funded by the national government days prior to the holding of the local elections, one (1) from the
shall be, as far as practicable, carried out in a spatially agricultural or industrial workers; and one (1) from other sectors
contiguous manner and in coordination with the including the urban poor, indigenous cultural communities, or
construction and repair of the roads and bridges of the disabled persons.
province and of its component cities and municipalities; (c) The regular members of the sangguniang
and panlalawigan and the sectoral representatives shall be elected
(ii) Coordinate the implementation of technical in the manner as may be provided for by law.
services by national offices for the province and its
component cities and municipalities, including public Sec468. Powers, Duties, Functions and Compensation. —
works and infrastructure programs of the provincial (a) The sangguniang panlalawigan, as the legislative
government and its component cities and municipalities; body of the province, shall enact ordinances, approve
(5) Exercise such other powers and perform such other resolutions and appropriate funds for the general welfare of the
duties and functions as may be prescribed by law or province and its inhabitants pursuant to Section 16 of this Code
ordinance. in the proper exercise of the corporate powers of the province
(c) The provincial governor shall receive a minimum as provided for under Section 22 of this Code, and shall:
monthly compensation corresponding to Salary Grade thirty (30) (1) Approve ordinances and pass resolutions necessary for
prescribed under R.A. No. 6758 and the implementing an efficient and effective provincial government and, in
guidelines issued pursuant thereto. this connection, shall:
(i) Review all ordinances approved by the
ARTICLE II: The Provincial Vice-Governor sangguniang of component cities and municipalities and
Sec466. Powers, Duties, and Compensation. — executive orders issued by the mayors of said
(a) The vice-governor shall: component units to determine whether these are within
(1) Be the presiding officer of the sangguniang panlalawigan the scope of the prescribed powers of the sanggunian
and sign all warrants drawn on the provincial treasury for and of the mayor;
all expenditures appropriated for the operation of the (ii) Maintain peace and order by enacting
sangguniang panlalawigan; measures to prevent and suppress lawlessness,
(2) Subject to civil service law, rules and regulations, disorder, riot, violence, rebellion or sedition and impose
appoint all officials and employees of the sangguniang penalties for the violation of said ordinances;
panlalawigan, except those whose manner of (iii) Approve ordinances imposing a fine not
appointment is specially provided in this Code; exceeding Five thousand pesos (P5,000.00) or
(3) Assume the office of the governor for the unexpired term imprisonment not exceeding one (1) year, or both in the
of the latter in the event of permanent vacancy as discretion of the court, for the violation of a provincial
provided for in Section 44, Book I of this Code; ordinance;

The Apocryphal Maggots:


Rainier, Chrisgel, Corina, Geoffry, Grace and Sylvie Blanche
The Flibbertigibbet Worms:
Golda, Gladys and Melyjane
CA VE AT: By simply r ea ding this re vi ew er a t the end o f the sem este r wi ll (hopef ully) guar antee you r pas sing this cou rse. D rink mode rat ely .
Amusin S C A r y
gly
THE C2005 LOCAL GOVERNMENT REVIEWER - 303 -
(iv) Adopt measures to protect the inhabitants of and priorities of the province as provided for under
the province from harmful effects of man-made or Section 18 of this Code, with particular attention to agro-
natural disasters and calamities, and to provide relief industrial development and country-wide growth and
services and assistance for victims during and in the progress and relative thereto, shall:
aftermath of said disasters and calamities and their (i) Enact the annual and supplemental
return to productive livelihood following said events; appropriations of the provincial government and
(v) Enact ordinances intended to prevent, appropriate funds for specific programs, projects,
suppress and impose appropriate penalties for habitual services and activities of the province, or for other
drunkenness in public places, vagrancy, mendicancy, purposes not contrary to law, in order to promote the
prostitution, establishment and maintenance of houses general welfare of the province and its inhabitants;
of ill repute, gambling and other prohibited games of (ii) Subject to the provisions of Book II of this
chance, fraudulent devices and ways to obtain money or Code and applicable laws and upon the majority vote of
property, drug addiction, maintenance of drug dens, drug all the members of the sangguniang panlalawigan, enact
pushing, juvenile delinquency, the printing, distribution or ordinances levying taxes, fees and charges, prescribing
exhibition of obscene or pornographic materials or the rates thereof for general and specific purposes, and
publications, and other activities inimical to the welfare granting tax exemptions, incentives or reliefs;
and morals of the inhabitants of the province; (iii) Subject to the provisions of Book II of this
(vi) Protect the environment and impose Code and applicable laws and upon the majority vote of
appropriate penalties for acts which endanger the all the members of the sangguniang panlalawigan,
environment, such as dynamite fishing and other forms authorize the provincial governor to negotiate and
of destructive fishing, illegal logging and smuggling of contract loans and other forms of indebtedness;
logs, smuggling of natural resources products and of (iv) Subject to the provisions of Book II of this
endangered species of flora and fauna, slash and burn Code and applicable laws and upon the majority vote of
farming, and such other activities which result in all the members of the sangguniang panlalawigan, enact
pollution acceleration of eutrophication of rivers and ordinances authorizing the floating of bonds or other
lakes, or of ecological imbalance; instruments of indebtedness, for the purpose of raising
(vii) Subject to the provisions of this Code and funds to finance development projects;
pertinent laws, determine the powers and duties of (v) Appropriate funds for the construction and
officials and employees of the province; maintenance or the rental of buildings for the use of the
(viii) Determine the positions and the salaries, province; and upon the majority vote of all the members
wages, allowances and other emoluments and benefits of the sangguniang panlalawigan, authorize the
of officials and employees paid wholly or mainly from provincial governor to lease to private parties such public
provincial funds and provide for expenditures necessary buildings held in a proprietary capacity, subject to
for the proper conduct of programs, projects, services, existing laws, rules and regulations;
and activities of the provincial government; (vi) Prescribe reasonable limits and restraints on
(ix) Authorize the payment of compensation to a the use of property within the jurisdiction of the province;
qualified person not in the government service who fills (vii) Review the comprehensive land use plans and
up a temporary vacancy, or grant honorarium to any zoning ordinances of component cities and
qualified official or employee designated to fill a municipalities and adopt a comprehensive provincial
temporary vacancy in a concurrent capacity, at the rate land use plan, subject to existing laws; and
authorized by law; (viii) Adopt measures to enhance the full
(x) Provide a mechanism and the appropriate implementation of the national agrarian reform program
funds therefor, to ensure the safety and protection of all in coordination with the Department of Agrarian Reform;
provincial government property, public documents, or (3) Subject to the provisions of Book II of this Code, grant
records such as those relating to property inventory, land franchises, approve the issuance of permits or licenses,
ownership, records of births, marriages, deaths, or enact ordinances levying taxes, fees and charges
assessments, taxation, accounts, business permits, and upon such conditions and for such purposes intended to
such other records and documents of public interest in promote the general welfare of the inhabitants of the
the offices and departments of the provincial province, and pursuant to this legislative authority, shall:
government; and (i) Fix and impose reasonable fees and charges
(xi) When the finances of the provincial for all services rendered by the provincial government to
government allow, provide for additional allowances and private persons or entities; and
other benefits to judges, prosecutors, public elementary (ii) Regulate and fix the license fees for such
and high school teachers, and other national activities as provided for under this Code.
government officials stationed or assigned to the (4) Approve ordinances which shall ensure the efficient and
province. effective delivery of basic services and facilities as
(2) Generate and maximize the use of resources and provided for under Section 17 of this Code, and, in
revenues for the development plans, program objectives addition to said services and facilities, shall:

The Apocryphal Maggots:


Rainier, Chrisgel, Corina, Geoffry, Grace and Sylvie Blanche
The Flibbertigibbet Worms:
Golda, Gladys and Melyjane
CA VE AT: By simply r ea ding this re vi ew er a t the end o f the sem este r wi ll (hopef ully) guar antee you r pas sing this cou rse. D rink mode rat ely .
Amusin S C A r y
gly
THE C2005 LOCAL GOVERNMENT REVIEWER - 304 -
(i) Adopt measures and safeguards against (b) The members of the sangguniang panlalawigan shall
pollution and for the preservation of the natural receive a minimum monthly compensation corresponding to
ecosystem in the province, in consonance with approved Salary Grade twenty-seven (27) as prescribed under R.A. No.
standards on human settlements and environmental 6758 and the implementing guidelines issued pursuant thereto.
sanitation;
(ii) Subject to applicable laws, facilitate or provide
for the establishment and maintenance of waterworks Caram vs. COMELEC (1993)
system or district waterworks for supplying water to
inhabitants of component cities and municipalities; Facts:
(iii) Subject to the availability of funds and to The LGC provided for the conversion of the Sub-province of
existing laws, rules and regulations, provide for the Guimaras into a province and the holding of the plebiscite wherein
establishment and operation of vocational and technical voters of the Province of Iloilo and the Sub-province of Guimaras
schools and similar post-secondary institutions; and, with were asked to resolve the issue of the conversion of the sub-
the approval of the Department of Education, Culture province to a regular province. Provincial Board of Canvassers of
and Sports and subject to existing laws on tuition fees, Iloilo issued a certificate proclaiming that the conversion of the
fix reasonable tuition fees and other school charges in Sub-province of Guimaras to a regular province
educational institutions supported by the provincial
government; Held:
(iv) Establish a scholarship fund for the poor but Comelec had no the authority to exclude voters of the Sub-
deserving students in schools located within its province of Guimaras from voting for the positions of the Governor,
jurisdiction or for students residing within the province; Vice-Governor, and Members of the Sangguniang Panlalawigan
(v) Approve measures and adopt quarantine representing the Second District of the Province of Iloilo because
regulations to prevent the introduction and spread of under s462 of the LGC, it was only in the case where the voters
diseases within its territorial jurisdiction; ratified the conversion of the Sub-province of Guimaras to a
(vi) Provide for the care of paupers, the aged, the regular province that the President was empowered to appoint the
sick, persons of unsound mind, abandoned minors, officials of the newly created province. The lawmakers failed to
abused children, disabled persons, juvenile delinquents, foresee that in the event the negative vote in the plebiscite
drug dependents, and other needy and disadvantaged prevailed, the Sub-province of Guimaras would continue to be a
persons, particularly children and youth below eighteen part of the Province of Iloilo and be represented by the officials of
(18) years of age; subject to availability of funds, the province elected in the May 11, 1992 elections. If YES votes
establish and support the operation of centers and prevailed, it would serve no useful purpose to undo what the
facilities for said needy and disadvantaged persons; and COMELEC had done and that there would be no legal basis to call
facilitate efforts to promote the welfare of families below special elections to give a chance to the voters of the Sub-province
the poverty threshold, the disadvantaged, and the of Guimaras to vote for the provincial officials of the Province of
exploited; Iloilo. Official ballots used in the Sub-province of Guimaras
(vii) Establish and provide the maintenance and contained a space for the voting of a Congressman for the Second
improvement of jails and detention centers, institute a Congressional District of Iloilo, a clear indication that the Guimaras
sound jail management program, and appropriate funds voters could vote for said official. There is no evidence that the
for the subsistence of detainees and convicted prisoners voters were not able to vote for congressman due to the lack of
in the province; information dissemination on the part of Comelec that they could
(viii) Establish a provincial council whose purpose do so.
is the promotion of culture and the arts, coordinate with NOTES:
government agencies and non-governmental No need for special elections. The voters had the chance to vote
organizations and, subject to the availability of funds, for congressman but they didn’t. The voters are presumed to have
appropriate funds for the support and development of abstained.
the same;
(ix) Establish a provincial council for the elderly
which shall formulate policies and adopt measures
mutually beneficial to the elderly and to the province; Q&A
and subject to the availability of funds, appropriate funds
to support programs and projects for the elderly; and Question
provide incentives for non-governmental agencies and When I make love, I often start to laugh in the middle of my
entities to support the programs and projects of the orgasm. I can't help it. It is slightly unnerving to my boyfriend
elderly; and though. He has never seen this before. Is this normal or are we
(5) Exercise such other powers and perform such other doing something wrong?
duties and functions as may be prescribed by law or
ordinance. Answer

The Apocryphal Maggots:


Rainier, Chrisgel, Corina, Geoffry, Grace and Sylvie Blanche
The Flibbertigibbet Worms:
Golda, Gladys and Melyjane
CA VE AT: By simply r ea ding this re vi ew er a t the end o f the sem este r wi ll (hopef ully) guar antee you r pas sing this cou rse. D rink mode rat ely .
Amusin S C A r y
gly
THE C2005 LOCAL GOVERNMENT REVIEWER - 305 -
No, you are not doing anything wrong. Many women laugh or Atty. Evangeline De Castro was the Chief of the Legal Affairs and
even cry during or after orgasm. Orgasm can release Complaint Services of the Division of City Schools of Manila
tremendous emotions during this time of enormous physical received a letter from Angel Aguirre, City Legal Officer of Manila,
w/ complaints against her for gross misconduct and conduct
release. Orgasm is also a short period of time during which we
unbecoming of a public officer. Aguirre summoned her to appear
are often at our most vulnerable. This can lead to all sorts of
before the City legal officer for investigation.
feelings rushing out. Because men are taught in our society to
suppress emotions more than women, it is less common for Issue: WON the City Legal Officer of Manila had jurisdiction to
guys -- but it can happen nonetheless. investigate the complaint against the Chief of the Legal Affairs and
It is also possible that you are simply becoming enormously Complaint Services of the Division of City Schools of Manila
ticklish right after orgasm, which is very common for both men Held:
and women. Even the slightest touch can be downright No. The Admin Code, lodged the power to appoint and discipline
uncomfortable due to this sensitivity. first-level EEs including De Castro with the regional director of
Basically, you can reassure him by explaining that this is only DECS. The LGC did not automatically repeal the Admin Code
occurring because you are having such an intense orgasm. If it and implied repeal not lightly presumed.
was a little hiccup you barely noticed, I doubt you would be
laughing your head off! The position of senior legal officer in the Division of City Schools is
not one of the offices covered by the city mayor’s POWER OF
APPOINTMENT under the LGC. There is also no specific provision
in the LGC showing that the POWER TO DISCIPLINE officials in
the Division of City Schools has been devolved from the regional
Appointive Local Officials Common to All Municipalities, director of DECS to the city mayor. There is also no proof that De
Cities, and Provinces – see Human Resource Development Castro is one of the devolved personnel w/c are automatically
reappointed by the local chief executive

Rapisora vs. Civil Service Commission (1993) The SOURCE OF WAGES is not the only criteria in determining
whether the payor may be deemed the ER. Therefore the
Facts: argument that De Castro’s salary is paid out of city funds, is not
CSC disapproved the permanent appointment of petitioner Edgar determinative of her status as a city EE. The most important factor
Rapisora as prov’l health Officer of Benguet for lack of educational is the CONTROL TEST. Who has the power to SUPERVISE and
attainment. CSC also asserted that it is not the Secretary of Health DIRECT the work of the EE concerned?
but the local chief executive, concurred in by a majority of the
members of the sanggunian, must appoint the prov’l health officer
by virtue of the new LGC. Leagues of LGUs and Elective Officials – LGC

Held: Sec491. Purpose of Organization. — There shall be an


CSC abused its discretion when it disapproved Rapisora’s organization of all barangays to be known as the liga ng mga
appointment for lack of educational attainment. As held in previous barangay for the primary purpose of determining the
cases, when necessary, education, experience or training may be representation of the Liga in the sanggunians, and for
used interchangeable to offset deficiencies. The necessity exists if ventilating, articulating and crystallizing issues affecting
the appointee’s training or experience is of such a level that the barangay government administration and securing, through
same would more than supplement the deficiency in education proper and legal means, solutions thereto.
considering the demands of the position in question. The decision
as to when such necessity arises rests upon the sound discretion Sec492. Representation, Chapters, National Liga. — Every
of the appointing authority, who is in the best position to determine barangay shall be represented in said liga by the punong
the needs of his department or agency. barangay, or in his absence or incapacity, by a sanggunian
member duly elected for the purpose among its members, who
The CSC is not empowered to determine or change the kind or shall attend all meetings or deliberations called by the different
nature of the appointment, for it is an essential discretionary power chapters of the liga.
and must be performed by an officer on whom it is vested, the only The liga shall have chapters at the municipal, city, provincial and
condition being that the appointee should possess the minimum metropolitan political subdivision levels.
qualifications required by law. The municipal and city chapters of the liga shall be composed of
the barangay representatives of municipal and city barangays
respectively. The duly elected presidents of component
Aguirre vs. De Castro (1999) municipal and city chapters shall constitute the provincial
chapter or the metropolitan political subdivision chapter. The
Facts: duly elected presidents of highly-urbanized cities, provincial
chapters, the Metropolitan Manila chapter and metropolitan

The Apocryphal Maggots:


Rainier, Chrisgel, Corina, Geoffry, Grace and Sylvie Blanche
The Flibbertigibbet Worms:
Golda, Gladys and Melyjane
CA VE AT: By simply r ea ding this re vi ew er a t the end o f the sem este r wi ll (hopef ully) guar antee you r pas sing this cou rse. D rink mode rat ely .
Amusin S C A r y
gly
THE C2005 LOCAL GOVERNMENT REVIEWER - 306 -
political subdivision chapters shall constitute the National Liga absence, by the vice-mayor or a sanggunian member duly
ng mga Barangay. elected for the purpose by the members, who shall attend all
meetings and participate in the deliberations of the league.
Sec493. Organization. — The liga at the municipal, city,
provincial, metropolitan political subdivision, and national levels Sec498. Powers, Functions and Duties of the League of
directly elect a president, a vice-president, and five (5) members Municipalities. — The league of municipalities shall:
of the board of directors. The board shall appoint its secretary (a) Assist the national government in the formulation and
and treasurer and create such other positions as it may deem implementation of the policies, programs and projects affecting
necessary for the management of the chapter. A secretary- municipalities as a whole;
general shall be elected from among the members of the (b) Promote local autonomy at the municipal level;
national liga and shall be charged with the overall operation of (c) Adopt measures for the promotion of the welfare of all
the liga on national level. The board shall coordinate the municipalities and its officials and employees;
activities of the chapters of the liga. (d) Encourage people's participation in local government
administration in order to promote united and concerted action
Sec494. Ex-Officio Membership in Sanggunians. — The for the attainment of country-wide development goals;
duly elected presidents of the liga at the municipal, city and (e) Supplement the efforts of the national government in
provincial levels, including the component cities and creating opportunities for gainful employment within the
municipalities of Metropolitan Manila, shall serve as ex-officio municipalities;
members of the sangguniang bayan, sangguniang panlungsod, (f) Give priority to programs designed for the total
sangguniang panlalawigan, respectively. They shall serve as development of the municipalities in consonance with the
such only during their term of office as presidents of the liga policies, programs and projects of the national government;
chapters, which in no case shall be beyond the term of office of (g) Serve as a forum for crystallizing and expressing
the sanggunian concerned. ideas, seeking the necessary assistance of the national
government, and providing the private sector avenues for
Sec495. Powers, Functions and Duties of the Liga. — The cooperation in the promotion of the welfare of the municipalities;
liga shall: and
(a) Give priority to programs designed for the total (h) Exercise such other powers and perform such other
development of the barangays and in consonance with the duties and functions as the league may prescribe for the welfare
policies, programs and projects of the national government; of the municipalities.
(b) Assist in the education of barangay residents for
people's participation in local government administration in order ARTICLE III: League of Cities
to promote united and concerted action to achieve country-wide Sec499. Purpose of Organization. — There shall be an
development goals; organization of all cities to be known as the League of Cities for
(c) Supplement the efforts of government in creating the primary purpose of ventilating, articulating and crystallizing
gainful employment within the barangay; issues affecting city government administration, and securing,
(d) Adopt measures to promote the welfare of barangay through proper and legal means, solutions thereto.
officials; The league may form chapters at the provincial level for the
(e) Serve as a forum of the barangays in order to forge component cities of a province. Highly-urbanized cities may also
linkages with government and non-governmental organizations form a chapter of the League. The National League shall be
and thereby promote the social, economic and political well- composed of the presidents of the league of highly-urbanized
being of the barangays; and cities and the presidents of the provincial chapters of the league
(f) Exercise such other powers and perform such other of component cities.
duties and functions which will bring about stronger ties between
barangays and promote the welfare of the barangay inhabitants. Sec500. Representation. — Every city shall be represented in
the league by the city mayor or in his absence, by the city vice-
ARTICLE II: League of Municipalities mayor or a sanggunian member duly elected for the purpose by
Sec496. Purpose of Organization. — There shall be an the members, who shall attend all meetings and participate in
organization of all municipalities to be known as league of the deliberations of the league.
municipalities for the primary purpose of ventilating, articulating
and crystallizing issues affecting municipal government Sec501. Powers, Functions and Duties of the League of
administration, and securing, through proper and legal means, City. — The league of cities shall:
solutions thereto. (a) Assist the national government in the formulation and
The league shall form provincial chapters composed of the implementation of the policies, programs and projects affecting
league presidents for all component municipalities of the cities as a whole;
province. (b) Promote local autonomy at the city level;
(c) Adopt measures for the promotion of the welfare of all
Sec497. Representation. — Every municipality shall be cities and its officials and employees;
represented in the league by the municipal mayor of in his

The Apocryphal Maggots:


Rainier, Chrisgel, Corina, Geoffry, Grace and Sylvie Blanche
The Flibbertigibbet Worms:
Golda, Gladys and Melyjane
CA VE AT: By simply r ea ding this re vi ew er a t the end o f the sem este r wi ll (hopef ully) guar antee you r pas sing this cou rse. D rink mode rat ely .
Amusin S C A r y
gly
THE C2005 LOCAL GOVERNMENT REVIEWER - 307 -
(d) Encourage people's participation in local government (h) Exercise such other powers and perform such other
administration in order to promote united and concerted action duties and functions as the league may prescribe for the welfare
for the attainment of country-wide development goals; of the provinces and metropolitan political subdivisions.
(e) Supplement the efforts of the national government in
creating opportunities for gainful employment the cities; ARTICLE V: Provisions Common to All Leagues
(f) Give priority to programs designed for the total Sec505. Funding. —
development of cities in consonance with the policies, programs (a) All leagues shall derive its funds from contributions of
and projects of the national government; member local government units and from fund-raising projects
(g) Serve as a forum for crystallizing and expressing and activities without the necessity of securing permits therefor:
ideas, seeking the necessary assistance of the national Provided, That the proceeds from said fund-raising projects and
government and providing the private sector avenues for activities shall be used primarily to fund the projects for which
cooperation in the promotion of the welfare of the cities; and the said proceeds have been raised, subject to the pertinent
(h) Exercise such other powers and perform such other provision of this Code and the pertinent provisions of the
duties and functions as the league may prescribe for the welfare Omnibus Election Code.
of the cities. (b) All funds of leagues shall be deposited as trust funds
with its treasurer and shall be disbursed in accordance with the
ARTICLE IV: League of Provinces board of director's resolutions, subject to pertinent accounting
Sec502. Purpose of Organization. — There shall be an and auditing rules and regulations: Provided, That the treasurer
organization of all provinces to be known as the League of shall be bonded in an amount to be determined by the board of
Provinces for the primary purpose of ventilating, articulating and directors. The funds of a chapter shall be deposited as chapter
crystallizing issues affecting provincial and metropolitan political funds and funds of the national league shall be deposited as
subdivision government administration, and securing, through national funds.
proper and legal means, solutions thereto. For this purpose, the
Metropolitan Manila Area and any metropolitan political Sec506. Organizational Structure. — To ensure the effective
subdivision shall be considered as separate provincial units of and efficient administration, the leagues for municipalities, cities
the league. and provinces shall elect chapter-level and national-level boards
of directors and a set of officers headed by the president. A
Sec503. Representation. — Every province shall be secretary-general shall be chosen from among the national
represented in the league by the provincial governor or in his league members to manage the day to day operation and
absence, by the provincial vice-governor or a sanggunian activities of the national league. The board of directors on the
member duly elected for the purpose by the members, who shall chapter or national level may create such other positions as may
attend all meetings and participate in the deliberations of the be deemed necessary for the management of the chapters and
league. of the national league. The national board of directors of the
leagues for municipalities, cities or provinces shall coordinate
Sec504. Powers, Functions and Duties of the League of programs, projects and activities of chapter and the national-
Provinces. — The league of provinces shall: level league.
(a) Assist the national government in the formulation and
implementation of the policies, programs and projects affecting Sec507. Constitution and By-laws of the Liga and the
provinces as a whole; Leagues. — All other matters not herein otherwise provided for
(b) Promote local autonomy at the provincial level; affecting the internal organization of the leagues of local
(c) Adopt measures for the promotion of the welfare of all government units shall be governed by their respective
provinces and its officials and employees; constitution and by-laws which are hereby made suppletory to
(d) Encourage people's participation in local government the provision of this Chapter: Provided, That said Constitution
administration in order to promote united and concerted action and By-laws shall always conform to the provisions of the
for the attainment of countrywide employment within the Constitution and existing laws.
province;
(e) Supplement the efforts of the national government in CHAPTER II: Leagues and Federation of Local Elective
creating opportunities for gainful employment within the Officials
province; Sec508. Organization. —
(f) Give priority to programs designed for the total (a) Vice-governors, vice-mayors, sanggunian members of
development of the provinces in consonance with the policies, barangays, municipalities, component cities, highly-urbanized
programs and projects of the national government; cities and provinces, and other elective local officials of local
(g) Serve as a forum for crystallizing and expressing government units, including those of the Metropolitan Manila
ideas, seeking the necessary assistance of the national Area and any metropolitan political subdivisions, may form their
government and providing the private sector avenues for respective leagues or federation, subject to applicable
cooperation in the promotion of the welfare of the provinces; and provisions of this Title and pertinent provisions of this Code;
(b) Sanggunian members of component cities and
municipalities shall form a provincial federation and elect a

The Apocryphal Maggots:


Rainier, Chrisgel, Corina, Geoffry, Grace and Sylvie Blanche
The Flibbertigibbet Worms:
Golda, Gladys and Melyjane
CA VE AT: By simply r ea ding this re vi ew er a t the end o f the sem este r wi ll (hopef ully) guar antee you r pas sing this cou rse. D rink mode rat ely .
Amusin S C A r y
gly
THE C2005 LOCAL GOVERNMENT REVIEWER - 308 -
board of directors and a set of officers headed by the president. Miguel vs. CA (1994) supra
The duly elected president of the provincial federation of
sanggunian members of component cities and municipalities Under the RRI (Rule 29), the election of the first set of officers of
shall be an ex-officio member of the sangguniang panlalawigan the national and local chapters of the Liga cannot be held unless a
concerned and shall serve as such only during his term of office constitution and by-laws for the Liga is first adopted and ratified by
as president of the provincial federation of sanggunian members the incumbent members of the board of the Pambansang
of component cities and municipalities, which in no case shall be Katipunang ng mga Barangay (the members of the board were the
beyond the term of office of the sanggunian panlalawigan presidents of the chapters). Inasmuch as no constitution and by-
concerned. laws had been drafterd by the committee and ratified by the
barangay national assembly when the said Ad Hoc Committee of
Sec509. Constitution and By-laws. — The leagues or QC met on April 1992, the said meeting and the elections
federations shall adopt a Constitution and by-laws which shall conducted therein were invalid.
govern their internal organization and operation: Provided, That
said Constitution and by-laws shall always conform to the
provision of the Constitution and existing laws.
Viola vs. Alunan III (1997)
Sec510. Funding. — The leagues and federations may derive
funds from contributions of individual league or federation Facts:
members or from fund-raising projects or activities. The local Cesar G. Viola, filed petition for prohibition challenging the validity
government unit concerned may appropriate funds to support of Art. III, §§1-2 of the Revised Implementing Rules and Guidelines
the leagues or federation organized pursuant to this Section, for the General Elections of the Liga ng mga Barangay Officers in
subject to the availability of funds. so far as they provide for the election of first, second and third vice
presidents and for auditors for the National Liga ng mga Barangay
and its chapters.
Galarosa vs. Valencia (1993)
Issue: WON §§1-2 of the Implementing Rules are valid
Held:
Facts: Yes. The creation of the additional positions is authorized by §493
Galarosa was incumbent president of the katipunang bayan or of LGC w/cn in fact requires — and not merely authorizes — the
Association of Barangay Councils (ABC) of Sorsogon and was board of directors to "create such other positions as it may deem
appointed ex officio member of the Sangguniang Bayan (SB) of necessary for the management of the chapter" and belies
Sorsogon pursuant to EO 342. Lasay filed for declaratory relief petitioner's claim that §493 limits the officers of a chapter to the
saying that Galarosa's term as an ex officio member of the SB of president, VP, 5 members of the board of directors, secretary, and
Sorsogon is coterminous with that of the said SB w/c expired on treasurer. Also, the creation of these positions was actually made
june 30. in the constitution and by-laws of the Liga ng mga Barangay
adopted by the 1st Brgy National Assembly.
Issue: WON Galarosa can continue to serve as a member of the
SB beyond 30 June 1992, the date when the term of office of the Congress can delegate the power to create positions such as
elective members of the SB of Sorsogon expired these. §493 embodies a fairly intelligible standard “deemed
Held: necessary for the management of the chapters,". There is no
Yes, on the basis of the "hold-over doctrine". There is no law which undue delegation of power by Congress. SC decisions have
prohibits them from holding over as members of the sangguniang upheld the validity of reorganization statutes authorizing the
bayan. In fact, the IRR of the LGC and two DILG Memorandum President of the Philippines to create, abolish or merge offices in
Circulars even expressly allow that hold-over authority to ABC the executive department.
presidents. The purpose of a hold over is to prevent a hiatus in
the government pending the time when the successor may be
chosen and inducted into office. Section 494 of the Local
Bito-onon vs. Fernandez (2001)
Government Code could not have been intended to allow a gap in
the representation of the barangays, through the presidents of the
ABC, in the sanggunian. Facts:
Bito-Onon was proclaimed winner as Executive VP of the Liga ng
mga Brgy Provincial Chapter of Palawan. Quejano filed post
NOTES:
proclamation protest w/ the Board of Election Supervisors (BES)
Q: Is this doctrine still relevant?
which decided against him. Quejano filed petition for review w/
A: Yes, because the local electons and the liga elections not
RTC. Onon filed MTD claiming that RTC have no JD to review
held at the same date. Thus, they don’t assume office at the
BES decisions in any post proclamation protest.
same date.
DILG MC 97-193: review of BES decision is w/ any regular courts
Guidelines by the Liga: review of BES decision is w/ National Liga
Board

The Apocryphal Maggots:


Rainier, Chrisgel, Corina, Geoffry, Grace and Sylvie Blanche
The Flibbertigibbet Worms:
Golda, Gladys and Melyjane
CA VE AT: By simply r ea ding this re vi ew er a t the end o f the sem este r wi ll (hopef ully) guar antee you r pas sing this cou rse. D rink mode rat ely .
Amusin S C A r y
gly
THE C2005 LOCAL GOVERNMENT REVIEWER - 309 -

Issue1: Does the president’s power of general supervision extend


to the liga ng mga brgy which is not an LGU
Held:
YES. DOJ ruled that the liga ng mga brgy is a gov’t organization,
being an association, federation or league or union created by law
or by authority of law whose members are either appointed or
elected gov’t officals.

The ligas are primarily governed by the LGCode. However, their


respective constitutions and by laws shall govern all matters
affecting the internal organization of the liga not otherwise provided
for in the LGCode, provided that such consti and bylaws shall be Par t IV: Miscel laneous and Final
suppletory to Book II Title VI of LGCode and shall always conform
to Constitution and existing laws.
Provisions
In authorizing the filing of the petition for review of BES decision w/ Provisions for Implementation – LGC
regular courts under MC 97-193 , the DILG sec in effect amended
and modified the Guidelines promulgated by the Liga which Sec511. Posting and Publication of Ordinances with Penal
provides that review of BES decision is w/ National Liga Board. Sanctions. —
The amendment of the guidelines is more than an exercise of (a) Ordinances with penal sanctions shall be posted at
supervision but is an exercise of the power of control w/c the prominent places in the provincial capitol, city, municipal or
president doesn’t have over the liga. barangay hall, as the case may be, for a minimum period of
three (3) consecutive weeks. Such ordinances shall also be
published in a newspaper of general circulation, where
available, within the territorial jurisdiction of the local government
Q&A unit concerned, except in the case of barangay ordinances.
Unless otherwise provided therein, said ordinances shall take
effect on the day following its publication, or at the end of the
Question
period of posting, whichever occurs later.
Can a person distinguish whether a woman is a virgin by the
(b) Any public officer or employee who violates an
color and size of her nipples? ordinance may be meted administrative disciplinary action,
without prejudice to the filing of the appropriate civil or criminal
Answer action.
I was scratching my head the first time I read this question a few (c) The secretary to the sanggunian concerned shall
months ago. Since that time, I have read the same question from transmit official copies of such ordinances to the chief executive
hundreds of readers! I think a new urban legend/old wives' tale officer of the Office Gazette within seven (7) days following the
must be in the works. approval of the said ordinance for publication purposes. The
No, you cannot tell whether a woman -- or a man, for that Official Gazette may publish ordinances with penal sanctions for
matter -- has had sex by the size, shape, or color of her or his archival and reference purposes.
nipples. I am making a guess here, but I think the origin of this
myth may come from the fact that women who have been Sec512. Withholding of Benefits Accorded to Barangay
Officials. — Willful and malicious withholding of any of the
pregnant do sometimes have larger, darker nipples. But this has
benefits accorded to barangay officials under Section 393 hereof
nothing to do with simply being sexually active.
shall be punished with suspension or dismissal from office of the
Folks, I have heard hundreds of people insist that you can tell if official or employee responsible therefor.
a woman is a virgin by everything from how she walks to
something about her eyes. The reality is that the only way to Sec513. Failure to Post and Publish the Itemized Monthly
know if someone has ever had sex is to ask them. Nothing Collections and Disbursements. — Failure by the local
magical happens to a person's body after they have sex for the treasurer of the local chief accountant to post the itemized
first time. Do people look different after they've ridden a horse monthly collections and disbursements of the local government
for the first time or had their first drink? Well, yes, but only for a unit concerned within ten (10) days following the end of every
few hours. :) month and for at least two (2) consecutive weeks at prominent
So, stop looking for ways to "know." Whether or not someone is places in the main office building of the local government unit
a "virgin" is such an ambiguous question as to almost be concerned, its plaza and main street, and to publish said
unanswerable anyway. itemization in a newspaper of general circulation, where
available, in the territorial jurisdiction of such unit, shall be
punished by a fine not exceeding Five hundred pesos (P500.00)

The Apocryphal Maggots:


Rainier, Chrisgel, Corina, Geoffry, Grace and Sylvie Blanche
The Flibbertigibbet Worms:
Golda, Gladys and Melyjane
CA VE AT: By simply r ea ding this re vi ew er a t the end o f the sem este r wi ll (hopef ully) guar antee you r pas sing this cou rse. D rink mode rat ely .
Amusin S C A r y
gly
THE C2005 LOCAL GOVERNMENT REVIEWER - 310 -
or by imprisonment not exceeding one (1) month, or both such The same penalty shall be imposed upon any officer charged
fine and imprisonment, at the discretion of the court. with the duty of collecting the tax due on real property who
willfully or negligently fails to collect the tax and institute the
Sec514. Engaging in Prohibited Business Transactions or necessary proceedings for the collection of the same.
Possessing Illegal Pecuniary Interest. — Any local official and Any other officer required by this Code to perform acts relating
any person or persons dealing with him who violate the to the administration of the real property tax or to assist the
prohibitions provided in Section 89 of Book I hereof, shall be assessor or treasurer in such administration, who willfully fails to
punished with imprisonment for six months and one day to six discharge such duties shall, upon conviction be punished by a
years, or a fine of not less than Three thousand pesos fine of not less than Five hundred pesos (P500.00) nor more
(P3,000.00) nor more than Ten thousand pesos (P10,000.00), or than Five thousand pesos (P5,000.00) or imprisonment of not
both such imprisonment and fine at the discretion of the court. less than one (1) month nor more than six (6) months, or both
such fine and imprisonment, at the discretion of the court.
Sec515. Refusal or Failure of Any Party or Witness to
Appear before the Lupon or Pangkat. — Refusal or willful Sec518. Government Agents Delaying Assessment of Real
failure of any party or witness to appear before the lupon or Property and Assessment Appeals. — Any government
pangkat in compliance with a summons issued pursuant to the official who intentionally and deliberately delays the assessment
provisions on the Katarungang Pambarangay under Chapter 7, of real property or the filing of any appeal against its
Title III of this Code may be punished by the city or municipal assessment shall, upon conviction, be punished by a fine of not
court as for indirect contempt of court upon application filed less than Five hundred pesos (P500.00) nor more than Five
therewith by the lupon chairman, the pangkat chairman, or by thousand pesos (P5,000.00), or by imprisonment of not less
any of the contending parties. Such refusal or willful failure to than one (1) month nor more than six (6) months, or both such
appear shall be reflected in the records of the lupon secretary or fine and imprisonment, at the discretion of the court.
in the minutes of the pangkat secretary and shall bar the
complainant who fails to appear, from seeking judicial recourse Sec519. Failure to Dispose of Delinquent Real Property at
for the same cause of action, and the respondent who refuses to Public Auction. — The local treasurer concerned who fails to
appear, from filing any counterclaim arising out of, or necessarily dispose of delinquent real property at public auction in
connected with the complaint. compliance with the pertinent provisions of this Code, and any
A pangkat member who serves as such shall be entitled to an other local government official whose acts hinder the prompt
honorarium, the amount of which is to be determined by the disposition of delinquent real property at public auction shall,
sanggunian concerned subject to the provisions in this Code upon conviction, be subject to a fine of not less than One
cited above. thousand pesos (P1,000.00) nor more than Five thousand
pesos (P5,000.00), or imprisonment of not less than one (1)
Sec516. Penalties for Violation of Tax Ordinances. — The month nor more than six (6) months, or both such fine and
sanggunian of a local government unit is authorized to prescribe imprisonment, at the discretion of the court.
fines or other penalties for violation of tax ordinances but in no
case shall such fines be less than One thousand pesos Sec520. Prohibited Acts Related to the Award of Contracts
(P1,000.00) nor more than Five thousand pesos (P5,000.00), Under the Provisions on Credit Financing. — It shall be
nor shall imprisonment be less than one (1) month nor more unlawful for any public official or employee in the provincial, city,
than six (6) months. Such fine or other penalty, or both, shall be or municipal government, or their relatives within the fourth civil
imposed at the discretion of the court. The sangguniang degree of consanguinity or affinity, to enter into or have any
barangay may prescribe a fine of not less than One hundred pecuniary interest in any contract for the construction,
pesos (P100.00) nor more than One thousand pesos acquisition, operation or maintenance of any project awarded
(P1,000.00). pursuant to the provisions of Title Four in Book II hereof, or for
the procurement of any supplies, materials, or equipment of any
Sec517. Omission of Property from Assessment or Tax kind to be used in the said project. Any person convicted for
Rolls by Officers and Other Acts. — Any officer charged with violation of the provisions of said Title shall be removed from
the duty of assessing real property who willfully fails to assess, office and shall be punishable by imprisonment of not less than
or who intentionally omits from the assessment or tax roll any one (1) month, nor more than two (2) years, at the discretion of
real property which he knows to be taxable, or who willfully or the court, without prejudice to prosecution under other laws.
negligently under assesses any real property, or who
intentionally violates or fails to perform any duty imposed upon TITLE II: Provisions for Implementation
him by law relating to the assessment of taxable real property Sec521. Mandatory Review Every Five Years. — Congress
shall, upon conviction, be punished by a fine of not less than shall undertake a mandatory review of this Code at least once
One thousand pesos (P1,000.00) nor more than Five thousand every five (5) years and as often as it may deem necessary, with
pesos (P5,000.00), or by imprisonment of not less than one (1) the primary objective of providing a more responsive and
month nor more than six (6) months, or both such fine and accountable local government structure.
imprisonment, at the discretion of the court.

The Apocryphal Maggots:


Rainier, Chrisgel, Corina, Geoffry, Grace and Sylvie Blanche
The Flibbertigibbet Worms:
Golda, Gladys and Melyjane
CA VE AT: By simply r ea ding this re vi ew er a t the end o f the sem este r wi ll (hopef ully) guar antee you r pas sing this cou rse. D rink mode rat ely .
Amusin S C A r y
gly
THE C2005 LOCAL GOVERNMENT REVIEWER - 311 -
Sec522. Insurance Coverage. — The Government Service autonomous regions until such time as the regional government
Insurance System (GSIS) shall establish and administer an concerned shall have enacted its own local government code.
appropriate system under which the punong barangay, the
members of the sangguniang barangay, the barangay secretary, Sec527. Prior Approval or Clearance on Regular and
the barangay treasurer, and the members of the barangay tanod Recurring Transactions. — Six (6) months after effectivity of
shall enjoy insurance coverage as provided in this Code and this Code, prior approval of or clearance from national agencies
other pertinent laws. For this purpose, the GSIS is hereby or offices shall no longer be required for regular and recurring
directed to undertake an actuarial study, issue rules and transactions and activities of local government units.
regulations, determine the premiums payable, and recommend
to Congress the amount of appropriations needed to support the Sec528. Deconcentration of Requisite Authority and
system. The amount needed for the implementation of the said Power. — The national government shall, six (6) months after
insurance shall be included in the annual General the effectivity of this Code, effect the deconcentration of
Appropriations Act. requisite authority and power to the appropriate regional offices
or field offices of national agencies or offices whose major
Sec523. Personnel Retirement and/or Benefits. — An official functions are not devolved to local government units.
or employee of the national government or local government
unit separated from the service as a result of reorganization Sec529. Tax Ordinances or Revenue Measures. — All
effected under this Code shall, if entitled under the laws then in existing tax ordinances or revenue measures of local
force, receive the retirement and other benefits accruing government units shall continue to be in force and effect after
thereunder: Provided, however, That such benefits shall be the effectivity of this Code unless amended by the sanggunian
given funding priority by the Department of Budget and concerned, or inconsistent with, or in violation of, the provisions
Management in the case of national officials and employees, of this Code.
and the local government unit concerned in the case of local
officials and employees. Sec530. Local Water Districts. — All powers, functions, and
Where the employee concerned is not eligible for retirement, he attributes granted by Presidential Decree Numbered One
shall be entitled to a gratuity from the national government or hundred ninety-eight (P.D. No. 198), otherwise known as "The
the local government unit concerned, as the case may be, Provincial Water Utility Act of 1973," to the Local Water Utilities
equivalent to an amount not lower than one (1) month salary for Administration (LWUA) may be devolved in toto to the existing
every year of service over and above the monetary value of the local water districts should they opt or choose to exercise, in
leave credits said employee is entitled to receive pursuant to writing, such powers, functions and attributes: Provided, That all
existing laws. obligations of the local government unit concerned to the LWUA
shall first be settled prior to said devolution.
Sec524. Inventory of Infrastructure and Other Community
Facilities. — Sec531. Debt Relief for Local Government Units. —
(a) Each local government unit shall conduct a periodic (a) Unremitted national collections and statutory
inventory of infrastructure and other community facilities and contributions. — All debts owed by local government units to the
undertake the maintenance, repair, improvement, or national government in unremitted contributions to the
reconstruction of these facilities through a closer cooperation Integrated National Police Fund, the Special Education Fund,
among the various agencies of the national government and other statutory contributions as well as in unremitted
operating within the province, city, or municipality concerned. national government shares of taxes, charges, and fees
(b) No infrastructure or community project within the collected by the local government units, are hereby written off in
territorial jurisdiction of any local government unit shall be full.
undertaken without informing the local chief executive and the (b) Program loans. —
sanggunian concerned. (1) Program loans secured by local government units which
were relent to private persons, natural or juridical, shall
Sec525. Records and Properties. — All records, equipment, likewise be written off from the books of the local
buildings, facilities, and other properties of any office or body of government units concerned: Provided, however, That
a local government unit abolished or reorganized under this the national government agency tasked with the
Code shall be transferred to the office or body to which its implementation of these programs shall continue to
powers, functions, and responsibilities are substantially collect from the debtors belonging to the private sector
devolved. concerned.
(2) Program loans granted to local government units by
national government agencies and which were utilized
Transitory Provisions – LGC by the local units for community development, livelihood,
and other small-scale projects are hereby written off in
Sec526. Application of this Code to Local Government full.
Units in the Autonomous Regions. — This Code shall apply to (c) Settlement of debts due to government financing
all provinces, cities, municipalities and barangays in the institutions (GFIs), government-owned and controlled

The Apocryphal Maggots:


Rainier, Chrisgel, Corina, Geoffry, Grace and Sylvie Blanche
The Flibbertigibbet Worms:
Golda, Gladys and Melyjane
CA VE AT: By simply r ea ding this re vi ew er a t the end o f the sem este r wi ll (hopef ully) guar antee you r pas sing this cou rse. D rink mode rat ely .
Amusin S C A r y
gly
THE C2005 LOCAL GOVERNMENT REVIEWER - 312 -
corporations (GOCCs), and private utilities. The national local revenue collection shall be computed starting from the year
government shall assume all debts incurred or contracted by 1988.
local government units from GFIs, GOCCs, and private utilities (f) Appropriations. — Such amount as may be necessary
that are outstanding as of December 31, 1988, in accordance to implement the provisions of this Section shall be included in
with the following schemes: the annual General Appropriations Act.
(1) Debts due GFIs. — The national government may buy
outstanding obligations incurred by local government Sec532. Elections for the Sangguniang Kabataan. —
units from government financing institutions at a (a) The first elections for the sangguniang kabataan to be
discounted rate. conducted under this Code shall be held thirty (30) days after
(2) Debts due GOCCs. — The national government may the next local elections: Provided, That, the regular elections for
settle such obligations at discounted rate through the sangguniang kabataan shall be held one hundred twenty
offsetting, only to the extent of the obligations of local (120) days after the barangay elections thereafter.
governments against the outstanding advances made by (b) The amount pertaining to the ten percent (10%)
the National Treasury in behalf of the government-owned allocation for the kabataang barangay as provided for in Section
and controlled corporations concerned. 103 of Batas Pambansa Blg. 337 is hereby reappropriated for
(3) Debts Due Private Utilities. — The national government the purpose of funding the first elections mentioned above. The
may settle these obligations at a discounted rate by balance of said funds, if there by any after the said elections,
offsetting against the outstanding obligations of such shall be administered by the Presidential Council for Youth
private utilities to government-owned corporation. Affairs for the purpose of training the newly elected sangguniang
GOCCs may in turn offset these obligations against the kabataan officials in the discharge of their functions.
outstanding advances made by the National Treasury in (c) For the regular elections of the sangguniang
their behalf. kabataan, funds shall be taken from the ten percent (10%) of the
In the case of obligation owed by local government units barangay funds reserved for the sangguniang kabataan, as
to private utilities which are not indebted to any GOCC or provided for in Section 328 of this Code.
national government agency, the national government may (d) All seats reserved for the pederasyon ng mga
instead buy the obligations of the local government units from sangguniang kabataan in the different sanggunians shall be
the private utilities at a discounted rate, upon concurrence by deemed vacant until such time that the sangguniang kabataan
the private utilities concerned. chairmen shall have been elected and the respective
(d) Limitations. — Obligations to the Home Development pederasyon presidents have been selected: Provided, That,
and Mutual Fund (Pag-ibig), Medicare, and those pertaining to elections for the kabataang barangay conducted under Batas
premium contributions and amortization payments of salary and Pambansa Blg. 337 at any time between January 1, 1988 and
policy loans to the Government Service Insurance System are January 1, 1992 shall be considered as the first elections
excluded from the coverage of this Section. provided for in this Code. The term of office of the kabataang
(e) Recovery schemes for the national government. — barangay officials elected within the said period shall be
Local government units shall pay back the national government extended correspondingly to coincide with the term of office of
whatever amounts were advanced or offset by the national those elected under this Code.
government to settle their obligations to GFIs, GOCCs, and
private utilities. The national government shall not charge Sec533. Formulation of Implementing Rules and
interest or penalties on the outstanding balance owed by the Regulations. — (a) Within one (1) month after the approval of
local government units. this Code, the President shall convene the Oversight shall
These outstanding obligations shall be restructured and an formulate and issue the appropriate rules and regulations
amortization schedule prepared, based on the capability of the necessary for the efficient and effective implementation of any
local government unit to pay, taking into consideration the and all provisions of this Code, thereby ensuring compliance
amount owed to the national government. with the principles of local autonomy as defined under the
The national government is hereby authorized to deduct from Constitution.
the quarterly share of each local government unit in the internal (b) The Committee shall be composed of the following:
revenue collections an amount to be determined on the basis of (1) The Executive Secretary, who shall be the Chairman;
the amortization schedule of the local unit concerned: Provided, (2) Three (3) members of the Senate to be appointed by the
That such amount shall not exceed five percent (5%) of the President of the Senate, to include the Chairman of the
monthly internal revenue allotment of the local government unit Committee on Local Government;
concerned. (3) Three (3) members of the House of Representatives to
As incentive to debtor-local government units to increase the be appointed by the Speaker, to include the Chairman of
efficiency of their fiscal administration, the national government the Committee on Local Government;
shall write off the debt of the local government unit concerned at (4) The Cabinet, represented by the following:
the rate of five percent (5%) for every one percent (1%) increase (i) Secretary of the Interior and Local
in revenues generated by such local government unit over that Government;
of the preceding year. For this purpose, the annual increase in (ii)Secretary of Finance;
(iii) Secretary of Budget and Management; and

The Apocryphal Maggots:


Rainier, Chrisgel, Corina, Geoffry, Grace and Sylvie Blanche
The Flibbertigibbet Worms:
Golda, Gladys and Melyjane
CA VE AT: By simply r ea ding this re vi ew er a t the end o f the sem este r wi ll (hopef ully) guar antee you r pas sing this cou rse. D rink mode rat ely .
Amusin S C A r y
gly
THE C2005 LOCAL GOVERNMENT REVIEWER - 313 -
(5) One (1) representative from each of the following: The days of getting an instant "hard on" whenever you see a
(i) The League of Provinces; doorknob are probably over.
(ii)The League of Cities; As men get older, they require more direct stimulation, take
(iii) The League of Municipalities; and longer to get an erection, and often don't have as firm erections.
(iv) The Liga ng mga Barangay.
However, this lack of speed is not actually a shortcoming. Many
(c) The Committee shall submit its report and
recommendation to the President within two (2) months after its men find, for example, that they have much better orgasm
organization. If the President fails to act within thirty (30) days control as they get older.
from receipt thereof, the recommendation of the Oversight Most importantly, having an instant erection is not synonymous
Committee shall be deemed approved. Thereafter, the with good sex. You have a world of knowledge and experience
Committee shall supervise the transfer of such powers and now. Almost no 20 year old can match a 50 year old as a lover!
functions mandated under this Code to the local government
units, together with the corresponding personnel, properties,
assets and liabilities of the offices or agencies concerned, with
the least possible disruptions to existing programs and projects.
The Auto no mous R egi on in Musli m Mind an ao
The Committee shall likewise recommend the corresponding
appropriations necessary to effect the said transfer. 1987 Constitution – Article X
For this purpose, the services of a technical staff shall be
enlisted from among the qualified employees of Congress, the Sec1. The territorial and political subdivisions of the
government offices, and the leagues constituting the Committee. Republic of the Philippines are the provinces, cities,
(d) The funding requirements and the secretariat of the municipalities, and barangays. There shall be autonomous
Committee shall be provided by the Office of the Executive regions in Muslim Mindanao and the Cordilleras as hereinafter
Secretary. provided.
(e) The sum of Five million pesos (P5,000,000), which
shall be charged against the Contingent Fund, is hereby allotted Sec15. There shall be created autonomous regions in Muslim
to the Committee to fund the undertaking of an information Mindanao and in the Cordilleras consisting of provinces, cities,
campaign on this Code. The Committee shall formulate the municipalities, and geographical areas sharing common and
guidelines governing the conduct of said campaign, and shall distinctive historical and cultural heritage, economic and social
determine the national agencies or offices to be involved for this structures, and other relevant characteristics within the
purpose. framework of this Constitution and the national sovereignty as
well as territorial integrity of the Republic of the Philippines.

Application of the Code to Local Government Units in the Sec16. The President shall exercise general supervision over
Autonomous Regions – LGC autonomous regions to ensure that the laws are faithfully
executed.
Sec526. Application of this Code to Local Government
Sec18. The Congress shall enact an organic act for each
Units in the Autonomous Regions. — This Code shall apply to
autonomous region with the assistance and participation of the
all provinces, cities, municipalities and barangays in the
regional consultative commission composed of representatives
autonomous regions until such time as the regional government
appointed by the President from a list of nominees from
concerned shall have enacted its own local government code.
multisectoral bodies. The organic act shall define the basic
structure of government from the region consisting of the
executive department and legislative assembly, both of which
shall be elective and representative of the constituent political
Q&A units. The organic acts shall likewise provide for special courts
with personal, family, and property law jurisdiction consistent
Question with the provisions of this Constitution and national laws.
I am almost 54 and I am in a relationship with a woman again The creation of the autonomous region shall be effective when
after being alone for many years. I can still get erections, but not approved by majority of the votes cast by the constituent units in
like I once did. How can I regain my previous erections? a plebiscite called for the purpose, provided that only provinces,
cities, and geographic areas voting favorably in such plebiscite
Answer shall be included in the autonomous region.
Assuming you are in good health and there are no other
problems, there is really not much you can do, but that is okay. Sec20. Within its territorial jurisdiction and subject to the
provisions of this Constitution and national laws, the organic act
In our youth-obsessed culture, we are often led to believe that
of autonomous regions shall provide for legislative powers over:
being like a twenty year old is the best way to be. The reality is
(1) Administrative organization;
that all men will take longer to get an erection as they get older. (2) Creation of sources of revenues;

The Apocryphal Maggots:


Rainier, Chrisgel, Corina, Geoffry, Grace and Sylvie Blanche
The Flibbertigibbet Worms:
Golda, Gladys and Melyjane
CA VE AT: By simply r ea ding this re vi ew er a t the end o f the sem este r wi ll (hopef ully) guar antee you r pas sing this cou rse. D rink mode rat ely .
Amusin S C A r y
gly
THE C2005 LOCAL GOVERNMENT REVIEWER - 314 -
(3) Ancestral domain and natural resources; other purposes, in a plebiscite called for that purpose in
(4) Personal, family, and property relations; accordance with Section 18, Article X of the Constitution.
(5) Regional urban and rural planning development; The new area of autonomy shall then be determined by the
(6) Economic, social, and tourism development; provinces and cities that will vote/choose to join the said
(7) Educational policies; autonomy. It is understood that Congress may by law which
(8) Preservation and development of the cultural heritage; shall be consistent with the Constitution and in accordance with
and the provisions of Republic Act No. 7160, the Local Government
(9) Such other matters as may be authorized by law for the Code of 1991, provide that clusters of contiguous-Muslim-
promotion of the general welfare of the people of the dominated municipalities voting in favor of autonomy be merged
region. and constituted into a new province(s) which shall become part
of the new Autonomous Region.
Sec21. The preservation of peace and order within the (2) Plebiscite Coverage. The plebiscite shall be
regions shall be the responsibility of the local police agencies conducted in the provinces of Basilan, Cotabato, Davao del Sur,
which shall be organized, maintained, supervised, and utilized in Lanao del Norte, Lanao del Sur, Maguindanao, Palawan,
accordance with applicable laws. The defense and security of Sarangani, South Cotabato, Sultan Kudarat, Sulu, Tawi-Tawi,
the regions shall be the responsibility of the National Zamboanga del Norte, Zamboanga del Sur and the newly
Government. created Province of Zamboanga Sibugay, and (b) in the cities of
Cotabato, Dapitan, Dipolog, General Santos, Iligan, Kidapawan,
RA 6734 (1989) AN ACT PROVIDING FOR AN ORGANIC ACT Marawi, Pagadian, Puerto Princesa, Digos, Koronadal, Tacurong
FOR THE AUTONOMOUS REGION IN MUSLIM MINDANAO and Zamboanga.
(a) PLEBISCITE QUESTION FOR VOTERS OF THE FOUR
ORIGINAL PROVINCES OF THE AUTONOMOUS
RA 8746 (1999) AN ACT PROVIDING FOR THE DATE OF THE
REGION. For the voters of the provinces of
REGULAR ELECTIONS OF REGIONAL GOVERNOR,
Maguindanao, Lanao del Sur, Sulu and Tawi-Tawi which
REGIONAL VICE-GOVERNOR AND MEMBERS OF THE
are already members of the autonomous region under
REGIONAL LEGISLATIVE ASSEMBLY OF THE ARMM
the provisions of Republic Act No. 6734, the Organic Act
For 1996 – 9 SEPT 99
for the Autonomous Region in Muslim Mindanao, the
For 1999 – 2nd Monday of September
question to be asked in the plebiscite of the voters
therein shall be as follows: Do you vote in favor of the
RA 9012 (2000) AN ACT RESETTING THE REGULAR amendments to Republic Act No. 6734, the Organic Act
ELECTIONS FOR ELECTIVE OFFICIALS OF ARMM TO THE for the Autonomous Region in Muslim Mindanao, as
SECOND MONDAY OF SEPTEMBER 2001 AMENDING FOR proposed under this Organic Act, which includes, among
THE PURPOSE REPUBLIC ACT NO. 8953 other things, the expansion of the area of the
autonomous region?
(b) PLEBISCITE QUESTION FOR THE VOTERS OF THE
RA 9054 (2001) - An Act To Strengthen And Expand The PROVINCES AND CITIES PROPOSED FOR
Organic Act For The Autonomous Region In Muslim Mindanao, INCLUSION IN THE EXPANDED AUTONOMOUS
Amending For The Purpose Republic Act No. 6734, Entitled 'An REGION. For the voters of the provinces of Basilan,
Act Providing For The Autonomous Region In Muslim Cotabato, Davao del Sur, Lanao del Norte, Palawan,
Mindanao', As Amended Sarangani, South Cotabato, Sultan Kudarat, Zamboanga
del Norte, Zamboanga del Sur, and the newly created
ARTICLE I: Name and Purpose Province of Zamboanga Sibugay, and the cities of
Sec1. The name of the Autonomous Region shall be the Cotabato, Dapitan, Dipolog, Digos, Koronadal, Tacurong,
Autonomous Region in Muslim Mindanao unless provided General Santos, Iligan, Kidapawan, Marawi, Pagadian,
otherwise by the Regional Assembly. Puerto Princesa, and Zamboanga, which compose the
The Autonomous Region in Muslim Mindanao shall be governed provinces and cities that are proposed for inclusion in
by the Regional Government. the expanded area of the autonomous region, the
question to be asked in the plebiscite of the voters
ARTICLE II: The Autonomous Region therein shall be as follows: Do you vote in favor of the
Area and Seat of Government inclusion of your province or city in the Autonomous
Sec1. Expanded Autonomous Region. — Region in Muslim Mindanao?
(1) The Autonomous Region in Muslim Mindanao which,
under the provisions of Republic Act No. 6734, the Organic Act Sec2. Results of the Plebiscite. —
for the Autonomous Region in Muslim Mindanao, is composed (a) In the four provinces. If the majority of the voters of
of the four provinces of Lanao del Sur, Maguindanao, Sulu and the four provinces of Lanao del Sur Maguindanao, Sulu, and
Tawi-Tawi, is hereby expanded to include the provinces and Tawi-Tawi vote in favor of the above-mentioned proposed
cities, enumerated hereunder, which vote favorably to be amendments, the amendments are deemed ratified. Otherwise,
included in the expanded area of the autonomous region and for the amendments are deemed rejected except as regards the

The Apocryphal Maggots:


Rainier, Chrisgel, Corina, Geoffry, Grace and Sylvie Blanche
The Flibbertigibbet Worms:
Golda, Gladys and Melyjane
CA VE AT: By simply r ea ding this re vi ew er a t the end o f the sem este r wi ll (hopef ully) guar antee you r pas sing this cou rse. D rink mode rat ely .
Amusin S C A r y
gly
THE C2005 LOCAL GOVERNMENT REVIEWER - 315 -
inclusion of the provinces and cities that vote for their inclusion Sec9. Preferential Rights of Inhabitants and Their
in the autonomous region as provided in this Organic Act, in Safeguards.
which case, the said provinces and cities shall become Sec10. Protection Of Women and Children.
members of the autonomous region. Sec11. Enhancement of Quality of Life
(b) In the provinces or cities proposed for inclusion in the Sec12. Progressive Tax System. — The Regional Assembly
expanded area of the autonomous region. A majority of the shall adopt an efficient and progressive system of taxation
votes cast in the plebiscite in every province or city in favor of which, among other things, shall provide incentives for the
the inclusion of the province or city as members of the prompt payment of taxes and penalize tax evasion and
expanded area of the autonomous region as provided in this delinquency.
Organic Act shall effect their membership in the autonomous Sec13. Equitable Share In National Budget and
region. Development Assistance.
Sec14. Rights to Initiatives, Consultations, Referenda
Sec3. Seat of Autonomous Government. — The regional and Plebiscites.
legislative assembly, hereinafter referred to as the Regional Sec15. Fundamental Rights and Duties of People.
Assembly, shall by law, fix the permanent seat of government of Sec16. Human Rights Commission.
the regional government in any province or city that is a member Sec17. Environment Protection and Sustainable
of the autonomous region, taking into consideration accessibility Development.
and efficiency in which its mandate may be carried out under
this Organic Act. ARTICLE IV: Powers of Government
Until the seat of the regional government is transferred as Sec1. Powers and Functions. — Subject to the provisions
provided above, its provisional seat shall be in Cotabato City. of the Constitution, the Regional Government shall exercise
The Regional Assembly elected after the plebiscite mentioned in those powers and functions expressly granted to it in this
this Organic Act, shall, within its term, identify the site of the Organic Act, or necessary for or incidental to the proper
permanent seat of the regional government. The central governance and development of all the constituent units within
government which shall also mean the national government the autonomous region consistent with the policy on regional
shall appropriate funds for the transfer of the provisional seat to and local autonomy and decentralization.
its permanent site as determined by the Regional Assembly. The Regional Government may enact its own regional
administrative code and regional local government code
ARTICLE III: Guiding Principles and Policies consistent with the Constitution. The powers and functions
Sec1. Integral Part of the Republic. already vested upon and the shares of the national taxes
Sec2. Peaceful Settlement of Conflicts. provided by Republic Act No. 7160, the Local Government Code
Sec3. Devolution of Powers. — The regional government of 1991, to provinces, cities, municipalities, and barangay in the
shall adopt a policy on local autonomy whereby regional powers autonomous region shall not be reduced.
shall be devolved to local government units particularly in areas
of education, health, human resource, science and technology Sec2. Corporate Entity.
and people empowerment. Until a law implementing this Sec3. Scope of Regional Assembly Legislative Power
provision is enacted by the Regional Assembly, Republic Act No. Exceptions. — The Regional Assembly may exercise legislative
7160 the Local Government Code of 1991, shall continue to power in the autonomous region for the benefit of the people
apply to all the provinces, cities, municipalities, and barangay and for the development of the region except on the following
within the autonomous region. matters:
The Regional Assembly may not pass any law to diminish (a) Foreign affairs;
lessen, or reduce the powers, functions, and shares in the (b) National defense and security;
internal revenue taxes of the said local government units as (c) Postal service;
provided by Republic Act No. 7160, the Local Government Code (d) Coinage and fiscal and monetary policies;
of 1991. (e) Administration of justice; It may, however, legislate on
Sec4. Charters Govern Cities. — matters covered by the Shari'ah. The Shari'ah shall apply only to
Sec5. Customs, Traditions, Religious Freedom Muslims. Its application shall be limited by pertinent
Guaranteed. constitutional provisions, particularly by the prohibition against
Sec6. Filipino and Islamic Values In Educational cruel and unusual punishment and by pertinent national
Policies. — The regional government shall adopt educational legislation that promotes human rights and the universally
policies that shall perpetuate Filipino and Islamic values and accepted legal principles and precepts;
ideals and the just aspirations of the Bangsa Moro with due (f) Quarantine;
respect to the beliefs, customs, traditions, and religions of the (g) Customs and tariff;
other non-Muslim inhabitants of the region be they Christians, (h) Citizenship;
Jews, Buddhists, or of any other religious denomination. (i) Naturalization, immigration and deportation;
Sec7. Improving Status of the Marginalized. (j) General auditing;
Sec8. Regional Government Authority Over Natural (k) National elections;
Resources.

The Apocryphal Maggots:


Rainier, Chrisgel, Corina, Geoffry, Grace and Sylvie Blanche
The Flibbertigibbet Worms:
Golda, Gladys and Melyjane
CA VE AT: By simply r ea ding this re vi ew er a t the end o f the sem este r wi ll (hopef ully) guar antee you r pas sing this cou rse. D rink mode rat ely .
Amusin S C A r y
gly
THE C2005 LOCAL GOVERNMENT REVIEWER - 316 -
(l) Maritime, land and air transportation, and Sec7. Representatives In Government-Owned Or-
communications; The autonomous government shall, however, Controlled Corporations.
have the power to grant franchises, licenses and permits to
land, sea and air transportation plying routes in the provinces or ARTICLE VI: The Legislative Department
cities within the region, and communications facilities whose Sec1. Regional Assembly.
frequencies are confined to and whose main offices are located Sec2. Election of Regional Assembly. — The Regional
within the autonomous region; Assembly shall be composed of Members elected by popular
(m) Patents, trademarks, trade names, and copyrights; vote, with three (3) members elected from each of the legislative
and districts.
(n) Foreign trade. Sec3. Sectoral Representatives.
Sec4. Term of Office. —(3) years which shall begin, at noon
Sec4. General Welfare Powers. on the 30th day of September next following the day of the
Sec5. Representation In Central government or national election and shall end at noon of the same date three (3) years
government Departments, Offices. thereafter and No member of the Regional Assembly shall serve
Sec6. Eminent Domain.. more than three (3) consecutive terms. Voluntary renunciation of
or removal from office for any length of time shall not be
ARTICLE V: Inter-Governmental Relations considered as an interruption in the continuity of his service for
Sec1. General Supervision of the President Over the the full term for which he was elected.
Regional Governor. — Consistent with the Constitution and Sec5. Filling of Vacancy. — In case of vacancy in the
basic policy on local autonomy, the President of the Republic Regional Assembly occurring at least one (1) year before the
shall exercise general supervision over the Regional Governor expiration of the term of office, a special election shall be called
to ensure that his or her acts are within the scope of his or her to fill the vacancy in the manner prescribed by regional law. The
powers and functions. member elected shall serve only for the unexpired term.
The power of supervision of the President over the provincial Sec6. Qualifications of Members of Regional Assembly.
governors and the mayors of the highly urbanized cities shall be — No person shall be a member of the Regional Assembly
exercised through the Regional Governor; over the mayors of unless he or she is:
the component cities and municipalities, through the provincial (1) A natural-born citizen of the Philippines;
governor and over the punong barangay, through the city or (2) At least twenty-one (21) years of age on the day of
municipal mayor. the election;
In addition to other acts which he or she may impose under the (3) Able to read and write;
Constitution and this Organic Act, the President may suspend, (4) A registered voter of the district in which he or she
reduce, or cancel the financial blocks or grants-in-aid, funds for shall be elected on the day he or she files his or her certificate of
infrastructure, and other forms of assistance intended for the candidacy; and
autonomous region (1) if the regional government fails to (5) A resident thereof for a period of not less than five (5)
account for the funds and financial assistance released to it by years immediately preceding the day of the election.
the central government or national government, within one Sec7. Oath of Office.
month from the end of every quarter in which the funds and Sec8. Salaries.
financial assistance had been released or (2) when measures Sec9. Forfeiture of Seat.
for the protection and enhancement of the civil, human, political Sec10. Disclosure of Financial or Business Interests
or religious rights of the lumads, Christians and other minorities Sec11. Prohibited Acts for Members of the Regional
in the autonomous region ordained by the Constitution and this Assembly.
Organic Act, are not respected or are violated or are not Sec12. Parliamentary Immunity.
implemented within one (1) year from its enactment. Sec13. Rules of Procedure; Discipline of Members and
The President may suspend the Regional Governor for a period Civil Service Rules.
not exceeding six (6) months for willful violation of the Sec14. Questioning Cabinet Members and Other Officials.
Constitution, this Organic Act or any existing law that applies to Sec15. Regular and Special Sessions.
the autonomous region. Sec16. Passage of Bills.
Sec17. Approval of Bills and Overriding of Veto.
Sec2. Cabinet Membership. Sec18. Submittal of Bills to the President and Congress.
Sec3. Shari'ah and Tribal Courts; Coordination With Sec19. Creation, Division or Abolition of Provinces,
Central Government or National Government. Cities, Municipalities or Barangay. — The Regional Assembly
Sec4. Representation of Autonomous Region in General in may create; divide, merge, abolish, or substantially alter
the Central Government or National Government. — boundaries of provinces, cities, municipalities, or barangay in
Sec5. Representatives In Executive Departments and accordance with the criteria laid down by Republic Act No. 7160,
Constitutional Bodies. — the Local Government Code of 1991, subject to the approval by
Sec6. Ex Officio Member of the National Security a majority of the votes cast in a plebiscite in the political units
Council. directly affected. The Regional Assembly may prescribe
standards lower than those mandated by Republic Act No. 7160,

The Apocryphal Maggots:


Rainier, Chrisgel, Corina, Geoffry, Grace and Sylvie Blanche
The Flibbertigibbet Worms:
Golda, Gladys and Melyjane
CA VE AT: By simply r ea ding this re vi ew er a t the end o f the sem este r wi ll (hopef ully) guar antee you r pas sing this cou rse. D rink mode rat ely .
Amusin S C A r y
gly
THE C2005 LOCAL GOVERNMENT REVIEWER - 317 -
the Local Government Code of 1991, in the creation, division, comprise the executive council of the autonomous government.
merger, abolition, or alteration of the boundaries of provinces, Sec7. Terms of Office of Elective Regional Officials. —
cities, municipalities, or barangay. Provinces, cities, (3) years, which shall begin at noon on the 30th day of
municipalities, or barangay created, divided, merged, or whose September next following the day of the election and shall end
boundaries are altered without observing the standards at noon of the same date three (3) years thereafter. The
prescribed by Republic Act No. 7160, the Local Government incumbent elective officials of the autonomous region shall
Code of 1991, shall not be entitled to any share of the taxes that continue in effect until their successors are elected and
are allotted to the local governments units under the provisions qualified. Term limits – not more than three (3) consecutive
of the Code. terms.
The financial requirements of the provinces, cities, Sec8. Oaths or Affirmation of Office of the Regional
municipalities, or barangay so created, divided, or merged shall Governor and Regional Vice Governor.
be provided by the Regional Assembly out of the general funds Sec9. Compensation of Regional Governor and Vice
of the Regional Government. Governor.
The holding of a plebiscite to determine the will of the majority of Sec10. Regional Governor Housing and Travel Allowance.
the voters of the areas affected by the creation, division, merger, Sec11. Succession to Regional Governorship in Cases of
or whose boundaries are being altered as required by Republic Permanent Vacancy.
Act No. 7160, the Local Government Code of 1991, shall, Sec12. Succession to Regional Governorship in Cases of
however, be observed. Temporary Incapacity.
The Regional Assembly may also change the names of local Sec13. Removal of Regional Governor or Regional Vice
government units, public places and institutions, and declare Governor.
regional holidays. Sec14. Recall of Regional Governor, Regional Vice
Sec20. Annual Budget and Infrastructure Funds. Governor or Members of Regional Assembly.
Sec15. Punishment for Disorderly Behavior.
ARTICLE VII: The Executive Department Sec16. Prohibition Against Holding of Other Offices or
Sec1. Executive Power. — The executive power shall be Conflict of Interests.
vested in a Regional Governor. He shall be elected by the Sec17. Appointments by Acting Regional Governor. —
qualified voters of the autonomous region. effective, unless revoked by the elected Regional Governor
Sec2. Regional Governor and Cabinet Members. — The within ninety (90) days from his or her assumption of office.
Regional Governor shall appoint the members of the cabinet not Sec18. The Regional Governor shall not issue appointments
exceeding ten (10) members, at least six (6) of whom shall remove personnel, or, unless authorized by the Commission on
come from indigenous cultural communities. Elections, undertake public works projects within the prohibited
Sec3. Qualifications of Regional Governor and Regional period before and after a regional election as provided by law.
Vice Governor. — No person may be elected Regional Sec19. Appointments by Regional Governor. — The
Governor or Regional Vice Governor of the autonomous region Regional Governor shall appoint, in addition to the members of
unless he or she is a natural-born citizen of the Philippines, a the cabinet and their deputies, the chairmen and members of
registered voter of the autonomous region, able to read and the commissions and the heads of bureaus of the Regional
write, at least, thirty-five (35) years of age on the day of the Government, and those whom he may be authorized by this
election, and a resident of the autonomous region for, at least, Organic Act, or by regional law to appoint. The Regional
one (1) year immediately preceding the election. Assembly may, by law, vest the appointment of other officers or
Sec4. Election of Regional Governor and Regional Vice officials lower in rank on the heads of departments, agencies,
Governor. — The Regional Governor and the Regional Vice commissions, or boards.
Governor shall be elected as a team by the qualified voters of The powers, functions, responsibilities, and structure of the
the autonomous region. A vote for a candidate for Regional departments, agencies, bureaus, offices, and instrumentalities of
Governor shall be counted as a vote for his teammate for the Regional Government including the corporations owned-or-
Regional Vice Governor. A vote for a Regional Vice Governor controlled by the Regional Government shall be prescribed and
shall be counted as a vote for his teammate for Regional defined by the Regional Assembly.
Governor. For purposes of their election, the candidates for Sec20. Power of Regional Governor Over Commissions,
Regional Governor and Regional Vice Governor shall belong to Agencies, Boards, Bureaus and Offices. —the Regional
the same political party or coalition of parties. The Commission Governor shall have control of all the regional executive
on Elections shall promulgate the necessary rule or rules to give commissions, agencies, boards, bureaus, and offices. He shall
effect to this provision of law. ensure that laws are faithfully executed.
Sec5. Regional Vice Governor Appointment to or Sec21. Fiscal Year; Submission of Budget.
Removal from the Cabinet. Sec22. Budget Approval; Automatic Reenactment.
Sec6. Executive Council; Deputy Regional Governors. — Sec23. Veto Power of the Regional Governor; Votes to
The Regional Governor shall appoint three (3) deputies each Override. — The Regional Governor shall have the power to
representing the Christians, indigenous cultural communities, veto any particular item or items in an appropriation or revenue
and the Muslims in the region. The Regional Governor, the bill, but the veto shall not affect the item or items to which he
Regional Vice Governor, and the three (3) deputies shall

The Apocryphal Maggots:


Rainier, Chrisgel, Corina, Geoffry, Grace and Sylvie Blanche
The Flibbertigibbet Worms:
Golda, Gladys and Melyjane
CA VE AT: By simply r ea ding this re vi ew er a t the end o f the sem este r wi ll (hopef ully) guar antee you r pas sing this cou rse. D rink mode rat ely .
Amusin S C A r y
gly
THE C2005 LOCAL GOVERNMENT REVIEWER - 318 -
does not object. The Regional Assembly may override the veto Sec12. Shari'ah Appellate Court Justices Tenure of Office.
by a two-thirds(2/3) vote of all its members. —until they reach the age of seventy (70) years, unless sooner
removed for cause in the same manner as justices of the Court
Sec24. of Appeals or become incapacitated to discharge the duties of
(a) Law to Authorize Use of Money. — No money shall be their office.
paid out of the regional treasury except in pursuance of an Sec13. Shari'ah Appellate Court Justices Compensation.
appropriation made by regional law. —same compensation and enjoy the same privileges as the
(b) Prohibitions Against Sectarian Purposes; Presiding Justice and Associate Justices of the Court of
(c) Special Fund. All money collected on any regional tax Appeals, respectively.
levied for a special purpose shall be treated as a special fund Sec14. Shari'ah Appellate Court Administrator and Clerk of
and paid out for such special purpose only. If the purpose for Court.
which special fund was created has been fulfilled or abandoned, Sec15. Prohibition Against Holding of Other Offices.
the balance, if any, shall accrue to the general funds of the Sec16. Shari'ah Appellate Court Official Seat.
Regional Government. Sec17. Shari'ah Appellate Court Proceedings.
(d) Trust Funds. Trust funds shall only be paid out of the Sec18. Shari'ah Courts.
regional treasury for the specific purpose for which said funds Sec19. Tribal Courts.
were created or received. Sec20. Jurisconsult in Islamic Law.
(e) Authorization by Regional Governor or Sec21. Customary Law.
Representative. No funds or resources shall be disbursed Sec22. Application and Interpretation of Laws. — The
unless duly approved by the Regional Governor or by his duly provisions of the Muslim code and the tribal code shall be
authorized representative. applicable only to Muslims and other members of indigenous
cultural communities respectively and nothing herein shall be
ARTICLE VIII: Administration Of Justice construed to operate to the prejudice of the non-Muslims and
Sec1. Exercise of Judicial Power. — The judicial powers non-members of indigenous cultural communities.
shall be vested in the Supreme Court and in such lower courts In case of conflict between the Muslim code and the tribal code
as may be established by law including the Shari'ah Courts in the national law shall apply.
accordance with Section 5 hereof. In case of conflict between the Muslim code or the tribal code on
Sec2. Justices from Autonomous Region. — It shall be the one hand, and the national law on the other, the latter shall
the policy of the central government or national government prevail.
that, whenever feasible, at least one (1) justice in the Supreme Except in cases of successional rights to property, the regular
Court and two (2) justices in the Court of Appeals shall come courts shall acquire jurisdiction over controversies involving real
from qualified jurists of the autonomous region. property located outside the area of autonomy. Muslims who sue
Sec3. Consultant to the Judicial and Bar Council. other Muslims or members of indigenous cultural communities
Sec4. Deputy Court Administrator who sue other members of indigenous cultural communities
Sec5. Shari'ah Courts. over matters covered respectively by Shari'ah or by tribal laws
Sec6. Shari'ah Public Assistance Office. may agree to litigate their grievances before the proper Shari'ah
Sec7. Shari'ah Appellate Court. or tribal court in the autonomous region. The procedure for this
Sec8. Shari'ah Appellate Court Composition. recourse to the Shari'ah or tribal court shall be prescribed by the
Sec9. Jurisdiction of the Shari'ah Appellate Court. — Supreme Court.
The Shari'ah Appellate Court shall: Sec23. Bases for Interpretation of Islamic Law.
(a) Exercise original jurisdiction over petitions for Sec24. Shari'ah Powers and Functions.
certiorari, prohibition, mandamus, habeas corpus and other
auxiliary writs and processes only in aid of its appellate ARTICLE IX: Fiscal Autonomy
jurisdiction; and, Sec1. Revenue Source. — The Regional Government shall
(b) Exercise exclusive appellate jurisdiction over all cases have the power to create its own sources of revenues and to
tried in the Shari'ah district courts as established by law. levy taxes, fees, and charges, subject to the provisions of the
Sec10. Shari'ah Appellate Court Decisions. Constitution and this Organic Act.
Sec11. Shari'ah Appellate Court Justices Qualifications Sec2. Fiscal Autonomy.
and Appointments. — The Justices of the Shari'ah Appellate Sec3. Regional Tax Code.
Court shall possess the same qualifications as those of the Sec4. Regional Economic and Financial Programs.
Justices of the Court of Appeals and, in addition, shall also be Sec5. Uniform, Equitable Taxation; Prohibition Against
learned in Islamic law and jurisprudence.The members of the Confiscatory Taxes, Fees.
Shari'ah Appellate Court shall be appointed by the President Sec6. Payment of Taxes.
from a list of at least three (3) nominees prepared by the Judicial Sec7. Extent of Tax Powers; Exceptions. — Unless
and Bar Council. The nominees shall be chosen from a list of otherwise provided herein, the taxing power of the regional
recommendees submitted by the Regional Assembly. Such government and of the provinces, cities, municipalities, and
appointments need no confirmation. barangay located therein shall not extend to the following:

The Apocryphal Maggots:


Rainier, Chrisgel, Corina, Geoffry, Grace and Sylvie Blanche
The Flibbertigibbet Worms:
Golda, Gladys and Melyjane
CA VE AT: By simply r ea ding this re vi ew er a t the end o f the sem este r wi ll (hopef ully) guar antee you r pas sing this cou rse. D rink mode rat ely .
Amusin S C A r y
gly
THE C2005 LOCAL GOVERNMENT REVIEWER - 319 -
(a) Income tax, except when levied on banks and other (a) Thirty-five percent (35%) to the province or city;
financial institutions; (b) Thirty-five percent (35%) to the regional government;
(b) Documentary stamps tax; and
(c) Taxes on estate, inheritance, gifts, legacies, and other (c) Thirty percent (30%) to the central government or
acquisitions mortis causa except as otherwise provided by law; national government.
(d) Customs duties, registration fees of vessel and Sec10. Treasury Bills, Notes and Other Debt Papers.
wharfage on wharves, tonnage dues, and all other kinds of Sec11. Economic Agreements.
custom fees, charges, and dues except vessels which are Sec12. Donations or Grants; Tax Deductible.
registered by their owners with the Regional Government and Sec13. Regional Tax Exemptions. —
wharfage on wharves constructed and maintained by the Sec14. Foreign or Domestic Loans.
Regional Government or the local government unit concerned; Sec15. Collection and Sharing of Internal Revenue Taxes.
(e) Taxes, fees, or charges and other impositions upon —
goods carried into or out of, or passing through the territorial
jurisdiction of the provinces, cities, municipalities, or barangay of ARTICLE X: Ancestral Domain, Ancestral Lands and Agrarian
the autonomous region in the guise of charges for wharfage, Reform
tolls for bridges, or otherwise, or other taxes, fees or charges in Sec1. Ancestral Domain; Lands of Indigenous Cultural
any form whatsoever upon such goods or merchandise except Communities.
tolls on bridges or roads constructed and maintained by the Sec2. The constructive or traditional possession of lands
provinces, cities, municipalities, or barangay concerned or by and resources by an indigenous cultural community may also be
the Regional Government. recognized subject to judicial affirmation, the petition for which
(f) Taxes, fees, or charges on agricultural and aquatic shall be instituted within a period of ten (10) years from the
products when sold by marginal farmers or fisherfolk; effectivity of this Organic Act.
(g) Taxes on business enterprises certified by the Board Sec3. "indigenous cultural community" refers to Filipino
of Investments or by the Regional Assembly as pioneer or non- citizens residing in the autonomous region who are:
pioneer for a period of six (6) and four (4) years, respectively (a) Tribal peoples. These are citizens whose social,
from the date of registration; cultural and economic conditions distinguish them from other
(h) Excise taxes on articles enumerated under the sectors of the national community; and
national internal revenue code, and taxes, fees, or charges on (b) Bangsa Moro people. These are citizens who are
petroleum products; believers in Islam and who have retained some or all of their
(i) Percentage or value-added tax (VAT) on sales, own social, economic, cultural, and political institutions.
barters, or exchanges or similar transactions on goods or Sec4. Cultural Communities.
services except as otherwise provided by law; Sec5. Ecological Balance.
(j) Taxes on the gross receipts of transportation Sec6. Unless authorized by the Regional Assembly, lands of
contractors and persons engaged in the transportation of the ancestral domain titled to or owned by an indigenous cultural
passengers or freight by hire and common carriers by air, land, community shall not be disposed of to non-members.
or water except as provided in this Organic Act; Sec7. No portion of the ancestral domain shall be open to
(k) Taxes on premiums paid by way of reinsurance or resettlement by non-members of the indigenous cultural
retrocession; communities.
(l) Taxes, fees, or other charges on Philippine products Sec8. Regional Land Reform.
actually exported, except as otherwise provided by law enacted
by the Congress; ARTICLE XI: Urban and Rural Planning and Development
(m) Taxes, fees, or charges on countryside, barangay Sec1. Urban and Rural Development.
business enterprises and cooperatives duly registered under Sec2. Indigenous Development Plans.
Republic Act No. 6810, the "Magna Carta for Countryside and Sec3. Equitable Development.
Barangay Business Enterprises" and Republic Act No. 6938, the Sec4. Urban Land, Land and Water Use.
"Cooperatives Code of the Philippines", respectively, and
(n) Taxes, fees, or charges of any kind on the central ARTICLE XII: Economy and Patrimony
government or national government, its agencies and Sec1. Regional Economy and Conservation of
instrumentalities and local government units except on Patrimony. — Consistent with the Constitution and existing
government-owned or -controlled corporations or entities that laws, the
are primarily organized to do business. Sec2. Economic Zones, Centers and Ports.
Sec3. Incentives for Investors.
Sec8. Sources Of Regional Government Revenue Sec4. Regional Economic Zone Authority; Freeports
Sec9. Sharing of Internal Revenue, Natural Resources Sec5. Use Development of Mines Minerals and Other
Taxes, Fees and Charges. — The collections of a province or Natural Resources Revenue Sharing; Exceptions. —
city from national internal revenue taxes, fees and charges, and (a) Regional Supervision and Control.
taxes imposed on natural resources, shall be distributed as (b) Sharing Between Central Government or National
follows: Government and Regional Government in Strategic Minerals

The Apocryphal Maggots:


Rainier, Chrisgel, Corina, Geoffry, Grace and Sylvie Blanche
The Flibbertigibbet Worms:
Golda, Gladys and Melyjane
CA VE AT: By simply r ea ding this re vi ew er a t the end o f the sem este r wi ll (hopef ully) guar antee you r pas sing this cou rse. D rink mode rat ely .
Amusin S C A r y
gly
THE C2005 LOCAL GOVERNMENT REVIEWER - 320 -
Revenues, Taxes, or Fees. Fifty (50%) percent of the revenues, Sec31. Barter and Counter-Trade.
each Sec32. Consumer Education and Welfare.
(c) Sharing Between Regional Government and Local Sec33. Local Labor and Goods.
Government Units in Strategic Minerals Revenues, Taxes, or Sec34. Foreign Investments.
Fees. The share of the Regional Government mentioned above
is hereby apportioned as follows: thirty percent (30%) to the Tourism Development
Regional Government; twenty percent (20%) to all the Sec35. Regional Tourism.
municipalities; and fifteen percent (15%) to all the barangays. Sec36. Tourism Office.
(d) Regional Assembly Authority to Grant Franchises and
Concessions and Empower Regional Governor to Grant Leases, ARTICLE XIII: Public Order and Security
Permits, and Licenses.. Sec1. Law and Order.
(e) Consultation with Cultural Communities, Needed. Sec2. Regional Security Force. — There is hereby created
Sec6. Use and Development of Natural Resources Open a Philippine National Police Regional Command for the
to Citizens. autonomous region, hereafter called the Special Regional
Sec7. Preferential Rights of Citizen-Inhabitants of Security Force (SRSF) or Regional Police Force, in short.
Autonomous Region. Sec3. Law Governing Regional Police Force. — The
Sec8. Rules, Regulations and Fees. — The Regional Regional Assembly shall enact laws to govern the Regional
Assembly shall by law regulate the exploration, utilization, Police Force consistent with the pertinent provisions of the
development, and protection of the natural resources, including Constitution and this Organic Act. The members of the Moro
the mines and minerals, except the strategic minerals as National Liberation Front who are integrated into the Regional
provided in this Organic Act. Police Force may be deployed in the autonomous region or
Sec9. Regulation of Small-Scale Mining. — elsewhere in the Republic as may be determined by the proper
Sec10. Regional Economic and Development Planning police authorities.
Board; Composition and Functions. Sec4. Regional Police Force Composition. — The PNP
Sec11. Pioneering Public Utilities and Cooperatives. Regional Command for the autonomous region/SRSF shall be
Sec12. Proclamation of State of Calamity Operation of composed of the existing PNP units therein, the MNLF
Public Utilities. — elements, and other residents of the area who may later on be
(a) Temporary take-over of operations. recruited into the SRSF.
(b) Proclamation of state of calamity. Sec5. Powers and Functions of Regional Police Force.
or national government or the Regional Government. — The Regional Police Force shall exercise within the
autonomous region the following powers and functions:
Sec13. Legislating Benefits, Compensation for Victims of (a) Enforce laws enacted by the Congress and by the
Mining and Mining Operations; Rehabilitation of Affected Regional Assembly relative to the protection of lives and
Areas. properties of the people;
Sec14. Reforestation; Support for Lumads or Tribal (b) Maintain law and order and ensure public safety;
Peoples. (c) Investigate and prevent crimes, arrest criminal
Sec15. Prohibition Against Toxic or Hazardous offenders, bring criminal suspects to justice, and assist in their
Substances. prosecution;
Sec16. Business Ownership. (d) Effect other arrests, searches, and seizures in
Sec17. Incentives, Tax Rebates and Holidays. accordance with the Constitution and pertinent laws;
Sec18. Transport and Communication Facilities, Priority (e) Detain persons for a period not exceeding what is
Projects. prescribed by law, inform the person so detained of all his or her
Sec19. Power Services Priority. rights under the Constitution, and observe the human rights of
Sec20. Pioneering Firms. all people in the autonomous region;
(f) Process applications for the licensing of firearms for
Agriculture, Fisheries and Aquatic Resources approval by the proper official of the Philippine National Police;
Sec21. Farming and Fishing Cooperatives. (g) Initiate drives for the licensing or surrender of
Sec22. Agricultural Productivity; Organic Farming. unlicensed firearms; confiscate unlicensed firearms after such
Sec23. Soil and Water Conservation. drives are over; prosecute or recommend to the President the
Sec24. Aquatic and Fisheries Code. grant of amnesty or pardon to possessors of unlicensed firearms
Sec25. Agriculture and Fisheries Bureau. who surrender them; and
(h) Perform such other duties and exercise all other
Trade and Industry functions as may be provided by law enacted by Congress or by
Sec26. Private Sector. the Regional Assembly.
Sec27. Cottage Industries. Sec6. Observance of Constitution and Laws.
Sec28. Banks and Financial Institutions. Sec7. Regional Police Framework and Organization. —
Sec29. Islamic Banks. The philosophical framework and structural organization of the
Sec30. Bangko Sentral Regional Bank. Regional Police Force shall be as follows:

The Apocryphal Maggots:


Rainier, Chrisgel, Corina, Geoffry, Grace and Sylvie Blanche
The Flibbertigibbet Worms:
Golda, Gladys and Melyjane
CA VE AT: By simply r ea ding this re vi ew er a t the end o f the sem este r wi ll (hopef ully) guar antee you r pas sing this cou rse. D rink mode rat ely .
Amusin S C A r y
gly
THE C2005 LOCAL GOVERNMENT REVIEWER - 321 -
(a) It shall be civilian in nature and character; (15) days after the occurrence of the events mentioned above
(b) The scope of its operations shall be regional. that need to be suppressed, prevented, or suppressed.
(c) It shall be headed by a regional director who shall be Sec13. Indigenous Structures.
assisted by two deputies, one for administration and one for
operations. ARTICLE XIV: Education, Science and Technology, Arts, and
(d) It shall have regional, provincial, and city or municipal Sports
offices; Sec1. Quality Education, A Top Priority. Educational
(e) At the provincial level, there shall be a provincial office Policies
headed by a provincial director who shall be a professional Sec2. Policies and Principles. — Consistent with the basic
police officer with the rank of police superintendent, at least; and state policy on education, the Regional Government shall adopt
(f) At the city or municipal level, there shall be an office the following educational policies and principles:
or station, which shall be headed by a Chief of Police who shall (a) Perpetuation of Filipino and Islamic values.
be a professional police officer with the rank of police (b) Inculcation of values of peaceful settlement of
superintendent for the city, and police inspector for the disputes.
municipality. (c) Optional religious instruction.
Sec8. Power of Regional Governor Over Regional Police (d) Inculcation of patriotism and nationalism.
Force. — The Regional Governor shall have the following (e) Vocational and special education.
powers over the Regional Police Force: (f) Scholarships, student loans and scholarships.
(a) To act as the deputy of the National Police (g) Funding for local education programs.
Commission in the region and as the ex-officio chair of the (h) Community participation in education.
Regional Police Commission; (i) Development of regional language.
(b) To exercise operational control and general (j) Media of instruction in schools. Filipino and English
supervision and disciplinary powers over the Regional Police (k) Teaching of Arabic as subject.
Force; (l) Auxiliary official languages
(c) To employ or deploy the elements of and assign or (m) Develop ethnic identity.
reassign the Regional Police Force through the regional director. (n) The Regional Government shall recognize the
The Regional Director may not countermand the order of the participation of private institutions of learning, including the
Regional Governor unless it is in violation of the Constitution Madaris (Arabic schools), in providing quality education to the
and the law; people of the region; and
(d) To recommend to the President the appointment of (o) (1) Basic education structure.
the regional director and his two deputies; Sec3. Regional Educational Curricula.
(e) To oversee the preparation and implementation of the Sec4. Education, Management, and Control of
Integrated Regional Public Safety Plan; Education. — The management, control, and supervision of the
(f) To impose, after due notice and summary hearings of regional educational subsystem shall be the primary concern of
the citizen's complaints, administrative penalties on personnel of the Regional Government.
the Regional Police Force except those who are appointed by The Department of Education, Culture and Sports, the
the President; and Commission on Higher Education, and the Technical Education
(g) Do everything necessary to promote widespread and Skills Development Authority, and other appropriate
support by the various communities making up the autonomous educational bodies of the central government or national
region for the Regional Police Force. government shall monitor compliance by the regional
Sec9. Regional Police Directors. educational subsystem with national educational policies,
Sec10. Regional Police Commission standards, and regulations.
Sec11. Regional Defense and Security.
Sec12. Calling Upon the Armed Forces. — The provisions Educational Structure
of the preceding sections notwithstanding, the Regional Sec5. The Regional Assembly may, by law, create, support
Governor may request the President to call upon the Armed and maintain a regional Department of Education, Culture and
Forces of the Philippines: Sports, and shall define its powers, functions and composition.
(1) To prevent or suppress lawless violence, invasion, or
rebellion, when the public safety so requires, in the autonomous Sec6. Private Schools Supervision. —
region in accordance with the provisions of the Constitution; Sec7. Educational Subsystem Structure.
(2) To suppress the danger to or breach of peace in the Sec8. Academic Freedom and Fiscal Autonomy.
autonomous region, when the Regional Police Force is not able Sec9. Education Centers.
to do so; or, Sec10. Tribal University System.
(3) To avert any imminent danger to public order and
security in the area of autonomy. Madrasah Education
The President may on his own accord send the Armed Forces of Sec11. Supervision of Madaris Schools.
the Philippines into the autonomous region to attain the above Sec12. Madrasah Educational System; Arabic as Medium
objectives of the Regional Governor does not act within fifteen of Instruction.

The Apocryphal Maggots:


Rainier, Chrisgel, Corina, Geoffry, Grace and Sylvie Blanche
The Flibbertigibbet Worms:
Golda, Gladys and Melyjane
CA VE AT: By simply r ea ding this re vi ew er a t the end o f the sem este r wi ll (hopef ully) guar antee you r pas sing this cou rse. D rink mode rat ely .
Amusin S C A r y
gly
THE C2005 LOCAL GOVERNMENT REVIEWER - 322 -
Sec13. Madaris. (1) Child health and development, including the support of
Sec14. Madaris Teachers Qualifying Examinations; the physically challenged and other disadvantaged
Compensations. persons in need of welfare services;
(2) Protection and development of the rights of women and
Science and Technology of indigenous population;
Sec15. Science and Technology. (3) Registration of births, marriages and deaths; and
Sec16. Priority Legislation. — (4) Fixing of regional public holidays.
(a) Give priority to science, research, inventions,
technology, education, and their development and utilization; Sec3. Housing Program
(b) Provide incentives, including tax deduction and Sec4. Re-integration programs responsive to the needs of
funding assistance, and encourage the participation of the former rebels who return to the fold of the law.
private sector in basic and applied scientific researches; Sec5. Family as Nation's Foundation.
(c) Regulate the transfer and promote the adoption of Sec6. Women's Rights.
technology from all sources for regional benefits; Sec7. Youth.
(d) Secure and protect the exclusive rights of scientists, Sec8. People's Organization.
inventors, scholars, writers, artists, and other gifted citizens to Sec9. Protection Of Labor.
their intellectual properties; and
(e) Ensure the full and effective participation of all sectors ARTICLE XVI: General Provisions
in the planning, programming, coordination, and implementation Sec1. The Regional Assembly is hereby empowered to pass
of scientific and technological researches and the acquisition, a law adopting an official regional emblem, seal, and hymn.
adoption, innovation, and application of science and technology Sec2. Disciplinary Authority Over Officials and
for development. Employees by — The Regional Government shall have primary
Sec17. Environmental Changes. disciplinary authority over officials and employees of the
Sec18. Non-Formal Education. Scholarship Grants and Regional Government.
Assistance Sec3. Oath of Office.
Sec19. Donations to Universities, Colleges and Schools. Sec4. Civil Service Eligibility. —
Sec20. Scholarship Programs. Sec5. Prohibition Against Employment of Military
Sec21. Financial Assistanc322òe for Disadntaged, Personnel in Civil Service.
Deserving Students. Sec6. Promulgation and Translation of Organic Act.

Funds for Education ARTICLE XVII: Amendments or Provisions


Sec22. Release of Education Funds. Physical Education Sec1. Organic Act may be reamended or revised by the
and Sports Development Congress of the Philippines upon a vote of two-thirds (2/3) of the
Sec23. Physical Education and Sports Development. Members of the House of Representatives and of the Senate.
Sec24. Sports Programs.
ARTICLE XVIII: Transitory Provisions
Cultural Heritage Sec1. Disposition of Certain Real Properties of the
Sec25. Cultural Heritage. Autonomous Region
Sec26. Protection and Promotion of Culture. Sec2. Personnel Absorbed by Regional Government.
Sec27. Bureau of Cultural Heritage. Sec3. Oversight Committee.
Sec4. Agencies and Offices Transferred to the
ARTICLE XV: Social Justice, Services, Institutions, and Other Autonomous Region.
Concerns Sec5. Notwithstanding the provisions of the preceding
Sec1. Promotion of Social Justice. section, the GSIS, SSS, the Pagtutulungan-Ikaw, Bangko,
Sec2. Industriya't Gobyerno (PAG-IBIG), and other funds of similar
(a) Social Services. The Regional Assembly shall, trust or fiduciary nature shall be exempt from the coverage of
consistent with the provisions of the Constitutions and existing this Organic Act.
national laws, enact measures to provide and promote social Sec6. Budgetary Law, Rules, and Regulations.
services. Sec7. First Regular Elections.
(b) Food and Drug Regulation. The Regional Assembly Sec8. The incumbent Regional Governor, Regional Vice
shall, by law, establish and maintain an effective food and drug Governor, and members of the Regional Legislative Assembly of
regulatory system. The rational use of drugs through an the Autonomous Region In Muslim Mindanao shall continue in
essential drugs list and the use of generic medicines or drugs, office pursuant to existing laws and until their successors shall
as well as the use of herbal medicines and indigenous health have been duly elected and qualified.
resources, whenever appropriate, shall be encouraged and Sec9. Within one (1) year from its organization, the Regional
promoted. Assembly shall, by law, create a code commission on Muslim and
(c) Other Legislation. The Regional Assembly shall also a code commission on tribal laws
enact legislation on the following: Sec10. Initial Funds for the Regional Government.

The Apocryphal Maggots:


Rainier, Chrisgel, Corina, Geoffry, Grace and Sylvie Blanche
The Flibbertigibbet Worms:
Golda, Gladys and Melyjane
CA VE AT: By simply r ea ding this re vi ew er a t the end o f the sem este r wi ll (hopef ully) guar antee you r pas sing this cou rse. D rink mode rat ely .
Amusin S C A r y
gly
THE C2005 LOCAL GOVERNMENT REVIEWER - 323 -
Sec11. Annual Assistance. Limbona vs. Mangelin (1989)
Sec12. Sectoral Representatives.
Sec13. Plebiscite and Effectivity of this Organic Act. Facts:
Sec14. Plebiscite Information Campaign. Limbona was expelled from membership in the Sangguniang
Sec15. Promulgation of Rules; Appropriations for Pampook, Regional Autonomous Government, Region XII,
Simultaneous Plebiscites. representing Lanao del Sur on the ground that he authorized the
Sec16. Zone of Peace and Development. — payment of salaries and emoluments to a certain Abdula without
Sec18. Repealing Clause. — All laws, decrees, orders, rules authority from the Assembly. Limbona brought petition at SC.
and regulations, and other issuances or parts thereof, which are
inconsistent with this Organic Act, are hereby repealed or Issue: ARE THE AUTONOMOUS GOVERNMENT OF MINDANAO
modified accordingly. SUBJECT TO THE JURISDICTION OF THE NATIONAL
Sec19. Effectivity Clause. — This Organic Act shall take COURTS? IN OTHER WORDS, WHAT IS THE EXTENT OF
effect after fifteen (15) days following its complete publication in SELF-GOVERNMENT GIVEN TO THEM
at least two national newspapers of general circulation and one Held:
local newspaper of general circulation in the autonomous Yes. LGU’s enjoy autonomy in these two senses: decentralization
region. of administration and decentralization of power.
Approved, March 31, 2001
(1) in decentralization of administration – an autonomous
NOTES government is under the supervision of the national
• Autonomous Region (AR) by itself is an LGU. But unlike government acting through the president (and the Dept. of
Local Government)
other LGU’s, AR has separate legal system. It has political
as well as administrative autonomy. It can have its own  If the Sangguniang Pampook is autonomous in this
LGC provided that it cannot decrease the power of local sense, it comes unarguably under the Court’s
government officials and the IRA under the present LGC. jurisdiction.
The legislative powers of AR is limited by the Constitution (2) in decentralization of power – an autonomous government
and the Organic Act. is subject alone to the decree of the organic act creating it
• 2 provinces can form AR. and accepted principles on the effects and limits of
• AR require plebiscite, therefor it is not automatic. autonomy.
• Sir: What is the point of being an AR if the President can  If the Sangguniang Pampook is autonomous in this
pry into internal affairs of AR? sense, its acts are beond the domain of the court in
the same way that internal acts, say, of the Congress
are beyond its jurisdiction.
Abbas vs. COMELEC (1989) supra
• An examination of PD No. 1618 creating the autonomous
Pursuant to Consti mandate, Congress enacted RA 6734 providing governments of Mindanao shows that they were never meant
for an organic act for ARMM. To implement the act, plebescites in to exercise autonomy in the second sense, that is, in which
13 provinces and 9 cities in Mindanao and Palawan were the central government commits an act of self-immolation.
conducted. 1. The P.D. mandates that the president shall have the
power of general supervision and control over
Under RA and Consti, the requirements for creation of autonomous Autonomous Regions
region are 2. The Sangguniang Pampook, their legislative arm, is
1. creation shall take effect when approved by a majority of made to discharge chiefly administrative services.
the votes cast by the constituent units in a plebiscite.
2. Only provinces and cities where a majority vote in favor
of the organic act shall be included in the autonomous Pandi vs. CA (2002)
region. The appointment and detailing of positions depend on which law is
3. The single plebiscite should be determinative of applicable at the time such appointment and detailing was made.
• WON there shall be an autonomous region in
Mindanao
• Which cities and provinces shall comprise it.
1. First period – Prior to the Organic Act of 1989
Governing law: E.O. No. 119 (Charter of the Department of
Health). The Ministry of Health was the appointing power of
The majority required in consti for creation of region is a simple
provincial health officers who were in reality national
majority of votes approving the organic act in individual constituent
government officials paid entirely from national funds. The
units and not a double majority of votes in all constituent units put
appointment of a provincial health officer was to a specific
together as well as in the individual constituent units.
region and the Minister (later Secretary) of health could

The Apocryphal Maggots:


Rainier, Chrisgel, Corina, Geoffry, Grace and Sylvie Blanche
The Flibbertigibbet Worms:
Golda, Gladys and Melyjane
CA VE AT: By simply r ea ding this re vi ew er a t the end o f the sem este r wi ll (hopef ully) guar antee you r pas sing this cou rse. D rink mode rat ely .
Amusin S C A r y
gly
THE C2005 LOCAL GOVERNMENT REVIEWER - 324 -
assign him to any province within the region upon the 1. the provincial governor appoints the provincial
recommendation of the Regional Director.
health officer if the latter’s salary comes from
provincial funds.
2. Second Period – Time after the enactment of the
Organic Act of 1989 but before the adoption of the 2. If the provincial health officer’s salary comes mainly
1991 LGU Code -Under the Organic Act of 1989, the
from regional funds, then the ARMM Local Code
power of the Secretary of Health to appoint provincial
applies, in which case the Regional Governor is the
health officers to a region, and to assign them to any
appointing power but he must appoint only from
province within the region, was not immediately devolved
among the 3 nominees of the Provincial Governor
to the Regional Government.

3. Third Period – Time after the enactment of the 1991


3. Moreover, the Provincial Governor exercises
supervision and control over provincial health
LGU Code but before the adoption of the ARMM Local
officer because the ARMM Local Code has classified
Code Under the 1991 LGU Code, the provincial health
him a s a provincial government official.
officers were made one of the officials the provincial
government to be appointed by the provincial governor if
This is the present state of law, which is actually the same
his salary came mainly from provincial funds.
as the law in the 4th period. The only difference is that the
Regional Assembly cannot amend the ARMM Local Code
However, the 1991 LGU Code, although a later law like the
to reduce or diminish this power of the Provincial Governor
RAC, did not amend the Organic Act of 1989 because the
because this devolved power, emanating from the 1991
Organic Act could only be amended through the ratification
LGU Code, is now part of the Organic Act of 2001.
process laid out in the Organic Act itself.

Thus, even after the passage of the 1991 LGU code, the NOTEs:
Secretary of Health continued to be the appointing power of AR allowed to legislate on their own, except that it musrt be
provincial health officers who remained national government consistent with the consti and the organic act.
officials. The Secretary also continued to exercise the authority
to assign Steps to follow:

Executive Order No. 133 was issued finally transferring the a. Check where Local Gov official is set
powers and functions of the DOH in the Autonomous Region to b. if ARMM, check their LGC
the Regional Government. Upon the effectivity of this EO, the c. If there is a provision in their LGC, check if consistent w/
administrative authority of the Secretary of Health to assign Consti and Organic Act
provincial health officers to any province within a region was d. If no provision, assume that it is the same as RA 7160.
transferred to the ARMM Secretary of Health as the regional e. If silent, make the necessary appointment.
counterpart of the National Secretary of Health.

4. Fourth Period – Time after the adoption o fthe ARMM Cord ille ra Adm inist rat iv e R egi on
Local Code but before the enactment of the Organic
Act of 2001 EO 220 - Creating A Cordillera Administrative Region,
Under the ARMM Local Code, the provincial health officer Appropriating Funds Therefor And For Other Purposes
in the ARMM, previously a regional official, has also
become a provincial government official. The Regional WHEREAS, pursuant to Section 1, Article X of the 1987
Governor appoints the provincial health officer from a list of Constitution, there shall be created an autonomous region in the
3 recommendees of the Provincial Governor. The ARMM Cordilleras; aisa dc
local code provides that the salary of the provincial health WHEREAS, Section 15, Article X of the Constitution provides that
officer shall be paid from regional funds. the autonomous region in the Cordilleras shall consist of
provinces, cities, municipalities and geographical areas sharing
5. Fifth Period – Time after the enactment of the Organic common and distinctive historical and cultural heritage,
Act of 2001 economic and social structures, and other relevant
Under the Organic Act of 2001, the powers and functions of characteristics within the framework of the Constitution and the
a Provincial Governor under the 1991 LGU Code are now national sovereignty as well as territorial integrity of the Republic
enjoyed, as a minimum by the Provincial Governor in the of the Philippines;
ARMM. WHEREAS, pursuant to Section 6, Article XVIII of the Constitution,
Thus, the President has the power to continue to exercise legislative
powers until the first Congress is convened;

The Apocryphal Maggots:


Rainier, Chrisgel, Corina, Geoffry, Grace and Sylvie Blanche
The Flibbertigibbet Worms:
Golda, Gladys and Melyjane
CA VE AT: By simply r ea ding this re vi ew er a t the end o f the sem este r wi ll (hopef ully) guar antee you r pas sing this cou rse. D rink mode rat ely .
Amusin S C A r y
gly
THE C2005 LOCAL GOVERNMENT REVIEWER - 325 -
WHEREAS, pursuant to Section 14, Article X of the Constitution, of Baguio. Until otherwise provided by the Cordillera Executive
the President shall provide for regional development councils or Board (CEB), the seat of the CAR shall be Baguio City. The
other similar bodies composed of local government officials, National Government shall provide appropriate offices therefor.
regional heads of departments and other government offices,
and representatives from non-governmental organizations within Sec3. Purposes. — The CAR shall have the following
the regions for purposes of administrative decentralization to purposes:
strengthen the autonomy of the units therein and to accelerate (a) Administer the affairs of government in the region as
the economic and social growth and development of the units in defined in Section 4 and 5 below;
the region; (b) Accelerate the economic and social growth and
WHEREAS, the incumbent President is sympathetic to the development of the units of the region; and
common desire of the peoples of the Cordilleras to be (c) Prepare for the establishment of the autonomous
immediately granted meaningful participation in the conduct of region in the Cordilleras.
their affairs, in order to enable them to prepare for regional
autonomy; Sec4. Scope of Authority and Responsibility. — The CAR
WHEREAS, the constitutional mandate for the creation of an shall have authority and responsibility in the region over the
autonomous region in the Cordilleras has been preceded by the following:
movement for local autonomy and administrative (a) Regional administrative system;
decentralization before and since the period of authoritarian rule (b) Economic, social and cultural development;
(September 1972 - February 1986), as manifested by the work (c) Agricultural, commercial and industrial development
of the 1971 Constitutional Convention, the 1976 resolution of and promotion of tourism;
governors and other leaders from the Cordilleras, and finally the (d) Infrastructure development;
regional autonomy representations at the 1986 Constitutional (e) Urban and rural development, protection of ancestral
Commission; domain and land reform;
WHEREAS, on September 13, 1986, the President of the (f) Regional educational system, including the
Philippines met with the representatives of the Cordillera establishment and maintenance of educational institutions and
Bodong Administration and the Cordillera People's Liberation the formulation of educational policies to cultivate the
Army, presided over the alasiw (exchange of peace tokens) to indigenous Cordillera cultures and inculcate traditional values;
signify the sipat (cessation of hostilities), and acknowledged (g) Health, sports, welfare and social services;
their aspirations for Cordillera autonomy; (h) Development of indigenous laws and political
WHEREAS, the immediate creation of a Cordillera Administrative institutions, particularly those of direct democracy and collective
Region is a sound and reasonable measure by which the people leadership, as well as the promotion of indigenous institutions
of the Cordilleras can immediately participate in the pursuit of and processes for conflict resolution and dispute settlement;
peace and development and enjoy the benefits thereof; (i) Preservation and enhancement of indigenous
WHEREAS, the Constitution envisions the building of a just and customs, traditions, languages and cultures;
humane society, and the National Government is pursuing the (j) Strengthening of the bodong system of tribal unity and
goals of national reconciliation, peace, unity and development in cooperation;
the country; (k) Protection and preservation of the cultural identity,
WHEREAS, pending the convening of the first Congress and the values, mores and norms of the various ethno-linguistic groups
enactment of the organic act for a Cordillera autonomous in the Cordilleras;
region, there is an urgent need, in the interest of national (l) Promotion of social justice and protection of human
security and public order, for the President to reorganize rights, particularly the rights of women, children, the elderly and
immediately the existing administrative structure in the disadvantaged groups, as well as the rights of people's
Cordilleras to suit it to the existing political realities therein and organizations; and
the Government's legitimate concerns in the areas, without (m) Such other matters as may be authorized by law or
attempting to preempt the constitutional duty of the first delegated by the President for the promotion of the general
Congress to undertake the creation of an autonomous region on welfare.
a permanent basis; cd i
NOW, THEREFORE, I, CORAZON C. AQUINO, President of the Sec5. Powers and Functions. — The CAR shall coordinate
Philippines, by virtue of the powers vested in me by the the planning and implementation of programs and services in
Constitution, do hereby order and decree: the areas enumerated in Section 4. Accordingly, it shall be
vested with, among others, the following powers and functions:
Sec1. Cordillera Administrative Region. — There is (a) Coordinate with the local government units as well as
hereby created a Cordillera Administrative Region, hereinafter with the executive departments of the National Government in
referred to as the CAR. the supervision of field offices and in identifying, planning,
monitoring, and accepting projects and activities in the region;
Sec2. Territorial Coverage. — For purposes of the CAR, (b) Appoint, supervise, control and discipline personnel of
the region shall consist of the provinces of Abra, Benguet, the CAR and of such other offices as may be funded by it;
Ifugao, Kalinga-Apayao and Mt. Province and the chartered city

The Apocryphal Maggots:


Rainier, Chrisgel, Corina, Geoffry, Grace and Sylvie Blanche
The Flibbertigibbet Worms:
Golda, Gladys and Melyjane
CA VE AT: By simply r ea ding this re vi ew er a t the end o f the sem este r wi ll (hopef ully) guar antee you r pas sing this cou rse. D rink mode rat ely .
Amusin S C A r y
gly
THE C2005 LOCAL GOVERNMENT REVIEWER - 326 -
(c) Manage and control funds, facilities and equipment April provided, however, that a regular session shall be held in
appropriated for the CAR; 1987.
(d) Advise the National Government on matters affecting The Chairman may call a special session as may be necessary.
the Cordilleras; A special session may also be convened upon the initiative of
(e) Undertake studies towards codifying the customary the majority of all the members of the Assembly.
laws of the tribes, including the pagtas of the bodong system; During the regular session, the Assembly shall:
and a) Discuss the annual report of the Executive Board and
(f) Promulgate and implement resolutions, rules and the proposed budget for the CAR;
regulations necessary to achieve effectively the purposes of this b) Initiate plans and programs for the Cordilleras;
Executive Order and to carry out the powers and functions of c) Discuss and resolve inter-tribal issues and conflicts;
the CAR. d) Formulate policies affecting the Cordilleras consistent
with national and local laws; and
Sec6. Peace and Regional Security. — Within the e) Identify priority projects and development program for
framework of the Constitution and applicable laws, the the region.
restoration and maintenance of peace within the region shall be The decisions of the Assembly shall be implemented by the
a major concern of the CAR. Cordillera Executive Board.
A regional security force shall be organized to assist in the
defense and security of the region subject to guidelines issued Sec10. The Cordillera Executive Board. — The Cordillera
for this purpose by the President after consultations with the Executive Board shall be the development body and
CAR, other organizations, and appropriate agencies of the implementing arm of the CAR. The President shall appoint the
Government. twenty-nine (29) regular members of the Board as follows: (a)
The defense and security of the region shall be the responsibility Mayor of Baguio City and the five (5) Governors of the provinces
of the National Government. enumerated in Section 2; (b) six (6) representatives from the
Cordillera Bodong Administration, one of whom shall be its chief
Sec7. Structure of the CAR. — The CAR shall have a executive; (c) twelve (12) representatives from the different
Cordillera Regional Assembly and a Cordillera Executive Board. ethno-linguistic groups in the Cordillera; and (d) five (5)
representatives from non-governmental organizations.
Sec8. The Cordillera Regional Assembly. — Within the All regional directors of the line departments of the National
framework of the Constitution, laws and policies of the National Government shall be non-voting ex-officio members of the
Government, the Cordillera Regional Assembly shall be the Executive Board.
policy-formulating body which shall articulate and harmonize the
interests and aspirations of the people of the Cordilleras. It shall Sec11. Executive Director. — The Cordillera Executive
be composed of not more than two hundred fifty (250) Board shall be headed by a full-time Executive Board who shall
representatives to be appointed according to the following be headed by a full-time Executive Director who shall be
guidelines: appointed by the President from among its regular members.
(a) Each municipality shall have one (1) representative; The Executive Director shall have the following functions:
(b) Baguio City shall have ten (10) representatives; (a) Act on behalf of the President as Chief Executive
(c) The non-governmental organizations shall be entitled Officer of the CAR;
to a total of eighteen (18) representatives twelve (12) of whom (b) Preside over the meetings of the Executive Board;
shall come from the major non- governmental organizations in (c) Initiate the proposed budget and annual report for the
the region to be determined according to the size of their CAR for the approval of the Executive Board;
membership and six (6) from province based non-governmental (d) Supervise, control and discipline personnel of the
organizations. Executive Board and of such other offices as may be funded by
(d) Each tribe shall send to the Assembly one (1) it;
representative. (e) Coordinate and supervise the Executive Committees
Tribal representatives shall be chosen by the tribes in a manner and the Cordillera Bodong Administration; and
consonant with the Constitution and in harmony with the (f) Perform all other functions assigned by law, the
indigenous decision-making processes in the Cordilleras. President, the Cordillera Regional Assembly or the Executive
The Assembly shall be headed by a Chairman who shall be Board.
appointed by the President from among its members. The
chairman shall be the titular head of the CAR. Sec12. The Executive Committees. — The Cordillera
The President shall also appoint members from non- Executive Board may create executive committees to assist in
governmental organizations as well as representatives of the the implementation of its powers and functions. Each committee
municipalities and of Baguio City upon nomination of their shall be headed by a member of the Executive Board.
respective municipal and city councils.
Sec13. The Cordillera Bodong Administration. — The
Sec9. Sessions. — The Assembly shall convene once every Cordillera Bodong Administration shall be incorporated into the
year for a five-day regular session starting on the 25th day of CAR as a commission and shall hereinafter be referred to as the

The Apocryphal Maggots:


Rainier, Chrisgel, Corina, Geoffry, Grace and Sylvie Blanche
The Flibbertigibbet Worms:
Golda, Gladys and Melyjane
CA VE AT: By simply r ea ding this re vi ew er a t the end o f the sem este r wi ll (hopef ully) guar antee you r pas sing this cou rse. D rink mode rat ely .
Amusin S C A r y
gly
THE C2005 LOCAL GOVERNMENT REVIEWER - 327 -
CBA. In the territorial units where it is effective, the CBA shall Sec19. Rules of Procedures. — The Assembly and the
have the following powers and responsibilities: Executive Board shall each adopt by a majority vote of their
(a) Promote respect for the customs and usages of the members their own rules of procedure as well as rules and
tribes concerned; regulations on discipline and privileges. They shall also
(b) Foster unity among the various communities in the prescribe rules and regulations regarding internal organization,
Cordilleras and promote regional confederation; sessions, meetings and quorum.
(c) Observe the traditional practice of direct democracy
and collective leadership in the Cordilleras within the context of Sec20. Projects. — The CAR, through its Executive Board,
and in harmony with administrative mechanisms of the National shall monitor the implementation of all ongoing national and
Government; local government projects in the region within the purview of
(d) Preserve and develop the communal social order and Section 5(a) hereof.
economic system;
(e) Perform all functions of the Executive Committees Sec21. Appropriations and Funds. — There is hereby
referred to in Section 12; and appropriated from available funds of the National Treasury the
(f) Perform such other functions as may be determined amount of five (5) million pesos for the operation of the CAR for
by the Executive Board. the fiscal year 1987. Thereafter, an annual appropriation for its
The definition and identification of the territorial units of the CBA budget shall be included in the General Appropriations Act.
shall be submitted to and confirmed by the Executive Board and The President and appropriate national departments and
adopted by the Assembly. agencies shall make available sources of funds for priority
The CBA shall be supervised by and be responsible to the physical, social and economic development programs and
Cordillera Executive Board. projects as recommended by the CAR.
The collection, custody, use and disbursement of public funds in
Sec14. CBA Budget. — The CAR, through the Executive the CAR shall be governed by its resolutions, rules and
Board, shall allocate part of its budget for the operation of the regulations of the Executive Board consonant with national laws,
CBA. rules and regulations.

Sec15. Compensation. — The Chairman and members of Sec22. Taxes and Resources. — The CAR shall receive an
the Assembly as well as the Executive Director and members of equitable share of the taxes and other government revenues
the Executive Board shall receive allowances and per diems as generated in the CAR territorial coverage. For this purpose, as
determined by the President in accordance with existing laws part of the BIR allotment in the region, the CAR and the local
and regulations. The city mayor and governors who are government units within the Cordillera Administrative Region's
members of the Board shall receive additional emoluments as territorial coverage shall have at least fifty percent (50%) share
may be allowed by law. of such taxes and other government revenues to be distributed
The Chairman of the Assembly shall receive such additional as follows: ten percent (10%) to barangays, ten percent (10%)
allowances as may be necessary to perform the functions of his to municipalities, fifteen percent (15%) to provinces or cities,
office. fifteen percent (15%) to the CAR.
The Executive Director and the heads of the Executive To ensure proper implementation of the principle of equitable
Communities shall receive an annual compensation to be sharing, the President shall direct the Department of Finance
determined by the Executive Board. and appropriate national departments and agencies to
coordinate with the CAR.
Sec16. Civil Service Rules and Regulations. — For
purposes of the CAR, exemptions from Civil Service rules and Sec23. Grants, Donations, Gifts. — Within the framework of
regulations may be provided by the Civil Service Commission. pertinent laws and regulations, the CAR shall be authorized to
receive grants, donations or gifts, provided that such grants,
Sec17. Period of Existence. — The CAR and its Assembly donations or gifts shall be administered, obligated and disbursed
and Executive Board shall exist until such time as the in accordance with the terms thereof, or in the absence of such
autonomous regional government shall have been established terms, in such manner as a majority of the executive Board may
and organized under an organic act passed by Congress in determine.
accordance with Section 18, Article X of the Constitution.
Sec24. Relationship with the National Government. — The
Sec18. Term of Office. — The term of office of the members President shall have the power of general supervision over the
and officers of the Assembly and Executive Board shall be CAR and the local government units therein and shall issue the
coterminous with the period of existence of the CAR. appropriate guidelines therefor. The President may also call
The city mayor, the governors, and the regional directors shall upon the appropriate executive departments and agencies of
hold office as members of the Executive Board only during the the National Government to assist the CAR as may be
term for which they were elected and/or appointed. necessary.
The Executive Director shall submit a semi-annual report to the
President.

The Apocryphal Maggots:


Rainier, Chrisgel, Corina, Geoffry, Grace and Sylvie Blanche
The Flibbertigibbet Worms:
Golda, Gladys and Melyjane
CA VE AT: By simply r ea ding this re vi ew er a t the end o f the sem este r wi ll (hopef ully) guar antee you r pas sing this cou rse. D rink mode rat ely .
Amusin S C A r y
gly
THE C2005 LOCAL GOVERNMENT REVIEWER - 328 -
legally and validly constitute the CAR. Ordillo et. al, then,
Sec25. Transitory Provisions. — complains.
1) The Executive Board shall conduct a study on the
territorial coverage of the Cordillera autonomous region to be Held: Ifugao cannot be the CAR alone.
established under an organic act to be passed by the Congress The keywords in Art. X, Sec. 15, (Const) — provinces, cities,
under Section 15 and 18, Article X, of the Constitution. This municipalities and geographical areas connote that "region" is to
territorial coverage may include provinces, cities, municipalities, be made up of more than one constituent unit. The term "region"
and geographic areas contiguous to the territory defined under used in its ordinary sense means 2 or more provinces. This is
Section 2. supported by the fact that the 13 regions into which the Philippines
2) The President may call upon the Assembly to assist is divided for administrative purposes are groupings of contiguous
her in constituting the Cordillera Regional Consultative provinces. Ifugao is a province by itself. To become part of a
Commission. region, it must join other provinces, cities, municipalities, and
Consonant with Section 3(c) hereof, the CAR shall contribute in geographical areas. It joins other units because of their common
a meaningful and appropriate manner to the work of the and distinctive historical and cultural heritage, economic and social
Cordillera Regional Consultative Commission and in the structures and other relevant characteristics. The Constitutional
preparation of the organic act by Congress for the Cordillera requirements are not present in this case.
autonomous region.
3) Pursuant to Section 2, the main offices of the CAR
shall be located at the Cordillera House in Baguio City until Cordillera Broad Coalition vs. COA (1990) supra
otherwise determined by the Cordillera Executive Board.
Facts:
Sec26. Separability Clause. — The provisions of this After the 1996 EDSA Revolution, Balweg, broke off on ideological
Executive Order are hereby declared to be separable, and in the grounds from the CPP-NPA. After President Aquino was installed
event any one or more of such provisions are held into office, she advocated a policy of national reconciliation. The
unconstitutional, such shall not affect the validity of other Cordillera People’s Liberation Army (CPLA) heeded this call.
provisions. Aqiuno and Balweg arrived at a joint agreement to draft an
Sec27. Repealing Clause. — For purposes of the CAR and Executive Order to create a preparatory body that could perform
for the duration of its existence, and all laws, acts, presidential policy-making and administrative functions and undertake
decrees, executive orders, proclamations and/or administrative consultations and studies leading to a draft organic act for the
regulations which are inconsistent with this Executive Order are Cordilleras. Pursuant to the joint agreement, E.O. 220, creating the
hereby repealed, amended or modified accordingly. Cordillera Administrative Region (CAR) was signed into law.
Sec28. Effectivity. — This Executive Order shall take effect During the pendency of this case, R.A. No. 6766 (Organic Act of
immediately upon signing and publication as required by law. CAuR) was enacted and signed into law. The Act recognizes the
Done in the City of Manila, this 15th day of July, in the year of CAR and the offices and agencies created under E.O. No. 220 and
Our Lord, nineteen hundred and eighty-seven. its transitory nature is reinforced. Petitioners contend that the
issuance of the EO pre-empted Congress’ task.

See RA 6766 - An Act Providing For An Organic Act For The Held: NO.
Cordillera Autonomous Region EO 220 does not create the autonomous region contemplated in
the Constitution. It merely provides for transitory measures in
anticipation of the enactment of an organic act and the creation of
See RA 8438 - An Act To Establish The Cordillera Autonomous an autonomous region. In short, it prepares the ground for
Region autonomy. This does not necessarily conflict with the provisions of
the Constitution on autonomous regions. The complex procedure
for the creation of an autonomous region in the Cordilleras will take
time. The President, in 1987 still exercising legislative powers, as
Ordillo vs. COMELEC (1990) supra
the first Congress had not yet convened, saw it fit to provide for
some measures to address the urgent needs of the Cordilleras in
Facts: the meantime that the organic act had not yet been passed and the
A plebiscite was conducted in the provinces of Benguet, Mountain autonomous region created. These measures are in E.O. No. 220,
Province, Ifugao, Abra and Kalinga-Apayao and Baguio City cast and they do not violate the Constitution.
their votes in a plebiscite pursuant to R.A. No. 6766 (Oragnic Act of
CAR). The COMELEC results of the plebiscite showed that only The bodies created by E.O. No. 220 do not supplant the existing
the Ifugao Province wanted the CAR. DOJ Sec. issued a local governmental structure, nor are they autonomous
memorandum for the President reiterating the COMELEC government agencies. They merely constitute the mechanism for
resolution and provided that since only the provinces and city an "umbrella" that brings together the existing local governments,
voting favorably shall be included in the CAR, the province of the agencies of the National Government, the ethno-linguistic
Ifugao being the only province which voted favorably will alone,

The Apocryphal Maggots:


Rainier, Chrisgel, Corina, Geoffry, Grace and Sylvie Blanche
The Flibbertigibbet Worms:
Golda, Gladys and Melyjane
CA VE AT: By simply r ea ding this re vi ew er a t the end o f the sem este r wi ll (hopef ully) guar antee you r pas sing this cou rse. D rink mode rat ely .
Amusin S C A r y
gly
THE C2005 LOCAL GOVERNMENT REVIEWER - 329 -
groups or tribes, and NGOs in a concerted effort to spur 1987 Constitution
development in the Cordilleras.
Sec11, ArtX. The Congress may, by law, create special
Neither did E.O. 220 contravene the Constitution by creating a new metropolitan political subdivisions, subject to a plebiscite as set
territorial and political subdivision. The CAR is not a public forth in Section 10 hereof. The component cities and
corporation or a territorial and political subdivision. It does not have municipalities shall retain their basic autonomy and shall be
a separate juridical personality, unlike provinces, cities and entitled to their own local executives and legislative assemblies.
municipalities. Neither is it vested with the powers that are The jurisdiction of the metropolitan authority that will hereby be
normally granted to public corporations (the power to sue and be created shall be limited to basic services requiring coordination.
sued, the power to own and dispose of property, the power to
create its own sources of revenue, etc.). The CAR was created Sec8, ArtXVIII Until otherwise provided by the Congress,
primarily to coordinate the planning and implementation of the President may constitute the Metropolitan Authority to be
programs and services in the covered areas. composed of the heads of all local government units comprising
the Metropolitan Manila area.
The CAR is in the same genre as the administrative regions
created under the Reorganization Plan, albeit under E.O. No. 220. RA7924 (1995) – An Act Creating The Metropolitan Manila
The operation of the CAR requires the participation not only of the Development Authority, Defining Its Powers And Functions,
line departments and agencies of the National Government but Providing Funds Therefor And For Other Purposes
also the local governments, ethno-linguistic groups and NGOs in
bringing about the desired objectives and the appropriation of
Sec1. Declaration of Policy. — It is hereby declared to be
funds solely for that purpose.
the policy of the State to treat Metropolitan Manila as a special
development and administrative region and certain basic
services affecting or involving Metro Manila as metro-wide
Badua vs. Cordillera Bodong Association (1991) services more efficiently and effectively planned, supervised and
coordinated by a development authority as created herein,
Facts: without prejudice to the autonomy of the affected local
Spouses Badua, allegedly own a farm land in faraway Abra and government units.
were forcibly ejected from the land by virtue of a "decision" of the Pursuant to this policy, Metropolitan Manila, as a public
Cordillera Bodong Administration of the Cordillera People's corporation created under Presidential Decree No. 824,
Liberation Army. Fearful for his life, the brave Leonor Badua went embracing the cities of Caloocan, Manila, Mandaluyong, Makati,
into hiding while his wife, Rosa, was arrested by the Cordillera Pasay, Pasig, Quezon, and Muntinlupa, and the municipalities of
People's Liberation Army and detained for two days. The Baduas Las Piñas, Malabon, Marikina, Navotas, Parañaque, Pateros,
filed this petition. San Juan, Tagig, and Valenzuela, is hereby constituted into a
special development and administrative region subject to direct
Issue: WON a tribal court of the Cordillera Bodong supervision of the President of the Philippines.
Administration can render a valid and executory decision in a
land dispute Sec2. Creation of the Metropolitan Manila Development
Held: Authority. — The affairs of Metropolitan Manila's shall be
NO. In the Ordillo case, the creation of the Cordillera Autonomous administered by the Metropolitan Manila Development Authority,
Region was rejected by all the provinces and city of the Cordillera hereinafter referred to as the MMDA, to replace the Metro
region, except Ifugao province, hence, the Cordillera Autonomous Manila Authority (MMA) organized under Executive Order No.
Region did not come to be. As a logical consequence of that 392 series of 1990.
judicial declaration, the Cordillera Bodong Administration, the The MMDA shall perform planning, monitoring and coordinative
indigenous and special courts for the indigenous cultural functions, and in the process exercise regulatory and
communities of the Cordillera region and the Cordillera People's supervisory authority over the delivery of metro-wide services
Liberation Army, do not legally exist. within Metro Manila without diminution of the autonomy of the
local government units concerning purely local matters.
Since the Cordillera Autonomous Region did not come into legal
existence, the Maeng Tribal Court was not constituted into an Sec3. Scope of MMDA Services. — Metro-wide services
indigenous or special court under R.A. No. 6766. Hence, the under the jurisdiction of the MMDA are those services which
Maeng Tribal Court is an ordinary tribal court existing under the have metro-wide impact and transcend local political
customs and traditions of an indigenous cultural community. They boundaries or entail huge expenditures such that it would not be
do not possess judicial power. viable for said services to be provided by the individual local
government units (LGUs) comprising Metropolitan Manila.
These services shall include:
Me tro Man ila Dev elo pm ent Au tho ri ty (a) Development planning which includes the preparation
of medium and long-term development plans; the development,
evaluation and packaging of projects; investments

The Apocryphal Maggots:


Rainier, Chrisgel, Corina, Geoffry, Grace and Sylvie Blanche
The Flibbertigibbet Worms:
Golda, Gladys and Melyjane
CA VE AT: By simply r ea ding this re vi ew er a t the end o f the sem este r wi ll (hopef ully) guar antee you r pas sing this cou rse. D rink mode rat ely .
Amusin S C A r y
gly
THE C2005 LOCAL GOVERNMENT REVIEWER - 330 -
programming; and coordination and monitoring of plan, program of Budget and Management (DBM), Housing and Urban
and project implementation. Development Coordinating Committee (HUDCC), and Philippine
(b) Transport and traffic management which include the National Police (PNP) or their duly authorized representatives,
formulation, coordination, and monitoring of policies, standards, shall attend meetings of the council as non-voting members.
programs and projects to rationalize the existing transport The Council shall be headed by a chairman, who shall be
operations, infrastructure requirements, the use of appointed by the President and who shall continue to hold office
thoroughfares, and promotion of safe and convenient movement at the discretion of the appointing authority. He shall be vested
of persons and goods; provision for the mass transport system with the rank, rights, privileges, disqualifications, and
and the institution of a system to regulate road users; prohibitions of a cabinet member.
administration and implementation of all traffic enforcement The chairman shall be assisted by a general manager, an
operations, traffic engineering services and traffic education assistant general manager for finance and administration, an
programs, including the institution of a single ticketing system in assistant general manager for planning and assistant general
Metropolitan Manila. Cdt manager for operation, all of whom shall be appointed by the
(c) Solid waste disposal and management which include President with the consent and concurrence of the majority of
formulation and implementation of policies, standards, programs the Council, subject to civil service laws, rules and regulations.
and projects for proper and sanitary waste disposal. It shall They shall enjoy security of tenure and may be removed for
likewise include the establishment and operation of sanitary land cause in accordance with law.
fill and related facilities and the implementation of other The assistant general manager for planning must have not less
alternative programs intended to reduce, reuse and recycle solid than five (5) years extensive experience in development and
waste. planning or must hold a master's degree in urban planning or
(d) Flood control and sewerage management which similar disciplines.
include the formulation and implementation of policies, The chairman and members of the Council shall be entitled to
standards, programs and projects for an integrated flood control, allowance and per diems in accordance with existing policies,
drainage and sewerage system. rules and regulations on the matter.
(e) Urban renewal, zoning, and land use planning, and
shelter services which include the formulation, adoption and Sec5. Functions and Powers of the Metro Manila
implementation of policies, standards, rules and regulations, Development Authority. — The MMDA shall:
programs and projects to rationalize and optimize urban land (a) Formulate, coordinate and regulate the
use and provide direction to urban growth and expansion, the implementation of medium and long-term plans and programs
rehabilitation and development of slum and blighted areas, the for the delivery of metro-wide services, land use and physical
development of shelter and housing facilities and the provision development within Metropolitan Manila, consistent with national
of necessary social services thereof. development objectives and priorities;
(f) Health and sanitation, urban protection and pollution (b) Prepare, coordinate and regulate the implementation
control which include the formulation and implementation of of medium-term programs for metro-wide services which shall
policies, rules and regulations, standards, programs and indicate sources and uses of funds for priority programs and
projects for the promotion and safeguarding of the health and projects, and which shall include the packaging of projects and
sanitation of the region and for the enhancement of ecological presentation to funding institutions;
balance and the prevention, control and abatement of (c) Undertake and manage on its own metro-wide
environmental pollution. programs and projects for the delivery of specific services under
(g) Public safety which includes the formulation and its jurisdiction, subject to the approval of the Council. For this
implementation of programs and policies and procedures to purpose, MMDA can create appropriate project management
achieve public safety, especially preparedness for preventive or offices;
rescue operations during times of calamities and disasters such (d) Coordinate and monitor the implementation of such
as conflagrations, earthquakes, flood and tidal waves, and plans, programs and projects in Metro Manila; identify
coordination and mobilization of resources and the bottlenecks and adopt solutions to problems of implementation;
implementation of contingency plans for the rehabilitation and (e) The MMDA shall set the policies concerning traffic in
relief operations in coordination with national agencies Metro Manila, and shall coordinate and regulate the
concerned. implementation of all programs and projects concerning traffic
management, specifically pertaining to enforcement,
Sec4. Metro Manila Council. — The governing board and engineering and education. Upon request, it shall be extended
policy making body of the MMDA shall be the Metro Manila assistance and cooperation, including but not limited to,
Council, composed of the mayors of the eight (8) cities and nine assignment of personnel, by all other government agencies and
(9) municipalities enumerated in Section 1 hereof, the president offices concerned;
of the Metro Manila Vice Mayors League and the president of (f) Install and administer a single ticketing system, fix,
the Metro Manila Councilors League. impose and collect fines and penalties for all kinds of violations
The heads of the Department of Transportation and of traffic rules and regulations, whether moving or non-moving in
Communications (DOTC), Department of Public Works and nature, and confiscate and suspend or revoke drivers' licenses
Highways (DPWH), Department of Tourism (DOT), Department in the enforcement of such traffic laws and regulations, the

The Apocryphal Maggots:


Rainier, Chrisgel, Corina, Geoffry, Grace and Sylvie Blanche
The Flibbertigibbet Worms:
Golda, Gladys and Melyjane
CA VE AT: By simply r ea ding this re vi ew er a t the end o f the sem este r wi ll (hopef ully) guar antee you r pas sing this cou rse. D rink mode rat ely .
Amusin S C A r y
gly
THE C2005 LOCAL GOVERNMENT REVIEWER - 331 -
provisions of RA 4136 and PD 1605 to the contrary (g) Perform such other duties as may be assigned to him
notwithstanding. For this purpose, the Authority shall enforce by the President or by the Council.
all traffic laws and regulations in Metro Manila, through its traffic
operation center, and may deputize members of the PNP, traffic Sec8. Functions of the General Manager. — The general
enforcers of local government units, duly licensed security manager shall:
guards, or members of non-governmental organizations to (a) Assist the chairman in the administration of the MMDA
whom may be delegated certain authority, subject to such and supervision of subordinate personnel;
conditions and requirements as the Authority may impose; and (b) Assist the chairman in the supervision of the operation
(g) Perform other related functions required to achieve of the various operating centers and units of the MMDA;
the objectives of the MMDA, including the undertaking of (c) Assist the chairman in the review of plans and
delivery of basic services to the local government units, when programs for the MMDA and for Metro Manila in the preparation
deemed necessary subject to prior coordination with and of the annual report of activities and accomplishments of the
consent of the local government unit concerned. MMDA; and
(d) Perform such other duties and functions as may be
Sec6. Functions of the Metro Manila Council. — lawfully delegated or assigned by the chairman from time to
(a) The Council shall be the policy-making body of the time.
MMDA.
(b) It shall approve metro-wide plans, programs and Sec9. Institutional Linkages of the MMDA. — The MMDA
projects and issue rules and regulations and resolutions shall, in carrying out its functions, consult, coordinate and work
deemed necessary by the MMDA to carry out the purposes of closely with the LGUs, the National Economic and Development
this Act. Authority (NEDA) and other national government agencies
(c) It may increase the rate of the allowances and per mentioned in Section 4 hereof, and accredited people's
diems of the members of the Council to be effective during the organizations (POs), nongovernmental organizations (NGOs),
term of the succeeding Council. It shall fix the compensation of and the private sector operating in Metro Manila. The MMDA
the officers and personnel of the MMDA, and approve the chairman or his authorized representative from among the
annual budget thereof for submission to the Department of Council members, shall be ex officio member of the boards of
Budget and Management (DBM). government corporations and committees of the department and
(d) It shall promulgate rules and regulations and set offices of government whose activities are relevant to the
policies and standards for metro-wide application governing the objectives and responsibilities of the MMDA which shall include
delivery of basic services, prescribe and collect service and but not be limited to Metropolitan Waterworks and Sewerage
regulatory fees, and impose and collect fines and penalties. System (MWSS), DOTC, DPWH, HUDCC and Department of
the Interior and Local Government (DILG).
Sec7. Functions of the Chairman. — The chairman shall: The MMDA shall have a master plan that shall serve as the
(a) Appoint, subject to civil service laws, rules and framework for the local development plans of the component
regulations, all subordinate officers and employees, who shall LGUs.
enjoy security of tenure and may be removed only for cause in The MMDA shall submit its development plans and investment
accordance with law. The chairman is hereby authorized to programs to the NEDA for integration into the Medium-Term
engage the services of experts/consultants either on full time or Philippine Development Plan (MTPDP) and public investment
part-time basis, as may be required in the performance of his program.
functions and duties as may be determined by him; The implementation of the MMDA's plans, programs, and
(b) Execute the policies and measures approved by the projects shall be undertaken by the LGUs, the concerned
Metro Manila Council and be responsible for the efficient and national government agencies, the POs, NGOs and the private
effective day-to-day management of the operations of the sector and the MMDA itself where appropriate. For this purpose,
MMDA; the MMDA may enter into contracts, memoranda of agreement
(c) Prepare the annual budget for the operations of the and other cooperative arrangements with these bodies for the
MMDA for submission to the Council; delivery of the required services within Metropolitan Manila.
(d) Submit for consideration of the Council such other The MMDA shall, in coordination with the NEDA and the
policies and measures as he may deem necessary to carry out Department of Finance, interface with the foreign assistance
the purposes and provisions of this Act; agencies for purposes of obtaining financing support, grants and
(e) Subject to the guidelines and policies set by the donations in support of its programs and projects.
Council, prepare the staffing pattern and fix the number of
subordinate officials and employees of the MMDA; and exercise Sec10. Sources of Funds and the Operating Budget Of
the power to discipline subordinate officials and employees MMDA:
under the provisions of law; (a) To carry out the purposes of this Act, the Amount of
(f) Prepare an annual report on the accomplishments of One billion pesos (P1,000,000,000) is hereby authorized to be
the MMDA at the close of each calendar year for submission to appropriated for the initial operation of the MMDA. Thereafter,
the Council and to the President of the Philippines; and the annual expenditures including capital outlays of the MMDA
shall be provided in the General Appropriations Act.

The Apocryphal Maggots:


Rainier, Chrisgel, Corina, Geoffry, Grace and Sylvie Blanche
The Flibbertigibbet Worms:
Golda, Gladys and Melyjane
CA VE AT: By simply r ea ding this re vi ew er a t the end o f the sem este r wi ll (hopef ully) guar antee you r pas sing this cou rse. D rink mode rat ely .
Amusin S C A r y
gly
THE C2005 LOCAL GOVERNMENT REVIEWER - 332 -
(b) The MMDA shall continue to receive the Internal violators, whatever sanctions it may impose must be "in such
Revenue Allotment (IRA) currently allocated to the present amounts and under such penalties as are herein prescribed."
MMA.
(c) The MMDA is likewise empowered to levy fines, and LOI 43 is inapplicable since what the LOI punishes is not a traffic
impose fees and charges for various services rendered. Cdasia violation but a traffic obstruction, which is an altogether different
(d) Five percent (5%) of the total annual gross revenue of offense. LOI 43 does not punish illegal parking per se but parking
the preceding year, net of the internal revenue allotment, or of stalled vehicles, i e., those that involuntarily stop on the road
each local government unit mentioned in Section 2 hereof, shall due to some unexpected trouble such as engine defect, lack of
accrue and become payable monthly to the MMDA by each city gasoline, punctured tires, or other similar cause.
or municipality. In case of failure to remit the said fixed
contribution, the DBM shall cause the disbursement of the same
to the MMDA chargeable against the IRA allotment of the city or Solicitor General vs. MMA (1991)
municipality concerned, the provisions of Section 286 of RA
7160 to the contrary notwithstanding. Facts:
upreme Court received several complaints regarding the
Sec11. Transitory Provisions. — To prevent disruption in the confiscation of the complainants’ driver’s licenses and even a
delivery of the basic urban services pending the full license plate by the traffic enforcers for alleged traffic violations
implementation of the MMDA's organizational structure and under Ordinance No. 7 of Mandaluyong, Memorandum from the
staffing pattern, all officials and employees of the interim MMA District Commander of the Western Traffic District of PNP, and
shall continue to exercise their duties and functions and receive Ordinance No. 11 of the Metropolitan Manila Authority
their salaries and allowances until they shall have been given
notice of change of duties and functions, and of being Issue 1: Won There Was Valid Delegation Of Power to MMA (&
transferred to another office or position. The Mun. Of Mandaluyong) to Issue Such Ordinance/S
All assets and properties presently in use or under the Held1:
accountability of the interim MMA and all its obligations, YES. There is a valid delegation of legislative power to promulgate
indebtedness, or liabilities shall be transferred to and assumed such measures, it appearing that the requisites of such delegation
by the MMDA created under this Act, subject to the conditions are present. (1) The completeness of the statute making the
that may be established by the Department of Budget and delegation and (2) The presence of a sufficient standard –
Management, Office of the President, and Commission on Audit. “convenience and welfare” of the public, particularly the motorists
The civil service laws, rules and regulations pertinent to the and passengers.
displacement of personnel affected by this Act shall be strictly
enforced. The national government shall provide such amounts
Issue 2: Won The Exercise Of The Delegated Power By The MMA
as may be necessary to pay the benefits accruing to displaced
(& Mun. Of Mandaluyong) Was Valid
employees at the rate of one and one-fourth (1 1/4) month's
Held2:
salary for every year of service: Provided, That, if qualified for
NO. The measures under consideration do not pass the first
retirement under existing retirement laws, said employees may
criterion that it must not contravene the constitution because they
opt to receive the benefits thereunder.
do not conform to existing law. The pertinent law is PD 1605.

PD 1605 does not allow either the removal of license plates or the
Metropolitan Transfer Command vs. Gonong (1990) confiscation of driver’s licenses for traffic violations committed in
Metropolitan Manila. In fact, PD 1605 prohibit the imposition of
Facts: such sanctions in Metropolitan Manila. Notably, Section 5 thereof
The rear license plate of David’s car was removed by the expressly provides that in case of traffic violations, the driver’s
Metropolitan Traffic Command while the vehicle was parked on license shall not be confiscated.
Escolta. He questioned the petitioner's act on the ground not only
that the car was not illegally parked but, more importantly, that NOTES:
there was no ordinance or law authorizing such removal. He asked Ordinance cannot impose higher penalty than the law. So what’s
that the practice be permanently enjoined and that in the meantime the point?
a temporary restraining order or a writ of preliminary injunction be
issued. Thoughts to ponder on: What if in Cebu they have ordinances
protecting women from domestic violence, what happens if a
Held: subsequent national law w/ lower penalties or decreasing the
A careful reading of the above PD 1605 will show that removal and grounds for violations?
confiscation of the license plate of any illegally parked vehicle is
not among the specified penalties. Moreover, although the
Metropolitan Manila Commission is authorized by the decree to
"otherwise discipline" and "impose higher penalties" on traffic MMDA vs. Bel-Air Village Association (2000)

The Apocryphal Maggots:


Rainier, Chrisgel, Corina, Geoffry, Grace and Sylvie Blanche
The Flibbertigibbet Worms:
Golda, Gladys and Melyjane
CA VE AT: By simply r ea ding this re vi ew er a t the end o f the sem este r wi ll (hopef ully) guar antee you r pas sing this cou rse. D rink mode rat ely .
Amusin S C A r y
gly
THE C2005 LOCAL GOVERNMENT REVIEWER - 333 -
Fact s:
Bel-Air Village Associaiton (BAVA) received from MMDA a notice Q: Can MMDA remove vendors from sidewalk?
requesting it to open Neptune Street to public vehicular traffic. A: No, because MMDA has no police power. Vendors are the
BAVA was also apprised that the perimeter wall separating the concern of the Sangguniang Bayan or Panglungsod not the
subdivision from the adjacent Kalayaan Avenue would be MMDA. LGU’shave the ordinances to govern sidewalk vendors.
demolished. BAVA institued a petition for injunction with TRO and They should ask MMDA to help them.
preliminary writ of injunction against MMDA.
Sexy Writing Exercise from How to Write a Dirty Story: Reading,
Issue: WON MMDA IS ENDOWED WITH POLICE POWER (note:
Writing and Publishing Erotica
police power cannot be exercised by any group or body not If you're thinking about writing an erotic story or love letter, but
possessing legislative power)
don't know where to start, try this exercise to get you started!
Held:
NO. The MMDA is, as termed in the charter itself, a “development
authority.” It is an agency created for purpose of laying down Fantasies Exercise
policies and coordinating with the various national government Give yourself two minutes to answer each of the following
agencies, people’s organizations, non-governmental organizations questions. When your time is up, stop, even if you haven't
and the private sector for the efficient and expeditious deliver of finished your sentence.
basic services in the vast metropolitan area. All its functions are • Write down an erotic fantasy about a sexual experience
ADMINISTRATIVE in nature. that you would have in a minute if it were offered to you,
The powers of the MMDA under RA 7924 are limited to the no questions asked. It should be something you would
following acts: formulation, coordination, regulation,
have no reservations or conditions about doing in real life.
implementation, preparation, management, monitoring, setting of
policies, installation of a system and administration. There is • Write down an erotic fantasy about a sexual experience
nothing in RA NO. 7924 that grants MMDA police power, let alone that you would only have under certain conditions. You
legislative power. could give yourself up wholeheartedly under those
conditions, but otherwise not at all.
The MMDA is not the same entity as the MMC in Sangalang.
Although the MMC is the forerunner of the present MMDA, the
• Write down an erotic fantasy about a sexual experience
charter of the MMC shows that the latter possessed greater that is completely satisfying to you in your imagination but
powers which were not bestowed on the present MMDA. that you could not do because it is either physically
impossible or something you could never bring yourself to
The legislative debates would show that the MMDA was not do in real life... Yet in your mind, it is completely hot and
intended as a political unit of the government or a public fulfilling
corporation endowed with legislative power. It is not even a Now you have three potential pieces of fiction, based on your
“special metropolitan political subdivision”. fantasies. Take another sheet of paper and answer the following:
• no plebiscite was conducted for its creation • What do you notice about the differences, or similarities,
• the chairman of the MMDA is not an official elected by
between your three fantasies?
the people, but appointed by the president with the rank
and privileges of a cabinet member. • Have you ever confided any of these fantasies to anyone?
• Part of the chairman’s functions is to perform such other • Is any one of your three fantasies more compelling than
duties as may be assigned to him by the President, another, sexually or creatively?
whereas in LGUs, the president merely exercises
supervisory authority. This emphasizes the Goal
administrative character of the MMDA.
• To articulate thoughts that are often unspoken and
Clearly, the MMC is not the same entity as MMDA. Unlike the unwritten.
MMC, the MMDA has no power to enact ordinances for the welfare • To pay intimate attention to your erotic unconscious.
of the community. It is the LGUs, acting through their legislative
councils, that possess legislative power and police power.
• To identify the elements in your erotic "stories" that both
propel and inhibit you.
NOTES:
Q: Can the LGU grant police power to MMDA: After You Finish This Exercise, You'll Never Believe Again...
A: No, because Congress had already denied them that power • That your fantasy life is barren.
and no law allows LGU’s to do that. • That fantasies can only be satisfied by acting them out.
SIR: LGU’s should be telling MMDA what to do NOT the other • That fantasies are not enhanced by conflicts, taboos and
way around. inhibitions. (Like literature, they thrive on all those things!)
A TRUE STORY
The Apocryphal Maggots:
At the last day of Loc Gov class, DG says:
Rainier, Chrisgel, Corina, Geoffry, Grace and Sylvie Blanche
The Flibbertigibbet Worms:
Golda, Gladys and Melyjane
“ If I could give a standing ovation alone for this
CA VE AT: By simply r ea ding this re vi ew er a t the end o f the sem este r wi ll (hopef ully) guar antee you r pas sing this cou rse. D rink mode rat ely .
class (meaning C 2005 of course), I would. You have
improved from the last time. And I think you’re
better than the other one. That’s why I tell my
freshmen class there is still hope for you. Look at
this section (again, referring to C-2005).”

Anda mungkin juga menyukai